You are on page 1of 769

Table of Contents

Foreword J oanna M Flynn ix


Preface Roger J Pepperell x
Contributors xi
Editorial Committee
Additional Contributors
Acknowledgements xvii
Introduction Vernon C Marshall 1
Role of the Australian Medical Council (AMC) Ian B Frank 9
Construction, Scoring and Validation of Assessments Neil S Paget 25
The AMC Multidisciplinary Clinical Assessment Task (MCAT) Format 31
Heather G Alexander
How to Use this AMC Handbook of Annotated MCATs Vernon C Marshall 34
MCAT Format Example: Candidate Information and Tasks, Performance
Guidelines 37
001 A cut to the thumb of a 22-year-old man
MCAT Candidate Information and Tasks, MCAT Performance Guidelines; 44
Five Principal Categories and Domains
1 CLINICAL COMMUNICATION (C) 45
1-A Communication, Counselling, and Patient Education 45
Introduction: Alan T Rose
~ MCAT Candidate Information and Tasks 002-021 51-67
~ MCAT Performance Guidelines 002-021 68-130
CIT PG
DETAILS OF MCAT SCENARIOS
002 Advice on breastfeeding versus bottle-feeding for a 28-year-old
pregnant woman 53 69
3 Advice on neonatal circumcision for a couple expecting their first child
53 72
54 75
55 77
56 79
57 81
58 84
58 87
59 90
60 92
61 95
4 Suspected hearing impairment in a 10-month-old child
5 Counselling a family after sudden infant death syndrome (SIDS)
6 Hair loss in a 38-year-old man
7 An unusual feeling in the throat in a 30-year-old man
8 Pain in the testis following mumps in a 25-year-old man
9 Contraceptive advice for a 24-year-old woman
10 Rape of a 20-year-old woman
11 Cancer of the colon in a 60-year-old man
12 Thalassaemia minor in a 22-year-old woman
i
CIT PG
ii
13 Pre-pregnancy advice to a 28-year-old woman with previous
thromboembolism
14 Pre-pregnancy advice to a 24-year-old woman with Type 1
diabetes mellitus
15 An anencephalic fetus diagnosed at 18 weeks gestation in a
25-year-old primigravida
16 A duodenal ulcer found on endoscopy in a 65-year-old man
17 Advice on autologous blood transfusion to a 55-year-old man
awaiting elective surgery
18 Advice on stopping smoking to a 30-year-old man
19 Excessive alcohol consumption in a 45-year-old man
20 Type 1 diabetes mellitus in a 9-year-old boy
21 Request for vasectomy from a 36-year-old man
1-B Case presentations and summaries to Examiner
Introduction: Vernon C Marshall
DETAILS OF MCAT SCENARIOS
022-029 Headache, neck lump, previous shoulder dislocation,
dysphagia, low back pain, knee pain, abdominal discomfort, gastric
ulcer with haemorrhage


62 99
131
132-135

2 CLINICAL DIAGNOSIS (D)
2-A The Diagnostic Process History-taking and Problem-Solving
Introduction: Reuben D Glass
~ MCAT Candidate Information and Tasks 030-043 ~
MCAT Performance Guidelines 030-043
DETAILS OF MCAT SCENARIOS
30 J aundice in a breastfed infant
31 A convulsion in a 14-month-old boy
32 Loud and disruptive behaviour of a 6-year-old boy
33 Tremor in a 40-year-old man
34 Headache in a 35-year-old woman
35 Lethargy in a 50-year-old woman
36 Syncope in a 52-year-old man
37 A painful penile rash in a 23-year-old man
38 Primary amenorrhoea in an 18-year-old woman
39 A skin lesion on the cheek of a 50-year-old man
40 A pigmented mole on the trunk of a 30-year-old woman
41 An itchy rash on the hands of a 19-year-old woman
42 Red painful dry hands in a 30-year-old bricklayer
43 Swelling of both ankles in a 53-year-old woman
137

137

142

- 154

155

- 195

143

156
144 159
144 161
145 164
145 167
146 170
147 173
148 177
149 180
150 182
151 184
152 186
153 189
154 191

62 102
63 105
64 108
65 111
65 115
66 121
67 125
67 129
CIT PG
2-B Physical Examination 196
Introduction: Vernon C Marshall and Barry P McGrath
~MCAT Candidate Information and Tasks 044-057 218 -23
3
~MCAT Performance Guidelines 044-057 234 -29
6
DETAILS OF MCAT SCENARIOS
044 Assessment of a comatose patient 219 235
045 Recent onset of poor distance vision in a 17-year-old male 220 241
046 A painful rash on the trunk of a 45-year-old child-care worker 221 246
047 Acute low back pain and sciatica in a 30-year-old man 222 248
048 Fever and a recent rash in a 30-year-old man 223 252
049 A heart murmur in a 4-year-old boy 224 255
050 A knife wound to the wrist of a 25-year-old man 225 257
051 Multiple skin lesions in a Queensland family 226 264
052 Subcutaneous swelling for assessment 228 274
053 Examination of the knee of a patient with recurrent painful
swelling after injury 229 280
054 Assessment of hearing loss, first noted during pregnancy in a
35-year-old woman 230 282
055 Examination of a 20-year-old woman who dislocated her
shoulder 6 months ago 231 286
056 Assessment of a groin lump in a 40-year-old man 232 289
057 Eye problems in an aboriginal community 233 293
2-C Choice and Interpretation of Investigations 297
Introduction: Reuben D Glass and Vernon C Marshall
~MCAT Candidate Information and Tasks 058-064 312- 319
~MCAT Performance Guidelines 058-064 320- 342
DETAILS OF MCAT SCENARIOS
058 Positive test for hepatitis C in a 26-year-old woman 313 321
059 Diagnosis of 'brain death' prior to organ donation 314 325
060 Breast biopsy concerns in a 20-year-old woman with a family
history of breast cancer 315 329
061 An elbow injury in an 11-year-old schoolgirl 316 331
062 Sudden onset of chest pain and breathlessness in a
20-year-old woman 317 334
063 Atypical ureteric colic in a 25-year-old man 318 337
064 Investigation for male factor infertility in a 25-year-old man 319 340


iii
CIT PG
2-D The General Consultation 343
Introduction: Barry P McGrath
~MCAT Candidate Information and Tasks 065-073 347-354
~MCAT Performance Guidelines 065-073 355-396
DETAILS OF MCAT SCENARIOS
065 Acute chest pain in a 60-year-old man 348 356
066 Palpitations and dizziness in a 50-year-old man 349 363
067 Muscle weakness and urinary symptoms in a 60-year-old man 350 368
068 Aches and pains in a 62-year-old man 351 371
069 Lack of energy in a 56-year-old suntanned man 352 374
070 Recent haematemesis in a 50-year-old man 352 377
071 Anaemia in a 28-year-old pregnant woman 353 380
072 Acute vertigo in a 50-year-old man 353 383
073 Urinary frequency in a 60-year-old man 354 394
2-E The Paediatric Consultation

397
Introduction: Peter J Vine
~MCAT Candidate Information and Tasks 074-077 401-403
~MCAT Performance Guidelines 074-077 404-416
DETAILS OF MCAT SCENARIOS
74 Neonatal jaundice in the first day of life 402 405
75 Immunisation advice to the parent of a 6-week-old baby 402 408
76 Dark urine, facial swelling and irritability in a 5-year-old boy 403 412
77 Fever and sore throat in a 5-year-old boy 403 414
2-F The Obstetric and Gynaecologic Consultation 417
Introduction: Roger J Pepperell
~MCAT Candidate Information and Tasks 078-082 419-422
MCAT Performance Guidelines 078-082 423-435
DETAILS OF MCAT SCENARIOS
78 Breech presentation in labour at 38 weeks in a 25-year-old woman 420 424
79 Vaginal bleeding in a 23-year-old woman 420 427
80 Cessation of periods in a 30-year-old woman on the oral
contraceptive pill (OCP) 421 430
081 Positive culture for Group B streptococci (GBS) at 36 weeks of
gestation in a 26-year-old woman 421 432
082 Vaginal bleeding after 8 weeks amenorrhoea, in a woman with
previous irregular cycles 422 434
2-G The Psychiatric Consultation 436
Introduction: Frank P Hume
~MCAT Candidate Information and Tasks 083-089 446-454
~MCAT Performance Guidelines 083-089 455-481
iv

CIT Pfi
DETAILS OF MCAT SCENARIOS

083 Medication changes for a 35-year-old woman with chronic

schizophrenia 447 456
084 Demand for urgent treatment for 'sudden hair loss' from a

29-year-old man 448 459
085 Poor work performance in a 30-year-old female police officer 449 463
086 Lifestyle stress in a 45-year-old man 450 466
087 Binge drinking in a 25-year-old man 452 470
088 Nausea, headache and feeling 'jittery' in a 30-year-old bank clerk 453 474
089 Collapse of a 30-year-old woman on the way to a court attendance 454 478
3 CLINICAL MANAGEMENT (M)

483
3-A Management Objecti ves, Therapeutics, Prevention and Public Health 483
Introduction: Alan T Rose, Michael R Kidd and Ronald McCoy

~MCAT Candidate Information and Tasks 090-100 489- -498
~MCAT Performance Guidelines 090-100 499- -536
DETAILS OF MCAT SCENARIOS

090 Acute right sided pain and haematuria in a 25-year-old man 490 500
091 Faecal soiling in a 5-year-old boy 491 503
092 Psoriasis in a 30-year-old man 492 507
093 Temporal arteritis in a 58-year-old woman 493 510
094 Acute idiopathic facial nerve palsy ('Bell Palsy') in a

40-year-old man 494 512
095 Dysuria and urinary frequency in a 40-year-old man 495 519
096 Eclampsia in a 22-year-old primigravida at 38 weeks of gestation 496 522
097 An abnormal glucose tolerance test (GTT) in a 34-year-old

primigravida 496 525
098 Bed-wetting by a 5-year-old boy 497 528
099 Acute gout in a 48-year-old man 497 531
100 Request for repeat benzodiazepine prescription from a

25-year-old man 498 534
3-B Clinical Procedures

537
Introduction: Peter G Devitt and Barry P McGrath

~MCAT Candidate Information and Tasks 101-104 543- -547
~MCAT Performance Guidelines 101-104 548- -563
DETAILS OF MCAT SCENARIOS

101 Resuscitation of a 24-year-old man after head and chest injury 544 549
102 Fluid balance assessment in a 50-year-old patient after

abdominal surgery 545 551
103 Evaluation of lung function by spirometry in a 22-year-old man 546 558
104 A suspected fractured clavicle in a 20-year-old man 547 561

V

CIT PG
INTEGRATED DIAGNOSIS AND MANAGEMENT (D/M)

565
4-A Clinical Perspecti ve and Priorities

565
Introduction: Bryan W Yeo

~MCAT Candidate Information and Tasks 105-112 570 -577
~MCAT Performance Guidelines 105-112 578 -600
DETAILS OF MCAT SCENARIOS

105 Abdominal pain and vaginal bleeding after 9 weeks amenorrhoea,

in a 39-year-old woman 571 579
106 Recent insomnia in a 25-year-old man 572 582
107 Dandruff or head lice in a 6-year-old girl? 573 585
108 Recent orchidectomy for a testicular neoplasm in a

28-year-old man 574 587
109 Postnatal fatigue and exhaustion in a 28-year-old woman 575 589
110 Fundus greater than dates in a 26-year-old woman at

30 weeks gestation 575 593
111 Tiredness and anaemia in a 55-year-old woman 576 596
112 Colonoscopy findings in a 24-year-old man with chronic diarrhoea 577 599
4-B Life-threatening Emergencies Priorities of Treatment

601
Introduction: Bryan W Yeo

~MCAT Candidate Information and Tasks 113-118 602- 608
~MCAT Performance Guidelines 113-118 609- 627
DETAILS OF MCAT SCENARIOS

113 A severely ill 4-month-old baby girl with fever 603 610
114 A lethargic febrile 2-year-old boy with a rash 604 612
115 Wheezing and breathing difficulty in a 5-year-old girl 605 614
116 Cuts to the wrist of a 25-year-old man 606 618
117 Severe postpartum haemorrhage in a 25-year-old primigravida 607 622
118 Emergency management of a snake-bite in a 20-year-old man 608 625
LEGAL, ETHICAL AND ORGANISATIONAL (LEO)

628
5-A Ethical and Legal Dilemmas

629
Introduction: Kerry J Breen

~MCAT Candidate Information and TasKS 119-124 633- 639
~MCAT Performance Guidelines 119-124 640- 659
DETAILS OF MCAT SCENARIOS

119 A man requesting disclosure of his wife's medical condition 634
641
120 Obtaining consent for leg amputation in a 35-year-old man after

a motor vehicle injury 635 644
121 Several bone fractures in a 9-week-old baby 636 647

VI

CIT PG
122 A parent requesting sterilisation of her intellectually disabled


daughter 637 649
123 Blood transfusion consent for a 33-year-old pregnant woman


with severe APH at 7 months 638 652
124 End-of-life request from a terminally ill patient 639 655
MCAT TRIAL EXAMINATIONS

661
Preparatory Instructions

661
Roger J Pepperell

16 Station Trial Assessment

~MCAT Candidate Information and Tasks T1-T16 664 -678
~MCAT Performance Guidelines T1-T16 679 -730
DETAILS OF MCAT TRIAL ASSESSMENTS

125 [T1] Meconium staining of liquor in labour in a 25-year-old


primigravida 665 680
126 [T2] A heart murmur in a 5-year-old girl 666 683
127 [T3] Vigorous vomiting by a 3-week-old boy 667 685
128 [T4] Urinary incontinence in a 50-year-old woman 668 688
129 [T5] Migraine in a 30-year-old woman 668 691
130 [T6] Past history of hip dislocation in a 35-year-old man 669 694
131 [T7] Tiredness in a 45-year-old man 670 696
132 [T8] Review of lung function tests in a 65-year-old man with


shortness of breath 671 700
133 [T9] Assessment of a 28-year-old primigravida at 34 weeks


with fundus less than dates 672 705
134 [T10] Delirium in a 25-year-old man after a burn injury 672 708
135 [T11] Chronic diarrhoea in a 45-year-old man 673 712
136 [T12] Fever, irritability and ear discharge in a 2-year-old boy 674 716
137 [T13] Review of cytology after aspiration of a breast lesion in a


28-year-old woman 675 718
138 [T14] Nocturnal hand discomfort in a 35-year-old schoolteacher 677 721
139 [T15] An attack of asthma in a 25-year-old man 677 724
140 [T16] Preparing a 30-year-old woman with suspected acute


appendicitis for surgery 678 728
8 Station Trial Retest Assessment

~MCAT Candidate Information and Tasks R1-R8 732 -739
~MCAT Performance Guidelines R1-R8 740 -765
DETAILS OF MCAT TRIAL RETEST ASSESSMENTS

141 [R1] Intravenous cannula insertion for antibiotic prophylaxis 733 741
142 [R2] Heartburn in a 35-year-old man 734 744
143 [R3] Spontaneous bruising and nosebleed in a 3-year-old boy 735 748

Vii

CIT PG
144 [R4] Nausea and vomiting in the first trimester in a 25-year-old
primigravida
736 750
145 [R5] Visual difficulties in a 50-year-old man
736 753
146 [R6] Cogniti ve state assessment of a 50-year-old barman
737 756
147 [R7] Jaundice in a 25-year-old man
738 760
148 [R8] Assessment of prominent leg veins in a 38-year-old woman
739 763
INTERACTIVE CLINICAL ASSESSMENT OTHER METHODS AND OSCE
MODIFICATIONS

767
Peter G Devitt and Heather G Alexander

149 Confusion and delirium after surgery in a 50-year-old man
771 773
150 Postoperati ve fever in a 45-year-old woman
151 The 4 station progressi ve OSCE
771
776
779
GLOSSARY OF TERMS AND ABBREVIATIONS

781
EPONYMS

790
APPENDICES
1. AMC Objecti ves of Medical Education

803
2. AMC Instructions to Standardised Patients and Clinical Examiners

806
3. MCC/AMC Clinical Task Categories; AMC Function/Process;
System/Region/Speciality; and Discipline classification

810
MCATs with full Domain listing and AMC Anthology Reference

814
MCATs by Discipline (Condition and page listings onl y)

843
MCATs by System/Region/Speciality (Condition and page listings onl y)

847
MCATs by Function/Process (Condition and page listings onl y)

856
Suggested Additional Groupings of MCATs for self-test trial assessments

862
Guidelines for further reading

863
EPILOGUE

867
INDEX

868


Viii
The AMC Multidisciplinary Clinical
Assessment Task (MCAT) Format
Heather G Alexander
The student is to collect and evaluate facts. The facts are locked up in the patient. To
the patient, therefore, the student must go.'
Abraham Flexner (1866-1959)
Medical Education, a Comparative Study
The MCAT is an integrated OSCE-style clinical examination where each candidate
proceeds through the same number of stations 16 stations in the full exam, 8 stations in
the retest.
CONTENT OF STATIONS
At each station, two minutes are allocated for preliminary reading outside the room. An
instruction sheet giving the candidate specific information and tasks required is provided.
This introduces the candidate to the consultation setting and clinical situation. It may also
include patient profile test results or an illustration. Specific tasks that the candidate will be
asked to perform are itemised. A duplicate copy of the instructions is provided in the
examination room.
This is followed by eight minutes performing the required task in a room
with a standardised patient. When the candidate first enters the room, the
observing examiner will check that the instructions for the station have
been read and will then introduce the candidate to the patient. The
examiner will then observe the performance and record the
candidate's performance on a tailored mark sheet. The standardised
patient may be a real patient or a simulated patient (role player) who
plays the role of either the patient or a relative. Doctor-patient
communication performance contributes to the assessment and requires
a well-trained role player. Where scenarios are based on physical
examination, the 'role player' may be a real patient.
The aims of the
station, the tasks
that candidates are
asked to perform,
the key issues and
assessment
domains defined
for the station are
all closely aligned.


031
FIGURE III.
History-taking
FIGURE iv.
Commencing the Physical Examination
The aims of the station, the tasks that candidates are asked to perform, the key issues
and assessment domains defined for the station are all closely aligned.
032
The MCAT scenarios developed for assessment purposes are designed to simulate closely real
life situations within medical consultations. These may be in a general practice setting, a hospital
emergency department, or a hospital inpatient or outpatient setting. Scenarios deal with different
phases of illnesses. Diagnostic scenarios include the diagnostic phases of history taking, physical
examination, and ordering and interpreting investigations. The management phases incorporate
patient explanation and education, advice and referral, therapeutics and preventive medicine,
clinical procedures and counselling. Scenarios are focused precisely so that the assessment
domains, key issues and critical errors are accurately related to the station aims and the tasks set
down in the candidate's instructions.
Members of the AMC clinical examination panel suggest MCAT clinical scenarios based on their
prevalence, seriousness, preventability and whether they can be simulated as real life situations
within the inherent time constraints. Scenarios are thoroughly reviewed and approved by the
multidisciplinary clinical panel prior to use. The current 16 or 8 station MCAT formats cover a
broad spectrum of skills in clinical medicine, psychiatry, surgery, obstetrics/gynaecology, and
paediatrics, including emergency, hospital and community practice medicine.
MCAT MARKING
In an MCAT, candidates are assessed at the level of a final year medical student, i.e. a doctor
about to commence an intern year (PGY1).
Mark sheets for examiner use.
The examiner scores the candidate's performance on a mark sheet which specifies the
assessment domains, key domains, and critical errors if appropriate. The assessment domains
match the tasks outlined on the instructions the candidates receive during the two minutes
preliminary reading.
The marking domains are identified from among a total of 14 covering:
approach to patient and responses to patient's questions;
patient counselling and education;
history-taking;
physical examination choice and technique;
physical examination accuracy;
choice of investigations;
interpretation of investigations;
diagnosis and differential diagnosis;
initial management plan;
explanation of clinical procedure;
performance of clinical procedure;
familiarity with test equipment;
commentary to examiner; and
answers to examiner's questions,
No single station is likely to have assessment in more than five of these domains.
Each domain has a 4-point marking scale:
Very satisfactory Clear pass
Satisfactory Pass
Unsatisfactory Fail
Very unsatisfactory Clear fail
033
An example mark sheet is included later with the example MCAT 001. (see page 44)
Critical errors are defined and derived from one or more of the key issues, when relevant. Not
all stations have critical errors. If the candidate makes a critical error the candidate is very likely to
fail that station, regardless of performance in other domains, unless performance in other
domains is outstanding and the critical error is deemed possibly related to lack of time or
misunderstanding of the task. MCAT performance is checked and reviewed by the Clinical Panel
of Examiners after each use in an examination. All details, particularly presence and definition of
critical errors, are reassessed and retained or modified in light of candidate performance and
examiner feedback.
Station failure would probably result from two or more 'unsatisfactory fail' assessments or one
'very unsatisfactory fail' assessment in a key issue domain, or from making a critical error in a
key issue domain.
After scoring each of the domains, the examiner will provide an overall (final) rating that is either
'Pass' or 'Fail' for each station.
All 16 MCAT scenarios are of equal weighting and for each scenario there are only two outcomes
pass or fail. Candidates must obtain a pass in 12 or more of the 16 stations, including a pass in
at least one paediatric and one obstetric/gynaecology station, to pass the MCAT as a whole.
Candidates scoring pass levels in nine or less of the 16 stations, or with failures in all three of the
paediatric or obstetric/gynaecology stations, fail the examination and must resit.
Candidates who pass 10 or 11 of the 16 stations (including a pass in at least one obstetric/
gynaecology station and one paediatric station) will be eligible for a pass/fail Retest Examination
of 8 stations. Retest candidates will be required to pass six or more of the eight retest stations to
pass the examination. Candidates scoring five or less passes will fail and be required to resit the
whole examination.
Heather G Alexander
J uly 2007

034
How to use this AMC Handbook of
Annotated MCATs
Vernon C Marshall
'In what may be called the natural method of teaching the student begins with the
patient, continues with the patient, and ends his studies with the patient, using books
and lectures as tools, as means to an end.'
Sir William Osier (1849-1919)
The MCAT self-test scenarios are arranged in groups under the principal categories and
domains tested. In each instance the reader is provided with a synopsis heading, outlining
the clinical problem/condition together with the information available to the candidate and
details of the task to be undertaken, exactly as this appears in the MCAT examination.
INSTRUCTIONS TO CANDIDATES
You may wish to attempt to complete the tasks in each of the major categories before
moving to the next group. If you would prefer to review tasks by system and region, or by
discipline, the appropriate groupings of these are listed in later pages. Page numbers of
individual MCATs are listed in the table of contents at the beginning of the book for easy
reference.
After reading carefully the information provided to you for each clinical scenario and the
required tasks, jot down how you will approach this consultation, how you will advise the
patient or relative of your findings and recommendations, and how you would structure
responses to queries from patient or examiner. Then turn the pages to check your
responses against the optimum Performance Guidelines, Examiner Instructions and
Commentaries. Note the station Aims, Key issues and Critical Errors outlined.
In this book the scenarios are grouped into five main categories. The groupings are to some
degree artificial in that communication skills are relevant to all scenarios. For example,
aspects of diagnosis, management, and patient counselling and education are frequently
combined to varying degree, but the groupings are arranged to emphasise and categorise
the principal domains even though most scenarios are assessed over multiple domains.
The five groupings below condense the total of 14 domain assessments into five categories
covering skills principally in:
1. Clinical Communication (C) with patient, relative and observer, and including a
number of domains: approach to patient, patient counselling/education, history-taking,
commentary to examiner, answers to patient's or examiner's questions, explanation of
procedure, case presentation and summary.
2. Clinical Diagnosis (D) includes history-taking, technique and accuracy of physical
examination, choice of investigations and their interpretation, diagnosis/differential
diagnosis.
3. Clinical Management (M) includes initial management plan, performance of
procedure/task, treatment and prevention of disease, clinical procedures.
4. Integrated Diagnosis and Management (D/M) includes clinical perspectives and
priorities, life-threatening emergencies, integrative reasoning skills and clinical
problem-solving.
5. Legal, Ethical and Organisational (LEO) includes scenarios where ethical and legal
issues are significant.
INTRODUCTORY GUIDELINES for candidates (see Table below)
are provided at the start of each of the main categories and their
domains.
After completing individual case scenarios you may find it helpful to
revise your knowledge of similar and linked conditions by referring
to appropriate clinical texts and references. The AMC Anthology of
Medical Conditions contains other self-test strategies for individual
conditions.
Try making up your own variations on the conditions tested, and
practise role playing and interactions with a colleague or in a group.
Once you are familiar with the mechanics and time constraints,
pace yourself through the trial examinations (one containing 16
stations and one containing 8 multi-disciplinary stations), and the
other suggested groupings provided later in the book, under
simulated examination conditions.
The Editorial Committee hopes you find the examples helpful and
extends its good wishes for a successful assessment.
The MCAT self-test
scenarios are arranged
in groups under the
principal categories
and domains tested. In
each instance the
reader is provided with
a synopsis heading,
outlining the clinical
problem/condition
together with the
information available to
the candidate and
details of the task to be
undertaken, exactly as
this appears in the
MCAT examination.
SCENARIO HEADINGS FOLLOWED IN THE AMC HANDBOOK OF CLINICAL
ASSESSMENT
The MCAT scenarios and performance guidelines are set out in a standardised sequence as
follows. Groups of self-test candidate information and tasks are arranged under principal
categories and domains tested.
Table 3 MCAT Introductory Guideline Scenario Headings
CONDITION
AND ID
NUMBER
A generic and non-diagnostic summary of the presenting symptom,
physical sign or investigation result in diagnostic-type cases, such as:
Assessment of acute abdominal pain in a 30-year-old woman.
Assessment of a vesicular rash in a 50-year-old man.
Review of liver function test results in a 50-year-old man with
jaundice.
The diagnosis or most likely diagnosis in management/counselling-type
cases, such as:
Management of shingles ('herpes zoster') in a 25-year-old
woman.
Counselling the relative of a patient after recent major surgery.
CANDIDATE
INFORMATIO
N AND
TASKS
Under this heading the background information and tasks are given
precisely as they appear in the MCAT examination.
Page references to the matching Performance Guidelines are given at
the foot of each Candidate Information and Tasks sheet.
YOUR TASKS ARE TO: Lists requested tasks for candidates.

035
036
Performance guidelines follow in similar category and domain groups linked to the preceding scenarios by ID
number and page reference.
PERFORMANCE GUIDELINES
CONDITION AND Princi pal category and assessment domains in detail; and
ID NUMBER classi ti cati on by function, system/region and discipline (see
Appendi x 3) are listed for each station just prior to the index.
AMC Anthol ogy of Medi cal Conditi ons reference is listed to aid further self-testing. The MCC/AMC
Clinical Task Category is also listed.
AIMS OF STATION A brief outline of station and assessment aims, matching the tasks. The
expected responses and levels of performance required to complete the
tasks successfully are outlined in the examiner instructions and
commentaries.
EXAMINER
INSTRUCTIONS
These provide the following:
Instructions from examiner to standardised pati ent
Candidate information and tasks and role player instructions are detailed
and provided to examiners and standardised patients so that there is
standardised behaviour across multiple patients. Cues assist in directing the
consultation pathway. The instructions are set out using lay terminology to
maintain realism, and outline:
Cli ni cal setti ng hospital emergency department, hospital ward
or outpatient department, primary care facility, community practice
office consultation.
Cli ni cal situati on description of illness and symptoms and
phase of the consultation.
Patient profil e age and gender, past history, family history,
habitus, as relevant to the case.
Opening statement one sentence provided as the patient's
opening gambit.
How to pl ay the role advice on further responses, posture,
gestures, affect, mood and ways to react to the doctor, including
where the task is a physical examination.
Questions to be asked by pati ent/role pl ayer set down in a loose
priority and which will depend on whether these have already been covered
by the doctor/candidate. Any examiner questions or prompts to the
candidate are also outlined, with the required responses.
EXPECTATIONS
OF CANDIDATE
PERFORMANCE
These are clarified for the examiner and match the tasks and the
domains.
KEY ISSUES These are selected from the assessment domains and expectations of
candidate performance for each case and highlighted accordingly.
CRITICAL ERROR(S) These list significant errors likely to lead to a fail performance.
COMMENTARY This discusses and comments further on the condition, highlighting
performance standards and common errors.

EXAMPLE CASE SCENARIO:
The following case scenario exemplifies the formatting for a combined Diagnosis and
Management MCAT.

037
MCAT FORMAT EXAMPLE:

Sample - Condition 001
Candidate Information and Tasks
Condition 001
A cut to the thumb of a 22-year-old man
You are the Hospital Medical Officer (HMO) in a hospital Emergency Department.
The patient injured his left thumb at work an hour ago. He is aged 22 years and works
as an orchard labourer and fruit picker. He is right handed. He was pruning fruit trees
today and the pruning knife slipped and he cut his left thumb. He was wearing cotton
gloves. The knife cut through the glove and cut the thumb as shown in the illustration
below. Bleeding was minor and controlled by a pressure dressing, which has been
removed for examination.
The wound appears as a linear knife cut as shown, the edges of which have sealed
after the initial bleeding which has now stopped.
YOUR TASKS ARE TO:
Examine him and assess the injury.
Explain to him the nature of the injury and your recommended management.
You may ask other questions of the patient as you proceed with the examination and
explanation.
Near the end of the eight-minute time allotted for your task, the examiner will ask
you one or two questions.
CONDITION 001. FIGURE 1.
Knife wound to the left thumb
The Performance Guidelines for Condition 001 can be found on page 40
038
Sample-Condition 001
Candidate Information and Tasks
CANDIDATE ADVICE
You should:
Prepare and document your responses and how you would approach this task. Test
yourself thoroughly after reading the MCAT Candidate Information and Tasks, before
proceeding to read the performance guidelines, examiner and patient instructions and
commentary which you will find on subsequent pages.
Follow a similar process for the other MCATs. The best way to develop proficiency in an
MCAT assessment is to work in pairs or as a group. Your colleague reads the performance
guidelines and plays the patient/relative, while you read the candidate information and
perform the tasks, while another group member takes the role of examiner/observer.
SUMMARY OF STUDY TASKS
Read the candidate information and task(s), preferabl y working with a colleague or
group.
Formulate and document a logical approach for responding to and solving the consultative
problem given.
Then read the performance guidelines that follow, and note the aims of the station,
expectations of your knowledge and performance, key issues and critical errors
and other points raised in the commentary. Check for any deficiencies in your performance.
Reread the introductions to the section in which the MCAT appears.
For this MCAT about a thumb wound, revise your knowledge of applied surface anatomy
relevant to wounds giving risk to underlying structures and how you should check for local
and distal effects of injury. Construct alternative scenarios for other wounds and self-test
yourself on these (for example, injuries to radial nerve in the arm, common peroneal nerve
in the leg). Revise the Anthology scenarios 113, 113H, 113J and 113K and complete the
self-test exercises.
Reinforce your understanding of the condition by completing other self-assessment tasks
(for example from the AMC Anthology of Medical Conditions) and construct at least one
other related task for solving.
Finally, one complete MCAT 16 station assessment and one complete MCAT eight station
assessment are provided later in the book as examples of whole examinations for trial.
039
Sample-Condition 001
Candidate Information and Tasks
Additional groupings of MCATs into further self-test trial examinations are also suggested at
the end of the book. MCATs are also grouped into one of the principal disciplines of
medicine, obstetrics/gynaecology, paediatrics, psychiatry, surgery if you wish to use the
book in this way. MCATs are similarly grouped into the relevant function and process and
into system/region/specialty. For these latter groups, MCATs are often listed more than
once when they cover more than one system or function.
Pace and test yourself through these.
Keep practising within a group of your peers until fully familiar with the routine. We hope
that you will find the self-discipline and requirements to adhere to logical clinical reasoning
pathways in approaching the wide range of clinical problems selected for this book will
stand you in good stead, not just for assessment examinations, but throughout your
subsequent career.
Vernon C Marshall

040
Sample - Condition 001 MCAT FORMAT
EXAMPLE:
Performance Gui delines
Condition 001
A cut to the thumb of a 22-year-old man
AIMS OF STATION
To assess the candidate's ability to use clinical reasoning skills to diagnose and
manage important injuries associated with skin wounds. In this instance, the knife cut
has severed the two extensor tendons to the thumb.
EXAMINER INSTRUCTIONS
The examiner will have instructed the patient as follows:
The examiner will draw the linear cut with a red marking pen on the role-playing
patient, and show the patient how to respond to requests to bend his thumb and
testing of sensation as follows:
You have not yet noticed and should not volunteer any information about limitation
of thumb movement, until specifically asked to extend each of the two end joints,
which you are unable to do. Sensation is normal.
You had a tetanus booster shot about a year ago for a leg graze and were
immunised against tetanus as a child.
Opening statement:
'Will it be okay for me to go back to work tomorrow with a dressing over it now it's
stopped bleeding, Doc?'
Other questions to ask:
If the candidate/doctor diagnoses tendon injury with normal sensation, you will
accept the recommendations for operation, and should ask about the operation
'Will I need an anaesthetic?'
(Appropriate answer Yes: regional block or general anaesthesia).
If no mention of a tetanus prophylaxis or antibiotics is made during the interview
you will subsequently ask
'Will I need another tetanus shot?'
(A booster dose of toxoid would be appropriate).
Examiner's questions to candidate:
At the end of 6-7 minutes, if the candidate has identified that a tendon injury has
occurred, the examiner should ask:
'What are the names of the injured tendons?'
(Extensor pollicis longus [EPL] and extensor pollicis brevis [EPB])
and
041
Sample - Condition 001
Performance Guidelines
'Which joint does each tendon act upon?'
(Interphalangeal [IP] and metacarpophalangeal [MP] joint respectively).
If no tendon injury has been identified just ask:
'If antibiotics are to be given, what would you choose?'
(Broad spectrum cover such as one dose of amoxycillin, cephalosporin or other
antibiotic).
EXPECTATIONS OF CANDIDATE PERFORMANCE
Cuts and stabs of various types commonly present to emergency departments.
Attending doctors must be aware of the anatomy of deeper structures at risk from
injuries at specific sites and the methods of diagnosing such injuries.
Diagnosis of injury to the two main long thumb tendons and recognition of the
treatment requirements for primary surgical repair in this 'tidy' (clean contaminated)
wound.
Explanation of treatment would optimally advise preparation for early surgery
using local (field) block or general anaesthesia.
Antibiotic and tetanus prophylaxis would be appropriate.
KEY ISSUES
Ability to identify deeper tendon injuries resulting from stabs or cuts.
Failure to appreciate that the whole thumb extensor mechanism (involving two
tendons) has been damaged would comprise a fail (unsatisfactory) in the domains
of examination technique and diagnosis.
Failure to name the tendons correctly would not necessarily be a fail performance,
providing the presence of tendon injury was diagnosed and appropriate advice
given in other areas. Failure to mention antibiotic or tetanus prophylaxis would
be unsatisfactory, but would not be considered a critical error in the presence of
a 'tidy' recent wound; such omission would most likely be corrected with
subsequent specialist referral for surgery and anaesthesia.
CRITICAL ERROR
Failure to test and identify the injury to the extensor tendons would comprise a clear
and irremediable fail for this station at a very unsatisfactory level.
042
Sample - Condition 001
Performance Guidelines
COMMENTARY
The knife cut has severed the two extensor tendons to the left thumb {extensor pollicis
brew's and extensor pollicis longus, from radial to ulnar side). These tendons form the
margins of the anatomical snuff box as illustrated. The tendons have been severed at
the knuckle level of the metacarpophalangeal joint. The patient has no obvious thumb
deformity but is unable actively to extend either the metacarpophalangeal (MP) joint or
the interphalangeal (IP) joint of the thumb. The digital cutaneous nerves have not
been cut and distal sensation is normal apart from tenderness around the cut.
Extension of the joints of the thumb occurs from the actions of:
Extensor pollicis longus (EPL) the ulnar-sided of the two thumb tendons running
on the dorsal aspect of the thumb. The long tendon of EPL runs obliquely across
the back of the hand after angulating around the tubercle of the radius (Lister
tubercle) before inserting into the base of the distal phalanx. EPL is the prime
mover and sole extensor of the terminal (interphalangeal) joint. By passing across
the metacarpophalangeal (MP) and carpometacarpal (CM) joints of the thumb.
EPL can also act as an accessory extensor of these joints. EPL, like other
superficial tendons, may be injured by cuts and penetrating injuries.
Extensor pollicis brevis (EPB) is the lateral of the two thumb extensors. EPB runs
in the same synovial sheath as the tendon of abductor pollicis longus on the lateral
surface of the radius and continues over the dorsal shaft of the metacarpal to
insert into the base of the proximal phalanx. EPB is the prime mover in extension of
the MP joint and an accessory extensor of the CM joint. Cuts around the knuckle of
the metacarpophalangeal joint are likely to sever one or both tendons. In this
patient, both EPL and EPB have been severed.
Abductor pollicis longus (APL). This stout tendon, often multiple or ridged like a
stalk of celery, inserts dorsolateral^into the base of the thumb metacarpal. APL is
the prime mover of radial abduction and extension of the thumb at the carpo-
metacarpal joint, separating the thumb from the other digits in the plane of the palm.
In this patient, radial abduction will be unaffected as APL has not been injured.
Sample - Condition 001
Performance Guidelines

CONDITION 001. FIGURE 2.
Normal Anatomy Left hand and thumb
The Examiner mark sheet for MCAT 001 follows.
043
044
Candidate ID card sighted Very Satisfactory - PASS Satisfactory - PASS

Unsatisfactory - FAIL Very Unsatisfactory - FAIL
*- KEY ISSUE Covers all essential Minor technical Candidate displays
Choice & Technique of Examination, aspects competently


faults but

one or more of the



Serious errors or omissions in
technique.
Organisation and Sequence - minimal errors or

examination following: CRITICAL ERROR?
Did the candidate carry out an omissions. completed reasonably. - significant omissions
appropriate focused and relevant - significant errors of
examination as per examiner technique
instructions?

- poor technique
- KEY ISSUE Identified most or Minor errors in One or more
Accuracy of Examination all findings


findings.

significant errors in

Serious errors or omissions in


findings: reported findings not
Did the candidate identify the accurately. findings. consistent with physical signs
physical findings accurately as

CRITICAL ERROR?
per examiner instructions?

^ KEY ISSUE Covered all essential Minor omissions Significant errors in Diagnosis not given. Serious
Diagnosis/Differential Diagnosis aspects competently


or errors in

explanations of

omissions or errors in
Did the candidate formulate and - minimal or no errors or explanations of findings. findings. Wrong interpretations of findings.
describe an appropriate diagnosis/ omissions. Logical, clear, Diagnosis and differential interpretations of findings. Clinical reasoning and diagnostic
differential diagnosis as per examiner well organised. diagnosis appropriate to Unclear and poorly skills markedly deficient. Very poor
instructions? the case even if not organised. Diagnosis organisation. Wrong diagnosis could

completely accurate.

inappropriate to the ;ase.
result in serious harm to the patient.
CRITICAL ERROR?
-. KEY ISSUE Covered all essential Minor errors but did Significant errors Serious errors or omissions.
Initial Management Plan aspects competently


not interfere with an

which did interfere

Inappropriate management
Did the candidate formulate and - minimal errors or adequate initial with an adequate and/or management proposed is
describe an appropriate initial omissions. Optimal management plan. management plan. potentially harmful to patient.
management plan as per examiner
instructions?
management plan.

CRITICAL ERROR?
Answers to Questions Covered all aspects Minor errors in Significant errors in

Serious errors or omissions in
Answers to questions asked by completely, minimal


answers to

answers to questions the answers given, or complete


examiner? errors or omissions.

questions.

indicating lack of
knowledge/expertise
these areas.
in
unfamiliarity with the subjects asked.
CRITICAL ERROR?

T X
U)
3
<
-i
T CD
3
A i 1
(
^+
1 n
1 A
1
II

o
X-
[11
rr
nX
ZS
cp
o
Tl
n


1
O
n
i
in
O
o o
GO
O

OVERALL RATING FOR THIS CANDIDATE FOR THIS STATION: PASS FAIL
Clinical Communication (C)

1-A: Communication, Counselling &
Patient Education
Alan T Rose
'Oh, that's your doctor, is it? What sort of a doctor is he?'
Well, I don't know much about his ability, but he's got a very good bedside manner!'
George du Maurier, Punch Cartoon, 15 March 1884.
1-A COMMUNICATION, COUNSELLING AND PATIENT EDUCATION
Communication is the exchange of messages and thoughts by speech, signals or writing.
Communication skills are employed to ensure that exchanges are readily and clearly
understood. Exchanges involve the sharing of information, ideas, emotions, and empathy.
Communication is the foundation on which medical consulting takes shape, supplemented
by the practitioner's skill in physical examination and diagnostic reasoning. Failure of
communication is an important contributor to clinical situations of perceived malpractice
and is the most important factor in a high proportion of medicolegal actions.
Most medical consultations and activities require the doctor and
patient to communicate rationally and effectively with each other.
045
Exceptions are when the patient is an infant or is intellectually
handicapped communication is then with a relative or carer
or when the patient is unconscious (including when
anaesthetised) or suffering from certain psychotic states, or
when doctor and patient do not share the same language.
Communication requires special techniques with patients who
are blind, mute, aphonic or aphasic. Impaired hearing may
affect either patient or doctor. The role of the interpreter, when
required, is also critical. Involvement of third parties (such as
relatives, friends, or outside agencies) requires the patient's
consent. In these situations the patient's legal right to
The AMC examination
process places
considerable emphasis
on assessment of
effective communication
between candidate and
patient during clinical
consultations, during
discussions with
relatives, and during
case presentations and
commentaries.
confidentiality and privacy must be respected.
The AMC examination process places considerable emphasis on assessment of effective
communication between candidate and patient during clinical consultations, during
discussions with relatives, and during case presentations and commentaries. Written
communications are important for letters of referral and discharge summaries, but are less
readily assessed within the current AMC format.
Verbal communication depends on a mutual understanding of the language being used
and the way it is articulated. This includes pronunciation, auditory level, speed, tone and
the unique voice qualities and cadences of the speaker. The AMC examination assesses
communication in the English language in a medical and clinical context. English is not the
first language for many IMGs, but all IMGs are required to have adequate clinical communi-
cation skills in English by medical registration boards.

Nonverbal communication (such as facial expression, posture, gesturing, silence, and
emoting) by either doctor or patient, also conveys messages as well as influencing the
understanding of what is being said and its emotional context.
Effective verbal and nonverbal communication in medical practice facilitates the establish-
ment of empathy and rapport, trust and confidence, mutual understanding, education about
the clinical condition, and satisfactory compliance with advice and treatment.
The term 'bedside manner', used to describe a doctor's communication skills, was first
used in a London 'Punch' cartoon by George du Maurier.
Wide variations in clinical communication skills occur because of each individual doctor's
inherent personality traits and individual approach to patients. These can be modified and
improved by education and self awareness, so that time is saved and any frustrations felt
by the doctor or dissatisfaction by the patient are minimised.
Similarly, the cultural characteristics of the patient (and of the doctor) can profoundly affect
the quality of doctor-patient communication. Doctors practising in Australia require
multicultural competence across all fields of medicine. Special care is required in the case of
Aboriginal and Torres Strait Islanders, and for culturally and linguistically diverse groups.
Communication skills, although important, are not sufficient. Good communication skills
must be accompanied by sound clinical skills, attitudes and professional behaviour. The
(fortunately) rare physician serial criminal murderers have usually been superb
communicators.
Other personal factors can interfere with the doctor's use of communication skills. Many
clinical realities are unpleasant to both patient and doctor. If the doctor retreats behind a
professional fagade of a stilted and portentous style of speech, or adopts a pompous or
pretentious attitude, or one interpreted as such, the patient can be daunted from further
enquiry. Rejection by the doctor of a patient's attitude or behaviour engenders lack of
understanding and trust. Value judgements of the doctor are best avoided or concealed.
Care and compassion should be evident but not forced or obtrusive. This is especially
important when treating users of illicit drugs or dependent alcoholics. Mention should also
be made of the so-called 'difficult patient' whose underlying but sometimes unrecognised
personality disorder reduces or eliminates the effectiveness of the communication skills
described below.
The application of communication skills
Effective communication is of most value when taking a history, providing patient education
about the condition diagnosed, giving advice about treatment, counselling patients, and
when discussing the patient's illness with anxious relatives or friends.
History-taking
There are two main methods. Transition from one to the other occurs depending on the
clinical setting and progress of this phase of the consultation. The aim is to define the
presenting problem to a point where the diagnosis moves from possible to probable to
definite.
Firstly, the nondirective or 'open-ended' approach: this allows and encourages patients
to outline the problem ('tell their story') in their own way while the doctor listens with
little interruption. Although apparent irrelevancies may be brought up, this method gives
an opportunity for patients to reveal concerns initially unstated. These concerns may
explain their real reason for attending and why they have come at this particular time.
046

This approach is most useful when the patient is consulting the doctor for the first time
about undifferentiated symptoms such as 'tiredness', 'bloating', 'being run down', 'sleep-
lessness', or 'requesting a checkup'.
Secondly, the directive, closed or interrogative approach: this confines the patient to
the doctor's agenda. This is appropriate in emergencies and, if the problem has already
been well defined, when a patient's progress is being reviewed in followup. The doctor
takes early control of the interview by the use of a series of direct or closed questions.
This approach is well summarised by the traditional term history 'taking'. The directive
approach risks the omission of significant information, particularly from a patient who is
anxious, reticent, embarrassed, or has feelings of shame or guilt.
Nonverbal communication is very useful in the nondirective approach and can replace some
parts of the verbal component, for example, expressing surprise by facial expression.
Transition from nondirective to directive mode occurs when the doctor begins to ask direct
or closed questions, but the two modes are usually phased or overlap. The type of question
used will change the direction of the interview. Using questions about pain as examples:
open ended 'Tell me about the pain?'
direct Where Is the pain?'
closed 'Have you had this pain before?'
leading 'The pain isn't severe?'
A series of direct or closed questions is usually necessary to complete the history regarding
occupation, past and family illness, domestic habits, medications, allergies and sensitivities,
where relevant.
Facilitation is also a valuable nondirective tool. Facilitation uses nonspecific inviting or
encouraging remarks, for example, 'go on', 'I'm listening carefully', 'tell me more about the
pain', 'anything else?', 7 see', or 'uh uh'.
Listening is an essential basis of communication. Adequate time must be given and the
doctor's nonverbal behaviour should indicate to the patient that the listening is attentive.
Note-taking should be discreet and avoid distraction. The use of a personal computer by the
doctor in the consulting room while taking a history requires, more than ever, that the doctor's
nonverbal behaviour assures patients that they are being 'listened to'. Silence is not the same
as listening. Silence may be the best response when there is an emotional or confrontational
component in the consultation.
Confrontation can defuse an issue ('Wouldn't we progress better if we leave aside for the
moment your previous dissatisfaction with treatment and try to work out how best to fix
things now?') but should be used with care. Frequently initial aggression or anger from the
patient is better deflected in the first instance.
Summarising briefly what the doctor believes the patient has said so far is often useful to
confirm that mutual understanding at that point in the interview is present.
As previously noted, these techniques are modified by the personality and instinctive
behaviour of the doctor, who may not always be fully aware of their effect on a patient. The
perceptive doctor attempts always to appreciate the patient's perspectives as well as the
doctor's own.
In summary, the following guidelines apply to history 'taking' from a patient presenting with
a nonurgent diagnostic problem.
047

Following the formalities (or informalities) of introduction, the doctor should:
begin with an open-ended approach;
listen carefully and attentively;
use facilitation and open-ended questions to encourage the patient;
limit direct questions early in the interview;
use indirect and reflected questions as appropriate;
use direct and closed questions to take control at an appropriate time;
take note of any display of emotion by the patient and respond appropriately;
briefly summarise what is being learnt or understood (or not) from the patient;
gradually increase control of the interview as it proceeds;
use nonverbal communication, to supplement verbal communication; and
be alert to the patient's nonverbal behaviour.
With loquacious and garrulous patients, transition from open-ended to direct questioning
needs to proceed expeditiously but tactfully because of time constraints. In many of the
MCAT clinical scenarios, direct questioning is essential to enable a focused history
to be taken within the time period available.
Communication skills are especially important when the patient has concerns other than
those expressed in the first statement to the doctor. Such hidden concerns may include
fears of cancer, heart disease, stroke, blindness, sexually transmitted infection, work
capacity, relationships, serious illness in an unborn child or infant, dissatisfaction with
treatment and feelings of grief or guilt to mention only a few.
Patient education
Once the diagnosis has been made, it should be stated to the patient using both medical
and lay terminology appropriate to the patient's understanding. Initial reassurance should be
given when appropriate ('I'm pleased to tell you that the biopsy showed no evidence of
cancer'). Failure to do so may allow patient anxiety to block the reception of other information.
Reassurance may be the only therapy that is necessary unnecessary prescribing may
follow if the doctor has not understood this basic need of the patient for reassurance.
The patient's knowledge and understanding of the condition should then be established so
that education can be pitched at the correct level. This includes the correction of incorrect
beliefs and responses to the patient's questions. The use of a chart, diagram or of printed
notes, may be additionally helpful.
As previously stated, anxiety may reduce the efficiency of absorbing and understanding
information. Patients will also differ in their interest and tolerance of information about their
ailments. Most will want to know if the condition is serious, whether it can be treated, when
they will recover and resume normal activities, and what forms treatment will take, even
though these questions may not be asked directly. Overloading the patient with information
can be counterproductive and have unwanted effects including the creation of pessimism
and anxiety. Further information can be added at a subsequent consultation. Handing out
previously prepared written material is helpful for most patients but is no substitute for
verbal education from the doctor.
Adverse information should be given in such a way so as to not destroy hope, and also to
assure the patient of the doctor's continuing support.
An adequate level of understanding is the only basis on which patients can share the
responsibility of decision-making about treatment and give informed consent. Many patients
048

however still prefer to leave all decisions to the doctor. If thought necessary, confrontation
can be used to bring to the attention of patients their own responsibilities in aiding effective
treatment.
Giving advice about treatment
The wide ranges of treatments, which may be indicated, include giving simple advice,
prescribing, minor or major procedures, the use of allied health professional services,
referral to another doctor or to a hospital service, and counselling. All require the use of
communication skills, either verbal, nonverbal, or written. When counselling is the main
component of treatment, additional special skills are required.
The duration of intended treatment should be advised to the patient and opportunities for
giving preventive advice taken whenever appropriate. This may involve other members of
the patient's family.
Prescribing should be supported by a statement of the name of the medication, which may
also be written down for the patient, the dose and timing, expected effects including the
most important side effects, and significant adverse reactions.
Patients will also wish to know general details about any procedure which has been
recommended, and what to expect when a referral has been made.
Patients should not be left wondering what happens next. Certainty about followup
arrangements, particularly after investigations or referral, should be made quite clear.
Neglect of these can have medicolegal consequences.
Patient Counselling
Counselling is a term widely used in the community to describe the provision of support to
individuals or groups who are experiencing significant emotional stress. Counsellors,
usually with special training in psychological skills, attend survivors or observers of
accidents and disasters and provide support.
Doctors frequently need to give 'bad news' to patients and relatives, and are involved with
bereavement and grieving responses. Allowing patients and relatives to work through their
feelings by means of facilitation and attentive listening, and providing support and
reassurance whenever possible, with unforced compassion and sympathetic
understanding, is required every day in clinical practice.
However, when 'patient counselling' is used in treatment, there are two types: directive and
nondirective. As with other communication skills these may overlap or be used in an
integrated way. However, one type usually predominates according to the clinical situation.
Directive counselling is when straightforward advice or instruction are given to the patient.
In contrast, nondirective counselling is a special communication skill which involves more
than patient education or giving advice about treatment. Its objective is that the patient,
instead of the doctor, is the final decision-maker and shares or accepts, in part or fully,
responsibility for the subsequent course and outcome of the problem. Nondirective
counselling begins with an accurate definition of the problem using the skills outlined under
nondirective history-taking, particularly listening and the use of silence. The doctor then
provides education about the problem including possible outcomes that are likely to follow
alternative forms of behaviour or non-compliance by the patient. Patients are then asked
what course they intend to follow, based on the options that have been discussed. The
process may occur rapidly or may take more than one consultation to work through. It is
essential for the doctor to maintain a nonjudgmental attitude throughout the process.
049

This skill is most useful in dealing with difficult behavioural problems such as smoking,
heavy drinking, eating disorders, drug use, marital problems and when dealing with
anxious parents.
When the use of addictive substances is involved, counselling has shortcomings as do
other techniques, but is always worth using results can be surprisingly effective.
Assessing communication skills
Communication skills are assessed continuously throughout the whole of every case
scenario and consultation. Other domains are usually assessed in sequential segments
(for example physical examination follows history, diagnosis follows investigations, and so
on). Particular communication skills are also defined within the domains in current use in
the AMC examination. These are:
Approach to patient
Empathy, comfort, consideration,
Explanation, using language that the patient understands (no jargon),
Checking for patient understanding,
Answering patient's, parent's or relative's questions, and
Obtaining verbal consent to proceed.
History
Taking an appropriately focused medical history.
Commentary to examiner
Describing the findings of physical examination with appropriate commentary; and
Presenting a case analysis in summary to the examiner.
Diagnosis/differential diagnosis
Describing an appropriate diagnosis/differential diagnosis plan.
Initial management plan
Describing an appropriate initial management plan.
Patient counselling/education
Educating the patient/relative/carer about the condition; and
Giving appropriate counselling.
Explanation of procedure
Explaining a procedure and its implications to the patient.
Answering questions asked by the patient or examiner
Most scenarios include cued questions to the doctor by the patient, parent or relative; and
Some scenarios have specific prompts or questions from the examiner.
Each of the above domains (plus others such as technique and accuracy of examination -
14 in all) attracts separate assessment by the examiner. The number of domains being
assessed in any individual MCAT rarely totals more than five. Communication skills may
affect performance in all domains. Experienced examiners, who are themselves skilled in
communication, have no difficulty in integrating communication components with those of
knowledge and attitudes and other clinical skills.
050
051
Communication, Counselling
& Patient Education
Conclusion
The effectiveness of virtually all consultations is enhanced by the doctor's understanding
and use of communication skills. The establishment of trust and confidence, empathy and
rapport, diagnostic precision, appropriate prescribing, patient education and
understanding, patient compliance, and self-help lifestyle modifications are all facilitated
when doctor and patient understand each other to an optimal level, as a result of the proper
application of communication skills.
Alan T Rose

1-A Communication, Counselling and Patient Education
Candidate Information and Tasks
MCAT 002-021
2 Advice on breastfeeding versus bottle-feeding for a 28-year-old pregnant woman
3 Advice on neonatal circumcision for a couple expecting their first child
4 Suspected hearing impairment in a 10-month-old child
5 Counselling a family after sudden infant death syndrome (SIDS)
6 Hair loss in a 38-year-old man
7 An unusual feeling in the throat in a 30-year-old man
8 Pain in the testis following mumps in a 25-year-old man
9 Contraceptive advice for a 24-year-old woman
10 Rape of a 20-year-old woman
11 Cancer of the colon in a 60-year-old man
12 Thalassaemia minor in a 22-year-old woman
13 Pre-pregnancy advice to a 28-year-old woman with previous thromboembolism
14 Pre-pregnancy advice to a 24-year-old woman with Type 1 diabetes mellitus
15 An anencephalic fetus diagnosed at 18 weeks gestation in a 25-year-old primigravida
16 A duodenal ulcer found on endoscopy in a 65-year-old man
17 Advice on autologous blood transfusion to a 55-year-old man awaiting elective surgery
18 Advice on stopping smoking to a 30-year-old man
19 Excessive alcohol consumption in a 45-year-old man
20 Type 1 diabetes mellitus in a 9-year-old boy
21 Request for vasectomy from a 36-year-old man
052
053
002-003
Candidate Information and Tasks
Condition 002
Advice on breastfeeding versus bottle-feeding for a 28-year-old
pregnant woman
CANDIDATE INFORMATION AND TASKS
You are a Hospital Medical Officer (HMO) in an antenatal clinic, seeing a 28-year-old
woman for her antenatal visit at 35 weeks of gestation. She wants to discuss infant feeding
with you. She has heard a lot about the benefits of breastfeeding, but her mother told her
recently that babies grow better with formula feeds. She is uncertain whether she should
breastfeed or formula-feed her baby.
YOUR TASKS ARE TO:
Discuss the advantages and disadvantages of breast-feeding and formula-feeding
with her.
Outline the steps involved in safe formula-feeding.
The Performance Guidelines for Condition 002 can be found on page 69
Condition 003
Advice on neonatal circumcision for a couple expecting their first child
CANDIDATE INFORMATION AND TASKS
A young couple, the wife pregnant with their first child, have come to see you in a general
practice to discuss with you the place of routine neonatal circumcision if their baby is a boy.
YOUR TASK IS TO: ]
Discuss with the couple the perceived risks and benefits of this procedure. J
The Performance Guidelines for Condition 003 can be found on page 72
054
Candidate Information and Tasks
Condition 004
Suspected hearing impairment in a 10-month-old child
( CANDIDATE INFORMATION AND TASKS )
You are working in a community health centre. Your next patient is a 10-month-old female
infant, baby Helena, seen with her mother, who has been referred by the local child health
nurse. The pregnancy and delivery were normal. The child presented to the nurse six
weeks ago for review and general screening including hearing. The nurse was concerned
that the baby has a hearing problem and wanted her checked by a doctor. The child's
parents have never had cause to worry about her hearing. She is the third child in a healthy
family and has been well, apart from a few upper respiratory infections.
She is crawling, does not walk yet, but pulls herself up to standing beside a small table.
Initial examination findings
A busy infant girl who objects to being restrained by her parent. She babbles during
assessment. Otoscopic examination is normal. No abnormal physical signs are present on
general examination. The parents are puzzled at the need for referral and seek information
about further investigation and management.
YOUR TASKS ARE TO:
Ask the parent for additional relevant and focused history.
Counsel the parent after you have obtained a further history.
Explain possible causes of any suspected hearing loss to the parent.
Discuss your plan of management with the parent.
The Performance Guidelines for Condition 004 can be found on page 75

Condition 005
Counselling a family after sudden infant death syndrome (SIDS)
CANDIDATE INFORMATION AND TASKS 1
You work in a general practice. You are counselling the family of a four-month-old male
infant who was rushed to the Emergency Department of the local hospital the day before
but was dead on arrival. The provisional diagnosis is sudden infant death syndrome (SIDS)
and the baby (Andrew) is to have a Coronial autopsy.
You had seen him for the first time two months previously, with his single mother, when he
was thriving and developing normally and had commenced immunisations. Two days
before his death you saw him again, this time with mild upper respiratory snuffles which
were causing minor difficulties with breastfeeding. However, over the next two days he
apparently improved, and his mother had advised you that he appeared normal and fed
well from the breast just prior to his death. You are unaware of any suspicious circum-
stances surrounding the death.
The family members have attended to seek details of why the baby died and why an
autopsy is necessary. The spokesperson for the group is the mother's sister, the aunt of the
infant. The mother is also present, but is too distressed to ask any questions herself.
YOUR TASKS ARE TO:
Answer the questions of the aunt relating to the death of the infant.
Counsel the aunt and family.
The Performance Guidelines for Condition 005 can be found on page 77
055
Candidate Information and Tasks
Condition 006
Hair loss in a 38-year-old man
CANDIDATE INFORMATION AND TASKS
You are a Hospital Medical Officer (HMO) working in a primary care clinic attached to a
teaching hospital. This 38-year-old male newsagent has just consulted you about recent
(2-3 weeks) hair loss from the scalp. One eyebrow is also affected. He is otherwise well
with no significant past or family history. The patient is very concerned about possible future
progression and wishes to ask you about the diagnosis and possible treatments. You have
completed an examination of the scalp. The findings are as depicted below.
YOUR TASKS ARE TO:
Discuss the condition with the patient.
Advise him about treatment.

CONDITION 006. FIGURES 1 AND 2.
The Performance Guidelines for Condition 006 can be found on page 79
056

Condition 007
An unusual feeling in the throat in a 30-year-old man
CANDIDATE INFORMATION AND TASKS
You are working in a general practice. Your next patient is a 30-year-old man who is
consulting you about a throat problem.
He is a previous patient of the clinic, is married with two children, parents and his siblings
are well. He smokes 10-15 cigarettes daily and takes 2-3 alcoholic drinks only at
weekends. He had a vasectomy two years ago and has had no serious illnesses.
YOUR TASKS ARE TO:
Take a focused history about his throat problem. The examiner will then give you the
examination findings.
Discuss the most likely causes of the problem and its nature with the patient.
Discuss whether any investigations are necessary and if so, what is most likely to be
found. .
The Performance Guidelines for Condition 007 can be found on page 81
057
058
008-009
Candidate Information and Tasks
Condition 008
Pain in the testis following mumps in a 25-year-old man
CANDIDATE INFORMATION AND TASKS
Your next patient in a general practice is a 25-year-old man who consulted you five days ago
because of painful swellings on both sides of his face associated with fever and malaise You
made a confident diagnosis of mumps.
You had previously diagnosed mumps in the patient's five-year-old son, a little less than three
weeks beforehand. The son has now fully recovered. The patient's other child, aged three years,
is well. You are aware that the couple have contemplated having another child.
You advised the patient to rest at home, suggested paracetamol as an analgesic, and asked him
to see you in a few days time before returning to work. The patient has come to see you today
because of a relapse of his fever associated with the onset of severe pain in his left testis.
You have found the left testis to be swollen to twice the size of the right one. It is very tender. The
right testis feels normal. The patient has a temperature of 38.4 C. His face is slightly swollen.
Apart from a tachycardia, there are no other abnormal clinical signs.
YOUR TASKS ARE TO:
Advise the patient of the diagnosis.
Advise him about treatment and prognosis.
Answer any questions asked by the patient.
The Performance Guidelines for Condition 008 can be found on page 84
Condition 009
Contraceptive advice for a 24-year-old woman
CANDIDATE INFORMATION AND TASKS
You are working in a general practice. A 24-year-old woman has come to see you for advice as to
the most appropriate pill she should go on for contraception for the next two to three years. She
knows that various types of pills are available and wants to know how to decide which is the most
appropriate pill for her.
YOUR TASKS ARE TO:
Take a further relevant and focused history.
Ask the examiner about findings you wish to elicit on general and gynaecological
examination.
Advise the patient of the appropriateness of oral contraceptive pill (OCP) therapy, which pill
should be given, and how it should be administered.
The Performance Guidelines for Condition 009 can be found on page 87

Condition 010
Rape of a 20-year-old woman
CANDIDATE INFORMATION AND TASKS J
You are a Hospital Medical Officer (HMO) in the Emergency Department of a metropolitan
general hospital. Your patient is a 20-year-old university student who is brought to the
Emergency Department of the hospital because she was raped by a man that she met at a
disco and who offered her a lift home. The rape occurred six hours ago after he had
stopped the car in an undeveloped area. She has decided not to involve the police as the
person concerned is known to her family. She has had no previous operations or illnesses
and no pregnancies.
YOUR TASKS ARE TO:
Take any further relevant history you require.
Ask the examiner about appropriate findings likely to be evident on initial general and
gynaecological examination.
Advise the patient of the investigations required and the management you would
propose.
The Performance Guidelines for Condition 010 can be found on page 90
059
Candidate Information and Tasks
Condition 011
Cancer of the colon in a 60-year-old man
CANDIDATE INFORMATION AND TASKS
You are working in a general practice. A 60-year-old man, whose father died of colon
cancer, consults you following a screening colonoscopy. This revealed a lesion shown in
the photograph given to the patient (see illustration below). The biopsy report confirms an
adenocarcinoma of the colon. The patient insists he has no symptoms and refuses to have
any operative treatment. However, he is still concerned enough to ask you what will happen if
nothing is done. The specialist who did the colonoscopy said the lesion was on the left side
of the colon.
The patient also wishes to know what are the prospects of cure if he changes his mind and
has the lesion removed by surgery, and would the surgery ever entail having a colostomy
(which he dreads)?
YOUR TASKS ARE TO:
Advise him what symptoms and signs may occur in the future, what complications
may develop and how they would be treated.
Address his concerns and counsel him about surgery.
You are not required to take any further history.

CONDITION 011. FIGURE 1.
Clinical notes: Biopsy of ulcerating tumour of rectosigmoid at 15 cm from anus.
Biopsy report: The specimens show numerous fragments of a moderately well
differentiated adenocarcinoma of the colon with invasion into the
submucosal tissues.'
Diagnosis: Adenocarcinoma of colon.
The Performance Guidelines for Condition 011 can be found on page 92

060
061
Candidate Information and Tasks
Condition 012
Thalassaemia minor in a 22-year-old woman
CANDIDATE INFORMATION AND TASKS
You are working in a general practice. Your next patient is a 22-year-old woman who
recently had a self-limiting febrile illness, which was suspected to be infectious
mononucleosis (IM). She is now fully recovered. Blood tests for IM were negative, but the
full blood examination showed a hypochromic microcytic anaemia of 108 g/L. The mean
corpuscular volume (MCV) was below normal (68 cubic microlitres normal 80-101). You
followed this up but there was no evidence of chronic blood loss (other than normal
menstruation). Serum iron and ferritin estimations were also normal. You suspected
/j-Thalassaemia minor ('Mediterranean anaemia') and this has been confirmed by
electrophoresis which showed an elevated Hb H
2
level (4.3%).
You are aware of her Greek descent and that she has just become engaged to be married.
Her fiance is also of Greek descent. The family history is that her mother, father and brother
are all alive and well. Her grandparents died in Greece and both were very old. One of her
father's brothers was reported to have died in childhood from an unknown cause. The
patient is very worried about being told she is anaemic, and as she is to be married shortly,
is worried about the effects on any of the children she hopes to have.
The patient has returned to discuss her results with you.
YOUR TASKS ARE TO:
Explain the nature of the condition to the patient.
Answer the patient's questions.
Advise the patient what should be done now.
The Performance Guidelines for Condition 012 can be found on page 95
062
013-014
Candidate Information and Tasks
Condition 013
Pre-pregnancy advice to a 28-year-old woman with
previous thromboembolism
CANDIDATE INFORMATION AND TASKS
You are working in a general practice. Your next patient is a 28-year-old woman, who had
one pregnancy 18 months ago, which was complicated by deep vein thrombosis and a
postpartum pulmonary embolus. She has come to see you for pre-pregnancy counselling
as she wishes to conceive again. At the time of a previous assessment twelve months ago,
she had ceased warfarin. When assessed six months ago, there were no sequelae or
symptoms and she had no signs of chronic venous insufficiency in the legs. There are no
abnormalities on physical examination and she is not overweight.
YOUR TASKS ARE TO:
Take any further relevant history you require from the patient.
Advise the patient on the management she will require before and during the
next pregnancy.
The Performance Guidelines for Condition 013 can be found on page 99
Condition 014
Pre-pregnancy advice to a 24-year-old woman with Type 1 diabetes mellitus
CANDIDATE INFORMATION AND TASKS
You are working in the primary care facility of a teaching hospital. Your patient is a woman
aged 24 years (para 0, gravida 0), a known diabetic for 15 years and well controlled on
insulin. She has come to see you for counselling and advice about possible future
pregnancies.
YOUR TASKS ARE TO:
Take any further relevant history you require.
Advise the patient of the information she needs to be given for pre-pregnancy
counselling.
The Performance Guidelines for Condition 014 can be found on page 102
Candidate Information and Tasks
Condition 015
An anencephalic fetus diagnosed at 18 weeks gestation in a
25-year-old primigravida
CANDIDATE INFORMATION AND TASKS
You are a Hospital Medical Officer (HMO) working in a primary care clinic attached to a
teaching hospital. Your next patient is a 25-year-old primigravida who has just had an
ultrasound performed at 18 weeks of gestation, which has revealed an anencephalic fetus
(as shown in illustration below). A maternal serum screening (MSS) was done at 16 weeks
and this had shown elevated levels of alpha fetoprotein.
YOUR TASKS ARE TO:
Take any further relevant history.
Advise the patient, in lay terms, of the relevance of the diagnosis and the subsequent
management you would propose in this pregnancy.
Advise the patient of the care you would recommend in a subsequent pregnancy.
You will not be expected to request examination findings, nor to arrange any further
investigations.
CONDITION 015. FIGURE 1.
Anencephalic fetus at 18 weeks of gestation
The Performance Guidelines for Condition 015 can be found on page 105
063
Candidate Information and Tasks
Condition 016
A duodenal ulcer found on endoscopy in a 65-year-old man
CANDIDATE INFORMATION AND TASKS
You are working in a general practice. You recently referred a 65-year-old man with a history of
self-medication for arthralgia and a subsequent six week history of epigastric pain and indigestion
to a gastroenterologist who performed an upper gastrointestinal endoscopy. The endoscopist told
the man he had detected a duodenal ulcer and gave him a photograph of the ulcer, taken during
endoscopy. The patient has come back to you seeking answers to several questions. The
photograph is as shown.
YOUR TASKS ARE TO:
Discuss the endoscopic findings with the patient in terms of the:
~ pathogenesis of the ulcer;
~ natural history and possible complications of the condition; and
~treatment options available to him.

CONDITION 016. FIGURE 1.
The Performance Guidelines for Condition 016 can be found on page 108
064
065
017-018
Candidate Information and Tasks
Condition 017
Advice on autologous blood transfusion to a 55-year-old man
awaiting elective surgery
CANDIDATE INFORMATION AND TASKS
You are working in a general practice. Your next patient is a middle-aged man booked for a
total hip replacement. You referred him to an orthopaedic surgeon who has arranged
elective surgery for his severely osteoarthritic hip. The patient has now come back to see
you, as he has some questions and in particular, is concerned about the risks of blood
transfusion (if required) and would like to find out about using his own blood for the
operation.
The patient wishes to discuss this with you, as he did not take in everything that was
explained by the surgeon.
YOUR TASKS ARE TO:
Explain the principles and indications for preoperative blood collection and intra
operative autologous blood transfusion.
Answer any questions from the patient about the blood transfusion procedure.
The Performance Guidelines for Condition 017 can be found on page 111
Condition 018
Advice on stopping smoking to a 30-year-old man
CANDIDATE INFORMATION AND TASKS
You are working in a general practice. The next patient is a 30-year-old man who has
returned to your practice for followup following a recent chest infection. He is a smoker (20
cigarettes per day). On his previous visit, you had told him that the 'best thing that he could
do for his health would be to stop smoking'. You have examined his chest which is clinically
normal.
At this visit, you are expected to follow up his response to your previous advice and counsel
him further about tobacco cessation.
YOUR TASKS ARE TO:
Assess his motivation to stop smoking.
Counsel him appropriately.
Discuss treatment options and general resources.
Respond to any questions he may have.
The Performance Guidelines for Condition 018 can be found on page 115
066
Candidate Information and Tasks
Condition 019
Excessive alcohol consumption in a 45-year-old man
CANDIDATE INFORMATION AND TASKS
You are working in a general practice. You are about to review a 45-year-old businessman
who consulted you two days ago about his drinking after seeing a TV program about the
harmful effects of alcohol.
At the previous consultation you established the following:
He is drinking excessively (at least five standard drinks every day);
This is of long standing at home, at work and socially;
He has problems at work;
He has trouble with his close family relationships;
His sexual performance is impaired;
He has had two minor traffic accidents in the last year;
He has a family history (grandfather) of alcoholism; and
On examination he is overweight (BMI 28 kg/m
2
), hypertensive 180/90 mmHg, and has
hepatomegaly.
You told him that his use of alcohol appears to be excessive and you ordered liver function
tests and a full blood examination. He is seeing you today for the results of the tests which
are as follows:
Liver Function Tests
Bilirubin Total 14 umol/L (<20)
ALP (Alkaline phosphatase) 50u/L (25-100)
AST (Aspartate transaminase) 45 u/L* (<40)
GGT (Gamma glutamyl transaminase) 63 u/L* (<50)
Serum albumin 32g/L (32-45)
Full Blood Examination
This showed a normal haemoglobin level (145 g/L) with a macrocytosis and elevated
mean corpuscular volume (MCV) of 106 fL (normal range 10-96) and some
variations in red cell size and shape (anisocytosis and poikilocytosis). Other features
were normal.
YOUR TASKS ARE TO:
Explain the results of the tests to the patient.
Discuss the effects of the excessive alcohol consumption.
Counsel him about his drinking.

You do not need to take any further history, nor perform any examination. The
Performance Guidelines for Condition 019 can be found on page 121
067
020-021
Candidate Information and Tasks
Condition 020
Type 1 diabetes mellitus in a 9-year-old boy
CANDIDATE INFORMATION AND TASKS
A nine-year-old boy, Roger, is admitted to the paediatric unit to which you are the Hospital
Medical Officer (HMO). This is his first presentation of insulin-dependent Type 1 diabetes
mellitus. His general condition is satisfactory, not requiring intravenous resuscitation, and
he has already commenced insulin therapy and has stabilised with good blood sugar
control.
As the Ward HMO, his mother has asked you for further information about his ongoing care
in relation to his diabetes from now on.
YOUR TASK IS TO:
Answer the queries the mother has, related to the ongoing care of Roger's diabetes.
The Performance Guidelines for Condition 020 can be found on page 125
Condition 021
Request for vasectomy from a 36-year-old man
CANDIDATE INFORMATION AND TASKS
You are working in a general practice. You are seeing a man aged 36 years who indicates
that he wishes to discuss vasectomy with you. You have enquired about his past medical,
family and social history (see below). He is asymptomatic across all his body systems.
Physical examination is normal, including his scrotum, testes and penis. Blood pressure is
120/70 mmHg, urinalysis normal. He is not overweight. You believe that he is in good
physical health.
You have already obtained the following patient details:
He has been married for 12 years and has two children (a boy aged seven and a girl
aged nine years). There are no marital problems of any kind;
He is a senior constable in the police force, does not smoke but is a moderate alcohol
drinker (three standard drinks a day);
He has no known drug sensitivities;
His mother, father and brother are well, as are his wife and two children; and
He is not on any medication.
YOUR TASKS ARE TO:
Explain the sterilisation procedure and its consequences to the patient.
Answer the patient's questions and provide counselling accordingly.
There is no need for you to take any additional history from the patient. The
Performance Guidelines for Condition 021 can be found on page 129
1-A
Communication,
Counselling & Patient
Education

1-A Communication, Counselling and Patient Education
Performance Guidelines
MCAT 002-021
2 Advice on breastfeeding versus bottle-feeding for a 28-year-old pregnant woman
3 Advice on neonatal circumcision for a couple expecting their first child
4 Suspected hearing impairment in a 10-month-old child
5 Counselling a family after sudden infant death syndrome (SIDS)
6 Hair loss in a 38-year-old man
7 An unusual feeling in the throat in a 30-year-old man
8 Pain in the testis following mumps in a 25-year-old man
9 Contraceptive advice for a 24-year-old woman
10 Rape of a 20-year-old woman
11 Cancer of the colon in a 60-year-old man
12 Thalassaemia minor in a 22-year-old woman
13 Pre-pregnancy advice to a 28-year-old woman with previous thromboembolism
14 Pre-pregnancy advice to a 24-year-old woman with Type 1 diabetes mellitus
15 An anencephalic fetus diagnosed at 18 weeks gestation in a 25-year-old primigravida
16 A duodenal ulcer found on endoscopy in a 65-year-old man
17 Advice on autologous blood transfusion to a 55-year-old man awaiting elective surgery
18 Advice on stopping smoking to a 30-year-old man
19 Excessive alcohol consumption in a 45-year-old man
20 Type 1 diabetes mellitus in a 9-year-old boy
21 Request for vasectomy from a 36-year-old man

068
069
002
Performance Guidelines
Condition 002
Advice on breastfeeding versus bottle-feeding for a 28-year-old
pregnant woman
AIMS OF STATION
To assess the candidate's ability to advise a young expectant mother on the advantages and
disadvantages of breastfeeding and bottle-feeding. This scenario tests the candidate's ability to
identify the conflict the young mother has in trying to respect what her mother has told her, while
knowing that this advice is contrary to her own feelings. It also tests ability to discuss logically the
advantages and disadvantages of the different feeding methods as well as testing knowledge on
safe bottle-feed preparation.
EXAMINER INSTRUCTIONS
The examiner will have instructed the patient as follows:
You are a 28-year-old mother having your first baby. You now have only five weeks to go and
although you always hoped to breastfeed your infant, you have had some doubts about its value
recently when your mother mentioned that formula-fed babies grow better than babies who have
been breastfed. You have come to discuss this. You realise that you may have to defend what
the doctor says to you (about breastfeeding being advantageous), and your own previously held
ideas about breastfeeding, against the ideas of your mother with whose opinions you have to
live.
Opening statement
'My mother feels that bottle-fed babies gain more weight than breastfed babies and therefore are
more healthy. What do you think, doctor?'
Questions to ask if not already covered:
'What are the advantages of breastfeeding? I would have thought it is easier to breastfeed.'
Is there anything special about breast milk? I always thought there was.'
'Do you have to prepare bottle feeds in any special way?'
'Are the formula feeds safe? I thought they contained cow's milk and what if you are allergic to
cow's milk?'
EXPECTATIONS OF CANDIDATE PERFORMANCE
The candidate should:
Be nonjudgmental, avoiding comments like 'Where on earth did your mother get such an
idea?'; rather asking 'Why do you think your mother made such a recommendation?'
Discuss that while breastfeeding is the optimal method of feeding the human infant, and that
the majority of mothers successfully breastfeed, a variety of reasons may prevent
breastfeeding in practice, including:
~illness in the mother;
~failure to establish lactation, which may be hormonally based;
~possible illness in the baby (e.g. cleft palate);
70
002
Performance Guidelines
~ prematurity, which requires the mother to express regularly to maintain her supply;
~ previous extensive breast surgery in the mother; and
~ heightened anxiety in the mother.
Explain that if for some reason breastfeeding is unsuccessful, formula-feeding is a safe
and very effective alternative.
Discuss that formulas are designed to contain the same nutritional components as
breast milk, but that exact reproduction is difficult as the concentration and components
of breast milk change throughout each feed.
Advise that there is no advantage of formula-feeding over breastfeeding.
Discuss the specific advantages of breastfeeding:
~the practical advantages of being able to feed almost whenever and wherever the
baby wants it without having to prepare formula, carry bottles around and without
problems with sterility;
~ increased resistance of the baby to infection, from immunological constituents in
breast milk including lymphocytes and antibodies; and
~ satisfaction derived from feeding the infant as well as the development of a close
relationship with the infant.
In response to her mother's comment, advise that weight gain is not the only criterion for
success as excess weight gain in the first 12 months of life may in fact be detrimental in
later life.
The candidate should stress the importance of optimal formula-feeding as follows:
~ sterility in preparing the bottle feeds is essential;
~ bottles need to be washed clean with a bottlebrush to ensure that all milk residue is
removed;
~ bottles and teats need to stored in solution (e.g. Milton), to ensure continuing
sterility, but the bottles need to be rinsed free of this solution prior to use;
~the fluid used to make the formula and to rinse the bottles should be cooled boiled
water;
~ each can of formula has explicit makeup instructions on the side of the can or packet;
if followed these will produce the exact required concentration;
~there is no place for any added scoops, which can be harmful;
~ the day's requirements are best made up at the one time, although each feed can be
made separately. If the former, the day's feed should be stored in the refrigerator;
~only one day's feed at a time should be prepared in advance; and
~each feed should contain approximately 30 ml more than it is anticipated the baby
may take, and any excess discarded at the end of the feed.
KEY ISSUES
Empathic answering of this young mother-to-be's questions.
Recognition that she is uncomfortable with what her mother has told her but is seeking
reassurance and support for her own view which she feels is accurate.
Satisfactory explanation of the advantages and disadvantages of the different methods
of feeding.
Candidates should know how formula feeds are prepared.
071
002
Performance Guidelines
CRITICAL ERROR - none defined
COMMENTARY
Ability to discuss impartially and accurately the relative merits, indications,
contraindications and techniques of infant feeding by breastfeeding and by
formula-feeding is a requisite for all medical graduates as outlined and is an area where
good communication skills are paramount.

072
003
Performance Guidelines

Condition 003
Advice on neonatal circumcision for a couple expecting their first child
AIMS OF STATION
To assess the candidate's ability to give impartial advice about neonatal circumcision.
EXAMINER INSTRUCTIONS
The examiner will have instructed the parents as follows:
You are a couple who are expecting your first baby. Your family members have suggested
that if the baby is a boy he should be circumcised. You are very unsure about this, as you
cannot see any reason why circumcision is essential. You have no religious beliefs that
dictate that circumcision must be done. You have therefore come to discuss the process
and learn what the advantages and disadvantages of the procedure are before making
your own decision on the matter.
Opening statement
We have come to discuss with you whether to have our baby circumcised.'
Questions to ask unless already covered:
What does the procedure involve?'
'What are the complications that can happen?'
What are the advantages of having it done?'
'Are there any times when it definitely should or shouldn't be done?'
'Has anyone looked into this in detail and come to any conclusions about it?'
EXPECTATIONS OF CANDIDATE PERFORMANCE
The candidate should:
give an impartial but informed explanation to the parents on the advantages and risks
involved in routine neonatal circumcision;
indicate that when religious grounds are stated as the reason for the procedure, these
are generally respected; and
realise that many are unaware of the actual process of circumcision and may ask for the
procedure more as a ritual; and stress that the parents should consider the advantages
and disadvantages of the procedure before making a decision.
During the discussion, the candidate would be expected to advise along the following
lines:
The perceived advantages of routine neonatal circumcision commonly quoted are:
~Reduced incidence of urinary tract infections (UTIs) in circumcised boys. While
circumcision may assist the uncircumcised boy who is suffering recurrent UTIs,
routine general circumcision of all boys is not indicated to achieve this, but can be
selectively applied at a later age if this situation exists.
~Reduction in the incidence of sexually transmitted infections (STIs) and HIV infection.
This remains a controversial point. The literature currently is divided on this issue, but
some evidence suggests the risk of HIV infection is lessened by circumcision.
073

003
Performance Guidelines
~Circumcision is indicated for phimosis or its prevention. This is true only if all
conservative methods of treatment have failed. Often, phimosis, developing after
birth, is secondary to inappropriate foreskin care and subsequent trauma and
scarring. If asked, the candidate should indicate the appropriate care of the foreskin,
which is minimal, until the foreskin can be retracted easily, which is usually by about
five years of age.
~Neonatal circumcision minimises the risk of subsequent development of carcinoma
of the penis. But poor penile hygiene associated with human papilloma virus infection
is the major contributor in adults, cancer being rarely seen in men who can retract
and clean the foreskin.
The recognised complications and disadvantages of routine neonatal circumcision
should also be discussed and include:
~haemorrhage;
~infection, including septicaemia/meningitis (rare);
~ulceration of the glans;
~inadvertent injury to the urethra;
~too much skin removed leading to unsatisfactory cosmetic appearance;
- anaesthetic complications; and
~secondary phimosis.
The complication rate has been estimated to occur with an incidence of between 1-5%
(range 0.2%10%); but these are figures at all ages, not just in the neonatal age range. The
skill of the operator is obviously of paramount importance. The most common complication
is haemorrhage.
The candidate should also discuss the absolute contraindications to routine neonatal
circumcision explaining each in turn:
~hypospadias and other congenital anomalies of the penis (e.g. epispadias);
~chordee;
~buried penis;
~sick infants, including jaundiced infants;
~family history of a bleeding disorder or known recognised familial bleeding disorder
possibility (e.g. haemophilia A); and
~inadequate expertise and facilities.
The candidate should be able to explain the procedure of circumcision and indicate
that:
~it is usually done under local anaesthesia; and
~if performed after six months of age, it is done under general anaesthesia.
KEY ISSUES
The ability to discuss in an unbiased manner the perceived advantages and
disadvantages of routine neonatal circumcision.
Capacity to summarise that the recommendations of various national and international
paediatric and paediatric surgical associations, who have extensively reviewed the
literature on the subject, do not support routine neonatal circumcision.

074
003
Performance Guidelines

CRITICAL ERROR - none defined
COMMENTARY
This topic is one that has been controversial for many years. While circumcision for religious
reasons has been done for many centuries, the issue in the scenario is related to the secular
trend towards routine neonatal circumcision of all males soon after birth. This has become more
of a ritual rather than for any recognised medical indication, although this tendency has reduced
in recent times. In fact there are many recognised complications of circumcision, not just routine
neonatal circumcision, but circumcision at any age. However there are recognised medical
indications for the procedure (e.g. established phimosis in boys and men).


CONDITION 003. FIGURES 1 AND 2.
Circumcision in ancient Egypt Phimosis
Many young parents are unaware of the issues involved and are often ill-informed by family
members who recommend that their infant should have the procedure performed without any
explanation as to why. They then request circumcision without any information about the
procedure and hence the need for informed discussion to allow them to make a rational decision
in the interests of their baby.
This scenario illustrates that medical practitioners are obliged to provide accurate information on
the risks and benefits of routine neonatal circumcision and should attempt to avoid any personal
bias. The decision should be left to the parents after a full and accurate discussion.
Unbiased up-to-date written material summarising the evidence should be made available to the
parents.
The review of the literature in relation to risks and benefits shows there is no evidence of
benefit outweighing harm for neonatal circumcision as a routine procedure.
The Policy Statement on Circumcision from the Paediatric and Child Health Division of the Royal
Australasian College of Physicians (September 2004) is recommended as an excellent summary
with 78 references to the evidence for and against circumcision. This is available on the open
website of the Royal Australasian College of Physicians under Policies (http://www.racp.edu.au).
075
004
Performance Guidelines
Condition 004
Suspected hearing impairment in a 10-month-old child
AIMS OF STATION
To assess the candidate's ability to deal with suspected hearing loss in an infant aged 10
months and ability to appropriately counsel the parent.
EXAMINER INSTRUCTIONS
The examiner will have instructed the parent as follows:
You have a 10-month-old girl called Helena. You have been referred by the local childcare
nurse for a suspected hearing loss in your daughter. You and your husband are concerned
as you feel baby Helena's hearing is normal. Your responses to questions asked are
outlined below. You will accept advice about referrals. If referral is not suggested, you
should ask do you think further tests would help?'
The candidate is expected to take a relevant history to determine if hearing loss may be
present as suspected by the child health worker despite the good sign of babbling, and to
provide succinct, accurate advice to concerned parents, who have noted nothing amiss
with her development or hearing.
The candidate should ask for history details as below (your response is indicated in
brackets):
Is there a family history of deafness?' (none);
Maternal problems in pregnancy 'Were there any problems with the mother during
pregnancy especially infectious diseases?' (none)
Perinatal problems 'Were there any health problems with the baby during or soon
after birth?' (none)
General development 'Is the baby growing and thriving?' (satisfactory same as the
other children)
'Does the baby respond to sounds, including loud sounds?' (seems to hear them and
respond)
'Does the baby respond when called by her name?' (usually)
'Does the baby turn towards the sources of the sounds?' (sometimes)
'Does the baby respond to television?' (sometimes).
In regard to any of the above aspects, the parents have noted no abnormalities in regards
to her response to sound.
076
004
Performance Guidelines
EXPECTATIONS OF CANDIDATE PERFORMANCE
The history should cover most of the questions detailed above.
The candidate should explain that:
Hearing loss is common in preschoolers most cases are mild and transient due to
conductive deafness from middle ear effusions.
Sensorineural hearing loss is less common but important to detect as early as possible.
Distraction tests are only a screening tool and do not diagnose deafness.
Often it is the parents who recognise that there is something wrong; but the child health
nurse is also a professional and her concerns must be followed up.
If there is a problem then the earlier the diagnosis the better.
The candidate should give the following advice:
Refer the child to a Paediatric Audiologist as soon as possible for more formal hearing
assessment by an audiogram.
Review following audiogram.
If normal, reassure and review hearing and early language development in about three
months.
If abnormal, refer to appropriate specialist for further evaluation.
The candidate should exhibit understanding for the parent's concern and provide guarded
reassurance and support.
KEY ISSUES
Appropriate history relevant to deafness.
Counselling with reference to early definitive screening for hearing.
Providing appropriate level of support and reassurance.
CRITICAL ERROR
Failure to refer for specialist assessment (audiogram) for definitive diagnosis.
COMMENTARY
The candidate is expected to proceed from the cue of 'possible hearing loss' to assess
whether other evidence, such as failure to turn to sound, is present to support this
diagnosis. In addition, enquiry regarding features suggesting associated developmental
delay is appropriate. Given a working hypothesis of possible hearing loss, features in the
history which could be causative, such as maternal illness in pregnancy, perinatal
problems such as jaundice or drug administration, should be evaluated. However, the
importance of early diagnosis and treatment of significant hearing deficit in this age group
make referral for definitive diagnosis mandatory despite no other concerning features
being present.
Adequate communication skills in dealing with a concerned patient are essential, used in
combination with appropriate knowledge-based clinical skills.
77
005
Performance Guidelines
Condition 005
Counselling a family after sudden infant death syndrome (SIDS)
AIMS OF STATION
To assess the candidate's ability in approach to the family and providing empathic
counselling in this tragic situation of presumed sudden infant death syndrome (SIDS) where
there are no suspicious circumstances. Candidates should outline the statutory
requirements (i.e. notifying police and coroner), in a caring and sympathetic manner.
EXAMINER INSTRUCTIONS
The examiner will have instructed the relative as follows:
You are the aunt of an infant who died, apparently of SIDS, the day before. The family
members including the young single mother have attended to ask questions about the
baby's death but the mother is still too distressed to ask them herself. You are the
spokesperson of the family.
Opening statement
We can't understand why Andrew has died!'
Questions to ask unless already covered:
Why do the police have to be involved? Do they think my sister killed her baby?'
Why does he have to have an autopsy?'
When will we get further information and results of this?'
When can we arrange his funeral?'
We feel so alone. Is there anyone we can talk to about this?'
'Should the snuffles have been treated?'
EXPECTATIONS OF CANDIDATE PERFORMANCE
Explanation of diagnosis
The most likely cause of the child's death is SIDS. Candidates should explain what is known
about SIDS along the following lines:
the frequency of SIDS has fallen from 1 in 500 live births to now approximately 1 in 1000;
the peak incidence occurs at about four months of age; and
there are no certain causes known. Many theories exist, but none is proven.
Explain that there can be other causes of sudden infant death (for example, overwhelming
infection), but the child's history does not suggest this cause. Andrew's snuffles were not a
warning sign and there is no suggestion that any medical treatment would have influenced
the outcome.
078
005
Performance Guidelines
Immediate management following a death due to apparent SIDS should be advised
empathically as follows:
Explain that the police and the Coroner must be notified by law because Andrew's death
was sudden and unexplained.
Explain that the role of the Police Officer is to assist the Coroner police are required
to interview all people concerned including the baby's general practitioner.
Explain the need for the autopsy in all cases, and that autopsies are done by very
experienced pathologists in an attempt to find out what causes SIDS, and to exclude
other possible causes of death.
Explain that tissues will be removed for further examination under the microscope.
Offer to contact the Coroner later to obtain information on the initial findings after the
autopsy has been performed or advise that the Coroner's office will contact the mother
at a later date to giver further information.
Offer to contact other family members for support for the mother.
The Coroner will decide if an inquest needs to be held, but with SIDS this is generally
not necessary.
Offer to contact the local SIDS Support Group, if one is available.
Future management
Followup contact with family and with the Coroner/pathologist to confirm diagnosis. Liaise
with support group in counselling the mother when results are available.
KEY ISSUES
Appropriate empathic explanation.
Ability to explain the involvement of appropriate authorities and support groups.
Offering to arrange for continuing followup, contact and support with the family.
CRITICAL ERRORS
Failure to display empathy in counselling.
Failure to recognise and explain need for coronial notification and autopsy.
COMMENTARY
Empathy in communication is essential in these tragic circumstances, together with
accurate knowledge of legislative requirements. All deaths under these circumstances
must be reported to the Coroner and the police must take statements. This is often the most
distressing part of the process for young parents and should be explained carefully to the
family why this process needs to happen by law.
The caring practitioner will also offer to liaise with the Coroner on behalf of the parents, and
in this way is often able to receive preliminary reports if the Coroner is agreeable to them
being released, which many Coroners are. Several pathologists who perform these
autopsies actually interview the parents themselves when the autopsy is completed.
The caring practitioner will offer to keep contact with the grieving couple or parent until
confident that this tragic event has been accepted.
079

006
Performance Guidelines
Condition 006
Hair loss in a 38-year-old man
AIMS OF STATION
To assess the candidate's ability to deal with a cosmetic problem, almost certainly alopecia
areata, for which treatment and prognosis are uncertain.
EXAMINER INSTRUCTIONS
The examiner will have instructed the patient as follows:
You are a 38-year-old male newsagent, married, with two children. Your general health is
excellent and you do not smoke or use alcohol. Past history is clear and there is no significant
family history including baldness.
You work long hours with the usual stress associated with running a small business, but
otherwise have no social or family problems. You are worried about your appearance because
of your contact with customers. You are concerned about the cause of the hair loss and are
very anxious to have treatment and also to be assured of effectiveness of treatment. Become
impatient if simple reassurance is the main advice given.
Opening statement
'What is happening to my hair?'
Questions to ask if not already covered:
'Will I go completely bald?'
'Will it improve?'
'How long will it last?'
'What can be done about it?'
Is treatment effective?'
'Should I see a specialist?'
'Could it have anything to do with my glands? OR with my thyroid gland?'
EXPECTATIONS OF CANDIDATE PERFORMANCE
The candidate should recognise that the patient has alopecia areata and is concerned about
his appearance; and should explain the nature of the condition in such a way that the possibility
of improvement or return to normal is emphasised but the natural history is unpredictable in
individual cases. The objective is to achieve patient understanding and acceptance of his
condition based on correct information. The patient's questions should be answered directly
with supportive explanation.
The candidate should explain that:
Initial management is by topical medication which aims to stimulate hair regrowth. Any one
agent should be used for 3-6 months before changing therapy.
~Potent topical corticosteroid applied once or twice daily e.g. betamethasone
dipropionate .05%.
~Intralesional corticosteroid such as triamcinolone acetonide 10 mg/mL is useful for small
areas on the scalp or eyebrows. Multiple injections are usually required.

080
006
Performance Guidelines

~Topical dithranol applied once daily, commencing at 0.5% increasing gradually to 2%
(avoid eye contact). ~Topical minoxidil (5% lotion) applied twice daily in cases not
otherwise responding
(but has very limited effectiveness).
Oral corticosteroids may be Mailed if topical treatment is ineffective, tapering dosage
downwards over two months.
If the condition worsens, disguising the hair loss with a wig may be considered.
Consideration of topical immunotherapy and ultraviolet phototherapy would require
referral to a dermatologist.
Some patients may require early referral for confirmation of the diagnosis and reinforce-
ment of advice about the likely course.
Followup arrangements should be made appropriately.
KEY ISSUES
Effective communication skills are very important in this case. Appropriate language,
verbal and nonverbal communication, and good interpersonal skills should be dis-
played by the candidate.
Showing empathy, sensitivity and perceptiveness, as well as being honest and
generating trust and confidence are particularly important in a chronically relapsing
condition such as this which significantly affects the patient's appearance.
Place of topical and systemic treatment and prognosis.
CRITICAL ERROR - none defined
COMMENTARY
Alopecia is a generic term for hair loss. Alopecia areata is a descriptive term for one or
more discrete circular areas of hair loss. These areas can occur anywhere on the body
including the beard area in males.
Alopecia universalis is where there is complete hair loss from the whole body and is a
variant of alopecia areata. The condition is a chronically relapsing autoimmune disease
with an extremely variable natural history. Patches on the scalp may regrow spontaneously,
remain unaltered or enlarge and coalesce into alopecia totalis (whole scalp).
In a case such as this scenario, there is approximately a 33% chance of complete regrowth
within six months and a 50% chance within one year. Approximately 80% of patients,
however, will eventually relapse. If the hair loss persists for years, the prospect of regrowth is
diminished, although the potential for hair regrowth always remains because the hair
follicle is not destroyed.
The aetiology is unknown, but a family history is present in 20% of cases and there is a
linkage with other organ-specific autoimmune diseases. There may be a specific trigger
such as a febrile illness or severe emotional stress (for example, the death of a family
member). Less severe day-to-day stress is not considered to be a trigger. The condition
itself is stressful, especially in females.
081
007 Performance Guidelines
Condition 007
An unusual feeling in the throat in a 30-year-old man
AIMS OF STATION
To assess the candidate's ability to take a satisfactory history about globus pharyngeus
disorder and to display perspective in selecting appropriate investigations, and skill in
counselling and educating a patient about the condition.
EXAMINER INSTRUCTIONS
Examiners should note that the scenario covers the initial consultation about the problem
and that ongoing management is not being assessed.
The examiner will have instructed the patient as follows:
You are a 30-year-old man, happily married with two children. Your parents and siblings are
in good health and apart from vasectomy two years ago, your past medical history is clear.
You smoke 10-15 cigarettes daily and only drink socially at weekends. You are consulting
the doctor about an unusual feeling in your throat.
Opening statement
I keep getting a lump in my throat. It feels like a knot.'
Provide more information as follows:
Without prompting:
Present your symptoms in a straightforward manner. You have become concerned since
the recent vomiting episode. You have had no previous worry about your health and you
have no idea what the nature of your complaint is, but you now wish to have it thoroughly
investigated.
Over the last 4-6 weeks you notice that:
your throat tightens up;
you have excessive saliva;
you clear your throat repeatedly;
a few days ago, it was very bad and you vomited; and
it occurs mostly after your evening meal. if questioned, answer as follows:
the feeling lasts 3-4 hours, usually until you go to bed;
you swallow more often when you have it;
there is definitely no difficulty swallowing solids or liquids which go down easily without
discomfort;
when your throat tightens, your voice can 'catch', and your eyes water; and
your voice is otherwise unaffected.
There is:
no hoarseness of the voice;
no sore throat, cough or nasal discharge or discharge from the back of the nose into the
throat (called postnasal discharge);
082
007
Performance Guidelines
no loss of weight, appetite change, abdominal pain or regurgitation (water brash);
no shortness of breath, palpitations, chest pain or dizzy spells;
no aggravation of symptoms brought on by lying down; and
You:
sleep well, have no other symptoms and do not take any medications;
believe you are in good health;
never take time off work;
feel fit;
have no past history of anxiety or other psychiatric illness;
do not feel depressed or anxious;
have no particular worries about your everyday life, wife and children;
are conscientious and hardworking but not a worrier; and
usually enjoy work, but have felt annoyed in the past few months when having to work
weekends you feel you should be with your family.
A cousin died from cancer of the larynx, three months ago. You used to be close as
children. He had been a heavy smoker and drinker in adult life, and you grew apart. You
were upset by his death, but no more than you would expect for normal grief. His death is
still on your mind, as well as the thought of cancer at times.
If the doctor reassures you without discussing the possibility of any investigations say: 'You
don t seem to be taking my complaint seriously'.
Alternatively, if the doctor advises multiple investigations all of which are to be done at once
say: 'Isn't there just a simple test to check my throat out?'
Examiner statement
When the candidate has completed the history or after five minutes the examiner should
say: 'Physical examination of this patient is completely normal. You should now
discuss the problem with the patient as stated in your tasks'.
EXPECTATIONS OF CANDIDATE PERFORMANCE
The focused history is expected to exclude dysphagia and hoarseness, any
disturbance in general health and to reveal the patient's concerns about the cause of his
cousin's death.
The diagnosis of globus disorder (subject to laryngoscopy) should be made, but may
not be directly stated to the patient. Examiners should use their discretion in assessing
how the candidate describes this functional disorder to the patient. Other than gastro-
oesophageal reflux, there is a list of much less likely differential diagnoses for this
patient.
Choice of investigations:
~ Laryngoscopy and pharyngoscopy must be done.
~ Chest X-ray with thoracic inlet views and barium swallow are acceptable.
~ Upper gastrointestinal endoscopy may be suggested for reassurance.
~ Bronchoscopy is not indicated.

007
Performance Guidelines

CONDITION 007. FIGURE 1.
Laryngoscopic vi ew normal vocal cords
Patient counselling and education: the correct diagnosis should be explained as most likely
being a nonserious condition with transient change in sensation or function of the throat;
~the condition is brought on by emotional factors;
~the condition needs limited investigations which should be explained; and ~
investigations are unlikely to reveal any serious process.
KEY ISSUES
Ability to take a focused history.
Ability to discuss a probable functional disorder with a concerned patient.
CRITICAL ERRORS
Failure to request laryngoscopy or upper pharyngeal/oesophageal endoscopy.
Failure to indicate to the patient that serious disease is extremely unlikely.
COMMENTARY
This patient is presenting with recent onset of mild symptoms localised in the same anatomical
region as his cousin's recent cause of death, the role of normal grief and worry about cancer
being largely unrecognised.
Globus pharyngeus disorder (globus hystericus) is a physiological symptom associated with
altered mood states, often grief, but not associated with any specific psychiatric disorder or
necessarily requiring psychiatric treatment. Elevated cricopharyngeal pressure or abnormal
hypopharyngeal motility may exist at the time of the symptoms. The same sensation may result
from gastro-oesophageal reflux or from frequent swallowing and mouth dryness.
Other causes of upper oesophageal or laryngeal compression are retrosternal goitre, carcinoma
of laryngopharynx, oesophageal or cricothyroid web (sideropenic dysphagia). Skeletal muscle
disorders, myasthenia gravis, myotonia dystrophica, and polymyositis are other potential
causes of dysphagia. None is likely in this case.
083
084
008
Performance Guidelines
Condition 008
Pain in the testis following mumps in a 25-year-old man
AIMS OF STATION
To assess the candidate's knowledge of mumps orchitis including natural history, management,
prognosis and preventive medicine aspects, and communication skills in dealing with an unwell
and anxious patient.
EXAMINER INSTRUCTIONS
The examiner will have instructed the patient as follows:
You thought you were recovering from mumps, having noticed the onset of facial swelling about a
week ago. You contracted the disease from your older son aged five years. You are now
consulting the doctor about a painful and very tender left testicle. Analgesics (Panadol) have
had little effect. The doctor has examined you and will now advise you about your condition.
You are in quite severe pain and feel most unwell. You are anxious about possible sterility (you
and your wife would like to have a daughter), whether the other testis will also be affected,
possible impotence, your infectivity (you have a wife and other younger son) and how long you
will be away from work. Be cooperative and willing to accept the doctor's advice if presented
clearly.
Questions patient should ask if not already covered (the candidate's expected
response is in brackets):
'What is the connection between the mumps and this trouble?' (Doctor should explain how
mumps virus is related to testicular problems viral aetiology)
What can I take for the pain?' (Pain killers that contain codeine compound analgesics)
'Are there any antibiotics or other drugs for this condition?' (Not at this stage)
Will it affect both of my testicles?'(Unlikely, usually only one is affected. Unless the other
testicle becomes affected, there would not be any expected influence on fertility. Sterility can
rarely follow if both testicles are affected)
Will we be able to have another child, if we decide to?' (Yes, fertility is not likely to be affected)
'Will my sex life be affected?' (No problems anticipated)
What will happen to the testicle eventually?' (Possibly reduction in size, but usually remains
fully functional. Function of the other testis is usually unaffected)
When will I be able to go back to work?' (Depends on how rapidly the pain and swelling
persists about 7-10 days)
Why wasn't my son affected in this way?' (Orchitis inflammation of the testicle is
extremely rare in childhood)
Will our younger son get mumps too?'(If he is immunised already with
measles-mumps-rubella (MMR) vaccines, he is at minimal risk. If he is not immunised, he is
unlikely to contract mumps from his brother, but he may still be infected from his father. The
younger son should be immunised if not already done)
'Are there any other complications of mumps?'(Very occasionally mild meningitis, which is
inflammation of the coverings of the brain)

008
Performance Guidelines
EXPECTATIONS OF CANDIDATE PERFORMANCE
The candidate should be confident with the diagnosis of mumps orchitis and should state
this to the patient. Torsion of the testis need not be considered under the circumstances.
A sympathetic and reassuring manner is expected. The candidate should indicate that the
patient's anxiety over testicular function (fertility and sexual activity) and infectivity to his
younger child, are recognised and that possible reduction in size of the left testis is not
likely to be of any consequence to fertility.
Pain control paracetamol with 8 or 30 mg of codeine (for example, Panadeine,
Panadeine Forte).
Steroids can be used to relieve severe pain but have no other effect on the illness. Their
use in mumps is controversial.
Advise general measures for a febrile illness adequate fluids, light diet, and rest, until
pain and swelling have subsided.
Local measures scrotal support, application of heat.
Provide information about immunisation for mumps live attenuated vaccine combined
with rubella and measles vaccines (MMR) is available. However, this will not immediately
protect the wife or son from patient's infectivity. Immunoglobulin is not effective. Mono-
valent vaccine is not available. Immunisation is advised for all children after 12 months
of age, with booster dose before going to school.
The risk of infectivity to other adults is very low.
In counselling, the candidate should tell the patient the diagnosis, determine the patient's
knowledge of the condition including any fears he may have, discuss these and reassure
him appropriately.
KEY ISSUES
Confidence in diagnosing mumps orchitis.
Recognising patient anxiety about possible infertility and impotence and giving
appropriate reassurance.
Giving an adequate explanation of the condition including the infectivity of mumps, and
its prevention by immunisation.
CRITICAL ERROR - none defined
COMMENTARY
Mumps has the following associations:
The infecting organism is a paramyxovirus, spread by droplet infection or direct contact.
Infectivity is 50-60% (sufferer to unimmunised person).
Incubation period is 12-25 days (usually about 18).
Period of infectivity is from 6 days before swelling of face occurs till 9 days after, so his
wife (if she has not had mumps) and younger son can still contract it from the father.
One attack gives lifelong immunity.
085

086
008
Performance Guidelines
Complications of mumps:
~ Common orchitis and aseptic meningitis; and
~ Rare encephalitis, arthritis, pancreatitis and oophoritis in females.
Orchitis complicating mumps:
~ occurs in 20-30% of postpuberal males;
~onset is 3-4 days after the parotitis is subsiding;
~ usually is unilateral. Subsides over one week;
~ significant atrophy of testis occurs in 50% of cases;
~ sterility is rare only if bilateral involvement;
~ sexual performance is not affected after recovery; and
~ mumps orchitis does not predispose to testicular malignancy.

009
Performance Guidelines
Condition 009
Contraceptive advice for a 24-year-old woman
AIMS OF STATION
To assess the candidate's ability to appropriately assess and advise a patient requesting
oral contraception.
EXAMINER INSTRUCTIONS
The examiner will have instructed the patient as follows:
You are aged 24 years, work as a secretary and live alone, but have a steady boyfriend.
You have never been pregnant and have no history of gynaecologic or medical problems.
You have been in a stable monogamous sexual relationship with your boyfriend for the past
12 months, using barrier contraception with condoms.
In response to specific questions, the following list of responses is likely to cover most
questions:
You have no previous deep vein thrombosis, active liver disease, breast cancer or
abnormal bleeding. Periods have always been irregular, occurring every 2-3 months, but
are otherwise normal.
Blood pressure has always been normal.
You have no history of migraine.
You are not on any medications, do not smoke and only drink alcohol occasionally.
Examination findings to be given by the examiner after the history are basically
normal, but the candidate is expected to ask specifically for the important
relevant findings:
Blood Pressure 120/80 mmHg
Pelvic and abdominal examination no abnormality.
Pap smear normal six months ago.
Breast examination normal. No hirsutes.
EXPECTATIONS OF CANDIDATE PERFORMANCE
The candidate should convey the substance of what follows to the patient:
A patient wishing to take the oral contraceptive pill (OCP) requires the following assess-
ments:
Exclude absolute contraindications on history. Need to exclude deep venous thrombosis,
oestrogen-dependent malignancies particularly breast cancer, active liver disease or
previous cholestatic jaundice, unexplained vaginal bleeding and focal migraines.
Check for relative contraindications to see whether the therapy can be justified. A wide
variety of relative contraindications has been described including hypertension, cigarette
smoking, diabetes, and very irregular cycles or oligomenorrhoea.
087

088
009
Performance Guidelines

Choice of pill:
~ Choose a low monophasic dose oestrogen pill (such as Microgynon 30). This has
a low breakthrough bleeding and low failure rate. It is also less costly than
Microgynon 20. Because her cycles are irregular (oligomenorrhoea) it is probably
better to choose a triphasic preparation such as Triphasilor Triquilaras these have
less post-pill amenorrhoea associated than a monophasic constant dose pill.
- If she had been an epileptic or on an antituberculous drug, a 50 ug
oestrogen-containing pill such as Microgynon 50should have been chosen. This is
because these drugs increase liver enzyme activity, resulting in an increased
metabolism of the hormones administered in the OCR
~If she had been hirsute, or with acne (consistent with polycystic ovarian syndrome)
an oestrogen-dominant pill such as Ovulen 0.5/50, or (probably more appropriate)
Diane 35if the patient can afford it, should have been advised.
~If she was still breastfeeding, or previous oestrogen-containing pills have produced
problems or she has had a previous thrombosis, a low dose progestogen pill should
have been chosen and a failure rate of 3% accepted.
In this instance, a low-dose oestrogen or triphasic pill would have been appropriate.
Need for followup in about three months. This is required to check the blood pressure
and to advise her as to whether the pill prescribed needs to be changed because of
persistent problems such as break-through bleeding.
Patient starts in red sector at time of next period. Contraceptive efficacy is satisfactory
after seven hormone tablets have been taken.
Explain about breakthrough bleeding, missed pills and diarrhoea and the appropriate
management of these: take the normal dose the following day, and take appropriate
additional precautions depending on circumstances.
Common side effects breakthrough bleeding for first three months, sore breasts in the
first 1-2 cycles.
[ KEY ISSUES
Ability to take an adequate history to exclude absolute contraindications to the OCP and
facts that would influence the pill chosen, and its dose.
Ability to advise a patient as to how to take the pill, the timing of its effectiveness and the
likely problems during its use.
CRITICAL ERROR
Failure to exclude absolute contraindications to OCP use.

009
Performance Guidelines
COMMENTARY
A young woman needs to be fully informed of all the benefits and side effects and risks of
taking the OCR She also needs to be carefully assessed to ensure that she has no
condition making her unsuitable to take the pill. Any patient prescribed the OCP must have a
full explanation of how to commence taking the pill, when it becomes effective as a
contraceptive, and what to do if a pill is missed accidentally.
As part of the assessment of a patient who is to be prescribed the OCR the type of pill and
its cost, should be taken into account as part of the advice to the patient.
Having excluded absolute and relative contraindications to use of the OCP (as is the case
in this patient), an appropriate low dose oestrogen pill should be advised with a low
breakthrough bleeding rate and low failure rate.
Alternatively a triphasic preparation could be used which has less post-pill amenorrhoea.
Appropriate advice along the above lines is required.
Common problems likely with candidate performance are:
failing to advise patients as to when contraception will be achieved following the
commencement of therapy;
what to do if a pill is missed or the patient gets diarrhoea; and
failing to advise the common side effects.
089

Condition 010
Rape of a 20-year-old woman
AIMS OF STATION
To assess the candidate's ability to appropriately assess and manage a woman who gives
a history of having recently been raped.
EXAMINER INSTRUCTIONS
The examiner will have instructed the patient to respond as follows:
menses are regular and normal and your last menstrual period started ten days ago.
you have not been taking or using any contraceptive agents.
you have had no previous operations, illnesses or pregnancies.
the rape followed a threat of severe injury if you did not comply. No bleeding followed
rape. He did ejaculate into the vagina.
you have been sexually active in the past, but not for the last six months. No previous
known pelvic infections.
you have no allergies to drugs or chemicals.
you do not use drugs of addiction, and are not on any medications at present.
Questions patient to ask unless already covered (candidate's expected responses as
outlined in expectations of candidate performance):
'Has he caused me any harm?'
'Will I be able to have children when I want to?'
'Do I need any treatment now?'
Initial examination findings to be given to the candidate by the examiner on request:
general examination: no evidence of bruising or trauma;
vulva: looks normal not bruised and not bleeding; and
speculum examination and PV have not yet been done. The candidate should now
discuss these with patient, and advise on management plan.
EXPECTATIONS OF CANDIDATE PERFORMANCE
The history should have:
determined the date of her last menstrual period; and
noted that she was not on the contraceptive pill.
The candidate should advise the patient along the following lines:
Explanation as to what examination is required and why. Speculum examination is
required to check that there are no lacerations in the vagina, to take swabs to exclude
infections as indicated below and to collect a specimen for pathologic analysis.
The need to exclude sexually transmissible infections and get a baseline blood sample
for HIV and syphilis. A cervical smear and culture should be done for gonococcus and
Chlamydia (or urine can be collected for polymerase chain reaction [PCR] analysis to
exclude Chlamydia). Penicillin (or azithromycin) and doxycycline should be given
prophylactically.
090
091

010
Performance Guidelines
The need to prevent pregnancy. As she is in the late follicular phase of the cycle, the
Yuzpe method of the oral contraceptive pill (two doses of two tablets given twelve
hours apart with associated metoclopramide [Maxolon], or high dose levonorgestrel
[Postinor]), should be given.
The need to review patient in three weeks to check whether she has conceived, and to
review test results and decide if any need to be repeated. Screening for syphilis and HIV
will need to be repeated in 1-3 months time. By that time, reassurance about subse-
quent pregnancy should be possible.
The need to collect a specimen from the vagina to see if spermatozoa are present and
keep the specimen for DNA analysis later. For legal purposes, there would need to be a
strict 'chain of security and continuity' of handling the specimen if the results are to be
admissible in court.
Offer referral for rape crisis counselling with the rape crisis team or medical social
worker.
If the candidate prefers to refer the patient to a doctor in a rape crisis centre immediately,
a summary as above must be given, indicating what action would be expected from the
staff at the rape crisis centre.
KEY ISSUES
Ability to assess a patient who has recently been raped.
Ability to arrange the appropriate followup investigations and care.
CRITICAL ERRORS
Failure to consider need for post-rape contraceptive methods and management.
Failure to refer to appropriate clinic or to discuss taking appropriate swab to exclude
sexually transmissible infections (STI) and taking specimen for DNA analysis.
Failure to consider use of prophylactic antibiotics to prevent pelvic inflammation with
an STI.
COMMENTARY
This patient will need to be treated with great empathy and support. As she does not wish
to involve the police at this stage, there may be no indication to collect forensic specimens,
but it would be appropriate to preserve any specimens collected in case she changes this
decision. She needs a full explanation as to the reasons for your examination (to exclude
trauma and STI), and she may well need the support of a social worker or a rape crisis
counsellor. She needs explanation that pregnancy might occur as a result of the rape, and
that the time of the incident in her menstrual cycle should be established, as well as the
possible use of post-coital contraception to prevent conception. She should also be
counselled that a review of testing in three months time will be necessary to follow up from
the initial potential infection. Antibiotics should be given as prophylaxis against STI.
Common problems likely with candidate performance are:
failure to review to exclude pregnancy and to arrange followup infection testing; and
failure to arrange appropriate counselling.
092
011
Performance Guidelines



Condition 011
Cancer of the colon in a 60-year-old man
AIMS OF STATION
To assess the candidate's ability to counsel an anxious patient, recently diagnosed with colon
cancer. Knowledge of the clinical presentation and natural history of carcinoma of the colon is
required, including the clinical features of left colon carcinoma, and the necessity for urgent
operation should acute bowel obstruction occur, in which case, a temporary colostomy may be
performed.
EXAMINER INSTRUCTIONS
The examiner will have instructed the patient as follows:
Your father died from bowel cancer in his late sixties. You were advised about followup periodic
colonoscopy, but as you had no symptoms, you did not attend for further studies after the first
one five years ago, which you were told was clear.
You recently attended for a further screening colonoscopy at your wife's insistence. You have
had no bowel symptoms, and no general symptoms. You were shocked and upset when told by
the gastroenterologist that a bowel lesion suspicious of cancer had been found, and that the
biopsy confirmed that cancer was present. You have been advised that referral for early surgery
is required, but you dread this prospect.
Now that the implications of the positive diagnosis have sunk in, you wish to discuss matters with
your general practitioner to check what is likely to occur if you continue to refuse surgery,
whether any other treatment is possible, and the likelihood of a colostomy being required.
Opening statement
' What will happen if I don't have surgery?'
Questions to ask unless already covered:
'What are the prospects of cure if I have the operation?
1

Is any other treatment apart from surgery possible
9
'
'Would I need a colostomy like my father had?'
At the conclusion of the interview if your concerns have been addressed adequately, thank the
doctor and ask him to arrange surgical referral.
EXPECTATIONS OF CANDIDATE PERFORMANCE
The candidate would be expected to know the natural history and clinical presentations of left
sided colon cancer as outlined in the commentary, and to address the patient's concerns
appropriately.
011
Performance Guidelines
093
KEY ISSUES
Discussion of natural history of left sided colon cancer if not treated.
The candidate is expected to know:
~that urgent operation is necessary if acute large bowel obstruction with caecal
distension ensues; and
~the general principles of surgical management for a rectosigmoid cancer.
CRITICAL ERRORS
Failure to counsel patient on natural history of untreated colon cancer.
Failure to advise need for urgent operation in the event of acute obstruction.
COMMENTARY
If surgery is not performed the tumour may become evident due to bleeding, bowel obstruction
or tumour spread.
Carcinoma of the left colon may present with the passage of red blood mixed with the stools (or
even bleeding apart from defaecation), increasing constipation, or alternating constipation and
spurious diarrhoea. If obstruction develops, subumbilical central pain ensues with persisting
constipation. If acute complete obstruction ensues in the presence of a competent ileocaecal
valve, then distension will follow with increasing pain. The risk of rupture of the caecum means
urgent operation is essential to decompress the bowel from the 'closed loop' obstruction.
Nasogastric tube suction and intravenous fluid replacement alone is incorrect management. A
colostomy may need to accompany emergency surgery at the surgeon's discretion, but would
usually be temporary unless operation revealed extensive unexpected tumour spread.
CONDITION 011. FIGURE 2. CONDITION 011. FIGURE 3.
Bowel obstruction gross caecal distension Metastatic carcinoma liver
094
011
Performance Guidelines



Cancers commonly arise in premalignant adenomatous polyps. The likelihood of malignant
change increases with increasing polyp size.
Spread of the tumour occurs by direct invasion through the bowel wall, which can be followed by
malignant ascites and peritoneal metastases.
Lymphatic spread in left colonic cancer, which significantly worsens prognosis, is via epicolic,
paracolic and preaortic nodes.
Blood spread occurs via the portal system to the liver and beyond. The prognosis after resection
of colon carcinoma relates directly to the degree of spread of the carcinoma as indicated in
Dukes classification:
Stage A: carcinoma confined to the mucosa, over 95% five year survival;
Stage B: carcinoma involves the muscle of the colonic wall, 75-80% five year survival:
Stage C: lymph node involvement, 50% five year survival; and
Stage D: spread into the peritoneal cavity or by blood spread to the liver and beyond,
25-35% five year survival.
Elective surgery on confirmation of diagnosis offers the best prospect of cure, and colostomy is
not usually required. If a colostomy is performed under conditions of elective surgery, it would
normally only be temporary.
In low left sided colonic cancer, adjuvant chemotherapy has been shown to improve survival in
selected cases.
Preoperative investigations normally check for general fitness for anaesthesia, absence of
anaemia and normal renal function. Abdominal imaging by computed tomography (CT) can
provide information regarding intra-abdominal spread as an aid to whether curative resection is
likely to be possible, and should be advised in this concerned patient to facilitate informed
consent for surgery.
Perioperative prophylactic antibiotics can minimise infective complications of surgery.
012
095
Performance
Condition 012
Thalassaemia minor in a 22-year-old woman
AIMS OF STATION
To assess the candidate's knowledge of the Mendelian inheritance of / -Thalassaemia minor, its
management, and the candidate's counselling skills in dealing with a sensitive issue involving her prospective
spouse.
EXAMINER INSTRUCTIONS

The examiner will have instructed the patient as follows:
The doctor will discuss the results of the tests and will explain to you the implications of you being a carrier of
the condition of / -Thalassaemia. You should be reassured that the condition is not serious to your own
health, but there are inheritance implications which will depend on your fiance's genetic status. If the doctor
does not suggest testing your fiance, ask:
Will this have any effects on our children?'
Should my fiance have any test done?'
EXPECTATIONS OF CANDIDATE PERFORMANCE
Candidates are expected to know that Thalassaemia is a recessive inherited trait so that the heterozygous
(carrier) state gives Thalassaemia minor, whilst the homozygous state results in Thalassaemia major.
Approach to patient:
~ This scenario deals with a sensitive issue, namely that if the patient's fiance is also a carrier, future
children may be affected, with a significant risk of / -Thalassaemia major.
~ Minimisation of anxiety by careful explanation, a supportive attitude and with guarded reassurance about
present day management is expected.
Interpretation of investigations:
The characteristics of / -Thalassaemia minor include:
~ FBE a symptomless hypochromic microcytic anaemia (rarely below 100 g/L) with decreased mean
corpuscular volume (MCV) and usually normal red blood cell count (RBC);
~ diagnosis needs confirmation by serum electrophoresis (elevated Hb A
2
); and
~ the anaemia does not respond to any form of iron therapy, unless the patient happens also to be
iron-deficient, which is very rare.
Initial management plan:
~ Reassure the patient regarding the effect on her health. Iron therapy is not indicated. Oral folic acid of 1 mg
per day will more than meet the requirements of the mildly increased red cell turnover.
012
Performance Guidelines



~ The candidate should offer to see her fiance to arrange appropriate testing, advise that the fiance
consults his own doctor, and offer to refer the couple to a geneticist or haematologist. Suggest also that
her brother should seek advice in regard to testing for the trait. If the fiance is not a carrier there is no
immediate problem, although the couple should be advised to inform their children of the situation at an
appropriate time, including how the carrier state is diagnosed.
~ This problem cannot be resolved at this consultation.
Followup with patient and also fiance (if possible) is required after results of fiance's genetic status are known.
Patient counselling/education:
The candidate should counsel regarding the following issues:
~ Thalassaemia minor is a recessive inherited trait ('carrier' state).
~ Nature of this condition and possible consequences if conjugal partner also carries the trait.
~ Information about -Thalassaemia major and risks of its occurrence in offspring if both partners are
carriers. Availability of antenatal diagnosis and management of pregnancy (including termination) if
fetus is shown to have -Thalassaemia major.

Patient counselling regarding inheritance implications.
Initial management plan.

Failure to advise that the prospective spouse should be investigated for carrier state.
Failure to understand principles of Mendelian recessive inheritance.

Thalassaemia is a common anaemia in certain areas of the world, including the Mediterranean region, India,
Southeast Asia and Africa. In Australia, there is a high prevalence of this condition in individuals descended
from these regions. At the molecular level, there are hundreds of globin abnormalities leading to either
alpha-chain or beta-chain underproduction. The underproduction of either alpha or beta chains results in a
microcytic red blood cell. There is also haemolysis because the imbalance results in excess globin chains,
which are oxidised and result in premature red blood cell removal in the spleen. -Thalassaemia major is a
rare, but very serious, congenital anaemia requiring lifelong transfusional support and patients with this
condition also need treatment to avoid complications of iron overload related to the frequent transfusions.
There is a markedly reduced life expectancy with devastating consequences for both the child and family.
The most important points being assessed are:
Knowledge of Mendelian inheritance (what is a recessive inherited trait and what are its implications?).
Meaning of
~ heterozygous inheritance: results in -Thalassaemia minor; and
~ homozygous inheritance: results in B-Thalassaemia major.
Advise that fiance be tested for -Thalassaemia minor
096
012
Performance Guidelines



The pattern of inheritance is as illustrated. She has the carrier state /J-Thalassaemia minor.
If fiance is negative for -Thalassaemia minor;

Either sex can be carriers. Half the children of either sex will be carriers ( -Thalassaemia minor).
None should have -Thalassaemia major.
If fiance is positive for -Thalassaemia minor:

Half the children of either sex will be carriers. One in four children will have -Thalassaemia major, which affects
children of either sex.
Management
If the haemoglobin test for -Thalassaemia minor in the fiance is negative:
~ No further action or tests are required.
~ Explanation of carrier state is required (can affect male or female children) each offspring will have an equal
chance of being carrier or noncarrier (normal).
~ If the fiance has a positive test for -Thalassaemia minor, they will need counselling about risks to fetus which
are 1:4 for -Thalassaemia major (25%) and 1:2 for -Thalassaemia minor (50% carrier status).
The diagnosis of -Thalassaemia major can be made by in utero genetic sampling at 12-14 weeks. If both partners
are carriers there is no cause for undue alarm because of increased awareness, diagnostic certainty, known risks and
availability of antenatal diagnosis and safe termination procedures (unless this is not an acceptable option).
097
PerformanceGuidelines

Beta-Thalassaemia is a condition involving the beta ( ) chains, with both homozygous and
heterozygousstatesbeingcompatiblewithsurvivaltotermbutwithmajordifferencesinoutlook.
Heterozygousalpha-Thalassaemia(whichinvolvesthealpha( a ) chains)isalsocompatiblewith
survival; however the homozygous state of alpha-Thalassaemia results in fetal hydrops and
death-in-utero.

CONDITION012.FIGURE3.
-Thalassaemiamajor:theslideshowsmicrocytosis,
anisocytosls,hypochromia,anormoblastandtargetcells
098

013
PerformanceGuidelines
Condition 013
Pre-pregnancy advice to a 28-year-old woman with previous
thromboembolism
AIMSOFSTATION
Toassessthecandidate'sabilitytoadviseapatientoftheappropriatemanagementsheshould
receive during her next pregnancy, when the last one was complicated by a pulmonary
embolusduringthepuerperium.
Because of her previous deep venous thrombosis (DVT) and pulmonary embolus, she is at
increasedrisk(probablyatleast20%)ofarepeatthrombosisinhernextpregnancy.
Theexaminerwillhaveinstructedthepatienttorespondasfollows:
Previous DVT and pulmonary embolus: your pulmonary embolus occurred on day three
after the birth. After this DVT in the right pelvic veins was diagnosed when the doctor
performedavenogram.
You were treated with intravenous heparin for two weeks, in full dosage, followed by full
dosewarfarintherapyforsixmonths.
Youhavenowbeenoffalltreatmentfor12months.
Contraception: you are using condoms. You had previously been on the contraceptive pill
priortothefirstpregnancywithoutproblems.
YouhavehadnopreviousDVTorpulmonaryembolusapartfromasoutlinedintheprevious
pregnancy.
Youhavehadnotraumaoroperationsonlegsorpelvis.
Thereisnoshortnessofbreathcurrently.
There are no varicose veins, but you do have occasional ankle oedema on the right side
(thesideofthepreviousthrombosis).
Your previous delivery was a spontaneous vaginal delivery of a live male infant weighing
3550grams.Asmallepisiotomywascuttofacilitatedelivery.Youdidnotbreastfeed.
Youhavenofamilyhistoryofanyclottingproblemsorthromboses.
During the last pregnancy, you were shown to be rubella-immune and took folic acid from
beforepregnancyandforthefirst16weeks.
Questionstoaskunlessalreadycovered:
Wi l l Igetanotherdeepvei nthrombosi sandpul monaryembol us?'
' DoIneedanti coagul anttreatmentduri ngthenextpregnancy?Ifso,what?'
Isthereanyri skfromanti coagul anttreatmentduri ngpregnancytomybabyorme?'
Investigationresults
Noinvestigationshavebeendonesinceyouceasedwarfarintherapyandnonehadbeendone
priortocommencinganticoagulantsafterthepulmonaryembolism.
099
013
100
PerformanceGuidelines



EXPECTATIONSOFCANDIDATEPERFORMANCE
Thecandidateshouldprovidethesubstanceofthefollowinginformationinlayterms:
The pregnancy should be managed in consultation with a consultant physician or
haematologistandanobstetrician.
There isaneedtoscreenherforaclottingpropensity(thrombophilia)priortoherbecoming
pregnant (tests should include the measurement of anticardiolipin antibody, lupus
anticoagulant, protein S, protein C, anti-thrombin 3, and factor V Leidin these tests will
screen for both inherited and acguired thrombophilias). If a thrombophilia is identified,
anticoagulanttherapythroughoutthepregnancyandthepuerperiumshouldbeadvised.
Even if there is no underlying problem discovered, she should still be treated with
anticoagulants at least during the puerperium, and it is safer to treat her throughout the
pregnancy. The treatment should therefore probably be started at about 14 weeks of
gestationandcontinueduntilatleast4-6weekspostpartum.
Optimalanticoagulanttherapyisbestgivenassubcutaneouslowdoseheparininadosageof
7500unitstwicedaily,orwithalowmolecularweightheparinsuchasenoxaparin,inadose
of20-40mg12hourly(inthepastitwouldhavebeencommonforheparintobechangedto
warfarin, in full dosage, between 16 weeks and 36 weeks, and then from one week
postpartum). Treatment with warfarin, however, has problems such as a 5% risk of
teratogenesiswhengiveninthefirsttrimesterandanincreasein miscarriagerate,fetaland
maternal haemorrhage, neurologic problems in the baby and stillbirth. For all of these
reasons,warfarinisnotusedduringpregnancy,althoughitiscommonlyusediftreatmentis
continuedpostpartum.
Avoidprolongedimmobilisationduringpregnancy,andconsiderusingcompressionstockings
bydaythroughoutthepregnancy.
Deliver in a controlled manner at about 38-39 weeks of gestation, rather than allowing a
spontaneouslabourtooccurinsomeonewhohasjusthadherdoseofheparin.Althoughthe
heparin dose is not full dosage, and unlikely to affect clotting tests, it is best to arrange
induction at a time when the morning dose of heparin can be withheld, with the treatment
beingreinstitutedafterdelivery.
Takeherfolicacidtherapyasonthepreviousoccasion.

Ability to recognise that she is at increased risk of a recurrent thrombosis in her next
pregnancyandrequiresatleastlowdoseheparinduringthepuerperiumifnotformostofthe
antenatalperiodaswell.
Recognitionofrelativerisksandindicationsforheparinandwarfarintherapy.
Recognitionofsignificantrisksofwarfarinduringpregnancy.

Failuretoscreenforaninheritedoracquiredcoagulationdisorder.
Failuretoadviseanticoagulanttherapyatleastfor4-6weekspostpartuminthenext
pregnancy.
Advisingwarfarintherapythroughoutpregnancy.
101
013
Performance
Guidelines



COMMENTARY

This young woman has survived a life-threatening pulmonary embolus in a previous


pregnancy. She must receive treatment to prevent a similar episode occurring in her next
pregnancy.Itisessentialtoexcludeanyacquiredorinheritedcoagulationdisorder.However,
in any event, she will need anticoagulant therapy for the majority of her next pregnancy. The
benefits of heparin must be carefully explained, as opposed to the risks of warfarin in
pregnancy.Itisalsoimportantthattheanticoagulanttherapybecontinuedbeyondthebirthof
herinfantforatleast4-6weeks.
Commonproblemslikelywithcandidateperformanceare:
failuretoenquireabouttheparticularsofthepreviousthrombosis/pulmonaryembolism.
suggesting that warfarin therapy should be given throughout the pregnancy, which can be
teratogenicifgiveninthefirsttrimester,andisimpossibletoreversequicklyinthelatethird
trimesterwhenlabourislikelytooccur


014
102
PerformanceGuidelines
Condition 014
Pre-pregnancy advice to a 24-year-old woman with
Type1diabetesmellitus
AIMSOFSTATION
Toassessthecandidate'sabilitytocounselanulliparousdiabeticwomanabouttheeffectsofher
diabetesonafuturepregnancy,theeffectoffuturepregnanciesonherdiabetes,andtheactions
sheshouldtaketominimisetheriskofpotentialproblems.

Theexaminerwillinstructthepatienttoreply,asindicatedbelow,whenherhistoryisbeingtakenby
thecandidate.
Yourdiabeteswasdiagnosedattheageof9years(thatis,15yearsago).
Yourdiabeteshasbeenwellcontrolledoninsulinandiscurrentlyverywellcontrolledonlong-acting
insulingiventwicedaily.
Your last review by the diabetic physician was 12 months ago. You manage your own insulin
dosagesandtestbloodsugarlevelsthreetimesaday.Generallyyouareabletokeeptheblood
sugarlevelsat6-8mmol/L.
Youhavenotbeentroubledwithurinaryinfectionsorvaginalinfections.
Yourvisionhasbeengoodandkidneyfunctionhasbeennormal.
Youhaveneverhadahyperglycaemiccoma,butoccasionallyhavehypoglycaemicepisodes.
Youhavenothadanyoftheotherpre-pregnancybloodtestsdone,suchasrubellaantibodies,full
bloodexamination(FBE),bloodgroup,indirectCoombstestorsyphilisserology.
Questionstoaskifnotalreadycovered:
' Wi l l Ibeabl etohaveababy?'
Wi l l mybabybeOK?'
Wi l l thepregnancyadversel yaffectmydi abetes?'
' Howdoesthedi abetesaffectmypregnancy?'
EXPECTATIONSOFCANDIDATEPERFORMANCE
Ingeneral,thecandidateneedstoadviseheroftheneedtohaveverygoodbloodsugarcontrolpriorto
conception and during the pregnancy, that the pre-pregnancy counselling applicable to all pregnant
womenisalsoapplicabletoher,thattheriskofcertainpregnancycomplicationsareincreased,thatthe
baby is also at increased risk, and that delivery at 38 weeks or sometimes earlier will probably be
required.

014
Performance Guidelines
Specific advice to the patient
The candidate should explain that:
Pre-pregnancy counselling must take the form of assessment of her diabetic control and looking for any evidence
of effects of the diabetes on target organs. Her haemoglobin A1C (HbA1c) should therefore be measured and her
blood glucose assessments reviewed to ensure they are satisfactory. It is particularly important to keep the blood
glucose levels within the range of 5-7 mmol/L during the first trimester to reduce the risk of fetal malformation to an
absolute minimum, and during the remainder of the pregnancy to reduce the fetal and maternal complications
(particularly macrosomia and unexplained fetal death).
Referral to her diabetic physician to check her general state is mandatory, with assessment for peripheral
neuropathy, renal function (renal function tests and 24 hour urinary protein), and a check of her optic fundi by her
ophthalmologist.
Providing all of these are normal, she could be advised to attempt to become pregnant, and commence folic acid
in a dosage of 0.5-1 mg per day from the time pregnancy is attempted until at least mid-gestation.
In addition, the routine tests which would normally be performed at the first antenatal visit are better performed
prior to pregnancy. These tests should include blood group and indirect Coombs test, full blood examination
(FBE), hepatitis screening, Venereal Disease Research Laboratory Test (VDRL). rubella serology, and midstream
urine specimen to exclude urinary infection. If a Pap smear has not been performed in the last two years, this
should also be performed. Any abnormalities found in these tests should be addressed prior to the pregnancy.
She should also be advised about the care which is likely to be required during pregnancy as follows:
~ Referral to a consultant obstetrician who will manage her in conjunction with a diabetic physician.
~ Her insulin requirement will markedly increase and she will need to keep her blood sugars between 5-7 mmol/L
to keep the fetal malformation rate to a minimum, and the macrosomia (large fetal size) rate to an acceptable
level.
~ The insulin requirement after delivery usually returns to pre-pregnancy requirements within 24 hours of delivery,
~ Ultrasound examination should be performed at 12 and 18 weeks looking for fetal abnormalities, and at 32
weeks looking for macrosomia.
~ Iron and folic acid therapy should be continued throughout the pregnancy.
~ In general, even where the diabetes is well controlled, delivery should be planned for 38-40 weeks in someone
who has had diabetes for 15 years and is on insulin therapy. Earlier delivery might be necessary if problems
occur during the pregnancy.
~ Despite adequate monitoring and care during pregnancy, the pregnancy is more likely to be complicated by
pre-eclampsia, polyhydramnios and macrosomia of the fetus. There is also an increased risk of unexplained
fetal death-in-utero late in pregnancy and of respiratory distress in the baby after delivery. All of these matters
need to be raised with the patient prior to pregnancy, so that she can make an informed decision as to whether
she wishes to proceed.


103

014
Performance Guidelines
KEY ISSUES
Ability of the candidate to counsel a diabetic woman, prior to a pregnancy, about the care she will require and the
methods of keeping the complication rate to herself and her fetus to a minimum.

Failure to ensure the patient is aware that her diabetic control prior to pregnancy should be good to ensure the
risk of fetal abnormality is kept as low as possible.
Failure to do pre-pregnancy blood tests (haemoglobin estimation, blood group, rubella antibodies and tests as
above) and failure to recommend pre-pregnancy and early pregnancy folic acid therapy (routine pre-pregnancy
counselling).

The most important aspect of managing pregnancy in an established insulin-dependent (Type 1) diabetic is that the
pregnancy must be managed in consultation with a physician specialising in diabetes and a specialist obstetrician.
The most important advice that the patient should receive is that pre-pregnancy and pregnancy control of blood sugar
levels is essential for an optimum outcome of the pregnancy.
Common problems likely with candidate performance are:
inability to advise the patient adequately concerning the special problems seen in pregnancy that can affect
either the baby or the mother;
lack of knowledge that good blood-sugar controi reduces the risk of unexplained fetal death-in-utero and fetal
macrosomia;
lack of knowledge that the insulin dose required will usually increase dramatically during the pregnancy, but fall
back to pre-pregnancy requirements within 24 hours of delivery; and
failing to suggest that the pregnancy should be managed in consultation with a diabetic physician and a specialist
obstetrician.



015
105
Performance Guidelines
Condition 015
An anencephalic fetus diagnosed at 18 weeks gestation in a 25-year-old
primigravida
AIMS OF STATION
To assess the candidate's ability to appropriately counsel a patient found to have an
anencephalic fetus at the time of an ultrasound examination at 18 weeks of gestation.

The examiner will have instructed the patient as follows:
This is your first pregnancy at 18 weeks. You are seeing the doctor to discuss your recent ultrasound
test. Screening blood tests two weeks ago raised concerns about the fetal condition.
The list of appropriate answers below is likely to cover most of the doctor's questions:
if asked about your periods, indicate they had been normal prior to the pregnancy;
you did not take any folic acid in early pregnancy;
no family history of neural tube defect (spina bifida,) in pregnancy;
if after explaining the diagnosis, the candidate explains termination of the pregnancy as an
appropriate action, or asks you what you would prefer, answer: I do not wish to continue with the
pregnancy if this can be arranged';
your blood group is O positive, indirect Coombs test negative; and
there is no history of asthma or other contraindication to prostaglandin therapy.
Opening statement
Is there anything wrong with my baby?'
Questions to ask if not already covered:
'Can I have the pregnancy terminated?' Ask only if the candidate informs you of the diagnosis
but doesn't mention termination of the pregnancy being an option.
Is this problem likely to occur again in a subsequent pregnancy?'
'How can I prevent the problem from occurring again?'
EXPECTATIONS OF CANDIDATE PERFORMANCE
Diagnosi
Fetal anencephaly is a developmental defect of the brain which occurs somewhere between
five and eight weeks of gestation. In this patient, the condition has been diagnosed
unequivocally at the 18 week ultrasound as illustrated. The condition is always fatal soon after
birth. The patient has the option to terminate the pregnancy forthwith, or continue until labour
occurs. If the latter is chosen, and if hydramnios occurs, the labour is likely to be premature,
otherwise labour post-term is common.



015
106
PerformanceGuidelines
Adviceexpectedtobegiventothepatient:
Theinformationconcerninganencephalyshouldbegivenasindicatedabove.
Asthepatienthasindicatedsheshouldwishtohavethepregnancyterminated,thisneedsto
bediscussed.Terminationofthepregnancycouldbeperformedbyusingprostaglandins,or
by the surgical procedure of dilatation and evacuation. This latter procedure has the
advantage of being performed under general anaesthetic with the procedure being over
when the patient wakes up. This procedure is quite difficult except in expert hands, and
cervical damage resulting in subsequent cervical incompetence in any subsequent
pregnancymayresultwhentheprocedureisdoneafter16weeksolgestation.
Prostaglandin termination may take several hours or even days and results in uterine
contractionssimilartothoseexperiencedinlabour,followedbyvaginaldeliveryofthefetus.
Thereisalsoapossibleneedforcurettagetoremoveanyretainedplacentalfragments.
Post-mortem examination should be performed on the fetus to check that no other
abnormality is present that might influence the advice given concerning the successor
otherwiseofanysubsequentpregnancy.
Theriskofrecurrenceofaneuraltubedefect(NTD)suchasanencephaly,orspinabifida,is
somewhere between 2% and 5%. Folic acid administration (in a dosage of 5 mg per day)
shouldbecommencedpriortoconceptionandcontinueduntilabout12weeksofgestation,
asthishasbeenshowntoreducetheriskofaneuraltubedefectUltrasoundexaminationina
subsequentpregnancyisimperative.
Maternalserumalphafetoproteinassessmentisalsousefulasascreeningtestandshould
beperformedatabout16weeksofgestationinanysubsequentpregnancy.

Abilitytoadvisethemother:
empathicallyofthefactthatherbabyhasalethalabnormality;
oftheappropriateoptionsregardingherfurthercareinthispregnancy;and
ofthemethodsavailabletoreducetherecurrenceriskofthisabnormality

Failureto:
recogniseandadvisethepatientthatthisisalethalabnormalitytothebaby:or
determinethepreferencesofthemotherinrespecttoterminationofpregnancy;or
counselthepatientappropriatelyconcerningmanagementinasubsequentpregnancy.

The most important aspect of managing this case is to understand the anxiety and
disappointment of the mother. She will need considerable support and understanding when
discussingtheabnormalitywithherandthemanagementoftheterminationofthepregnancy
that she has requested. While helping her to deal with the extreme disappointment of the
outcomeofthispregnancy,thecandidateshouldbepositiveintermsofprevention(folicacid)
andscreeningtestsinasubsequentpregnancy.

015
Performance
Guidelines



Commonproblemswithcandidateperformanceare:
Notfocusingenoughontheactualproblemwhentakingthehistory,butaskingforinformation
suchasthedateofthelastmenstrualperiod,irrelevantpasthistory,socialhistoryetc.This
justtakestimetodoandreducesthetimeavailablefortheremainingtasks.
Advisingthepatientthatasuctioncurettewouldbethepreferredmethodoftermination.This
is not the case at 18 weeks of gestation, although it would be appropriate for a pregnancy
terminationbeingperformedatlessthan15weeksofgestation.
Advising that maternal serum screening at 11-12 weeks gestation in the next pregnancy
wouldbeappropriatetoexcludeanotherNTD.Thistestwouldbeappropriatetoassessthe
likelihoodofachromosomeabnormalitybutanalpha-fetoproteinassessmentat15-16weeks
gestationisalsonecessaryforrecognitionofalikelyNTD.Earlierultrasoundexamination,at
11-12weeksofgestation,mayallowadiagnosisofanencephalytobemade,butwouldnot
excludespinabifida.Anunderstandingoftherecentmethodsandtimingofgeneticscreening
inearlypregnancyisrequired.
CONDITION015.FIGURE2.
Ultrasoundshowinganencephalicfetus
107
108
016
Performance Guidelines



Condition 016
A duodenal ulcer found on endoscopy in a 65-year-old man
A I M S OF STATION
To assess the candidate's ability to counsel a patient on the aetiology and complications duodenal peptic
ulceration and the principles of management.
EXAMINER INSTRUCTIONS
The examiner will have instructed the patient as follows:
You are a retired bank clerk. You have had dyspepsia (epigastric pain after meals) for abo six weeks. Your
general health has been reasonably good apart from some rec generalised aches and pains in your joints.
You have been taking Nurofen"
1
table (ibuprofen), which you obtained from the pharmacist, which have not
given much relief, i do not smoke or drink alcohol.
Questions to ask if not already covered by doctor:
'Why have I developed the ulcer?'
Is it contagious? Can my family members be affected?'
'What treatment do I need to get rid of this duodenal ulcer?'
'Would those NuroferP tablets have anything to do with this?'
Is it likely to come back once it has healed? If so, can this relapse be prevented?'
Is the ulcer cancerous?'
'Would surgical treatment ever be required?'
EXPECTATIONS OF CANDIDATE PERFORMANCE
The candidate should explain that:
The majority of duodenal ulcers are a consequence of mucosal damage caused Helicobacter pylori and
gastric hydrochloric acid. The relationship of H. pylori a gastric acid to gastric ulcer is less definite.
Non-steroidal antiinflammatory dru (NSAIDs) commonly contribute to peptic ulcer formation.
Management of peptic ulc depends on the underlying cause. For Helicobacter pylori-related ulcers,
triple ther (usually a proton pump inhibitor and two antibiotics) will heal more than 90% of ulc within one
week of starting treatment and, provided the organism is eradicated (u breath test at six weeks), the
chance of ulcer recurrence is low. For NSAID-relat ulcers, the patient will only require acid suppression
to achieve ulcer healing.
Recommended therapy for an H. pylori-related ulcer includes a proton pump inhib' (for example,
omeprazole) plus antibiotics (clarithromycin or amoxycillin plus metr dazole given for 1-2weeks),
followed by continued antisecretory therapy foruptoat of 4-8 weeks.
Cessation of potential aggravating factors such as NSAID use and smoking is important. Whilst tobacco
and alcohol may hinder the healing of ulcers, they don t contribute to the development of the ulcer. The
patient should replace ibuprofen alternative analgesia such as paracetamol. The bacterial infection is
not highly contagious to other family members.
016
109
Performance Guidelines
Peptic ulcers are common, more so in men. About 70% are related to H. pylori. Left untreated, most ulcers
will spontaneously heal, but relapse, often after several months. Thus peptic ulcer disease can be a lifelong
problem, with about 10% of patients developing complications (bleeding, perforation or gastric outlet
obstruction).
Most patients can be managed with medical therapy and without surgery. Complications of peptic
ulceration, which require surgical intervention, include perforation, gastric outflow obstruction and severe
persistent bleeding. Malignancy occurring in a chronic peptic ulcer is uncommon (1-2% in gastric ulcer) but
would need examination and monitoring to ensure permanent healing. Haemorrhage from peptic ulcers
produces haematemesis and/or melaena. Bleeding is usually self limiting, but more likely to persist in older
patients with arterial medial sclerosis. In these patients, endoscopic haemostasis using a heater probe or
injection of adrenaline, is often successful in preventing further bleeding. Surgery is indicated when
excessive or persistent bleeding occurs and should be considered when blood loss exceeds 3000 mL.
However, each patient should be evaluated individually.

Knowledge of the usual medical therapy and risk factors for patients with peptic duodenal ulceration.
Understanding that surgery is required for perforation, obstruction and for patients with severe, persistent
bleeding or intractable pain.
With the advent of the proton pump inhibitors and the identification of the pathogen Helicobacter pylori, the
management of peptic ulcer disease has undergone a considerable change in the last two decades. From
being a chronic, indolent problem, frequently requiring surgical intervention, peptic ulcer disease is now a
relatively benign and easily treated condition.

Informing the patient that the duodenal ulcer is likely to be malignant.

The mucosal lining of the stomach and duodenum is normally protected from acid-attack and autodigestion by
a layer of mucus. This layer of mucus forms an 'unstirred layer' which is constantly replaced by secretion from
the underlying cells. Bicarbonate is also secreted from the surface cells to help with mucosal defence. Both
mucus and bicarbonate secretion are modulated through the actions of prostaglandins, cyclic AMP (cAMP)
and several other agents.
There are a number of ways in which the integrity of this mucous layer can be broken. Replenishment of the
layer can be interrupted through interference with prostaglandin synthesis and reduction in bicarbonate
secretion. This is an important side effect of NSAIDs. The layer itself may be disrupted and acid allowed to
infiltrate and damage the underlying mucosa. The most common mechanism of this kind of damage is brought
about by the presence of Helicobacter pylori. To survive in the hostile acid environment, this bacteria surrounds
itself in a neutral zone, brought on by the breakdown of urea (through the enzyme urease) and formation of
ammonium ions. This change weakens the integrity of the mucous layer.
016
Performance Guidelines
110
CONDITION 016. FIGURE 2. CONDITION 016. FIGURE 3.
Helicobacter pylori associated peptic Urease test on biopsy with sample
ulceration
The existence of H. pylori and its role in peptic ulcer disease was only appreciated in the early 1980s, mainly
through the seminal contributions of B Marshall and R Warren from Perth, Australia, leading to their receipt of
the Nobel Prize in Medicine for 2005.
On a worldwide basis, most peptic ulcers will be related to the presence of H. pylori. Whilst the prevalence of
H. pylori is high in many communities especially amongst those in lower socioeconomic groups the rate
of ulcer formation is still relatively low. When a peptic ulcer is not related to the presence of H. pylori, the most
likely aetiological factor will be an NSAID. A great deal of false expectation occurred with the development of
the Cyclo-oxygenase-2 (COX-2) inhibitors (such as celecoxib), from belief that this class of compound would
be associated with a lower incidence of gastrointestinal side effects. This has not proven to be the case.
Ideally, all patients with a suspected peptic ulcer should have an endoscopy performed. The aim of the
procedure is two-fold: first, to confirm the diagnosis; and secondly, to test for the presence of H. pylori. As part
of the diagnosis, any ulcer in the stomach must be biopsied to exclude malignancy. Duodenal ulcers are not
associated with malignancy.
If H. pylori is present, part of the treatment will be to eradicate the infection. Provided this is done and the ulcer
is healed, the chance of recurrence is low. If the ulcer does recur this is probably because the patient has
become reinfected. If H. pylori is absent, the likely cause of the ulcer is an NSAID. Treatment consists of
withdrawing the NSAID and giving the patient an acid-suppressing agent for long enough for the ulcer to heal.
Treatment for 4-8 weeks will lead to H. pylori eradication and ulcer healing in over 90% of cases.
If the initial ulcer was a duodenal ulcer, success of treatment can be judged by relief of symptoms and a breath
test to check for H. pylori eradication (if the infection was present in the first place). If the cause of the problem
had been a gastric ulcer, the patient should be endoscoped again, and the ulcer biopsied again if still present.
017
Performance Guidelines
111
Condition 017
Advice on autologous blood transfusion to a 55-year-old man awaiting elective
surgery
AIMS OF STATION
To assess the candidate's ability to explain the principles of preoperative blood collection for autologous
intraoperative transfusion.
EXAMINER INSTRUCTIONS
The examiner will have instructed the patient to ask questions of the candidate as follows (unless
the matter has already been covered):
The answers expected by the candidate are in brackets.
Opening statement:
'Are there any advantages in using my own blood it I need a transfusion?' (Specifically minimising infective and
incompatibility risks)
Questions to ask if not already covered:
'How long is the blood good for?' ( Up to five weeks)
'How much do they take?' (Up to 2 litres over a period of 2-5 weeks)
'Don't I need all my own blood?' (Your blood rapidly regenerates, being a renewable tissue from the bone
marrow)
'Won't it make me very weak?' (Not significantly)
'What are the advantages of my blood over blood bank blood?''(It is your own. which is fully compatible).
EXPECTATIONS OF CANDIDATE PERFORMANCE
The candidate should display ability to counsel about the following issues:
The candidate is expected to explain the normal functions of bone marrow and blood, and that blood is a
renewable tissue with the main functions of oxygen transport to tissues and maintenance of circulatory
volume.
Blood donors can give readily 10-15% of the blood volume (450 mL) and the fluid volume is rapidly replaced
within hours from the body's reserves. Replacement of the red cells starts immediately after donation and
the slight anaemia resolves within weeks as the blood cells are replaced.
Blood can be stored safely for reuse within a few weeks (up to five weeks), so that one. two or more
donations can be collected over the weeks prior to operation. The blood units can then be stored and saved
to be used during operation as whole blood or reconstituted red cells to replace operative blood loss
requirements in operations (such as this patient is having) on large joints or blood vessels, where blood loss
can be such as to require transfusion.
The procedure removes the risk of disease transmission (particularly by viruses) or incompatibilities and
allergies inherent in standard homologous blood transfusion from another donor.
1 1 2
017
Performance Guidelines
The procedure is recommended in patients with no medical contraindications to blood
donation, who are to have a planned major elective surgical operation on a defined date some
weeks ahead at which blood losses are expected to be moderate or high.
The procedure of donation does not cause any increase in liability of operative problems or
complications and the modestly lowered haemoglobin level causing temporary 'thinning' of
the blood may be protective against thrombotic clotting complications such as deep venous
thrombosis, pulmonary embolism, heart attack or stroke, because the thinned blood flows
more freely.
The procedure would not necessarily remove all likelihood of requiring an additional
non-autologous transfusion, which will depend on volume of operative loss, but will
significantly reduce such prospects. Non-autologous/homologous blood transfusion itself is
used to save many thousands of lives each year, and although not entirely free of risk, is
rigorously supervised and a safe procedure in Australia.
Preoperative autologous blood collection for subsequent operative use depends on a verified
and non-cancellable time for surgery, so there is always a possibility that the blood would be
unavailable for use if the operation was inadvertently long delayed or cancelled for any
reason.
KEY ISSUES
Appropriate explanation of blood being a renewable tissue, thus allowing for preoperative
collection and storage including:
~advantages of autologous over homologous blood transfusion: and
~circumstances in which autologous transfusion may be considered.
CRITICAL ERROR - none d e f i n e d
COMMENTARY
There are two components to the task:
To provide education on the transfusion of blood and blood products.
To explain the benefits, risks and principles of autologous blood transfusion.
In industrialised communities, blood transfusion is now extremely safe. Blood is usually collected
from volunteer donors only. In Australia there is no remuneration or any form of inducement other
than a demonstration of community spirit. All blood samples are carefully screened for important
communicable diseases, including human immunodeficiency virus (HIV), hepatitis B and C virus
(HBV, HCV) and syphilis. There will always be a risk of infection and of concern is the rare
hepatitis G virus and the prion transmission agent responsible for Creutzfeldt-J acob disease
(CJ D). Some countries will not accept donors who have come from countries where 'mad cow'
disease has been identified. Screening for HIV does not give absolute freedom from risk, as a
window period occurs after primary infection until antigen is detectable and seroconversion
occurs after several weeks.
Blood is only given to patients when strictly necessary. Donors are screened for high risk
circumstances and the donated blood tested, and the blood is grouped, screened for antibodies
and cross-matched with the potential recipient. There are strict guidelines for the administration
and monitoring of blood and any adverse events are documented and reported. Blood
transfusion is used sparingly for elective surgery and otherwise healthy patients can tolerate
haemoglobin concentrations down to 80 g/L.
113
017
Performance Guidelines



Autologous transfusion is only considered when:
the date of the surgical procedure can be virtually guaranteed:
the patient's haemoglobin concentration exceeds 110 g/L (135 g/L in women); and
it is likely that a substantial amount of blood will be lost during the procedure.
Concerns about the safety of blood transfusion, particularly in the 1980s about risks of transmission of HIV
infection, stimulated interest in avoiding or reducing the use of donor blood, particularly single unit transfusions
(Give blood by the gallon, not by the gill).
Several techniques can be used to avoid the need for bank blood.
Autologous blood transfusion is one such technique, which reduces the risk of transfusion of viral infection
and also of alloimmunisation and incompatibility reactions.
Stored blood at 4"C has a 'shelf-life' of up to five weeks, at which time around 70% of the red cells still survive
normally.
With pre-deposit autologous transfusion, patients can donate 2-5 units (of450mL) at approximately weekly
intervals before elective surgery. Alternatively, deliberate preoperative haemodilution can be induced by
removing one or two units immediately before surgery to be used during surgery to replace operative losses.
Collection and administration of blood for autologous transfusion is an expensive exercise. Because the donor
(the patient) is not a standard blood donor, if the blood is not used for any reason, it cannot be put into the
general donor pool and will be wasted. Starting a maximum of five weeks before the planned procedure, the
patient donates a unit (450 mL) a week and is given ferrous sulphate supplements. Another variant on
autotransfusion, as noted above, is to take off 1 L of blood immediately prior to operation, haemodiiute the
patient with crystalloid and then use the freshly removed blood as required.
Other techniques that can be used include intraoperative autotransfusion (operative
blood salvage) and erythropoietin. Blood salvage techniques to collect and retransfuse blood lost during
surgery can be employed, providing the operative site is known to be free of bacteria, intestinal content, or
tumour. Intraoperative autotransfusion can be used in major trauma and vascular surgery. As the equipment
required to run such systems is expensive, intraoperative autotransfusion does not usually make significant
dollar savings over standard banked blood.
Recombinant human erythropoietin can be used to stimulate the body's own blood reserves prior to elective
surgery. The technique can be considered for chronic renal failure patients who are anaemic and as a
blood-saving strategy in major surgery.
In Australia and UK. where blood transfusion is generally perceived as being safe, and where emergency
surgery and pressures and operating list revisions make it difficult to set definite dates of elective surgery, the
techniques of autologous transfusion are applied only in a minority of cases. This contrasts to other
communities, such as USA. where the use of autologous transfusion is more common (up to 5% in some
regions).
The future: The use of allogenic blood will always involve the need for compatibility testing. The shelf life of
allogenic blood is limited and although current techniques of blood transfusion are very safe, disease
transmission from viruses is unlikely to be completely eliminated because of false negative results during the
window period.
A clinically effective red blood cell substitute would thus be beneficial in terms of universal compatibility,
immediate availability, freedom of disease transmission, and long-term storage.
Major potential candidates are haemoglobin solutions and perfluorochemical emulsions.
017
Performance Guidelines
114


The haemoglobin molecule, the tetramer, chemically binds and carries oxygen (1 g Hb binds 1,39
ml_ 0
2
and is fully saturated at ambient pressure). Oxygen is unloaded from Hb in capillaries at 40
Torr.
The unmodified tetramer is potentially nephrotoxic and vasoconstrictive. Efforts to modify the
tetramer to improve safety include polymerisation and other techniques.
Continuing research into synthetic blood substitutes thus continues in an attempt to produce a
non-toxic temporary fluid combining volume replacement, and intravascular oncotic stability
equivalent to that exerted by plasma proteins with oxygen-carrying capacity. In the meantime, the
major alternative volume replacement fluids available are albumin solutions, solutions utilising
other colloids (gelatin, dextrans), and simple balanced electrolyte solutions.
018
Performance Guidelines
115
Condition 018
Advice on stopping smoking to a 30-year-old man
AIMS OF STATION
To assess the candidate's knowledge of nicotine-dependence and the ability to obtain relevant
information to counsel the patient appropriately and to answer his questions about the
withdrawal process and treatment options.
EXAMINER INSTRUCTIONS
The examiner will have instructed the patient as follows:
You are a 30-year-old information technology specialist, employed in a State Government
Department. You are engaged to be married and have attended your general practitioner as a
followup to a recent episode of an upper respiratory infection associated with a cough.
You have been a cigarette smoker since your mid-teens and currently smoke 20 cigarettes a day
(on average). With your impending marriage, you have been considering stopping smoking. On
the previous visit to your general practitioner, the doctor had briefly suggested that you should
stop smoking for the benefit of your health.
Opening statement:
Start the interview by saying: I've been thinking about your advice last time about my smoking. I
would like to stop. What can you do to help me?'
Appear interested, engaged and genuinely motivated at this time to stop smoking Listen carefully
to the advice and information provided by the doctor, and respond appropriately. Ask questions
and seek clarification, depending on the content provided.
Questions to ask if the topic has not already been covered by the candidate:
Is it too late for me to stop? Is the damage already done?'
'How easy is it to stop?'
'Do / have to stop abruptly or can I just cut down gradually?'
'Do hypnotherapy or acupuncture or alternative remedies like herbs or vitamins work?'
'What are the risks/benefits of nicotine patches/gum or mood tablets? Are they expensive?
What are the side effects?'
'What should I expect by way of withdrawal symptoms?
Will the treatment affect my sex life?'
Do I need to do a course or join a quit smoking group?'
Is the desire to smoke inherited? Will I pass it on to my children?'
018
Performance Guidelines
116
EXPECTATIONS OF CANDIDATE PERFORMANCE
The candidate should display appropriate empathy, rapport and enthusiasm for explaining the
benefits and pitfalls of smoking cessation.
Cigarette dependence is a chronic relapsing condition which, once established, involves a long
term, even lifetime struggle to achieve abstinence. Motivation to stop in this patient should be
assessed by the candidate asking direct questions about his intentions and desire to stop (for
example, on a scale from 'not at all' to Very much'). The patient is in the preparation' stage of
behaviour change (planning to stop in the near future) and is hoping to learn 'action' strategies
that will help him succeed in overcoming his dependence on nicotine.
In order to assess how dependent the patient is on nicotine, the candidate should not only ask
about the number of cigarettes a day the patient smokes, but focus more on the pattern of
smoking during the day (smoking more in the first hours of waking is more significant and
suggests a greater degree of dependence).
Questions which would be appropriate from the candidate to the patient are:
'How many cigarettes a day do you smoke?
1

How soon after you wake up do you have your first cigarette?'
'Do you find it difficult not to smoke in nonsmoking areas?'
Is the first cigarette of the day the hardest to give up?'
'What is your pattern of smoking during the day?'
'Do you smoke even if you are so ill that you cannot get out of bed?'
Have you tried to stop smoking for good in the past but found you could not?'
The patient's answers to these questions will shape the advice that the candidate then gives. A
'yes ' response to the last question suggests that the patient will need help to stop smoking. When
a patient smokes 20 or more cigarettes/day and has to have the first smoke within half an hour of
waking up then the patient is likely to benefit from nicotine replacement therapy (NRT) or
bupropion.
ADVICE ON NICOTINE WITHDRAWAL
Within 24 hours of reducing or stopping nicotine intake you may experience:
depression or otherwise feeling unwell;
insomnia;
restlessness and irritability;
anxiety and difficulty concentrating;
drop in heart rate over time;
increase in appetite so that you may gain up to 3 kg over the next 12 months:
cravings for sweet things; and
cravings for cigarettes.
About 5% of nicotine-dependent individuals can stop smoking unaided and less than 25% of
people succeed at their first attempt to quit. Overall the successful quit rate is about 45%
eventually.
Withdrawal symptoms:
peak in intensity over the first four days of abstinence;
most residual symptoms improve significantly within a month;
hunger and weight gain may persist for a year or so.
018
Performance Guidelines
117
KEY ISSUES
Counselling abilities.
Awareness of principles of a tobacco quitting programme.
CRITICAL ERROR
Lack of awareness of the key elements of a nicotine quitting programme.
COMMENTARY
Principles of a tobacco cessation programme follow. Candidates should have a broad knowledge
of the content, and are expected to be aware of, and incorporate most of. the following key
elements into their counselling.
KEY ELEMENTS OF CIGARETTE/NICOTINE WITHDRAWAL PROGRAMME
The key elements of a cigarette/nicotine withdrawal programme include:
Set a definite QUIT BY date (within two weeks of making the decision to quit).
Aim for total abstinence not just 'cutting down'.
Review previous attempts at quitting and what went wrong!
Inform family and friends, particularly other smokers, of the plan,

Avoid alcohol, which is an important trigger for smoking, and similarly review coffee intake.
Anticipate and discuss likely individual pitfalls and difficulties (for example, weight gain or
depression).
Practise problem solving as a way of dealing with 'what do I do if/when'.
Encourage the use of nicotine replacement therapy unless there are contraindications (e.g.
coronary artery disease or pregnancy).
Recommend starting or increasing physical activity and the importance of a balanced diet.
Schedule followup visits and supportive phone calls.
NICOTINE REPLACEMENT THERAPY (NRT)
The aim of nicotine replacement therapy (NRT) is mainly to ameliorate nicotine withdrawal.
Neither patches nor gum give the arterial 'high' concentration of cigarettes and the overall dose of
nicotine they provide is about 40% of that provided by cigarettes, but they are not accompanied
by tar. carcinogens or carbon monoxide. Smokers extract about 1 mg of nicotine per cigarette
independent of the brand used, although each cigarette may contain up to 14 mg of nicotine.
Costs of NRT may influence patient choice, but are generally cheaper than continuing to smoke.
Forms of NRT include:
NICOTINE GUM contains nicotine 2 or 4 mg per piece in a sugar-free resin base. The gum
should be chewed slowly, then left between cheek and gum before being repositioned and
chewed intermittently for up to 30 minutes, 10 times a day. Because nicotine is poorly
absorbed from an acid environment, acid drinks such as fruit juice should be avoided. Mouth
soreness or dyspepsia may occur. Lozenges are an alternative.
018
Performance Guidelines
118
TRANSDERMAL PATCHES, preferred by many patients, come in a variety of dosage strengths from 7
mg to 21 mg and in preparations designed to be used for 16 or 24 hours. They are designed to release
nicotine slowly through the adhesive layer of the patch to the skin and hence into the circulation. Patches
are applied each morning on a rotational basis to non-hairy skin sites. Skin reactions or rashes may be
severe enough to warrant discontinuation. They are not available on the Pharmaceutical Benefits
Scheme, and are used for 8-12 weeks.
NICOTINE INHALERS are less popular. Gaseous nicotine is released after deep inhalation through the
mouthpiece of a plastic cartridge. About 4 mg of nicotine is released by the device which is single use
only and cannot be reused or recycled, which may mean 10 or more cartridges need to be used per day
for up to 6 months.
All forms of NRT are effective as aids to stopping smoking. There is no reliable evidence to recommend gum
over patches or inhaled, each roughly doubling the chances of successfully quitting. They may be used
together but monotherapy is preferred. The initial strength of nicotine dosage will depend on severity of
dependence as well as average daily intake of nicotine from cigarettes.
Whilst NRT is effective by itself in achieving abstinence, behavioural support from the doctor, family and
friends will increase the success rate.
OTHER SMOKING CESSATION STRATEGIES AND AIDS INCLUDE:
Bupropion (slow-release Zyban) is an atypical antidepressant with both noradrenergic and
dopaminergic activity.
~Mechanism of action as an aid to smoking cessation is not related to its antidepressant action, but to
common addiction pathways.
~Treatment starts at 150 mg/day for the first three days and then increases to 150 mg twice daily.
~The standard treatment period is nine weeks which is subsidised under the Pharmaceutical Benefit
Scheme in Australia. Maintenance may last a further six months.
~Nausea, insomnia and dry mouth are the commonest early side effects.
- Bupropion is absolutely contraindicated in patients with a history of epilepsy: and is relatively
contraindicated when there is a history of Type 1 or 2 diabetes.
~It must not be prescribed during pregnancy.
~Sustained-release bupropion has been shown to be efficacious in producing abstinence in cigarette
smokers either with or without adjunctive psychological interventions.
Clonidine is uncommonly used to moderate withdrawal symptoms.
Nortriptyline 10 mg/day has also been trialled as an alternative to bupropion.
Pamphlets, stickers, badges and audiovisual and multimedia tapes in different languages are available
through the various state health departments, which can supplement face-to-face counselling.
018
PerformanceGuidelines
General Health 0109 - 2004

SMOKING
Quitting smoking is one of the most important actions you can take to protect yourself
from chronic disease and early death. One in two lifetime smokers will die from their
addiction. Half of these deaths will occur in middle age. Quitting at any age will give
major health benefits and reduce your risk of tobacco related illnesses. With planning
and determination you can quit and stay a non-smoker.
Health effects of smoking
Tobacco smoke contains more than 4,000
chemicals, Harmful ones include:
Nicotine - is an addictive drug and can make it
hard, but not impossible, to quit. It also affects
vour heart rate and blood pressure.
Carbon monoxide - replaces some of the oxygen
in your blood, leaving your muscles, heart and
brain with less oxygen.
Tar - contains many cancer causing chemicals.
Lower tar or 'light' cigarettes are not any better
as you are likely to take deeper putts, more
often, to get the amount of nicotine you need.
Smoking increases the risk of cancer, heart disease,
stroke and lung disease. It affects fertility levels in
men and women and can lead to impotency in
men, and miscarriage and complications in
pregnancy and labour. Smoking affects your
immune system and is cause of many other
conditions such as blindness and osteoporosis.
Harm to others
Environmental tobacco smoke comes from both the
burning end of a cigarette and from the smoke
breathed out by a smoker. This harms not just the
jjf smoker, but also family members and
coworkers. Passive smoking causes heart disease
and lung cancer in non-smokers living with
smokers.
Children exposed to passive smoking are more
likely to suffer from health problems including
asthma, meningococcal disease, coughs and chest
infections. Smoking by the mother is a major risk
factor for sudden infant death syndrome (SIDS or
'cot death).
Do you know why you want to
stop smoking?
It's important to be clear about your reasons. These
are some of the best reasons to quit. You will:
Breathe, and taste and smell food better within
weeks
Improve vour circulation, immune system and
the health of your heart and lungs
Have fewer days of illness and fewer health
complaints than continuing smokers
Provide a positive example for children and
others
Save money, a minimum of $2300 per year for a
pack a day habit.
Before you quit
Chemicals in cigarettes change the way
some medications work. We know that
stopping smoking can be stressful. So if
vou have suffered from depression, anxiety
or other mental illness, and/or are taking
medication, speak to your doctor before
quitting.
Pharmacy Serf Care is a program of the Pharmaceutical
Society of Australia

CONDITION 018. FIGURE 1. Smoking
cessation pamphlet
1 1 9
018
Performance
Guidelines
CONDITION018.FIGURE2.
Examplesofsomenicotinereplacementtherapies(NRT)available
120
019
121
PerformanceGuidelines
Condition 019
Excessivealcoholconsumptionina45-year-oldman
AIMSOFSTATION
To assess the candidate's knowledge of hazardous drinking, including early presentation,
sequelae, types of dependency, and skill in counselling a person who has been drinking
hazardouslyoveralongperiodoftime.

Theexaminerwillhaveinstructedthepatientasfollows:
You are a 45-year-old businessman who has become concerned about the harmful effects of
alcohol.Youhavebeenaregular,nowdaily,drinkerformanyyears,butyouconsideredyourself
tobean'averagedrinker'foryourpersonalsituationinlife.
You consulted this doctor two days ago. The doctor enquired about your use of alcohol and
otherconcernsatlength,andthenexaminedyou.Thedoctorsaidyouruseofalcoholwasof
concern and asked you to have some blood tests. You have returned today to discuss the
results.
You should be rather passive, exhibiting acceptance of the doctor's advice, while showing a
contemplative demeanour that suggests that you may not have made up your mind about
modifyingorstoppingyouralcoholintake.
EXPECTATIONSOFCANDIDATEPERFORMANCE
The candidate's approach to the patient should be nonjudgmental and supportive. The
candidateshould:
Explaintestresultsandtheirinterpretation
~ The elevated Gamma glutamyl transaminase (GGT) and Aspartate transaminase (AST),
elevatedmeancorpuscularvolume(MCV)andmacrocytosisshouldconfirmthecandidate's
clinicalsuspicionofliverdiseaseduetoexcessiveharmfuldrinking.
- The candidate should make this clinical suspicion clear to the patient and explain that
excessivedrinkingisalsolinkedwithhishypertensionandexcessiveweight.
Discusstheeffectsofexcessivedrinkingincounsellingandeducatingthepatient
~Otherphysicalsequelaeongastrointestinaltract,cardiovascularsystemandcentralnervous
system.
~ Family problems, work problems, sexual problem, minor accidents, as revealed from the
history obtained at the previous consultation. Be complimentary about patient's initiative to
consultabouthisdrinking(thisispositiveandhelpsprobabilityofcompliance).
122

019
PerformanceGuidelines



The 'CAGE' or 'AUDIT' questionnaires could be applied during this discussion, or during
counselling:
~Cuttingdownonyourdrinking?:Annoyedbycriticismofdrinking?;Guiltyaboutyour
drinking?;Eyeopenerneededinthemorning?
~AlcoholUseDisorders/dentificationTest.
1
CounselthepatientaboutsafedrinkingusingNationalHealthandMedicalResearchCouncil
(NHMRC)Guidelinesformen:
~lowrisk:lessthan4standarddrinksperday;
~hazardous:5-6standarddrinksperday;and
~harmful(highrisk):morethan6standarddrinksperday.
Counsel the patient that the test results show that his drinking level is above safe drinking
levels:
~ A 'standard drink' contains approximately 10 grams of alcohol. When conventional
glasses/containersareused,thevolumeofeachdrinkreducesasthestrengthincreases.
Adjustmentsmustbemadeforthedifferentstrengthsofbeerbecausecans/bottlesarethe
samesize.
~ The test results show that he is demonstrating clearly harmful and progressive effects of
excessivedrinking.
Explaintypesofdependency:
~Soci al (appliestothispatient);
~Psychol ogi cal (mayapplytothispatient);
~Physi cal (probablyappliestothispatient).
Discussthesignificanceofamountanddurationofalcoholuseindeterminingriskoiphysical
dependency.Hallmarksofphysicaldependencyinclude:
~increasingalcoholtolerance(whichdecreasesinlaterstages);
~withdrawalsymptoms(tremor,sweating,hyperarousalthroughtodel i ri umtremens);and
~relief from withdrawal symptoms by further alcohol consumption, or agonist
(benzodiazepine).
Discussthesocial,behavioural,emotionalandcognitivesequelaeofexcessive
alcoholuseinfamily,work,socialandindividualsettings.
Explainthatexcessivehabitualconsumption/dependencecanbeassociatedwithtargetorgan
damagewithoutpsychologicalorsocialdisordersandviceversa.
As initial management, advise a period of abstinence to test presence and/or degree of
physicaldependency.Indicateavailabilitytoassessandtreatanysymptomswhichmayarise
suchastremor,sweating,excitabilityandcraving.
Asfollowupmanagement,thecandidateshould:
~suggestfurtherconsultationshortly;
~offertoseewife;

1Curtin:ResearchCentres:UniversityResearchInstitutes:NationalDrugResearchInstitute.ScreeningforIhazardousalcohol
useanddependenceinpsychiatricin-patientsusingtheAUDITquestionnaire,Hulse.IG.K.,Saunders.J.B.,Roydhouse,R.M.,
Stockwell,T.R.andBasso,M.R.,2000.http://espace.list.curtin.edu.au/archive/00000076/
019
Performance
Guidelines
Self Help 0506-2004


ALCOHOL
Alcohol can interact with some medications and certain medical conditions can be made worse bv
drinkingalcohol.InAustralia,alcoholproblemsaffectthehealthandwellbeingofmanyindividuals,
families and communities. Many people drink in ways that put themselves at risk of alcoholrelated
harm.TheAustralianAlcoholGuidelinesrecommendlevelsforlowriskdrinking.
How much is too much?
Drinkingatriskyorhighrisklevelsmeans
drinkingmanydrinksinoneday.Youdonot
needtodothisregularlytocauseharm.
Hxcessivealcoholconsumptioncancausehealth
problemsinthelongerterm.Youdonotneedto
getdrunktorthistocauseharm.
Who are the Guidelines for?
TheGuidelinesindicatethelowriskdrinkinglevels
forthegeneralpopulation.Theyareforpeoplewho
arenotonmedications,donothavemedical
conditionsthataremadeworsebydrinking,andare
notelderly,pregnantorbreastfeeding.Ifyouweigh
lessthan60 kgformenor50 kgforwomen,orare
abouttodriveoroperatemachinery,thelevels
recommendedintheguidelineforthegeneral
populationshouldbereduced.
What do the Australian Alcohol
Guidelines recommend?
Peoplewhodrinkregularly(almostdaily),more
thaneightstandarddrinksaday,shouldseetheir
doctorbeforeattemptingtochangetheirdrinking
habits.
Averagenumberof
drinksperday
Men
4orless
Women2or
less
Maximumnumberof
drinksperweek
28 14
Maximumnumberof 6 4
drinksinanyoneday
Numberofalcoholfree
daysperweek
12 12
What is a standard drink?
Thepicturesbelowshowthenumberofstandard
drinksfoundintypicalserving
containers.Becarefulglasses
ofwineandspiritscanvary
widelyinsizeandalcohol
content.
375ml

fullstrengthbeer


180ml


averagerestaurantserveofwine

1.8
.WmlspiritnipI
300mlalcoholicsoda1.2

Check the label
Allalcoholicbeverageshavethenumber
standarddrinksinthecontaineronthe
Usethistocalculatethenumberof
standarddrinks.
of
label.
Pharmacy Serf Care is a program of the Pharmaceutical Society of
Australia

CONDITION 019. FIGURE 1.
Alcohol ingestion guidelines
1
019
124
PerformanceGuidelines



~ mention availability of Alcoholics Anonymous. Weight reduction and control of
hypertensionwillneedtobeundertakenbutcannotbesuccessfulunlessuseof alcohol
ismodified;and
~ make clear to the patient that he has to come to his own decision about what to do, but
thathecanexpectongoingsupportinfollowup.
The above points serve as guidelines only. The content, order and emphasis offered by the
candidate will vary and examiners will need to give a global assessment of the candidates
interpretationofthetestresultsandcounsellingskillswithregardtohazardousdrinking.
Interpretationofresultsofliverfunctiontests(LFTs)andfullbloodexamination(FBE).
Explanationofeffectsofhazardousandharmfuldrinking.
Advisingaperiodofabstinencetoestablishtypeanddegreeofdependency.

Takingajudgmentalattitudeandblamingpatientforhiscondition

As well as knowledge of hazardous drinking, this case allows assessment of the candidate's
communication skills and knowledge of counselling as the principal form of management in a
behaviouralproblem,in whichthetypeofdependencyisuncertain atthisstage.Thepatient is
likelytohavetruephysicaldependency;thenextstepinmanagementistotestthisassumption
by asking the patient to try abstinence and see what happens. Subsequent management will
dependontheresultandthewillingnessofthepatienttoacceptandcontrolthetypeanddegree
ofdependencyonalcohol.
020
125
Performance
Guidelines



Condition 020
Type1diabetesmellitusina9-year-oldboy
AIMSOFSTATION
Toassessthecandidate'sknowledgeofpracticalaspectsofchildhooddiabeticcarewhichis
notverydissimilartoadultType1diabeticcareandthecandidate'sabilitytoanswerqueriesof
a concerned parent. The questions chosen are similar to those asked by parents who are
anticipatingtheproblemsthattheymayencounterinthemonthsafterinitialdiabeticeducation
anddischargefromhospital
EXAMINERINSTRUCTIONS
Theexaminerwillhaveinstructedthemotherasfollows:
As many candidates have a tendency to refer any question they are uncertain of to a higher
authority(forexample,theirregistrarorconsultant),youshouldnotacceptthatanswer,butask
whatsortofthingstheconsultantislikelytosayaboutthatquestion.
Each question asked by the parent requires an answer from the candidate so that examiners
shouldkeepaclosewatchonthetime,andindicatetotheparenttomovetothenextquestionif
thecandidateiseitherspendingtoomuchtimeondetailedinformation,ordoesnotprovidethe
appropriateexplanation.Thiswillensurethatthecandidatehastheopportunitytocoverallofthe
questionsintheallottedtime.
Satisfactorycandidateswillanswereachofthefollowingquestionsaskedandcovermostofthe
followingpoints.
Questionstobeaskedbythemother(inthisorder)
OpeningStatement
'Will he need insulin injections each day from now on?'
If thecandidate indicates(correctly)thatthechild willrequiredaily insulin,askhow oftenand
thenask
'Who is going to be giving Roger's insulin from now on?'
How do I assess the day to day control of his diabetes?'
'What do I need to do about his school?'
'Will he be able to go to school camps? What should I do about them?'
He went to his first sleepover party a few weeks ago could he still go on these now?'
'Can he play sport?'
The above questions are designed not only to test the candidate's knowledge but ability to
reassure the parent that with appropriate tuition, she will be confident in handling her child's
diabetesandinteachingotherpeoplewithwhomhewillbeincontacttounderstandandmanage
hisdiabetes.
020
PerformanceGuidelines



EXPECTATIONSOFCANDIDATEPERFORMANCE
Openingstatement/Needforinsulininjections?
Thecandidateshouldindicatethatthechildwillbeonlife-longinjectionsofinsulinandshould
goontodescribethatthisisusuallyinaregimenoftwicedailyinsulininjectionsconsistingofa
combination of a short-acting insulin and an intermediate-acting insulin to allow for a 24-hour
cover. At this age it is usually inappropriate for children to be on basal/bolus insulin regimen.
Thisisbestleftuntilteenageandwhenthechildisreadytoacceptfourinjectionsperday.
Q2Whoinjects?
Thecandidateshouldadvisethatbothparentsifavailable,andperhapsanoldersibling,should
learn the injection technique in case of parental sickness or absence. Similarly they should
understandhowandwhentogiveglucagon.CandidatesshouldindicatethatthePaediatricUnit
staff and educators will ensure that parents are confident of drawing up and administering
insulin before discharge, and if necessary will arrange help at home from the District Nursing
Serviceafterdischarge.
Q3Monitoring?
The candidate should indicate that blood sugar levels are monitored several times daily by a
glucometer (illustrated), the details of which will be shown to the parent. These levels are
usually assessed before each meal and before bed at night. This allows the clinician and the
parent, when confident, to be able to review the blood sugar levels and to recognise if an
adjustment in the insulin dosage is required. This is usually adjusted based on a trend over
severaldaysratherthanonaday-to-daybasis.
Thedoctorshouldalsoindicatethaturinetesting forketonesshouldalso bedoneiftheblood
sugarlevelsarepersistentlyhigh,orifthechildisunwellforanyreason.
CONDITION020.FIGURE1.
Glucometerkitforbloodglucosemonitoring
126
127
020
Performance
Guidelines



04Schoolliaison?
ThecandidateshouldadvisethattheparentshouldnotifyRoger'sschoolofhisdiagnosisand
indicatethattheschoolwillprobablybevisitedsoonafterthechild'sdischargefromhospitalby
theWarddiabeticeducatorinassociationwiththeparenttoinstructtherelevantschoolstaffasto
the important features related to possible complications at school (the most likely one being
hypoglycaemiaandhowtodetectandmanageit).
Theparentwillalsobe advisedtotakeeveryopportunity to reviewthese importantaspectsof
care with the school staff and to ensure that any new staff are made aware of the child's
condition.Similarlytheschoolshouldbegivenphonenumbersto ensurethatthe parents,the
child'sgeneralpractitionerandifnecessarytheHospital/Children'sWardcanbecontactedinan
emergency. In an emergency most schools will contact the Ambulance service who will be
capableofmanagingthesituationonarrival.
Q5SchoolCamps?
Management at school camps may vary often depending on the age of the child. With young
children,parentsareoftenencouragedtoattendascampparentsandthiswouldbeappropriate
forthisfamilyifoneorotherparentisabletodoso.
Management is related to the confidence and experience of the teachers involved and how
comfortable the parents are with the teacher's knowledge. The Paediatric Unit, in conjunction
withtheparentwillsupporttheteachersbyeducationandtelephonesupport.Manycampsfor
youngchildrenareheldrelativelyclosetotheschooldistrict.
If the camp is in a distant town, supervising teachers with good clinical acumen are a
prerequisite,butalettershouldbesuppliedtoalocalmedicalpractitionerexplainingthechild's
diagnosis and providing details of the insulin regimen, blood sugar levels and telephone
contacts.
Some school camps have permanent staff who have children attend with a variety of medical
conditions (for example, epilepsy and diabetes), and these staff are given special detailed
instructionsastothecomplicationsandmanagementoftheseconditions.Theparentshouldbe
encouragedtoenquireastotheknowledgeofthestaffinvolved.
Theparentshouldbeadvisedthattheaimofthetreatmentandeducationprogramistoallowthe
childtoliveasnormalalifeaspossibleandRogershouldbeencouragedtoparticipateinallthe
schoolactivities.Thisusuallyispossible;buttheparentshouldbesatisfiedthatthepersonnel
involvedarecognisantwiththetreatmentforthechild.
Q 6 Sleepovers?
Theseshouldalsobeencouragedforreasonspreviouslydiscussed.Sleepoversshouldusually
be at the home of a family who know the child and parents well; so that there can be a frank
informativediscussiononthechild'smanagement,whichishighlylikelytobeknowntothehost
family.Similarly,thisshouldusuallybeatahomeclosetothechild'shomesothatifnecessary
thechild'sparentmaybeabletocalloveranddothebloodtestaswellasgivetheinsulininthe
evening and next morning, until the child is old enough and reliable enough to do these by
himself.
Q 7 Sport?
Theparentshouldbeassuredthatthereisnoreasonwhythechildcannotplaymostsports.He
willbeexcludedfromsomeactivities(forexample,pilotinganaeroplane).Agoodcandidatewill
mentionthatinsulindosesmayneedtobeadjustedpriortoactivesporttoallowfortheincreased
glucosemetabolismassociatedwithphysicalactivity.
020
128
Performance Guidelines



The candidate should be able to counsel the parent in a reassuring manner that parents will not be left
entirely on their own in the management of their child, and that help is but a phone call away. Most candidates
should be able to handle this discussion with ease and confidence by applying general principles in their
discussion and this should comprise part of the overall assessment of the candidate.
Most of the detail for this case is provided in the examiner's instructions. Much of the advice to be given is
common sense in relation to the ongoing care of a person with diabetes and should not be a problem to the
competent candidate. The scenario is designed to assess both the knowledge of the topic and the candidate's
ability to provide accurate information in a reassuring manner to a parent who is trying to cope with a
diagnosis, which to most parents is initially devastating and upsetting.

Ability to answer the specific questions of the parent accurately and sensibly
CRITICAL ERROR
Failure to discuss symptoms and treatment of insulin-induced hypoglycaemia

The emphasis in many of the answers should be allowing the child to lead as normal a life as possible, so that
school and social activities should be maintained wherever possible. Under most circumstances this can be
achieved.
The key to success in these situations is whether parents are well educated in their child's condition and can
confidently instruct others accurately in the management of the child. Families who achieve this are generally
very successful in managing their child's diabetes confidently and appropriately; and the child is generally
able to progress satisfactorily through childhood with minimal restriction, if any, in lifestyle.

129
021
Performance Guidelines



Condition 021
Request for vasectomy from a 36-year-old man
AIMS OF STATION
To assess the candidate's ability to explain the surgical procedure of vasectomy, its
complications, effectiveness, reversibility and effects on sexual performance.
EXAMINER INSTRUCTIONS
The examiner will have instructed the patient as follows:
Your wife aged 34 years, who works as a secretary, wishes to cease oral contraception
because of weight gain. She has taken 'the pill' since the birth of your son now aged seven
years. You also have a daughter aged nine years. Your wife would prefer not to have a tubal
ligation.
You are happily married with a mutually satisfactory sexual relationship and neither of you
wishes to have another child. You have no extramarital relationships. You want a-vasectomy
but have reservations about the operation because of possible complications, reliability,
absence from work, and any adverse effect on your sexual performance.
Questions to ask unless already covered
(the candidate's expected responses are in brackets):
Is it 100% effective?' (No, recanalisation of vas can occur rarely).
How do you know it has worked?' (Semen analysis showing no spermatozoa after 20 or so
ejaculations, test repeated on at least two occasions).
Can the operation be reversed?' (Reversal requires microsurgery, success uncertain.
Because sperm antibodies can occur, and these reduce fertility, even if the tubes can be
connected satisfactorily, success is not 100%).
'Are there any complications?' (Wound infection, temporary discomfort, bruising, and
haematoma).
Will there by any change in my sexual performance?' (No. not after recovery from
operation).
'How long afterwards can we resume normal sex?' (As soon as you are comfortable. A
condom must be used until the result of the postoperative sperm count is known; or wife
should continue to use the pill until the sperm count is clear).
Does my wife have to sign anything^' (Preferable if both provide written permission).
What is actually done?' (Description of identification and division of vas).
Will my wife notice any difference?' { No) .
What happens to the semen?' (Semen ejaculated but no sperm).
'Can it be done as a day procedure?' (Yes).
'Will I have a general anaesthetic?' (General or local used with sedation).
021
130
Performance Guidelines



EXPECTATIONS OF CANDIDATE PERFORMANCE
Approach to patient
Give reassurance about result of checkup (good health).
Be supportive and give honest advice about the procedure.
Provide information in a clear and concise manner.
Deal with patient uncertainties.
Explanation of the procedure
Surgical procedure of vasectomy how it is done.
Inconvenience and complications.
Postoperative sperm counts.
Effectiveness and reversibility although reversal is sometimes possible, vasectomy should effectively be
regarded as a method of permanent sterilisation.
Timing and effect on sexual activity/performance.
Use of diagram, patient education material (e.g. Royal Australasian College of Surgeons patient information
brochure).
Counselling
Recommend involvement of patient's wife offer to discuss procedure with her.
Advise that she should preferably also give written permission.
Discussion of alternatives, (i.e. condom or diaphragm with spermicidal).
Clarification that they need to use contraception until seminal analysis is negative.

Adequate explanation to the patient of the procedure and its followup.

False reassurance that the procedure is 100%effective before at least two negative sperm counts.
False reassurance that a vasectomy is easily reversed.
Refusal to discuss sterilisation.

Appropriate consent for vasectomy is required. Permission of the patient's wife is not obligatory but is strongly advised
by medical defence organisations.
The candidate may have religious or cultural objections to contraception in general or vasectomy in particular. If so.
candidates have been advised to inform the patient of their position at the beginning of the consultation. However,
medical knowledge of the procedure is still expected together with the communication skills required in explanation.
This recognises the patient's right to seek the procedure and the doctor's right not to remain nvolved beyond this
consultation, but to suggest appropriate referral after initial discussion.
131
1-B: Case Presentations to Examiner

Vernon C Marshall
'Begin with an arresting sentence; close with a strong summary; in between speak simply,
clearly and always to the point; and above all be brief.'
William J Mayo (1861-1939)
American physician
Case Presentation and Discharge
Summary scenarios 022-029
02
2
Headache
02
3
Neck lump
02
4
Previous shoulder dislocation
02
5
Dysphagia
02
6
Low back pain
02
7
Knee pain
02
8
Abdominal discomfort
02
9
Gastric ulcer with haemorrhage
Verbal Case presentation
to an observing examiner or
senior colleague is a
fundamental part of clinical
medicine. It may consist of an
informal exchange of
information between colleagues
on a ward round or it may be a
more formal presentation to an
examiner or to a group in
examination circumstances.
Whatever the circumstances, it
is essential that the listener's
interest and attention are kept
and that information is clearly
accurately and sensitively
transmitted.

132
1-B
Case Presentations to Examiner
CONDITION 022


CASE PRESENTATIONS
A number of AMC MCAT communications with examiners will involve answering questions or
prompts asked by the examiner to clarify points of diagnosis or management. These usually
require brief and succinct candidate answers.
Example 022
Examiner: What do you believe is the most likely diagnosis from the information you have
found so far?'
Candidate: 'Tension headaches. '
Examiner: 'Are any alternative diagnoses likely?'
Candidate: I think the clinical presentation fits tension headaches rather than migraine. I don't
think a serious cause like temporal arteritis or raised intracranial pressure is likely.
Examiner: 'Please now advise your patient of the likely diagnosis and your management plans.
In most of these multiple short assessments your oral presentation to the observing examiner
will be necessarily brief, with most of your communication interchange being with the
standardised patient.
However, practice in delivering summarised case presentations to an observing examiner (as
could occur in an extended or composite station) is well worthwhile as an aid to improving your
communication skills, and to organising and presenting clinical reasoning skills in data
acquisition, assimilation and interpretation, to best effect and in a logical manner.
You may find it easiest to do this along disease-centred diagnosis approaches by
succinctly summarising history, examination findings, diagnostic and investigational plans and
management separately or sequentially.
Alternatively, a biopsychosocial approach, centred on the individual patient's presenting
condition and problem and the demographic and psychosocial environment, may be preferred.
Verbal Case presentation to an observing examiner or senior colleague is a fundamental part
of clinical medicine. It may consist of an informal exchange of information between colleagues
on a ward round, or it may be a more formal presentation to an examiner or to a group in
examination circumstances. If the patient is also present and listening, the presentation needs
to be given in such a way as not to upset the patient.
Whatever the circumstances, it is essential that the listener's interest and attention are kept and
that information is clearly, accurately and sensitively transmitted.
Relevant data should be gi ven in a systematic and concise form highlighting the
presenting problem, and evol ving into a coherent whole.
To summarise a patient's problem fully, begin with patient identification (age is essential) and
reason for presentation.
Proceed to a narrative, which may sequentially summarise examination findings, investigation
findings, systems review, risk factors, and diagnostic and treatment plans; or may be confined
to one of these domains, depending on circumstances.
If asked to describe what task you are doing as you proceed (such as performing a physical
examination), make sure you both describe and perform.
133
1-B
Case Presentations to Examiner
CONDITIONS 023-027


Example 023
Candidate: 'The patient has a lump in the right anterior triangle of the midneck which feels like
a lymph node enlargement. I shall proceed to examine the skin of face and scalp,
the oropharynx and nasopharynx, and the external ear for any primary pathology'.
Then go ahead and do what you said you will do. One of the most common errors is failing
to observe appropriately. 7 am now looking for muscle wasting. Next I shall test reflexes and
power... 'and missing the obvious muscle atrophy which is present, by not actually looking.
Example 024
Candidate: (To the examiner) 'The patient has had a past history of dislocated shoulder of his
dominant right arm and my task is to assess the current status.
I shall begin by inspection of contour looking in particular for wasting or deformity
None is apparent (make sure you have looked). / shall now test the range of active
movements, comparing these to the opposite normal side. '
(To the patient) 'Please face me and move your arms as I do'.
Example 025
Examiner:
Candidate:




Examiner:
Candidate:



Examiner
:
'What is your provisional diagnosis?'
'The patient is aged 60 years and has a history of progressive painless dysphagia
for the past 6 months associated with weight loss. I think the most likely diagnosis
is an oesophageal lesion, probably a neoplasm. You have informed me that
physical findings are noncontributory in the chest, neck and abdomen. He needs
further investigation by diagnostic imaging or endoscopy. I shall explain this to him
and arrange appropriate referral and followup... '.
'Which of the two investigations would you choose?'
'Endoscopy usually is most definitive. Preliminary contrast imaging may help by
identifying the site of a stricture and can show extralumenal aspects like extrinsic
compression, so I shall order a contrast swallow now as well as arranging
consultation for an endoscopy'.
'Please now advise the patient of your recommendations'.
Example 026
Examiner: Please summarise your history and findings and your provisional diagnosis '
Candidate: 'The patient is aged 25 years and has had low back pain with sciatic radiation after
a lifting strain at work two weeks ago. He has painful limitation of back movements
and symptoms and signs suggesting lower lumbar nerve root impingement. I
believe the most likely diagnosis is a lumbar disc prolapse with L5 radiculopathy'.
Example 027
Examiner: Please summarise your findings on physical examination so far'.
Candidate: 7 was asked to examine the right knee area in this young man with a past history of
twisting strain to his knee and persisting pain on its inner side. He walks with a
painful limp favouring the right leg. He finds it difficult to bear weight on that side,
and painful to kneel or squat'.
134
1-B
Case Presentations to Examiner
CONDITION 028


He cannot fully extend the right knee, the deficiency is 15. and flexion range is
normal. I cannot detect any joint effusion, and there is no local swelling. Joint
stability is stable testing the collateral ligaments and cruciate ligaments.
Patellofemoral mobility and tracking appears normal; patellofemoral friction is not
painful. He has localised tenderness over the joint line anteriorly on its inner side.
There is no muscle wasting. The other knee appears normal'.
Examiner: What is your provisional diagnosis?'
Candidate: 'Injury to the medial intraarticular cartilage'.
Example 028
Examiner: 'Please summarise the history you have obtained so far and your provisional
diagnosis from the history'.
Candidate: 'Mrs S is a 65-year-old widowed pensioner with a number of problems.
'She has a recent history of increasing abdominal discomfort and bloating
related to meals over the past four weeks, with episodes of greater discomfort
which have woken her from sleep intermittently. The pain is epigastric and diffuse
without radiation. She has occasional reflux of bitter fluid, she has not vomited nor
passed blood. She gets some relief from a glass of milk. She neither smokes nor
drinks alcohol. 'She has a number of relevant associated problems.
'She has had rheumatoid arthritis, affecting predominantly the hands, for 15 years
and has been treated with anti-inflammatory agents and steroids and has required
periodic increased steroid dosage for exacerbations. She is currently on 5 mg
prednisolone and Celebrex.
Type 2 Diabetes: Non-insulin dependent diabetes mellitus was diagnosed 5
years ago. She is on diet and oral hypoglycaemics and blood sugar control has
been good BS 5-6 mmol/L. She has regular eye and foot and blood checks, and
has been told these are all satisfactory.
'She had a myocardial infarction 10 years ago with no sequelae. She remains on
low dose aspirin.
'She has hypertension which has been controlled by an ACE-inhibitor. 'She has
always been overweight despite dieting.
'There is a family history of diabetes and of coronary artery disease. There is no
other relevant medical past history.
'She was widowed 5 years ago She lives by herself and her three grown children
live interstate. Her rheumatoid arthritis has progressively worsened and has caused
increasing difficulties in activities of daily living
Examiner: How would you plan to proceed?'
Candidate: Her multiple medical comorbidities and problems have been mentioned and are likely
to be contributory to the presenting problem.
'She has a number of risk factors for peptic ulcer disease. I believe an upper
gastrointestinal endoscopy will be needed to check for peptic ulcer, gastritis or
reflux oesophagitis as a cause other abdominal discomfort. Gall stones should also
be excluded by ultrasound. '
The above is a good demonstration of a problem-based summary.
1-B
Case Presentations to Examiner
CONDITION 029


Scenarios requiring presentation of diagnostic case summaries to the examiner are exemplified in Conditions
65 (chest pain) and 66 (palpitations and dizziness) as well as in scenarios in other sections.
In a patient whom you have been treating as an inpatient, the communication required may be a written
discharge summary
Example 029
Copy of Discharge Summary to local medical officer. Discharge
Summary
Name: Mr B, aged 66years, DOB. 16.8.1937. Hospital Record No . . .
Inpatient dates: 13.2.2004-20.2.2004
Problem number Title of problem
135
Gastric ulcer with haemorrhage ~ Haematemesis and
melaena ~ Endoscopic fulguration
Rheumatoid arthritis
Myocardial infarction

Onset
#
1
#
13.2.2004

1988
1992

Status
Active

Active
Inactive
#1 Gastric ulcer with haemorrhage haematemesis and melaena
Mr B, a 66-year-old pensioner, has been taking anti-inflammatory drugs for rheumatoid arthritis diagnosed in
1988. His medications have been Celebrexwith intermittent steroid courses for exacerbations. At time of
admission had been on prednisolone for past two weeks. He had a past history of myocardial infarction in
1992 with no sequelae.
He presented with a history of having vomited blood on two occasions, and passed a tarry melaena stool
earlier that day. He was haemodynamically stable on admission. Urgent endoscopy was arranged which
showed a shallow gastric ulcer in the prepyloric region with a bleeding point which was fulgurated. Biopsy of
the edge was performed which confirmed a benign gastric ulcer. He was begun on a proton pump inhibitor
(omeprazole) and his steroids and NSAID were discontinued.
He was discharged after being observed in hospital for a week without recurrence of bleeding.
Prognosis is guarded in view of associated risk factors of arthritis likely to require resumption of
anti-inflammatory and steroid treatment. He will require long term acid inhibition medication.
#2 Rheumatoid arthritis
Rheumatoid arthritis since 1988 affecting mainly the hands and wrist. Significantly disabled by pain, stiffness
and deformity causing loss of function. Referred to rheumatology clinic for advice on continuing management.
Medications on discharge:
Omeprazole 40 mg daily
Paracetamol tablets two q.i.d.
Followup
Gastroenterology outpatients one week
Rheumatology clinic one week
Local medical officer, Dr S.B


136
2 Clinical Diagnosis (D)

2-A: The Diagnostic Process History-taking and Problem-
solving

Reuben D Glass
'Listen to the patient, he is telling you the diagnosis.'
Sir William Osier (1849-1919)
History-taking usually provides the most important diagnostic evidence. Obtaining this
evidence requires the communication skills previously discussed in Section 1 Clinical
Communication'.
Good clinicians organise knowledge so they can quickly retrieve and use relevant data, by
grouping knowledge into 'chunks',
1
often groups of three items of information. In addition to
recognising diseases by patterns, they recognise groups of symptoms and signs, which have
value in discriminating between one disease and another, or between one group of diseases
and another. The alert clinician also notes unexpected variations from a common pattern. For
example, a child with fever is usually flushed. The child with an infection who is pale is
unusual; the pallor may result from the circulatory insufficiency of septicaemia. Experienced
clinicians gather important evidence early in a consultation. A number of cues are noted within
seconds of meeting a patient. Where are we meeting? Is the
patient male or female, young or old, relaxed, anxious, or
unconscious? What else do I see? What is the opening
sentence or two of the presenting complaint? The combination
of verbal and nonverbal information, which impresses the
clinician when first meeting the patient, has been called the
'dominant cue'. It is the starting point for action or information-
gathering, by triggering ideas from the clinician's memory. The
most frequent error in the diagnostic process arises from faulty
triggering. It is important to check what the patient means by
what was said.



When solving clinical
problems, clinicians tend to
review hypotheses either by
some form of rule, by
informally weighing
probability, by considering the
soundness of the cause-and-
effect relationships of the
evidence, or by a comb I
nation of methods.
'Framing the problem' is an art requiring practice to avoid going on a false trail. For example:
a 45-year-old man, who had an appendicectomy for appendicitis two years ago now presents
with symptoms of an acute small bowel obstruction. The most likely and most common cause
is adhesions. In such instances spontaneous resolution often follows conservative
management. The careful clinician will ensure that less common but weightier causes
external hernia or colonic malignancy, which would require corrective urgent surgery are
considered and excluded.
The dominant cue may provoke urgent action by triggering a process of pattern-recognition,
which is close to an end-diagnosis. This is essential in life-threatening situations. For example,
if a patient presenting after a motor crash is gasping for breath and has a weak pulse with one
side of his chest moving less than the other, then does he have

1 Glass, Reuben D. Diagnosis: A Brief Introduction. Oxford University Press, Australia. 1996
137
138
2-A
The Diagnostic Process History-taking and
Problem Solving


a life-threatening tension pneumothorax requiring urgent needling? Pattern-recognition is also helpful in dealing
efficiently with common diseases, provided the clinician remains alert for deviations from the regular pattern.
Excessive reliance on pattern-recognition may lead to failure to consider diagnostic alternatives methodically.
If the dominant cue does not demand immediate action, further evidence is gathered. The clinician changes from a
rapid, pattern-recognition mode of functioning, to a measured consideration of hypotheses. The first task is to
elaborate details of the patient's presenting complaint and confirm that the dominant cue has been interpreted
correctly. Then provisional hypotheses, which the cue suggests, are explored. Groups of observations may raise the
possibility of related diseases.
Problem-oriented questions probe a subject in depth when the dominant cue suggests that this is warranted,
keeping in mind the likelihood of a disease, and the value of treating it. Most questioning attempts to elicit symptoms
to support a diagnosis, but the absence of a feature in history or examination may be of great diagnostic value.
During the diagnostic encounter, the clinician is mindful of the events that have influenced the patient's life, leading to
the presentation for medical attention. Screening questions, to assess whether a problem may exist in another
system or subject area, form part of a complete assessment. These questions reflect a doctor's style. In a full
consultation, the background of the patient needs to be known. This involves a history of past illnesses, enquiry about
other family members, their health and occupations, and interpersonal relationships.
Rather than relying on question lists for system reviews, experienced clinicians choose questions that give the highest
yield for the specific situation, collecting the most critical information early in the interview. They structure data,
processing a limited amount of information at one time in meaningful packets. It is often helpful to summarise the
events that led to the patient's presentation for medical attention, and to repeat this concisely to the patient.
This helps to ensure that the clinician understands what the patient meant by what he said, For example, the patient
may have said 7 seem to have peed an awful lot lately'. The clinician may summarise this as 'you have been
concerned about passing large amounts of urine'; and the patient may respond
l
no, I don't do much, it's just I have to
go often'. By repeating the information in unambiguous but nontechnical language to the patient, this clinician has
avoided the traps of incorrectly mentally coding the dominant cue as 'polyuria instead of 'frequency'. On other
occasions, the clinician's summary may trigger a further comment from the patient, about some important item that
was not mentioned.
If the situation does not require immediate action, the clinician usually develops a number of hypotheses often
three about how the patient's symptoms might be explained. Thus abdominal pain may be due to an
intra-abdominal cause, or result from referred pain due to intrathoracic or spinal causes. The data is checked to see if
it explains the various hypotheses. If so, the hypothesis becomes active; otherwise it is rejected. Beginners tend to
jump to conclusions too rapidly, considering one hypothesis at a time, until the evidence against it is overwhelming,
only then allowing them to move to another hypothesis. Experts 3onsider a number of possibilities and rank them.
This process occurs early in diagnostic :hinking. The ultimate refinement is called a 'differential diagnosis'. Early
lists of hypotheses nay include some specific diagnoses (for example, 'acute appendicitis'), but are mainly ideas
about disease groupings (for example, 'intra-abdominal mischief). Compiling a set Df active hypotheses often
involves including some inclusive category (such as 'something
139
2-A
TheDiagnosticProcessHistory-takingand
ProblemSolving


odd'),whichservesuntilattentionisdrawntothatsituationandisevaluatedmorefully.Thussuddenvomiting
withoutprecedingnauseaisunusualandraisesthepossibilityofintracranialdisease.Diseaseshaveavariety
of forms and their manifestations often do not follow the typical textbook example. Experience provides a
picture of the variations. A common strategy is to alternate between questions aimed at confirming a
hypothesis,andothersaimedatdistinguishingitfromotherhypotheses.Hypothesesremainactivelongerif
they include common situations, or if they involve 'high stakes' that is, potentially serious diagnosis.
Logically,thismeansthathypothesesfavourreadilytreatableorremediablesituationseveniftheyareunlikely
(thoughevenmoreiftheyareprobable).
Aswithhistory-taking,clinicalexaminationcanbedividedintoproblem-orientedand
screening signs. Problem-oriented signs are best given in clusters, which relate to disease groups. Here,
normal, or negative findings can also be given in some detail. Less detail is generally appropriate in the
screeningcomponentoftheexamination.Anoutlineofpathwaysofdiagnosticandmanagementplanningis
showninFigure1.
Whensolvingclinicalproblems,clinicianstendtoreviewhypotheseseitherbysomeformofrule,byinformally
weighingprobability,byconsideringthesoundnessofthecause-and-effectrelationshipsoftheevidence,or
byacombinationofmethods.Fordescriptivepurposes,considerationofeachhypothesismaybedividedinto
stages. To determine the probability of disease,theclinician needs an awareness ofthe prevalence of the
disease in the population, and the usefulness of the observation. Thus stronger evidence is required for
diagnosingararediseasethanforacommonone.
Diagnosticreasoninginvolvesconsideringthevalueofitemsofinformation.Artificialmethodscanbeusedto
analyse the process. One method is 'revising the odds'. Before an observation is made, the 'prior odds' of
disease (chances of disease compared with nondisease) are expressed as a number or fraction. This is
multipliedbythe'likelihoodratio'(chancesofobservationindisease,comparedwithnondisease;or'leverage
oftheevidence'),togive'revisedodds'.Asimplermethodinvolves'weighingtheevidence',wheretheinitial
ideaandsubsequentobservationsaregivenscoreswhichareadded.
Diagnosticerrorsarisewhencliniciansconfusepopulationswithdifferingfrequencyofdisease.Malariamay
betheprobablediagnosisofafebrilepatientinatropicalareawherethediseaseisendemic,butinasimilar
patientinatemperatezone,moreevidencewillberequiredtoconfirmthatdiagnosis.Hoofbeatsusuallyimply
horses,notzebrasbuttheoddsrequirerevisioninAfrica.
Itisalsoimportanttorealisethattheabsenceofafeaturedoesnotnecessarilyhavethesameleverageor
weightasitspresence:therelativevalueofeachmaybecompletelydifferent.Thusseverecrushingcentral
chestpainassociatedwithpallorstronglysuggestsmyocardialinfarction,however,asthisfeatureisabsentin
half of the patients with acute infarcts, the absence of the 'classical pattern' is of limited diagnostic value.
Similarly, the presence of a bruit over the thyroid is diagnostic of hyperthyroidism. The absence of a bruit,
however,isonlyweakevidenceagainstthediagnosis.Sometimesabsenceofafeaturea'normal'finding
may be of great value in differential diagnosis. For example, absence of anorexia or nausea is evidence
against the diagnosis of appendicitis, though the presence of these features is of little diagnostic value.
Knowledgeablecandidates will highlight discriminatory 'normal' findingsinproblem definition,in contrast to
'normalfindings'onscreening.
Theneedforactiondependsonthelikelihoodofadisease,andthevalueoftreatingit.Thusevaluatingfully
thepossibilityofararediseasethatisamenabletotreatmentisimportant,althoughperhapslessimportantif
treatmentisunsatisfactory.
140
2-A
TheDiagnosticProcessHistory-takingand
ProblemSolving


Gatheringclinicalorlaboratoryinformationaboutapatientcostsmoneywhetherfromdoctororpatienttime,
inconvenience,orequipment.Whileinformationmustbecompleteforthepurposeofdecidingmanagement,
it needs to be obtained efficiently. In deciding on patient management, two threshold levels of likelihood of
disease are important. Below the lower level, no amount of new evidence would suggest that treatment is
warranted. Above the upper threshold, no additional evidence is going to alter the treatment decision. While
some redundancy of information is appropriate when there is significant doubt or risk, unnecessary
investigationsshouldbeavoided.Attimes,theymayevenbemisleading.For example,ifacuteappendicitis
is confidently diagnosed on clinical evidence, a normal leucocyte count should not be taken as a
contraindicationforsurgery.
PathwaysinthediagnosticprocessaresummarisedinFigure1.Examinationcandidatesshouldrecognise
thatquestionshavebeendesignedtotestskillsindifferentaspectsof the diagnosticprocess,generallyusing
role playing patients. Candidates should read and follow instructions carefully. On occasions, they may be
askedtoelaborateonapatient'shistory;onothers,toexamineaparticularorganorsystemorregion.Other
scenarios may give all relevant information, requiring a candidate to deduce the diagnosis, or differential
diagnosticpossibilities,andexplainthemanagementtothepatient.Inallcases,theability toachieverapport
withthepatientisexpected.

ReubenDGlass
2-A
TheDiagnosticProcess
History-takingandProblemSolving


SECTION2-A.FIGURE1.
Pathways in the diagnostic process. In practice, the clinician uses all these methods.
Examinationscenariosareoftendesignedtotestonecomponentoftheprocess.

Arrangefollowupifrequired
Patternrecognition.Rapiddiagnosticpathway,essentialforurgentsolutions.Recognitionofatypical
patternmaybeusedforcommonproblems.
Problem-orienteddiagnosisgeneratingandevaluatinghypothesis.
Afullsystematicenquiryisneededforcompletepatientcare.
141
142
2-A
TheDiagnosticProcessHistory-taking
andProblem-solving


2-A TheDiagnosticProcessHistory-takingandProblem-solving
CandidateInformationandTasks
MCAT030-043
30 Jaundiceinabreastfedinfant
31 Aconvulsionina14-month-oldboy
32 Loudanddisruptivebehaviourofa6-year-oldboy
33 Tremorina40-year-oldman
34 Headacheina35-year-oldwoman
35 Lethargyina50-year-oldwoman
36 Syncopeina52-year-oldman
37 Apainfulpenilerashina23-year-oldman
38 Primaryamenorrhoeainan18-year-oldwoman
39 Askinlesiononthecheekofa50-year-oldman
40 Apigmentedmoleonthetrunkofa30-year-oldwoman
41 Anitchyrashonthehandsofa19-year-oldwoman
42 Redpainfuldryhandsina30-year-oldbricklayer
43 Swellingofbothanklesina53-year-oldwoman
143
030
Candidate Information and Tasks



Condition 030
Jaundice in a breastfed infant

CANDIDATE INFORMATION AND TASKS
Baby Helen is brought to see you in a general practice setting, as her mother is concerned about her continuing
jaundice. Helen is now two weeks old and was born at term by easy vaginal delivery weighing 3.7 kg. Apgar scores
were 9 and 10 (at 1 and 5 minutes respectively).
She became jaundiced in the neonatal period starting on day three. Investigations then revealed no blood group
incompatibility, both mother and baby being group O positive and no red blood cell (including enzymes) abnormality.
The infant was treated with phototherapy for two days. Since discharge from hospital at eight days of age the jaundice
has persisted and the mother is concerned. Baby is feeding well from the breast. Current weight is 3.9 kg.
Examination findings
The baby was active and clinically normal apart from the jaundice when you saw her yesterday. You arranged
investigations as set out below. The mother has now returned with the baby to discuss the results and your advice
about treatment.

Investigation results
Serum bilirubin Total: 250 umol/L
~ Conjugated: less than 10 umol/L
Neonatal thyroid screening normal
Urine culture sterile
Full Blood Examination (FBE) normal

YOUR TASKS ARE TO:
Obtain any further necessary history you require. You should not take more than 2-3 minutes to do this.
Discuss the results of investigations with the mother.
Explain the diagnosis to her and advise about future management.

The Performance Guidelines for Condition 030 can be found on page 156
144
031-032
Candidate Information and Tasks



Condition 031
A convulsion in a 14-month-old boy

CANDIDATE INFORMATION AND TASKS
Benjamin, a 14-month-old boy, has been brought in to the hospital Emergency Department by his
parent following an episode at home the previous evening. His parent explains that he had been
unwell all day with a high fever (40 C), and while he was being cuddled, he was staring and did
not respond to his name. They noted that his body twitched all over for several seconds and the
whole episode lasted 60 seconds. He then went off to sleep and slept for the rest of the night.
Examination findings
Benjamin is alert and normal neurologically. He has a low grade fever and signs of an upper j
respiratory tract infection.
YOUR TASKS ARE TO:
Take any further history to ascertain the most likely cause for this episode.
Explain your diagnosis and subsequent management to the child's parent.

The Performance Guidelines for Condition 031 can be found on page 159

Condition 032
Loud and disruptive behaviour of a 6-year-old boy

CANDIDATE INFORMATION AND TASKS
You are seeing a 6-year-old boy, J onathan, for the first time with his mother, who complains how
active he is. He is in his second year at school, and his teacher has commented that he is
disruptive and loud in class.
YOUR TASKS ARE TO:
Take a focused history from the mother to determine the possible causes for the child's
presentation.
Indicate to the mother your probable diagnosis and a brief plan of management. The
Performance Guidelines for Condition 032 can be found on page 161
145
033-034
Candidate Information and Tasks



Condition 033
Tremor in a 40-year-old man

CANDIDATE INFORMATION AND TASKS
You are working in a general practice. Your next patient is a 40-year-old man who is consulting
you because of a tremor ('the shakes').
YOUR TASKS ARE TO:
Take a focused history about his condition.
After completing the history, discuss possible diagnoses with the patient.
The examiner will then question you about the physical findings you would check to clarify
the diagnosis, and any investigations you would arrange.

The Performance Guidelines for Condition 033 can be found on page 164

Condition 034
Headache in a 35-year-old woman

CANDIDATE INFORMATION AND TASKS
Your patient, a 35-year-old woman, is consulting you in the Emergency Department of the local
hospital about headaches. You have not seen the patient before.

YOUR TASKS ARE TO:
Take a focused history from the patient.
Request from the examiner the essential features of the physical examination you would look
for in this patient as the next stage in diagnosis. The examiner will inform you of the results.
Tell the patient what you consider to be the most likely diagnosis and what investigations, if
any, should be undertaken.

The Performance Guidelines for Condition 034 can be found on page 167
035
Candidate Information and Tasks



Condition 035
Lethargy in a 50-year-old woman
CANDIDATE INFORMATION AND TASKS
You are consulting in a general practice setting. Your next patient is a middle-aged widow (see
Figures 1 and 2 below) who is presenting for a 'check-up'. She looks apathetic and lethargic on
first impression.
YOUR TASKS ARE TO:
Take a focused history.
Ask the examiner for the results you would wish to elicit on a focused physical
examination.
Give your diagnosis and differential diagnosis to the examiner, and indicate what further
investigations you would require.
CONDITION 035. FIGURE 1. CONDITION 035. FIGURE 2
The Performance Guidelines for Condition 035 can be found on page 170
146
147
036
Candidate Information and
Tasks



Condi t i on 036
Syncopeina52-year-oldman
CANDIDATEINFORMATIONANDTASKS
You are working in a general practice. Your next patient is a 52-year-old technician who is
consultingyouaboutrecenttransientlossofconsciousness.

YOURTASKSARETO:
Takeahistoryfromthepatient.
Asktheexaminerforthefindingsofthefocusedphysicalexaminationyouwouldperform.
Telltheexamineryourdiagnosisandthereason(s)forthis.
Indicatetothepatienthowyouwouldproceedinyourfurtherassessmentofhiscondition.


ThePerformanceGuidelinesforCondition036canbefoundonpage173



Condition 037
A painful penile rash in a 23-year-old man
CANDIDATE INFORMATION AND TASKS
You are the Hospital Medical Officer (HMO) in a hospital primary care clinic. A 23-year-old man
presents with penile pain and a penile rash.
YOUR TASKS ARE TO:
Take a focused history to assess the presenting problem.
Explain to the patient your provisional diagnosis and recommended management.
The penis appears as in the illustration below.
CONDITION 037. FIGURE 1.
The Performance Guidelines for Condition 037 can be found on page 177
148
149
038
Candidate Information and
Tasks



Condition 038
Primary amenorrhoea in an 18-year-old woman
CANDIDATE INFORMATION AND TASKS
You are working in a general practice. Your next patient is an 18-year-old woman who is
concerned because she has never had a menstrual period.

YOUR TASKS ARE TO:
Take any further relevant history you require.
Ask the examiner for relevant findings you wish to ascertain on general and gynaecologic
examination.
Advise the examiner of investigations you wish to order and your provisional diagnosis.
Counsel the patient appropriately.

The Performance Guidelines for Condition 038 can be found on page 180
039
Candidate Information and Tasks



Condition 039
A skin lesion on the cheek of a 50-year-old man
CANDIDATE INFORMATION AND TASKS
You are the Hospital Medical Officer (HMO) in a hospital primary care clinic. A retired
50-year-old builder presents with a skin lesion on his right cheek.
YOUR TASKS ARE TO:
Assess the lesion by a focused history and physical examination.
Present your case summary and diagnosis to the examiner.
Explain to the patient your recommended management.
The lesion appears as in the illustration below.

CONDITION 039. FIGURE 1.

The Performance Guidelines for Condition 039 can be found on page 182
150
040
Candidate Information and
Tasks



Condition 040
Apigmentedmoleonthetrunkofa30-year-oldwoman

CANDIDATEINFORMATIONANDTASKS
You are working in a general practice. A fair complexioned 30-year-old schoolteacher seeks
advice concerning a 'mole' on her trunk. The skin lesion appears as in the illustration of her
backshownbelow.
YOURTASKSARETO:
Assessthelesionbyafocusedhistoryandphysicalexamination.
Explaintothepatientyourdiagnosisandrecommendedmanagement.
CONDITION040.FIGURE
ThePerformanceGuidelinesforCondition040canbefoundonpage184
151
041
Candidate Information and Tasks



Condition 041
Anitchyrashonthehandsofa19-year-oldwoman

CANDIDATEINFORMATIONANDTASKS
You are a Hospital Medical Officer (HMO) in a general medical outpatient clinic. A 19-year-old female
computerstudentpresentswithanitchyrashonherhands.Therashhasbeenpresentforaboutoneweek
andappearsasshowninthephotograph.Therearenootherabnormalexaminationfindingsapartfromthe
rash.
YOURTASKSARETO:
Takeafurtherfocusedhistory.
Explainthemostlikelydiagnosistothepatientandhowthiscanbeconfirmed.
Advisethepatientabouttreatment.
CONDITION041.FIGURE1.
ThePerformanceGuidelinesforCondition041canbefoundonpage186
152
042
Candidate Information and
Tasks



Condition 042
Redpainfuldryhandsina30-year-oldbricklayer
CANDIDATEINFORMATIONANDTASKS
YouaretheHospitalMedicalOfficer(HMO)inahospitalprimarycareclinic.A30-year-oldmanpresentswith
reddryhands.Therashappearsasintheillustrationbelow,andisonthefrontandbackofbothhands
YOURTASKSARETO:
Takeahistoryaboutthepresentingproblem.
Explaintohimyourdiagnosisandthepossiblecausesofthecondition.
Outlineyourmanagementoftheproblem.
CONDITION042.FIGURE1.
ThePerformanceGuidelinesforCondition042canbefoundonpage189
153
043
CandidateInformationandTasks



Condition 043
Swellingofbothanklesina53-year-oldwoman
CANDIDATEINFORMATIONANDTASKS
Youareworkinginageneralpractice.Thepatientyouareabouttoseeisa53-year-oldclerical
workerwhoisconsultingyouaboutbilateralswollenankles,asillustratedbelow.
\
YOURTASKSARETO:
Takearelevanthistoryfromthepatient(youhavefiveminutestodothis).
Tellthepatientyourworkingdiagnosisastowhytheanklesareswollenafterconcluding
thehistory.
Describetotheexaminertheessentialfeaturesyouwouldlookforinphysical
examinationtoconfirmyourprovisionaldiagnosis.
CONDITION043.FIGURE1.
ThePerformanceGuidelinesforCondition043canbefoundonpage191
154
155
2-A
TheDiagnosticProcess
History-takingandProblem-solving


2-ATheDiagnosticProcessHistory-takingandProblem-solving
PerformanceGuidelines
MCAT030-043
30 Jaundiceinabreastfedinfant
31 Aconvulsionina14-month-oldboy
32 Loudanddisruptivebehaviourofa6-year-oldboy
33 Tremorina40-year-oldman
34 Headacheina35-year-oldwoman
35 Lethargyina50-year-oldwoman
36 Syncopeina52-year-oldman
37 Apainfulpenilerashina23-year-oldman
38 Primaryamenorrhoeainan18-year-oldwoman
39 Askinlesiononthecheekofa50-year-oldman
40 Apigmentedmoleonthetrunkofa30-year-oldwoman
41 Anitchyrashonthehandsofa19-year-oldwoman
42 Redpainfuldryhandsina30-year-oldbricklayer
43 Swellingofbothanklesina53-year-oldwoman
030
156
Performance
Guidelines

Theexaminerwillhaveinstructedtheparentasfollows:
Condition 030
Jaundiceinabreastfedinfant
AIMSOFSTATION
To assess the candidate's knowledge of the causes of persisting neonatal jaundice, ability to make an
appropriatediagnosisandtoconveythistothepatient

This is your first baby. You have always intended to breastfeed, and you established breastfeeding
successfully. BabyHelen hasbeennormal in all respects since birth. She has gained weightand is having
normalbowelactionsmotionsandurinearebothnormallycoloured.Youareahealthy,breastfeeding
mother,nonsmoker,onnomedication.
Questionstoaskunlessalreadycovered:
' Does Hel en need t o go under t hose l i ght s agai n?'
Ist her e somet hi ng wr ong wi t h my mi l k, shoul d I st op br east f eedi ng?'
If the candidate indicates that breastfeeding should be ceased for 1-2 days to see if the jaundice
decreases,ask'Wi l l t hi s af f ect my capaci t y t o cont i nue br east f eedi ng?'
EXPECTATIONSOFCANDIDATEPERFORMANCE
Thecandidateshouldrecognisethesignificanceofthebaby'sunconjugatedhyperbilirubinaemiaandcometo
themostlikelydiagnosis(breastmilkjaundice)withoutneedforfurtherinvestigations.Establishingthatawell
baby with persisting jaundice has normal-coloured urine and motions and no abnormality on examination
virtuallyclinchesthediagnosis.
Thecandidateshouldexplainthat:
Themostlikelydiagnosisisbreastmilkjaundice.
Theconditionisself-limitingandrequiresnotreatment.
The diagnosis can be confirmed by suspending (not stopping) breastfeeding for 24-48 hours which will
resultinafallintheserumbilirubinafterwhichbreastfeedingcanbecontinued.Thisisacceptablebutnot
necessary.
Themothershouldbeadvisedtoexpresshermilkinordertomaintainlactationiftemporarysuspensionof
breastfeedingisadvised.
Emphasisethatthereisnothingwrongwithhermilk.Notreatmentisindicatedandfurtherphototherapyis
notindicated.
030
PerformanceGuidelines

KEYISSUES
Establishingthaturineandstoolsareofnormalcolour.
Accurateinterpretationofimportantpathologyresults.
Reassurancetomotherthathermilkisnotharmfultoherbaby.
Accurateexplanationofthepossiblecausesforthejaundiceandlogicallyexcludingotherimportant
diagnoses.

Notappreciatingthesignificanceofpredominantunconjugatedhyperbilirubinaemiaandinsistingthatthe
babyhasbiliaryatresiaorhaemolyticdisease.
Insistingthatthebreastmilkisunsatisfactoryforthebabyandrecommendingpermanentcessationof
breastfeeding.

Jaundiceinthenewbornisverycommon,affectingatleast50%offullterminfantsandasignificantlyhigher
proportion of premature infants. Generally termed physiological jaundice, this is an adaptation process to
extrauterinelifethattakessomedaystomature.Mostphysiologicaljaundiceresolvesbytheendofthefirst
weekoflifeinfullterminfantsandupto10daystoafortnightinpreterminfants.
Withsuchacommoncondition,guidelinesarethereforerequiredtodeterminewhataspectsofjaundicefall
outsidethisphysiologicalrange.Thereareamultitudeofcausesforjaundicebesidesphysiologicaljaundice
andallmustbeconsideredbeforeassumingthatthejaundicehasabenignorigin.
However,jaundicegenerallycanbeconsideredpathologicalifitoccursoutsidetheaboveagerangeorismore
pronouncedthanexpected.Usuallyphysiologicaljaundicedoesnotbecomeobviousinfullterminfantsuntil
daythreeoflifeandbylatedaytwoinprematureinfants.Jaundicethatismanifestinthefirst24hoursoflife
usuallyhasapathologicalcauseandthismustbesought.
The most common cause for this in our community is a haemolytic process, usually ABO incompatibility
althoughthereareseveralothermorerarecausesofhaemolysisthatmayalsopresentatthistime(suchas
hereditaryspherocytosisandglucose-6-phosphatedehydrogenase[G6PD]deficiency).Rhisoimmunisationis
usuallyknownpriortobirthfromthescreeningtestsdoneonthemotherduringherpregnancy.
Theotherextremeofthenaturalhistoryofjaundiceisprolongedjaundice,thatis.jaundicethatpersistslonger
than the time when it would have been expected. There are many possible causes for prolonged jaundice
which can usually be differentiated by whether the jaundice is predominantly unconj ugat ed or
conj ugat ed, thelatteroftenindicatingamoresignificantaetiology,forexample,biliaryatresia.

1
030
158

Performance
Guidelines

Breast milk jaundice fits into the unconj ugated variety but should be considered as a diagnosis by
exclusionoftheotherimportantcausesofunconjugatedhyperbilirubinaemia.Thereare,however,important
clinical features that also would support this diagnosis Obviously the babies are breastfed, but are thriving
andgainingweightwell.Theirbowelmotionsandurinearenormalcolour,afeaturethatmustbedetermined
whenassessingtheseinfantsPhysicalexamination,exceptforthejaundice,isentirelynormalandimportant
investigationstoexcludeothercauses,forexample,hypothyroidism,arenormal.
Breast milk jaundice is a benign condition, which requires no treatment except explanation and support.
Breastfeeding should not be interrupted although some authorities indicate that the diagnosis can be
confirmedbyceasingbreastfeedingtemporarilyanddemonstratingasignificantfallinthebilirubinlevel.This
israrelynecessaryandtendstogivethemessagetoyoungmothersthattheirmilkisharmfultotheirinfant,
whichisnottrue.
Thecauseisunknown,withmanytheoriesexpoundedinanattempttoexplainthisphenomenon,althoughit
is thought to be due to a factor in breast milk that causes increased enteric absorption of bilirubin. The
jaundicemaypersistforuptothreemonthsbutoverthattimethebabythrivesandremainswell.Despiteits
persistenceformanyweeksinsomeinfantsitmaydisappearinjustafewdaysinothers.
031
159


Performance
Guidelines
Condition 031
A convulsion in a 14-month-old boy
AIMS OF STATION
To assess the candidate's ability to relate appropriately with the parent of a boy who had a febrile convulsion
the previous evening, by taking a relevant history making and explaining the diagnosis, and advising
preventive measures.
The examiner will have instructed the parent as follows
You are the parent of 14-month-old Benjamin who had a convulsion with generalised twitching the previous
evening. You did not know what the episode was at the time but as he went off to sleep, you let him sleep
and brought him in the next day to be checked. He now seems well and is back to normal, but you are
concerned as to what the episode was as it frightened you when it occurred.
If asked about his development, reply that all has been normal.
His birth history was normal born at term weighing 3600 g (good Apgar birth scores).
He has been well since birth and has not been sick until last night.
There is no family history of epilepsy.
Your own younger sister had a similar episode when aged two years, but has had no further problems
(this is potentially important information but should be provided only if the candidate asks).
Opening statement
Benjamin seems fine now that he is less feverish. '
Questions to ask if not already covered:
'Why would he have this episode?'
If the candidate indicates that this was a febrile convulsion, then ask 'what is a febrile convulsion?'
Is this epilepsy? Should he have some medicine for this?'
'Does he need to have any tests done?'
Will it occur again?'
What should we do if it does occur again or when he gets a high temperature?'
EXPECTATIONS OF CANDIDATE PERFORMANCE
Although this boy has had a simple febrile convulsion, the candidate should question specifically to exclude
any risk factors for epilepsy or recurrence, including birth history developmental history, and family history of
febrile convulsions or epilepsy.
The candidate should:
Indicate that this is most likely a simple febrile convulsion
Explain that a febrile convulsion is a common reaction to fever in young children about 3% of the
population have a seizure associated with fever.
160
031
Performance Guidelines
Emphasise that generally this is a benign condition.
Point out that the condition commonly runs in families.
Explain that the fever is almost always caused by a viral infection, which would usually manifest as an upper
respiratory tract infection in an adult, but immature brains are susceptible to the effects of high fever.
Explain that such convulsions do not cause brain damage or subsequent epilepsy.
Give advice as to a 30%risk of recurrence especially in the first 24 hours after this episode and advise the parent
what to do if it occurred again (tepid sponging and antipyretics when he is feverish).
Advise against any drug treatment (except an antipyretic), as this is unnecessary.
Reassure that this is most unlikely to be epilepsy. Only around 3%of children who have had febrile convulsions
subsequently develop epilepsy, being mainly in the high risk group (positive family history of epilepsy, prolonged
convulsion, a focal element, abnormal development before the seizure).
Candidate may mention the use of rectal diazepam for recurrent febrile seizures, but this should be reserved for
special circumstances and its use is not without risk.
KEY ISSUES
Appropriate questioning and history-taking.
Appropriate education and reassurance.
Advice on preventive measures.
CRITICAL ERROR
Suggesting on the strength of this episode of a brief febrile convulsion that he has epilepsy.
COMMENTARY
In this scenario, the dominant cue is the recognition of the change in sensorium in this child while he has a high fever.
Pattern recognition leads to the likely diagnosis and should allow the candidate to make a definitive diagnosis and
be confident in the advice given to the parent. The scenario tests not only knowledge of a common condition, but also
skill in being able to impart a logical explanation in language that the parent can understand without causing alarm. It
also requires reassurance and confidence in the diagnosis. While epilepsy may be raised as a possibility, the clinical
features at the time, the normal appearance of the child when examined and absence of risk factors, make this highly
unlikely at this stage. The candidate should therefore recognise this as a simple febrile convulsion with an excellent
prognosis.
A simple febrile convulsion is a common condition and candidates must be aware of risk factors that would suggest an
alternative diagnosis of epilepsy should be considered: prolonged convulsion lasting greater than 15 minutes; a focal
element to the seizure; a family history of epilepsy; or abnormal neurological behaviour in the child prior to the seizure.
Any of these features would throw doubt on the diagnosis of simple febrile convulsion. This is the process of
diagnostic reasoning.
None of these features is present in this child and hence the diagnosis is clearly most likely to be a simple febrile
convulsion.
032
PerformanceGuidelines



Condition 032
Loudanddisruptivebehaviourofa6-year-oldboy
AIMSOFSTATION
ToensurethatthecandidateconsidersallotherpossibilitiesastowhyJonathanisbehavingas
heis,beforeconcludingthathemostlikelyhasattentiondeficithyperactivitydisorder(ADHD).
Thecandidatewillthereforebeassessedonabilitytoaskpertinentquestionsthat confidently
excludeotherimportantdiagnoses.

Theexaminerwillhaveinstructedtheparentasfollows:
YouarethemotherofJonathanagedsixyearsandareconcernedabouthowactiveheis.Both
teachershehashadinthefirsttwoyearsatschoolhavecommentedonhowdisruptiveheisin
class.
The candidate is required to take a concise history from you to try to determine the possible
causesforJonathan'sbehaviour.Listedbelowaresuggestedanswersthatyoushouldprovide
inresponsetoquestionsfromthecandidate.
Atsixminutes,theexaminerwillindicatetoyoutoaskthecandidatewhatistheproblemand
howshouldJonathanbemanaged.
Foreaseofresponse,questionsaregroupedbasedonhowthecandidatemayquestionyou.
Hyperactivityandotherassociatedsymptoms
Jonathanhasalwaysbeenveryactivefromthetimehewalked(13months).
Evenasaninfanthewasarestless,irritablebabywhowasdifficulttofeed.
Hissleepingpatternsincebirthhasbeenverybizarrewithfrequentwaking.
Heisimpulsiveandoftenactswithoutthinkingwhattheconsequencesmightbe.
Heiseasilyfrustratedandrarelyplaysatanyparticularactivityforanylengthoftime.
Histeachercomplainsheneversitsstillandtendstowanderaroundtheclassroom,whichat
timesisdisruptivetotherestoftheclass.
Schoolprogress
Hisreadingandlearningarebehindtheleveloftheotherchildrenintheclass.
Heconstantlytalksloudlyinclass.
Hetendstointerruptfrequentlyandhasgreatdifficultytakingturns.
Hehastroublekeepingfriends,asheissoboisterous.
Hispreschoolteachercommentedonsimilarbehaviour.
Homesituation
Heisthefirstinthefamily;hehasafourandahalf-year-oldsisterwhoalreadyisreading,
almosttothelevelthatJonathanhasachieved.
Helivesinacaringfamilyandyouhavetriedjustaboutanythingthatanyonehassuggested
tohelphim.
Thedescriptiongivenbytheteachersisexactlyasheisathomeaswell.
Hecannotconcentrateontasksforanylengthoftime.
161
162
032
PerformanceGuidelines
He tends to disrupt his younger sister's play.
He demands attention from you frequently, not being able to take 'no' for an answer which you
are finding exhausting.
You consider your home situation to be very stable with both parents active in supporting each
other and rearing the children. There are no parental interpersonal problems.
Famil y history
His father is a plumber who struggled at school, left school as early as he could to take up his
apprenticeship and who found it difficult to study at Technical College (TAFE).
Father is described by his own parents as being very similar to J onathan at J onathans age.
Father even now tends to leave tasks unfinished around the home. This is not a problem at
work where he is supervised.
There is no family history of intellectual disabilities or deafness.
Mother was previously a secretary for a builders' supply firm.
Pregnancy
You had a normal pregnancy. J onathan's birth weight was 3600 g at term after normal labour;
good Apgar scores; only mild and transient jaundice in the neonatal period.
You noticed very active movements even in utero compared to when you were pregnant with
his sister.
J onathan thrived and gained weight well, development was normal except slow in speech
development compared to his younger sister.
Past medical history
Physically well.
No serious illnesses (in particular no history of meningitis, encephalitis).
No history of head injury, although he appears uncoordinated and constantly seems to run
into things.
You have had his hearing and vision tested and were assured both were normal (give this
information ONLY if asked).
When indicated by the examiner, you should ask: What do you think is causing this, and
what can we do about it?'
EXPECTATIONS OF CANDIDATE PERFORMANCE
This scenario is designed firstly to assess the candidate's ability to explore the reasons a ! child
may present with hyperacti vity. While it has been written to suggest ADHD, the assessment is
essentially based on whether the candidate approaches the problem in a logical manner,
exploring all other possibilities for the behaviour. It is not at all critical that the correct diagnosis is
made, but rather that the child's history is fully explored to exclude:
Physical or congenital lesions.
Behaviour secondary to neonatal problems.
Subsequent medical or physical illnesses including significant head injury.
Reaction to unfavourable home circumstances or child rearing and family environment.
Difficulties in the child's adaptation to school.
A visual or auditory problem.
032
Performance Guidelines



Importantly, the candidate should question as to similar behaviour in other family members (see note on
father's history)
In outlining briefly the probable diagnosis and plan of management, the candidate should:
Give a logical explanation as to the possible causes for Jonathan's presentation having excluded organic
pathology and indicate that ADHD is a strong possibility (the diagnosis of ADHD is not essential to pass,
provided history has been satisfactory).
Suggest a neurodevelopmental assessment be performed with an experienced psychologist if possible.
If hearing and vision have not been enquired about, suggest having hearing and vision assessed (can then be
told this has been done and is normal).
Briefly outline the management of ADHD by behavioural modification with or without stimulant medication.
Indicate willingness to review and monitor progress in a supportive manner.

Obtaining a logical and focused history to exclude various possible causes for Jonathan's behaviour.
Showing empathy with the parent's frustration.
Having a clear approach to the management plan.

CRITICAL ERROR
Coming to a premature conclusion of ADHD and recommending stimulant medication, without having explored the
history for other causes.

This presentation of childhood hyperactivity is common in both paediatric and general practice and skill is required
to sort out the myriad of causes for the behaviour exhibited. This is done by a careful history exploring the child's
past history, current developmental status, and family and school situation. Monitoring progress over several
consultations may be required to give a positive diagnosis of ADHD.
The great majority of children presenting in this manner have major family dynamics problems and the scenario
could easily have been modified to indicate this. However the current scenario is designed to indicate that a
diagnosis of ADHD is by exclusion of all the other causes for this symptom; and that stimulant medication is not the
immediate treatment in this situation, a tendency for which has developed in the general community and is often the
expectation of family members who have spoken with neighbours or read the lay press. The doctor is often under
great pressure to prescribe these medications at first visit and this should be resisted.
There are no specific tests available that make a diagnosis of ADHD. Diagnosis is made from the pattern of
behaviour, which can be confirmed by psychometric testing. While behavioural modification is recommended,
parents have often already arranged this prior to the consultation, and also have often tried a variety of exclusion
diets that in some cases may be helpful, but overall are not. Parents should be supported to try exclusion diets if
they are keen to institute them; but the practitioner should monitor progress and make a critical appraisal as to the
success or otherwise of this treatment and support the parent accordingly.
163
033

PerformanceGuidelines
Condition 033
Tremorina40-year-oldman
AIMSOFSTATION
Toassessthecandidatesskillindefiningapresentingsymptomoftremor,makingaprobability
diagnosis from the history and selecting with discrimination which aspects of physical
examinationandinvestigationswillclarifythediagnosis.
Theexaminerwillhaveinstructedthepatientasfollows:
Youarea40-year-oldmanwhohasconsultedageneralpractitionercomplainingoftheshakes'.
Thedoctorwillseekdetailsofyoursymptomsandwillaskquestionsaboutyourhealthstatus,
medicalandsocialhistory.
Openingstatement
' I' vegottheshakesdoctor.Ispillmydrinksometimes' .
Providethefollowinginformationwithoutprompting:
I have had the shakes since my early 20s. It happens when I get nervous about
something, but it hasn' t been a problem until recently (6-12 months). Sometimes
my head shakes and often I spill my drink when I put a glass to my mouth. I feel
much better after a couple of beers. Recently I heard something about Parkinson's
diseasewhichcancausetheshakes,soIthoughtIshouldseeadoctor' .
Inresponsetofurtherquestioning:
' It doesn't seem to bother me when I get up in the morning and doesn' t stop me
goingtosleep.Itcangoawayforafew daysthencomes back.SeemstobewhenI
am doing something with my hands like using a knife and fork, or writing. The
newspaper shakes when I am trying to read it. Sometimes I have trouble lighting a
cigarette. My right hand is the worst. I don' t have any stiffness and I don' t have
trouble moving from one position to another, nor in walking. I can control the
shakesbygrippingthingsfirmly.'
You have noted that your hands and fingers shake if you hold your arms out in front of your
body.Youmayaskthedoctorifyourvoicecouldbeaffectedbecauseyouhavenoticedsome
shakinessinyourvoice,butonlyoccasionallyandofminordegree.
Shakeswerefirstnotedinyourdominantrighthand.Lefthandislessaffectedandnot until
recently.
Youhavealwaysbeenatenseandnervyperson,stressmakestheshakesworse.Youliketo
havea'fewbeers'especiallyattheweekendsbutneverget'drunk'.
Deny any other symptoms affecting the central nervous system, cardiovascular system,
respiratory,gastrointestinalorurinarysystems,butadmittoadiminishedlibidoand difficulty
inmaintaininganerectionifaskeddirectlyorif thedoctorprovidesan opportunityforyouto
doso.Nolossofweight.
You work as a storeman. You are married with two teenage children. You smoke 10-15
cigarettesdailyanddrinkuptofive375mLcansoffullstrengthbeeronmostdays.Youhave
hadnoseriousillnessesoroperations.
164
033


Performance Guidelines
Family history: Father died from lung cancer aged 58 years. He also used to get the 'shakes'. He was a
non-drinker. Mother died of stroke aged 62 years. Her father had Parkinson disease. Your brother, sister, wife and
two children all keep in good health.
You are anxious about the cause of your symptoms. You are ignorant of long term harmful effects of alcohol. If the
doctor facilitates your story and maintains an open-ended approach whilst you are giving details of the shakes',
continue to amplify your symptoms. If, however, the doctor controls the interview too early, by only asking
questions and not listening to your story, just answer the questions asked. If asked, admit to concern about
Parkinson disease because of your grandfather. The same applies to your alcohol use: be reluctant to confirm or
reveal the true level of your alcohol intake. You have never been charged with exceeding .05, and have never had
an injury or motor accident associated with alcohol use.
EXPECTATIONS OF CANDIDATE PERFORMANCE
The candidate should assess the patient's tremor as follows:
History (see patient instructions)
By the process of listening (using an open-ended approach followed by direct questioning), to develop the two most
likely diagnostic pathways: benign tremor and tremor associated with heavy drinking. Other differential diagnoses
such as an anxiety state. Parkinsonism and thyrotoxicosis are much less likely from the history. Past, family and
social histories should be sought.
Discussion with patient after the history
This may or may not be specific. It is included to assess the candidate's diagnostic approach to the patient.
Knowledgeable candidates will be reassuring because of their confidence in the likelihood of a benign tremor.
Questions from the examiner after six minutes with expected responses:
What are your differential diagnoses?'
~ Benign or essential tremor (familial); effects of heavy drinking: Parkinson disease: cerebellar disease; thyrotoxicosis
'At this stage what do you consider to be the most likely diagnosis?'
~ Benign tremor or alcoholic tremor
"What are the essential physical signs you would look for in this patient?'
~ Hepatomegaly and any stigmata of chronic liver disease, cerebellar signs, increased muscle tone, tachycardia,
cardiomegaly
'What investigations would you advise assuming the physical examination to be normal?'
~ Full Blood Examination (FBE), liver function tests, possibly thyroid function tests.

Approach to patient establishing trust and confidence by having a non-judgmental attitude, listening to
patient's concerns and being reassuring.
History comprehensive but focused, using appropriate communication skills Diagnosis must include benign
tremor and alcoholic tremor.
165
166
033
PerformanceGuidelines



Answertoexaminerquestions
~Physicalexaminationshouldincludecheckingforhepatomegalyandstigmataof
chronicliverdisease,tachycardiaandcerebellarsigns.
~Investigationsmustadviseliverfunctiontests,butthyroidfunctiontestsnot
essential.

CRITICALERRORS
Failure to indicate the most likely diagnosis is essential tremor.
Failuretoadviseliverfunctiontests.

COMMENTARY
Essentialtremorisoneofthemostcommonneurologicaldisorders,withprevalenceincreasing
withage.Anautosomaldominantfamilyhistoryispresentin50-60%ofpatientsandthegenetic
basisisunknown.Functionalimagingrevealsabnormalcerebellaractivityandnohistologicalor
structural changes have been identified. Age of onset is bimodal. with the largest peak in the
seconddecade,andasmallerpeakinthefifthdecade.Thecharacteristicfindingisapostural
andkinetictremoroftheupperlimbswhichinterfereswithfinemanualtasks.Headtremorisalso
present in 40%. Less commonly legs are involved or there is voice tremulousness. With
advancingage,thetremorfrequencyoftenslowsandamplitudeincreases,leadingtoacoarse
tremorwhichcanbedisabling,althoughthisisuncommon.
Patientswithbenignessentialtremoroftendrinkasameansofcontrollingthetremorasalcohol
hasanamelioratingeffectin50%ofcases.
Inthiscase,thepatientmayalsobesufferingfromtheeffectsofprolongedheavydrinking.This
wouldrequirefurtherassessmentwiththeinvestigationsrecommendedabove.
034
167
PerformanceGuidelines
Condition 034
Headacheina35-year-oldwoman
AIMSOFSTATION
Toassessthecandidate'scommunicationskillsindefiningthenatureofthepatient'sheadache.
Inaddition,knowledgeof typesandcausesofheadacheandthe essentialcomponentsofan
appropriatelyfocusedphysicalexaminationaretested.
ExaminerInstructions
Theexaminerwillhaveinstructedthepatientasfollows:
You are consulting the doctor about headaches. You are aged 35 years and work as a
telecommunications manager. You have not seen this doctor before. Use the following
informationtorespondtothedoctor'senquiries.
OpeningStatement
Iwanttofi ndoutwhati scausi ngmyheadaches.'
Historywithoutprompting:
Youhavebeensufferingfromintermittentheadachesforatleastfiveyears,attacksoccurring
irregularlyeveryfewmonths,lastingforafewdays.TheyaretemporarilyrelievedbyPanadol,
up to six a day at a maximum. You have not previously sought advice. You are concerned
becauseinthepastfewmonths, theheadacheshaveoccurredmoreoften,that is, everyfew
days,andnowlastlonger,formostoftheday.
Inresponsetoquestioning:
Theheadachehasnotincreasedinintensityonlyinfrequencyandduration.
Itfeelslikeatightbandorpressurearoundorontopofyourhead,oradullache,not
pulsating.
Itaffectstheforeheadandboththetemplesandradiatestothebackofthehead.
Itusuallystartsinthemorningandlastsallday,gettingworsebyevening,notinterferingwith
sleep.
ThepainreducesaftertakingPanadol,afteryoureveningmeal,andwithrestandlocalheat.
Onascalefrom1to10,theheadachepainratesabout3to4.
Headachesarenotaccompaniedbynausea,vomiting,visualdisturbance,photophobia,or
associatedwithmenstrualcycle.
Theyarenotrelatedtoposture,exerciseorpositionofheadorneck.
Youcancontinuetoworkandtodohouseholddutiesduringattacks.Youhavelostnotime
fromwork.
ReviewofGeneralHealth
Youconsideryourgeneralhealthtobesatisfactory.
Youdonotoftenseekmedicaladvice.
Youdonotregardyourselfasa'nervous'typeandhavenotnoticedanychangeinyourusual
mood.
Yourmarriageandfamilylifeissatisfactory.
168
034
PerformanceGuidelines
Negativeresponsetoquestionsaboutallbodysystemsexceptasalreadystated.
Menstrualcyclenormal.
OtherSignificantInformation
Youareverybusy,rushthroughwork,domesticandfamilyduties,andhavelessrelaxationtimenow.Youare
aperfectionistbynatureandarefindingitmoredifficulttocopewithalloftheactivitiesofyourgrowingfamily.
You have a dual income and no financial problems. You recentl y saw a TV documentary about a
personwhohadacerebral tumour.
CurrentmedicationPanadol(paracetamol)500mg1-2tabletstorelieveheadachetakennotmorethan
threetimesdaily.Noknowndrugsensitivities.Nonsmoker.Alcoholisusedoccasionallyatweekends.
Appearworriedandtenseanddisplayconcern.Youhadpreviouslybeencomplacentabouttheheadache.
Youarenowseriouslyworriedabouthavingacerebraltumour.Indicatethesiteoftheheadache(forehead,
temples, and occiput). Be prepared to be reassured by the doctor if history and physical examination are
adequate.Ifnot,pressthedoctortohavetestsorbereferredtoaspecialist.Notethatyouarealsoatapeakin
yourfamilyresponsibilities.
EXPECTATIONSOFCANDIDATEPERFORMANCE
Thecandidateshouldassessthepatient'sheadachealongthefollowinglines:
History(seepatientresponses)
Characteristicsofheadaches,includingassessmentofseverity.
Recentchangeinchroniccondition.
Identifyacutepatientconcernthataseriouscausemaybepresent(cerebraltumour).
Recognitionoftypicalcharacteristicsoftensionheadache.
Identificationofrelevantpsychosocialandenvironmentfactors(lifestylestress)
Essentialfeatureswhichcandidatesshouldlookforonphysicalexamination(suppliedfromexamineron
request)
Inspectionofheadandneckandtestingforneckstiffness.
Neurologicalexaminationmaybelimitedbutmustincludeophthalmoscopy.
Cardiovascularexaminationmustincludebloodpressure.
THEEXAMINERSHOULDINFORMTHECANDIDATETHATNOABNORMALITIESAREFOUNDON
PHYSICALEXAMINATION.
Communicationskills
Useoffacilitation,activelisteningandrelevantenquiry,tofullydefinethenatureoftheheadacheandits
associations;
Communicateunderstandingandconcernthatthereisarecentchangeinachronicproblem;and
facilitatedisclosureofrelevantpsychosocialhistoryandworryaboutbraintumour.Diagnosis/Differential
Diagnosis
Tensionheadache(musclecontractionheadache)isthemostlikelydiagnosis.Otherdiagnostic
possibilitiesinclude:
~Migraine.
169
034
Performance Guidelines



~ Cluster headache.
~ Raised intracranial pressure (excluded by history typical of tension headache and
negative examination findings).
~ Cervical spondylosis.
Patient counselling
Headache is of muscle contraction type due to tension and associated with personality and daily pressures of work
and family life. No special investigations are indicated at this stage. Referral to neurologist for opinion is
acceptable. Reassurance should be strongly given with offer to follow up results of this first contact with the doctor.
Investigations are not indicated.
Discussion and education about the role and function of investigations is indicated.
Referral for computed tomography (CT) brain or magnetic resonance imaging (MRI) would detract from overall
performance but is not a key issue especially if a patient requires additional reassurance that the headache is not
due to a cerebral tumour. X-ray skull is not indicated.
KEY ISSUES
Use of communication skills to elicit the most relevant and important points in the history.
Focused physical examination in a patient complaining of longstanding headache.
Confidence in diagnosis of tension headache based on typical history, normal physical examination, patient
concern about a serious cause and lifestyle factors.
Recourse to investigations (CT or MRI) unnecessary at this stage but allowable to diminish patient concern.
Arrange followup to assess therapeutic effect of this initial assessment.
CRITICAL ERRORS
Failure to request blood pressure and ophthalmoscopy findings.
Failure to indicate the most likely cause is tension headache and that a serious cause is most unlikely.
COMMENTARY
This case is deliberately set in a hospital Emergency Department where the time constraint matches the eight minutes
allowed for the candidate to complete the focused tasks set.
The candidate who takes immediate control over the interview by asking a series of direct questions about site,
duration, intensity etc. may successfully reach the diagnosis of tension headache, but is likely to overlook the patient's
recent concern about a serious cause and miss the real cause (lifestyle factors) with two consequences:
referral for unnecessary investigations which also increases the patients anxiety about a serious cause; and
missing the opportunity to convert the diagnostic approach into the appropriate therapy by improving self
understanding by the patient.
The initial response from the candidate should be to facilitate the history given by the patient with an open-ended
approach, listening, and encouraging the patient to tell the whole story including concerns and life situation.
035
170
PerformanceGuidelines


Condi t i on 035
Lethargyina50-year-oldwoman

AIMS OF STATION
To assess the candidate's history-taking skills and diagnostic acumen in a patient with the symptoms and
signsofhypothyroidism.Thecaseisdeliberatelypresentedasanundifferentiatedproblembutthepatient's
initialunpromptedstatementsshouldleadtothecorrectdiagnosticpathwaywithconfirmationofsuspected
hypothyroidismbyafocusedselectivephysicalexamination.
Examiner Instructions
This case requires the patient to reveal the symptoms of hypothyroidism in a slow and hesitant, but
neverthelesspositivemanner,inresponsetoappropriatehistory-taking.
Theexaminerwillhaveinstructedthepatientasfollows:
Youarea50-year-oldwidow.
Openingstatement
' My daught er want s me t o have a check- up because she says I am al ways t i r ed. '
Respondtothedoctor'senquiriesasfollowswi t hout pr ompt i ng.
Youhavenotfeltwelllately(bevagueabouttheduration).
Youfeelweakespeciallyyourarmsandlegs.
Youalsofeellethargic.
Yourdaughtersaysyouarenotinterestedinanything,gotosleepduringthedayandcan'tbebothered
talkingtopeople.
Yourvoicehasbecome'croaky'peoplesayithaschangedoverthepastyear.
Youarealwaysconstipatedandthisseemstobegettingworse.
Yourperiodsstoppedlastyearandwerescantyandirregularforayearbeforethat.
Youhaveputonweight.
Yourresponsesareapathetic,lackingexpression.
Youareslowtoreactwithacroaky,husky,thickvoiceandpoormemory.
Yourespondslowlytodoctor'squestions,butappropriatequestionsevokecorrectresponses.
Donotbeevasive.Showpaucityofbodymovements.
Inresponsetootherquestions:
Nohistoryofthyroidsurgery.
Youfeelcoldallthetime.
Yourhairhasbecomethinner.
Youfindithardtoconcentrate.
Yourmemoryisnotgood.
Youareabletomanageyourownpersonalcarebuteverythingisaneffortandtakeslonger.
Yougetconstipatedifyoudon'ttakeColoxylregularly.
035
171




Performance Guidelines
Your joints feel stiff and the muscles are sore.
You wake up early in the morning and can't get back to sleep.
Answer in the negative to any other questions about your health except to indicate that you feel you are gradually
going downhill.
If asked, state that you have just come to stay with your daughter because you were unable to carry on living alone.
Past medical history and family history nothing of note. Parents died from old age.
Current medication
'Tonic' obtained from Pharmacist by daughter.
Aspirin irregularly for the rheumatism.
Dioctyl sodium 120mg (Coloxyl) 'for my bowel' 1 or 2 tablets daily, taken for about a year.
Physical examination the examiner should give the findings for selective and specifically requested
components of the physical examination.
Appearance as illustrated
~ Looks tired and dull. Expressionless face, coarse features and skin.
~ Overweight BMI 29 kg/m
2
~ Pulse rate 56/min regular, blood pressure 130/70 mmHg
~ Thyroid not palpable
~ Skin dry sparse axillary hair
~ Cold hands and feet
~ Power and tone reduced in arms and legs
~ Reflexes sluggish with delayed ankle jerks
At six minutes the examiner will ask the candidate for the diagnosis/differential diagnosis and proposed investigation.

EXPECTATIONS OF CANDIDATE PERFORMANCE
Diagnosis/differential diagnosis
Candidate's response to examiner's request for the diagnosis: candidate should strongly suspect
hypothyroidism as indicated by pattern recognition and from the patient's symptoms and signs. Other possibilities
such as depression, anaemia, early dementia may be mentioned but should be considered unlikely.
Investigations
Must request thyroid function tests and full blood examination.
Key Issues
History
Choice and sequence of examination
Diagnosis/differential diagnosis
Appropriate investigations

172
035
Performance
Guidelines


CRITICALERROR
Failuretoconsiderhypothyroidismindifferentialdiagnosis.
COMMENTARY
Spontaneous atrophic hypothyroidism often gives gradually progressive symptoms as ir this
case.Thepathologyisdestructivelymphoidinfiltrationofthethyroidglandleadingtcatrophy
withnovisibleenlargement,orinsomepatients,associatedwithgoitre.
Diagnosisissuspectedbytheconstellationofsymptomsandsignsasexhibitedinthispatient
andwouldbeconfirmedbyelevationofserumTSHwithloweredT4levels.Theconditionisan
organ-specificimmunedisorderandrespondswelltothyroxinetreatment,beginningwithalow
dose(50ugdaily)andincreasingslowlytothedoserequiredtorestoreTSHtonormal.

036
173
Performance Guidelines



Condition 036
Syncope in a 52-year-old man

AIMS OF STATION
To assess the candidate's ability to take a focused history regarding transient loss of consciousness with
possible causes in mind. The candidate should also know the essential components of a selective physical
examination which should identify a probable cause and be able to specify the investigations which would
confirm the diagnosis
Examiner Instructions
The examiner will have instructed the patient as follows:
You are a 52-year-old technician who is consulting a doctor about recent transient loss of consciousness.
Opening statement
I was playing tennis yesterday when I suddenly blacked out. My friends thought I was dead!'
If given the opportunity by the doctor, follow with this information:
'/ was enjoying my usual Sunday morning game of tennis. It was a hot day. I had been serving
and the game was pretty fast when I suddenly blacked out. There was no warning and I must
have 'come to' pretty quickly because my friends told me they were about to start pushing on my
chest. They couldn't tell whether I was breathing or not and they said they couldn't feel my pulse.
Anyway I woke up and felt that nothing had happened except for this graze on my elbow. I
decided not to play on although I felt ok'.
Provide the following information in response to specific, appropriate questioning:
You feel well today.
This was the first such attack. You have not had any previous fainting or dizzy spells
No convulsions or fitting from your friend's description. Before or after the attack you had no palpitations or
awareness of heart beating abnormally, no vertigo, no headache, no disturbance of your vision, no
numbness or tingling.
No one said anything about your colour.
You had no loss of control of your bladder or bowel function during the attack.
You get short of breath whilst playing and this has been more noticeable lately. You have attributed this to
your age. You don't get short of breath lying down or at night.
You also have some 'muscle soreness' in your chest, which sometimes comes on when you are playing, this
has been noticed over recent months, but it is not severe and goes away when you stop playing tennis
between games. This occurs if a game is strenuous or prolonged and is a tight feeling.
You don't feel it anywhere else, just across your chest.
There has been no swelling of ankles.
Negative responses to all questions reviewing body systems.
174
036
Performance Guidelines



Other significant information
Your last cholesterol check was three or four years ago when you last saw a doctor. Youwere
told that it was in the 'high normal' range and advised to reduce intake of fatty foods.
Blood pressure always normal.
No regular medication.
If asked about your past history, family history, habits, social history or other information,
respond as for yourself.
You are a mature middle-aged person without previous health concerns. Because you were
unaware of the details of yesterday's events (except for the graze on your elbow) and
because you feel so well, you are not unduly concerned.
Do not reveal the chest soreness or the excessive shortness of breath on exertion, without
proper enquiry from the doctor.
After obtaining the results of the physical examination the doctor should give you an opinion
as to the cause of your symptoms. Accept what the doctor says.

EXPECTATIONS OF CANDIDATE PERFORMANCE
History
Eliciting the triad of symptoms of syncope, exertional dyspnoea and angina which raise a
high index of suspicion of aortic stenosis.
Excluding symptoms suggestive of other causes, particularly epilepsy (see differential
diagnosis).

Physical examination (provide these on request)
Cardiovascular examination
pulse 70/min regular
blood pressure 118/88 mmHg lying and standing
jugular venous pressure normal
auscultation of neck - systolic bruit (transmitted) over both carotid
arteries at base of neck
loudest on right.
heart
~prominent left ventricular impulse
~apex beat not displaced
~ejection systolic murmur (3/6) best heard over aortic area
~radiates to neck and apex
Neurological examination normal
There are no other abnormal physical findings.
The examiner should limit the candidate's requests for physical examination findings to the
cardiovascular and central nervous systems (the latter is normal).
After six minutes ask the candidate the most likely diagnosis, then direct the candidate to
discuss these with patient.
03
PerformanceGuidelines



Diagnosis(opiniontopatient)
Thatthelossofconsciousnesswasmostlikelyduetoanabnormalityinoneoftheheartvalves
which requires investigation by ECG and echocardiogram (X-ray chest and full blood
examination are addtionally acceptable), and referral to a cardiologist (may use term aortic
stenosiswithfurtherexplanation).

Historymustelicitdetailsofsyncope,exertionaldyspnoeaandchestdiscomfort.
Examinationmustrequestpulse,bloodpressure,presenceofcarotidandcardiacmurmurs.
ChoiceofinvestigationsmustincludeECG,andechocardiogram
Diagnosismustrecognisethatthispatient'ssyncopehasaseriousunderlyingcause.
Syncopal episodes are common, accounting for 3-5% of attendances at Emergency
Departmentsandaffecting15-25%ofthepopulationoverany10yearperiod.Theprevalenceof
syncope increases with age and can cause significant morbidity in the elderly. An attack of
syncopeisassociatedwithmajormorbiditysuchasfracturesandmotorvehicleaccidentsin6%
ofcases,andminorinjuriessuchaslacerationsandbruisingoccurinaboutathirdofcases.Itis
essential to distinguish syncope from seizures and syncope caused by benign causes from
syncope caused by serious underlying illness. This middle-aged patient with syncope evinces
thetriadofsymptomsclassicalofaorticstenosis.

Failuretoaskaboutcardiovascularsymptoms.
Failuretorequestexaminationfindingsforbothcarotidandcardiacbruits.

Causes of syncope include neurally-mediated syndromes (such as vasovagal/ vasodepressor
syncope and carotid sinus syncope), orthostatic syncope (including volume depletion, drugs,
autonomicfailuresyndromes),cardiacarrhythmias(bradycardiasandtachycardias),structural
heartdiseaseandcerebrovasculardisease.
Takingacarefulhistory,includinganeyewitnessaccount,iscriticallyimportantinsyncopeand
canpreventinappropriateandcostlyinvestigations.Thehistoryinthismiddle-agedpatientwith
syncope has the important feature of coming on with exertion and against a background of
cardiovascular symptoms of chest pain and shortness of breath pointing towards a cardiac
cause.Theabsenceofpalpitationsisagainstthediagnosisofcardiacarrhythmiabutthiscould
stillbeapossibilityonthehistory.Thecandidateisexpectedtouseappropriatequestioningto
try to exclude epilepsy and other neurological causes. Carotid sinus syncope and
vasovagal/vasodepressor syncope may be considered as differential diagnoses but other
features of these conditions are not apparent (for example, relationship with head turning or
warningsymptoms).
175
036
176
PerformanceGuidelines



The next step is a careful cardiovascular examination. The candidate is expected to look for evidence of
structural heart disease, in particular for features of aortic stenosis. The examination requested should be
systematic,startingwiththeperipheralpulseandbloodpressure,movingontoanexaminationofthecarotid
pulses. The nature of the cardiac apex beat, and a requestfor details on the heart sounds and the cardiac
murmur,itscharacteristics,siteandradiationarekeyfindingsthathelptorefinethediagnosticprocess.The
most important investigations to perform in this patient are an ECG and an echocardiogram. Referral to a
cardiologistwouldbeanappropriatesteptotake.
A lack of understanding of the condition would be exhibited if a candidate requests a whole range of
investigationssuchas:7woul d l i ke to perform ful l bl ood count, el ectrol ytes, blood gl ucose, caroti d
Doppl erstudi es,el ectroencephal ogram,Hotterorl oopmoni tori ngand coronaryangi ography.'

177
037
Performance Guidelines



Condition 037
A painful penile rash in a 23-year-old man

AIMS OF STATION
To assess ability to diagnose and manage genital herpes. This patient has penile herpes simplex. The
patient also needs to be assessed in terms of other possible sexually transmissible infections.

EXAMINER INSTRUCTIONS
The examiner will have instructed the patient as follows:
Opening statement
I've got a problem with my penis, doctor. '
You are a carpenter. Volunteer the following history without prompting:
You have had penile pain for two days.
It started as intermittent tingling, but is now constant.
You have noticed today a lumpy penile rash with blisters.
In response to questions from the doctor indicate that:
You have had no serious past illnesses and are on no medications.
You have no allergies.
There is no history of mental illness.
Sexual history you have no steady partner. You are heterosexual and you last had sex with a woman
you met at a disco a week ago.
Do not volunteer information about sexual behaviour unless asked specifically
Questions to be asked of the candidate unless already covered:
What is wrong with me?'
'What is the cause?'
'Can I pass it on?'
'How can it be treated?'
'Will I be cured?'

EXPECTATIONS OF CANDIDATE PERFORMANCE
The candidate is expected to take an appropriate sexual history and to be able to identify the vesicular
penile rash as most likely due to genital herpes simplex and to confirm diagnosis by viral testing.
Testing for associated sexually transmissible infections as part of the differential diagnosis should include
Venereal Disease Research Laboratory (syphilis), chlamydial, gonococcal and HIV testing.
Patient counselling and education is important to reduce risk of transmission; and management advice
should be:
Confirm diagnosis with virological testing.
Explain the Cause to the patient.
178
037
Performance Guidelines
Assess the risk of possible exposure to other sexually transmissible infections, for which the patient should be
screened.
Counselling to reduce risk of transmission of any sexually transmissible infections is essential.
Antimicrobial therapy: reduces the length of illness and may decrease virus transmission. First clinical episodes of
genital herpes, when severe, are treated with a five day course of oral aciclovir, or similar agent.
Topical treatments can help symptomatic management: for example, topical povidone-iodine; lignocaine.
Supportive treatment: rest, salt baths, ice packs, analgesics, wear loose clothing.
Further investigations
~ The risk of other sexually transmissible infections, including HIV infection needs to be investigated.
~ This may involve swabbing oral, genital and anal areas for Chlamydia and other infections.
All testing needs to be done with informed consent.

KEY ISSUES
Ability to identify rash as herpes simplex.
Ability to take a sexual history and investigate possible concomitant sexually transmissible infections.
Counselling to reduce risk of further transmission of herpes simplex virus. Ability to treat herpes infection.

CRITICAL ERROR
Failure to assess for other sexually transmissible infections.

COMMENTARY
Herpes Simplex Virus (HSV) is one of a family of herpes viruses, which includes HSV 1 and HSV 2, varicella zoster
virus (VZV), Epstein-Barr virus (EBV), cytomegalovirus (CMV) and human herpes virus 6. HSV 1 and HSV 2 cause
patterns of disease which differ clinically and epidemiologically, but share some characteristics. HSV 1, HSV 2 and
herpes zoster all establish permanent latency in sensory nerve ganglia following the primary infection.
HSV 1 is more common and is usually acquired during childhood presenting most commonly as acute
gingivostomatitis, but can occur anywhere on the skin. In the eye. keratoconjunctivitis is a feared infection because it
can cause scarring of the cornea and loss of vision.
HSV 2 is largely associated with genital infection and is most common in young, sexually active adults. The risk of
infection increases with numbers of sexual partners. Transmission is by vaginal, oral or anal sex. Condoms reduce the
risk of transmission. The infection appears at the site of virus entry, on the glans penis or penile shaft in men, or on the
vulva or vaginal mucosa in women. Perianal and rectal lesions can develop as a result of anal intercourse. Primary
herpes simplex infections have an incubation period of 3-6 days but this may be longer.
179
L
037
Performance Guidelines



During the first attack, there may be a tingling and burning over the affected area.
The rash is vesicular, often occurring in crops,
After a day, the vesicles leave small red, painful ulcers. These heal within a few days.
There may be associated fever, myalgia and local lymphadenopathy.
The first (primary) attack lasts around two weeks.
Recurrence occurs in about 50%and may be associated with shooting pains in the buttocks and legs.
Recurrences often occur at times of stress and tiredness. Systemic manifestations are uncommon.
Recurrences are also more frequent in presence of immunosuppression including HIV infection.
HSV infection may be associated with other sexually transmissible infections.
038
180
PerformanceGuidelines
Condition 038
Primaryamenorrhoeainan18-year-oldwoman
AIMSOFSTATION
Toassessthecandidate'sabilitytodefinethecauseofprimaryamenorrhoeainayoungwomanwhohasgone
throughanapparentlynormalpuberty

EXAMINERINSTRUCTIONS
Theexaminerwillhaveinstructedthepatientasfollows:
Youhavecometoseethedoctorbecauseyouareworriedthatyouhavenotyethadaperiod.Youare18years
old.
Listofappropriateanswersinresponsetolikelyquestions
Evidenceofpubertalchanges:
~Youhadagrowthspurtthreeyearsago.
~Breastchangesstartedthreeyearsago.
~Pubichairbegantodevelopthreeyearsago.
Youarenowofsimilarheighttopeersfromschool.
Youhavenohistoryofabdominalorpelvicpain.
Yourmotherhadherfirstperiodattheageof17years.
Yoursister(aged14)hasnotyetstartedtohavebreastdevelopment.
Youhavehadnooperationsorsignificantillnesses.
Youhaveneverbeensexuallyactive.
Questionstoaskunlessalreadycovered:
' Whyhasn' tmyperi odstartedyet?'
' Al l myfri endsatschool startedyearsago.Wi l l Ieverhaveperi ods?'
' Wi l l Ibeabl etohaveababy?'
Examinationfindingsfromexaminer
Theseshouldbegiventothecandidateonrequestforspecificcomponentsoftheexamination.
Herweightis48kgandshehasnormalheightforweight.
BreastsshowTannerstage5ofpuberty.Pubicandaxillaryhairalsoshowstage5.
Bloodpressure120/80mmHg.
Vulvalinspectionnormalappearance.Hymenintact,butapparentlyperforate.Lower vaginaabovehymen
appearsnormal.
EXPECTATIONSOFCANDIDATEPERFORMANCE
Pelvicexaminationpervaginumshouldnotbedoneandshouldnotbespecifiedby candidate.
03
181
Performance Guidelines



Investigations required
Ultrasound examination (abdominal-pelvic ultrasound, not vaginal): to check the development of the uterus and
vagina and to confirm that these are normal
Hormone tests: should order follicular stimulating hormone (FSH), luteinising hormone (LH), prolactin, and
oestradiol levels.
Chromosome analysis is not necessary if uterus is shown to be normal and above hormonal levels are normal.
Advice to patient (the candidate should convey the substance of what follows to the patient):
Reassurance that this is likely to be just a physiological delay in the first period as all else appears normal on
examination. The investigations which have been arranged should confirm this. The first period usually occurs
two years after the first breast development, but can be delayed for three years or longer as normal variation in
menarche.
She will be reviewed after investigations have been established as normal and again in 12 months time. If a
period has still not occurred, estimation of oestradiol levels should be repeated.

Ability to define the most likely cause of primary amenorrhoea (delayed menarche).
Ability to arrange the appropriate investigations.

CRITICAL ERRORS
Inadequate history to evaluate current pubertal status.
Performance of pelvic vaginal examination as she is virginal. Requesting that pelvic examination should be done
would be a significant and potentially failing error.
Failure to order abdominal ultrasound. Pelvic (vaginal) ultrasound is also inappropriate
Failure to order hormonal analyses of FSH, prolactin and oestradiol.

This case illustrates the situation where a slightly delayed menarche can be a normal situation particularly where
there is a familial trait. It is therefore essential to obtain an appropriate history, both in regard to her own history and
that of other family members. Noninvasive investigations will enable this patient to be reassured in the presence of
other pubertal changes.
Common problems likely with candidate performance are:
Failure to recognise that an apparently normal puberty is occurring. Therefore the cause of the primary
amenorrhoea is likely to be just a slight delay in the first period with everything else being normal. Other less
likely possible causes are obstruction to the outflow of blood from the uterus to the exterior by an intact hymen or
vaginal septum or an absence of development of the uterus. Ultrasound examination is essential to make these
latter diagnoses.
Failure to reassure the patient and failure to advise review in 12 months time if a period has still not occurred.
182
039
Performance Guidelines



Condition 039
A skin lesion on the cheek of a 50-year-old man
AIMS OF STATION
To assess the candidate's ability to diagnose a facial skin lesion suspicious of basal cell carcinoma
EXAMINER INSTRUCTIONS
The examiner will have instructed the patient as follows:
You are a builder aged 50 years. Most of your working life has been spent outdoors.
You have had a firm nodule on your cheek for a year or so, which has slowly increased in size. It has
recently developed an ulcer in the centre of it, and has bled a little. You think this may have followed you
picking at it. You are otherwise healthy and well with no serious past illnesses.
You have no allergies and are on no medications. Otherwise answer as for yourself.
Opening statement:
'I've had this thing on my cheek for about a year doctor. '
EXPECTATIONS OF CANDIDATE PERFORMANCE
The candidate should recognise from the history and physical findings that this is a 'suspicious' lesion which
requires excision, and should advise the patient accordingly.
KEY ISSUES
Ability to present a focused case summary.
Ability to identify and manage a 'suspicious' skin lesion, in this case basal cell carcinoma.
CRITICAL ERROR
Failure to diagnose that this is a 'suspicious' skin lesion needing excision.
COMMENTARY
This scenario has been arranged in the format of a summary case presentation to the examiner of a suspicious
skin lesion, followed by advice to the patient about treatment. It tests accuracy of observation and ability to
summarise the problem concisely. The diagnosis should be straightforward. The most likely diagnosis is an
ulcerating basal cell cancer and the most appropriate treatment is local excision with an adequate margin.
Differentiating a basal cell cancer from a squamous lesion or other skin malignancies is less important than
identifying the lesion as requiring appropriate histologic diagnosis after adequate excision.
039
183
Performance Guidelines



Common clinical features of basal cell carcinoma (BCC) are:
~ BCC is the most common skin cancer comprising 80%of skin cancers, occurring usually in people older than
35 years, more frequently in males and in fair-skinned people,
~ BCC occurs most commonly on sun-exposed areas: face (mainly), neck, upper trunk, limbs (10%); but can
occur on covered areas as well. ~ Ulceration produces the characteristic 'rodent' ulcer as illustrated by this
example
~ BCCs grow slowly over years, and can occur in various forms: nodular, ulcerated, morphoeic, cystic or
pigmented, - They do not metastasise, but local spread can cause problems with surrounding
structures as they can spread deeply around nose, eye, or ear ('nothing burrows like
a basal cell').
Patients need to be educated about avoiding direct sunlight when the sun is at its strongest: wear a broad
rimmed hat; wear a shirt; and use sun block cream when exposed.
184
040
Performance Guidelines


Condition 040
A pigmented mole on the trunk of a 30-year-old woman
AIMS OF STATION
To assess the candidate's ability to diagnose a pigmented focal skin lesion and to identify a 'suspicious' lesion.
EXAMINER INSTRUCTIONS
This patient has a 'suspicious' pigmented skin lesion: a melanocytic naevus suggestive of a dysplastic naevus.
Excluding melanoma by excision is critical to successful case management.
The examiner will have instructed the patient as follows:
You are a 30-year-old schoolteacher.
You have had a dark spot on your back for many years. You think you were born with it. The spot has
become darker over the last few months so you thought you would have it checked out. You have a few
other dark spots, but they have not changed.
You are otherwise healthy and well, and have had no serious past illnesses. You have no allergies, and are
on no medications.
Opening statement:
'It seems to have changed and become more itchy lately, doctor. '
EXPECTATIONS OF CANDIDATE PERFORMANCE
The presence of change in appearance and irritative symptoms in a previously stable pigmented naevus,
should raise concerns about malignant change, especially in a large lesion with appearances suggesting a
dysplastic naevus. The candidate should refer the patient for excisional biopsy.
KEY ISSUES
Ability to identify a 'suspicious' pigmented lesion, possibly at risk of malignant melanoma.
Ability to manage pigmented skin lesions appropriately.
CRITICAL ERROR
Failure to suspect malignant potential.
COMMENTARY
Most pigmented skin lesions are benign. About one third of melanomas arise in pre-existing naevi. Removal of
such naevi is important for melanoma prevention. The incidence of melanoma is rising in Australia and around
the world. Incidence is higher in fair-skinned people; appears to be related to brief intense sunlight exposure as
well as effect of chronic exposure.
185
040
Performance Guidelines



Most people have an average of 5-10 benign melanocytic naevi. Multiple dysplastic naevi carry a greater risk of
malignant change.
Clinical features of 'suspicious' dysplastic melanocytic naevi include:
Large (> 5 mm) irregular moles, such as this patient exhibits, appearing usually on the trunks of young adults.
Lesions with irregular, ill-defined borders, irregular pigmentation, background redness, variable colours brown,
black, tan, pink.
Most are stable and do not lead to melanoma, but excision is indicated if any diagnostic concerns.
Dysplastic naevus syndrome is diagnosed because of the presence of multiple, large, irregular pigmented naevi,
mainly on the trunk. It is important to exclude malignant melanoma
Signs indicative of possible malignant melanoma include:
any change in size of a presenting lesion (lateral spread or thickening);
change in shape;
change in colour (brown, blue, black, red, white and combinations of these colours);
change in surface;
change in the border;
bleeding or ulceration; and
other symptoms (itching).
Development of satellite nodules and lymph node involvement are late signs.
Differential diagnosis of pigmented skin lesions includes:
haemangioma (thrombosed);
dermatofibroma (sclerosing haemangioma);
pigmented seborrhoeic keratosis;
pigmented basal cell carcinoma;
junctional and compound benign melanocytic naevi;
blue naevi;
dysplastic naevi; and
lentigines.
Management:
In this case, the solitary dysplastic naevus may have no significant malignant potential at this stage. However,
because of the size of the lesion and the patient's concern, this lesion should be excised.
Suspicious pigmented lesions should have complete excisional biopsy, and not be treated by cryotherapy.
041
186
PerformanceGuidelines
Condition 041
Anitchyrashonthehandsofa19-year-oldwoman
AIMSOFSTATION
To assess the candidate's ability to diagnose, confirm and treat scabies, and to prevent
recurrence.
ExaminerInstructions:
Theexaminerwillhaveinstructedthepatientasfollows:Opening
statement
' Iwanttogetridofthisrashonmyhands.'
Follow with ' It started about a week ago and I can t stop scratching my hands
becauseoftheitch.'
Inresponsetoquestionsthedoctormayask:
~Youhavenothadanythinglikethisbefore,
~Norashoritchinesselsewhereonyourbody.
~Theitchisintenseandmadeworsebywarmingyourhands,aswhenwashingupin
hotwaterorbathingorshowering.
~Theitchisworstatnightandinterfereswithsleep.
~Yourhandshavenotbeenincontactwithanyirritants,chemicalsorplants.
~Yourgeneralhealthisexcellent.
~Nopasthistoryofanyseriousillness.
~Noknownallergies.
~Nohistoryofmentalorbehaviouraldisturbance.
~Norecenttravelawayfromhome.
~Nomedicationexceptoralcontraception.
~Yourboyfriendwithwhomyouaresexuallyactivehashadasimilarrashthoughnotasbad
andhehasnotsoughtmedicaladviceaboutit.
Scratchandrubthebacksofyourknucklesandbetweenthebasesofyourfingers.Answerthe
doctor's questions in a straightforward manner including about the relationship with your
boyfriend.Donotrevealthisspontaneously.
EXPECTATIONSOFCANDIDATEPERFORMANCE
Approachtopatient.Displayinterestandintentiontodealeffectivelywiththecondition.Be
nonjudgmental about possible sexual transmission of scabies from boyfriend. Provide
reassurancethatconditionissimplycuredandnotserious.Compliancewiththewholeofthe
treatmentregimenshouldbeobtained.
History.Identifysiteandseverityofitchandquestionaboutsexualactivityafterotherpossible
sourceshavebeenexcluded.
Confirmationofdiagnosis.Thecandidatemaydiagnosescabiesfromillustrationandhistory
asgivenabove,butshouldadvisethepatientthatdiagnosismustbeconfirmedbytakingskin
scrapingsfromthelesionsformicroscopy.
041
187
PerformanceGuidelines
Exami ner shoul d i ntervene at thi s poi nt by stati ng ' Pl ease assume that the ski n scrapi ngs are
posi ti ve for scabi es, and advi se the pati ent accordi ngl y. 'Theexaminershouldonlystatethisifthe
candidatehasmentionedtheneedforskinscrapingstodiagnosescabies.
Patienteducationandcounselling.Inthiscase,theconditionistransmittedbyclosecontactduringsexual
activity. Description of the scabies mite is expected with reassurance that the condition is not serious,
although very uncomfortable, and is readily treatable. Patient should advise boyfriend to seek medical
advice.
Management.
~Applicationofpermethrincreamorlotion5%(Lyclear)orbenzylbenzoateemulsion25%(Ascabiol)to
entirebodyfromjawlinedownincludingnails,flexuresandgenitals.Leavepermethrincreamorlotion
overnightthenwashoffthoroughly,butbenzylbenzoatelotionshouldbeleftonfor24hours.
~Avoidhotbathsorscrubbingbeforeapplication
~Treathouseholdcontactsevenifnonsymptomatic
~Washclothingandbedclothesinhotwaterandexposetosuntodry
~Repeattreatmentinoneweekifinfestationisconsideredtobesevere
~Avoidintimatecontactwithboyfrienduntilhehasalsobeenproperlytreated
KeyIssues
ApproachtopatientAbilitytoestablishsatisfactoryrelationshipwithpatienttoachievecomplianceand
cooperationofpatienttogetboyfriendtoseektreatment.
HistoryAbilitytotakeanappropriatehistoryincludingsiteandseverityofpruritusandsexualpartneras
sourceofinfection.
DiagnosisShouldadvisemicroscopyofskinscrapingstofacilitatediagnosis.
ManagementProvideadequateadviceforpropertreatmentandadvisethepatienttoavoidintimate
contactwithboyfrienduntilhehasbeentreated
CriticalError
Failuretosuspectscabiesortotakeactiontoconfirmdiagnosis.
Commentary
Scabies is a highly contagious infestation which is spread through close contact including sexual contact.
Scabies can affect entire households, especially if overcrowded, although this is now uncommon. It is
characterisedbywidespreadinflammatorypapulesandseverepruritusanditcanbeendemicamongschool
childrenandinstitutionalisedolderpatients.
Thefemalescabiesmite(illustratedbelow)burrowsjustbeneaththeskininordertolayhereggsandthendies.
Theeggshatchintomiteswhichspreadoutacrosstheskinandliveforabout30days.Amiteantigeninthe
excretainducesahypersensitivityrash.
CONDITION041.FIGURE2. CONDITION041.FIGURE3.
Scabiesmite(Sarcoptesscabi ei ) Penilescabies
Clinicalfeaturesincludeintenseitching,worseatnightandwhenhandsandbodyarewarm(for
example,afterashower),withanerythematouspapularrashusuallyonhandsandwrists.The
rashalsocanoccurinwebspaces,onmalegenitaliaasillustrated,onelbows,axillae,feetand
ankles,ornipplesoffemales.Diagnosisisconfirmedbymicroscopyofskinscrapings.
188
Condition 042
Red painful dry hands in a 30-year-old bricklayer
AIMS OF STATION
To assess the candidate's ability to diagnose occupational dermatitis and advise an initial management plan.
This patient has occupational contact dermatitis secondary to concrete exposure.
After 6 minutes, if the candidate has not identified the condition as contact dermatitis ask
the questions:
What is the likely cause of the condition?'
'How would you manage this condition?'
The examiner will have instructed the patient as follows:
You are aged 30 years, and have been working as a bricklayer/contractor for about a year.
Opening statement
'I've got problems with this rash on my hands. '
Following without prompting:
Your hands have been itchy and dry for some months now, and are getting worse. The rash is on no other part
of the body. You are otherwise healthy and well, with no serious past illnesses. You have no allergies. You are
on no medications.
State if questioned about the relationship of rash to work: the rash definitely improved significantly after a
holiday from work.
Your brother has skin problems but you are not sure what type.
189
190
042
PerformanceGuidelines



EXPECTATIONSOFCANDIDATEPERFORMANCE
Historymustelicitoccupationalhistory.
Diagnosisshouldsuspectallergiccontactdermatitisanditscauseinthispatientfromthe
historyandfromthephysicalfindingsasillustrated,whichinvolvepalmaranddorsalsurfaces
ofbothhands.Confirmationofdiagnosisisbypatchtestingbydermatologist(notessential).
Managementexplaintothepatientthattherashwillpersistaslongasthereisexposureto
cementalthoughitsseveritymaybereducedbythefollowinginitialmanagement:
~Washonlywithwaterandavoidsoap.
~Patdryafterwashing
~Applytopicalcorticosteroidcreamtogaininitialcontrol.Oralprednisoloneisreservedfor
severecases.
~Oralantibioticsmayberequiredforsecondaryinfectioninseverecases
~Considerusingemollientagentsforfutureprevention.
~Forcementdermatitis,specificmeasuresinvolveavoidingcontactwithwetcement:using
barrier creams before putting on gloves (do not use barrier creams on damaged skin);
usingprotectivegloveswhenworkingandwashinghandsafterbeingexposedtocement.
KEYISSUES
Abilitytoidentifythetypeandcauseofthedermatitis.
Abilitytomanageoccupationalcontactdermatitis.
Considerationofcementasmostlikelycauseofdermatitis.
CRITICALERROR
Failuretosuspectcausalworkassociationindiagnosis.
COMMENTARY
Allergic contact dermatitis is due to a delayed hypersensitivity reaction. While physical
appearanceoftheskincanbesimilartootherformsofdermatitis,rashsiteandexposurehistory
arecriticalfordiagnosis,managementandprevention.
Trigger factors only affect some people. Common trigger factors include cosmetic ingredients
including perfumes and preservatives, topical antibiotics, topical anaesthetics, topical
antihistamines, plants (rhus. grevillea, primula, poison ivy), metal salts (nickel sulphate,
chromtes as occur in cement and concrete), dyes, rubber/latex, epoxy resins, glues,
acrylates,coral.
Incementdermatitis,individualscanbecomesensitisedtochromatesaltsatanytime,evenafter
workingwithcementformanyyears.
043
Performance Guidelines
Condition 043
Swelling of both ankles in a 53-year-old woman
AIMS OF STATION
To assess the candidate's ability to take a detailed history concerning swelling of the ankles,
knowledge of possible causes and the components of the physical examination necessary to
reach a firm provisional diagnosis.
The examiner will have instructed the patient as follows:
You are a 53-year-old clerical worker and are consulting the doctor about swelling of your
ankles. The doctor will take a history about this complaint but will not examine you
Opening statement
'I have come to see you about swelling of my ankles. '
Provide the following without prompting
'Over the past eight weeks my ankles have been swelling. I usually notice this is worse at the
end of each day. They have mostly gone down by the mornings. My ankles have never swelled
up before'.
Provide the following in answer to appropriate questioning
The swelling is the same in both lower legs, there is no discolouration of the skin.
There has been no pain in your legs.
If asked about shortness of breath: you believe that you are not as fit as you used to be
because you become noticeably breathless when walking up stairs or hurrying. This passes
when you rest.
Regarding exercise: you gave up playing tennis about a year ago because you became very
breathless for a short time after a rally and also you felt exhausted afterwards.
Recurring palpitations: for some years you have noticed that your heart seems to 'bounce
around in your chest' particularly when you are going off to sleep (thumps and misses beats).
Your heart also seems to race after any strenuous exertion although this settles down after a
few minutes. You have not counted your pulse rate but you are sure that it is faster than
normal. You have the feeling that it may not be regular at times, but you find it hard to be sure.
No associated dizziness or blackouts.
No suggestion of a fever, no chills or shakes.
No cough or blood in sputum.
You have not had any recent chest pain with or without exercise, you may comment that this
is why you haven't worried about the other symptoms. If asked about chest pain in the past
say ' Four years ago I had a bad pain in the centre of my chest. I was on holidays at the time.
The pain lasted about two hours and I felt unwell for a few days afterwards. '
You sleep well, lying flat in bed: you do not snore.
191
192
Review of general health
You consider yourself to be in good health. You have never suffered any serious ill health.
You have not had a medical check-up recently. 'After all. my father was 90 when he died'.
If asked other specific questions, reply in the negative. You have never had any kidney
problems (for example, blood in urine) or liver disease (jaundice).
Review of relevant systems
Positive responses are confined to the cardiovascular system.
In particular, no gastrointestinal symptoms including no rectal bleeding.
Other significant information
You are very busy at work.
You work for a large legal firm as a legal secretary.
The only exercise you have these days is when gardening and this does not cause any
problems, unless you are digging for more than a short time, then you get 'puffed'
You have noticed this over the past six months.
Patient profile
You are married.
Your spouse is well.
You have three married children.
You smoked 20 cigarettes a day from age of 18 years and stopped a year ago.
You drink three glasses of wine daily.
You are not taking any medication.
You eat a normal, well-balanced diet.
Famil y history
Mother died aged 77 years (stroke).
Father died aged 90 years.
No brothers or sisters.
Past medical history
No serious illnesses.
No operations.
No history suggestive of rheumatic fever.

Blood pressure has been checked several times in recent years and was always normal.
Other instructions
Appear calm and not unduly concerned about your swollen ankles.
You have attributed them to your age.

Be cooperative, but do not disclose all of the cardiovascular symptoms without facilitation,
prompting and appropriate questioning by the doctor, as indicated above.
You are not worried about heart trouble because you no longer smoke and, apart from the
short episode 4 years ago, do not have chest pain.
You have never suspected that your various symptoms could be connected and would not
have attended without the insistence of your spouse.
043
193
PerformanceGuidelines

EXPECTATIONSOFCANDIDATEPERFORMANCE
History
Thisshouldincludeareasonablenumberofquestionsdetailedinthepatient'sadviceabove.
Some questions out of each section should be included, but clearly time limitations will
influencethechoiceandnumber
Thehistorymustatleastcoverkeyquestionsrelatingtopossiblecardiac,hepaticandrenal
causesfortheoedema.
Venous thrombosis, causing inferior venacaval obstruction or bilateral lower limb deep
venousthrombosisisunlikelybutneedstobeconsidered.
Possiblediagnosisgiventopatientafterhistorymustincludecardiacfailureasthemostlikely
condition.
Other potential causes could also include hepatic and renal disease (consider cirrhosis,
nephroticsyndromeormalignancyasmostunlikelycausesinthispatient).
If,afterfiveminutesthecandidatehasnotstartedtodiscusswiththeexaminersomeofthe
likelycausesforthesymptoms,encouragethecandidatetodoso
Afterfiveminutes,ifthecandidatehasnotalreadydoneso,instructthecandidatetotelthe
patienttheworkingdiagnosis,andthenasktheexaminerforphysicalfindingstoconfirmthis.

Examination
The examiner is not required to provide specific examination findings but should
encouragethecandidatetorelatetheexaminationfindingssoughttothepreviouslystated
diagnosticpossibilities.
Theseshouldinclude:
~temperature;
~pulserateandrhythm;
~bloodpressure;
~jugularvenouspulseandpressure;
~mucousmembranes;
~cardiacexamination(apexbeatandauscultation);
~respiratoryexamination(anyreferencetoeffusion,adventitioussoundsorrub
acceptable);
~liver,spleen
~inguinalregionandlowerlimbs(symmetryofoedema,discolouration,tenderness,heat);
and
~urinalysismustberequestedorcomeupsometimeintheassessment.
Candidatesarenotexpectedtoindicatetheinvestigationsrequiredinthisstation,although
candidatesmayindicatethetestsrequiredtoconfirmtheproposeddiagnosis.
043
Performance Guidelines



KEY ISSUES
Ability to take an appropriate history.
Ability to explain to the patient why she has swollen ankles and shortness of breath.
Ability to provide a sensible differential diagnosis.
Ability to state precisely what would be sought on physical examination and why.

This station assesses the candidate's ability to take a comprehensive, but ordered and concise
history in a patient with recent onset of bilateral leg oedema. It also examines clinical reasoning
abilities in understanding the potential causes of leg oedema and proceeding in a logical way to
accumulate the relevant positive and negative features of the history in order to form a
satisfactory probability diagnosis.
Congestive heart failure can present in a subtle way with symptoms of right heart failure, such as
bilateral leg oedema, which is worse after prolonged standing and reduces with supine rest. As
in this case, there is often a coexisting history of left heart failure symptoms, such as exertional
dyspnoea. It is very important in a patient with possible congestive heart failure not to be
satisfied with this as a complete diagnosis, but to ask the questions 'Why has this patient
developed heart failure? What is the underlying cause? This will require an understanding of the
pathophysiology of heart failure.
Heart failure is difficult to define. Various definitions include the following:
A pathophysiological state in which an abnormality of cardiac function is responsible for the
failure of the heart to pump blood at a rate commensurate with the requirements of the
metabolising tissues.'
A clinical syndrome caused by an abnormality of the heart and recognised by a characteristic
pattern of haemodynamic, renal, neural and hormonal responses.'
A biomechanical definition is that the failing heart exhibits a reduction of power such that it
cannot maintain a normal cardiac output without abnormal elevation of systemic and/or
pulmonary venous pressures.
The underlying causes of heart failure are many and it is useful to consider these under the
following group headings:
Primary myocardial disease (ischaemic heart disease, cardiomyopathy).
Pressure overload (hypertension, aortic stenosis).
Volume overload (aortic regurgitation, mitral regurgitation, ventricular septal defect, high
output states).
Obstruction to ventricular filling (mitral stenosis).
Restriction of ventricular filling (hypertrophic cardiomyopathy, constrictive pericarditis).
194
043
Performance Guidelines



In this station the patient presents with important history features of probable ischaemic heart
disease and cardiac arrhythmia. The onset of atrial fibrillation is often a precipitant of heart
failure, and this is particularly true in patients who are older, have hypertension and/or
diabetes, or underlying mitral valve disease. In patients who have stiff (noncompliant) hearts -
due to age, left ventricular hypertrophy, hypertension, diabetes, ischaemic heart disease or a
combination of these the first presentation may be due to cardiac diastolic dysfunction
where heart failure is caused by an increased resistance to filling of one or both ventricles.
Atrial fibrillation is often a cause of diastolic heart failure because of the effects on ventricular
filling with loss of the atrial systole and an increased ventricular rate. With diastolic dysfunction
of the left ventricle, the presentation is usually with breathlessness on exertion and episodes of
acute pulmonary oedema. In this patient, the presentation with leg oedema is an indication of
right heart failure, which in the great majority of patients is secondary to a longstanding
problem with the left heart, the so-called 'backward failure'.
CONDITION043.FIGURE 2.Pitting
oedema in CCF
195
2-B: Physical Examination

Vernon C Marshall and Barry P McGrath
'One of the unexpected and disturbing results of the development of increasingly precise and useful diagnostic
measures in the laboratory and X-ray departments is a significant and often alarming decrease in emphasis on the
training of the medical student to perform with excellence the average comprehensive physical examination.'
Journal of the American Medical Association (1962)

LOOK! MOVE! FEEL! LISTEN! MEASURE! COMPARE! INTERPRET!
This aide-memoire comprises the seven champions of physical examination. Used in the correct sequence (and
remember the above sequence, eyes first and foremost and only then fingers, hands and ears), they form the basis of all
physical examination techniques. Whether one is examining the whole patient, a focal region (head and neck, back, chest,
abdomen, limbs) or a body system (integument, cardiovascular, respiratory, gastrointestinal, neuroendocrine), an ordered,
well practised and logical sequence is essential. Sound technique facilitates accurate findings and diagnostic acumen.
Physical examination still matters and, along with a careful history, will confirm the diagnosis in the majority of
consultations despite the plethora and utility of available investigations. In preparing themselves to be good noticers and
good examiners of physical signs, clinicians should gain practice in:
Pattern recognition: the ability to define and group a constellation of features in order to diagnose, for example,
shock, hyperthyroidism or cardiac failure.
Focused examination of an area or region such as a limb, the neck, the chest or the abdomen. Here one must
concentrate on checking features normal and abnormal-of the multiple local structures which comprise focal
components of several body systems grouped at a common site. A sound knowledge of clinical anatomy is an
essential prerequisite.
Examination of multiple sites and areas logically, sequentially and expeditiously to provide global assessment of a
body system (cardiovascular, gastrointestinal, etc) A sound knowledge of clinical physiopathology is an essential
prerequisite.
This introductory segment and the MCATs following provide selected examples of these techniques.
Skill development in physical examination is sequential throughout undergraduate medical education and extends into
independent and specialist practice. Like the acquisition of any skill, medical practitioners in their attempts to become
skilled clinicians must:
have a good understanding of correct methodology;
assiduously develop the correct techniques;
have the right equipment and know how to use it;
know the range of normality and what constitutes abnormality;
be aware of the limitations of clinical signs, but use adjuvant investigations thoughtfully and selectively; and
practise, and practise frequently.

196

2-B
Physical Examination



Central to correct physical examination, and unique to the health domain, is the manner in which
the examiner interacts with the patient. Common problems that are observed in candidates
undertaking physical examination include the following:
lack of empathy and skill in engaging the patient;
failure to spend time in general inspection of the patient, thus missing
important aspects of pattern recognition;
Physical examination still matters, andalongwitha
careful history, will confirmthe diagnosis in the
majorityof consultations, despite the plethora and
utilityof available investingations.
causing undue discomfort to the patient;
incorrect techniques;
failure to develop a careful, systematic approach;
a slipshod approach, missing important signs along the way;
inaccuracy of sign characterisation and of measurements;
missing obvious pathology by overlooking physical signs;
finding things that are not there;
over-interpretation; and
inability to provide a succinct, accurate clinical summary.
PHYSICAL EXAMINATION REGIONAL EXAMINATION
The integument
The skin is the largest body organ. Skin rashes should be assessed as macular, papular,
maculopapular, vesicular or pustular, itchy or nonitchy. Rashes are commonly allergic, irritative
or infective. Atopic eczema is a blotchy ill-defined red macular rash which can progress to papule
and pustule formation. Irritative contact dermatitis can be wet (intertrigo, nappy rash), or dry and
associated with hyperkeratosis, lichenification and pigmentation. Infective rashes are legion and
range through bacterial (impetigo, acne), fungal, or viral (molluscum contagiosum, herpes
simplex and zoster, HIV). Involvement of scalp or nails may occur (psoriasis). The distribution of
the rash (e.g. pretibial erythema nodosum) and associated features (e.g. focal skin ischaemia in
vasculitis; central clearing in fungal lesions), give important diagnostic clues.


SECTION 2-B. FIGURE 2.
Acne vulgaris
SECTION 2-B. FIGURE
1. Flexural eczema
197
2-B
Physical Examination
198
SECTION 2-B. FIGURE 3. SECTION 2-B. FIGURE 4.
Molluscum contagiosum Microsporum canis
('ring worm')
Focal skin lesions are also of immense variety. In Australia, malignant skin lesions are common,
particularly in higher latitudes and in fair-haired and pale-skinned individuals. Basal cell cancers
are the most common cancers, and although mostly seen on the face and other exposed parts,
can occur anywhere. By contrast squamous cancers are almost always confined to sun
exposed areas. Melanomas are the most serious lesions; their incidence is increasing in
Australia and in most parts of the world, so picking up dysplastic or premalignant lesions is
important.
Most focal skin lesions are, however, benign and include benign melanocytic and other naevi,
calluses and viral warts. Solar keratoses, seborrhoeic keratoses, dermatofibromas (sclerosing
hemangiomas), 'senile' melanocytic and purpuric freckling, and cherry angiomas (Campbell de
Morgan spots), are seen with increasing frequency with increasing age.
SECTION 2-B. FIGURES 5 AND 6.
Neurofibromatosis Type I von Recklinghausen disease of nerves Note
numerous cutaneous neurofibromas (molluscum fibrosum)
2-B
Physical Examination
SECTION 2-B. FIGURES 7 AND 8. SECTION 2-B. FIGURE 9.
Portwine stains cavernous haemangiomas Nodular portwine stain
Cutaneous neurofibromas form part of the syndrome of von Recklinghausen disease of
nerves (neurofibromatosis). The syndrome is usually readily identified by pattern recognition.
Solitary cutaneous neurofibromas are also often found apart from the inherited syndrome.
Congenital 'portwine' stains (cavernous haemangiomas) have a classical appearance and
may become nodular with age.
It is usually possible following a focused and accurate history and examination to classify
lesions into clearly benign', 'clearly malignant', and 'suspicious' with the latter two needing
appropriately wide excisional biopsy.
Subcutaneous lumps
These are mostly benign and often merely need accurate diagnosis and reassurance. The
diagnostic features of most importance are site, physical characteristics, and relationships of
the lump to its surroundings (which includes the regional nodes).
Critical features to note are the Ss, Cs, Ts, Fs, and Ps.
Site, Size, Shape, Surroundings.
Contour, Consistency, Colour, Compressibility, Cough impulse.
Tenderness, Temperature, Transillumination.
Fluctuation, Fixity, Fields.
Pulsation, Percussion.
The lump should always be layered' is it in subcutaneous fat, and if so is it attached to
overlying skin, or underlying fascia and musculature?
199
The mobility of subcutaneous lumps in relation to their superficial and deep surroundings is
important in picking up infiltrative rather than expansile enlargement. The former is very
suggestive of malignant or inflammatory fixation and fibrosis. The lump's 'mobility' or fixity helps in
checking whether it is below deep fascia, attached to nearby bone or vessel or nerve, in the
abdominal parietes or intra-abdominal. Most lipomas, 'sebaceous' and other cysts, ganglia,
bursae, lymph node swellings, hernias, vascular swellings and other subcutaneous lumps will be
readily diagnosable if the above simple rules of focused assessment are combined with basic
knowledge of local anatomy and likely pathologies.
Head and neck lumps
With neck lumps it is particularly important always first to observe the effects of movement:
swallowing, coughing, protruding the tongue, and tensing underlying muscles such as
sternomastoid or trapezius. Remember to examine the accessible nasopharynx and oropharynx.
The laryngopharynx and oesophagus are not accessible to your examining hands and fingers,
but remember the importance of endoscopic evaluation in the diagnosis of occult primary
neoplasms presenting as neck lumps. With neck lymph node swellings always keep in mind the
possibility of:
lymphatic spread from areas outside head and neck (chest and lungs, the abdomen or
genitals); and
focal presentation of systemic lymphoid pathology.

SECTION 2-B. FIGURE 10. SECTION 2-B. FIGURE 11.
Hodgkin l ymphoma Nodal metastasis from papillary
carcinoma thyroid
Examination of a cytologic aspirate will often clarify the diagnosis and point the way for further
diagnostic tests. For example, squamous neoplastic cells in a neck lymph node point to a
potential primary neoplasm of skin, laryngopharynx, oesophagus, or lung, rather than from
thyroid or stomach. Cytology may be specific for melanoma. If suggestive of adenocarcinoma,
cytology commonly points to a lung, stomach, colon, breast, or testicular origin of the primary.
Cytology is particularly useful in diagnosis and classification of lymphomas.
Careful application of the above techniques facilitates identification of the common head and
neck lumps and their primary pathologies. The most common swellings will involve lymph nodes,
thyroid, salivary glands, or developmental lesions (branchial cysts, cystic hygromas,
sternomastoid 'tumour'). Rarer lesions include chemodectomas such as carotid body tumours
and neurilemmomas.
200
2-B
Physical Examination



Examination of the hands and wrists
This assessment will include structural and functional changes across multiple systems. A logical approach is to think
successively of the various tissue layers, checking for structure and function of each. Inspect carefully for deformities,
and any abnormalities of skin and nails, then probe deeper. Test active and passive movements of each joint, always
checking active movements first. Palpate carefully and carry out clinical testing for vascular insufficiency,
musculotendinous disorders, bone and joint problems, and neurologic abnormalities.
Common conditions encountered include:
Skin and nails: circulatory, neurotropic and occupational changes; a large variety of dermatoses and nail changes;
pitting; infective lesions (Osier nodes, etc.); and vasculitis (nailfold capillaries).

SECTION 2-B. FIGURES 12 AND 13. Osier nodes in
bacterial endocarditis
Subcutaneous fasciae: Dupuytren nodularity and contracture, carpal tunnel syndrome.
Muscles, tendons and sheaths: Volkmann contracture (long forearm muscles) and short hand muscle
contractures (intrinsic-plus deformity); trigger finger (stenosing tenosynovitis), De Quervain tenosynovitis;
spontaneous tendon rupture (dropped finger, thumb); and ganglia (dorsal, ventral, digital).
Bones and joints: changes of osteoarthritis (Heberden and Boucher nodes, carpometacarpal joint of thumb);
rheumatoid arthritis (synovial thickening, rheumatoid nodules, metacarpophalangeal subluxations and ulnar
deviation fingers. Z-thumb, swan-neck. boutonnire, mallet finger deformities); and gout.
SECTION 2-B. FIGURES 14 AND 15.
Hands in rheumatoid arthritis Rheumatoid nodules
201
2-B
Physical
Examination
Nerves: check median, ulnar and radial nerve motor, sensory and autonomic function;! differentiate
peripheralnervelesionsfrommorecentrallylocatedcervicalnerveroot,and|upperorlowerbrachialplexus
lesions.
SECTION 2-B. FIGURES 16 AND 17.
Testinginterosseifunction Thenaratrophy
Vessels:observeforvascularischaemicdigitallesions,palpatepulses,checkdominant!arterialsupply
(Allentest),checkforproximallesions(cervicalrib,listenforaxillarybruit)!
With hand and wrist trauma, check for bone and joint injuries, and local and distal tendonj nerve and
vasculareffects.
Functionalassessment:testgripstrengthindominantandnondominanthand:testlpower,precision,and
hookgripsandoppositionoffingersandthumb.Finallyaskpatienttoperformeverydaytasksofusinga
key,undoingbuttons,writing,andcombinghair.
Remember that any regi onal examination (for example, of head and neck, abdomen,; chest, limbs)
necessarily involves assessment of several systems. A systems-based examination, by contrast,
involvesexaminationofseveralregions.Notethedifferingfocusedtechniquesrequiredinperformingan
abdomi nal examinationfromexaminationofthegastroi ntesti nal system

PHYSICALEXAMINATIONSKILLS:EXAMINATIONOFTHEMAINBODYSYSTEMS
The structured approaches which follow providesuccinctinformation on how to perform aq examination of
eachofthemainbodysystems.Theaimistoprovideguidanceforathoroughexaminationofeachsystem
suchthatimportantsignsarenotoverlookedReadersareprovidedwithlearningobjectivesforeachsystem
andabriefguideonhowtoprepareboththemselvesandthepatientinordertoconductthesystemspecific
examination. This material is based on the Clinical Skills curriculum for Monash University Faculty of
Medicine,NursingandHealthSciences.
Genericlearningobjectives
Conductphysicalexaminationsacrossthefollowing:
~Integument(seepreviousdescription)
~Neurologicalsystemandmentalstatus
~Cardiovascularsystem
~Respiratorysystem
~Gastrointestinalsystem
202
203
2-B
Physical Examination



~Haematologicalsystem
~Endocrinologicalsystem
~Rheumatoiogicalsystem
~Renalandurogenitalsystem
Interpretandintegratehistoryandphysicalexaminationfindingstoarriveatanappropriate
diagnosisordifferentialdiagnosisincommonlypresentingcomplaintsandconditions.
Describeanduseclinicalreasoningskills.
Preparingthepatient
Establishpatient'slevelofcommunicationcapacity.
Introductions:
~Setthescene.
~Explainyourstatus.
~Exhibitahumaninterestinthepatient.
~Gainpatientpermission.
Demonstrateprofessionalism.
Showsensitivitytopatient'smodesty,healthstatusandcomfort.
Involvepatientintheprocesswithclearinitialexplanationandstepwiseinstructionsregarding
whatyouaredoingandwhyandwhatyouwishthepatienttodo.
Establishwhatdifficultiesanddiscomfort(especiallypain)thepatientmayhavebeforeand
duringtheconductofthephysicalexamination,andavoidcausingpainwhereverpossible.
Whatequipmentisneeded?
Have your own basic set of items to aid in eliciting signs. Items marked with an asterisk are
standardrequirementsforpersonaluse
watchwithstopwatchorsecondhand.*
stethoscope with capacity to detect low frequency (bell) and high frequency (diaphragm)
sounds.*
penciltorch.*
disposabletonguedepressors.*
measuringtape.*
reflexhammer(QueenSquarepattern,bestwithalarge-sizerubberhead).*
pinsthesemustbesingleuseonlyandmustnotbehypodermicneedlesordiabeticlancets.
Neurotipsareexcellent.*
cottonwool.*
128or256Hztuningforkforvibrationtesting.*
Snellenchartfortestingvisualacuity.
mini-mentalstateexamination(MMSE)card.
sphygmomanometers will be available in all wards and clinics and other items will also be
availableforrelevantstations,butitemsstarredyoushouldhaveforpersonaluse.
204
2-B
Physical Examination



1. THE NEUROLOGICAL SYSTEM
1.1 Objectives
Objectives for a neurological examination
Perform a stage-appropriate, technically competent neurological examination, incling
~mental status
~speech - gait
~cranial nerves
~limbs
Localise neurological disorders based on the results of physical examination.
Other objectives
Demonstrate stage-appropriate knowledge of the selection and use of standard neurological investigations
(magnetic resonance imaging [MRI], computed tomography [CT], single proton emission computed
tomography [SPECT], positron emission tomography [PET], electroencephalography [EEG], nerve
conduction studies [NCS], electromyography j [EMG], lumbar puncture [LP]) based on the results of history
and physical examination,
1.2 Preparation
What specific equipment is needed?
Essential
A red-topped pin for visual field examination
A bright pocket torch (a focusing torch with a halogen bulb, [e.g. mini-Magliteor
:
similar] is best)
Visual acuity chart (Snellen) the half-size 3 metre chart is the most practical for ward work
Desirable
Ophthalmoscope
512 or 1,024 Hz tuning fork for hearing tests
Glasgow Coma Score card as an aide mmoire
Usually readily obtainable
Cotton wool (for corneal reflex testing)
Large size paper clip (straighten, bend in centre, then bend tips at right angles to ft a serviceable 2-point
discriminator)
1.3 Physical examination
1.3.1 The neurological examination
Assessment of mental status
~level of consciousness
~attention (e.g. digit span)
~language (comprehension, repetition, spontaneous speech, naming)
~memory
~visuoconstructional ability
~executional ability
~MMSE (for scaling)
2-B
Physical
Examination



SECTION 2-B. FIGURE 18.
Trendelenburgtest
Assessmentofspeech
~dysphasia/dysarthria/dysphonia
Observationofgaitandposture
~freegaitandturning
~tandem(heeltotoe)gait
~Rombergtest
~toe/heelstanceandwalk,risingfromsquatorchair.
Trendelenburgtest
Cranialnerveexaminationinvolves
~olfaction(notroutinelytested,anosmiausuallydue
toolfactorynerveorbulbinjury)
~vision:acuity,visualfields(redpin),colourvision,
fundoscopy
~pupils:shape,size,symmetry,reactivity(lightand
accommodation)
~eyemovements:smoothpursuit(H-shape),diplopia,
nystagmus
~trigeminal:cornealreflex,cutaneoussensation,motorfunction,jawjerk
~facial:facialmovements,strengthofeye/mouthclosure,cornealreflex
~hearingandbalance:whisperedvoice,otoscopy,tuningforktests;vertigo;nystagmus
~palatal:sensation,gagreflex/palatalmovement,cough
~accessory:sternocleidomastoids,trapezius
~hypoglossal:tongueprotrusion/fasciculation
SECTION 2-B. FIGURE 19.
Papilloedema
SECTION 2-B. FIGURES 20 AND 21.
Righthypoglossalnervepalsy
Examinationofthelimbs
~observefordeformity/wasting/fasciculation/adventitiousmovements
~tone(spasticity,extrapyramidal)
~power
~reflexes(tendon/cutaneous)
~coordinationandrhythm
~sensation
jointposition/vibration
pinprick/temperature
2-pointdiscrimination
205
206
2-B
Physical Examination



2. THE CARDIOVASCULAR SYSTEM
2.1 Objectives for a cardiovascular examination
Inspect for general and peripheral signs of cardiovascular disorder
Accurately record vital signs pulse and blood pressure
Assess the jugular venous pulse
Perform comprehensive central examination of the heart
Detect and differentiate normal and abnormal impulses, heart sounds and murmurs
Examine the lung bases, abdomen and lower limbs for signs of heart failure
Examine the central (carotid and aorta) and peripheral arterial pulses and listen for bruits
Provide an accurate summary of your findings
2.2 Preparation
Specific to the cardiovascular examination
~ have adequate exposure of the patient's chest wall
~ comfortably position the patient in the supine, 45 degree and sitting positions,
2.3 Physical Examination
2.3.1 The cardiovascular examination
Observe general appearance - colour
~ respiration
~ peripheral swelling
Observe and feel the hands
~ colour
~ warmth
~ fingernails
Feel and listen: the arterial pulse radial: character, rate, rhythm, symmetry, brachial, the vessel wall
Measure and interpret blood pressure
Observe the face, tongue, sclera and conjunctivae
The neck
~ JVP: height, character, waveform
~ carotid arteries, feel and listen
~ trachea: position
The precordium
~ inspect for scars, pulsations
~ feel apex beat, and over 4 valve sites (impulses, thrills)
~ listen for heart sounds, murmurs 4 sites
~ special manoeuvres for mitral, aortic murmurs
The chest
~ percuss and auscultate lung bases
2-B
Physical Examination



The abdomen
~look, feel, and percuss liver edge (check for pulsation and movement with breathing)
~look, feel, and percuss spleen
~look, feel, and listen to aorta
~examine femoral pulses (radial-femoral delay)
The limbs
~inspect skin
~check for pitting oedema
~check pulses and peripheral circulation
~check venous system
3 THE RESPIRATORY SYSTEM
3.1 Objecti ves for a respiratory examination
Inspect for signs of respiratory disorders, respiratory distress
Accurately record vital signs
Recognise clubbing
SECTION 2-B. FIGURES 22 AND 23.
Finger clubbing

Recognise different breathing patterns
f accessory muscles, diaphragmatic
~t
nd lower limbs for evidence of right heart failure
atic disease lymph nodes in neck and axillae, liver, bony tenderness
tion of findings
~paradoxical, asymmetrical, recruitment o
dysfunction
Examine the thorax
~the chest wall and spine
he lung fields
~central cardiac examination
Assess J VP, the abdomen a
Assess for metast
Provide an accurate summary and interpreta
207

11
3.2Usefulspecificequipment
Peakflowmeter
SECTION 2-B. FIGURES 24 AND 25.
Peakflowmeter
3.3Physicalexamination
3.3.1Therespiratoryexamination
Lookforuseofsputumcup,inhalers,oxygen
Generalinspectionofpatient
~obesity/cachexia
~inspiredoxygenrequirements
~cyanosis
~respiratorydistressandventilatorypattern
Inspecthands
~nicotinestaining
~clubbing
~peripheralcyanosis
~metabolicflap
~pulmonaryosteoarthropathy
Bloodpressuremeasure,determineifthereisparadox
Headandneck
~JVPheight,characterandwaveform( cor pul monal e)
~mouth/tongue(centralcyanosis)
~trachea(tug,deviation)
~lymphnodegroups
Cardiacexamination
~apexbeatposition
~parasternalheave( cor pul monal e)
~heartsounds
208
2-B
Physical Examination



Thorax
~chest inspection scars, deformity, kyphosis, barrel chest, rib crowding (anterior
upper chest)
~the lung fields (start posteriorly)
~chest expansion demonstrate symmetry/asymmetry, check for flail segment.
~percussion Compare sides for normality, dullness, hyper-resonance
~auscultation vocal resonance, normal and abnormal breath sounds (bronchial
breathing) and added breath sounds (wheezes or crackles)
~anterior chest repeat lung fields examination
~test for upper lobe expansion, symmetry
~percuss over clavicles
~percuss and auscultate upper chest, axillae and laterally (remember right middle lobe
region)
~percuss spine and spring ribs for bony tenderness
~assess sacral oedema
Abdomen
~liver span look for ptosis, feel for pulsatile liver
Lower limbs
~oedema, rashes
Bedside lung function testing
~forced expiratory time (obstructive disorders)
~counting time (restrictive and/or obstructive disorders)
~peak flow measurement (special test)

4 THE GASTROINTESTINAL SYSTEM
4.1 Objectives for a gastrointestinal examination
Inspect for general and peripheral signs of gastrointestinal disease
Accurately record vital signs (including lying and standing blood pressure)
Recognise
~anaemia and hypovolemia
~jaundice, ascites and signs of chronic liver disease
~abdominal veins (Caput Medusae)
~hepatomegaly and splenomegaly
SECTION 2-B. FIGURES 26 AND 27.
Scleral jaundice Ascites chronic liver disease
209
2-B
Physical Examination



Detailedassessmentofthe
~abdomen
~periphery
AssessmentofJVPandheartforevidenceofrightheartfailure
Assessmentformetastaticdisease
Summaryandinterpretationoffindings
4.2Physicalexamination
4.2.1Thegastrointestinalexamination
Positionthepatientcorrectly(bedflat,singlepillow,abdomenandchestexposed)
Generalinspectionofpatients
~jaundice
~weightandwasting
~abdominaldistensionandperipheraloedema
~skin(pigmentationandbruising)
~mentalstate(encephalopathy)
Inspectthehands
~nails(leuconychia,clubbing)
~palms(erythema,anaemia,Dupuytrennodularity)
~flap(asterixis)

SECTION2-B.FIGURE28.
Leuconychia
Inspectthearms
~bruising,petechiae,scratchmarks
~spidernaevi
~pulseandbloodpressure
Inspecttheface
~eyes:jaundice,anaemia,xanthelasma
~parotidglands
~mouth:dentitionandbreath(fetor)
~tongue
210
2-B
Physical
Examination



Inspecttheneckandchest
~cervicalandsupraclavicularnodes
~spidernaevi
~gynaecomastiaandbodyhair
~JVP
4.2.2Theabdomen
Inspection
~scars,herniae(inspectwithcoughingandstrainingbeforepalpation).
~distensionorlocalswellings(inspectondeepbreathing)
~prominentveins
~skinlesionsandstriae
~periumbilicalorflankdiscolouration(Cullensign,GreyTurnersign)

SECTION2-B.FIGURE29. SECTION2-B.FIGURE30.
Dilatedabdominalveinsinportalhypertension CombinedCullenandGrey
Turnersignsinacute
pancreatitis
Palpation
~superficialpalpation(tenderness,rigidity,outlineofanymasses)
~deeperpalpation(definemasses;liver,spleen,kidneys,otherabnormalmasses)
~measurementoforgan(s)ifenlarged
~rollontorightsidetopalpatespleen
Percussion
~visceraloutline
~ascitesandshiftingdullness(awayfromexaminer;midlinetoleftflank)
~listenbowelsounds,bruits,hums
Inspectthegroin(seekpatient'sspecificpermission)
~genitalia
~lymphnodes
~hernias
211
2-B
Physical Examination



Inspect the lower limbs
~ oedema
~ bruising
~ neurological signs (alcohol)
Other
~ ask to perform a rectal (PR) examination
~ temperature chart
~ urine analysis check this routinely for all systems
5 THE HAEMATOLOGICAL SYSTEM
5.1 Objectives for a haematological examination
inspect general appearance
inspect the hands and face and eyes
examine the lymph node groups: epitrochlear, axillary, facio-cervical, supraclavicular, abdominal, inguinal
assess for bone tenderness
perform an abdominal examination
examine the legs
perform a urinalysis with dipsticks
provide an accurate summary of your findings oral and written
5.2 Physical examination
General appearance (position patient lying on the bed with one pillow)
~ geographical and ethnic origin thalassaemia
~ pallor anaemia
~ bruising distribution and extent
~ jaundice haemolysis
~ scratch marks/pruritus lymphoma or myeloproliferative disorders
SECTION 2-B. FIGURES 31 AND 32.
Spontaneous bruising and abdominal wall Rectus sheath haematoma confirmed
haematoma from warfarin on CT
212
213
2-B
Physical Examination



Hands
~ nails koilonychia, dry, brittle, ridged, spoon-shaped nails due to iron deficiency
~ pallor nail beds anaemia
~ rheumatoid arthritis or other connective tissues disorders, anaemia of chronic disease
~ gout myeloproliferative disorders
~ pulse tachycardia
~ anaemic patients have increased cardiac output and compensated tachycardia
because of reduced oxygen-carrying capacity of blood
~ purpura macular bruising within the skin, which can vary in size
~ petechiae pinhead bruising on the dependent parts of the body
~ ecchymoses large bruises
Epitrochlear lymph nodes
~ must always be palpated
~ place the palm of the right hand under the patient's right elbow. Examiner's thumb can then be placed over the
area that is proximal and anterior to the medial epicondyle
~ enlarged epitrochlear lymph node is suggestive of Non-Hodgkin lymphoma
Axillary lymph nodes
~ five main groups of axillary lymph nodes anterior and posterior, central, lateral and medial
Face
~ eyes
- scleral jaundice haemolysis
- haemorrhage platelet or bleeding disorder
- injection polycythaemia
- conjunctival pallor anaemia
~ mouth
- gum hypertrophy leukaemia especially acute monocytic leukaemia
- gum bleeding
- atrophic glossitis megaloblastic anaemia, iron deficiency anaemia
- Waldeyer ring lymphatic tissue involving the tonsils and adenoids enlarged in Non-Hodgkin lymphoma
Cervical and Supraclavicular Lymphadenopathy
~ sit patient up and examine from behind and in front
~ eight groups submental, submandibular, jugular chain, posterior triangle, occipital, postauricular, preauricular and
supraclavicular
Bone tenderness
~ tap spine
~ press ribs
~ gently press sternum and clavicle
~ enlarging marrow due to infiltration by myeloma, lymphoma or carcinoma
2-B
PhysicalExamination



Abdominalexamination
~splenomegalypalpate,percussandmeasure
~hepatomegalypalpate,percussandmeasure
~para-aorticlymphnodes
~inguinallymphnodestransverseandverticalgroups
~testicularmasses
Legs
~bruising
~pigmentation
~scratchmarks
~legulcershaemolyticanaemia
~neurologicalabnormalitiesvitaminB
12
deficiency
6THEENDOCRINESYSTEM
6.1Objectivesforanendocrineexamination
Inspectforgeneralphysicalfeaturesassociatedwithendocrinedisorders
Developskillsinsymptompatternrecognitioninendocrinediagnosis
Identifytypicalappearancesofpatientswithhypothyroidism,hyperthyroidism,acromegaly,
Cushingsyndrome,Addisondisease,Klinefeltersyndromeandhypogonadism

SECTION2-B.FIGURES33AND34.
Coarsefacialfeaturesandskeletalenlargementcharacteristicofacromegaly.
Tailortheexaminationtothespecificorgansystem
Evaluatesignsofhormoneover-secretionorunder-secretion
Provideanaccuratesummaryofyourfindingsoralandwritten
6.2Physicalexamination
6.2.1Theendocrineexamination
Generalinspection:observeforfeaturesofspecificendocrinedisorders:e.g.Cushing
syndrome,acromegaly,diabetesmellitus,hypoglycaemia,thyrotoxicosis,hypothyroidism
Vitalsignsbloodpressure(posturalhypotension)andpulse(bradycardia/tachycardia)
214
2-B
Physical Examination



Inspectandfeelhands
~overallsize
~lengthofmetacarpals
~abnormalitiesofnails
~tremor
~palmarerythema
~sweatingofpalms
Examineaxilla
~lossofaxillaryhair,acant hosi s ni gr i cans, skintags
Inspecteyes
~visualfields
~fundi
Face
~hirsutism/hairless
~skingreasiness,acne,plethora
Mouth
~protrusionofchin
~enlargementoftongue
~buccal/lippigmentation

SECTION 2-B. FIGURE 35.
Peutz-Jegherssyndrome
Neck
~ alwayslookfirstandcheckeffectofmovements
~ examineforthyroidenlargementsmoothlydiffuse,multinodular,oruninodular
~ palpatelymphnodesfrombehindandfrominfront
~ feelforthrill,listenforbruitoverthyroid









215
2-B
Physical Examination
Chestwall
~hirsutism/lossofbodyhair
~reductioninbreastsize/gynaecomastia
~nipplepigmentation

SECTION2-B.FIGURE36.
Gynaecomastia
Abdomen
~scars,purpura,striae,masses,hepatomegaly,cirrhosis,lipohypertrophy
~hirsutism
~externalgenitalia
~centralfatdeposition
Legs
~reflexes,tone
~diabeticchanges
Bodymassindex(BMIkg/m
2
)
7RHEUMATOLOGICAL/MUSCULOSKELETALSYSTEM
7.1Objectivesforarheumatologicalexamination
Performaccuratefocusedphysicalexaminationofjoints,bones,tendons,musclesandbursae
Followsequenceofl ook,move,feel ,l i sten,measure,compareandInterprettoridentifying
normalandabnormalfindings
Assessjointsoflimbs,spineandfaceidentifyevidenceofarthritisandwhetheracuteorchronic,
monarthriticorpolyarthritic
Identifynormallocomotorsystemanatomyandjointmovementranges,andanomaliesincluding
disordersofstance,gaitanddeformities
Identifyextra-articularmanifestationsofsystemicrheumatologic/connectivetissuedisorders
Comparesidesinunilateralabnormalitiesandeffectsofdominant/nondominanthandinupperlimb
disorders



216
2-B
PhysicalExamination
SECTION2-B.FIGURE37. SECTION2-B.FIGURE38.
Kneejointexamination PositiveThomastestlefthip
8RENALANDUROGENITALSYSTEM
8.1Objectivesforaurogenitalexamination
Performaccuratefocusedphysicalexaminationofmaleandfemalegenitaliaandidentifyabnormalities
Performabdominal,vaginalandrectalexaminationwithaccurateinterpretationofsigns
Identifysignsofacuteandchronicrenalinsufficiencyandtheircauses
Performfocusedinguinoscrotalexaminationwithaccurateinterpretation
Identifyurinoscopyasaglobalscreeningtestofwideutility
Identifysignsandsitesofurinaryinfections
Identifyanddiagnosesitesandcausesofhaematuria,pyuria,bacilluria
Identifyanddiagnosesexuallytransmittedinfections
VernonCMarshallandBarryPMcGrath
217
218
2-B
PhysicalExamination



2-B PhysicalExamination
CandidateInformationandTasks
MCAT044-057
44 Assessmentofacomatosepatient
45 Recentonsetofpoordistancevisionina17-year-oldmale
46 Apainfulrashonthetrunkofa45-year-oldchild-careworker
47 Acutelowbackpainandsciaticaina30-year-oldman
48 Feverandarecentrashina30-year-oldman
49 Aheartmurmurina4-year-oldboy
50 Aknifewoundtothewristofa25-year-oldman
51 MultipleskinlesionsinaQueenslandfamily
52 Subcutaneousswellingforassessment
53 Examinationofthekneeofapatientwithrecurrentpainfulswellingafterinjury
54 Assessmentofhearingloss,firstnotedduringpregnancy,ina35-year-oldwoman
55 Examinationofa20-year-oldwomanwhodislocatedhershoulder6monthsago
56 Assessmentofagroinlumpina40-year-oldman
57 EyeproblemsinanAboriginalcommunity
219
044
CandidateInformationandTasks



Condition 044
Assessmentofacomatosepatient
CANDIDATEINFORMATIONANDTASKS
Thisyoungpatienthasbeenfoundunconsciousthismorningathomebyaflatmate.Theflatmate
isunabletoprovideanyhistory,onlybecomingaflatmateaweekago.Whenfound,thepatient
wasinbedandtherewasnoexplanationastowhythepatientmighthavebecomeunconscious.
ThepatientisnowintheEmergencyDepartmentwhereyouareabouttodoanexamination.The
airway is patent, and the patient is breathing without difficulty, the blood pressure is stable
(140/70mmHg)andtemperatureis37.5C.

YOURTASKSARETO:
Performanexaminationtodeterminethelevelofunconsciousnessandtotrytoidentifythe
cause.
Telltheobservingexaminerwhatyouaredoingandwhy.Thiscanbeasyouproceedorat
theendofeachcomponentofyourexamination.
Towardstheendoftheexamination(afterapproximatelysixminutes),youwillberequired
toprovidetheexaminerwithanassessmentoflevelofunconsciousness,alistofpossible
andlikelyexplanationsforthepatient'sunconsciousstateandtheinvestigationsyouwould
arrange.

ThePerformanceGuidelinesforCondition044canbefoundonpage235
220
045
Candidate Information and Tasks



Condition 045
Recentonsetofpoordistancevisionina17-year-oldmale

CANDIDATEINFORMATIONANDTASKS
You are working in a general practice. Your next patient is a 17-year-old apprentice who is
complainingofpoordistancevisionofrecentonset.Hecannolongerreadnotices,streetsigns,
scoreboardsetc.atadistance.Hesaysthisismostinconvenientandisgraduallygettingworse.
Botheyesareaffected.Hehasaskedyouifhemaybeshort-sightedlikehisfatherandhisolder
brother. He wants to be tested to check for short-sightedness or any other problems, to ask
whether he will need glasses or contact lenses, whether surgery can help and whether he
shouldseeanopticianoraneyespecialistdoctor.

YOURTASKSARETO:
Examinethepatient'seyestoexcludeseriouseyedisease.
Testthepatient'svisualacuityusingtheSnellentestchartprovidedandstateyour
findingstothepatient.
Explaintheproblemtothepatient.

Youdonotneedtotakeanyfurtherhistory.
ThePerformanceGuidelinesforCondition045canbefoundonpage241
046
CandidateInformationandTasks

Condition 046
Apainfulrashonthetrunkofa45-year-oldchild-careworker
CANDIDATEINFORMATIONANDTASKS
You are a medical officer in a hospital primary care clinic. A 45-year-old child-care worker
presentswithapainfulrashonthetrunk,asillustratedbelow.
YOURTASKSARETO:
Takeahistoryaboutthepresentingproblem.
Explainyourdiagnosisandthenatureoftheconditiontothepatient.
Advisethepatientaboutmanagement.
(Neartheendofthetimeallotted,theexaminerwillaskyouoneortwoquestions).
CONDITION046.FIGURE1.
ThePerformanceGuidelinesforCondition046canbefoundonpage246
221
222
047
CandidateInformationandTasks



Condition 047
Acutelowbackpainandsciaticaina30-year-oldman

CANDIDATEINFORMATIONANDTASKS
Youareworkinginageneralpractice.Yournextpatientisa30-year-oldself-employedlandscapegardener
whoiscomplainingofdisablingleftsidedlowbackpain.Thepaincameonsuddenlyyesterdaywhilstliftinga
heavyrock.Thepainisalsofeltdownthesideoftheleftthighandlegandtheoutersideofthefoot.Itismade
worse by coughing and movement. The patient could not sleep last night despite taking two Panadeine
tablets(paracetamol500mgcodeinephosphate8mgpertab).Thepatienthaspreviouslybeeninexcellent
healthandhasnootherrelevantpastorfamilyhistory.
Abnormalexaminationfindingsare:
Hehasdifficultystandingorwalkingonhistoesontheleftside.Hehasseverelimitationtoleftstraightleg
raising,withapositivestretchtest,diminishedleftanklejerkanddiminishedsensationtolighttouchonthe
outeraspectoftheleftfoot,andpainfullimitationoflumbarspinemovements,particularlyflexion/extension
andleftlateralbending.
YOURTASKSARETO:
Advisethepatientofthemostlikelydiagnosisandmanagementrequired.
Counselthepatientaboutwhenhecanreturntoworkandanynecessary
modificationsthatmayberequired.
Thereisnoneedforyoutotakeanyadditionalhistory,norrequestanyfurtherexaminationfindings.Allthe
informationyouneedisdetailedabove.
ThePerformanceGuidelinesforCondition047canbefoundonpage248
048
Candidate Information and Tasks



Condition 048
Fever and a recent rash in a 30-year-old man
CANDIDATE INFORMATION AND TASKS
You are working in a hospital primary care clinic. A 30-year-old man who works as a fashion
consultant in a clothing store is presenting to you with fever and rash, onset two days ago. The
rash appears as in the illustration below. It is a generalised erythematous maculo-papular rash.
You have just finished examining him. Your other findings on physical examination were a fever
of 38.5 C, an inflamed palate, a palpable spleen and generalised tender lympha-denopathy in
the neck, axillae and groins.

YOUR TASKS ARE TO:
Take a further focused history from the patient.
Explain to the patient the possible nature of his condition and how you intend to proceed.
Briefly discuss differential diagnosis and investigations with the examiner.
CONDITION 048. FIGURE 1.
The Performance Guidelines for Condition 048 can be found on page 252
223
224
049
CandidateInformationandTasks



Condition 049
Aheartmurmurina4-year-oldboy
CANDIDATEINFORMATIONANDTASKS
Youareworkinginageneralpractice.A4-year-oldboyhasbeenseenwithhismother.Hewastakento
another doctor with a cold whilst the family were on holidays and a soft cardiac murmur was heard. His
parentswereaskedtobringhimtoseethefamilydoctor,todecideifanythingfurtherneedstobedone.
Hisgeneralhealthandexercisetoleranceareexcellentandheisonthe50thcentileforheightandweight.
Hehasneverbeencyanosed.Thereisnohistoryofheartdiseaseintheimmediatefamilybutacousinhad
ahole-in-the-heartoperation.Hisparentsfeelhehasnoconcerningsymptoms.
Onexaminationyouhaveconfirmedasoftvibratorymidsystolicmurmur(grade2/6)locatedbetweenthe
lowerleftsternaledgeandtheapex,whichvarieswithrespiration.Fullphysicalexaminationisotherwise
completelynormal.Youhavefinishedyourhistory-takingandexaminationandareabouttodiscussthings
withthechild'smother.

YOURTASKISTO:
Explainyourdiagnosisandfurthermanagementtothechild'smother.

ThePerformanceGuidelinesforCondition049canbefoundonpage255
050
CandidateInformationandTasks



Condition 050
Aknifewoundtothewristofa25-year-oldman
CANDIDATEINFORMATIONANDTASKS
YouareworkingasaHospitalMedicalOfficer(HMO)inahospitalEmergencyDepartment.The
patient you are seeing has presented with a history of a knife wound to the left wrist from an
assailant after an argument in a pub. He has been brought to hospital by an ambulance. The
wound bled profusely at first and was controlled by a pressure dressing which is still on the
wound. The ambulance personnel described the wound as a nasty deep knife wound and its
extentisillustratedinthephotograph.
You are about to examine the patient for evidence of damage to important structures. Do not
remove the dressing. Assume that the illustration represents accurately the extent of the skin
wound.

YOURTASKSARETO:
Performafocusedandrelevantexaminationtodeterminethelikelyextentofinjury.Explain
totheexaminerwhatyouaredoing,andwhy,asyouproceed,orattheconclusionofthat
segmentoftheexamination.
Describeyourfindingsandyourdiagnosisoftheinjuriestotheexaminer.
Theexaminerwillaskyouoneortwoquestionsattheconclusionofyourcommentary
CONDITION050.FIGURE1.
ThePerformanceGuidelinesforCondition050canbefoundonpage257
225
051
CandidateInformationandTasks



Condi t i on 051
MultipleskinlesionsinaQueenslandfamily
CANDIDATEINFORMATIONANDTASKS
Youareworkinginageneralpracticeinasmallcountrytown.A58-year-oldfarmer,wholives
withhisfamily,160kmoutsideoftown,comestoseeyouasheisconcernedabouthisfamily
members,havingseenatelevisionprogrammeaboutskincancer.Hehastakenphotographsof
his family's various skin lesions and asks for your advice about the need for them to seek
medicalattention,andwhetherattendance isurgent.Theyareallverybusyharvestingcrops
andwillbesoforseveralweeks.
Thefarmerpresentsthefollowingphotographsshowing:
1.Thelipofhis35-year-oldson 2.Theneckofhis50-year-oldbrother

CONDITION051.FIGURE1.
CONDITION051.FIGURE2.
3.Thefaceofhis82-year-oldfather 4.Thelegofhis56-year-oldwife
CONDITION051.FIGURE3. CONDITION051.FIGURE4.
226
051
CandidateInformationandTasks



5.Thechestofhis52-year-oldbrother 6.Thefaceofhis22-year-olddaughter

(whodrinksalargeamountofalcohol)
CONDITION051.FIGURE5.
CONDITION051.FIGURE6.

YOURTASKSARETOADVISEHIMASFOLLOWSAFTERREVIEWINGTHE
PHOTOGRAPHS:
Indicatewhichlesionsarelikelytobebenign,andwhicharelikelytobemalignantor
suspiciousofmalignancy.
Indicatewhichmember(s)ofthefamilyrequire(s)themosturgenttreatment.
Indicatethemodeofspreadofanymalignantlesionsyoudiagnose.

ThePerformanceGuidelinesforCondition051canbefoundonpage264
227
228
052
CandidateInformationandTasks



Condi t i on 052
Subcutaneousswellingforassessment

CANDIDATEINFORMATIONANDTASKS
Youareworkinginageneralpractice.Yourpatientisseekingadviceaboutasubcutaneous
swelling which has been present for about 10 years. The patient thinks it may have grown
slowlyoverthisperiodbutnotmuchchangeinsizehasoccurred.Ithasneverbeenpainfulor
otherwisesymptomatic.Thepatientisnotparticularlyconcernedaboutitbutiscuriousastoits
cause.

YOURTASKSARETO:
Performanappropriatelyfocusedandrelevantphysicalexaminationinorderto
determinethenatureofthelump.
Describeyourfindingstotheexaminerasyouproceed.
Telltheexaminerthelikelydiagnosis.
Explainyourfindingsanddiagnosistothepatientandindicatewhatfurtherevaluation
and/ortreatmentisrequired.

Youmayaskrelevantquestionsofthepatientduringyourexamination,butyourprinciptaskis
toperformaphysicalexaminationandcometoadiagnosis.
ThePerformanceGuidelinesforCondition052canbefoundonpage274



Condition 053
Examination of the knee of a patient with recurrent painful swelling
after injury

CANDIDATE INFORMATION AND TASKS
The patient you are about to see in a general practice setting has a history of twisting the right knee six months ago when he
caught his foot on a piece of broken pavement. He fell on the knee and it became swollen and painful on the inner side. The
swelling caused a painful limp for a few days and then subsided with easing of symptoms.
Since then he has had intermittent attacks of pain on the inner side of the knee with swelling, which settles within 24 hours,
and has had difficulty in straightening the leg fully. He is, on occasion, apprehensive when twisting to the right.
Between attacks of pain he can walk normally with only a minor feeling of pain on the inner side of the knee. He is otherwise
well.
This is the first time he has consulted a doctor about this problem.

YOUR TASKS ARE TO:
Perform a focused and relevant physical examination of the knees, giving a commentary to the
observing examiner as you proceed, describing what you are doing and why, and your findings.
After seven minutes, you will be expected to present a diagnostic/differential diagnostic plan to the
examiner.

The Performance Guidelines for Condition 053 can be found on page 280
229
^1
230
054
Candidate Information and Tasks



Condition 054
Assessment of hearing loss, first noted during pregnancy, in a
35-year-old woman

CANDIDATE INFORMATION AND TASKS
You are working in a general practice and your next patient is a young woman who gave birth to her
first child one month ago. She is complaining of loss of hearing, which she first noted about
midway through her pregnancy. It has become progressively worse since and affects both ears.
She is otherwise well and her infant (breastfed) is thriving.
YOUR TASKS ARE TO:
Take a further focused history concerning her hearing loss (limit this to one minute).
Examine the patient and test her hearing, telling the examiner what you are doing, including
your findings.
Tell the examiner the type of hearing loss present.
Inform the patient of the most likely cause of her hearing loss.
Suggest to the patient what further action is indicated for her hearing loss, including a
prognosis.

The Performance Guidelines for Condition 054 can be found on page 282
CandidateInformationandTasks

Condition 055
Examination of a 20-year-old woman who dislocated her
shoulder6monthsago
CANDIDATEINFORMATIONANDTASKS
You are a Hospital Medical Officer (HMO). Your next patient dislocated her shoulder playing
competitive basketball six months ago. It was a typical anterior dislocation which was
complicated by a nerve injury and was treated by closed reduction, several weeks
immobilisationinasling,subsequentphysiotherapyandagymnasiumprogramme.
Thepatienthasreturnedforacheckupatthehospitaloutpatientdepartment,hastoldyouthe
shoulder now seems to be working fine, and that she would like to recommence playing
basketballnextseason.
YOURTASKSARETO:
Performanappropriatelyfocusedandrelevantphysicalexaminationofthearea.
Describeyourfindingstotheobservingexaminerasyouproceed.
Discussfutureactivitieswiththepatient.
Inthefinaltwominutesyouwillbeaskedquestionsbytheexaminer.
CONDITION055.FIGURE1.
Filmofpreviousdislocation6monthsago
ThePerformanceGuidelinesforCondition055canbefoundonpage286
231
056
CandidateInformationandTasks



Condition 056
Assessmentofagroinlumpina40-year-oldman
CANDIDATEINFORMATIONANDTASKS
Youareworkinginaprimarycareclinicattachedtoateachinghospital.Yournextpatientisa
50-year-oldmanwhoworksasabuilder'slabourer.Twoweeksagohefeltapaininhisright
groinafterheavyliftingatworkandaweeklaternoticedalumpinthegroinwhichhadnot
been there before. The lump is not acutely painful, but is uncomfortable on exertion or
walking.Discomfortiseasedonlyingdown.
Heisingoodgeneralhealth,withoutrelevantpasthistoryandhasnoproblemswithlungsor
heart,bladderorbowels.

YOURTASKSARETO:
Performafocusedphysicalexaminationtoassessthelump,whichisillustratedbelow.
Giveyourdiagnosisandmanagementplantothepatient.
Youdonotneedtotakeanyfurtherhistory.
CONDITION056.FIGURE1.
ThePerformanceGuidelinesforCondition056canbefoundonpage289
232

Condition 057
EyeproblemsinanAboriginalcommunity

CANDIDATEINFORMATIONANDTASKS
YouareadoctorworkinginageneralpracticeinaremotesettingintheNorthernTerritory.You
are about to see a nurse who has recently joined the staff of the general practice clinic. The
nursemadeatimetoseeyoutodiscusseyeproblemsshehasnoticedinthelocalAboriginal
community.
The nurse has taken digital photographs of eye problems that were noticed in a number of
affected individuals (see figures below of four separate individuals). In the upper two
photographstheuppereyelidiseverted.Thenursewantsyoutoexplainwhatcancausethese
appearances,andwhatcanbedoneabouttheprobleminthelocalcommunity.

YOURTASKSARETO:
Studythephotographsanddescribetheabnormalitiestotheclinicnurse.
Explaintothenursewhatdiseaseisillustratedinthephotographs,anditsepidemiology.
Discusswiththenursehowtheproblemshouldbemanaged.
Answeranyquestionsthatthenursemayhave.
CONDITION057.FIGURE2. CONDITION 057. FIGURE

CONDITION057.FIGURE3. CONDITION057.FIGURE4.
Figures1and2werephotographstakenafterevertingtheuppereyelid.

ThePerformanceGuidelinesforCondition057canbefoundonpage293
233
2-B

PhysicalExamination


2-BPhysicalExamination

PerformanceGuidelines

MCAT044-057

044Assessmentofacomatosepatient
045Recentonsetofpoordistancevisionina17-year-oldmale
046Apainfulrashonthetrunkofa45-year-oldchild-careworker
047Acutelowbackpainandsciaticaina30-year-oldman
048Feverandarecentrashina30-year-oidman
049Aheartmurmurina4-year-oldboy
050Aknifewoundtothewristofa25-year-oldman
051MultipleskinlesionsinaQueenslandfamily
052Subcutaneousswellingforassessment
053Examinationofthekneeofapatientwithrecurrentpainfulswellingafterinjury
054Assessmentofhearingloss,firstnotedduringpregnancy,ina35-year-oldwoman
055Examinationofa20-year-oldwomanwhodislocatedhershoulder6monthsago
056Assessmentofagroinlumpina40-year-oldman
057EyeproblemsinanAboriginalcommunity


















234

044
235
Performance Guidelines
Condition 044
Assessment of a comatose patient
AIMS OF STATION
To assess the candidate's ability to examine and diagnose a patient presenting with coma.

The examiner will have instructed the patient as follows:
You are to play the part of a young person found in a coma in your flat this morning breathing without difficulty
with a Glasgow Coma Score of 10 out of 15 (see following pages about Glasgow Coma Scale).
You are wearing shorts and T-shirt and are feigning unconsciousness and stupor on a hospital bed.
Your responses should be:
Maintain your level of consciousness and responses as follows during the candidate's examination. Keep
your neck stiff when candidate attempts to flex it.
~ Eye opening: eyes should be closed. Do not open them spontaneously or to verbal
command but open them in response to painful stimulation.
~ Best motor responses: no response to verbal command. Localise pain when
stimulated move arms towards source of pain or withdraw limb if stimulated.
~ Best verbal responses: use of inappropriate words. When painful stimuli applied
say 'piss off', or 'damn'or 'shit'.
The candidate will probably:
Do a general examination looking for evidence of injury.
Examine your eyes and pupils, and will open your eyelids to do this and shine a torch.
Examine your response to commands and painful stimuli.
Examine you for neck stiffness (which you have).
Check your pulse and breathing. Blood pressure and temperature have been given as normal.
Check your arms for evidence of intravenous drug abuse.
In summary, you are being examined to check the level of coma and possible causes for this.
You are feigning a partially responsive coma, with a Glasgow Coma Score of 10-11 out of the normal score
of 15, breathing spontaneously, reacting by localising to pain, and with inappropriate verbal response
when stimulated.
Remain in this role throughout the examination.
Remember, your neck is stiff if flexion is attempted.
236
044
Performance Guidelines



EXPECTATIONS OF CANDIDATE PERFORMANCE
The candidate is examining, in the Emergency Department, a comatose young person found in bed this morning,
who is haemodynamically stable.
The candidate is expected to:
Examine for evidence of injury to the head or elsewhere.
Look for evidence of neck stiffness.
Examine eye movement by gently opening the lids.
Examine pupillary size and response to light (direct and consensual).
If candidates say they are going to test the corneal response, indicate that there is normal eye closure to cotton
wool testing. Similarly, if the candidate wishes to look at the fundi or ear drums, advise that they are normal.
Examine breathing pattern no hyperventilation (as in a hyperglycaemic coma) or hypoventilation (as in a
drug overdose) is present.
Examine for evidence of intravenous drug use or insulin injection sites in patients with diabetes.
Check for pulse rate, rhythm and character, which are normal.
Arrange immediate blood sugar estimation this may be asked by the candidate as part of the examination.
Alternatively, it should be done as part of the investigations recommended.
After six minutes, the examiner will ask the candidate three questions:
1. 'What is the Glasgow Coma Scale level?' Answer: around 10-11 out of possible 15 (Table 1)
2. 'Name at least four possible causes of the coma?' Acceptable causes would be:
~ drug overdose
~ meningitis
~ cerebral vascular accident (subarachnoid haemorrhage)
~ diabetic hypoglycaemia or hyperglycaemia
~ head injury
~ psychiatric problem
3. 'What investigations would you do?'all these are required urgently:
~ brain computed tomograph (CT) / magnetic resonance imaging (MRI) (if available-lumbar puncture generally
should NOT be done until the results of head imaging are available. If results from CT/MRI are not
available, lumbar puncture using a 25 gauge needle would be appropriate in view of neck stiffness).
~ blood and/or urine for drug screen
~ serum electrolytes and blood glucose
~ oxygen saturation
044
237
Performance Guidelines
KEY ISSUES
Ability to perform a focused, relevant and accurate examination to aid determination of the level and cause of the
coma.
~Neck stiffness should be tested and identified.
~ No response to verbal command but response to pain.
~ The candidate should indicate appropriate knowledge of the Glasgow Coma Scale.
Ability to provide an adequate differential diagnosis.
Ability to describe an initial investigation plan.
~ Mandatory investigations should include Brain CT or MRI. drug screen and blood sugar level. If CT/MRI is not
readily available, a lumbar puncture should be performed if there is no evidence of papilloedema.


Failure to determine reasonably the level of coma by the Glasgow Coma Scale score.
Failure to check for neck stiffness.


Coma is a state of deep unconsciousness where the patient shows no meaningful response to external stimuli. The
comatose patient has no verbal response, does not obey commands and does not open the eyes spontaneously or in
response to command.
Stupor is also a state of inaccessible consciousness without awareness, but the stuporous patient shows some
response to painful stimuli. Coma and stupor, and other levels of deep unconsciousness, are best graded on the
Glasgow Coma Scale. This has three elements: eye opening, and best verbal and motor responses to standard
stimuli (Table 1).
238
044
Performance Guidelines



CONDITION044. TABLE 1. Glasgow
Coma Scale score
CRITERIA SCALE
Eyes open

Spontaneously 4
To speech or verbal command 3
To pain 2
No response 1
Best motor response

- To verbal command

Obeys 6
- To painful stimuli

Localises pain 5
Withdrawal 4
Abnormal flexion (decorticate rigidity) 3
Extension (decerebrate rigidity) 2
No response 1
Best verbal response

Oriented 5
Confused conversation 4
Inappropriate words 3
Incomprehensible sounds 2
No response 1
TOTAL SCORE Range of 3-15
NOTE: Ascore of 15 represents a fully responsive and conscious patient. Ascore of 3, the
lowest level, a deeply comatose patient unresponsive to external stimuli. Ascore of
3 of course does not indicate 'brain death' or a 'vegetative state or any other
prognostic features as a single reading
Guidelines for neurological examination and conscious state chart are shown in Figures 1 and 2
1
.

1Reproduced fromHunt Pand Marshall V, Clinical Problems in General Surgery, Butterworths 1991


CONDITION 044. FIGURE 1.
Conscious state and head injury chart 1

1Reproduced from Hunt P and Marshall V, Clinical Problems in General Surgery, Butterworths, 1991.
239
044
Performance Guidelines

CONDITION044.FIGURE2.
Guide to recording neurological observation chart1

240
1Reproduced from Hunt P and Marshall V, Clinical Problems in General Surgery, Butterworths, 1991
Condition 045
Recent onset of poor distance vision in a 17-year-old male
AIMS OF STATION
To assess the candidate's knowledge of myopia, and ability to test visual acuity and distance vision using a
Snellen test chart.
The examiner must check the myopic patient's visual acuity in each eye before the examination commences.
The patient should have mild myopia and does not require any special instructions other than the knowledge
of having his eyes tested and providing appropriate responses. The doctor/candidate will explain and perform
the procedures.
EXPECTATIONS OF CANDIDATE PERFORMANCE
Exclude serious eye disease
The candidate should indicate that the following would be examined:
eyelids (ptosis, retraction of upper or lower lids)
conjunctiva (chemosis, injection, pallor)
cornea (ulceration)
anterior chamber (blood or pus)
sclera (jaundice)
orbit (tenderness, paraesthesia)
eyeball (intraocular pressure, glaucoma)
The candidate should indicate use of the ophthalmoscope to:
test the red reflex (to exclude cataract);
examine the retina (detachment, exudates, haemorrhage, new vessel formation);
examine the optic disc (bulging, blurring of margins): and
examine the macula (exudates).
The pupil will not be dilated. The candidate is expected to describe the proposed use of the opthalmoscope to
the examiner who will then say 'fundoscopic examination is normal'.
A thorough examination of the eye will also include instillation of fluorescein (cornea), dilatation of the pupils
(appropriate view of the posterior chamber), tonometry (intraocular pressure) and the pinhole test.
The pinhole test
A pinhole test card should be placed in an obvious position and used by the candidate for both eyes If visual
acuity is not improved by looking through a card with a 1 mm pinhole, the defective vision is not
solel y due to a refractive error. Macular degeneration, cataract and glaucoma will need to be
excluded. If the unaided visual acuity is less than 6/12, the patient should be referred to an
ophthalmologist.
241
242
045
Performance Guidelines



Test visual acuity
The term 'visual acuity' refers to the clarity of vision (from the Latin acuitas or sharpness). Visual acuity is expressed as
a proportionate relationship between the subject's vision and a person with normal vision. The subject is asked to
read from a Snellen chart. The chart, with letters of different sizes on each of its ten or eleven lines, is placed 6 metres
(20 feet) from the subject.
An individual with visual acuity of 6/6 (or 20/20 if feet are used) is just able to identify a letter whose height subtends 5
minutes of arc at the eye. Such letters are found on one of the lower lines of the Snellen chart. Acuity of this degree is
referred to as normal vision. Being able to discern letters below this line shows increased visual acuity and if the
individual can only decipher letters above this line, the visual acuity is diminished.
A near-sighted (myopic) individual will have better visual acuity at close distance, whereas a far-sighted (hyperopic)
person will have better visual acuity at far distance.
With the onset of presbyopia, near visual acuity diminishes and reading glasses are required.
Testing visual acuity
Visual acuity is measured using the Snellen chart, displaying letters of progressively smaller size. Visual acuity is
recorded in the form of a fraction but it is NOT a fraction in the mathematical sense of the word. The numerator
indicates the distance of the patient from the chart (e.g. 6 metres), and the denominator indicates the distance at which
the normal eye can read the line. Normal vision is 6/6 (20/20).
Visual acuity of 6/6 means that the test subject sees the same line of letters at 6 metres (20 feet) as that seen by a
person with normal sight at 6 metres (20 feet), whereas 6/12 (20/40) vision means that the test subject sees at 6
metres (20 feet) what a normal person sees at 12 metres (40 feet). Because the visual nomenclature used does not
reprsenta mathematical fraction, it is incorrect to say that 6/12 represents 50%of normal sight. In fact, for legal
assessment of visual impairment, 6/30 is regarded as a 50%impairment.
Visual acuity of 6/5 (20/15) vision is better than normal 6/6 (20/20). A person with 6/5 (20/15) vision can see objects at
6 metres (20 feet) that a person with normal vision sees at 5 metres (15 feet).
Note that the Snellen notation applies only to distance vision. Near vision is recorded using font size, usually in this
country the American point-type. Thus normal reading vision is N5 (5 point type). Newsprint is N8 (8 point type).
Levels of vision
6/6 Normal vision. This is the visual requirement for a fighter pilot
6/12 The visual requirement for a Driver's Licence in Australia
6/60 Legal blindness.
Testing should be done as follows:
The patient faces a Snellen chart at 6 metres distance. Formal testing requires a distance of 6 metres (20 feet),
necessitating use of a large room or a small room with a mirror to adjust for the distance. A 6-metre chart should
always be employed for formal visual acuity testing. Preliminary office testing can employ a 3-metre chart.
243
045
Performance Guidelines
Explain procedure to patient: Start reading at top (largest) line of letters. If only the top line can be read, acuity is
6/60. If the patient is unable to read the top line, the chart should be moved closer to the patient, 1 metre at a time,
until the top line can be read. If the top line can be read at 3 metres, this is recorded as 3/60. If the patient cannot
see 1/60, he is asked to identify a moving hand, and this is recorded as 'Hand Movements' (HM). If unable to see a
moving object, a light is shone in the eye and the patient is asked if he can appreciate the light, and this is recorded
as 'Perception of Light' (PL). If the patient can point to the light accurately, this is recorded as 'PL with accurate
projection'. If the light is not seen, the acuity is NLP (No Light Perception).
Visual acuity corresponds to the lowest line which can be read. The small numbers corresponding to the lowest line
which can be read give the denominator the distance in metres at which a person with normal vision can read
the line. In a line with 5 or more letters, the patient should correctly identify 3 letters to be regarded as having read
the line.
Examine each eye separately by using an occlusive card in a systematic way which includes asking the patient to
read the lines backwards when testing the second eye.
If the visual acuity is worse than 6/6, the candidate should perform a pinhole test. Ask the patient to hold a piece of
paper with a 2 mm hole in it over the uncovered eye. This manoeuvre utilises the 'pinhole camera effect' and results
in an improvement in visual acuity if a refractive error is the cause of the diminished acuity. The patient should be
referred for refraction and prescription of glasses.
The candidate should give findings to examiner in the conventional way, normal vision being 6/6. The smaller the
ratio, the poorer the vision (6/12, 6/24, etc).
Explain problem
Nature of myopia.
Management options.
KEY ISSUES
Exclusion of serious eye disease with ophthalmoscope and pinhole test.
Correct use of the Snellen test chart.
Accuracy of examination (compare with examiner's findings).
Diagnosis must state myopia, or short-sightedness or near-sightedness.
Patient counselling/education cause, treatment, need for periodic check of intraocular tension when over 40
years of age.
CRITICAL ERRORS
Failure to exclude serious eye disease.
Failure to mention myopia as a possible diagnosis.
244
045
Performance Guidelines



COMMENTARY
A comprehensive history and careful physical examination will provide a diagnosis in most common ophthalmic
disorders.
Although unlikely in this case, the possibility of the patient's complaint of recent impaired vision being due to a
serious cause (namely retinal detachment, glaucoma, cataract or macular degeneration) should be considered by
the candidate Ophthalmoscopic examination and the use of the pinhole test cover these concerns at this stage of
assessment of vision.
Myopia (near-sightedness or short-sightedness) is a common inherited condition which in most cases is due to the
axis of the eyeball being too long so that the visual image is focused in front of the retina. Less often the refractive
power of the lens is too strong.
The condition is readily corrected by the use of a concave (minus) lens. Onset can be in childhood, but more
commonly in late teens. The condition tends to worsen in early adult life and then stabilises.
The prescription:
The refractive error of the eye can be expressed in numeric terms. The power of the lenses necessary to correct
vision is measured in units called dioptres (see below). The first number in a spectacle prescription designates the
amount of myopia (minus numbers) or hyperopia (plus numbers). The second number (if present), indicates the
amount of astigmatism. The third number indicates the axis of the steepest meridian of the cornea (e.g. +3.00/-2.50
X 170'). The fourth number is the additional correction needed to bring the focal point of the eye to the reading
distance.
The dioptre is the unit of measurement of the strength of a lens. A lens deviates light and the amount of deviation is
proportional to the amount of curvature and the density of the lens. A lens of power of 1 dioptre has a focal length of
1 metre (i.e. parallel rays of light are brought to a focus 1 metre from the lens). As the refractive power of a lens
decreases, the focal length increases. The strength of a lens = 1/focal length. Thus a 4-D lens has a focal length of 'A
metre. The power is a negative number of a concave lens (myopia, near-sightedness) or a positive number for a
convex lens (hyperopia, far-sightedness).
The corrective lens can be prescribed by an optician, although an initial assessment by an ophthalmologist is
preferable to exclude any other cause of visual impairment, especially retinal detachment and macular degeneration,
both more common if myopia is severe.
Contact lenses can be worn to correct myopia, without the risks of surgical correction.
Corrective operations (excimer laser surgery) can produce excellent results by altering corneal curvature and thus
the refractive power of the eye. This procedure is not without significant risk. Otherwise glasses will need to be worn.
Reading is not affected much until middle age. Myopia can affect the accurate measurement of intraocular pressure,
and therefore intraocular pressure should be checked periodically to detect chronic open angle glaucoma which is
asymptomatic in the early stages. This applies to all patients, and particularly to myopes.
The 'acute red eye', although not relevant to this case, is an important and urgent clinical problem. Most of the
causes of red eye (conjunctivitis, foreign body, inflammation ulceration, glaucoma and subconjunctival
haemorrhage) are associated with pain and/or trauma and can be excluded on the history alone in this patient.

Suddenlossofvisionisusuallyassociatedwithavascularorneurologicalproblemandagain,
these types of problem are not relevant in the present case. Diabetes mellitus must be
considered,aspresentationofthisdisorder withanophthalmologicalcomplication mayoccur.
Somediabeticspresentwithcataract;otherswithmatureonsetdiabetesmaypresentwithpoor
centralvisionduetooedemaofthemacula.Theassessmentoftheophthalmoscopeandpinhole
testshouldexcludetheseriousdisordersthatcanbeassociatedwithgradualvisualloss(and
otherssuchasretinaldetachment,glaucoma,cataractandmaculardegeneration).

CONDITION 045. FIGURES 1 AND


2.Visualacuitycharts(nottoscale)
245
246
046
Performance Guidelines



Condition 046
Apainfulrashonthetrunkofa45-year-oldchild-careworker

AIMSOFSTATION
Toassessthecandidate'sapproachtoapatientwithadermatomalrashfromherpeszosterplusweightloss
andtiredness,whichcouldbeincidentalbutmaybeassociatedwithunderlyingmalignancy.Thesesymptoms
needtobefurtherassessed.

EXAMINERINSTRUCTIONS
Theexaminerwillhaveinstructedthepatientasfollows:
Youareachild-careworkerinakindergarten.Youaresingleandlivebyyourself.
Openingstatement:
' I' vehadapai ni nmyl owerchestandnowtherei sthi srash.'
Answerquestionsaboutyourconditionasfollows:
Youhavebeenfeelingabitunwellforafewdays.
Youhavehadaburningpainoveryourlowerchestandflankforafewdays.
You noticed today that you have developed a blistery rash that runs in a line around your chest and
abdomenintheareawherethepainstarted.
Inaddition,youhavelost6kginweightoverthepastfewmonthsandhavebeenfeelingmoretiredthan
usual (you have nothing to add to this general statement. You have noted no disturbance to any body
systemfunction).
Youhavehadnoseriouspastillnesses,thereisnorelevantfamilyhistory.Youhavenoallergiesandare
onnomedications.
Describethepainandtheskinrashwithoutprompting.Donotvolunteertheweightlossandassociatedrecent
tirednessunlessquestionedfirstaboutHowhasyourheal thbeeni ngeneral ?' orsomethingalongthose
lines.Youhavebeenconsideringhavingacheckupbuthavenoothersymptoms,andyouhadnotconsidered
thattheremightbesomethingseriouslywronguntilthepainandthisrashappeared.

EXPECTATIONSOFCANDIDATEPERFORMANCE
History
~Typicalhistoryandrash(seeFigure1)ofherpeszosterwithprodromalpreherpeticneuralgia.
Diagnosis
~Mustmakediagnosisofherpeszoster.Mustshowconcernoverrecentweightlossandtiredness.
Initialmanagement
~Treattherashwithsymptomaticmeasuressuchascalamineorcoldcompressesand adryinglotion.
~Useanalgesicswithorwithoutcodeine.
~Treatwithantiviralmedicationsifpatientpresents(asinthisinstance)withinfirs!72hoursoftherash
aciclovir,famciclovirorvalaciclovir.
046
247
Performance Guidelines



~ Monitor for the development of postherpetic neuralgia which may require further management.
~ Examine patient and perform investigations for any possible precipitating cause. In this case, the weight loss and
tiredness demand further investigation (no details are required at this stage).
Patient education and counselling
~ Explain the cause to the patient (i.e. relationship of herpes zoster to varicella [chicken pox]).
~ Explain that the condition is only mildly contagious, but that chickenpox can be acquired by those persons in close
contact with the patient who have not previously had chicken pox. Therefore appropriate infection control
measures need to be taken including management of occupational and community contacts (for example, she is a
child-care worker in a kindergarten and young children and babies should not be exposed to vancella zoster virus).
After 5-6 minutes, if the candidate has not discussed these issues, the examiner will ask
~ 'Are there any unusual features of the condition in this patient?
~ 'How would you manage this particular patient?'
KEY ISSUES
History-taking must elicit weight loss and tiredness.
Diagnose herpes zoster/shingles
Management must consider use of aciclovir or other related antiviral drugs.
Must advise further assessment regarding weight loss and tiredness and discuss implications of infectivity.
CRITICAL ERRORS
Failure to diagnose herpes zoster.
Failure to consider the possibility of an additional underlying cause in this patient.
Failure to assess implications for contacts in community and work settings.
COMMENTARY
Herpes zoster (shingles) is caused by reactivation of varicella zoster virus (VZV) acquired originally through primary
infection with chicken pox.
The condition is more common in people over 50 years of age.
The virus is found in the dorsal root ganglion. In most cases the reason for reactivation is unknown, although
occasionally this can be related to an underlying malignancy such as a lymphoma, leukaemia or
immunosuppression including HIV infection.
Occasionally patients may get rare complications including meningoencephalitis.
Post-herpetic neuralgia is an important sequel. The incidence of post-herpetic neuralgia increases with age,
affecting around 30-50%of adults aged 70-79 years.
248
047
PerformanceGuidelines



Condition 047
Acutelowbackpainandsciaticaina30-year-oldman

AIMSOFSTATION
Toassessthecandidate'sabilitytodiagnoseandtreattheproblemofacuteexertion-relatedlowbackpain
andsciatica.

EXAMINERINSTRUCTIONS
Theexaminerwillhaveinstructedthepatientasfollows:
Youareaself-employedlandscapegardeneraged30years.Youhaveconsultedthisdoctorbecauseofthe
suddenonsetofseveredisablingpaininyourlowerbackyesterdaywhichmoveddownyourleftthighandleg
intoyourfoot.Itcameonwhenyouliftedaheavyrockandyouhavenotbeenabletowork.Youcouldnot
sleeplastnightdespitetakingPanadeinetablets.Ithurtstomoveandtocough.
Youusuallykeepinexcellenthealthwithnoseriousmedicalproblemsinthepast.
Thedoctorhastakenyourhistoryandexaminedyou.Hewillexplaintheproblemandwhatyouhavetodo.
Showconcernabouthowyouaregoingtobeabletoworknowandinthefuture.
Appeartobeinseverepain-situncomfortablyberestless.
Statedissatisfactionwithlevelofpainreliefyoucouldnotsleeplastnight.
Expectthedoctorto'dosomething'togetridofthepain.
Resistadvice(irrationally)nottogotoworkeveninasupervisorycapacitybecauseofimportantjobs
needingtobefinished.
Becomecompliantifthedoctorexplainsthesituationandgivesappropriateadvice.
Questionstoaskunlessalreadycovered(candidate'slikely
responseisdetailedinbrackets):
What has happened t o my back?' (Explain'slippeddisc'intervertebraldiscprolapsewith
herniationofnucleuspulposususeofadiagramcanbehelpful).
How does t hi s happen?' ( Ver y common,relatedtostressonbackwhilstlifting).
How l ong wi l l I be away f r om wor k?' (Dependsonprogress.UsuallysettlesrapidlywithI
adequaterest.Ifso,offworkfor1-2weeks.Ifpaindoesnotsettle,mustbeinvestigatedbyCTorMRI).
' Shoul d I see a chi r opr act or ?' (Definitelynotatthisstage;manipulationmayworsenthe
condition).
Wi l l I be abl e t o l i f t heavy obj ect s i n t he f ut ur e?' (Giveadviceonhowtoliftwithgooi
self-maintenancestrategies).
' Wi l l I al ways have a bad back?' { No, likelihoodofrecoveryisgood).
'Do/need t o see a Speci al i st ?' (Notatthisstage,willbearrangedifsymptomspersist!
' Can' t I have an oper at i on t o f i x i t and r el i eve t he pai n?' (Usuallynotnecessary,butwil
dependonprogress).
' What el se can I t ake f or t he pai n?' (Panadeineforte
03
).
047
Performance Guidelines



EXPECTATIONSOFCANDIDATEPERFORMANCE
Diagnosisandexplanationofcondition
Anatomyoflumbarspine(thisisanL5/S1levelproblem,involvingtheS1nerveroot)
Causesofpainparticularlydiscprolapsewithnerveimpingement/irritation(radiculopathy).
Expectedcoursebothshortandlongtermmostresolvecompletely
Adiagramwouldassist.
Immediatemanagement
Adequaterestisessential(3-4daysrestathome,butupandaboutastolerated)
Pain-relievingmedicationPanadeine,Panadeineforteorsimilar(includingNSAID).
Subsequentphysiotherapyandback-strengtheningexercises.
Indicationsforfurtherinvestigationlackof,slow,orincompleteresolution.ThenneedsCT
orMRI.
Avoidanceofmanipulation.
Gentle traction may have a place in treatment if progress slow would be advised after
specialistreferral.
Emphasisonpositiveapproach.Prognosisforrecoverywithinafewweeksisgooddespite
anklejerkbeingaffected.
Need for investigation this is particularly important if there is no improvement, or there is
continuingevidenceofneurologicormuscleweakness(CTacceptable,MRIpreferred,plain
X-raygiveslimitedinformationonly).
Physiotherapy stretching and arching active mobilising exercises appropriate once initial
symptomsease.
Orthopaedic,neurologicalorrheumatologicconsultationwillberequiredforlackof
resolution.
Preventivemeasures
'Back education' including advice regarding bending and lifting, and the value of walking,
swimming.
Futuremanagement
Reassessmentinshortterm(2-3days).Thisisessential.

Abilitytodeterminethelikelycauseofthesciaticaandtoexplainthecausetothepatient.
Adequateknowledgeofthemanagementofapatientwithacutesciaticaincludingwhatfurther
investigationsorreferralarerequiredandwhentheseshouldbedone.
Abilitytoadvisethepatientaboutworkpracticemodificationsrequiredtopreventarecurrence
oftheproblem.
Abilitytoadviseearlyrestandshorttermreview.
249
250
047
Performance Guidelines



CRITICAL ERRORS
Failure to make correct diagnosis of a likely disc lesion.
COMMENTARY
Low back pain is a very common problem in Australian society. The incidence increases with age and is more
common in manual workers than sedentary workers.
A major problem in spinal assessment is the fact that there is often a poor correlation between clinical presentation
(the patient's history and examination finding) and the imaging findings. Imaging abnormalities will be found with
increasing frequency in individuals with or without accompanying symptoms from their third decade onwards.
Back pain, acute and chronic, is thus one of the most common of all conditions encountered but precise pathology is
very frequently lacking. The portmanteau and nonspecific term 'Mechanical low back pain' is useful in that it codifies a
very common condition from which almost all individuals will suffer at some time of their lives. In such instances the
precise pathology is indeterminable and no specifically diagnostic imaging or other test is available.
Back pain may (or may not) follow an identifiable injury or strain as occurred in this patient Pain is usually
self-resolving over a period of days or weeks, but may become recurrent, relapsing or chronic, and is influenced by
cultural, psychological, socioeconomic and other personal factors in its incidence and persistence. Against such a
background, it is hardly surprising that the condition and its preferred treatment remain controversial.
The outcome of physical treatments such as massage, manipulation, heat, light, sound/ultrasound, electricity and
magnetism (and surgery) are each difficult to separate from placebo and are prone to fashion and fetish. Clinical
studies are possible and literature search and meta-analysis can be helpful and reveal (for example) that laser
treatment of low back pain is free of concerning side-effects, but gives short term outcomes no different from placebo,
and is expensive and not cost-effective.
Distinguishing true radicular sciatic pain ('sciatica') due to nerve root compression requires symptoms of pain of
lancinating or cramping type, extending usually from low back and buttock down the leg to foot and toes
corresponding to sensory disturbance within the dermatomal distribution of appropriate nerve roots (most commonly
L5 or S1 ), exacerbated by straining or coughing, with positive nerve tension signs, and sometimes with objective
motor weakness and sensory loss corresponding to the appropriate motor nerve root. Such a constellation of
objective signs (as in this patient) is virtually pathognomonic and diagnostic of nerve root foramen compression (from
intervertebral | disc prolapse, facet joint arthropathy or other encroachments on the relevant nerve root or spinal
canal). Confirmation of the diagnosis can usually be made by noninvasive imaging, of which MRI is the most accurate.
Persistence of unrelieved pain after one month is an | indication of the need for a full history and examination
(including diagnostic imaging), concentrating on the search for pointers of more serious pathology (malignancy,
referred back pain from intra-abdominal lesions, bone infections, or cauda equina symptoms such as interference with
bowel or bladder control).
This case scenario has been chosen to exemplify the classical syndrome of nerve impingement radiculopathy, the
most likely diagnosis being compression from an intervertebral disc prolapse between L5 and S1.
047
Performance Guidelines
Bycontrast,mostcasesofsimplemechanicalbackpainduetomusculoligamentoussofttissuestraininjuries
willresolvewithinonetotwoweekswithexplanationandencouragement,earlymobilisationwithoutbedrest(
' don' t t akebackpai nl yi ngdown) andsimpleanalgesics,aidedwhereindicatedbyashortcourseof
physicaltherapyconcentratingonearlymobilisationandanactiveexerciseprogram,andpatienteducation
regardinggoodbackstrategies.
Plain radiographs for patients with persisting chronic pain rarely are of clear cut diagnostic value, but may
show loss of disc height, gas formation in the nucleus pulposus, adjacent vertebral marginal sclerosis and
osteophytes, or other radiological signs of lumbar vertebral spondylosis affecting the facet joints. However,
similarradiologicalsignsorevidenceofminorspondylolysisorspondylolisthesisarealsopresentcommonlyin
nonsymptomaticmiddle-agedorelderlypeople.
MRIistheinvestigationofchoicefordefiningspinalpathologywhensurgeryisbeingconsidered.Surgeryis,
however, indicated in only a very small percentage of patients with low back pain and it is quite rare to
demonstratetreatablenewpathologyinpatientswithchroniclowbackpain,whichhaslastedformorethana
year.
Associated job dissatisfaction, depression, obesity and socioeconomic deprivation are commonly found in
such instances. Long-term treatments with laser, shortwave diathermy, ultrasound, acupuncture,
transcutaneouselectricalnervestimulation,formalphysiotherapyorchiropractichavenotconvincinglybeen
demonstratedtohaveotherthanplaceboeffects.
Theeffectsofrepeatedimage-guidedfacetjoint,epiduralornerverootforaminalinjectionsoflocalanaesthetic
or corticosteroids are also disappointing in the long-term. Percutaneous semisurgical procedures
(radiofrequencyrhizolysis)alsoseemoflittleconvincinglong-termvalue.
Surgicaltechniqueshaveimprovedinthesmallgroupofpatientsrequiringsurgery,andreleasesurgeryfor
focalmajornerverootcompressionsconfirmedbyimagingcanbedramaticallyeffective.Bycontrast,spinal
fusiontechniquesforchroniclowbackpainarevarious,resultscanseldombeguaranteedandpersistingpain
aftersurgeryiscommon.
251
048
Performance Guidelines



Condition 048
Feverandarecentrashina30-year-oldman

AIMSOFSTATION
To assess the candidate's diagnostic approach to a young man presentingwith fever and rash of 48 hours
durationwithsignsofsplenomegalyandlymphadenopathy.


Theexaminerwillhaveinstructedthepatientasfollows:Openingstatement:
' Ithi nkImayhavedevel opedani nfecti on.'
Followwith:
~Youhavebeenfeelingunwellfortwodays.
~Youhaveafeverandasorethroat.
~Youhavealsodevelopedarashoverthepastonetotwodays.
~Therashisalloveryourbody,andisespeciallyapparentonthefaceandtrunk.
Inresponsetoquestionsthedoctormayask:
~Therashisnotitchy.
~Youhave'achesandpains'throughoutyourbodyinthelegs,armsandback.
~Youhaveaheadacheandbrightlightshurtyoureyes.
~Youhavepreviouslyconsideredyourhealthtobegood.
~Youhavehadnoseriouspastillnessesorfamilyhistoryofrelevance.
~Youhavenoallergiesandtakenomedications.
~Nohistoryofmentalillness,nohistoryofbloodtransfusion.
~Youdonotuseinjectabledrugs.
~Youhavebeeninasexualrelationshipwithanothermanfortwoyearsandhavebeenhavinganalsex
withoutcondomsforafewmonthsnow.Youhavealsohadanumberofcasualsexualrelationshipsin
thepastsixmonths.
~YouhaveneverhadanHIVtestinthepast.
Answer the doctor's questions honestly. Be open about your homosexuality but do not reveal this without
specific questioning by the doctor. Be very concerned about the possibility of HIV infection when this is
mentioned,andanxioustoproceedwithinvestigationsatonce.
EXPECTATIONSOFCANDIDATEPERFORMANCE
Thekeytodiagnosisisthehistoryofunprotectedanalsex.
Approachtopatient:thehistoryshouldbetakenanddiagnosticpossibilitiesdiscussedinamatteroffact
and nonjudgmental way. Support forthe patientshould be shown whenthepossibleseriousness of his
conditionisdiscussed.
History:mustobtaindetailedsexualhistory.
Explanation to patient: his condition is possibly due to one of a number of viral infections, such as
infectiousmononucleosis.However,themostlikelyinfectioniswithhumanimmunedeficiencyvirus(HIV)
andthismustbeconfirmedorexcludedbylaboratoryinvestigations.ThesemustincludeHIVserology.

252
048
253
Performance
Guidelines

InformedconsentisrequiredforHIVtesting:ensurepatienthaspretestcounselling.
~Laboratorytestsmaynotbeclearduringtimeofacuteseroconversionillnessandmayrequire
consultationwithHIVlaboratoryand/orspecialistunittomanagehiscondition.
~IfHIVinfectionisconfirmed,referraltoaspecialistinfectiousdiseasesunitisrequired
formanagementduringseroconversionillness.
~Othertestsareindicated(seedifferentialdiagnosis)andthecandidatemaymention
otherviralcausesofthispatient'sfever,rash,sorethroat,lymphadenopathyand
splenomegaly.
~Theexaminershouldinterveneatthisstageandsay:' Weshoul dnowdi scussthedi fferenti al
di agnosi sandappropri atei nvesti gati ons.'
DifferentialdiagnosisapartfromHIV:
~Epstein-Barrvirusinfection;
~secondarysyphilis;
~toxoplasmosis;
~rubella;
~cytomegalovirus(CMV);
~herpessimplexinfection;
~disseminatedgonococcalinfection;
~hepatitisA.B,C,DorE;and
~otherviralinfections.
Investigationsthesearerelatedtothedifferentialdiagnosisandshouldinclude:
~fullbloodexaminationandEpstein-Barrserology:
~testsforrubella,CMVinfectionandtoxoplasmosis;
~VenerealDiseaseResearchLaboratories/syphilisserology;
~liverfunctiontests;and
~testsforhepatitisA,B,C,DorE.

KEYISSUES
Historymustincludesexualhistory
InvestigationsmustincludeHIVserology.
DifferentialdiagnosismustincludeHIVinfection.
ApproachtopatientmustdiscussinformedconsentforHIVtesting.

CRITICALERRORS
FailuretoconsiderHIVinfectionasalikelycauseofthispatient'spresentation.
FailuretodiscussinformedconsentforHIVtesting.
254
048
Performance Guidelines



COMMENTARY
Diagnosis of HIV infection requires a careful history to identify potential high-risk behaviour and recognition of the
constellation of clinical symptoms and signs. The rash of acute HIV infection is usually an erythematous,
maculopapular rash.
From 40-90%of patients who have acquired HIV infection will develop an acute febrile illness within the first six weeks
of infection, often sooner. Common symptoms include: fever, night sweats, malaise, myalgia, arthralgia, headache,
photophobia and sore throat. Neurological manifestations including headache and photophobia are common as well
as transient neurological signs including peripheral neuritis and other central nervous system manifestations. These
symptoms usually last for less than two weeks.
Other nonspecific viral sequelae such as mucosal ulceration, desquamation and herpes simplex may also occur.
Acute symptoms are self-limiting. The condition resembles infectious mononucleosis but is seronegative for infectious
mononucleosis. Chronic lethargy, depression and irritability may persist after initial illness. Key to diagnosis in this
patient is checking for recent risk exposure history of unprotected oral or anal sex. reuse of contaminated needles or
other exposure, such as occupational exposure.
049
255
Performance Guidelines



Condition 049
A heart murmur in a 4-year-old boy

AIMS OF STATION
Toassess the candidate's ability to diagnose an innocent heart murmur in a young child and to advise a concerned
parent.

EXAMINER INSTRUCTIONS
Opening statement:
'What is the matter with my child?'
The examiner will have instructed the parent as follows:
You are the parent of an only child, puzzled and concerned at being told that the child may have something the matter
with his heart. Be prepared to accept reassurance if the explanation is adequate. If not, insist on referral and ask what
tests might be performed.
EXPECTATIONS OF CANDIDATE PERFORMANCE
This is almost certainly an innocent murmur. No concerning symptoms or signs are present which might suggest an
alternative diagnosis. Parents need reassurance that the child is normal and that normal physical activity is allowed.
Referral to a paediatrician/paediatric cardiologist is only indicated if parents wish it, or seem unconvinced. The
consultant would consider echocardiography.
It would be reasonable do to a chest X-ray and ECG, depending on degree of parental concern. This is unlikely to
show any abnormality and may be reassuring, and may then render unnecessary further referral to a cardiologist.

KEY ISSUES
Ability to assess confidently the features of an innocent heart murmur.
Avoidance of unnecessary extensive investigation.

CRITICAL ERROR - none defined

COMMENTARY
Cardiac murmurs in young children are very common. It is estimated that careful auscultation under ideal
circumstances will detect an innocent soft murmur in over 50%of normal four-year-olds. Hence medical facilities
would be overwhelmed if all of these murmurs were referred for specialist assessment.
Primary care physicians should be confident in distinguishing innocent functional murmurs from those that are
associated with an organic heart lesion. Rheumatic fever in our community is unusual these days unless practising in
areas where large numbers of Aboriginal or Torres Straight Islander peoples are treated. So the usual task is to
differentiate an innocent murmur from one due to an organic heart lesion, most likely of congenital origin.
256
049
Performance Guidelines



As in many situations in paediatrics, the diagnosis can usually be determined by a careful history and examination. The
child with an innocent murmur, is well and thriving, has a normal exercise tolerance, is not cyanosed, and does not
suffer from recurrent chest infections.
Physical examination reveals:
a soft midsystolic murmur, which is an almost musical high-pitched murmur at the base of the heart with no
radiation;
the murmur varies with posture and respiration, and has no associated thrill; and
the murmur has no diastolic component.
In comparison, an organic murmur may be:
loud;
associated with a palpable thrill;
radiating either to the axilla or neck;
associated with cyanosis; and
associated with significant symptoms.
In determining the possible aetiology, the clinician should seek information along these ines to determine if any of
these features exist in the history, and must perform a thorough examination.
If all features indicate an innocent heart murmur, no investigations are warranted. The parent should be reassured of
the innocent nature of the murmur and that the practitioner will continue to observe the child until the murmur
spontaneously disappears, usually between the ages of five and seven years.
If the parents are still concerned despite adequate explanation and reassurance, referring the child to a paediatrician
who is skilled in assessing murmurs is acceptable. If necessary, the paediatrician will refer the child to a paediatric
cardiologist for full cardiological investigations, including echocardiography.
050
257
PerformanceGuidelines



Condition 050
Aknifewoundtothewristofa25-year-oldman

AIMSOFSTATION
Toassessthecandidate'sabilitytodiagnosetendonandnerveinjuriesinadeepwound

EXAMINERINSTRUCTIONS
Theknifewoundisacrossthewristjustabovethecreaselineasintheillustrationandthe
candidatecanobservethis.
Theexaminerwillindicatetothecandidate
' Thebl eedi nghasbeenstopped bythedressi ng. Proceedtoyour exami nati ontoascertai nthe
extentofi nj urydescri bi ngyourfi ndi ngstome,theexami ner.'
Theexaminerwillhaveinstructedthepatientasfollows:
YouhavepresentedtotheEmergencyDepartmentwithaknifewoundtotheleftwristproduced
by an assailant after an argument in a pub. You lifted your arm to protect your face and he
slashedyourwristwithaknife.Itbledalotatfirst,butyourfriendsreducedthisbylocalpressure
and the ambulance staff put on a dressing which controlled the bleeding, but which you think
madeyourhandfeelnumb.Youareunabletomoveyourfingersfreely.
Specifically,youhavelostsensationandmusclepowerasfollows:
Sensationtotouchandpinoverthewholepalmaraspectofyourhand,fingersandthumb.The
numbnessandlossoffeelingextendsontothebackofthefingersandthumb,overthenails
andtheendofthejoint.
You should hold your hand as depicted in the illustration so all the fingers and thumb are
stretchedoutstraight.Whenaskedtoflexyourfingersandthumb,youareunabletodosoat
theendtwojointsofthethumb,andunabletobendanyofthethreejointsofyourfourfingers.
Youareabletostretchthemoutstraightagainifthecandidatebendsthemforwards.
You also cannot flex your wrist (bend it forward), but are able to extend it (bend your wrist
back).
Ifaskedtoputyourthumbacrosstotouchtheotherfingers,youcannotmovetheotherfingers
towards the thumb (cannot bend fingers and thumbs inwards towards the palm); and you
cannotmovethethumbacrossthepalmtowardsthebaseoftheringandlittlefingers.
If asked to do the movement of abduction of the thumb by lifting the thumb away from the
palm,youcannot.
Youcanonlymovethethumboutwardsandawayfromtheotherfingersintheplaneofthe
palm.
Ifaskedtoholdacardbetweenyourfingers,ortomoveyourlittlefingerawayfromtheothers,
youcannot.
258
050
Performance Guidelines



EXPECTATIONSOFCANDIDATEPERFORMANCE
Thecandidateisexpectedtodiagnoseaccuratelythedeepandextensiveinjuryto:
themediannerve
theulnarnerve.
allofthefollowingflexortendons,whichhavebeenseveredabovethewrist:
~wristflexors:f l exor car pi r adi al i s, ul nar i s, ( pal mar i s l ongus)
~fingerflexors
- f l exor di gl t or umsuper f i ci al l stoallfourfingers(normallyflexesproximalinter-phalangeal
[PIP]joint)
- f l exor di gl t or um pr of undus toallfourfingers(normallyflexesdistalinterphalangeal[DIP]
joint)
~thumbflexor:f l exor pol l i ci s l ongus (normallyflexesthumbinterphalangeal[IP]joint).
Arteriesprobablyoneorboth,buttheseinjurieshavenotdisturbedtheviabilityofthehand.
NeurologiceffectsParalysisofallthenarandhypothenarsmallmusclesofthehandpreventing
~palmarabductionofthethumb( abduct or pol l i ci s br evi s mediannerve)
~abductionoflittlefinger( abduct or di gi t i mi ni mi ulnarnerve)
~flexionofmetacarpophalangeal[MP]jointsoffingers(lumbricals,interosseiulnar
andmediannerves);andofMPjointofthumb(flexorpollicisbrevis)
~abduction/adductionoffingers(interosseiulnarnerve)
~oppositionmedianandulnarnerves
~ulnaradductionofthethumb(adductorpollicisulnarnerve)
Sensorylossisofcombinedmedian/ulnarnerveinjury.
Knowledgeable candidates may recognise that the dorsal cutaneous branch of the ulnar nerve has been
spared.
Candidates are expected to conduct a logical and systematic examination to detect nerve, tendon and
vascular injury. Candidates should achieve the diagnosis of combined median and ulnar nerve and flexor
tendon injury. Knowledge of all of the individual muscle groups is not expected but candidates should be
aware of the effects of median and ulnar nerve division and the appropriate tests (sensory and motor) to
detectthese.Candidatesshouldalsobeexpectedtorecognisethatthefailuretoflexthedistaljointsofthe
fingersandthumbareduetoconcomitanttendoninjury,andnottotheeffectsofnervedamagetomedianand
ulnarnervesatthelevelofthecutjustabovethewrist.
Attheendofthecandidate'scommentary,oratsevenminutes,theexaminerwillask:
' Why i s he unabl e t o hol d a car d bet ween hi s f i nger s?'
~Answer:Becausetheulnarnerveinjuryhasparalysedtheinterossei.
' Why i s he unabl e t o f l ex t he end j oi nt s of f i nger s or
t humb?'
~Answer:Becausethelongtendonshavebeendamaged.
KEYISSUES
Abilitytocorrectlyidentifythestructuresdamaged,byanappropriatelyfocusedexamination.
050
259
Performance
Guidelines

CRITICALERROR
Failuretoidentifythecombinationofnerveandtendoninjury.

COMMENTARY
Cuts to the wrist and handsfrom knives, glass breakages and other sharpitems need carefulevaluationto
identifydamagetoimportantunderlyingstructures,particularlymajorbloodvessels,nervesandtendons.
Ofthethreemainnervesofthearm(median,ulnarandradial),themedianandulnarnervesruntothehandon
thevolaraspectofforearmandwrist,carryingmotorfibrestotheintrinsicshortmusclesofthehand,sensory
fibres to the vital grasping surfaces of thumb and fingers, and autonomic sympathetic fibres subserving
sweatingandvasomotorresponses.Themediannerve,asitsnameimplies,runsamidlinecoursethroughout
the forearm and lies in close proximity to the tendon of flexor digitorum superficialis running to the middle
finger. The important sensory and motor branches are given off after the nerves have entered the hand by
passingunder(mediannerve)oraround(ulnarnerve)thecarpaltunnel.
Theulnarnerveliesmoredeeplyontheulnarsideofforearmandwristflankingtheulnararteryonthesurface
of the deep long flexor muscle (flexor digitorum profundus). Clearly both nerves were at risk from the cut
illustrated. The superficial terminal branch of the radial nerve, by contrast, is at this stage a much less
importantnerve,withnomotorfibres.Itrunstothebackofthehandandfingersalongtheradialsideofthe
forearm,supplyingsensationtothedorsumofhandandonlythebacksoftheradialthreedigitsforashortway
alongtheirlength.Theulnarnervegivesadorsalsensorybranchtosupplytheotheroneandahalfortwo
ulnardigits.Ifyouextendyourthumbandtensethetendonofextensorpol l i ci sl ongusyoumaybeableto
feeltheterminalbranchoftheradialnervecrossingthesnuffboxsuperficialtothetauttendonbyrunningyour
fingeralongthetendon.Theradialnerveisthusunlikelytohavebeenatrisk.
Testingfordamagetotheothertwonervesisusuallyeasyandrapidwithacooperativepatient.
Mediannerve
Thepulpoftheindexfingerisvirtuallyalwayssuppliedbythemediannerve.Canthepatientfeelyoutouch
here(withabluntpin,orwool,oryourownfinger)?
Thepulpofthelittlefingersimilarlyissuppliedbytheulnarnerve,sorepeattheprocesstheretocheckforulnar
nervedamage.
To confirm your suspicions, now check the motor functions the median nerve first. In checking for motor
paralysisduetonerveinjury,thinkofthemusclesinnervatedbythenervedistaltotheinjury,findanaction
whichisperformedbyonemuscleonly,checkthataction,andifpossible,seeandfeeltheresponsiblemuscle
contracting.
For median nerve, abductor pol l i ci s brevi s (APB), the short abductor of the thumb, is virtually always
suppliedbythemediannervejustafterenteringthepalm.Youtestitsactionbyaskingthepatienttomovethe
thumbdirectlyupwards,withthepalmflat,awayfromthepalmandtheotherfingers,keepingthethumbinside
themarginoftheindexfingersothethumbpushesstraightupagainstresistance.
260
050
Performance Guidelines



That movement is palmar abduction. Only APB can perform it, you can test the power and you can see and
feel the muscle contract. The branch to the muscle is given off immediately after the median nerve enters the
palm after passing under the flexor retinaculum. In patients with longstanding carpal tunnel syndrome with
median nerve compression affecting the motor fibres as they go through the tunnel the muscle may waste and
atrophy as illustrated elsewhere in the book. But don't of course expect wasting, or the deformities arising from
such wasting, in an acute injury.
The ulnar nerve has a much greater effect on the motor function of the hand than does the median it
supplies at least a dozen important small muscles, compared to the handful from median (conversely the
sensory loss from median nerve injury is much more significant than from ulnar nerve damage).
Use two tests here they will help remind you of the muscles involved.
Can the patient abduct the little finger away from the other fingers against resistance? This is done by
abductor digiti minimi, supplied by ulnar nerve by its deep palmar branch.
Can the patient hold a piece of paper between outstretched fingers? This is done by the interossei, all
supplied by the ulnar nerve.
There are many other tests for other muscles supplied by the ulnar nerve pinch test for adductorpollicis
(Froment sign), the deficiencies seen in opposition (Sunderland sign), and so on but further tests are not
needed, having already made the diagnosis of an injured ulnar nerve at wrist level or above, corresponding to
the site of the cut.
In cooperative patients, this is easy and conclusive but supposing the patient was drunk and uncooperative,
or stuporous, or a young child, and cannot or will not cooperate with you.
In this case you can still diagnose a nerve injury from effects on the sympathetic efferent fibres. If the nerve
carrying them is cut, they too will be paralysed, and the affected skin in the distribution of the nerve will be dry
and unable to sweat. This can be demonstrated elegantly by sprinkling a starch powder over the skin and
observing the colour change. Also check if any differences can be seen or felt distal to the cut compared to the
other hand a small point but sometimes quite helpful.
Next check for damage to the next important group of structures the long tendons to
the thumb and fingers and the tendons to the wrist. These lie in three layers from superficial to deep.
The wrist flexors:
~ Flexor carpi radialis the largest and most visible tendon
~ Flexor carpi ulnaris the most ulnar sided
~The inconstant palmaris longus between them.
You can check these by asking the patient to flex his wrist. If he can do so. he may of course be using deeper
finger flexors, which because they cross multiple joints, can act as accessory flexors of the wrist. In this
instance no wrist flexion is possible.
The superficial and deep long flexor tendons to thumb and fingers. Flexor digitorum superficialis (FDS) and
profundus (FDP), each with four tendons, and the solitary tendon of flexor pollicis longus (FPL).
~ FDS This group of four tendons to index, middle, ring and little fingers flexes the proximal
interphalangeal joints. It is difficult to test their independent action becauseof the last and deepest layer
flexor digitorum profundus; these can act as accessory flexors of the more proximal joints, which
they also cross to reach the end of the digit The deepest tendons are prime movers of the most distal
joints of thumb and fingers.
050
Performance Guidelines



~ FDP and FPL Test the abilities to flex the end joints of the digits first thumb index through to little finger. In this
patient, no spontaneous movement against resistance is possible all of these deep tendons have been severed.
Clearly given this finding, it is very likely that the more superficially placed flexor digitorum superficialis tendons
have been also severed, and you will find this confirmed when you test the ability of the patient to flex the proximal
interphalangeal joint (which in the absence of action of FDP is left as the only muscle which can flex the joint).
Identifying damage to the flexor digitorum superficialis in the finger is easy in this patient with a cut wrist, because the
flexor digitorum profundus is also cut and cannot confuse things by itself acting as a flexor of the proximal IP joint.
But what if the FDP is not damaged? How can the action of flexor digitorum superficialis on the PIP joint be checked in
such circumstances if the sole injury is to FDS?
Answer: Totest the action of the superficial finger flexors in the presence of intact deep flexors is a difficult task and
needs a knowledge of the anatomical arrangements of the muscles, and in particular, the deep layer of flexor digitorum
profundus. Note that with fingers extended, it is possible to flex the index finger at its end joint independent of other
fingers, just as with the end joint of the thumb. But to flex the end joint of the middle, ring or little finger alone is rather
difficult the end joints of adjacent fingers tend also to flex, unless concentrating or holding them down. This is
because, of the four separate tendons of FDP in relation to the muscle, the one to the index finger is virtually a
separate muscle (flexor indicis), whereas the other tendons are communally joined until just above the wrist
This fact can be used to advantage to eliminate the influence of the deeper tendons of FDP to these three fingers on
the proximal joint as follows try this trick.
Hold down flat all fingers but one of a colleague, then ask them to flex the remaining finger at the proximal IP joint to a
right angle as illustrated. By restraining the long tendons of FDP to the other fingers, and preventing their movement,
you have very effectively inactivated the remaining FDP tendon to the middle finger. You can easily check that FDP is
not having any effect by flicking the terminal phalanx with your finger note that the distal IP joint is freely floppy and
the only muscle now causing flexion of the PIP joint is FDS.
CONDITION 050. FIGURE 2. Testing for
function of FDS alone
261
050
Performance Guidelines



An injury to the long flexor tendons should have already been suspected in this patient on inspection alone. In the
normal hand at rest, the fingers and thumb are progressively flexed into the palm from index to little finger with the
thumb at right angles as illustrated. This is the position of rest of the hand with a balanced postural tone of flexors and
extensors. In your patient (a trained role player) the fingers are extended instead of curled and the thumb is also
extended, and the whole hand looks very unnatural; because the imbalance caused by the unopposed natural resting
tension of the finger extensors, with all the long flexors cut, has distorted the normal position of rest.

CONDITION050. FIGURE 3. CONDITION050. FIGURE 4.
Figures 3 and 4 show position of rest from palmar and radial aspects

CONDITION050. FIGURE 5.
Note position of fingers and thumb after
long tendon injury
The inabilities to move the terminal two joints of the fingers and the terminal jointot the thumb are not, and could not be, due
to the injury to the median or ulnar nerves
The branches to the extrinsic long flexors come off from much higher in the forearm and are unaffected by nerve injury
at the wrist, which can only affect the function of intrinsic muscles in the hand. These effects on the terminal joints are
due to tendon injuries.
262
263
050
PerformanceGuidelines



Moving proximally, also note that he cannot flex the metacarpophalangeal joints of the fingers (or of the
thumb). This action in the fingers is done by the lumbricals, supplied by branches from median and ulnar
nervesinthepalm,andthisinabilityisduetothenervedamage,asarealltheothertestsforintrinsicmuscle
functiontestingthenarandhypothenarmusclesandinterosseiapartfromthosewehavealreadydone.These
additionaltestsalsogivecharacteristicsigns:onattemptedulnaradduction,pinchtestofthethumb(Froment
sign)andfailureofoppositionoflittlefinger(Sunderlandsign).Theselattersignsareaccentuatedandeven
moreobviousinpatientswithlongstandingeffectsofmusclewasting,whichneednotconcernusfurtherin
thispatient.
ThemostsuperficialmusclesarethewristflexorsFCRandFCUandPL.Thepatientcannotflexthewrist
activelyeitherbecauseallofthesearedividedaswell(plusthelongfingerandthumbflexorswhichcanof
courseactasaccessoryflexorsofthewristaswellasprimemoversoftheirrespectivefingerorthumbjoints).
Finallythebloodvesselstheverysuperficialradialarteryandmoredeeplyplacedulnarartery.Theseare
verylikelybothtohavebeencutbutvascularspasmandcompressionmayhavecausedbleedingtostop.
Examinethecolourofthefingersandtestcapillaryrefillingafterpressure,buttheanastomosesandcollateral
circulationacrossthewristareveryefficientanditisveryunlikelythatthehandwillbegrosslyischaemiceven
ifbotharterieshavebeendivided.
Thefinaldiagnosis,aftercheckingthatsensationtothebackofthehandandproximalbackoffingersisintact,
confirmingthatradialnerveanddorsalbranchofulnarnervehaveescapedinjury,is:
Severedeepknifewoundofwristseveringallvolarlongflexortendonstowristandhandandseveringmedian
andulnarnervesaverysevereinjuryrequiringearlyreconstructivesurgery.Fortunatelythisisa'tidy'wound
withoutmajorcontaminationandthereisnocontraindicationtoprimaryrepair.
Treatmentwillnecessarilyrequireasubsequentintensiverehabilitationprogrammeofinitialrest,withearly
mobilisationandsupportivephysiotherapyovermanymonths.Thefinalfunctionaloutcomewillbeverymuch
influencedbyhisoccupationifheisaconcertpianist,thoughtsaboutvocationalretrainingshouldstartearly.
051
264
PerformanceGuidelines



Condi t i on 051
MultipleskinlesionsinaQueenslandfamily
AIMSOFSTATION
Toassessthecandidate'sabilitytodiagnoseavarietyofcommonbenignand'suspicious'skin
lesionsandtoadviseonmanagement.
EXAMINERINSTRUCTIONS
A careful history of how long the lesions had been present would normally be required; this
scenariofocusesonpatternrecognitionfromphysicalappearance.
EXPECTATIONSOFCANDIDATEPERFORMANCE
Thefarmerisveryconcernedaboutsixmembersofthefamilyandhasphotographsofeachof
thelesions.Hehascomeinfromhisfarm,whichisalongwayfromthetown.
Thephotographsdemonstrate:
Figure1.Hissonhasalesionsuspiciousofsquamouscellcarcinoma(SCC)ofthelip.
Figure2.His50-year-oldbrotherhasalesionsuspiciousofbasalcellcarcinoma(BCC)
oftheneck.
Figure3.Hisfatherhasaseborrhoeickeratosisonhisface,whichisbenign.
Figure4.Hiswifehasalesionsuspiciousofmalignantmelanomaoftheleg.
Figure5.His52-year-oldbrotherhasabenignspidernaevusofthechest.
Figure6.Hisdaughterhasalesionsuggestiveofabenignmelanocyticdermalnaevus
oftheface.
The SCC, BCC and melanoma, assuming diagnosis is confirmed by excision, are malignant
Theseborrhoeickeratosis,thespidernaevusandthemelanocyticdermalnaevusarebenign
andprobablyrequirejustreassurance.
Hiswife,whohasthemalignantmelanoma,requiresthemosturgenttreatment.Theexcisionof
herlesionshouldnotbedelayed,eventhoughshewouldprefertodelaytreatmentforseveral
monthsbecausetheyarebusyonthefarm.
TheSCCandtheBCCshouldalsoberemovedwithoutexcessivedelay.
The BCC only spreads directly, but local infiltration may be extensive, although this occurs
slowly.TheSCCspreadsdirectlyandmainlybylymphatics,andoccasionallybybloodspread.
Malignantmelanomaisthemostseriousofthelesionsandspreadslocally,bylymphaticsand
by blood spread. Widespread metastases can occur even from a small lesion. The risk of
spread is proportional to the depth of the melanoma seen on microscopic examination. The
prognosisisfavourableifthedepthislessthan0.75mm.
KEYISSUES
Thecandidateshouldindicatewhichlesionsarelikelytobebenign(seborrhoeickeratosis,
melanocyticdermalnaevusandspidernaevus)anddonothavetobeexcised;andwhich
aresuspiciousofmalignancy(SCC,BCCandmelanoma)andshouldbeexcised.
051
265
Performance Guidelines



CRITICALERROR
Failuretosuspectthatthewife'slesionisamalignantmelanoma,andthatsurgicalexcisionshouldoccur
withoutdelay.

COMMENTARY
This scenario illustrates six of the most common focal cutaneous lesions seen in the Australian population
(benignmelanocyticnaevus,seborrhoeickeratosis,spidernaevus,basalcellcancer,squamousceilcancer,
malignant melanoma). Clearly benign, longstanding lesions are the most commonly seen pigmented skin
'moles';andmostcanbeconfidentlydiagnosed.
Benignskinlesions:
Benignmelanocyticnaevi(intradermal,junctionalandcompound)
Theseareclassifiedaccordingtothesiteofthebenignpigment-containingmelanocytes.Junctionalnaevihave
melanocytesatthejunctionofepidermisanddermis,theyareflatterthantheothermorematurenaeviandmay
bewhollymacular.
Intradermal and compound naevi have melanocytes intradermally, or at both epidermal and dermal levels.
Macroscopicallytheyvaryfromalightordarkbrownnodule(oftencontaininghair,ahelpfuldiagnosticpoint
hairymolesarealmostinvariablybenign).

CONDITION051.FIGURE7. CONDITION051.FIGURE8.
Benignmelanocyticnaevus Benignmelanocyticnaevusofneck
Seborrhoeickeratoses('seborrhoeicwarts')
Theselesionsarisefromtheepidermisastheresultofproliferationofkeratinocytes.Theyareoftenmultiple.
Thereisnodermalinvolvementandthekeratosesaresosuperficialthattheyareoftensaidtohavea'painted
on'appearance.Seborrhoeickeratosesoccurinolderpeopleandaremostoftenfoundonthetrunk,although
theymaybefoundonthefaceandscalp.Thelesionsareraisedorflatandplaque-like,withawaxytexture.
Haemosiderindepositionintheplaquesmayproduceabrownish-blackcolour.Occasionallythelesionmaybe
situated on a part of the body that makes it prone to trauma, but the only real reason for excision of a
seborrhoeic keratosis is for cosmetic purposes. They are quite benign and their fissured, variegated, rough
texturedappearanceusuallyallowsconfidentdiagnosis.
051
266
Performance Guidelines
CONDITION051.FIGURE9. CONDITION051.FIGURE10.
Multipleseborrhoeickeratosesoftrunk Seborrhoeickeratosesofback
Spidernaevi
Thesesmalllesionshavearedcentralspotsurroundedbyflaringtelangiectases.Pressureonthecentral
arteriole causes blanching. Multiple ones on the upper trunk, upper limbs, or face can be stigmata of
alcoholicliverdisease.
Papillomas
Otherwiseknownasskintags,papillomasmaybefoundatanysiteandareeithersessileorpedunculated
overgrowthsofskinseenfrequentlyaroundflexuralareasofaxillaorgroin.Theymaybeexcisedforcosmetic
reasons.
Pyogenicgranulomas
Theselesionsmayariseinresponsetominortrauma.Atthesiteofpunctureoftheskinthereisamassof
rapidlygrowinggranulationtissuewhichcharacteristicallyformsanexophyticgrowth.Thismayappearover
afewweeksandbleedseasilyoncontact.Treatmentisbyexcisionandcurettageoftheareaunderlyingthe
granuloma.
CONDITION051.FIGURE11. CONDITION051.FIGURE12.
Pyogenicgranulomaofpalm Pyogenicgranulomaoffinger
051
PerformanceGuidelines



Verrucae
Verrucae are most commonly seen in children and are caused by viruses. Common sites are
hands and soles of feet. The lesions may spread to adjacent sites or other individuals. They
consistofraisedandroundedkeratinisedprojectionsabovetheskinsurface.Thoseonthesole
are commonly 'endophytic' due to weight-bearing. Histologically there is hyperplasia of the
epidermisandincreasedkeratinisation.Treatmentcanbedifficult,astheverrucaislikelytoreturn
ifthevirusisnotcompletelyeradicated.Asmanyverrucaewillcompletelyregress,ifthelesions
are asymptomatic, they are better left alone. Those warts that occur around the genital and
perinealregionsareknownascondyloma acuminata. Theyarecausedbythehumanpapilloma
virusandspreadbysexualcontact.
Keratoacanthomas
These lesions are characterised by rapid growth, a macroscopic appearance resembling a
squamouscellcarcinomaandspontaneousregression.Akeratoacanthomausuallyoccursonthe
face(oftenonthenoseorear)orhandandappearsoverthecourseofafewweeks.Thecentreof
thelesionulceratesandmaycontainaplugofkeratin.Histologicallytheselesionscanresemble
squamouscellcarcinoma,butareidentifiedbyacentralcoreofproliferatingcellsextendingdown
into the dermis. The site of the tumour and its rapid development should make the diagnosis.
Keratoacanthomasshouldbeexcisedandsentforhistologicexaminationtoexcludesquamous
cellcarcinoma.Accuratehistologicdiagnosisisusuallypossibleifthewholelesionisprovided;
onlyasmallmarginofexcisionisrequired.

CONDITION051.FIGURE13.
Keratoacanthomaofface
Fibrohistiocytictumours:dermatofibroma,xanthoma
Thereisaconsiderablehistologicalrangeofsofttissuetumoursandthetwobenignlesionsthat
maybeconsideredoftrueskinorigin arethecutaneousfibroushistiocytoma(dermatofibroma)
andthexanthoma.Adermatofibromaisarelativelycommonskinnoduleandtypicallyoccurson
thelegsofyoungormiddle-agedwomen.
267
051
PerformanceGuidelines
CONDITION 051. FIGURE 14.
Dermatofibromaofleg
The lesions are usually raised from the skin surface and about 1 cm in diameter. Most are
asymptomatic but they can be itchy and tender which is the usual reason for excision.
Xanthomas occur when an area of skin becomes infiltrated by lipid-filled macrophages or
histiocytes. They may occur at any site on the body and the most common form is the
xanthelasma.Thesesoft,yellowplaquesarecharacteristicallyfoundattheinnercanthusofthe
palpebralfissure.
Appendagetumours
Cylindromas (arising from sweat gland cells) and other skin appendage tumours are rare.
Treatmentisexcision,toconfirmthediagnosis.
Premalignantneoplasmsofskin:
Actinickeratoses
Actinic (solar) keratoses are the result of solar damage and are characteristically found on
areasofthebodymostatriskofprolongedsunexposure.Thebackofthehandisacommon
site. The lesions occur most frequently in older people and those who work outdoors. Fair
skinned people living in the tropics and subtropics are most at risk. The actinic keratosis
represents a gradual dysplastic change in the epidermis and underlying dermis. There is a
build-up of excessive keratin in the epidermis and elastosis in the dermis. Actinic keratoses
appear as scaly lesions with hyperaemic bases that bleed easily with trauma. They can be
treatedbycryotherapy,applicationofacytotoxiccreamorexcision.Leftuntreated,15-20%of
actinickeratoseswillprogresstosquamouscellcarcinoma.
CONDITION051.FIGURE15.Multiple
actinickeratosesofhands
268
051
Performance Guidelines



Bowen disease
This is an unusual condition and presents as a scaly red plaque with clearly defined margins. The surface is
keratotic and often crusted and fissured. The lesion is not related to solar damage and in some instances
arsenicals have been implicated in the aetiology Bowen disease may occur on any part of the body and is a
premalignant condition and represents squamous cell carcinoma in-situ. There is hyperplasia of the
epidermis, atypical epithelial cells are present and infiltration of this layer with pleomorphic malpighian and
giant cells. Treatment is with cryotherapy or cytotoxic creams. Larger lesions and those that are suspicious
or frankly malignant are best treated by excision. Skin grafting may be required.
Malignant skin neoplasms
7 49333535
CONDITION 051. FIGURE
16. Bowen disease of skin
The skin is the largest organ of the human body and not surprisingly it is the most common site of tumours.
Skin cancer is the most common malignancy in fair-skinned people. Tumours can arise from any of the skin
structures epidermis, dermis, connective tissue, glands, and muscle or nerve elements. Although not all
skin tumours are neoplastic, from a management perspective, the suspicion of malignancy must always be
uppermost in the clinician's mind when dealing with a skin tumour, be it pigmented or not.
Malignant skin lesions are very common in Australia with a susceptible population and
excessi ve solar exposure. By contrast, melanoma and other skin malignancies are
uncommon in indigenous Aboriginal peoples.
Basal cell carcinomas (BCC)
This is the most common type of skin cancer and is almost totally confined to fair skinned people. Basal cell
carcinomas are rare in Asiatic peoples and almost never occur in dark skinned people. They tend to occur in
people over the age of 40 and are usually found on areas of the body subject to chronic exposure to the
sun, particularly the face. Characteristically these tumours are found on the face above an imaginary line
running from the corner of the mouth to the ear.
The tumours are slow-growing and may take years to get to sufficient size to bother the patient. Left
untreated, a basal cell carcinoma will spread relentlessly and destroy all the surrounding tissues without
ever metastasising. The tumour characteristically has a raised, rolled edge which often takes on a pearly
appearance. Most basal cell carcinomas are the same colour as the adjacent skin, but some are heavily
pigmented and mistaken for
269
051
PerformanceGuidelines



melanomas.Theremaybecentralregressionofthetumourwithulceration,producingtheso-called'rodent
ulcer.'OthermorphologicalpatternsofbasalcellcarcinomasincludethoseresemblingBowendiseasewitha
thin pink plaque (erythematous basal cell carcinoma) and those known as sclerosing tumours with white
plaqueandafinepearlyedge(morphoeacarcinoma).Theerythematousvarietyofbasalcellcarcinomatends
tooccuronthetrunk.
Apart from nodular BCC (the most common presentation), ulcerative, sclerosing/cicatrising ('brush-fire'),
cystic,psoriatiform,comedoformandpigmentedvariationsarecommonlyseen.

CONDITION051.FIGURES17AND18.
UlcerativeBCC MorphoeicBCC

CONDITION051.FIGURES19AND20.SclerosingBCC
behindearPigmentedBCC
Small basal cell carcinoma can be treated by cryotherapy, topical chemotherapy or radiotherapy.
Radiotherapy is used for cancers in areas where surgical resection would be difficult and risk damage to
surroundingstructures,suchastearductsandeyelids.Radiotherapyshouldnotbeusedforlesionsadjacent
tocartilage,whichmightundergoradionecrosis.Themajordisadvantageofthesetypesoftreatmentisthatno
tissueisobtainedforhistologicalanalysis.
Theoptimumtreatmentforbasalcellcarcinomasandparticularlyforlesionsgreaterthan1cmdiameteris
surgicalexcision.Amarginofatleast1mmofnormaltissueisrequired.Tominimisetissueloss,particularly
forlesionsontheface,atechniqueofserialslicingcanbeemployed.Whilstthisistime-consuming,theserial
excision and immediate microscopic examination of the resected tissue will allow an intraoperative
assessmentofclearanceoftumourindepthandwidth.Largerbasalcellcarcinomaswillrequireskingraftinga
reconstructivesurgery.
270
Squamouscellcarcinomas(SCC)
Thesearethesecondmostcommoncanceroftheskin.Whilstbasalcellcarcinomasarisefrom
the basal layers, squamous cell carcinomas arise from the keratinocytes of the epidermis.
Sunlight is an important aetiological factor and solar keratoses and Bowen disease are
precursors. Squamous cell cancers can also occur in scars or chronic ulcers (Marjolin ulcer).
Most squamous cell carcinomas occur in the fair skinned people, but these tumours are also
found in darker-skinned people, particularly in depigmented skin following scarring. These
cancersareusuallyseenintheolderpopulation,butwithexcessivesunexposureinchildhood,
squamous cell carcinoma is also a disease of young adults. As most of the tumours are sun
exposure-related,theytendtooccuronexposedpartsofthebody,particularlytheheadandthe
handsandonthelips,invariablyinvolvethelowerlip.
On the ears, squamous cell cancers occur particularly on the outer helix, in contrast to BCC
whichoccurintheretroareolarsulcus.

CONDITION051.FIGURE21. CONDITION051.FIGURE22.
Squamouscellcarcinomaoflip SCCofear
Squamouscellcarcinomahasavariablenaturalhistory.Thosetumoursthatarisefromactinic
keratosescanbequiteslowgrowing,whilethosethatcomplicateBowendiseasetendtobemore
aggressive. Whilst a squamous cell carcinoma can morphologically resemble a basal cell
carcinoma,thecrucialdifferenceistheabilityoftheformertometastasise.
Optimal treatment of squamous cell carcinoma of the skin is surgical excision. Although the
tumours do metastasise to lymph nodes there is no evidence that prophylactic lymph node
dissection confers any benefit. For those patients who undergo a curative resection, the
prognosisisgood,witha95%5-yearsurvivalrate.
Malignantmelanomas
The most malignant of all skin tumours, more common in exposed skin, occurring throughout
adult life,particularlyprevalentin fair-skinnedpopulations oftropicalclimates,butincidenceis
increasinginmostcountries.
Anybrownorblackmoleshowinganincreaseinsize,irritation,bleeding,nodularityorulceration
shouldberegardedassuspectandshouldbeexcisedwithanadequatemargin
Spreadtoregionalnodesiscommonandmarkedlyworsensprognosis.Bloodbornemetastases
tolungs,liver,brainandsmallbowelarecommon.
Prognosisworsenswithincreasingdepthofinvasion.Severalmacroscopictypesarerecognised
withprogressivelyworseningprognosis.

271
051
Performance Guidelines



CONDITION 051. FIGURES 23-25

Hutchinson melanotic freckle (lentigo maligna melanoma)
Superficial spreading melanoma (most common type)
Nodular melanoma
Nonpigmented amelanotic melanoma.
Kaposi sarcoma
Classical Kaposi sarcoma is found in elderly males of Mediterranean or East European origin
and tends to run an indolent course. The disease associated with AIDS and other acquired
immunodeficiency states runs a more aggressive course. Kaposi sarcoma is a spindle cell
tumour and is characteristically a multicentric angiomatous lesion of the skin. The lesions vary in
appearance from nodule or macule to plaque and may be several centimetres in diameter. In its
aggressive form the body may be covered in confluent, violaceous skin nodules.
CONDITION 051. FIGURE 26.
Kaposi sarcoma
272
051
PerformanceGuidelines



VisceralinvolvementisuncommonwiththeclassicalformofKaposisarcoma,butgastrointestinal
andpulmonarydiseaseoftenoccursinAIDS-relatedKapositumour.Localisedcutaneouslesions
canbetreatedwithradiotherapy,cryotherapy,intralesionalchemotherapyortopicalretinoids.
Otherprimarycutaneoustumours
Malignant histiocytoma (dermatofibrosarcoma protuberans) is a lesion with a tendency to
biphasicgrowthspurtsandlocalrecurrenceafterexcision.

CONDITION051.FIGURE27.
Malignanthistiocytoma
There are numerous other uncommon primary cutaneous tumours, but only three need to be
considered because of their similarity to basal cell carcinoma. The three are Merkel cell
carcinoma, microcystic adnexal carcinoma and sebaceous gland carcinoma Merkel cell
carcinoma is of neuroendocrine origin and is an aggressive tumour with a high rate of local
recurrence. It may resemble a basal cell carcinoma, both in its appearance and preferential
distributionontheheadandneck.
The other two tumours are rare, slow growing and prone to local recurrence if not adequately
excised.
Secondarytumours
Theskinisacommonsiteofmetastaticdeposits,particularforaerodigestivetractneoplasms.In
mostinstancestheskindepositswillonlybecomemanifestaftertheprimarydiseasehasbeen
diagnosedortreated.Occasionally,acutaneousmetastasismaybethepresentingfeatureofan
otherwiseasymptomatictumourofthelungoroesophagus.
273
274
052
PerformanceGuidelines



Condi t i on 052
Subcutaneousswellingforassessment

AIMSOFSTATION
Toassessthecandidate'sabilitytoperformanappropriatelyfocuseddiagnosticexaminationofa
subcutaneouslump.

EXAMINERINSTRUCTIONS
Theexaminerwillhaveinstructedthepatientasfollows:
Thecasescenarioisarealpatientwithalongstandingsubcutaneousswellingthefindingsand
diagnosis are to be checked by the examiner personally prior to commencement of the
examination.
The lump will usually be a lipoma, sebaceous cyst, ganglion or bursa, or occasionally a less
commondiagnosis.
Real patients should just answer questions as asked and will expect to be reassured about
conservativetreatmentbeingoffered,ornoteadviceaboutpossiblesurgery.
Openingstatement:
'What is this lump?'

EXPECTATIONSOFCANDIDATEPERFORMANCE
Majorpointsoftechniqueandaccuracyinexaminationare
Establishingthelump'sphysicalcharacteristicsparticularlycontourandconsistency.
'Layering'thelumpisitinsubcutaneousfat,attachedtoorbeneathdeepfascia,andifthe
latterisitarisingfrommuscle,tendon,ligament,bone,nerveorbloodvessels?
Doesthelumppulsateandisthisintrinsicortransmittedpulsation?
What are its attachments? Superficially is it attached to the skin and deeply what are the
effectsoftensingorcontractingunderlyingmusclesortendons.
Isitafluid-filledcysticlump?Themosthelpfultestherewillbewhetheritistransilluminable,a
test often inadequately performed. This test must be performed properly by correct torch
placementbehindthelumpandmustbedonebysuitablydarkeningthesurroundsturning
off lights and covering with sheet or blanket as required. When brilliantly positive, it gives
irrefutable evidence of contents being liquid or gas. Gas cysts do occur in lung, neck and
bowel;butinasubcutaneoussite,thefluidwillalmostalwaysbealiquid,andusuallyaclear
serous liquid (such as in scrotal cystic swellings, bursae and tendon sheath swellings, or
branchial cysts) rather than pultaceous material or blood. A negative transillumination sign
doesnotofcourseexcludeafluidcollectionasthecystmaycontainacomplexandviscous
fluid,ormayhaveathicklining.
275
052
PerformanceGuidelines



Candidates should familiarise themselves with the normal extent of transilluminability of other
tissues, which vary rather like the scale of sonicity characteristics of an ultrasound. Try
illuminatingthefingerandareasofnormalskintoseetheextentofnormaltransilluminationoffat
andothertissues.Lipomasarenottransilluminablyseparatefromsurroundingfattytissues,nor
are most sebaceous' cysts transilluminable, because the content is viscous or pultaceous
keratin.
Testingforfluctuationisoftenpoorlyperformedalso.Thelumpmustbecapableofbeingfixed
bytwofingersattheperimeterwhileathirdfingercompressesitcentrally.Iftheotherfingers
are displaced and expanded symmetrically, and if this occurs when tested in several
directionsandinplanesatrightanglestoeachother,thenthisagainisirrefutableevidenceof
contained liquid (by virtue of the incompressibility of liquids which causes transmission of
outside pressure in all directions). Lipomas and other soft compressible solids may give an
impressionoffluctuance,buttheydonotexhibittruefluctuation,merelyeffectsofdeformation.
Ultrasound confirmation and positive yield of liquid on needle aspiration are definitively
diagnosticofacysticcollection. Indeeperlumps, suchasthose inbreastorthyroid,where
neithertransilluminationnorfluctuationisrelevantorpossible,thesetechniquesbecomethe
bestdiagnosticaids.
Is the lump vascular? Candidates should not omit feeling for vascular pulsation, or a
transmitted venous impulse, or listening for a vascular bruit or hum. Remembering normal
vascularsurfaceanatomymakesegregiouserrorslesslikelysuchasmissingananeurysm.
Doesthelumpshowemptyingandrefillingaftercompressionorwithjointmovements?This
importantsignshouldalertthecliniciantoapossiblebursalcommunicationwithanunderlying
jointandcandidatesshouldknowwhichbursaearelikelytocommunicatewithwhichjoints.
Testyourknowledgeoftheprepatellarbursa,thesuprapatellarbursa,thepretibialbursa,the
anserinebursa,thesemimembranousbursaandaBakercyst:allarenearthekneejoint,but
onlysomecommunicatewith(orarepartof)thejointsynovium.
Isthelumpattachedtotheskin?Ifthissignisunequivocallypositive,thelumpisverylikelyto
be a 'sebaceous' cyst or one of its variants, like a pilomatrixoma ('calcifying epithelioma of
Malherbe').Thissignissometimeseasytoelicitandisaccompaniedbyobviousskindimpling
orapunctum.Butoften,inareaswheretheskinisthickandrelativelyfixedtodeeperlayers
suchasonthebackofneckorscalp,thetestisequivocalandtheclinicianmustrelyonother
findings such as contour and consistency to help diagnose subcutaneous lumps, and to
differentiatebetweenlipomasandkeratinous'sebaceous'cysts.
Attheendofthecandidate'sexamination,orafterfiveminutes,theexaminerwill:
ASK' Whatisyourdiagnosis?'
ASK' Isthereanysignificantriskofmalignantchange?'JUeanswerisNOforlipomas,
sebaceouscysts,gangliaandbursae.
ASK' Isthereanysignificantriskofinfection?' TheanswerisYESforinfection
complicatingbursaeandsebaceouscysts,butnotlipomasorganglia.ADVISEthecandidate
' pleasecounselyourpatientaboutthelump. '
052
PerformanceGuidelines



KEYISSUES
Thecandidateshouldbeableto:
Perform an appropriately focused and accurate physical examination of the subcutaneous
lump.Anappropriateandoptimalexamination willdetermineinwhichtissue layerthelump
lies, its physical shape, size, contour and consistency, and its relationship to adjacent
anatomicalstructuressuchasskin,muscleandtendons,joints,vesselsandnerves.Testing
forskinanddeeperattachmentsandforfluctuationandtransillumination,whereappropriate,
willbeobservedfortechniqueandaccuracy.
Displayappropriatereasoningskillsinmakingthecorrectdiagnosis:
~ Most lipomas, sebaceous cysts, ganglia and bursae will not be difficult to diagnose.
Distinction between sebaceous cysts and lipomas may be easy and aided bydiagnostic
clues, such as an obvious punctum. and knowledgeable candidates will recognise the
differences and the potential risks of infective complications of sebaceous cysts and
bursae.
~Knowledgeablecandidateswillbeabletomakeaconfidentdiagnosisandtocounselthe
patientbrieflybutappropriately.Thestationishoweverpredominantlytoserveasatestof
technique and accuracy of physical examination, and of appropriate clinical reasoning
skills.

CRITICALERRORS
Veryunsatisfactoryexaminationtechnique.
Majorerrorsinaccuracyoffindings.

COMMENTARY
The most common subcutaneous swelling is a lipoma. Sebaceous (epidermoid) cysts, bursae
andgangliaarealsocommon.
Subcutaneouslipomas(Figures1and2)areslowgrowing,soft,painless,tabulatedandmobile
swellingsbeneaththeskin.
CONDITION052.FIGURE1. CONDITION052.FIGURE2.
Subcutaneouslipomaofback Subcutaneouslipomaunderarm
276
052
Performance Guidelines



EpidermoidCyst(keratinouscyst,implantationdermoidcyst,pilosebaceous
folliclecyst,'sebaceouscyst')
Epidermal(epidermoid)cystsarecommon.Theycanoccuratanyageandatanysitealthough
theytendtobeseeninolderpeopleandmostoftenontheface,scalportrunk.Theyhaveseveral
differentcauses.Someareinclusionorimplantationcysts,traumaticinorigin,andothersresult
fromtheocclusionofthepilosebaceousunit.Traumaticimplantationcyststendtooccuronthe
handsandfingers.Otherswillbefoundatthesiteofsurgicalscars.Someepidermoidcystsare
associatedwithhereditarysyndromes(e.g.Gardnersyndrome).Thecystislinedwithsquamous
epithelium and full of desquamated debris, which has a characteristic soft cheesy texture and
offensive odour when infected (Cock peculiar tumour). These cysts are often and mistakenly
called 'sebaceous' cysts. A true sebaceous cyst is rare and arises from a sebaceous gland. A
keratinouscystisapreferableterm.

CONDITION052.FIGURE3. CONDITION052.FIGURE4.
'Sebaceous'cystwithpunctum
An epidermoid cyst tends to be elevated and many, but not all, will have a central punctum.
These cysts may discharge or become infected. Uncomplicated cysts may be enucleated,
whereasaninfectedcystshouldbeincisedanddrained,withlaterexcision.
CONDITION052.FIGURE6.Large
'sebaceous'cystofscalp
CONDITION052.FIGURE5.
Multiple'sebaceous'cystsofscrotum
277
052
PerformanceGuidelines



'Sebaceous'cysts(Figures3,4,5and6)movewithandnotseparatefromtheskin.Theyoccur
withinanyareaofhair-bearingskin.Theyareusuallyround,fluctuantandnon-transilluminable
with a smooth nonlobulated contour which differentiates them from lipomas. The other
differentiations(skinattachmentversusthesubcuticularmobilityoflipomas)arenotalwayseasy
signstodetectinareasofthickskinlikethebackoftheneck-but'sebaceous'cystsalwayshave
afocalpointofskinfixationwithorwithoutapunctum,andhavearoundnon-lobulatedcontour.
Dermoidcyst(congenitalinclusioncyst,hamartomatouscyst)
Dermoidcystscanbefoundascystictumoursoftheovaryorwithinthecraniumandspineandfor
thepurposesofthissection,insubcutaneoustissues.Apartfromtheconfusinguseofthisterm,
the dermoid cyst is a congenital lesion and those found in the skin and subcutaneous layer
usuallyoccurontheface,neckorscalp.Theyarethin-walledcystsandcontainfattymaterialand
occasionally,hair.Althoughthesecystscanappearatanyage,thoseonthefaceandneckare
usuallyevidentatbirth.Thoseonthefaceoccurmainlyaroundtheeyesandareoftenattachedto
theunderlyingperiosteum.Theymayalsobefoundinthemouthandupperneck.Dermoidcysts
are true hamartomas and develop when skin and skin structures become trapped during
embryonicdevelopment,suchasatlinesoffusionanteriorlyinthemidlineandaroundtheeyesin
theheadandneck.Treatmentisbyexcision.Imagingmaybenecessary,toassessthedegreeof
involvementofunderlyingstructures.
Ganglia (Figures 7, 8 and 9) present as deeply placed subcutaneous lumps around joints or
tendonsheaths.Theymaybemademoreprominentbytendoncontractionortensingandonjoint
movement. These are helpful diagnostic tests in the optimal examination sequence of: Look,
Move, Feel, Listen. The common ganglia those around wrist or ankle do not communicate
withtheadjacentjoints.Theyare,however,oftenformedbycysticdegenerativechangeinthe
fibrousjointcapsuleorfibroustendonsheath,sotheirremovalnecessitatesopeningthejointor
sheath.
CONDITION052.FIGURE7.
Ganglionoflateralaspectoffoot
278
052
Performance Guidelines
CONDITION052.FIGURES8AND9.Ganglion
ofwristthemostcommonsite
Bursaearecysticsacsbetweentheskinandunderlyingbonyprominencesortheyseparateand
aidglidingofadjacenttendonsandligaments.Somebursaecommunicatewithjointsortendon
sheaths.

CONDITION052.FIGURE10. CONDITION052.FIGURE11.
OlecranonBursitis Doublepathology'sebaceous'cystof
neckwithsubmandibularsalivarygland
swellingbehindit
Candidates should show appropriate perspective in counselling. Many or most of these lumps
requirenoactivetreatmentotherthanreassurance.
Subcutaneouslumpsareverycommonandtypicalexamplesasindicatedinthefigures.Knowing
that the lump has been present for a long time without significant symptoms or change in
characterisreassuringandmakesabenignconditionmostlikely.
279
053
280
PerformanceGuidelines



Condi t i on 053
Examination of the knee of a patient with recurrent painful
swelling after injury
AIMS OF STATION
To assess the candidate's technique of physical examination of the knee joint and the accuracy of examination.

EXAMINER INSTRUCTIONS
The examiner will have instructed the patient as follows:
Opening statement:
'What is wrong with my knee?'
You have a history of twisting your right knee six months ago when you caught your foot on a piece of broken
pavement. You fell on the right knee which became swollen and painful on the inner side of the knee. The
swelling caused a painful limp for a few days then subsided.
Since then you have had intermittent attacks of pain felt on the inner side of the right knee with swelling which
settles within 24 hours. You have difficulty in straightening your right leg fully and occasionally have
apprehension twisting to the right.
You are otherwise well and between attacks can walk normally with only a minor feeling of pain on the inner side
of your knee.
You should complain of tenderness at the inner joint line anteriorly when the right knee is examined.
You cannot fully straighten the affected knee because of pain (a deficit of around 15 of extension).
EXPECTATIONS OF CANDIDATE PERFORMANCE
The candidate should examine both knees:
Expected technique of knee examination:
Checks stance and gait.
Checks active range of movement initially flexion/extension (range/power) notes positive signs of inability
to fully extend affected knee and medial joint tenderness
Checks passive range of movement with care, ensuring that the range of active movement is not exceeded.
Checks for tenderness at joint line and around margins over sites of attachments of collateral ligaments and
patellar ligament Should identify tenderness anteriorly, at the joint line, on the inner aspect of the right knee.
Checks for joint effusion (patellar tap' and 'bulge test' for cross fluctuation).
Checks patellofemoral mobility and tracking
Checks integrity of ligaments appropriately: valgus and varus strain to slightly flexed joint for collaterals;
anterior and posterior glide (drawer) test for cruciates all of these
are normal
053
281
PerformanceGuidelines
Checksquadricepsforstrengthandwasting
Comparessymptomaticsidewithnormalside.
Examinesbackofjoint(poplitealfossa)aswellasfrontandsides.
Diagnosis/DifferentialDiagnosis
Probableinjurytomedialintra-articularmeniscus(medialcartilage)
Alternative:traumaticosteochondritis/synovitisrightknee.
Unacceptable:cruciate/collateralligamentrupture.

KEYISSUES
Performafocusedandaccuratephysicalexaminationofthekneejoints(physicalexaminationskills).
Formulateadiagnostic/differentialdiagnosisplanappropriatetotheclinicalproblem(clinicalreasoning
skills).

CRITICALERRORS
Failuretotestmovementsoftheleftkneetocomparewiththeother(affected)side.
Failuretotestligamentintegrity.

COMMENTARY
Thekneejointisthemostcomplexsynovialjointinthebody.Traumaticsofttissueinternalderangementsof
theknee(IDK)arecommonafterdomestic,recreationalandsportinginjuriesgiving:
injuriestotheintra-articularcartilages(morecommonlytothemedialmeniscus);
tearsofthecollateralligamentsfromvalgusorvarusstrains;
cruciateligamenttears;
traumaticsynovitis;and
chondromalaciaorosteochondritisdissecans.
Candidatesshouldnotomitexaminingthebackoftheaffectedknee.Occasionallycandidatesmaymistakenly
examine the normal knee instead of the affected one after asking the patient to turn over a moderately
seriousmistakeinducedbynervousnessandlackofconcentration.
282
054
PerformanceGuidelines



Condition 054
Assessmentofhearingloss,firstnotedduringpregnancy,ina35-year-old
woman
AIMSOFSTATION
Toassessthecandidate'sknowledgeoftypesofhearinglossandtheirdifferentiationonexamination.
EXAMINERINSTRUCTIONS
Openingstatement:
' Whatiswrongwithmyhearing?'
Theexaminerwillhaveinstructedthepatientasfollows:
yougavebirthtoyourfirstchildamonthago;
aboutmidwayduringthepregnancyyoubecameawareofreducedhearing;
thishasbecomeprogressivelyworseandisthereasonforyouconsultingthedoctortoday;
yourinfantisbreastfedandthriving;and
youhavenoothercomplaints.
Thedoctorisexpectedtoaskyouaboutafamilyhistoryofdeafnessbutprovidethisinformationonlywhen
asked.Yourmotherhadoperationsfordeafnessonbothearsmanyyearsago.Yourfather,brotherandsister
arenotdeaf.
Thedoctormayaskyoufurtherquestionsaboutyourhearingloss:
bothearsareaffected;
ithasnootherspecialcharacteristics' j ustgetti ngdeaf:
youhavenothadexposuretoveryloudmusic(e.g.heavymetalmusicviaearphones),orindustrialnoise;
and
youhavenopasthistoryofearinfections.
Regardingseverity:
youhavedifficultyhearingthebabycryifheisnotinthesameroomasyouare;
youhavetohavethetelevisionvolumeturnedup(partnercomplainsitistooloud):and
youhavenoticedthatyouseemtohearabitbetterwhenthereisalotofoutsidenoise.
Otherpoints:
TheexaminerwillNOTALLOWthedoctortousetheotoscopeprovidedtoexamineyourearcanalsbut
willaskaboutitsuseandwhatconditionsarebeinglookedfor.Candidatesshouldindicatetheywilllook
forcompleteocclusionoftheearcanalbywax(cerumen).
Hearing capacity the doctor will whisper numbers or words in your ear, masking your other ear
respondbysayingthatyouCANNOTHEARthesesoundswhentheybecomesoft.
Tuningforktests:
~Whenplacedontopofyourheadsay' sameonbothsi des' ,inresponsetothedoctor'squestion.
283
054
Performance
Guidelines

~Whenplacedbesideyourearandthenpressedonthebonebehindyourear,saythat' thel atteristhe


l ouder.' Thedoctormaythenrepeatthetestwiththeforkbesideyourear,totestairconduction,asking
youtosaywhenthesoundcannolongerbeheard.Respondaccordinglyafterabout10seconds' can' t
hear i t now' . Thedoctorshouldthenpressthetuningforkonthebonebehindyoureartotestbone
conduction.Respondbysaying7canhearthat' .
~Ifthecandidatedoesthetestinthereverseorder(i.e.testsboneconductionpriortoairconduction)react
as follows: 7 can hear that' (on bone). When it can no longer be heard say 7 can' t hear that
anymoreandwhenairconductionistested,say7can' thearthatei ther' .
Thecandidateshouldadvisethatreferralforanaudiogramorotolaryngological(ENT)opinionisnecessary.
EXPECTATIONSOFCANDIDATEPERFORMANCE
Abilitytodistinguishtypesofhearingloss(inthispatientconductionlossduetootosclerosis).
KEYISSUES
Skillinuseoftuningforkteststodefinetypesofhearinglossconductiveversussensorineural.
Abilitytoexplaintothepatienttheproblemanditsmanagement.
CRITICALERRORS
Failuretocorrectlyusetuningforkinassessinghearingloss.
Failuretoadviseareferralforaudiometryand/orENTopinion.
COMMENTARY
' Therearetwoki ndsofdeafness.Onei sduetowaxandi scurabl e;theotheri snotduetowax
andi snotcurabl e.'
Si rWi l l i amWi l de(1815-1876).fatherofOscarWi l de
Wehavemadesomeprogressindiagnosisandtreatmentsincetheabovestatement!
Thisstationispredominantlyatestofskillinclinicalassessmentofhearinglossrequiringthatthecandidate
hasabasicknowledgeoftypesofhearingloss:conductiveversussensorineural,andthatconductivedeafness
due to wax occlusion or other causes must be excluded. The patient evinces conductive deafness due to
otosclerosis.
Deafnessisacommonprobleminolderpeopleinourcommunity.Theonsetofbilateraldeafnessinayoung
woman during pregnancy is uncommon. The positive family history of a mother requiring surgery for her
hearinglosspointstowardsaninheritedcause,namelyotosclerosis.Theonsetofthisconditionisgenerallyin
earlyadulthoodandmayprogressrapidlyinpregnancy.Thestapesfootplatebecomesankylosedintheoval
windowcausingconductivetypedeafness.Patientsmaynoticetheyhearmoreclearlyinnoisysurroundings.
Theconditioncanbetreatedbyprostheticstapedectomyandveingrafting.
054
284
Performance Guidelines



The eighth cranial nerve has two functional parts, the vestibular and the cochlear components. The cochlear branch
subserves hearing. Afferent cochlear fibres from the inner ear pass through the internal auditory meatus and enter the
upper medulla at the level of the inferior cerebellar peduncle to reach the dorsal and ventral cochlear nuclei. Fibres
from these nuclei cross to the other side and end in the inferior colliculus. Fibres from that body go to the medial
geniculate body and the auditory radiation to the temporal cortex. It is important to note that there are bilateral
connections in the cochlear nucleus and above.
Sensorineural deafness occurs when there is damage to the cochlear nerve fibres anywhere from the inner ear to the
cochlear nuclei. The most common cause is degenerative changes in the elderly. Other causes include a fracture of
the petrous temporal bone and an acoustic neuroma.
Conduction deafness is caused by blockage of the ear canal and by damage or disease of the tympanic membrane or
ossicular chain or fluid in the inner ear.
Establishing type of deafness conductive versus sensorineural.
Hearing should first be tested clinically for each ear with the examiner's finger occluding the other ear. Few people now
have a ticking watch, so the examiner stands to the patients side and whispers numbers, which are repeated by the
patient.
The meatus, canal and drum are inspected with an otoscope, retracting the ear upwards and backwards to straighten
the canal. This excludes other nonacute causes of conductive deafness (such as wax, osteomas, otitis externa,
chronic otitis media). In this instance candidates will be informed that otoscopy is normal.
Deafness may be due to impaired conduction of sound through a muffled middle ear (conductive or middle ear
deafness); or to a lesion of auditory nerve, cochlear or brain (perceptive or sensorineural deafness).
A tuning fork of high pitch (256 Hertz or greater) is used to compare hearing by bone conduction and air conduction.
Normally air conduction is better than bone conduction. In nerve deafness air conduction remains better than
bone conduction in the affected ear or ears. In middle ear/conduction deafness, bone conduction becomes better than
air.
RinneTest
The vibrating tuning fork is placed on the mastoid, then at the auditory meatus: and the patient is asked which is louder.
Air conduction (AC) is normally louder and is also iouder in nerve deafness. Hearing the fork louder on bone
conduction (BC) indicates conductive deafness (BC > AC).
Alternatively, put the fork on the mastoid until no longer audible, and then put it outside the meatus. The sound will in
normal individuals be heard again and will also be heard again in nerve deafness, but not in conductive deafness.
Weber test
This can be very useful in unilateral deafness. The fork is placed on the centre of forehead in the midline; ask whether
this is louder in one ear or equal. Normally the sound is heard equally in both ears. Occlude one ear with a finger or ear
plug, and the sound will become louder in the affected ear (conductive deafness). In nerve deafness the sound is heard
better on the normal side. So in unilateral conduction deafness Weber test localises to the affected side, in nerve
deafness to the normal side.
285
054
Performance Guidelines



In bilateral conductive or bilateral nerve deafness the sound will be the same. Unilateral nerve deafness must be due
to a lesion of the nerve itself, as cortical radiations are bilaterally and diffusely represented in the temporal cerebral
cortices.
Common causes of conductive deafness are wax, otitis media, otosclerosis and Paget disease. Nerve deafness can
be due to cochlear degeneration, acoustic nerve tumour, drug ototoxicity, or trauma (fracture of petrous temporal
bone).
Knowledge of features of otosclerosis is required to identify the likely cause.
Otosclerosis is a common cause of bilateral symmetrical hearing loss in adults. The stapes footplate is ankylosed in
the oval window. The condition is familial (autosomal dominant), more common in women and worsens with
pregnancy so that patients may present during pregnancy. Patients may notice they hear more clearly in noisy
surroundings, whereas in perceptive hearing loss background noise worsens hearing. Investigation by audiometry will
be diagnostic. The condition can be treated by prosthetic stapedectomy and vein grafting. A hearing aid is less
effective for this condition and effectiveness gradually diminishes.
055
286
Performance Guidelines



Condition 055
Examination of a 20-year-old woman who dislocated her
shoulder 6 months ago

AIMS OF STATION
To assess the candidate's ability to perform a focused examination of the shoulder joint and of
axillary nerve function.

EXAMINER INSTRUCTIONS
The examiner will have instructed the patient as follows:
You dislocated your right shoulder six months ago whilst playing competitive basketball. The
dislocation was reduced successfully without anaesthesia at courtside.
You initially noticed you had an area of numbness (with loss of sensation) the size of your fist
over the lateral side of the upper arm below the shoulder tip, and you couldn't raise or keep your
arm above your head. You were told this was due to a nerve injury. Over the next month the
feeling gradually returned and the power in the arm came back.
By four weeks you were having active shoulder exercises out of the sling, and by four months
you were able to move the shoulder quite normally. You then began a graduated gymnasium
programme under the supervision of your physiotherapist. Your current programme includes
weight work and you have a full range of movements without loss of power as compared with the
other side.
You have resumed fully your normal activities of daily living and are keen to return to playing
basketball when the season starts again in three months. Your physiotherapist and gym
supervisor feel you are ready to return to this sporting activity but have suggested you get a final
clearance from your doctor.
Your shoulder will be examined by the candidate and it is now normal and without discomfort.
The candidate will give the findings to the examiner and then will discuss things with you.
Near the end of the assessment, the examiner will ask questions concerning shoulder function
and what nerve was originally damaged.
Appropriate prompts could be used as follows if the candidate does not provide you with the
information you require regarding returning to basketball.
Questions to ask unless already covered:
'Am I able to restart sport in three months, doctor?'
'Do I need any other tests done?'
'Would it help if I saw a specialist in Sports ' Medicine?'
'Is it likely to happen again?'
287
055
PerformanceGuidelines

EXPECTATIONSOFCANDIDATEPERFORMANCE
Thecandidatewouldbeexpectedtocheck:
Rangeofmovementflexion,extension,abduction,adduction,internalandexternalrotation
andcircumductionallwillbenormal
Powerofmovementnormal
Scaleofmusclepowerused:
~0=nil
~1=flicker
~2=activemovementpossiblebutnotagainstgravity
~3=activemovementpossibleagainstgravity
~4=activemovementpossibleagainstgravityandresistance
~5=fullpowerabsenceofwasting,particularlyofdeltoidmuscle,whichissupplied
byaxillarynerve
Sensationnownormal.

Aftersixminutes,theexaminerwillask(preferredanswersinparentheses):
'Isshoulderfunctionandmovementnownormal?' (Yes).
' Whichnerveisatmostriskfromtheusualtypeofshoulderdislocation?' (Axillary
nerve).Ifthecandidateisunabletoindicatewhichnerveisinvolved,averyunsatisfactory
markshouldbegiveninthediagnosiscategoryofassessment.Asknowingthenameofthe
involvednerveisnota"KEYISSUE"itwouldnotmeanafailmarkoverallmustbeawarded,
but failing to test for axillary nerve function would be a critical error and generate a fail
assessment.
' Couldyoupleasenowfinishyourdiscussionwiththepatientaboutherdesireto
returntosport?' (Allseemssatisfactoryforyoutoreturntoplayingbasketball.Thereisa
small likelihoodthattheshoulder dislocationwillhappenagain.It wouldbe advisablefor
you to have the shoulder strapped before each game to reduce the likelihood of a
recurrenceoftheproblem.Yourphysiotherapistwillbeabletoteachyouthebestmethod
of strapping, and you then should be able to doit yourself, or get your coach to do it for
you).

KEYISSUES
Examinationoftheshoulderareaindicatingtheappropriatetechniquetobeusedtoevaluate
theshoulderandaxillarynervefunction.
Displayappropriatecounsellingskillswhenadvisingthepatientconcerningherdesiresto
returntosportingactivities.

CRITICALERRORS
Inabilitytoassessadequatelythenormalrangeofmovementoftheshoulderjoint.
Failuretotestthesensoryandmotorfunctionscoveredbytheaxillarynerve.
Givinginappropriateadviceconcerningthelikelihoodofrecurrentdislocation.
288
055
Performance Guidelines



COMMENTARY
A dislocated shoulder is a common injury in body contact sports. The shoulder joint is the most mobile synovial joint in
the human body. Protection is supplied by the overlying acromion and clavicle, and by the rotator cuff musculature that
closely envelops the joint. The major weak spot is below, where protection by surrounding muscles is less, and the
capsule is more lax to allow freedom of full flexion and abduction.
The common mechanism of dislocation is therefore displacement of humeral head from the glenoid downwards and
forwards. The axillary nerve runs between the muscles immediately below the capsule from front to back and is
therefore at hazard from stretching injury in shoulder dislocations. The axillary nerve gives motor branches to deltoid
and to teres minor, and supplies sensation as the upper lateral cutaneous nerve of the arm. It is important to test for the
integrity of the vulnerable nerve before reduction is undertaken and afterwards; just as it is to test sciatic nerve function
before and after reducing a posterior dislocation of the hip (the most common hip dislocation).
056
Performance Guidelines
Condition 056
Assessment of a groin lump in a 40-year-old man
AIMS OF STATION
To assess the candidate's ability to perform a focused inguinoscrotal assessment, and to
diagnose and advise on management of a reducible groin hernia.

EXAMINER INSTRUCTIONS
The real patients, with a variety of groin lumps will play themselves. The examiner will check the
physical findings prior to the assessment. The patient illustrated has a reducible right inguinal
hernia.

KEY ISSUES
Performance of an appropriately focused inguinoscrotal assessment with appropriate technique
and accuracy

CRITICAL ERRORS
Failure to display appropriate clinical skills in diagnosis of a reducible groin hernia
Causing significant patient discomfort by rough technique.

COMMENTARY
Inguinoscrotal lumps are very common and present throughout life from birth to old age.
The history of onset of pain following a lifting strain and followed by a lump is very suggestive of a
groin hernia, and the appearance of the lump also supports this diagnosis.
The examination should be thorough but gentle. The groin area needs to be exposed and the
patient examined unclothed below the waist. It may be convenient to start with the patient
standing, as small hernias are often made more prominent when standing.
However, full examination and definition of all inguinoscrotal lumps is best performed with the
patient comfortably lying; and with an anxious, apprehensive or modest patient it is best to start
the examination with the patient lying supine. Examination with the patient standing should not be
omitted, however, as various swellings (such as varicocele, saphena varix) may only be apparent
on standing.
Inspection and palpation of the area enable one to answer:
'Is a groin hernia present?'A groin lump with an expansile impulse on coughing confirms the
diagnosis in this case. The lump is seen and felt to expand uniformly and expansively when the
patient coughs. The impulse ceases and the lump lessens or disappears when he relaxes.
'Is the hernia reducible?' Check this visually and by feel. Always ask the patient himself to
reduce the swelling before you try in lumps of long standing he will be much more adept
than you!
'Is it an inguinal or femoral hernia?' This question is mainly of concern to the treating surgeon,
but usually the differentiation is clear. This lump's relations to the groin land-
289
056
PerformanceGuidelines



marks with an inguinal hernia are that the swelling is above the inguinal ligament and
mediallyplacedinregardtothepubictubercle.Whenyoufeeltheimpulseitisclearthatitis
arisingfromtheexternal(superficial)inguinalring,thekeytodiagnosis.
Thispatient'sherniaisclearlyaninguinalhernia.Itarisesfromtheexternalring,hasan
expansilecoughimpulseandreducesonlyingdown.
If the lump had come out from the region of the saphenous opening below the inguinal
ligamentandmorelaterally(4cmbelowandlateraltopubictubercle)itwouldhavebeena
femoralhernia.Sometimesinobesepeopleitisimpossibletobequitesureclinicallywhich
typeofherniaispresent.

CONDITION056.FIGURE2. CONDITION056.FIGURE3.
Rightinguinalhernia Leftinguinalhernia

CONDITION056.FIGURE4. CONDITION056.FIGURE5.
Rightfemoralhernia Largerrightfemoralhernia
CONDITION056.FIGURE6. CONDITION056.FIGURE7.
Bilateralfemoralhernias Largeleftscrotalhydrocele
290
056
PerformanceGuidelines



'Is the inguinal hernia direct or indirect?' The brief answer is often you cannot tell clinically,
and the diagnosis is made at operation for hernias confined to the groin alone. But if the hernia
is a larger one, which clearly extends well down into the scrotum, the answer becomes
obvious. Only indirect inguinal hernias descend into the scrotum by virtue of the anatomy of
the sac, which is within the spermatic cord. Whereas the sac of a direct hernia, which never
descends, is behind the spermatic cord. The candidate must check that an inguinoscrotal lump
reduces completely on lying down there may in fact be two lumps a reducible hernia
coming down from above and an unreducible scrotal hydrocele below!

CONDITION 056. FIGURE 8. CONDITION 056. FIGURE 9.
Bilateral large inguinal hernias, Large right indirect inguinoscrotal hernia
probabl y indirect
Patients with chronic obstructive airways disease and a chronic cough often develop acquired
bilateral direct inguinal hernias. These present as small swellings confined to the groin which
bulge directly forward through the enlarged external inguinal rings as illustrated.
CONDITION 056. FIGURE 10. CONDITION 056. FIGURE 11.
Bilateral direct inguinal hernias Varicocele
291
292
056
Performance Guidelines



After having arrived at the correct diagnosis (a reducible right inguinal hernia), the examination is not yet finished.
Multiple pathologies are very common. Both right and left sides must be checked both inguinal and femoral orifices
to check that the hernia is unilateral.
Additionally carefully check the spermatic cords/testes and their coverings to exclude an additional testicular swelling,
hydrocele, or epididymal cyst.
Feel femoral pulses and check that no abnormal lymph node enlargements are present. Do not forget to stand the
patient up to check for venous swellings or small hernias
Once a full diagnosis is made, consider treatment. Although surgery is not obligatory for all hernias, it is likely to be
appropriate in this manual worker. So, refer him appropriately, explaining that the benefits of surgery usually outweigh
risks (of which a number exists).
293
057
Performance Guidelines



Condition 057
Eye problems in an Aboriginal community
AIMS OF STATION
To assess the candidate's ability to interpret photographs of trachoma, to assess their
knowledge of the disease, and their ability to advise on appropriate management of
the condition,
EXAMINER INSTRUCTIONS
The examiner will have instructed the nurse as follows:
You are a trained nurse and have come to work in a small community in outback
Australia. You have no previous experience of this type of work and in particular, you
have little knowledge of the diseases often endemic in remote Australian Aboriginal
communities.
You have been in the community for a couple of weeks and have noticed that a
significant proportion of the community appears to have eye problems. You have taken
photographs of some of these individuals and have brought the images to the
community doctor so that you can get a medical opinion on the problems and how
they might be managed.
Questions to ask unless already covered:
'Do you think these might all be due to the same problem?'
'How does the infection get from one person to another?' (if infection is mentioned as
a cause)
'What will happen if the condition is not treated?'
'Is this condition found anywhere else and how common is it?'
'Is it preventable
9
If so, how can it be prevented?'
EXPECTATIONS OF CANDIDATE PERFORMANCE
The candidate should be able to:
recognise that eversion of the upper eyelid facilitates diagnosis;
describe the changes in the photographs:
recognise that the condition is trachoma, because of the setting of an indigenous
Australian Aboriginal community, with classical appearances of trachoma:
describe the agent responsible for the disease (Chlamydia trachomatis), and the
vector of transmission (flies, hand contact, fomites);
understand the pathological changes produced by the organism; and
describe some simple and appropriate measures that might be employed to reduce
the risk of infection.
KEY ISSUES
Interpretation of clinical photographs of eye changes produced by trachoma.
Knowledge and understanding of trachoma, its mode of transmission and measures
used to reduce the risk of infection.
294
057
Performance Guidelines

CRITICAL E R R O R
Failure to recognise trachoma
COMMENTARY
Globally, trachoma is the most common infectious cause of blindness and is a preventable disease. It is endemic in
Africa, the Middle East, Asia and Aboriginal communities of Australia. More than six million people are blind as a
result of trachoma. It is a disease of poverty and the marginalised members of society.
The condition was known from ancient times as a contagious disease, and given the name trachoma (Greek),
meaning a 'rough' swelling.
Trachoma is caused by an obligate intracellular Gram-negative bacterium. Chlamydia trachomatis. The disease is
usually transmitted by direct contact or fomites between children and their mothers, or others involved in the care of
children, and by flies. Poor facial hygiene facilitates spread by attracting flies and there are often recurrent bouts of
infection within a family. Recurrent infection will cause chronic conjunctival inflammation, which is followed by
scarring of the tarsal conjunctiva. As scars mature, the tarsal plate becomes distorted and entropion (turning in of the
eyelid) develops and this results in trichiasis (misdirection of the eyelashes towards the globe). Chronic irritation of
the globe will lead to corneal abrasions, infection, opacification and finally, blindness.
Candidates should be able to identify the stages of trachoma, easily remembered by the acronym FISTO
Follicles;
Inflammation;
Scarring;
Trichiasis; and
Opacity of the cornea.
Follicles are the sign of active trachoma infection, and represent the sites of replication of the causative organism.
The diagnosis of trachoma is confirmed by the presence of more than 5 follicles. They lead to scarring after multiple
attacks (dozens).
The scarring of the deep surface of the lid distorts the lid and results in lashes rubbing on the cornea, which ulcerates
and becomes infected and scarred. The resulting blindness is permanent.
Candidates should know that azithromycin is specific for the causative organism Chlamydia trachomatis.
The active disease is usually seen in young children, with inflammatory changes most apparent in young adults. The
scarring effects of infection develop in middle-age, when the patients present with trichiasis and corneal opacity.
A simplified grading scheme for trachoma which can be taught to and used by community health workers, was
introduced by the World Health Organisation (WHO) in 1987.
295
057
Performance Guidelines



Grade Definition
TF Five or more follicles seen on the tarsal conjunctiva of the everted upper lid. (Figures 1, 5)
Tl Intense inflammation of the upper tarsal conjunctiva, obscuring the view of more thanhalf of the deep tarsal blood
vessels on the everted upper eyelid. (Figure 6)
TS White lines of subconjunctival scarring visible on the surface of the everted upper eyelid. (Figure 2)
TT Trichiasis, at least one eyelash rubbing on the globe, or evidence of recent
removal. (Figures 3, 4, 7)
CO Corneal opacity, obscuring at least part of the pupil margin. (Figures 3, 4, 7)
The candidate should be able to discuss public health measures. Some practical advice for a community nurse would
be along the lines of the SAFE strategy, developed by the WHO.
Surgery: identification of individuals in the community who might benefit from correctional eyelid surgery for
trichiasis and entropion.
Antibiotics: children should be examined (with eversion of the upper eyelid) for trachoma and all members of a
family where there is active trachoma should be treated with oral azithromycin
Facial cleanliness communities, and particularly affected families, need to be educated about the disease, its
mode of spread, and that simple measures such as ensuring facial cleanliness will reduce the risk and severity of
trachoma.
Environmental upgrade: any improvement in water supplies, household sanitation, personal and community
hygiene will reduce the risks of infection. Improved cleanliness in sleeping areas with fly and dust control should be
emphasised.
Additional trachoma examples are shown below. Upper eyelid eversion to inspect for follicles facilitates early
diagnosis.
057
Performance Guidelines

CONDITION 057. FIGURE 5. CONDITION 057. FIGURE 6.
Inflammatory follicles on the under Inflammatory changes on the everted
surface of the upper tarsus (TF) upper eyelid (Tl)
in closeup

CONDITION 057. FIGURE 7.
Entropion with trichiasis (TT) and corneal
opacification (CO)
296
2-C:Choice and Interpretation of
Investigations

ReubenDGlassandVernonCMarshall
' Itisnotofdecisivesignificancewhethertheclinicianconfrontsanoverwhelmingora
modestamountofmaterial,ifonlyheunderstandshowtoexploitit:inotherwords,he
must be in a position to put the right questions and to find the right methods for
answeringthem.'
RudolfVirchow(1821-1902)
Disease,LifeandMan
Modern medicine relies considerably on the results of special investigations. Previously, the
traditionalstock-in-tradeoftheastuteclinicianhasbeenthemannerinwhichhisunaidedclinical
senses of observation, hearing and touch are used in making diagnoses and in formulating
strategiesofmanagement.Buttimesandclinicalpracticehavechanged.Investigationsplayan
increasingly important (though still not all-important) part in the practice of medicine. Major
growthofspecialinvestigationshasoccurred inthefieldsoflaboratorymedicineandinorgan
imaging.Theclinicianinsuchinstancesoftenassumestheroleofconsumer,andmustbealert
tothepossibilitythatanopiniongivenbyanother(orthedataissuedbyamachine)isnotalways
absolutelyreliableandmayoccasionallybewrong.Thisshortintroductionaimstogiveageneral
idea of the circumstances in which special investigations are essential, useful, profitable,
redundant,orpotentiallydangerous,andtohelpintheirchoiceandinterpretation.
Patientsoftenassumethatwhenatestisordered,itwillanswerwhetherdiseaseispresentor
not.Clinicianssometimesmakethesamemistake.Atestmaybeusefulforhelpingtoconfirm,or
helpingtoexcludethepossibilityofadisorder,butwillseldomgiveaperfectanswer.Afrequent
erroristosupposethatagiventestisequallyusefulforconfirmingorexcludingdisease,butthis
isusuallynottrue.Theclinicianwhoordersatestshouldalwaysbemindfulofthereasonforits
use.Ifthetestresultwillnotalterthepatient'smanagement,thereislittlepointinorderingit.
Diagnostic accuracy of any test will depend upon how well it performs in comparison with its
performance against another so-called 'gold standard'. The histological diagnosis of cancer is
the usual yardstick against which the performance of another less invasive test is measured.
More often, however, one must compare the accuracy of several available tests to determine
whichisbest.
TESTRESULTSANDTHEIRIMPLICATIONS
297
SensitivityandSpecificity
Theperformanceofatestmaybestudiedinasurveyofaparticular
population.
The test sensitivity is the percentage of patients known t o have a
particulardisease,whosetestprovespositive( t r ue posi t i vi t y) The
downsideofhighsensitivityisapotentiallyhighfalsepositivityrate:
theproportionwithoutthediseasewhoalsotestpositive.
The test specificity is the percentage of patients in that population
known t o be f r ee ofthedisease,inwhomthetestprovesnegative
{ t r ue negat i vi t y) Thedownsideofhighspecificitywouldbeahigh
false negativity rate: the proportion with the disease who also test
negative.
Afrequenterroristo
supposethatagiven
testisequallyusefulfor
confirmingorexcluding
disease,butthisis
usuallynottrue.The
clinicianwhoordersa
testshouldalwaysbe
mindfulofthereasonfor
itsuse.Ifthetestresult
willnotalterthepatient's
management,thereis
littlepointinorderingit.
298
2-C
Choice and Interpretation of
Investigations


The following examples explore these concepts: Example 1
A group of 1,000 patients is tested for the presence of a certain disease (Table 1 ).
Results
SECTION 2-C. TABLE 1.
Test results in 1,000 patients affected with a disease.

Disease Present Disease Absent Total
Test Positive 18 (a) 20 (b) 38 (a+b)
Test Negative 2 (c) 960 (d) 962 (c+d)
Total: 20 (a+c) 980 (b+d) 1.000 (a+b+c+d)
The sensitivity is a/(a+c) which in example 1 is 18/20= 90%
The specificity is d/(b+d) which in example 1 is 960/980=98%
Results of research studies are often presented in this way. Sensitivity and specificity are epidemiological measures,
which need to be adapted for clinical use.
Predictive values
In considering individual patients with unknown disease status on presentation, the clinician wishes particularly to
know the predictive value of the test: that is, the likelihood of a positive or negative test confirming or excluding the
disease.
Positive Predictive Value is the proportion of persons testing positive who actually have the disease. (The false
positives comprise the 'false alarm rate').
Negative Predictive Value is the proportion testing negative who in fact do not have the disease. (The false
negatives comprise the 'false reassurance rate').
The clinical interpretation of the test may be given as:
The positive predictive value is a/(a+b) which in example 1 is 18/ 38 = 47%
The negative predictive value is d/(c+d) which in example 1 is 960/ 962 = 99.8%
The false alarm rate is b/(a+b) which in example 1 is 20/ 38 = 53%
The false reassurance rate is c/(c+d) which in example 1 IS
2/962 = 0.2%
Implications:
This test has a high specificity (98%). and thus is very helpful in excluding disease. If it gives a negative result, there is
a very high likelihood that disease is absent (99.8%negative predictive value). Conversely, there is a very low chance
that this negative result will give false reassurance (0.2%false reassurance rate). It has a lower sensitivity (90%),
and is much less useful in confirming disease. It misses 10%of cases with the disease and is more likely to be
positive and thus gives a false alarm in patients without the disease (53%) than in those with the disease (47%).
299
2-C
Choice and Interpretation of
Investigations


The ideal test should have absolute sensitivity all people within the test group with the condition will give a positive
test result.
The ideal test should also be absolutely specific (the converse of sensitivity) all people within the test group who do
not have the condition will give a negative result. If a test had a sensitivity of 1 and a specificity of 1 its probability of
error would be zero, and positive and negative predictive values would be 100%. With such an ideal test there would
be no false positives and no false negatives. The ideal test should also be attended by a very low level of random or
systematic errors of measurement. It should be highly reproducible: the error within and between observers should
be very low. No test absolutely measures up to all these ideals.
Biochemical tests are based on results from a healthy population sample. These results will usually have a Gaussian
normal distribution bell-shaped curve pattern when plotted. Laboratory reports often imply that results falling outside
two standard deviations from the mean are 'abnormal'. Such reports should be treated with caution, as by statistical
definition, one in 20 healthy individuals would fit this category. A similar spread of results is obtained when examining
patients with a particular disease. As there is overlap between the values in healthy and diseased patients, the choice
of a cut-off point between a positive and negative test result is artificial. In such tests, specificity and sensitivity are
likely to be inversely related. For example, in screening tests for prostate cancer, a serum prostate specific antigen
(PSA) level of 4 ng/mL is frequently used as the cut-off level. The test has a range of levels, and at high levels the risk
of prostatic cancer being present is increased. At lower levels more patients will be identified who have a condition
other than prostate cancer (prostatitis, benign prostatomegaly etc.). Reducing the cut-off level of the test to 2 ng/mL
would increase the test sensitivity but reduce the specificity. This will increase the number of false positives and create
anxiety for a greater number of patients who would subsequently be worked up to decide whether the test was truly
positive for prostate cancer.
Implications:
A highly sensitive test is an appropriate one for screening a population for an
abnormality. A negative result in a highly sensitive test will effectively rule out the diagnosis. The fact that the test is not
absolutely specific for the abnormality but also identifies a number of individuals, who are normal or have some other
condition, can subsequently be taken care of by applying to the identified group a further test which is highly specific
(but not so sensitive). In the highly specific test on individuals picked up by the sensitive screening test, a positive
result will effectively give a definite diagnosis. In the prostatic example above the subsequent test would be a tissue
biopsy confirming cancer.
Faecal occult blood testing for bowel malignancies is another example of a test with a relatively high sensitivity but
poor specificity, which will require further assessment (endoscopy) of those identified with a positive result. The
sensitivity is determined also by the biology of the condition. In colorectal cancer, a faecal occult blood test will not
detect neoplasms which do not bleed.
Other relevant aspects of testing are the frequency and importance of a disease and its duration and natural history.
Unfortunately, measures of predictive value have limitations in clinical practice, as they depend on the
prevalence of the disease in the population (Disease prevalence:
the number of people with the disease in the test population at the time of testing. This should not be confused with
disease incidence: the number of new cases of the disease occurring in the population over a set time interval).
2-C
Choice and Interpretation of
Investigations


The utility of a test is influenced by population differences. Suppose the same number of affected patients (20) with
the disease illustrated in Example 1 in a population of 1,000, were spread about a population 10 times the size
(10,000 Example 2, Table 2):
The prevalence of the disease is
20
1000
20
10000
= 2% (Table 1)
=0.2%(Table 2)
(a+c)/ (a+b+c+d) which in Example 1 is
and in Example 2 is
Example 2
SECTION 2-C. TABLE 2.
Test results in 10,000 patients in a region of lower disease prevalence.

Disease Present Disease Absent Total
Test Positive 18 (a) 200 (b) 218 (a+b)
Test Negative 2 (c) 9,780 (d) (c+d)
Total: 20 (a+c) 9,980 (b+d) 9,782 10,000 (a+b+c+d)
The test sensitivity (90%) and specificity (98%) are unchanged in Example 2 from Example 1 However, in this larger
population, with a disease prevalence of 0.2%(compared to a prevalence of 2%in Example 1 ), the negative
predictive value has increased to almost 100%, while the positive predictive value is now only 8%.

The predictive value of a positive test depends on the frequency of disease in the population

Example 3: breast cancer evaluation
The following table gives results of a highly specific test (T: needle core biopsy) performed in 600 women showing
focal mammographie abnormalities, to diagnose or exclude the worst case disease (D: breast cancer).
300
Test(T) Patients with cancer Patients found free of

Needle core biopsy subsequently confirmed cancer subsequently Total

(D positive) (D negative)

Needle core biopsy 143 2 145
Positive for cancer a b

Needle core biopsy 15 440 455
Negative for cancer c d

Total Breast cancer 158 Benign Conditions 442 600
SECTION 2-C. TABLE 3.
Test results for 600 women with focal mammographie abnormalities.
2-C
ChoiceandInterpretationof
Investigations
Sensitivity of test (true positivity) a/( a+c) 143 /158 91%
Specificity of test (true negativity)
d /(b+d) 440/ 442
99%
False Positivity Rate b/ (a+b) 2/ 145 1 %
False Negativity Rate c/( c+d) 15 /455 3%
Positive Predictive Value a/( a+b) 143/145 99%
Negative Predictive Value d/(c+d) 440/455 97%
False Alarm Rate b/(a+b) 2/145 1 %
False Reassurance Rate c/(c+d) 15/455
3%
Prevalence of breast cancer in test population (a+c)/( a+b+c+d) 158/600
26%
Implications:
Positive and negative predictive values are seen to be strongly affected by the characteristics of the test
(sensitivity and specificity) and by characteristics of the population (disease prevalence). Here 26% of those
showing focal abnormalities on mammography (the original screening test) ultimately proved to have cancer.
Core biopsy had very good (99%) positive predictive value, but not quite so good (97%) negative predictive
values. A small but not insignificant group (3%) had cancer found at operation done to remove the
mammographie abnormality, after preoperative false reassurance.
Likelihood ratios and scoring systems
Use of likelihood ratios, which may be considered as measuring the 'leverage' of a test, is a method of
separating the characteristics of a population from the inherent value of a test. In assessing the result of a test,
the clinician may follow the following thought process:
Depends on test characteristics

Depends on population
Before performing a test, the clinician should have an idea of the possibility of disease in
the population. This can be expressed mathematically as prevalence probability (a+c)/(a+b+c+d), or more
usefully here as 'prior odds', which is the ratio comparing the number of patients with disease to the number of
patients without disease, or (a+c)/(b+d)
The usefulness of a test is measurable by the likelihood ratio. The likelihood ratio of a positive test is the
frequency of a positive test in disease compared with its frequency without disease, or [(a)/(a+c)]/[(b)/(b+d)].
The value in example 1 is thus [(18/20)]/[(20/980)]. or approximately 50.
301
2-C
Choice and Interpretation
ofInvestigations


Multiplyingtheprioroddsbythelikelihoodratiogivesrevisedodds,orchancesthatdiseasewillbepresent,
comparedwithabsent:

Inexample1,theprioroddsare20/980,orapproximately0,02infavourofdisease(or50:1against);revised
oddsare0.02x50=1,i.e.1:1ora50:50'toss-up.
Inexample2,theprioroddsare20:9980or0.002infavourofdisease(or500:1against);revisedoddsare0.002
x50=0.01infavourofdisease(or10:1against).
Implications:
Itshouldbenotedthatthelikelihoodratiooftenhasadifferentvalueforapositiveoranegativetest.Ifthetestis
negative,the likelihood ratio (negative) isthefrequency of a negative test without disease,compared with a
negativetestindisease,or[ d/ ( b+d) ] / [ c/ ( a+c) ] . Thisisnotthereciprocalofthelikelihoodratio(positive);a
testwillnotbeequallyusefulforconfirmingorexcludingdisease.
Inexample1,thelikelihoodratio(negative)is[(960/980)/(2/20)]orapproximately10.Inthatexample,where
theprioroddsofnodiseasewere50:1,therevisedoddsagainstdisease,givenanegativetest,arenow500:1.
Inexample2,theprioroddsofnodiseasewereapproximately500:1,therevisedoddsarenow5000:1.
Various scoring systems have been proposed, which follow a similar logic, using addition of mathematically
derivedscores,ratherthanthemultiplicationofoddsandratios.Whilesomebelievethatadditionof'weights'in
this way may parallel the clinician's thought processes more closely, these systems have not been widely
adopted.
TestError
Thepossibilityoferrorinanytestmustthusalwaysbeconsidered.Mostdisturbingofallistheprospectofa
mistakeinthedistinctionbetweenlifeanddeath.Nowthatthetransplantationofcadaverictissueshasbecome
aclinicalreality,andbecauseprocurementofsuchtissueswithinashorttimeofdeathisessentialtosuccess,
theunequivocalearlydiagnosisofbraindeathhasdemandeddiagnostictestsofscrupulousandunparalleled
stringency.Aswillbeseeninthebookexamplerelatingtobraindeath,notethatvirtuallyallthetestsforbrain
deathinvolvedirectobservationofthepatientbyatleasttwoexperiencedcliniciansratherthantheapplication
ofpotentiallyimperfecttechnologies.
USEFULNESSOFTESTS
1
The more tests that are performed on a patient the more likely it is that one or another will give a falsely
abnormalresultbypurechance.Anotherimportantfactoristhatmultipledeviationsfromnormalityarecommon
inanyindividual,especiallyinanageingpopulation.So,ifmultiplediagnostictestsareappliedtoagivenpatient
in a mindless scattergun fashion, it is almost inevitable that one or more will be reported as positive. These
positiveresultsmaybetrueorfalse,importantorunimportant,relevantorirrelevanttothepatientsproblem.
Relatively simple clinical problems can all too easily become lost sight of in the maze created by multiple
investigations.



1Glass,R.D.,Di agnosi s: a br i ef i nt r oduct i on, OxfordUniversityPress,Australia,1996
302
2-C
Choice and Interpretation of
Investigations


Diagnostic utility of a particular test is thus determined by:
the prevalence and importance of the condition tested for;
the diagnostic accuracy of the test and how it compares to that of other well established
tests;
the invasiveness of the test and the risk to the patient from its performance;
the test's cost and availability; and
whether the outcome of the test will influence the management of the patient's complaint.
Case Example
The following case study demonstrates the optimal integration of morphologic and functional
imaging and biochemical investigations with diagnostic and management plans and pathways in
a patient with an unexpected incidental finding on initial investigation.
A 66-year-old man presented to his general practitioner after an episode of right upper quadrant
abdominal pain, which had resolved after four hours. Clinical examination was noncontributory.
An abdominal ultrasound was ordered with a provisional diagnosis of gallstones and associated
biliary pain. The ultrasound showed a normal gall bladder and biliary system free of stones, but
revealed a focal round solid mass 5 cm in diameter, above the right kidney, inferior to the liver,
and in the position of the right adrenal gland. (Figure 1 )

SECTION 2-C. FIGURE 1.
Ultrasound findings
An abdominal CT was next done to delineate more accurately the pathology. Intravenous and oral
contrast material were used to enhance imaging. (Figure 2)
The mass was confirmed to be a focal 5 cm round, solid mass within the right adrenal gland,
extrinsic to the posteroinferior right lobe of the liver, and to the right of, and lateral to, the inferior
vena cava, and superomedial to the right kidney. The mass had smooth borders and appeared
well encapsulated without any evidence of infiltration of surrounding structures. The left adrenal
was normal and no other intra-abdominal mass or pathology was noted.

303
2-C
Choice and Interpretation
of Investigations
SECTION 2-C. FIGURE 2.
Computed tomography (CT) of abdomen
Was this adrenal mass relevant to his symptoms, or was It an 'Incidentaloma '? Small benign
adrenal adenomas of no clinical relevance are very common in patients of this age, but are
mostly less than 2 cm in diameter. This one is larger (5 cm) and masses of 5 cm or above are
more suspicious of malignancy. Removal is often advised for larger tumours even it
nonsymptomatic.
He was referred for specialist opinion.
Further detailed history with focused questioning was helpful. For the last 8 years he had
noticed 'funny turns ' episodically. These caused him to feel dizzy and nauseated, and were
associated with thumping and pounding in his chest, and a throbbing in his head thai lasted for
several minutes.
The experienced clinician recognised that such a pattern would be consistent with
catecholamine surges produced by an adrenal medullary tumour a phaeochromocytoma.
Family history can be important in phaeochromocytomas associated with multiple endocrine
neoplasia Type 2, with associated parathyroid hyperplasia (C cells) and medullary thyroid
cancer. No such family history was obtained here; and clinical examination of neck was normal
as was the serum calcium level.
There was no family history of hypertension. His father died of a stroke and his mother of
gastric cancer, both in their 80s. He had a past episode of a bleeding peptic ulcer 30 years ago
(duodenal ulcer is associated with phaeochromocytomas) and minor symptoms currently of
urinary hesitancy.
His prostate felt normal on rectal examination. He had no cough, chest pain, dyspnoea or
sputum and did not smoke (adrenal metastases from lung or other primary sites need to be
remembered).
304
305
2-C
Choice and Interpretation of
Investigations


The range of endocrine overactivities from an adrenal tumour can involve singly or in
combination:
Glucocorticoids from adrenal cortex: Cortisol excess (Cushlng syndrome). There were no
stigmata of hypercortisolaemia clinically no moon face, no hypertension, no body fat
redistribution, no cataracts.
Mineralocorticoids from adrenal cortex: aldosterone excess (Conn syndrome). Hypertension is
the common association here, together with hypokalemia. His electrolytes were normal.
Adrenal sex hormones from adrenal cortex: these may give virilisation in women.
Catecholamines from adrenal medulla (adrenaline and noradrenaline): phaeochromo-cytoma.
This seems most likely from the history. Again hypertension is a common association. Thus
the three most common adrenal tumours each can cause secondary hypertension, although
his blood pressure was normal on review. However hypertension may be episodic, particularly
with phaeochromocytoma.
Functional endocrine investigative studies were arranged. Modern endocrinological tests have
high specificity and sensitivity. Initial screening urinary or serum analyses are most reliable
following an episode of symptoms but do not of course localise the site of origin. A battery of
screening tests was ordered, as well as standard preoperative screening tests.
Full blood examination normal, Hb 151 g/L
Cardiovascular status ECG sinus rhythm, no evidence cardiac ischaemia
Echocardiogram no ventricular hypertrophy
Electrolytes normal
Coagulation studies no abnormalities
Blood glucose mild elevation
Specific urinary catecholamine excretion analysis can be for end products (VMA
vannylmandelic acid), or for adrenaline and noradrenaline themselves (normally 80%
adrenaline and 20% noradrenaline).
Urine 24 hour analyses for excretion of adrenaline and noradrenaline were done in this
case and were markedly elevated, with high levels of both noradrenaline and adrenaline
noradrenaline 1160 mmol/day (45-600), adrenaline 720 mmol/day (5-80).
As adrenal tumours may produce more than one hormone, a check for aldosterone effects was
also done measuring plasma aldosterone, renin activity levels, and aldosterone/renin ratio. All
were within normal range. Hydrocortisone (Cortisol) levels were also normal.
The diagnosis of right adrenal phaeochromocytoma was thus definitively established, and
Conn and Cushing syndrome excluded. But was the tumour only at one site?
Phaeochromocytoma often causes attacks of episodic hypertension only and can be at multiple
sites. It is designated the 10% tumour 70% are bilateral, 10% are at extra-adrenal sites, 10%
are malignant. Previously, extended open laparotomy was required to check fully for other
abdominal sites the opposite adrenal, the retroperitoneum down to the pelvis, the urinary
bladder, and in the presacral area around the great vessels. Open surgery required large
incisions, major surgical dissection of hazardous tissue planes, and inpatient stays of up to 2
weeks.
The advent of nuclear medicine has enabled accurate preoperative functional imaging to confirm
one or multiple sites precisely. Extra-adrenal tumours secrete noradrenaline only and are almost
always associated with hypertension, but the opposite adrenal always needs to be excluded as a
source.
2-C
Choice and Interpretation of
Investigations


Final investigation: Radionuclide localising scan
Metaiodobenzylguanidine scan (MIBG): A radioiodine labelled agent (MIBG). which is taken up
by catecholamine precursors, is injected. Abdominal scintigraphy will localise the functioning
tumour as illustrated (Figure 3). The test is specific and sensitive.

SECTION 2-C FIGURE 3.
Functional nuclear scan for
catecholamines, showing hot spot
below liver on right
The scan confirmed a single hot area at the site of the right adrenal with no activity in the left
adrenal or elsewhere. Preparation of the patient for surgery now began. Preoperative elective
catecholamine blockade over a period of 1-2 weeks has now virtually eliminated the hazard of
operative adrenal crisis due to a catecholamine surge with life-threatening hypertension.
Adrenaline and noradrenaline stimulate a and /3 (vascular and cardiac) receptors. Initial
a-receptor blockade was begun with phenoxybenzamine, followed by (1-receptor blockade
(propranolol) after a-blockage had occurred.
During surgery nitroprusside and phentolamine should be available to control bleed pressure
swings precisely. Laparoscopic surgery is particularly applicable to well localised functioning
adrenal tumours. Excellent views are obtained, separation of the tumour from major adjacent
vasculature is facilitated, required hospital stay is reduced, and rapid convalescence ensured.
Only very large tumours, malignant tumours, or evidence of tumours at multiple sites are
contraindications to a laparoscopic approach. The operation was uneventful with removal of the
right adrenal and its contained tumour. His convalescence was straightforward with discharge
from hospital within 2 days of surgery.
306
2-C
Choice and
Interpretation of
Investigations

SECTION 2-C. FIGURE 4.
Adrenalectomy specimen
Diagnostic utility was appropriate in this instance for each of the carefully planned sequential
investigations across a spectrum of imaging, biochemical, and radionuclide tests, leading to
precise diagnosis and focused surgery with every expectation of complete cure.
OFFICE TESTS USED IN PATIENT ASSESSMENT
Body Temperature Measurement
This test illustrates many of the points discussed. Temperature recording is easily performed and
inexpensive, can be done accurately, is non-invasive and effectively free from risk. In terms of
detecting abnormality, it is extremely sensitive, but very low in specificity. It is a cost effective test
that can and should be applied to almost all clinical problems. A significantly elevated body
temperature indicates (in the absence of factitious malingering) an organic inflammatory or
infective ailment. The test is thus an excellent all round screening measure.
Urinal ysis
This shares many of the performance characteristics of temperature measurement and can be
applied at minimum cost to virtually all clinical patients. Observer error has been reduced to a
minimum by the development of user-friendly dipsticks. These can provide highly specific and
highly sensitive identification of glycosuria, proteinuria, biliuria, haematuria and other
abnormalities. Medieval manuscripts used depictions of inspection of a urine flask (urinoscopy)
as a convenient symbol of the medical practitioner a convention based on clinical reality at the
time. Modern technology has enhanced rather than diminished the utility of urinoscopy/urinalysis
in diagnosis.
Urinary positivity for glucose will depend upon renal threshold. Blood glucose fingerprick analysis,
now also readily available by user-friendly office instruments (glucometer), allows rapid
identification of hyperglycaemia and can often establish the presence of diabetes. An elevated
random blood glucose level over 11 mmol/L will effectively confirm the diagnosis of diabetes.
Glycosuria and glycaemia here serve as complementary screening and diagnostic tests high
sensitivity screening augmented by specific diagnostic testing.
307
308
2-C
Choice and Interpretation of
Investigations


Urine tests for pregnancy diagnosis
Another very widely used urinary test for office or home use, which gives results within a few minutes, is urinary
detection of human chorionic gonadotrophin (hCG) using monoclonal and polyclonal antibody test strips. Positive
testing for pregnancy can occur from the first day of the missed period with a sensitivity of 25 mu/ml. Sensitivity and
specificity progressively increase thereafter as the pregnancy progresses.
In many clinical consultations with women of childbearing age with abdominal pain, particularly if accompanied by
menstrual irregularities, a spot urinary pregnancy test is prudent and often diagnostically helpful.
Urine tests for diagnosis of ovulation time
Home monitoring of urinary luteinising hormone (LH) antibody from 17 days before the expected period can detect the
LH surge indicating that ovulation will occur within 24-36 hours.
Electrocardiograph/electrocardiogram (ECG) Previously a quite sophisticated test, ECG is now increasingly
available for on-the-spot office consultation and provides sensitive and specific information regarding cardiac rate and
rhythm and cardiac function, complementing cardiologie history and examination.
Ultrasound/Doppler probes
Hand held battery-operated ultrasound probes can be used to more accurately identify arterial or venous blood flow,
aiding diagnosis of peripheral arterial insufficiency, arteriovenous shunting, or venous obstruction and incompetence.
In the wards they can aid physical examination to detect a full or empty urinary bladder in postoperative patients.
Mass screening of populations, used in our community to identify common life-threatening diseases (cancer of
breast, colon) and cardiovascular disease (coronary artery disease and stroke), involves mammography, faecal occult
blood examination, measurement of blood pressure and serum lipids, and other programmes. The effectiveness of
these programmes is determined by sensitivity and specificity of the tests employed and their cost, plus the
prevalence and importance of the disease in the community. The ultimate goal of such screening programmes is to
diminish mortality by early detection of diseases, or by detecting persons who are at high risk of developing disease
and introducing preventive strategies. The additional requirement of cost-effectiveness in achieving such goals often
requires years of prospective study.
309
2-C
Choice and Interpretation of
Investigations


USE OF INVESTIGATIONS IN CLINICAL PROBLEM-SOLVING: CHOICE OF
INVESTIGATIONS
Tests used to aid clinical diagnosis of the patients presenting problem should utilise discriminative strategies rather
than the cumulative strategy of performing more and more tests in the hope that something will turn up. Tests used
discriminatively and with appropriate perception and perspective will enable the diagnostic process to move along
appropriately focused lines to best advantage.
Data collection from focused history and physical examination leads to the discriminative clinician asking the
questions:
What is the patient's presenting condition?
What is the diagnosis?
What else could it be?
Have I enough certainty to stop testing and go on to treatment?
If more tests are required which are the best and in what sequence, and over what time interval?
For example, in diagnosing headache, acute headache is often part of an upper respiratory tract infection presentation
associated with other symptoms of general loss of well being. Chronic and recurring headaches are most usually due
either to tension headache, migraine or cervical dysfunction/spondylosis. Of the many other causes, warning flags
should be looked for in the clinical assessment to exclude temporal arteritis, subarachnoid haemorrhage and cerebral
tumour. A persisting headache is also an associated symptom secondary to a wide range of other conditions. CT head
scan is only required in the minority of instances, when an intracranial lesion causing cerebral compression with
increased intracranial pressure is suspected.
Low back pain and neck pain are most often due to temporary soft tissue musculo-ligamentous strains ('nonspecific
mechanical back pain'). Precise anatomical or pathologic diagnosis is often not possible. The place of investigations is
confounded by the facts that degenerative change in discs and facet joints which could be associated with
symptomatic pain are found in a significant proportion of nonsymptomatic individuals, especially those aged over 40
years, where the prevalence is likely to be at least 30%. Contrariwise in patients with chronic low back pain, no
significant organic pathology is demonstrable in around 30%of patients.
Plain X-rays will exclude serious bony lesions and may give evidence of soft tissue pathology, but CT (or nowadays
and increasingly, MRI scanning) gives the most accurate assessment of soft tissues; and despite its expense the latter
investigation of MRI is usually the preferred investigation in chronic spinal pain. Ultrasound is of limited use in spinal
pain, but ultrasound is usually the investigation of first choice in acute or chronic shoulder pain following injury.
However, ultrasound is very observer-dependent, and again MRI is likely to be more sensitive and specific.
With acute abdominal pain, a small group of patients with a catastrophic syndrome of 'acute abdominal surgical
emergency' requires urgent surgery with minimal preoperative investigations. In this case surgery is the major and
most urgent investigation, and leads directly to diagnosis and management of such causes as acute abdominal aortic
aneurysm rupture, and acute ischaemic strangulation of bowel. Upper abdominal pain which is less urgent, or chronic,
can be investigated by plain or contrast X-ray. ultrasound, isotope studies, endoscopy, CT or MRI together with a host
of biochemical and other laboratory tests. If gallstones are thought to be the most likely pathology causing abdominal
2-C
ChoiceandInterpretationofInvestigations
painultrasoundisthemostappropriatefirstinvestigation.Ultrasoundisnoninvasive,canbeusedbothinemergencyand
elective situations, displays the gallbladder wall and contents as well as the bile duct system and picks up many
associated or alternative diagnoses (particularly in liver, kidneys, pancreas and spleen). If peptic ulcer is thought most
likely, endoscopy is usually the best initial investigation and aided by biopsy can distinguish benign from malignant
lesions.PainthoughttoarisefromthepancreasislikelytorequireearlyCT.Theuseofcombinednoninvasivemodalities
such as helical CT or magnetic resonance cholangiopancreatography (MRCP) can now provide high sensitivity and
specificitywithhighresolutionimaging.
Newer techniques such as multi-slice CT (MSCT) are becoming the examinations of choice for assessment of various
bodysystemsandorgans.
MSCT allows greater information to be gained due to the thinner multiplanar slices acquired, which can be reviewed in
multipleplanesorinthreedimensionstogivesuperblydetailedimagesasillustrated(Figures5-9).
2

SECTION2-C.FIGURE5. SECTION2-C.FIGURE6. SECTION2-C.FIGURE7.
Spine Extremity Abdomen
SECTION2-C.FIGURE8. SECTION2-C.FIGURE9.
Angiography CTAngiography
2FiguresreproducedbypermissionofMIAVictoria,amemberofl-MED/MIANetwork;AGuidetoMulti-SliceCTScanning.

310
311
2-C
ChoiceandInterpretationof
Investigations


Inpatientswithjaundice,liverfunctiontestsareusuallyoflimitedvalueindiagnosingthecause
of the jaundice, but they provide information which must be taken into account in formulating
further diagnostic and management plans. Ultrasound and helical CT or MRCP comprise
investigative mainstays. Important investigations prior to surgical management include blood
coagulation tests and tests of renal function. Patients with suspected bowel pathology can be
investigated either by radiology or endoscopy. Colonoscopy is preferred to diagnose mucosal
lesions.
For focal subcutaneous lumps and focal skin lesions, in many instances no investigations are
requiredandanaccuratediagnosiscanbeobtainedfromtheclinicalhistoryandexamination.If
thelumporskinlesionisclinicallysuspiciousthenthedefinitiveinvestigationisoftenhistological
examination of an operative specimen. Preliminary diagnostic investigations are often done to
determine more accurately the physical nature of deep lumps. Ultrasound can differentiate
between cystic and solid lesions, while CT or MRI will give more precise diagnosis. Once the
decision is made that microscopic examination is necessary, fine needle aspiration cytology
(FNAC) is often a highly specific test, particularly for patients presenting with breast lumps
(palpableorpickedupbyimaging),orsubcutaneouslymphnodeswellings.Aspirationcytology
gives cytological rather than histological diagnosis; but using flow cytometry and assessing
surfacereceptorstodifferentiatesubsetsofTandBlymphocytescandiagnoseanddifferentiate
polyclonalandmonoclonallymphomas,asdiscussedinSection4-A.
Cytological studies may also demonstrate the likely origin of metastatic lesions by finding
squamousneoplasticcellsinalymphnodeneckswelling,orbyfindingapapillarythyroidlesion
innecklymphnodes.Primarygrowthsofpharyngolarynxandthyroidcanbesmallandoccultin
association with larger nodal metastases. Accurate cytology can point the way to a further
appropriatesequenceofdiagnosticinvestigations.Finallycorebiopsybypercutaneousneedling
iswidelyusedand isthe preferreddiagnosticmethodforsolidbreastlumpsand otherdeeper
lumpswheretissuediagnosisisrequired.
Intheexampleswhichfollow,candidatesshouldexercisecareinthechoiceandinterpretationof
investigationinordertodirectandfocusdiagnosticandmanagementpathways.

ReubenDGlassandVernonCMarshall
312
2-C
Choice and Interpretation
ofInvestigations

2-CChoiceandInterpretationofInvestigations
CandidateInformationandTasks
MCAT058-064
58 PositivetestforhepatitisCina26-year-oldwoman
59 Diagnosisof'braindeath'priortoorgandonation
60 Breastbiopsyconcernsina20-year-oldwomanwithafamilyhistoryofbreastcancer
61 Anelbowinjuryinan11-year-oldschoolgirl
62 Suddenonsetofchestpainandbreathlessnessina20-year-oldwoman
63 Atypicaluretericcolicina25-year-oldman
64 Investigationformalefactorinfertilityina25-year-oldman
313
058
Candidate Information and Tasks



Condition 058
Positive test for hepatitis C in a 26-year-old woman
CANDIDATE INFORMATION AND TASKS
You are working in a general practice. Your next patient is a 26-year-old woman who has been sent to see you
because she was tested for hepatitis B and C and for HIV when she attended the Red Cross blood bank as a blood
donor one week previously and was found to be hepatitis C positive.
She has just been notified that she was found to be hepatitis C positive and advised to see her local doctor for further
assessment. Other blood tests were negative for both hepatitis Band HIV.
She had never given blood before, and had not been tested for any of these infections previously.
YOUR TASKS ARE TO:
Take a relevant history from the woman.
Advise her about subsequent management and likely prognosis.

The Performance Guidelines for Condition 058 can be found on page 321
314
059
Candidate Information and Tasks



Condition 059
Diagnosis of 'brain death' prior to organ donation
CANDIDATE INFORMATION AND TASKS
You are working in a general practice. Your next patient is a secondary school science teacher.
Next week a doctor from the Australian Kidney Foundation is going to speak at a class seminar to all the Year 11
students about organ transplantation and the donation of organs and tissues from donors who have died. The teacher
is to chair the seminar which has been titled The Gift of Life'.
The teacher has heard about 'brain death' and found the protocol below on the internet and printed it out. He wants you
to explain it to him in understandable language to help him comprehend the implications and facilitate his
chairmanship.
CONDITION 059. TABLE 1.
Brain death protocol.

PREDETERMINED CRITERIA BEFORE TEST
Core body temperature > 35 C
No central nervous system (CNS) depressant drugs for > 48 hours (longer if CNS depressants given in large
amount or for a long time)
No neuromuscular blocking drugs for > 12 hours
No endocrine problems, eg hypothyroidism, hypopituitarism
PaC02 > 50 mmHg
No hypoglycaemia
TESTS
1. Pupils fixed and unresponsive to light
2. Absent corneal reflexes
3. Absent pain response in cranial nerve distribution
4. Absent gag reflex on endotracheal tube movement
5. Oculocephalic reflexes absent (no 'dolls' eyes' response)
6. Vestibulo-ocular reflexes absent (no nystagmus)
7. No spontaneous respirations after 10 minutes (patient ventilated on 100% oxygen at a rate of 4 breaths/min with a
tidal volume of 7 mL/kg). Arterial blood gases taken at 5 and 10 minutes.

BRAIN DEATH
Diagnosis to be made by two doctors independently including the intensive care consultant. Neither will be a member
of the transplant team where organ donation is considered.
Two groups of tests, preferably separated by 24 hours.
The results of examination must be recorded in the case notes or a suitable devised form.

YOUR TASK IS TO:
Discuss the subject with him and respond to his queries.

The Performance Guidelines for Condition 059 can be found on page 325
315
060
Candidate Information and Tasks



Condition 060
Breast biopsy concerns in a 20-year-old woman with a family
history of breast cancer
CANDIDATE INFORMATION AND TASKS
Your next patient for office consultation in a primary care community practice clinic is for review of a 20-year-old single
woman whom you saw four weeks ago with a complaint of cyclical mastalgia for the last six months. Physical
examination of the breasts was normal. She had no previous history of breast problems.
Her 50-year-old mother, also a patient of the clinic, had a Stage 1 breast cancer treated by mastectomy and axillary
dissection five years ago and is well on follow up.
You ordered an ultrasound of the breasts in this young woman, which showed an impalpable, focal well-circumscribed
solid parenchymal lesion in the right breast 1 cm in diameter consistent with a fibroadenoma.
You referred her to the female surgeon who treated her mother, who suggested an ultrasound-guided percutaneous
core biopsy to confirm the imaging diagnosis of benign fibroadenoma. The patient was also reassured that if this
showed, as expected, a benign lesion, surgery would not be required, and she could be observed clinically with
periodic ultrasound assessments.
The patient is unhappy with this advice and feels she would like the lump removed and has come back to you to
discuss this further. She is worried that the lump may be malignant or will become so, and feels that just taking a piece
of it will leave her still worried.

YOUR TASK IS TO:
Discuss her concerns with her and advise her on the future management you would propose.

The Performance Guidelines for Condition 060 can be found on page 329
061
CandidateinformationandTasks



Condition 061
Anelbowinjuryinan11-year-oldschoolgirl

CANDIDATEINFORMATIONANDTASKS
YouareworkinginahospitalEmergencyDepartment.Emilyan11-year-oldschoolgirl,fellatschoolinjuring
herrightelbowwhichisswollenandpainful.YouarrangedforX-rayswhichhavebeentakenandareshown
below. You are interviewing Emily's mother after examining Emily and her X-rays. The elbow region was
swollen,painfulandtender,withmarkedpainonattemptedmovement.Therewerenosignscausingconcern
onexaminationofthehands.
YOURTASKSARETO:
Advisetheparentregardingdiagnosisandtreatment.
Answerquestionsfromtheobservingexaminerneartheendoftheinterview.
CONDITION061.FIGURE1. CONDITION061.FIGURE2.
ThePerformanceGuidelinesforCondition061canbefoundonpage331
316
062
CandidateInformationandTasks

Condition 062
Suddenonsetofchestpainandbreathlessnessina20-year-oldwoman

CANDIDATEINFORMATIONANDTASKS
This young woman has presented to the Emergency Department of the local hospital with the
sudden onset of right sided chest pain and breathlessness while walking to work. She is
otherwise in good health and is a nonsmoker. Physical examination of the chest showed no
definite abnormality. Her breathlessness is less now. A chest X-ray has been taken, and is
illustratedbelow.
YOURTASKSARETO:
Examineandinterpretthepatient'schestX-ray.
Explaintothepatientthediagnosisandhowsheshouldbetreated.
There is NO need to take any further history from the patient NOR repeat the physical
examination.
CONDITION062.FIGURE1.
ThePerformanceGuidelinesforCondition062canbefoundonpage334
317
063
Candidate Information and Tasks



Condi t i on 063
Atypical ureteric colic in a 25-year-old man
CANDIDATE INFORMATION AND TASKS
You are a medical officer in the hospital followup clinic. A few days ago your next patient, a
25-year-old driver, previously in good health, attended the Emergency Department with very
severe acute colicky mid-line lower abdominal pain. Abdominal examination was normal.
Because the patient's urine tested positive for blood, a diagnosis of atypical ureteric colic was
made. The pain was controlled by an injection of pethidine. A plain X-ray of the abdomen was
normal so an intravenous pyelogram (IVP) was arranged. The films are available for you to
review (see below), but a formal report from the radiologist has not yet been received.
The patient is seeing you today to find out the result of the IVP. He is now well and has been
straining his urine but no calculus has been found.
YOUR TASKS ARE TO:
Examine the IVP film, and give a commentary to the examiner.
Explain the X-ray findings to the patient.
Advise the patient about further management.
CONDITION 063. FIGURE 1.
Intravenous pyelogram
The Performance Guidelines for Condition 063 can be found on page 337
318
319
064
Candidate Information and Tasks



Condition 064
Investigation for male factor infertility in a 25-year-old man

CANDIDATE INFORMATION AND TASKS
A married couple (husband 25, wife 23 years) have been trying to conceive for the last 12 months.
Examination of both the husband and the wife is normal. Investigations arranged by you, from a
general practice setting, have shown she is ovulating each month, and has patent Fallopian
tubes.
The husband's recent semen analysis is not normal. His result is as follows:
SEMEN ANALYSIS
Collected after three days of abstinence.
Examined 30 minutes after collection by masturbation, normal values in brackets
Volume 6 mL (2-6 mL)
Count 2 million/mL (Greater than 20 million/mL)
Motility 20% (Greater than 40%)
Velocity 20 microns/second (Greater than 30 microns/second)
Abnormal morphology 95% (Less than 80%)
Antisperm antibodies nil (Nil)

The husband has come to see you today for the result of the semen specimen. His wife is aware
of her results. She was unable to come today.
When you examined him previously, you found no abnormality on general or genital
examination. Both testes were normal in size (20 mL estimated volume), felt normal in
consistency, there was no indication of a varicocele or hydrocele.

YOUR TASKS ARE TO:
Take a further relevant and focused history from the husband in regard to the results
obtained.
Advise the husband regarding the couple's fertility problem. The
Performance Guidelines for Condition 064 can be found on page 340
320
2-C
ChoiceandInterpretationof
Investigations


2-CChoiceandInterpretationofInvestigations
PerformanceGuidelines
MCAT058-064
58 PositivetesttorhepatitisCina26-year-oldwoman
59 Diagnosisof'braindeath'priortoorgandonation
60 Breastbiopsyconcernsina20-year-oldwomanwithafamilyhistoryofbreastcancer
61 Anelbowinjuryinan11-year-oldschoolgirl
62 Suddenonsetofchestpainandbreathlessnessina20-year-oldwoman
63 Atypicaluretericcolicina25-year-oldman
64 Investigationformalefactorinfertilityina25-year-oldman
321
058
Performance
Guidelines



Condition 058
PositivetestforhepatitisCina26-year-oldwoman
AIMSOFSTATION
To assess the ability of the candidate to take a focused history assessing the possible
mechanismforherbecominghepatitisCpositive,andthentoappropriatelyadvisethepatientin
regardtothemodeofcontractingthedisease,thetestsrequiredtoassessthecurrentactivityof
thedisease,thelikelylong-termoutcome,whetheranytreatmentislikelytobehelpful,thelikely
possibilityoftransmittingthediseasetootherpeople,andtheneedfornotificationofthedisease,
EXAMINERINSTRUCTIONS
Theexaminerwillhaveinstructedthepatientasfollows:
The candidate will be expected to take an appropriate history from you to determine how the
hepatitisCinfectionoccurred,andwhetheryouarelikelytospreadittoanotherindividual.The
candidate will also be expected to provide you with information concerning the investigations
now required to assess any potential adverse effect of the hepatitis C virus on your body and
whether you have cleared the infection spontaneously. Advice concerning your subsequent
management will also be given and the need for notification of the disease to the local state
healthdepartmentwithinfivedays.
Thefollowinghistoryislikelytobesoughtfromyou
(giveanswerstospecificquestionsasoutlinedbelow):
InformationinregardtolikelycauseofthehepatitisCinfection
~Youwereanintermittentintravenousdrug-useroveratwo-yearperiod,butlasthad
a'shot'aboutsixyearsago.
~Onoccasionsyouhadsharedneedleswithafriend.
~Youhaveneverhadabloodtransfusionorgivenbloodproducts
~Youhavehadtwosexualpartnersinyourlife.Thefirstrelationshiplastedthreeyears
andthesecond,thecurrentone,haslastedfouryears.Youmarriedyourcurrent
partnertwoyearsago.
~Nofamilyhistoryofhepatitisofanysort.
~Nopreviousoperationsorillnesses.
~Notattoosorbodypiercing
~Youworkinahospitalasacleaner.
~Youneverhadaneedlestickinjury
Youfeelwell,haveanormallevelofenergyandnodifficultiesatwork
Inresponsetoanyquestionsaboutsymptoms(suchaschangeinappetite,changeinweight,
skin changes, abdominal discomfort, bowel function) reply that there have been no such
problems.
Youralcoholintakeis1or2glassesofwineaday.
Youaretakingtheoralcontraceptivepill(Microgynon30)andwishtohaveachildinabout
twoyearstime.
Youhavenopasthistoryofclinicalhepatitis.Youhaveneverbeenjaundiced.
322
058
PerformanceGuidelines
After taking your history, the candidate should explain to you that the infection is likely to have occurred as
a result of your intravenous drug use.
You are likely to be advised that you need blood tests to define whether the infection has cleared
spontaneously from your body (polymerase chain reaction [PCR] test) and whether it has had any effect on
your liver function (liver function tests). Knowledge of these results will then determine what the
subsequent risks to you are.
Questions to ask if not already covered:
'Do I need to have any more tests?'
'Will I be able to have a baby?'
'Can anyone catch this infection from me?'
'Must you notify this to the health department?'
EXPECTATIONS OF CANDIDATE PERFORMANCE
History taking should include:
Information in regard to the likely cause of the hepatitis C infection as outlined in instructions
to patient.
Current status
She has no current symptoms of liver disease: no tiredness, bruising, itch, appetite change,
abdominal discomfort, gastrointestinal bleeding, leg swelling.
Alcohol intake is moderate.
Investigations
Investigations required are those to assess any effect of hepatitis C on her liver, and whether the
actual viral infection has spontaneously cleared, (i.e. liver function tests and a polymerase chain
reaction test for hepatitis C virus [HCV PCR]).
In addition to the above tests, the possibility of blood group immunisation due to the use of
shared needles needs to be assessed by the indirect Coombs Test. If positive this will influence
the care required in a pregnancy.
Counselling
In counselling about hepatitis C and risk to the patient, the candidate is expected to know that
hepatitis C is a viral infection transmitted mainly via infected human blood. In most patients the
diagnosis is made only when the disease is established and chronic. In this patient, the exposure
was almost certainly at the time of intravenous drug use 6-8 years ago. In order to best identify
the risk of liver disease, LFTs and HCV PCR should be performed
An HCV PCR should be performed to help determine if the patient has spontaneously cleared
the infection.
In any patient, if serum alanine aminotransferase (ALT) is persistently normal (three
estimations over a six month period) the prognosis is good and it is likely no long-term
adverse liver effects will ever be found. In this patient if ALT is normal on the first test, given
the likely exposure was many years ago, the patient can be reassured, but further ALT
monitoring should still be advised.
058
323
PerformanceGuidelines

IfALTiselevated,referraltoagastroenterologistwouldbeappropriateforfullliverassessment,
includingbiopsy,asantiviraltherapy,includinginterferonandribavirin,maybeofuseifthere
issignificantfibrosisinthebiopsyspecimen.Ultimatelyliverfailureandlivertransplantation
mayberequiredinasmallpercentageofcases,andthepatientisalsoatincreasedriskof
hepatocellularcarcinoma.
When the above aspects have been sorted out, a decision can be made regarding the
advisabilityofapregnancy.Pregnancyshouldnotbealloweduntilatleast6monthshaselapsed
aftercessationofantiviraltherapy,ifthishasbeengiven,duetotheteratogenicityoftheribavirin.
If pregnancy is to be allowed, the oral contraceptive pill should be ceased and the pregnancy
awaited.Thechanceofthebabybeinginfectedbyverticaltransmissionduringthepregnancyis
about5%inpatientswhohaveapositivePCRforHCV.
CounsellingabouthepatitisCandrisktoothers
Prevention of infection of others can only be achieved by ensuring all people who come in
contact with her blood take appropriate precautions, and that the sharing of needles is never
done. There is no risk of hepatitis C transmission by hugging, kissing, casual contact, sharing
foodoreatingutensils.Howeveritisimportanttoavoidsharingobjectswithpotentialforblood
contamination, such as razors and toothbrushes. The risk of vertical transmission is very low.
Therearenorecommendationsagainstbreastfeeding.Theriskofspreadingthisinfectionduring
sexualactivityisextremelylow,andthereislittleornoevidencethatcondomusagewillbeof
valueinprotectingherhusbandfromhisverylowriskofgettinginfectedinthisway.HepatitisCis
anotifiablediseasenotificationisconfidential.
Patienteducation
The good candidate will seek to provide the patient with appropriate supplementary patient
educationmaterial.

KEYISSUES
Takingafocusedhistoryinregardtodeterminingthesourceoftheinfection.
Advisingthepatientappropriatelyregardingsubsequentcare,theriskofliverpathology,and
the need to ensure blood transmission does not occur, as this would be likely to result in
hepatitisCinfectionintherecipient.

CRITICALERRORS
FailuretorecognisetheneedforLFT(ALT)assessments.
Advisingthepatientofabenigncourseofdiseaseinallinstances.

COMMENTARY
Hepatitis C is a single stranded RNA virus. Risk factors for hepatitis C infection include intra-
venousdruguse(70%),sexualexposure(-10%),bloodtransfusion(6%),occupationalexposure
(3%), unknown (-10%). The risks of tattoos, bodypiercing and intranasal cocaine are not well
defined.Theviralinfectionisestablishedandchronicatthetimeofdiagnosisinmostpatients.If
identifiedearly,treatmentwithinterferonwithin3to6monthsofinfectioncanpreventchronicity
in98%ofpatients.
324
058
Performance Guidelines



The natural history of hepatitis C infection is that 15-50%resolve completely with no adverse end result and normal
liver function (i.e. PCR is negative and liver function and ALT are normal). The remaining 50-75%will have chronic
infection (PCR positive). Twenty percent of those with an elevated ALT will develop cirrhosis, of which 1-5%will
develop hepatocellular carcinoma, and 20%will develop liver failure requiring transplantation. LFTs and HCV PCR
therefore need to be done to assign the patient to the appropriate group.
This station requires that the candidate has knowledge of the natural history of hepatitis C and how this infection is
detected, monitored and treated. Good communication skills are required to address sensitive issues in a situation
where the patient is likely to be very anxious, having just been informed about a potentially serious infection. The
station examines the ability of the candidate to take a focused medical history, relating to potential source of the
infection, and any effects on her health. The patient needs to be advised about the necessary investigations (blood
tests for HCV PCR and LFTs) and why these are required. Counselling skills are evaluated as the candidate talks with
the patient about the possible effects of the hepatitis C virus on her health and the potential of passing on the infection
to others. There is a good opportunity at this first consultation to establish a good rapport, to give some basic education
about hepatitis C. to provide some reassurance about transmission risk, and to set the scene for the next visit when the
ordered test results will be discussed. Candidates should be aware that hepatitis C is a notifiable disease with
confidentiality maintained.
059
325
Performance Guidelines



Condition 059
Diagnosis of 'brain death' prior to organ donation
AIMS OF STATION
To assess the candidate's knowledge of the principles of diagnosis of brain death and its
certification.
To assess the candidate's communication skills in public education by discussing and explaining
aspects of cadaver organ donation in the context of brain death with a layperson.
EXAMINER INSTRUCTIONS
The standardised 'patient' in this instance is a secondary school science teacher with enquiries as
described. Responses and questions will depend upon the clarity of explanation and information
from the doctor.
Questions to be asked unless already covered:
'What does'brain death'mean?'
What are these predetermined criteria about?'
'Can you explain these tests to me?'

EXPECTATIONS OF CANDIDATE PERFORMANCE
You would expect the imminent medical graduate to understand clearly the principles, although not
necessarily the detail, of the diagnosis of brain death and its implications in gaining of consent for
cadaver organ donation as outlined in the commentary.
KEY ISSUES
Ability to discuss principles of 'brain death' to a lay person.
Ability to discuss principles of cadaveric organ donation for transplantation
CRITICAL ERROR - none defined

COMMENTARY
The candidate should be familiar with legislation in Australia (which is broadly similar to that
pertaining in most developed countries) providing for legal certification of death by either of two
methods:
permanent and irreversible cessation of heart beat and loss of cardiac function; and
permanent and irreversible loss of brain function
The concept of 'brain death', as an alternative to 'cardiac death' has important implications in
transplantation of organs and tissues from a cadaver donor. Removing organs once brain death
has been diagnosed and certified improves significantly the prospect of immediate function of the
organ graft in the recipient after revascularisation in its new host. Immediate graft function is
essential for successful heart and liver transplantation, and highly desirable in grafts of kidneys,
lung, pancreas, bowel and other tissues.
059
326
PerformanceGuidelines



Removaloforgansfromthebrain-dead,heart-beatingcadaverwithpermanentandirreversible
apnoea (cessation of spontaneous breathing) due to brain stem death, whose respiration is
maintainedbyartificialmechanicalventilationofthelungs,minimisesthecriticaltimeof'warm'
ischaemiabeforecoldperfusionoftheremovedorgans.Thus,immediatefunctionofthegraftin
itsnewhostcanbeanticipatedprovidedtotalischaemictimesafterorganremovaldonotexceed
thetoleratedtimeperiodsfortheindividualorgansaround6hoursforheartsandupto24-48
hoursforliver,kidneysandotherorgans.
Clearlythediagnosisofirreversibleandpermanentlossofbrainandbrainstemfunctionmustbe
anunequivocalandcertainone,withabsolutelynoprospectoferror.Weneedtestsofabsolute
specificityandsensitivity.
Theseveralcriteriaandtestslistedoutlinethewaysofensuringthatirreversiblelossoffunction
has occurred both in the higher cortical brain functions, and also in the functions of the brain
stem,wheretherespiratorycentreandcranialnerveoriginsareclustered.
Thecriterialistedfordiagnosisofbraindeathfirstrequiretheappropriateclinicalsetting(usually
massiveheadinjuryoracatastrophicstroke),andthepresenceofdeepandtotallyunresponsive
comawithpermanentlossoffunctionoftherespiratorycentre,sonospontaneousbreathingcan
occur because the brain stem centre for breathing has been irreversibly destroyed (and in the
absenceofrespiratoryactivity,cardiacarrestinevitablyfollowswithin30minutesorlessunless
ventilationisrestoredbyartificialventilation).
The other criteria are the exclusion of other possible contributors to prolonged coma
(hypothermia,continuingcentralnervoussystemparalysisfromdrugsorcurare-likerespiratory
depressants,orgrossmetabolicandendocrinedisturbances).
Theteststhenemployedaslistedinthecriteriaofbraindeatharediagnosticofdestroyedand
absentbrainstemreflexes,involvingsuccessivelythemidbrain,ponsandmedulla,sothatlossof
brain stem function is progressively confirmed from above downwards, testing reflexes
subservedbycranialnerves2through12viatheirsensoryandmotorpathways,andthebrain
stemreflexarcsfromhighesttolowestlevel.
Permanentirreversibleapnoea(failureofspontaneousbreathing)duetodeathoftherespiratory
centreisconfirmedovera10-minuteintervalinthepresenceofahighlevelofbuildupofcarbon
dioxide in the blood, which in the presence of a responsive respiratory centre will stimulate
spontaneousbreathing.
Criteriaforconfirmationoffindingsisbytwogroupsoftestsseparatedbyanappropriateperiodof
observation, and confirmed by two independent doctors, including a senior and experienced
clinician.

Meticulous application of these defined and universally accepted worldwide criteria has
ensured that brain death can be diagnosed clinically with absolute confidence and without
anyriskofmisdiagnosis.
Diagnosisofbraindeathismadebymeticulousclinicalobservationsandtestsanddoesnot
requireelaboratetechnologyforcertaintyofdiagnosis.

The doctor here has been put on the spot by the bluntness and directness of the science
teacher'srequest.Howshouldtherequestbestbehandled?Itmaybebestinitiallytobroadenthe
discussion into a general outline of the usual setting of cadaver organ donation and the tragic
circumstancesofsuddenandunexpecteddeathofalovedonehighlightingthemanysensitive
human,ethicalandculturalissueswhichmakeempathieand
327
059
PerformanceGuidelines



compassionatecommunicationbetweenthetreatingandtransplantmedicalteamsandgrieving
relatives and next of kin so essential. Certainly all doctors should be conversant with the laws
governing consent for donation given by the next of kin, but the request would come normally
fromanexperiencedseniormemberofthe intensivecareteam,andnotfromajuniorHospital
Medical Officer (HMO). Initial discussion with the candidate could then be followed by a
subsequentbriefinginwhichthedoctorcouldreadupaboutthemorespecificdetailsofdiagnosis
ofbraindeath andthescienceandethicsofcadaverorgantransplantationbeforemeetingthe
teacheragain.
This station is a rather extreme example of the increasingly common practice of patients
presentingtodoctorswithprintedinternetreportsrelatedtotheirpresentingproblem,oftenquite
detailedbutnot alwaysappropriateintheirperspectiveorapplicationtotheparticularproblem
posedbythepatient.
In this instance, the isolated table the teacher brought was presumably taken from a larger
generalaccount,whichwouldbemorerelevantthanthespecificsofagonochemicaleventsand
thespecificsoftestsusedtodiagnosepermanentandirreversibledeathofthebrainandbrain
stem. The essential principle is that the criteria and tests provide the legal basis for medical
certificationofthedeathofaperson(becausethebrainisdead)eventhoughotherfunctionsof
the body (heartbeat and machine-driven ventilation) are still going on with the support of
machinesandtransfusionssotheperson(whoisnowactuallyacadaver)'looksalive'.
Knowledgeablecandidatesshouldbeabletogiveageneraldescription,similartotheabove,of
braindeath.Inparticular,that:
all the vital brain stem centres have been destroyed, including the respiratory centre and
consciousness centre, so the condition presents as a totally unresponsive individual with
permanentlossofconsciousness,withpermanentlossofthecapacitytobreathe:
thevarioustestsdescribedaretestsofthesebrainstemandhigherfunctionstobecertainthat
allarepermanentlyandirreversiblydestroyedoverarepeatedperiodofobservation:and
all other potential influences on consciousness (like effects of drugs) have been eliminated
withcertainty.
Doctors should also know that the condition of brain death, its certification and its legal and
ethicalimplicationshavebeenratifiedbyallmajorreligions,andthatthetimeofbraindeathis
seenbytheologianstoequatewiththetimeatwhichthesoulleavesthebody.Itisalsoimportant
to understand that brain death means death of the individual as surely as does recognition of
deathbycessationofheartbeat.
Objections from next of kin to obtaining consent for organ donation after certification of brain
deathareusuallyculturalandemotionalandassociatedwithfearofmutilationofthebody.

The vegetative state comprises the condition of deep coma with present but ineffective
spontaneous breathing and with retention of other brain stem activities and reflexes, requiring
artificial feeding (and often respiratory assistance), and responding to a variety of stimuli. It is
NOTbraindeath.Knowledgeablecandidateswillstressthisfact.
328
059
Performance Guidelines



Much confusion in the public mind was initially stimulated by misinformed or unsubstantiated, (but sensational)
reporting of doctors removing organs from struggling and responsive patients in whom brain death was said to be
wrongly diagnosed.
Doctors always need to employ great empathy and compassion in obtaining consent in indicating to the relatives of
the brain dead individual what organs are to be removed for transplantation, and in answering direct questions from
them in regard to these and other matters.
329
060
Performance Guidelines



Condition 060
Breast biopsy concerns in a 20-year-old woman with a family history of breast
cancer
AIMS OF STATION
To assess the candidate's counselling and educational skills in a patient with concerns
about familial breast cancer risk.
EXAMINER INSTRUCTIONS
The examiner will have instructed the patient as follows:
You are very worried that this lump could be a cancer You feel it should be removed so
you don't have to worry about it anymore. If the doctor's reassurances are clear and
convincing, you are prepared to change your mind. If not, ask if you can have a second
surgical opinion
Opening statement:
'I think this lump should be removed'.
Questions to ask if not already covered:
'How can you be sure it's not a cancer?'
'Isn't it likely to turn into a cancer'?
'Cant I just have it out and then forget about it?

EXPECTATIONS OF CANDIDATE PERFORMANCE
Reassurance about likely benign diagnosis,
Reassurance that impalpable fibroadenomas are very common in nonsymptomatic
women on imaging and do not reguire excision and that they are not cancers and
do not become cancerous.
Reassurance that the biopsy takes several representative pieces and can save
unnecessary surgery and avoid potentially unsightly scarring.
Reassurance that with a homogeneous lesion such as this, the biopsy could be relied
upon to give a definitive diagnosis.
Sympathy for concerns of patient about cancer, and about continuing clinical and
ultrasound monitoring; reassurance of noninvasive nature of ultrasound monitoring:
reassurance of noninvasive nature of ultrasound
Assurance that if patient is still concerned, the surgeon would be likely to accede to
her wishes, and if not, she could be referred for a second opinion.
Whatever the patient decides, periodic followup with clinical and imaging reviews will
be advisable because of her family history and her concerns.
KEY ISSUES
Counselling and communication skills in dealing with an anxious patient.
Knowledge of pathology and natural history of breast fibroadenomas.
330
060
Performance Guidelines
CRITICAL ERROR - none d e f i n e d
COMMENTARY
The scenario illustrates a common problem. Breast cancer is very common in Australian women
and around 1 in 14 women will develop breast cancer in their lifetime. The risk is increased in the
presence of family history of breast cancer in first degree relatives, as is the case here.
This young patient requires regular clinical screening and appropriate imaging. The ultrasound
ordered by the family doctor was appropriate as initial investigation. Note that her original
problem (cyclical mastalgia) has now been replaced by the more serious problem of 'I have a
breast lump which could be cancerous' Her natural reaction (which might be the correct solution
to the problem) is 'I want it out.'
Benign impalpable (or palpable) fibroadenomas and other benign parenchymal lesions are very
common in this age group. We know from mass screening that benign lesions occur throughout
all stages of life, and that the natural history of fibroadenomas may be to remain unchanged, to
increase in size or to regress. No convincing evidence exists that benign fibroadenomas are
premalignant, and much collateral evidence on screening programme followup suggests that
they are not. Whether total excisional biopsy or partial core biopsy should be performed will
depend on circumstances, but an appropriate core biopsy would take five or more representative
samples and would be expected to give a definitively accurate diagnosis with minimal likelihood
of either a falsely negative or falsely positive result, and with minimal morbidity in experienced
hands. This lesion is impalpable and both clinical findings (normal breasts) and imaging findings
(typical ultrasound appearance of a benign lesion) already favour a benign fibroadenoma. But
these findings alone are no! enough, and pathological confirmation by biopsy is required
additionally to make our reassurance quite positive ('triple test check' clinical, imaging, and
pathology all confirmed and negative for cancer). Pathology can be determined by fine needle
aspiration cytology (FNAC) or by percutaneous image-guided needle core biopsy. Choice will
depend on circumstances and availability of expert cytology and pathology services; but core
biopsy will give a tissue diagnosis and has higher sensitivity and specificity, so is generally
preferred.
If the 'triple test' is negative, the lesion is virtually certain to be benign. The patient will require
continuing periodic clinical and imaging review from her family doctor and surgeon. The
surgeon's advice was therefore appropriate and concise, but she has not convinced the patient
that it is the right plan. The utility of any advice regarding management is only relevant and
helpful if patient acceptance is present. If this patient remains unconvinced and unhappy, despite
repeated reinforcement by the family doctor, clearly the best decision may be to agree to her own
wishes that the lesion is removed. This will require an image-guided needle localisation
operation. It is unlikely that the surgeon would not agree to this, even though she (the surgeon)
correctly regards core biopsy and observation as the best option.
061
Performance Guidelines



Condi t i on 061
An elbow injury in an 11-year-old schoolgirl
AIMS OF STATION
To assess the candidate's ability to identify a supracondylar humeral fracture on X-ray and advise
regarding treatment.

EXAMINER INSTRUCTIONS

The examiner will have instructed the parent as follows:
Your 11-year-old daughter, Emily, fell at school and now has a sore, painful swollen right elbow.
The candidate has finished examining your child and has examined the X-rays.
Opening statement and questions from the parent:
'Is the arm broken, doctor?'
'What treatment will she need?'
'What about school and writing?'

EXPECTATIONS OF CANDIDATE PERFORMANCE
The candidate would be expected to advise the parent and describe the diagnosis, initial
treatment plan and followup in response to the questions along the following lines: (Expected
responses in parentheses)
'Is the arm broken?' (Yes. Emily has fractured the arm bone [humerus] just above the elbow.
There is minimal displacement and no complications. She should get excellent results with full
functional recovery).
'What treatment will she need?'(\Ne will apply a back slab/plaster/splint to the elbow with a
bandage and sling. No anaesthetic will be needed. Pain relief will be ensured by paracetamol
as required, dose for age. She will need to keep the elbow in plaster for several weeks [4-6
weeks] as illustrated. She will need a first followup tomorrow to check plaster and fingers, and
that she is comfortable with the plaster and sling. The parent should report earlier if hand or
fingers swell further. She should sleep
with arm supported on a pillow.
Subsequent unrestricted use of hand
and fingers should be encouraged with
self-maintenance finger stretches).
'What about school and writing?' (Can
write as soon as finger movements
allow this. Can return to school when
pain eases in a day or two).
CONDITION 061. FIGURE 3. X-ray
after application of backslab
331
332
061
Performance Guidelines



The examiner should ask the following question at seven minutes:
'What complication is most to be feared in these fractures if they are displaced?' (Vascular njury to the brachial artery).
KEY ISSUES
Recognition of fracture on X-ray.
Understanding principles and practice of management of an undisplaced uncomplicated closed supracondylar
fracture.
Understanding of potential complications in this type of fracture.
CRITICAL ERRORS
Missing the diagnosis of fracture
Failing to arrange appropriate review and followup.
Failure to know of risk of vascular complications in displaced supracondylar fractures.
COMMENTARY
Supracondylar humeral fractures are common in children following falls on the arm or hand. Undisplaced fractures or
those with minor displacements can be treated without need for reduction by immobilising the elbow using a padded
backslab or plaster (leaving shoulder and wrist and hand free to move) in the position of function of partial
elbow-flexion of around 100 flexion.
Severely displaced fractures will require reduction under anaesthesia and similar splintage after alignment is checked.
A serious complication to be watched for is injury to the brachial artery from the anteriorly displaced upper fragment,
giving ischaemia of the hand and fingers shown by pallor, insensitivity and absent radial pulse.
Unless circulation is clearly restored after reduction, such ischaemia must be treated by open exploration of the
fracture site and the injured artery with restoration of adequate blood flow by vascular surgery, otherwise Volkmann
ischaemic contracture of forearm muscles can occur.
Fortunately in the majority of cases, displacement is minor or alignment is readily corrected and no vascular
complications are present; but circulation must always be checked by review after 24 hours, and parents and patients
advised to report earlier if symptoms of numbness, finger swelling or severe hand pain occur.
Immobilisation is usually only needed in children for 4-6 weeks and active mobilising exercises then begin.
Emily would be expected to make a full functional recovery after this injury and did so as llustrated (Figure 4).
061
Performance Guidelines
CONDITION 061. FIGURE 4.
333
334
062
Performance Guidelines



Condition 062
Sudden onset of chest pain and breathlessness in a 20-year-old woman

AIMS OF STATION
To assess the candidate's ability to recognise the right-sided pneumothorax on the chest X-ray
and explain the diagnosis to the patient. The candidate needs to reassure the patient and then
explain how the problem will be managed.

EXAMINER INSTRUCTIONS
The examiner will have instructed the patient as follows:
You developed chest pain while walking to work. A chest X-ray has just been done at the local
hospital and you are about to be informed of the result of this.
Opening statement
'I've got a bit of a pain in my chest and I feel a bit breathless' indicate site of pain which is
on the right below the clavicle and at the back in the same area.
You were walking to work when the pain came on suddenly. You were also breathless. The pain
is:
sharp and stabbing (if asked it is not tight, heavy or gripping);
not made worse with every breath
worse if you take a deep breath;
also radiating to your shoulder tip; and
moderately severe at onset, but easing now
You also have:
A feeling of breathlessness (not severe) at rest as well as on exertion.
An irritating dry cough - not severe or distressing
You are a nonsmoker and drink alcohol on social occasions only (2-4 standard drinks). You have
no known drug sensitivities. You live 2 km from the hospital and there are several others at home
most of the time.
Indicate area where pain is felt, upper chest, both back and front. Also indicate that you are
concerned about the cause of the pain and what the doctor will do to relieve it. Be cooperative
and answer the doctor's guestions without evasiveness
You have a moderate-sized pneumothorax, a partial collapse of the lung. The candidate must
make the correct diagnosis, explain it to you and how the problem will be managed Inserting a
catheter to take the free air out of the chest is a possible response, and admission to hospital
may be recommended. If so, ask if you could be treated at home.
335
062
Performance Guidelines



EXPECTATIONSOFCANDIDATEPERFORMANCE
Candidateshouldrespondalongthefollowinglines:
Responsetopatient
Pneumothoraxisthediagnosis,confirmedbychestX-ray.Apneumothoraxofthis sizemay
notneedactivetreatment.Itwouldbeprudentandreasonabletoadmitthepatientovernight
for observation and serial X-ray. Sending the patient home if she lives nearby is a less
acceptableoptionwithapneumothoraxofthissize.Recurrenceispossibleafterspontaneous
pneumothorax,andtherecurrencerateisapproximately35%onthesamesideand10-15%
onoppositeside.Mostrecurrencesoccurwithin12months.
Thegeneralconsensusregardingtheneedforintercostaldrainageisasfollows:
~If25-30%orlesslungcollapseandnosymptomsobserve.Thisisareasonableoptionin
thispatient.
~If25-30%orlesslungcollapseandpersistingsymptomsdrain.Thepatientmayfallinto
thiscategorywithobservation.Thepneumothoraxisaround30%andhersymptomsare
currentlymild.
~Ifgreaterthan30%collapse,whethersymptomaticornotdrain.
Displayingclinicalknowledgeandskills
Aetiologyofpneumothoraxruptureofblebonsurfaceoflung.
Nature of pain associated with pneumothorax possibly due to tear of adhesion as lung
collapses.
Associatedbreathlessnessdependsonsizeofpneumothorax.
Confirmatory investigation chest X-ray diagnostic. This is a moderate (25-30%)
pneumothorax.Itmayneedaformalchestdrain.InsertingacatheterwithaHeimlichvalveis
anoptiontobediscussedshouldthepneumothoraxincreaseinsize.
DemonstratingCommunicationskills
Reassuringapproachtopatientanxiousaboutthecauseofthepain.
Thepainshouldberecognisedasbeingofrespiratoryorigin,ratherthancardiac.

KEYISSUES
CorrectinterpretationofchestX-ray.
Explainingthediagnosisandappropriatemanagementtothepatient.

CRITICALERROR
FailuretoidentifythepneumothoraxonthechestX-ray.
062
PerformanceGuidelines



COMMENTARY
Spontaneouspneumothoraxisusuallyduetotheruptureofapreviouslynonsymptomaticbleb
on the pleural surface of the lung. Symptoms of breathlessness and local discomfort are
proportional to the size of the pneumothorax which is often small, in which case no active
interventional treatment is required. Elective intercostal catheter drainage is indicated for a
large (> 30% chest volume) initial pneumothorax or progressive increase in size on serial
X-rays.
The common smaller size pneumothoraces are often difficult to identify on plain X-ray, even
witherectfilmsandmagnifiedviews.Largerpneumothoraces,asillustrated,areusuallyeasyto
identify.
CONDITION062.FIGURE2. CONDITION062.FIGURE3.CONDITION062.FIGURE4.
Examples of pneumothoraces and haemopneumothorax on plain X-ray (Figures 2,3) and
chestCT(Figure4)
336
337
063
Performance Guidelines



Condition 063
Atypical ureteric colic in a 25-year-old man

AIMS OF STATION
To assess the candidate's ability to interpret an X-ray of an intravenous pyelogram (IVP).
explain the findings to the patient, and give further advice about future management
to the patient.
EXAMINER INSTRUCTIONS
The examiner will have instructed the patient as follows:
You are a 25-year-old driver who has always kept in good health. A few days ago you
suddenly developed very severe lower abdominal pain for which you attended the
Emergency Department at this hospital. You were diagnosed as most probably having
a stone in one ureter (the tubes which connect the kidneys to the bladder). The pain
was relieved by an injection and has not returned. An X-ray of your kidneys was
arranged (IVP). You were told to strain your urine but so far nothing has been found.
Today you are attending the follow up clinic for the result of the X-ray.
Be yourself. Be more concerned about the kidney abnormality (which the candidate
should explain to you) than about the possibility of a stone in the ureter.
Questions to ask if not already covered:
'What does this mean for the future?'
'Why is my kidney in the wrong place?' ask this if the candidate advises you that
your right kidney is not in its normal position.
'Is the stone still in there somewhere?'
'Will I get another stone?'
'Is this kidney likely to develop a cancer?'
EXPECTATIONS OF CANDIDATE PERFORMANCE
The candidate should in commentary to the examiner interpret the IVP appropriately
and indicate that the abnormality shown is a left-sided crossed fused ectopic kidney
with separate calyceal systems and ureters. This anomaly is obvious on the film. The
abnormal position of the right kidney must be described. There is no evidence of a
calculus in either ureter on the single film available
The candidate, in discussion with the patient, should:
check and confirm that no stone has been found on straining the urine. Reassure
patient that on the X-ray there is no sign of any urinary calculus. Avoid alarming the
patient about the congenital renal abnormality.
explain that the stone is likely to have been passed spontaneously. No need to strain
urine anymore. Ensure adequate fluid intake in future especially in hot weather.
Report further symptoms. Inform future medical attendants of the renal abnormality,
especially if suffering from an abdominal complaint. Estimation of serum calcium to
exclude hyperparathyroidism would be appropriate.
338
063
Performance Guidelines



Patient Counselling/Education the abnormal position of the right kidney would explain the atypical nature of
the pain. Otherwise the renal abnormality is of no significance and needs no treatment except awareness of its
presence in the case of trauma to the left side of the abdomen or left sided abdominal pain.
The condition is congenital (present from birth) and variations in kidney position are quite common. The patient should
be reassured concerning the future, although warned that recurrence of renal/ureteric colic may occur if further stone
formation occurs.
KEY ISSUES
Interpretation of investigations must identify abnormal position of right kidney on the X-ray.
Initial management plan no further action required as stone has most likely passed,
Patient counselling/education reassurance, explanation of renal abnormalities. Ensure patient awareness if
any future abdominal pain occurs.
CRITICAL ERROR
Failure to describe abnormal position of right kidney.
COMMENTARY
Although intravenous urography/pyelography (IVP) is now largely replaced by computed tomography (CT) when
scanning for suspected urinary calculi, in this case a urogram is used to assess the candidate's ability to interpret an
X-ray finding which is quite obvious if anatomical knowledge is sound.
Congenital anomalies of the kidney and its vascular and urinary drainage systems are relatively common: up to 10%
of infants may be born with some anomaly of the genitourinary system.
Unilateral renal agenesis (congenital absence of one kidney) occurs in about 1 in every 1000 births, and may be
accompanied in a male by the absence of the vas deferens on the affected side. In the female, uterine and vaginal
abnormalities commonly co-exist.
The kidney begins intrauterine development in the pelvis, ascending to its adult position on the posterior abdominal
wall by birth, and acquiring fresh blood supply from progressively higher blood vessels with exclusion of others as
differential growth of body segmental somites occurs.
Ectopic kidneys
One or both kidneys may be in an abnormal position. Most ectopic kidneys are pelvic in position, and may present as
a pelvic mass, or be felt on rectal or vaginal examination. An ectopic kidney may be on its own side, or on the side of
the normal kidney (crossed ectopia), and may be fused with the normal kidney or pelvic in site as is this instance of
crossed fused renal ectopia. In 'pancake' kidney a single pelvic renal mass is served by two collecting systems and
ureters. Ectopic pelvic kidneys usually receive their blood supply from local vessels; the ureter of the displaced kidney
often crosses to its own side and opens into the bladder in the normal position.
339
063
Performance Guidelines



As kidneys ascend from the pelvis they normally remain separate. If they come into contact and adhere, a horseshoe
kidney may result, the kidneys being joined by an isthmus, which the ureters need to cross to descend.
Anomalies of the urinary collecting and drainage systems can predispose to urinary obstruction from hydronephrosis
or calculus.
Obstructive renal and ureteric pain (renal 'colic') is often an acute, constant and unremitting severe pain felt from the
site of the kidney towards bladder, penis and testis. An abnormal site of the kidney with anomalous referral of pain can
cause difficulties in diagnosis until functional imaging reveals the anomalous anatomy.
340
064
Performance Guidelines



Condition 064
Investigation for male factor infertility in a 25-year-old man

AIMS OF STATION
To assess the candidate's ability to advise a husband with an abnormal semen specimen of the
subsequent evaluation and management required for the couple to best achieve a pregnancy.

EXAMINER INSTRUCTIONS
The examiner will have instructed the patient as follows:
The doctor will generally be expected to advise you (the husband) of the significance of the
semen findings, what further evaluation is required, and what treatment is likely to be given in an
attempt to achieve a pregnancy. If there is no reversible factor present, and the semen analysis
does not improve with time it will be necessary for you and your wife to consider the place of
IVF, or the use of donor sperm not involving the use of IVF.
List of appropriate answers:
You are happy to have other tests done, or undertake treatment if this will lead to an
improvement in your semen specimen or achieve a pregnancy.
You work in an office writing computer programs for the banking industry.
You have had no contact with any chemicals.
You have had no surgery to your testes or inguinal region.
You have never had any testicular trauma.
You do not smoke and have 3-4 glasses of alcohol, usually wine, per week.
You are not on any drugs and have never taken any tablets except Panadolwhen you have
a headache.
You have never used any drugs of addiction or hashish.
You have never used anabolic steroids.
You had mumps when aged 10 years There was no testicular involvement (give this latter
information only when specifically asked).
You have not had any viral illness, or high fever, nor were you given antibiotics over the last
three months (these could have resulted in the current semen specimen being abnormal).
You do not use saunas.
If asked whether you would accept the use of donor sperm to achieve a pregnancy in your
wife, indicate 'no'.
If asked whether you and your wife would accept the use of IVF to achieve a pregnancy,
indicate 'yes'.
341
064
Performance Guidelines



Questions to ask unless already covered:
'Why is my test so bad?'
'Can't you do something to improve it?'
'If there are two million sperm present, why doesn t a pregnancy occur?
'Will a change in my diet help?'
'Will IVF be required for all pregnancies my wife and I want?' Only ask this question if the candidate has already
discussed the use of IVF.
EXPECTATIONS OF CANDIDATE PERFORMANCE
The history should cover the likely causes of the abnormal semen specimen, as detailed in the patient answers.
The candidate should advise along the following lines.
One semen specimen is insufficient to make a meaningful prediction of fertility potential. Preferably three
specimens obtained about three months apart are required. If these show the same findings as the first
one, then clearly there is a problem which is almost certainly a major factor in the infertility.
It is unlikely a cause of the abnormal semen specimen will be found.
A number of blood tests should be performed to provide information as to the likely reversibility of the problem. This
would include at least the measurement of serum FSH and testosterone levels. If the FSH is high, spontaneous
improvement in the analysis is less likely.
If the semen analysis improves spontaneously with time, the possibility of achieving a pregnancy is increased.
There is no documented evidence for the use of hormone or other treatment, in improving the semen specimen,
There is a definite place for the use of IVF, with intracytoplasmic sperm injection in the oocyte (ICSI). This has a
pregnancy rate of about 20-40%/cycle. IVF without the use of ICSI has poor results (about 2-5%pregnancy rate
per cycle of transfer).
There is a place for the use of donor sperm and performing artificial insemination, if this had been acceptable.
Pregnancy rate is about 20%per cycle of insemination. Use of donor semen is cheaper and more straightforward
than other methods of treatment such as IVF, but the baby would not contain any of the husband's genetic material.
Intrauterine insemination using his poor semen sample has a very poor success rate (about 1-2%
pregnancies/cycle of insemination).

KEY ISSUES
Need for appropriate history from husband.
Knowledge of appropriate tests to assess him, and of the possibility of improvement with time.
Need for empathie counselling.
Ability to understand that a definitive cause is unlikely to be found.
342
064
Performance Guidelines



CRITICAL ERRORS
Failure to advise that at least a second semen specimen (3 months after the first) must be examined.
Failure to recognise that persisting severe abnormality of the semen specimen as currently obtained will result in
a very low pregnancy rate.
Failure to understand that ICSI (within IVF) is the best method of achieving pregnancy using his genetic material.

COMMENTARY
In the advice to this young man, it must be recognised that a single sperm test is unreliable as a basis on which to
make a meaningful fertility prediction. This test must be repeated 2-3 months later and preferably again after a further
3 months. The comprehensive aspect of the counselling is based upon the assumption that a repeat specimen would
show a similar abnormality.
Common problems likely with candidate performance are:
Failure to repeat the semen specimen analysis a few months later (i.e. lack of understanding that one semen
analysis's result is of little predictive value).
Failure to ask questions to define the possible causes of the abnormal semen specimen.
Failure to ask whether the use of donor semen would be acceptable, as this is very effective and cheap, although
the child produced would not obtain DNA from the
husband
2-D:TheGeneralConsultation

BarryPMcGrath
' Inthefi el dofobservati on,chancefavoursthepreparedmi nd.'
Loui sPasteur(1822-1895)

ObjectivesofthemedicalconsultationHowIDoIt'
The medical consultation is the cornerstone of medical practice. A properly conducted
consultationestablishesaneffectivedoctor-patientrelationship.Theconsultationisthebasisof
the diagnostic and treatment formulations, the vehicle for a patient's education about health
promotion,clinicalproblems,diseaseprocessesandtestinterpretations.Itcanalsomotivatea
patient to follow treatment recommendations. It is usually not a 'one-off encounter; the first
consultation is generally followed by further consultation visits at variable intervals which can
extendovermanyyears.Thefollowingarethebroadobjectivesofamedicalconsultation:

Establishorbuildonaneffective,professionaldoctor-patientrelationship.
Determineandevaluatethepatient'sphysicalandpsychologicalsymptoms.
Identifyabnormalphysicalandmentalstatesigns.
Definetheclinicalproblem(s)thepatient'sprincipalcondition(s).
Chooseandinterpretappropriateinvestigations.
Reachthecorrectdiagnosis(thedoctorisprogressivelydevelopingandtestinghypotheses).
Explaintothepatientthenatureofthecondition,itsphysical,psychologicalandsocial
consequences.
Reachagreementwiththepatientonaplanofmanagement.
Institutetreatment.
Arrangereferraltoothercliniciansor
healthworkersappropriately.
Devisemethodsofrelievingpainand
suffering.
Somecommonlyencounteredproblems
duringaclinicalconsultationare:
failuretoobservecommoncourtesies;
failuretoestablishlevelsofcomprehension
andcommunicationcapabilities;
ignoringemotionalrponsesandconcerns;
overuseofdirected,closedquestions;
excessiveuseofleadingorloaded
questions;
vagueorcomplexquestions;
usingjargon;
disjointedquestioning;
abrupttopicchanges;
lackofexpressivenessininterviewer's
bodylanguageorvoice;
notdiscerningpatient'sideasandbeliefs
abouttheproblem;
narrowingthefocusofenquirytoosoon.
Provideadviceonhealthpromotion.
Reassuretheworriedwell.
343
344
2-D
The General Consultation



Setting the scene for a medical consultation
This may take place at any number of settings: general practice, Emergency Department,
hospital ward, outpatient/ambulatory care, and specialist consulting rooms. The goals of the
consultation will vary with the setting and the urgency of the clinical problem(s). Whatever the
setting, the doctor must respect the patient's safety, privacy, dignity, modesty, physical and
psychological well-being. Attention should be paid to the interview setting (for example, the
seating arrangement). Medical interviewers must be appropriately dressed, with a professional,
friendly demeanour and introduce themselves in a way that identifies their roles. The interviewer,
if a medical student or if not the patient's usual medical practitioner (for example, a trainee), will
need to seek the patient's permission to conduct the consultation.
Communication skills
First impressions, as in most human encounters, are very important. An expert medical
interviewer may adopt a variety of techniques, with an underlying self-questioning approach:
'How can I connect with this patient?' Frequently a patient will attend with a partner, family
member or friend. Establishing whether or not the patient wishes, or indeed, if it is appropriate, to
proceed to interview in the presence of 'a significant other' needs to be established early.
Depending on the circumstances, the interviewer may seek to conduct the medical consultation
with the patient alone initially and then subsequently involve the 'significant other' or family
members, but again only with the patient's permission.
Patient-related factors and doctor-related factors can influence doctor-patient communication.
It is critically important to establish, as soon as possible, if there are any impediments to
communication such as dementia, physical disability (such as deafness, blindness, stroke),
language or cultural attitude.
A skilled medical interviewer will exhibit:
an encouraging, warm and empathie manner;
nonjudgmental attitude;
good eye contact;
respect for patient's dignity, awareness of any discomfort;
alertness and responsi veness to nonverbal as well as verbal cues;
good listening skills;
use of mainl y open questions; and
note-taking that does not interfere with patient rapport. Some
commonly encountered problems include:
failure to observe common courtesies:
failure to establish levels of comprehension and communication capabilities:
ignoring emotional responses and concerns;
overuse of directed, closed questions, excessive use of leading or loaded questions;
vague or complex questions;
using jargon;
disjointed questioning;
abrupt topic changes;
lack of expressiveness in interviewer's body language or voice;
not discerning patient's ideas and beliefs about the problem; and
narrowing the focus of enquiry too soon.
2-0
The General Consultation

Theseissuesrelatingtocommunicationskillsarevitallyimportantandarere-emphasisedhere.
They are also addressed in a number of excellent text books and in many of the case
scenarios.
1,2,3,4
Clinicalreasoninginmedicalhistory-taking
Clinical reasoning mostly involves an efficient, hypothetic-deductive process. The diagnosis is
often made early in the medical consultation. Usually a list of alternative hypotheses, or
differential diagnoses, are considered, ranked and further addressed Additional information is
obtained from the physical examination and specific investigations which serve to confirm the
diagnoses, determine their severity and effects and to exclude alternative (differential)
diagnoses.Thefollowingpointsaregermanetotheprocessofclinicalreasoning:
Iterativenatureofprocess:thediagnostichypothesisiscontinuallybeingstrengthened
refined,modifiedortotallyreformulatedonthebasisofresponsestoquestions.
80%ofclinicaldiagnosesarereachedonthebasisofhistoryalone.
Clinicalexaminationoftenprovidesconfirmatoryinformation;insomecasesanewdiagnosis
isdefined.
Investigationsprovidethediagnosisinonlyabout10%ofinstances.
Theprocessofproblemidentificationissummarisedintheaccompanyingfigure
SECTION2-D.FIGURE1.
Processofproblemidentification
1 MRSanders,CMitchell,GJAByrne(eds).Medi cal Consul t i ng Ski l l s- Behavi our al and I nt er per sonal Di mensi ons of Heal t h Car e
AddisonWesleyLongmanAustraliaPty.Ltd.MelbourneAustralia1997.
2 SAColeandJBird(eds).The Medi cal I nt er vi ew The Thr ee- Funct i on Appr oach. MosbyInc.StLouisMissouriUSA.2000.
3 MMloyd,RBoor(eds).Communi cat i on Ski l l s f or Medi ci ne. ChurchillLivingstone.NewYorkUSA.1996.
4 The Cl i ni cal Encount er : A Gui de t o t he Medi cal I nt er vi ew and Case Pr esent at i on. MosbyInc.StLouisMissouri1999.
345
346
2-D
TheGeneralConsultation



Thestructureofthemedicalhistory
Thisisusuallyarrangedalongthefollowinglines:
basicinformationaboutthepatient
thepresentingcomplaint:
~historyofthepresentingcomplaint
~descriptionofthepresentingcomplaint:
- site
- severity/intensity
- quality/character
- timecourse
- setting/context
- aggravatingandrelievingfactors
- associatedfeatures
- riskfactors
othermedicalproblems:
~relatedtopresentingcomplaint
~additionalproblems
medication,habitsandallergies
systemsreview
pastmedicalhistory
familyhistory
socialandpersonalhistory
psychiatrichistory
BarryPMcGrath
347
2-D
TheGeneralConsultation



2-DTheGeneralConsultation
CandidateInformationandTasks
MCAT065-073
65 Acutechestpainina60-year-oldman
66 Palpitationsanddizzinessina50-year-oldman
67 Muscleweaknessandurinarysymptomsina60-year-oldman
68 Achesandpainsina62-year-oldman
69 Lackofenergyina56-year-oldsuntannedman
70 Recenthaematemesisina50-year-oldman
71 Anaemiaina28-year-oldpregnantwoman
72 Acutevertigoina50-year-oldman
73 Urinaryfrequencyina60-year-oldman
065
CandidateInformationandTasks



Condi t i on 065
Acutechestpainina60-year-oldman

CANDIDATEINFORMATIONANDTASKS
YouareworkinginahospitalEmergencyDepartment.Youareaskedtoseea60-year-oldman
complainingofacutechestpain.

YOURTASKSARETO:
Takeaconcise,relevantandfocusedhistory.
Presentasummaryofthepatient'shistoryfortheexaminer,whowillthengiveyouthe
findingsonphysicalexaminationwhichyourequest.
Telltheexamineryourprovisionaldiagnosisandthereasonsforthis.
InterprettheECGtotheexaminer(theECGwillbegiventoyouatabout7minutesinto
thisconsultation).
Instituteemergencytreatment.
CONDITION065.FIGURE1.
ThePerformanceGuidelinesforCondition065canbefoundonpage356
348
066
CandidateInformationandTasks



Condi t i on 066
Palpitationsanddizzinessina50-year-oldman

CANDIDATEINFORMATIONANDTASKS
YouareworkinginahospitalEmergencyDepartment.Youareaskedtoseea50-year-oldman
complainingofpalpitationsanddizzinessoverthepastthreedays.
Hehasnotseenadoctorforthepast10yearsandatthatlastassessmenthewastoldhisblood
pressurewaselevated.Hiscurrentbloodpressureis150/96mmHg.
Thesymptomsarestillpresentwhenyouseehimtotakehishistory.Heislyingdownonatrolley.

YOURTASKSARETO:
Takeaconcise,relevantandfocusedhistory.
Presentasummaryofthepatient'shistorytotheexaminer,whowillthengiveyouthe
findingsonphysicalexamination.
Telltheexamineryourdifferentialdiagnosis.
InterprettheECGtotheexaminer(theECGwillbegiventoyoubytheexaminerabout7
minutesintotheconsultation).
CONDITION066.FIGURE1.
ThePerformanceGuidelinesforCondition066canbefoundonpage363
349
350
067
CandidateInformationandTasks



Condition 067
Muscleweaknessandurinarysymptomsina60-year-oldman

CANDIDATEINFORMATIONANDTASKS
Youareworkinginageneralpractice.Yournextpatientisa60-year-oldmanwhoiscomplaining
oftiredness,weaknessandurinarysymptoms.

YOURTASKSARETO:
Takeahistoryfromthepatient.
Ask the examiner for the findings of a selective and focused physical examination you
wouldperform.
Statetotheexamineranyrelevantinvestigationsyouwouldorder.
Brieflyexplaintothepatientwhatyoubelievetobethecauseofhissymptomsandthe
firststepinmanagement(youarenotexpectedtodiscusstreatmentindetail).

ThePerformanceGuidelinesforCondition067canbefoundonpage368
351
068
Candidate Information and Tasks



Condition 068
Aches and pains in a 62-year-old man
CANDIDATE INFORMATION AND TASKS
You are working in a general practice You
r
next patient is retiree and aged 62 years. He
;
s
consulting you about aches and pains, and you have just finished taking a history, which was as
follows:
Over the last six weeks, he has had a gradual onset of pain across the upper part of the back,
neck and shoulders which feel stiff. In the past week or so his hips are also feeling stiff and sore.
Since retirement two years ago he has been playing golf at least three times per week and
thought that he may have been overdoing it. At first the pain was just an aching feeling, but is
now more definite pain but hard to describe. It is continuous, worsened by movement and is
keeping him awake at night. The aching and stiffness is worse early in the morning and he finds
it difficult to get out of bed because of muscle weakness and pain which improves during the day.
Pain is not relieved by aspirin or Brufen, nor worsened by coughing. There is no radiation to the
arms. Pain is felt in the muscles but not in the joints, although these feel stiff especially after
inactivity. He has not played golf for a week. He has noted a little difficulty in lifting himself up
from a chair. Muscles feel 'as if they are losing their strength'.
He has felt much more tired than usual over the last few weeks, especially after golf. His appetite
is not as good as usual. He thinks he may have lost a little weight and sometimes feels hot and
slightly sweaty at night in bed.

YOUR TASKS ARE TO:
Specify to the examiner the essential features you would like to know from a focused physical
examination of this patient. The examiner will give you the results and ask you questions
about your provisional diagnosis and further investigations.
Answer the questions put to you by the examiner.
Advise the patient of your diagnostic and management plans.

You do not need to take any further history.
The Performance Guidelines for Condition 068 can be found on page 371
352
069-070
Candidate Information and Tasks



Condi t i on 069
Lack of energy in a 56-year-old suntanned man
CANDIDATE INFORMATION AND TASKS
You are consulting in a general practice setting in Melbourne, Victoria. Your next patient is a 56-year-old industrial
chemist. He is complaining of tiredness, although he has just returned from holidays in Queensland and appears quite
suntanned.


YOUR TASKS ARE TO:
Take a concise, relevant and focused history.
After four minutes, tell the examiner what would be the most significant clinical signs you would search for on
physical examination, including office laboratory tests. The examiner will respond with these findings for this
patient.
Advise the patient of your opinion about possible causes for his tiredness, and how you intend to proceed to
make a firm diagnosis.

The Performance Guidelines for Condition 069 can be found on page 374

Condi t i on 070
Recent haematemesis in a 50-year-old man
CANDIDATE INFORMATION AND TASKS
You are an intern at the hospital Emergency Department. This 50-year-old patient has presented having had a
haematemesis of about 500 ml_ of fresh blood two hours ago, accompanied by a transient feeling of light headedness
and sweating.
The patient has given you a past history of a previous admission six months ago with a similar episode of
haematemesis which settled spontaneously. An endoscopy was done and the patient was told there were dilated
veins at the lower end of the gullet and was advised not to drink alcohol. The patient tells you that he has been trying
to give up alcohol with limited success.
On the basis of the history you have just finished taking, and his prior episode, you believe that the patient may have
had a haematemesis from oesophageal varices with portal hypertension and chronic liver disease as the explanation
for the current problem

YOUR TASKS ARE TO:
Perform a relevant and focused physical examination of the patient.
Explain your actions and what you are looking for to the examiner.
Describe your findings as you proceed.

You are not required to take any further history.
The Performance Guidelines for Condition 070 can be found on page 377
353
071-072
Candidate Information and Tasks



Condition 071
Anaemia in a 28-year-old pregnant woman CANDIDATE
INFORMATION AND TASKS
This 28-year-old pregnant woman, who is attending a general practice in which you work, has
just been found to have a haemoglobin level of 80 g/L when tested at 26 weeks of gestation.

YOUR TASKS ARE TO:
Take any further relevant history you require.
Ask the examiner about relevant findings likely to be evident on general and obstetric
examination.
Advise the patient of the tests required to define the most likely diagnosis and the
subsequent management you would advise.

The Performance Guidelines for Condition 071 can be found on page 380

Condition 072
Acute vertigo in a 50-year-old man

CANDIDATE INFORMATION AND TASKS
You are working in a primary care facility attached to a teaching hospital. This 50-year-old man is
consulting you about intense dizziness. He is a previous patient who is overweight, and is on
medications for control of hypertension and hyperlipidaemia. He appears unwell and distressed,
with a slight drooping of the left eyelid. His wife drove him to the hospital.

YOUR TASKS ARE TO:
Take a focused and relevant history.
The observing examiner will then give you the significant findings on physical examination.
Discuss your diagnosis and management plan with the examiner.
The Performance Guidelines for Condition 072 can be found on page 383
354
073
CandidateInformationandTasks



Condition 073
Urinaryfrequencyina60-year-oldman

CANDIDATEINFORMATIONANDTASKS
Youareworkinginageneralpractice.Yournextpatientisa60-year-oldman.Hehasattended
thepracticeinfrequentlyinthepast.Todayheisconsultingyouabouturinarysymptoms.

YOURTASKSARETO:
Takeafocusedhistorywithregardtothepresentingsymptoms.
Give the examiner a summary of the patient's presenting history with the most likely
diagnosis.
Ask the examiner what aspects of physical examination are most likely to confirm this
diagnosis and any initial office tests you would perform. The examiner will respond
accordingly.
Tell the patient your diagnostic conclusions, what investigations are indicated and the
reasonsforthese.

ThePerformanceGuidelinesforCondition073canbefoundonpage394
355
2-D
TheGeneralConsultation



2-DTheGeneralConsultation
PerformanceGuidelines
MCAT065-073
65 Acutechestpainina60-year-oldman
66 Palpitationsanddizzinessina50-year-oldman
67 Muscleweaknessandurinarysymptomsina60-year-oldman
68 Achesandpainsina62-year-oldman
69 Lackofenergyina56-year-oldsuntannedman
70 Recenthaematemesisina50-year-oldman
71 Anaemiaina28-year-oldpregnantwoman
72 Acutevertigoina50-year-oldman
73 Urinaryfrequencyina60-year-oldman
356
065
Performance Guidelines



Condition 065
Acutechestpainina60-year-oldman

AIMSOFSTATION
Toassessthecandidate'sabilitytotakeamedicalhistoryinanoldermalepatientpresentingto
theEmergencyDepartmentwithchestpainoftwohoursduration.Thecandidateneedstobe
awareofthepotentialseriousnessofthesituation,theimportanceoftakingafocusedhistoryto
distinguishbetweencardiacandnon-cardiacsourcesofchestpain,whilstbeingawareofthe
patient'sdiscomfortandtheneedtotakestepstorelievethis.Asinclinicalpractice,theearly
performance of an ECG and its correct interpretation is a key step in the assessment of this
patient.

EXAMINERINSTRUCTIONS
Theexaminerwillhaveinstructedthepatientasfollows:
You have acute and worsening chest pain. You are 60 years of age.
Openingstatement
' Ihaveaverybadtightnessinmychest.'
Characterisationofsymptom:
Site:
Severity:Timecourse:
Context:

Aggravatingfactors:
Relievingfactors:
Associatedsymptoms:
Otherhealthproblems:


Systemsreview:



Pasthistory:Drugs:

Habits:
FamilyHistory:
central,retrosternal,radiatingtolowerjaw.
severe8/10
cameontwohoursago,steadilygettingworse
recentanginafortwomonths;thispainstartedwhenplaying
thirdsetoftennis

none,noassociationwithrespiration
anginine,oxygenwhengiveninEmergencyDepartment
sweating,nausea,breathlessness
overweight, diabetes (5 years); hypertension (3 years): high
cholesterol(3years);paininleftlegonwalking500metres(1year).
centralchestpainwhenwalkingoncoldmorningsforthepasttwo
months:shortofbreathonexertionandbreathlessatnight(three
days);tiredness(twodays).Noepigastricpain,oesophagealreflux
ordysphagia.Recentblackbowelmotions(fivedays).Ifno
questionsaboutyourbowels,volunteerthisinformation.
Myocardialinfarctthreeyearsago
Forbloodpressure(enalapril/hydrochlorothiazide);diabetes
(metformin);aspirin;lipid-loweringagents
Smokeruntilthreeyearsago;alcoholintake1glasswineperday.
Nilrelevant.

357
065
Performance Guidelines



Theexaminerwillprovidephysicalexaminationfindingstothecandidateasfollows:
Heisanoverweightmanofstatedagewhoisinacutedistresswithpainandwhoisanxiousandsweating.
Blood pressure is150/96 mmHg, pulse rate 96/min and regular. Heartsounds dual rhythm,no murmur.
Therearenosignsofheartfailure.Examinationotherwisenoncontributory

EXPECTATIONSOFCANDIDATEPERFORMANCE
Approachtopatient
The candidate is expected to demonstrate professionalism, empathy and to seek relief of the patient's
discomfortwithuseofoxygenwhilsttakingthehistory.Awarenessofthepotentialseriousnessofthesituation
asthehistoryevolvesstillrequiresthecandidatetobecalm,confidentandreassuring.
History-takingskills
The candidate is expected to fully characterise the chest discomfort, its time course, the context and
associatedsymptoms.Thisneedstobedoneinasensitiveandfocusedway.usingamixtureofopen-ended
and direct questioning. The cardiovascular risk factors must be determined, including the past history of
myocardial infarction. The occurrence of this pain in the context of recent chest pain on exertion and
breathlessnessneedstobedefined.
Ifthehistoryofrecent'blackbowelmotions'isnotobtainedbythecandidate,thepatienthasbeenaskedto
bringthistothecandidate'sattention.
Whatshouldthedoctorbethinking?
Meetingthepatient: anoverweight,anxious-looking,sweatyolderman
withchestpain:urgentassessmentneeded:focusonkeyquestions
relatingtopossibilityofischaemicheartdisease.
Thepresentingproblem: fitspatternofacutemyocardialinfarction.
Checkouthismedicationlist:indicatesheisdiabetic,hypertensive,high
cholesterol.
Othercardiovascularriskfactors:previousacutemyocardialinfarction;claudication:
smoker.
Othermedicalproblems: Type2diabetes;hypertension;centralobesity;
hypercholesterolaemia;probablemelaena.
Physicalexamination: fitspatternofacutemyocardialinfarction
Treatmentstartsimmediately:Thisisamedicalemergencyrequiringmanagement
byanexpertteam(whatisthecandidate'sroleintheteam?);
commenceoxygentherapy,aspirin,glyceryltrinitrateandmorphine;
monitorpulse,bloodpressure.ECG;assessforthrombolytic
therapy.
065
Performance Guidelines



Ability to provide a concise clinical summary
This should be along the following lines and reflect the manner in which a junior doctor would
describe the key features of the history to a registrar or consultant.
The patient is an overweight, anxious looking, sweaty 60-year-old man with chest pain
described as 'a very bad tightness', 8/10 severity, in the central, retrosternal region, radiating to
the lower jaw but not to the arms. The pain came on when playing tennis and has been
increasing steadily over the past 2 hours, associated with shortness of breath, sweating and
partly relieved by anginine and oxygen. In addition he has had exertional chest pain over the
past 2 months and shortness of breath, orthopnoea and tiredness over the past few days. A
concerning symptom is his 5 day history of passing black bowel motions, which is suggestive of
gastrointestinal blood loss.
He is at very high risk of acute coronary ischaemia, having had a prior myocardial infarct, and
with risk factors of diabetes, hypertension, high cholesterol. EC G findings need to be checked
and anaemia considered as a precipitating factor.
Diagnosis
The most likely diagnosis is acute myocardial infarction. The key features that suggest this
diagnosis are the characteristics of the chest discomfort in a patient with significant risk factors
and prior myocardial infarction.
Interpretation of ECG
The ECG shows the following features:
Sinus rhythm, rate 96/min.
There are features of acute inferior myocardial infarction shown by the Q waves in leads II,
III, AVF and ST segment elevation in these leads as well.
CONDITION 065. FIGURE 2.
ECG of patient
Tests: confirm acute myocardial infarction; assess anaemia
358
359

065
PerformanceGuidelines



KEYISSUES
Approach to patient sensitivity to the patient's discomfort and a calm and professional
manner.
Theabilitytotakeanappropriateandfocusedmedicalhistoryshowinganawarenessofthe
likely causes of chest pain and the main characteristics that distinguish cardiac and
noncardiac sources of chest pain. The candidate needs to show an appreciation of
cardiovascularriskfactors,andanefficientabilitytocharacteriseassociatedsymptomsandto
definethecontextinwhichthesymptomofchestpainhasarisen.
Commentarytoexaminerasuccinctsummarywhichbringstogetherthekeyfeaturesofthe
presenting complaint, the context in which it has arisen, the associated symptoms and the
cardiovascular risk factors. The candidate should identify the potential significance of the
historyofmelaena.
Diagnosis/Differential diagnosis the candidate must consider the diagnosis of acute
myocardialinfarctionandwhynoncardiaccausesofthechestpainarelesslikely.
Interpretation of investigation the most important findings on the 12-lead ECG must be
defined:sinusrhythm,acuteinferiormyocardialinfarction.

CRITICALERRORS
Failuretoconsiderthediagnosisofacutemyocardialinfarctiononthehistory.
Failuretodefinethemaincardiovascularriskfactorspriormyocardialinfarction.Type2
diabetesmellitus,hypertensionandhypercholesterolaemia,smoking.
FailuretocorrectlyinterprettheECGfeaturesofmyocardialinfarction.

COMMENTARY
Thepatient'spresentationishighlysuggestiveofanacutemyocardialinfarction.
1
-
2
Cardiovascularriskassessment
Apriorhistoryofacardiovasculareventisthemostimportantpointertowardsarecurrentevent.
Type 2 diabetes mellitus is associated with a 10-fold increased risk of acute myocardial
ischaemia. For hypertension and hypercholesterolaemia and a history of smoking, the risk is
alsosignificantlyincreased.
3
.
4,5,6,7


1 Managementotunstableanginaguidelines,http://www.heartfoundation.com.au/
2 TherapeuticGuidelinesCardiovascularVersion42003
3 GrundySM,PasternakR,GreenlandP,SmithS,FusterV.Assessmentofcardiovascularriskbyuseofmultiple-risk-factorassessmentequations.
Ci r cul at i on 1999,100:1481-92.
4MRC/BHFHeartProtectionStudyofcholesterolloweringwithsimvastatinin20,536high-riskindividuals:arandomizedplacebo-controlledtrial.Lancet
2002,360:7-22
5 GenuthS,EastmanR.KahnR.KleinR,LachinJ,LebovitzH,etal.ImplicationsoftheUnitedKingdomprospectivediabetesstudy.Di abet es Car e
2003,26Suppl1:S28-32.
6 NealB,MacMahonS.ChapmanN.EffectsofACEinhibitors,calciumantagonists,andotherblood-pressure-loweringdrugs:resultsofprospectively
designedoverviewsofrandomisedtrials.BloodPressureLoweringTreatmentTrialists'Collaboration.Lancet 2000.356:1955-64.
7 National Health Committee revised guidelines for smoking cessation 2002. Wellington: National Health Committee; 2001.
http://www.heartfoundation.com.au


065
Performance Guidelines



Acute coronary ischaemia syndromes
All the acute coronary syndromes share the underlying pathology of an atherosclerotic plaque
which becomes active acutely with rupture of the plaque with resultant platelet adhesion,
thrombosis, vasoconstriction and inflammation. The exact syndrome depends on the extent of
thrombosis, the degree of distal embolisation of platelet thrombi and the resultant myocardial
necrosis. When the thrombus that occurs on a ruptured plaque completely occludes the
coronary artery, the result is severe transmural myocardial ischaemia with ST elevation on the
ECG. This may cause sudden death from ventricular fibrillation. If the coronary occlusion is not
relieved, myocardial infarction develops progressively over the next 6-12 hours. This is often
associated with evolution of evidence of transmural myocardial infarction on the ECG as shown
by the development of Q waves.
The acute coronary syndromes are differentiated on the basis of extent and duration of chest
pain, ECG changes and biochemical markers. They are divided into two syndromes: (1)
associated with ST elevation on the ECG (ST elevation myocardial infarction, STEMI) and (2)
those without ST elevation (non-ST elevation myocardial infarction, NSTEMI) associated with
either ST depression, T-wave inversion or no changes on the ECG. NSTEMI is differentiated
from unstable angina on the basis of biochemical evidence of myocardial necrosis (elevated
troponin level).
The following figures give examples of different patterns of myocardial infarction:

CONDITION 065. FIGURE 3.
Acute anterolateral myocardial infarction
Features indicating acute anterolateral infarction are:
ST elevation in leads I, aVL, V
2
-V
6
; and
Q waves in aVL, V
2
, V
3
and loss of R waves across chest leads.
360
065
PerformanceGuidelines

CONDITION065.FIGURE4.
Acuteinferiormyocardialinfarction
Featuresofacuteinferiormyocardialischaemia/infarctionare:
STsegmentelevationinII,III,andaVL;and
Theslowrateisalsocommoninthiscondition.

CONDITION066.FIGURE5.
Acuteposterior-inferiormyocardialinfarction

Featuresofposterior-inferiormyocardialinfarctionare:
QwaveandSTelevationininferiorleads(II,III,aVF);and
theprominentRwavesinV1(labelledC1)andQwaveswithSTelevationin
V5,V6indicatepostero-lateralinfarction.
361
Centralchestdiscomfortisacommonpresentationofcardiacdisease,butitmayalsobedueto
disease of the gastrointestinal tract, lungs or a musculoskeletal disorder. The features of the
chestdiscomfort/pain,thecontextinwhichthesymptomoccurs,theassociatedsymptomsand
thepatient'spredispositiontocardiacversusnoncardiacdiseasebasedonanassessmentof
cardiovascularriskfactors,mustallbeconsidered.
Intakingahistoryrelatingtochestdiscomfort,anumberofkeydescriptorsneedstobedefined
to determine if its origin is cardiac ischaemia. A common sequence of enquiry would be as
follows:
' Whati sthedi scomfortl i ke?Descri bei ti nyourownwords.'
' Howseverei si te.g.ascoreoutof10?'
' Showmewhereyoufeel i t?Doesi tgoanywhereel setheabdomen,theback,theneck,
thej aw,thearms?'
'Whendiditstart?Howhasit progressed?Howlonghasitbeenpresentorhowl ongdi di t
l ast?'
' Doesanythi ngmakethedi scomfortworse?Doesanythi ngmakei tbetter?'
' Doyouhaveanyothersymptoms?Shortnessofbreath?Di zziness?Pal pi tati ons?
Sweating?Nauseaorvomiting?'
Inadditionthereareanumberofquestionsthatwillbeusedintryingtodetermineifthereisa
non-cardiaccause:
'Doyougetacidindigestionorreflux?'
' Wastheonsetofthediscomfortrelatedtoameal?'
' Doesithurttotakeadeepbreath?'
'Isthechestsoretotouch?'

362
363
066
PerformanceGuidelines



Condition 066
Palpitationsanddizzinessina50-year-oldman

AIMSOFSTATION
To assess the candidate's ability to take a medical history in a patient presenting to the
Emergency Department with episodes of palpitations and dizziness. Careful history-taking is
essentialinassessingsuchpatients.Thecandidateneedstodefinetheattacks,thenatureofthe
arrhythmia (rate, rhythm, onset, offset, context) and the close association between the two
symptoms. Also critical to the assessment is an understanding of potential risk factors and
prcipitants of cardiac arrhythmias. Underpinning the history-taking, the candidate needs to
haveknowledgeofthecausesofcardiacarrhythmiasandthemanifestationsofdifferenttypesof
arrhythmias.
ObtaininganECGduringanattackanditscorrectinterpretationisakeystepintheassessment
ofthispatient.

EXAMINERINSTRUCTIONS
Theexaminerwillhaveinstructedthepatientasfollows:
You are lying on a trolley in the Emergency Department. You were brought to hospital by
ambulance.
Openingstatement
'I'vebeengettingattacksofpalpitationsanddizzinessoverthepastthreedays.'
Inresponsetospecificquestioning,providethefollowinginformation:
HistoryofpresentingcomplaintsThepalpitationsanddizzinessseemtocomeontogether.
Youhavethemnow.
Palpitations-Thesearedescribedasafastbeatingoftheheartgoingintotheneck.Ifaskedto
tapouttherhythmonthedesk,givearapidregularbeatofabout150/min.Therehavebeen
fourattacksoverthepastthreedays,eachlastingforabouttwohours.Theattackscomeon
suddenlyandstopsuddenly.Threeoftheepisodesoccurredaftertheeveningmealandthe
fourthwhilstdriving.Nothingyouhavetriedseemstostoptheattack.Thereisnoassociated
chestpainbutyouaremildlyshortofbreathandsweatduringandforashorttimeaftereach
attack.Thereisnoflushing,headacheornausea.
DizzinessThisisalight-headed,nearfaintingexperiencewhichcomesonwithinaminuteof
thepalpitationsandlastsforthedurationoftheattack.Youfeelyouhavetoliedown.
Systems review There is no history of heat intolerance, nervousness or tremor. You are
overweightwithnorecentchangeinweight.Bowelfunctionisnormal.
HabitsYousmoke20cigarettesperday;drink4or5glassesofwinewiththeeveningmeal
and5cupsofcoffeeperday.Youareonnomedications.
Social history You have a sedentary solitary lifestyle. Your job is stressful as a company
secretaryandthecompanyyouworkforisrestructuring.
FamilyhistoryNosignificantfamilyhistoryofheartdiseaseorcardiacarrhythmias.
364
066
Performance Guidelines



EXPECTATIONS OF CANDIDATE PERFORMANCE
Approach to patient
The candidate is expected to demonstrate professionalism, empathy and good communication skills.
History-taking skills
The candidate is required to carefully define the two symptoms and how they relate. The specific features of the
palpitations, the context in which they occur, the potential risk factors and prcipitants are all important elements of
this patient's history. In addition the candidate is expected to explore possible underlying cardiac diseases, in this
case particularly the possibilities of hypertensive heart disease or alcoholic cardiomyopathy.
Ability to provide a concise clinical summary
This should be along the following lines and reflect the manner in which a junior doctor would describe the key
features of the history to a registrar or consultant.
'The patient is a 50-year-old company secretary who presents with his fourth attack of palpitations and dizziness over
the past 3 days. Three of the attacks have occurred in the evenings after his meal and one whilst driving. Each attack
lasts approximately 2 hours, they come on suddenly and stop suddenly and the dizziness, which he describes as a
near-fainting experience, always accompanies the palpitations. The nature of the palpitations is that they appear to
be rapid, approximately 150/min and regular. He is currently experiencing an attack. The attacks are also associated
with shortness of breath and sweating, but no chest pain.
Considering possible underlying causes for his attacks, he has a history of high blood pressure but no known cardiac
disease. He has a high alcohol intake and has recently been under stress at work. He is also at risk of ischaemic heart
disease because of his smoking, obesity and sedentary lifestyle. There is no evidence on history to suggest
thyrotoxicosis. '

The examiner will provide physical examination findings to the candidate as follows:
Physical examination
He is an overweight, anxious man in some distress while sitting or lying on a couch. Pulse is 150/min and regular,
blood pressure is 150/96 mmHg. Heart sounds show dual rhythm with no bruits and are synchronous with the
pulse. There are no signs of cardiac failure. Examination otherwise is noncontnbutory

Diagnosis
The most likely diagnosis is paroxysmal atrial arrhythmia, probably atrial flutter. The key features that suggest this
diagnosis are the sudden onset and offset, the rapid, regular palpitations and the rate. The potential causes for this
arrhythmia are hypertensive heart disease, alcoholic cardiomyopathy, ischaemic heart disease or occult
thyrotoxicosis.
The differential diagnosis includes atrial fibrillation, supraventricular tachycardia, ventricular tachycardia.
066
PerformanceGuidelines
InterpretationofECG
TheECGshowsthefollowingfeatures:
CONDITION066.FIGURE2.Atrial
flutterwithvariableblock
KEYISSUES
Approach to patient Sensitivity to the patient's discomfort and a calm and professional
manner.
HistoryTheabilitytotakeanappropriateandfocusedmedicalhistorywithcarefuldefinition
of the symptom characteristics and showing an awareness of the likely causes and
prcipitantsofcardiacarrhythmias.
CommentarytoexaminerThisneedstobeasuccinctsummarywhichbringstogetherthe
key features of the presenting complaint, the context in which it has arisen, the associated
symptomsandthearrhythmiariskfactors.
Diagnosis/Differential diagnosis The candidate must consider the diagnosis of atrial
arrhythmiaandthepotentialcontributionsofhypertensionandalcohol.
Interpretation of investigation The most important findings on the 12-lead ECG must be
defined:atrialflutterwithvariableblock.
CRITICALERRORS
Failuretoconsiderthediagnosisofatrialtachyarrhythmiaonthehistory.
FailuretocorrectlyinterprettheECGfeaturesofatrialflutter.
365
066
PerformanceGuidelines



COMMENTARY
1

2 3

Palpitations are the symptom of an abnormal awareness of heart rate. They may be due to a
changeintherate,rhythmorforceoftheheartbeatsorsomecombinationofthese.Itisimportant
to ask the patient to tap out with a finger what is noticed when the palpitations arise. Anxious
patientsmaybeawareoftheirnormalheartbeat.Isolatedforcefulbeats('thumpinthechest')are
usuallycausedbyectopicbeats.Patientsmayalsoreporttheirheart'missesabeat',usuallydue
either to a compensatory pause after a ventricular ectopic beat or a nonforceful ectopic beat.
Awareness of a fast heart rate usually occurs when this is of recent origin. It is important to
determinewhetherthisisregularorirregularandwhetherthereareanyassociatedsymptoms.
Thedevelopmentofsymptomsinapatientwithanatrialarrhythmiawilldependontherate,the
rhythm,underlyingcardiacdiseaseandpatientcharacteristics.Acommonproblemencountered
in older patients with hypertensive heart disease who develop atrial fibrillation is that the
presenceofapoorlycompliant(stiff)leftventriclerendersthemquiteintoleranttothisarrhythmia,
wherethereislossofthecontributionofatrialcontractiontoventricularfilling,leadingtoleftheart
failure.
Atrial flutter usually presents with 2:1 atrioventricular block and a regular ventricular rate of
150/min.Itisoftenmisdiagnosedassupraventriculartachycardia.Rarelyconductionoccurs1:1,
givingaventricularrateof300/minandseveresymptoms.Muchmorefrequentlygreaterdegrees
ofAVblockarepresentgivingventricularratesof100(3:1block)or75(4:1block).Patientsmay
beasymptomaticexceptwhentheratechanges(e.g.from4:1to2:1block).Anexampleofatrial
flutter with 4:1 block is seen in the figure below (Figure 3). Note the characteristic saw-tooth
appearanceofthePwaves.

CONDITION 066. FIGURE
3.
Atrialflutterwith4:1block
In untreated patients with a normal AV node, atrial fibrillation (AF) usually presents with an
irregular ventricular rate of 160-180/min. Older patients with impaired AV conduction can often
presentwithlowerrates.Apartfromthefirstepisode,wherethenaturalhistoryisnotclear,atrial
fibrillationtendstofallintooneofthreeclinicalpatterns(theso-called'threePs').Patientsmay
progressfromonetoanother.Thesepatternsare:
ParoxysmalAF(episodeswhichcomeonsuddenlyandusuallyrevertspontaneouslywithin48
hours);
PersistentAF(episodespersistfordaysorweeksunlessactivemeasuresaretakentorevert
tosinusrhythm);and
1 TherapeuticGuidelinesCardiovascularVersion42003.
2 WyseDG.WaldoAL,DiMarcoJP,DomanskiMJ,RosenbergY.SchronEB,etal.Acomparisonofratecontrolandrhythmcontrolinpatientswith
atrialfibrillation.New Engl and Jour nal of Medi ci ne 2002,347:1825-33.
3 HankeyGJ.Non-valvulararialfibrillationandstrokeprevention.Medi cal Jour nal of Aust r al i a 2001.274:234-39.
366
066
Performance Guidelines



Permanent AF (inability to sustain sinus rhythm for any length of time or decision made not to try to revert the rhythm).
Patients with persistent and paroxysmal AF have at least the same risk of thromboembolism as patients with
permanent AF. An example of atrial fibrillation is illustrated (Figure 4). Note that the rhythm is irregularly irregular and
that no P waves can be seen.

CONDITION 066. FIGURE 4. Atrial
fibrillation
It is important to identify and manage underlying causes of atrial tachyarrhythmias (for example, hypertension,
thyrotoxicosis, heart failure, mitral valve disease, atrial septal defect).
Treatment of these two common arrhythmias needs to be considered under two headings: treatment of the arrhythmia
itself and prophylaxis against thromboembolic complications. The pharmacotherapeutic approaches to atrial fibrillation
and flutter are very similar, however atrial flutter commonly responds very easily to a low energy direct current shock or to
pace cardioversion and is often relatively insensitive to antiarrhythmic drugs.
367
067
368
PerformanceGuidelines



Condition 067
Muscleweaknessandurinarysymptomsina60-year-oldman

AIMSOFSTATION
To assess the candidate's ability to take a focused history regarding muscle weakness,
symptomsofprostatismandthepatient'sconcernsabouttheircause,whilealsobeingawareof
the possibility of an adverse drug reaction. The candidate should know the essential
components of a selective physical examination and essential investigations to confirm the
diagnosisandexcludeotherconditions.

EXAMINERINSTRUCTIONS
Theexaminerwillhaveinstructedthepatientasfollows
Youarea54-year-oldnewsagent.
Openingstatement
' I' m feeling weak and tired which is not like me. I' m also ha ving trouble with my
waterworks.'
Followthis,ifnotinterruptedbydirectquestioning,bytellingthedoctorthat:
Symptomshaveonlydevelopedoverthelastsixmonthsorso.
Youhavefelttiredandhavenotedafeelingofweaknessinyourmusclesoverthepastfew
weeks. This is not constant and not severe. All your muscles seem to be affected. It gets
worse towards the end of the day and you have attributed this to tiredness (long working
hours)andincreasingage.Youhavealsohadsomecrampsinyourcalfmuscles.
Overthelastsixmonthsyouhavealsofeltanincreasinglystrongurgetopassurinewhen
standingupaftersitting(e.g.ongettingoutofyourcarorafterwatchingTVforanhourorso).
Youalsohavehadtopassurinemorefrequentlyatnight.Somenightsyouhavetogetupat
threeorfourtimesandthenhavetroubleinstartingthepassageofurine.
Inresponsetoappropriatequestioning:
Thestreamofurineispoorandyoufindithardtofinish,withannoyingdribbling.Sofaryou
havenotlostcontrolorsoiledyourself.Passingurineisnotpainful.Theurinedoesnotsmell
abnormally. Sexual intercourse and ejaculation are not affected except for reduced
frequencyinrecentyears.Youhavelearnttoemptyyourbladderbeforegoingoutorsitting
forlongperiods.Youthinkthatreductionintheamountofbeeryoudrinkafterworkfrom4to
5glassesto1or2hashelped.
Youhavealwayskeptingoodhealth.
Other body systems are normal. In particular, no cardiovascular or other neurological
symptomsandnorelatedsymptomssuchastremororstiffness.Noweightloss.
Othersignificantinformation:
You commenced treatment for 'mild blood pressure' about eight months ago. but from a
differentdoctor.Yourmedicationishydrochlorothiazide25mg(Dithiazide)eachmorning.
Youareonyourfeetalldayinthenewsagency.YoustillplaytennisonSundaybutthepower
inyourgamehas'gone'.Nomarital,familyorfinancialproblems.
If asked about your past history, family history, habits or social history, respond as for
yourself.
369
067
Performance Guidelines



You are very puzzled by your muscle weakness. You are also concerned about the urinary symptoms and worried
that you could have prostate cancer. You have also thought of the possibility that your symptoms might be caused by
your medication.
The doctor may not seek all this information. If asked other questions, respond as for yourself.
After obtaining the results of the investigations from the examiner, the candidate will briefly explain the cause of your
symptoms to you. Do not question the doctor, simply accept what is said.

EXPECTATIONS OF CANDIDATE PERFORMANCE
Approach to patient
~ Use of appropriate communication skills to define the salient points of the history.
History
~ Identification of muscular weakness, cramps, urinary frequency, urgency and dribbling and medication (for
hypertension use of thiazide diuretic should be elicited). Patient concern about cancer should be recognised.
Physical examination
~ The candidate should ask the examiner for certain findings based on diagnostic possibilities suggested by the
history. Results will be provided for specific requests as follows:
CONDITION067. TABLE 1.
Examination findings

General appearance looks well
Pallor absent
Pulse 72/min regular
Blood pressure lying 154/92 mmHg and standing 148/90 mmHg
Heart normal
Abdomen normal
Neurological examination (limited)
power of limbs possibly slightly reduced
tone normal
reflexes normal
sensation normal
PR the prostate is enlarged. Features which should be sought:
degree of enlargement moderate
both lobes yes
consistency firm but not hard
tenderness no
surface smooth
nodularity/induration no
Urine office testing normal on chemical testing
370
067
PerformanceGuidelines

Thefollowinginvestigationsshouldbesuggested:
~Serumpotassium
~Haemoglobinandfullbloodexamination
~Prostatespecificantigen(PSA)
~Microscopyandcultureofmidstreamurine
Explanationtopatient
~Howlowpotassiumduetouseofadiuretictabletfortreatmentofelevatedbloodpressure
couldbethecauseoftheweakness.Effectsarefullyreversible.
~Enlargementofprostateisthecauseoftheurinarysymptoms.Reassurethatmalignancy
is very unlikely but that referral to a urologist is advisable for probable operative
treatment,whichwillincludeexaminationoftissueforcancercells.
~ Cease Dithiazide and perform followup checks of blood pressure for further
management.
KEYISSUES
History-takingtoidentifyweakness,prostatism,fearofcancer,currentmedication.
Examination for pallor, pulse and BP, heart, abdomen, neurological (limited), rectal
examination.
Investigations including serum electrolytes and creatinine, PSA, FBE, ECG. urine
microscopyandcultureandcytology.
Explanationtopatientthatthelikelydiagnosisishypokalaemia(reversible)asthecauseof
muscle weakness together with benign prostatomegaly. Reassure the patient regarding
cancer.
CRITICALERRORS
Failuretodorectalexamination.
Failuretosuggestappropriateinvestigations.
COMMENTARY

Muscleweaknessandfatiguearecommonsymptomswithmultipleaetiologies.
Inthisscenariothefirstdominantcueistheassociationoftirednessandweaknesswithurinary
symptoms.Theselattersymptomsandthesignsofbenignprostaticenlargementsuggest
bladderneckobstructionrequiringfurtherinvestigationandreferral.
Theotherdominantcueispickedupbysystemsreviewgivingtheinformationthatthepatient
hasbeenonathiazidediureticforeightmonths,andhissymptomsofmusdeweaknessbegan
afterstartingthismedication.
Thediureticpolyuriamayhavebroughttolightpreviouslynonsymptomaticprostaticpathology,
and caused potassium loss. Even so-called potassium-sparing thiazide diuretics can be
associatedwithpotassiumdepletion,whichcouldbecontributingtohismuscleweaknessand
musclecrampsthroughhypokalaemia.
Investigations of serum electrolytes (particularly potassium levels), renal function tests, urine
cytologyandculture,andfullbloodexaminationwouldbemandatoryinapatientofthisagewith
symptomsasdescribed.
371
068
PerformanceGuidelines



Condition 068
Aches and pains in a 62-year-old man

AIMS OF STATION
To assess the candidate's knowledge of the clinical presentation of polymyalgia rheumatica and
the way in which this diagnosis is confirmed or excluded.

EXAMINER INSTRUCTIONS
The examiner will have instructed the patient as follows:
You are a retired office worker and will be advised by the candidate of the diagnostic and
management plans.
EXPECTATIONS OF CANDIDATE PERFORMANCE
Physical examination findings to be sought:
Essential features of focused physical examination to be given to candidate on
request by the examiner.
~Temperature 37 C, normal
~Pulse 70/min regular
~Blood pressure 140/80 mmHg
~Muscle groups of neck, trunk, upper and lower limbs should be examined.
- Active movement of neck, shoulder and trunk muscles causes discomfort.
- Normal power and tone and coordination of movements.
- Examination of joints, particularly hands, shoulders, neck, sacroiliac joints and hips.
These show no abnormalities and a full range of movement.
~Examination of lymph nodes, abdomen, and respiratory systems is expected to exclude any
medical conditions that could possibly give rise to this constellation of symptoms (e.g.
lymphoma, carcinoma) normal findings.
~Rectal examination to check prostate normal.
After providing results of physicai examination, the examiner will ask the candidate
'What is your provisional diagnosis and differential diagnosis?'
'What further tests will you advise?'
'Please now give to the patient your diagnostic and management plans. '
Diagnosis/Differential diagnosis
Polymyalgia rheumatica should be suspected from the history. The examination does not reveal
any specific diagnostic features but erythrocyte sedimentation rate (ESR) or C-reactive protein
(CRP) would be expected to confirm the diagnosis. Underlying malignancy should be a
consideration.
372
Management
The candidate is expected to indicate to the patient that if the blood tests confirm the suspected diagnosis of
polymyalgia rheumatica, then the patient is likely to have a good response to a limited course of prednisolone which
may need to be given for up to two years, but that any such treatment, how it is given and monitored, must await the
results of the tests. The patient should be advised to report any severe headaches, visual disturbance or pain in the
jaw when eating, since giant cell arteritis can occur together with polymyalgia rheumatica.
KEY ISSUES
Focused physical examination which must include musculoskeletal system plus rectal examination.
Investigate with ESR and/or CRP.
Polymyalgia rheumatica as the most likely diagnosis.
CRITICAL ERROR
Failure to request ESR and/or C-reactive protein.
COMMENTARY
Polymyalgia rheumatica and giant cell arteritis are linked conditions of unknown aetiology. The incidence varies with
ethnicity and these conditions are more common in people of Northern European descent.
Polymyalgia rheumatica commonly presents in middle aged or elderly patients with diffuse symptoms of muscle pain
particularly in the neck, shoulders and hip girdles. The myalgia is symmetric and often begins in the shoulders.
Muscle strength is normal but can appear diminished because of pain. There is often a disparity between the severity
of myalgia reported and the physical findings. There are often constitutional symptoms including weight loss, malaise
and depression; spiking fevers are rare. The diagnosis in this instance would be confirmed by investigations,
specifically ESR and C-reactive protein, and full blood examination (FBE).
Treatment of polymyalgia rheumatica is with oral prednisolone, initially in high dosage.
Differential diagnosis to be considered would include:
~ Chronic fatigue syndrome: This condition is a 'medically unexplained condition'. It is usually seen in younger
patients, may follow a viral infection and the dominant feature is incapacitating fatigue with other medical
symptoms of subjective memory impairment, headaches, poor sleep, generalised muscle pains,
postexertional malaise lasting more than 24 hours, lymph node tenderness. It is best viewed as a symptom
complex resulting from interaction of physical and psychosocial factors. The ESR, CRP and FBE tests are
normal.
~ 'Fibromyalgia': Another of the 'medically unexplained' conditions, characterised by aching pains across the
shoulders and upper back, skin tenderness, poor sleep pattern and often additional constitutional symptoms.
ESR, CRP and FBE are normal.




373
068
Performance Guidelines



~ Polymyositis: This is an uncommon inflammatory muscle disorder that may be associated
with an underlying neoplasm in older patients. The most frequently encountered mode of
presentation is the onset of painful muscles and proximal muscle weakness, often
commencing in the neck, shoulder girdle and proximal limb muscles, associated with some
atrophy with disproportionate weakness. ESR. CRP and FBE abnormalities may be
indistinguishable from polymyalgia rheumatica but elevated creatine kinase and abnormal
autoantibodies are characteristic. A positive muscle biopsy is diagnostic.
~ Underl ying malignancy: prostate, breast in females, multiple myeloma, lung cancer.
374
069
Performance Guidelines



Condi t i on 069
Lack of energy in a 56-year-old suntanned man
AIMS OF STATION
To assess the candidate's ability to diagnose the cause of tiredness and lack of energy in a 56-year-old man.

EXAMINER INSTRUCTIONS
The examiner will have instructed the patient as follows:
You are a 56-year-old industrial chemist.
Opening statement
'I have felt very tired lately and for no apparent reason. '
Story in Detail
Without prompting you have felt tired and lethargic since you retired about a year ago You attributed this to a lack
of mental stimulation from what was a demanding job. About three or four months ago you also noticed aches and
pains in your joints which have persisted. You also realised that you had lost some weight which is why you decided
to see the doctor. You have just returned from holidays in Queensland. You have noticed that you have developed a
suntan even though you don't spend much time outdoors, and did not do much swimming on holiday.
In response to specific questions respond as indicated:
The tiredness is constant and not improved by resting or sleeping (you sleep well).
The aches and pains are mainly in your shoulders, hips and knees. There is some tenderness and swelling in
wrists and elbows and pains in the shoulders. The muscles are not sore.
The weight loss is 3-4 kg.
You have also noticed palpitations at times mainly when going off to sleep, when your heart seems to speed
up and miss beats for a few minutes at a time.
Your sexual activity has been less than before; you thought due to your age.
You don't feel depressed
In response to all other questions deny any other symptoms.
You do not smoke or drink any alcohol.
There have been no significant past illnesses.
There is no significant family history but you were adopted and know little about your
parents.
375
069
Performance Guidelines



EXPECTATIONS OF CANDIDATE PERFORMANCE
Approach to patient The candidate should show skill in:
~ listening and facilitation of presenting symptoms; and
~ using direct and indirect questioning in a logical, relevant and non-threatening
manner.
History-taking The candidate is expected to take a comprehensive history in a patient presenting with the
symptoms of tiredness, arthralgia and weight loss. The additional symptoms of palpitations, change in skin colour
and loss of libido, in the absence of other symptoms or significant past history, indicate there is a multisystem
disorder. However most candidates will require the examination findings to assist in the diagnostic formulation.

Examination requests should be made for specific diagnostic features. The examiner will respond to a request with
the following specifics:
~ Distribution of hyperpigmentation generalised over body but not mucos
~ Joints swelling and some tenderness of wrists, elbows and knees.
Limitation of range of movements and tenderness of the
shoulder joints
~ Pulse irregular, atrial fibrillation, confirmed by office ECG
~ Blood pressure 140/90 mmHg
~ Heart no additional findings, no signs of cardiac failure.
~ Abdomen 5 cm enlargement of liver firm nontender liver edge
~ Genitalia testes softer and smaller than usual for age
~ Urine positive for glucose. Diabetes confirmed by random blood
sugar of 12 mmol/L
Diagnosis/Differential diagnosis The patient presents the classical clinical picture of haemochromatosis
~ Other causes of increased pigmentation Addison disease, Cushing disease, hyperthyroidism, cirrhosis,
porphyria, chronic renal failure, malnutrition/malabsorption, drugs causing photosensitivity (for example,
psoralens, phenothiazines, certain antibiotics, amiodarone).
Choice of investigations The candidate should indicate the need for:
~ full blood examination and erythrocyte sedimentation rate.
~ creatinine and electrolytes
~ serum iron studies (especially transferrin saturation)
~ liver function tests
~ test for gene for haemochromatosis (HFE) gene
If the candidate suspects haemochromatosis the next steps in the confirmation of the diagnosis should be explained
to the patient. Details of treatment are not required in this case. Specialist referral would be expected for further
assessment and management.
If the candidate does not recognise the significance of the constellation of symptoms and signs, their choice of
investigations and/or referral will indicate the level of performance.
376
069
Performance Guidelines



KEY ISSUES
History-taking the candidate is expected to exhibit appropriate history-taking skills and obtain the key
features of the illness given the patients initial presenting complaints of tiredness, joint pains and weight loss.
This will require a systematic approach to history-taking and the multisystem nature of the complaints requires a
comprehensive but concise history.
Choice and sequence of examination examination of the joints and of the features of skin pigmentation is
expected. The candidate should indicate the need to examine the pulse, cardiovascular system, abdomen and to
look for evidence of endocrine dysfunction.
Diagnosis/Differential diagnosis the condition of haemochromatosis may not be apparent to the candidate
on the history and examination findings. However a consideration of the causes of the multisystem disease with
skin pigmentation, arthritis, cardiac, liver, and endocrine systems disorder should be sensibly discussed.
Choice of investigations these should be appropriate to the investigation of the multisystem disease.
CRITICAL ERROR - none defined
COMMENTARY
In primary haemochromatosis there is increased absorption of iron from a normal diet. Repeated blood transfusions
can cause secondary haemochromatosis. The primary form is an autosomal recessive condition, known also as
'Bronze Diabetes'.
The homozygous state is present in 1:150 in Australia with 1 in 10 carriers (1 in 300 blood donors have iron overload)
and is more common in people of Celtic or Northern European background. Typical manifestations are bronze skin
pigmentation, diabetes (60%), cardiomyopathy, liver damage and pituitary failure. Fatigue, arthralgia and abdominal
pain are leading symptoms, while detection of atrial fibrillation, hepatomegaly, testicular atrophy and hyperglycaemia
make a clinical diagnosis possible.
The critical confirmatory investigations are iron studies serum iron, total iron binding capacity, ferritin, transferrin
and transferrin saturation, plus testing for the gene for haemochromatosis (HFE gene).
377
070
PerformanceGuidelines



Condition 070
Recenthaematemesisina50-year-oldman
AlMSOFSTATION
To assess the candidate's clinical perspective in examining a patient presenting to the
EmergencyDepartmentwithanacutehaematemesis.
Tocheckabilitiestoexamineforevidenceofchronicliverdisease.
EXAMINERINSTRUCTIONS
Theexaminerwillhaveinstructedthepatientasfollows:
Thecandidateshavebeentoldthatyouhavevomitedupalargequantityofbloodtwohoursago
and they have been instructed to undertake a relevant physical examination. They are not
expectedtotakeafurtherhistoryfromyou.Theyhavejustfinishedtakingahistory.
Onthisoccasionyouvomitedalargeamountoffreshbloodtwohoursagoyouthinkit might
havebeenapintorso(500ml_).Youfelttemporarilyfaintandbrokeintoasweat.Youhavenot
vomitedsince.Yourwife hasdrivenyoutohospitalwhere theHospitalMedicalOfficer(HMO)
hastakenyourhistoryYouarelyingonthecouchundressedtoyourunderclothesandwearinga
hospitalgown.TheHMOwhohastakenyourhistoryisabouttoexamineyou.
Youhavegivenapasthistoryofapreviousadmissionsixmonthsagowithasimilarepisodeof
vomitingbloodwhichsettledspontaneously,andyouweredischargedafterafewdays.Youhad
anendoscopythroughthemouthandyouweretoldyouhaddilatedveinsatthelowerendofthe
oesophagusleadingintothestomach.Youwerewarnedabouttheeffectsofcontinueddrinking.
You'vebeentryingtogivethisupbutyouhavehadlimitedsuccess.Itislikelythatthecandidates
will want to measure your blood pressure and feel the pulses in your arms. They will also
examineyourhands,face,chestandabdomen.
Inapatientwithliverproblems,thefindingswillbeevidentonexamination,andwillhavebeen
previouslycheckedbytheexaminer.
EXPECTATIONSOFCANDIDATEPERFORMANCE
Candidatesshouldfirstlookforevidenceofhaemodynamiccompromise(lookingforevidenceof
hypotension and postural drop, pulse, peripheral perfusion). Once candidates indicate they
wouldtakethebloodpressure,theycanbetoldthattheBPis110/70mmHgandpulse90/min.
As the bleeding occurred only two hours ago, this assessment is particularly important.
Candidatesshouldindicatethattheywoulddoarectalexaminationlookingforamelaenastool,
andwillbeinformedthereisafreshmelaenastoolontheglove.
378
070
PerformanceGuidelines

Afterassessmentofstablehaemodynamicstatusasfirstpriority,thecandidate
should:
Putthepatientatease,correctlypositionhimsupinewithappropriateexposuretoexamine
thewholeabdomen,groin,headandneckandupperlimbs.
Makeanappropriateexaminationlookingforevidenceofchronicliverdisease(examination
of hands liver palms, leuconychia, spiders, clubbing), easy bruising, spider naevi
elsewhere, gynaecomastia, parotid enlargement, oral cavity and tongue, ascites, portal
hypertension(dilatedveinsandsplenomegaly),testicularatrophy.
Examineforliverflap.
Palpatetheabdomenadequatelyforhepaticandsplenicenlargement.
Checkforevidenceofascites,bypalpationforshiftingdullnessorfluidthrill.
Percussforevidenceofliverandsplenicenlargement.
Auscultateabdomenforvenoushum,bruit,bowelsounds.
Providealogicaldescriptionconcerningtheexamination.
Performtheexaminationinalogicalsequence.
KEYISSUES
Performing a satisfactory physical examination pertinent to an episode of acute
haematemesisinapatientinwhomevidenceofchronicliverdiseaseshouldbesought
Accuracy of examination will be a key issue for the mark sheet when a real patient is
involved.

Satisfactorycommentarytoexaminer.
CRITICALERRORS
Failuretoassessthehaemodynamicstateofthepatient.
Failuretolookforevidenceofliverfailureandportalhypertension.
COMMENTARY
Inthisimportantandcommonemergencyroomsettingtherearethreeissuesthedoctormust
focuson:
CheckingtheABC(Airway,Breathing,Circulation)ofimmediateresuscitation.
Assessmentofthecardiovascularstateofthepatientandprovisionofpromptresuscitation,
ifnecessary.
Identificationofthecauseofthehaemorrhage.
This patient has had a large haemorrhage and the airway couid be compromised. In this
scenario the patient appears fully conscious and is able to give a detailed history, so it is
unlikelythatthereisamajorproblemwiththeairwayorbreathing.
Thusthephysicalexaminationmuststartwithmeasurementofthebloodpressure(lyingand
sitting, if necessary) and pulse, and a clinical evaluation of how well the periphery and vital
tissues are perfused. Is the patient shocked, cold and clammy, with a shutdown peripheral
circulation?Ifthepatientisshocked,thephysicalexaminationmustceaseatthisstageandthe
patientmustberesuscitated.
070
Performance Guidelines



Provided the patient is stable, a methodical examination may be undertaken to look for the cause of the haemorrhage.
Although the patient's past history has suggested that the cause is alcoholic liver disease, portal hypertension and
bleeding oesophageal varices, this should not be assumed as many patients with known varices will be bleeding from
another cause. The examination should look for:
signs associated with chronic liver disease:
signs of possible liver failure;
signs of portal hypertension; and
any other clues suggesting a different aetiology for the haemorrhage.
Signs associated with chronic liver disease apart from hepatomegaly include nail changes (leukonychia), salivary
gland enlargement, testicular atrophy, gynaecomastia and spider naevi. If the liver is failing, the patient may have
ascites and encephalopathic changes. Encephalopathy may have a variety of presentations, ranging from minor
mental impairment and flap, through to coma.
Portal hypertension may be manifest by the signs of hypersplenism (purpuric haemorrhage), splenomegaly and
collateral venous channels. The latter may be visible in the anterior abdominal wall as communications between the
umbilical vein and the epigastric venous channels flowing back into the systemic circulation. Of more sinister import
are the oesophageal mucosal collaterals that form between the portal and azygos systems through decompression
along the left gastric (coronary) vein.
Occasionally, the physical examination will reveal other signs that might be associated with haemorrhage, for
example, the hereditary haemorrhagic telangiectasia associated with the Osler-Weber-Rendu syndrome.
Variations on this theme are also used, in which a real patient with liver disease is to be assessed after admission to
the ward and institution of an intravenous drip while blood is being typed and cross-matched. The instructions will in
that case state that the patient is now haemodynamically stable and the emphasis of the task is to assess the patient
for evidence of chronic liver disease, which is expected to be present. The assessment now concentrates on the
technique and accuracy of physical examination as key issues.
In this emergency department scenario the emphasis is FIRSTLY on assessment of stable or unstable haemodynamic
status in a patient with recent haematemesis.
CONDITION 070. FIGURES 1 AND 2.
Abdominal distension Ascites
379
380
071
PerformanceGuidelines



Condition 071
Anaemia in a 28-year-old pregnant woman
AIMS OF STATION
To assess the candidate's ability to define the possible causes of anaemia in pregnancy and to
arrange appropriate investigations and advise the patient concerning the diagnosis and
treatment. The most likely diagnosis is iron deficiency anaemia due to the demands of three
pregnancies in a short time interval, but other causes of anaemia including thalassaemia, and
folic acid deficiency associated with a multiple pregnancy need to be excluded. Having made
the appropriate diagnosis iron therapy should be prescribed.

EXAMINER INSTRUCTIONS
The examiner will instruct the patient as follows:
The list of responses below is likely to cover most of the questions you will be asked.
List of appropriate answers to questions
Previous obstetric history you have had three pregnancies during the last four years. No
postpartum haemorrhage.
You did not take iron tablets during these pregnancies: your haemoglobin was always
greater than 100 g/L when previously tested.
You noted no excessive blood loss before or between pregnancies periods have not been
heavy.
You have had no bleeding from the bowel, and there has been no suggestion of malaria or
hookworm infestation; you have always lived in Southern Australia.
You had an ultrasound examination at 18 weeks of gestation, this showed a singleton
pregnancy was present, and confirmed the period of gestation.
Diet you eat meat occasionally, you don't like green vegetables. No iron tablets have
been taken during the pregnancy.
No vaginal bleeding has occurred during the pregnancy.
There is no family history of /^thalassaemia or of anaemia generally. You are Australian
born as were your parents, and there is no Mediterranean heritage in the family
You have not had a full blood examination (FBE) done before in this pregnancy.
You have a supportive partner who assists at home.
Questions to ask if not already covered:
'Why have I become anaemic?'
'Will my anaemia harm my baby?'
'Do I need a blood transfusion?'
'How quickly will my haemoglobin come up?'
381
071
Performance Guidelines




Examination findings to be given to the candidate from the examiner on request
Apart from looking pale, general examination is normal. The uterus is enlarged to about 4 cm above the umbilicus
and measures 26 cmabove the pubic symphysis.
Investigation results
None has been recorded for this pregnancy other than the ultrasound.

EXPECTATIONS OF CANDIDATE PERFORMANCE
Advice to patient (the candidate should convey the substance of what follows to the patient):
She needs an FBE to check on the form of anaemia which is present.
She requires assessment of her iron status serum iron or ferritin levels should also be checked.
If the FBE suggests possible /Mhalassaemia minor, haemoglobin electropheresis will also be required.
Treatment with iron tablets should begin after taking blood for investigation (Ferro-Gradumet or Fefol). Two
tablets should be taken a day; she should be warned about the possible effects of these in causing constipation
and dark stools. There is no need for parenteral iron therapy at this time, or blood transfusion.
A satisfactory response to oral iron therapy should be able to be achieved well ahead of the time that delivery is
likely. The haemoglobin should be checked again in two weeks, along with a reticulocyte count. If the haemoglobin
does not increase satisfactorily, referral to a haematologist for advice concerning diagnosis and treatment would be
appropriate. Providing the anaemia can be treated satisfactorily, there should be little effect on the pregnancy. In
the absence of adequate treatment the placenta becomes larger, however the babies are usually smaller.

KEY ISSUES
Ability to evaluate appropriately a patient who has become anaemic during pregnancy.
Ability to commence treatment and arrange appropriate followup in such a patient.

CRITICAL ERRORS
Failure to make a provisional diagnosis of probable iron-deficiency anaemia due to the demands of successive
pregnancies.
Failure to administer oral iron therapy.
Recommending blood transfusion at this time.
382
071
Performance Guidelines



COMMENTARY
This case illustrates a common problem of iron deficiency anaemia in a young woman who has had a number of
pregnancies in a short period of time. This is the most common form of anaemia under these circumstances and
whilst other less common forms of anaemia should be considered, it is important to commence treatment for simple
iron deficiency anaemia whilst awaiting the results of investigations. It is also important to remember that blood
transfusion is not indicated under these circumstances in mid-pregnancy.
Common problems likely with candidate performance are:
Failing to focus on other causes of anaemia when taking the history failing to ask about menstrual loss, loss
from other sites, and failing to consider the possibility of thalassaemia minor.
Failing to arrange appropriate blood tests which would include haemoglobin electrophoresis if the anaemia is
hypochromic and microcytic without evidence of iron deficiency, and the assessment of serum iron or ferritin
levels.
383
072
Performance Guidelines



Condition 072
Acute vertigo in a 50-year-old man

AIMS OF STATION
To assess the candidate's ability to diagnose an acute vascular 'stroke' presenting with vertigo.

EXAMINER INSTRUCTIONS
The examiner will have instructed the patient as follows:
Opening statement
'I feel so dizzy that I can hardly stand up. It's like being on a merry-go-round! '
Story in detail without prompting
'This morning just over an hour ago, I was having breakfast when I felt a pain in the left side of my face (indicate the left
side). Then I started to feel numb up and down the other side of my body (indicate the right side) and I became so
dizzy that I couldn't even sit up. let alone stand up. '
'My head felt as if it was exploding, and my speech was funny and slurred. '
'My wife got me into bed and then I felt sick and vomited. I had to lie very still or I wanted to vomit. It was like being very
seasick. So I rested while she rang you. If I turn my head to the side the dizziness gets worse. I found it hard to get in
and out of the car. I keep falling this way (indicate to the left). Am I having a stroke?'
In answer to the doctor's questions
You are feeling a little better now but would prefer to lie down.
Everything seems to be moving and spinning around you.
The pain has gone from your face but the cheek (left) now feels numb too.
You still have a feeling of numbness down your right side, involving the trunk and limbs.
No persisting headache or neck stiffness.
No hoarseness (if asked about swallowing, you did feel that it was difficult to swallow your saliva, but that feeling
has now gone. You suspect that your taste has been affected).
No previous episodes. No more vomiting. Your speech has now returned to normal after initial slurring
Review of general health
Apart from being overweight and having treatment for high blood pressure and high cholesterol, you feel you have
been in good health. Your blood pressure has been variable. Lipid levels stable.
Appear apprehensive and agitated. Hold firmly onto the desk or chair to keep yourself steady. Lean towards your left
side. Although you have just suffered a cerebral event your ability to give a satisfactory history is not impaired. Give a
full account of your symptoms unless interrupted by the doctor taking control of the interview at too early a stage. You
are very concerned that you are having a stroke.
384
072
PerformanceGuidelines



Socialhistory
Youaremarried.Youandyourwifeliveinyourownhome.Youdonotsmoke.Youworkasa
postman.Youdrinkalcoholonlyoccasionally.
Familyhistory
Motherdiedfromstrokeat65years.
Fatherwasdiabetic,diedfromheartattackaged59years.
Majorcontinuinghealthproblemsare:
~Hypertensionforabout10years.
~Hyperlipidaemiaidentifiedabout6yearsago.
~Bothconditionshavebeenwellcontrolled.
Currentmedication
Norvasc(amlodipine)10mgoncedaily(calciumchannelblocker).Avapro(irbesartan)150
mgoncedaily(angiotensinIIreceptorantagonist).Lipitor(atorvastatin)10mgoncedaily
(CoAreductaseinhibitor).
Afterthehistoryisfinished,theexaminerwillhandtothecandidateaseparatesheetgivingan
outlineofphysicalfindingsassetoutintheboxbelow.

PhysicalFindings
Cardiovascularexaminationisnormalbloodpressure145/85mmHg,pulse80/minand
regular.
The main findings on neurologic examination are that he has an ataxic gait and postural
unsteadiness without significant change on closing eyes. He has some incoordination of
movement of the left arm and hand, but no motor weakness or other motor signs are
present.
CranialNerves
Eyemovementsandpupilreactionsarenormalasisfundoscopy.
Nystagmustotheleftonlookingtotheleftispresent.
AleftHornersyndromeispresent(ptosis,miosisofpupil).
Painsensationtopinprickislostontheleftsideofthefaceandthedirectcornealreflex
isabsent.Powerofthemusclesofmasticationisnormal.
Hearingisnormalinbothears.
Appreciationofpainandtemperaturesensationisreduceddownthewholeoftheright
sideofthebodybelowtheface.
Vibrationandjointpositionsenseandlighttouchsensationarenormal.
EXPECTATIONSOFCANDIDATEPERFORMANCE
Abilitiesincommunicationskillsandindiagnosticproblemsolvingarerequired.
Thispatientisabletogiveaveryfullpictureoftheonsetofthecondition.Theskillrequiredisin
listeningcarefully,promptingorfacilitatingwhen necessaryandleavingquestionstoconfirm
clinicalsuspicionuntilafterthepatienthasfinished.
385
072
PerformanceGuidelines



Presentationtotheexaminerrequiresdiagnosticproblem-solvingskillsabout:
Causesofvertigoofsuddenonsetincludingstrokeortransientischaemicattack.
Knowledgeoftheclinicalpicturepresentedbyobstructionofthebloodsupplytothebrain
stemandcerebellum.
Significanceofcrossedsigns,particularlylossofpainsensationtoleftfacebuttoopposite
sideoftrunkandlimbs.
Appreciationofcardiovascularriskinthispatient.
Abilitytouseclinicalreasoningskillstoexplainneurologicalsignsfoundonphysical
examination.
Appreciatingtheneedforfurtherassessmentonanurgentbasis.
Choosingtheinvestigationmosturgentlyrequired.
Responsetoobservingexaminer
Thecandidateshouldrecognisethatvertigoisofcentralbrainstem/cerebellaroriginratherthan
peripheralvestibularorigin.
Itsacuteonset,associatedsymptomsandsignsandlackoffurtherprogressionsuggestvascular
obstructionratherthanhaemorrhage.Thecerebellumandbrainstemaretheareasinvolved.
Differentialdiagnosis
Allunlikely,butotherpotentialcausesofvertigoinclude
acutelabyrinthitis
benignparoxysmalvertigo
Menieresyndrome
migraine
cerebraltumour
multiplesclerosis
Responsetopatientandimmediatemanagement
Thereisaneedforimmediatehospitaladmission.Advisethatthepatienthashada'mildstroke'
as suspected, but that confirmatory investigation is necessary. This should include urgent
assessmentbyaspecialistphysician/neurologist.
Itwouldbereasonabletoreassurethepatientaboutthefuturebuttoemphasisetheneedtopay
attention to the underlying risk factors which will require ongoing management after recovery
fromthisevent.
Theinclusionofaneurologicalcaseofthiscomplexitymaybemorethreateningtocandidates
thanothercases.Examinersareaskedtotakethisintoaccountwhenmarking.Ifthecandidate
obtainsadetailedhistory,makesareasonableattemptatexplainingthefindingsonneurological
examination, recognises that the presence of neurological signs as described, in addition to
nystagmus, is indicative of a brain stem lesion and realises that this is a serious disorder of
cerebrovascular origin involving the cerebellum/brain stem and that it requires urgent
investigation,thenaclearpasslevelwouldbeachieved.
The posterior inferior cerebellar artery (PICA) syndrome is, however, well documented and
should not be an unduly difficult diagnosis for a well prepared candidate to suspect from the
historyandconfirmbythephysicalfindings.
386
072
PerformanceGuidelines



Investigations
Magnetic resonance imaging (MRI) should be advised associated with hospital admission.
Computed tomography (CT) with CT angiography is also acceptable. Other investigations can
be undertaken later.
Problem sol ving ability
Clinical reasoning skills.
Data assimilation from history and examination.
KEY ISSUES
Recognition of an acute cerebral vascular event affecting the vertebrobasilar system.
Appreciation that this combination of symptoms and signs implies brain stem/cerebellar
disease.
Immediate management including appropriate investigation.
Knowledge of the disease process
Recognition that the patient has had a serious cerebrovascular incident (i.e. 'stroke').
Understanding that the pathology is in the area of the vertebrobasilar arterial system.
Knowledgeable candidates may recognise the likelihood of posterior inferior cerebellar artery
(PICA) obstruction, most likely due to thrombosis of the vertebral artery. Embolism is also
possible. Relationship of this episode to the patient's cardiovascular risk factors should be
recognised.
CRITICAL ERRORS
Failure to recognise likelihood of cerebral/cerebellar vascular lesion.
Failure to advise hospital admission.
COMMENTARY
In this scenario, the sudden onset of vertigo has not been associated with tinnitus or hearing
loss, so the vestibulocochlear system seems likely to be intact.
True vertigo (a sense of rotation between patient and surroundings) is in this instance
accompanied by ataxia (=Greek, without order, in particular a disturbed gait) suggesting an
acute cerebellar disturbance. A cerebellar source is also suggested by the motor
incoordination, without any weakness, in upper and lower limbs.
What else is going on? Other clinical features suggest that a unilateral lower brain stem
disorder is also present Sensory loss to pain is crossed between the face and the
body. There is loss of pain sensation on the left side of the face, but in the trunk and limbs
there is dissociated anaesthesia sensation of pain and temperature is impaired on the
right side
All forms of ascending sensation for projection into the contralateral cerebral hemisphere come
together at the level of the medulla, (where decussation of the uncrossed fibres of vibration and
joint sense [and half touch] in the posterior columns occurs to join previously crossed pain
and temperature fibres [and half touch] running in the spinothalamic tracts) as illustrated in
Figure 1. The combination of cerebellar ataxia and crossed sensory loss suggests a left sided
lower midbrain and cerebellar lesion
072
PerformanceGuidelines

CEREBRAL
HEMISPHERE

Internal
Capsule

MIDBRAIN
Allsensationmodalitiesnow
conjoinedandcontralateral
PONS
Vn.Sensory,
nucleus
Position, vibration, 1/2 touch
crossinuppermedulla
Cuneate & Gracile
tubercles medial
lemniscus
Spinothalamictract(s.t)
Pain
Temperature1/2Touch

SpinalCord
SensoryAscendingPathways
Condition072.Figure1

Areanyotherbrainstemnucleiorlongtractsinvolved?

Yes,thecervicalsympatheticoutflowisinterrupted.Descendingexcitatorysympatheticfibresto
the cervicothoracic outflow are also concentrated in the medulla. There is a left Horner
syndrome(whichfitsaleftsidedlesion)catchingthesympatheticheadandneckoutflow.Loss
ofsensationtotheleftcheeksuggeststhattheleft5thnervesensorypainnucleusisinvolved,
withlossalsoofthecornealreflex.
387
072
PerformanceGuidelines



Thefindingsfitaleftposterolaterallowerlateralmedullaryandleftcerebellarlesion
Thiswouldbeexplainedbyafocalinfarctinvolvingthevertebrobasilarsystem,nofthecarotid
anditsbranches.
Theanatomyofthebloodvesselsandcranialnervesisasillustrated(Figures2-5).

-----C
MEDULLA
PONS
- A
MIDBRAIN
VENTRALVIEW-BLOODVESSELS&CRANIALNERVES

va.vertebralartery
b.a.basilarartery
a.i.e.a.anteriorinferiorcerebellarartery
p.i.e.a.posteriorinferiorcerebellarartery
a.c.aanteriorcommunicatingarteryasa
anteriorspinalartery
__________________________________________________________________________
CONDITION072.FIGURE2.
388
072
Performance Guidelines

Theacutenessofonsetsuggestsembolismorthrombosis.Therearenocardiacarrhythmiasto
favourembolism,soanacutethrombosisaffectingaleftsidedarterysupplyingcerebellumand
brain stem is most likely (distal left vertebral artery). The patient has coexisting vascular risk
factors.Theabsenceofprogression,andsuddenonsetmakeahaemorrhagicstrokelesslikely.
The differential diagnosis would include other causes of vertigo: Meniere syndrome, chronic
petrositis,cerebellopontineangletumour,andotherneurologicalproblems.Noneisaslikelyasa
vascularstroke.
Knowledgeablecandidatesmayrecognisethatthisclusterofsymptomsandsignsisclassicalof
thrombosisoftheposteriorinferiorcerebellarartery(PICAsyndrome).
Thecerebellumandbrainstemreceivetheirbloodsupplyviathesuperiorandinferiorcerebellar
arteries,arisingfrombasilarorvertebralarteries.
TherelevantvascularandcrosssectionalanatomyisindicatedinFigures2-5

Colliculi
3
rd
&4
th
nerve
Ascending
contralateral
sensory
pathways
Descending
sympathetic
fibres
4
Descending
ipsilateral
upper motor neurone
pathways

CROSS-SECTIONMIDBRAIN-A

CONDITION072.FIGURE3.
389
072
Performance Guidelines
CONDITION 072. FIGURE 4.
390
072
PerformanceGuidelines


CROSS-SECTIONMEDULLA-C

CONDITION072.FIGURE5.
The patient's MRI is shown and demonstrates a left sided focal cerebellopontine vascular infarction. The
patientmadearapidrecovery.
CONDITION072.FIGURE6.
MRIofpatient'sheadshowingleftcerebellarandbrainstemischaemicinfarction
391
392
072
PerformanceGuidelines



Cerebellarfunctionsincludeipsilateralstabilisationofmotormovementsandcoordinationand
balance.Differentiationofdifferentcausesofataxiacanbehelpedbyassociatedsymptoms--
sensoryataxiaduetolossofpositionsenseisworseindarkconditions.
CerebellarDisturbancescause:
Cerebellarataxicgaitwithastaggeringbroadbasedgaitlikeadrunkensailor,andatendency
tofalltothesideofthelesion.Thegaitofsensoryataxiafrombilateraldorsalcolumnlosswith
lossofpositionsenseisbycontrastahigh'stamping'gaitwithpositiveRombergism(instability
standingwitheyesshut).
Cerebellar incoordination Various tests evince incoordination of upper and lower limbs -
'past-pointing';'finger-nosetests' witheyesopen orclosed;'dysdiadochokinesia'onrhythmic
pronation-supination,oralternatehandslapping;orknee-shin-ankleplacementbyotherfoot,or
rhythmicflexion-extensionofankle.
Additionalbrainstemdamagemaybefound,forexample:
ipsilateral5thnervemusclesofmastication,facialsensation
ipsilateral7thnervelossoftastetosideoftongueandmotorweakness
ipsilateral8thnervedisturbanceofhearing
ipsilateral9thnervedifficultieswithswallowing
ipsilateral10thnervedysarthria
ThePICAsyndromeclassicallypresents,asinthiscase, witha dramaticonset ofcerebellar
signswithataxiaandvertigo,usuallywithouttinnitusordeafness.
AssociatedcerebralsympatheticparalysiswithanipsilateralHornersyndrome(ptosis,miosis,
anhydrosis, enophthalmos) from medullary brain stem involvement is common, as is an
ipsilateral loss of facial sensation to pain due to 5th nerve involvement, and other medullary
brainstemnucleimaybeaffected.
Loss of pain and temperature sensation from opposite (right) side of the body due to
involvementofleftspinothalamictractisalsoseen.
Dissociatedanaesthesia(diminutionofpainandtemperaturesensationwithretentionoftouch
andofotherformsofsensation)isclassicalofabrainstemorspinallesionbelow
the pons, and occurs most notably in Brown-Squard Syndrome (hemisection of cord) with
findingsasillustrated(Figure7):
Focalipsilaterallowermotorneuronlesionatthelevelofthespinalcordinjury.
Ipsilateraluppermotorneuronlesionparalysisbelowtheinjury.
Ipsilateraldorsalcolumnsensoryloss(positionandvibrationsense)belowtheinjury
Contralateralspinothalamicloss(painandtemperature)belowtheinjury
072
Performance
Guidelines
CONDITION 072. FIGURE 7.
393
394
073
PerformanceGuidelines



Condition 073
Urinaryfrequencyina60-year-oldman

AIMSOFSTATION
To assess the candidate's history-taking skills, knowledge of the symptomatology and
confirmatorytestingformaturityonsetType2diabetesandtheinvestigationswhichshouldbe
undertakeninarecentlydiagnoseddiabetic.

EXAMINERINSTRUCTIONS
Theexaminerwillhaveinstructedthepatientasfollows:
Youareaged60years.Youareconsultingyourgeneralpractitionerabouturinarysymptoms.
You also have concerns about cancer (your father had prostate cancer) and loss of sexual
function,buttheseshouldnotbeimmediatelyrevealedtothedoctor.
OpeningStatement:
Iseemtoneedtogotothetoilettopassurinemoreoftenlatelydoctor.
Inresponsetothedoctor'senquiries,respondasfollowsbutdonotvolunteerallthis
informationwithoutappropriatepromptingbythedoctor.
Overaboutthelastthreemonthsyouhavebeenpassingurinemoreoftenduringthedayand
havetogetoutofbedtopassurineatleasttwiceeachnight.Youalsosufferfromlegcramps,
worse at night, and your feet have felt slightly numb. You have felt thirstier lately and your
mouthhasbeendry.
Youhavebeenworriedthatyoursymptomsareduetoprostatetroublebecauseofyourfather's
historyandrecentpublicityaboutprostatecancer
Reviewofgeneralhealth
You have lost 4 kg in weight, over the past three months. Admit to feeling tired recently
'MaybeI'mjustworried.' Admittolossoflibidoandinabilitytoobtainandsustainanerection
overlast3-4months.Admittorecentdeteriorationineyesight,ifasked.IsupposeIneednew
glassesatmyage.'
Reviewofrelevantsystems
No other deviations from normal. No dysuria. No incontinence. Normal stream. No other
symptoms suggestive of prostatism with bladder neck obstruction. No chest pain or
breathlessness.
You are a previous patient but not well known to the doctor. Be pleasant, straightforward,
exceptforsomeembarrassmentoversexualactivity.Youareworriedaboutprostatecancer.
Thedoctormayaskadditionalquestionsaboutyou.Ifso.respondasfollows:
Smokinghabits:
Alcoholuse:Drug
sensitivities
Familyhistory:
Nonsmoker
Twocanslightaledaily
Nil
Fatherdiedfromastrokeaged80yearsalsohadprostatecancer.
MotherinnursinghomeDementia.
Pastmedicalhistory:noseriousillnesses.
395
073
PerformanceGuidelines



Physicalexaminationfindingstobegiventothecandidatefromexamineronrequest
Heissignificantlyoverweightwithabdominalobesity.Bloodpressureis140/90mmHg,pulseis
regular.Hehasmildbluntingofallsensorymodalitiesinhisfeet.Neurologicalexaminationis
otherwise normal. Genital and rectal examinations are normal, without evidence of prostatic
enlargementornodularity.
EXPECTATIONSOFCANDIDATEPERFORMANCE
Summarisetheproblemspresentedbythepatient:
~urinarysymptoms,thirst,weightloss;
~numbnessinfeetandvisualdisturbance:
~fearofcancer;
~erectiledysfunction/reducedlibido;and
~maturity-onsetdiabetesasthemostlikelydiagnosis.
Requesturinalysisandrandombloodsugarusingglucometer.

ESSENTIALOFFICEINVESTIGATIONSTOBEPROVIDEDBYEXAMINERWHEN
REQUESTED
Urinalysispositiveforglucose(++++),ketones(+),negativeforprotein.
Randombloodsugarshouldbedoneintheconsultingroomwithglucometer.Resultof21
mmol/Leffectivelyconfirmsdiagnosisofdiabetesmellitus.
OTHERINVESTIGATIONS
urea,creatinineandelectrolytelevels;
glucosetolerancetesttoconfirmdefinitivediagnosis(althoughthelevelofrandomblood
sugarputsthediagnosiseffectivelybeyonddoubt);
microscopyandcultureurinecheckingformicroalbuminuria;
serumlipidscholesterol,triglycerides,Low/HighDensityLipoprotein(LDL/HDL)andratio;
ECGtocheckforpresenceofundiagnosedischaemicheartdisease;
glycosylatedHbshouldbedoneasbaseline;
fullbloodexaminationanderythrocytesedimentationrate:and
prostatespecificantigen(PSA)levelindicatedinviewofhisfamilyhistoryandconcerns.

KEYISSUES
DiagnosisofType2diabetesmellitusbyappropriateconsultation(history,examinationand
officetests).
Appropriatefurtherinvestigationofnewlydiagnoseddiabetic.
Recognitionoffearofcancerandsexualdysfunction.

CRITICALERROR
Failuretotesturineormeasurerandombloodsugaratthisconsultation
396
073
Performance Guidelines


COMMENTARY
The constellation of symptoms of polyuria, thirst, weight loss, sensory and visual disturbances, and erectile
dysfunction, should raise suspicion of maturity-onset Type 2 diabetes mellitus.
Urinary chemical testing and random blood sugar assessment applied to overweight adults over 45 years will pick up
at least as many nonsymptomatic undiagnosed diabetics as are found on symptomatic presentation.
2-E:ThePaediatricConsultation

PeterJVine
' Chi l drenarenotsi mpl ymi cro-adul ts,buthavethei rownspeci fi cprobl ems.'
BetaSchi ck(1877-1967)
There are many very positive features about working with
childrentheyaremuchlesscomplexthanadultsandusually
under most clinical situations have only one presenting
complaint, unencumbered by a series of complicating past
eventsorofage-relateddisease.
Medical care and assessment of children is often a
multidisciplinaryprocess.Contrarytopopular
Recognitionofthefeatures
inthehistoryandinthe
clinicalsignsthatindicate
thatachildissignificantlyill
isaskillthatmustbe
developedbyanyonecaring
beliefinsomecircles,childrenarenotjustscaled-downadults,butrathertheirneedsareunder
the influence of a variety of variable factors, all of which have a profound effect on the
developmentofthechildasheorsheprogressestoadulthood.Alloftheseinfluencestherefore
mustbetakenintoaccountwhenconsultingwithchildrenandthesevarydependingontheage
ofthechild.
ChildreninAustraliahavegenerallybeensparedthetraumasexperiencedbytheirpeersinthird
worldcountries,orthosetornapartanddisruptedbywarandnaturaldisaster.Refugeechildren
come from other lands rather than our own, and have their own problems related to this
background. However within our indigenous populations, the health of the children often is
equivalent to those in developing countries with high infant mortality, a high incidence of
conditions uncommonly seen in the urban populations (for example, rheumatic fever), and a
reducedadultlifespan,
Australiaisamulticulturalnationwith25%ofthepopulationbeingbornoverseasaccordingto
the latest Census.
1
Many children are first generation Australians born in this country to
immigrant parents. Our capital cities in particular have people from many nations residing in
themwhohaveculturalbeliefsandpracticesofwhichdoctorsneedtobeaware.Exceptinthe
caseofolderchildrenwherethedirecthistoryfromthechildismostappropriate,thepaediatric
history is usually given by a third person, commonly a parent or caregiver. Often with new
arrivals,thehistoryisgivenbyyetanotherintermediary,aninterpreter,whomayormaynotbea
relative,whichmayaddyetanotherdimensiontotheconsultation.
Manymedicalpractitionersadmittobeingratherfrightenedattheprospectofcaringforchildren,
astheprocessissodifferentfromthatrelatedtoadultswherethehistoryisobtainedfromthe
patients themselves. Other doctors are apprehensive at being able to perform an adequate
examinationofachild.Nowhereelseinmedicineisitsoessentialtohaveexpertobservation
skillsthaninpaediatrics.Manydiagnosescanbemadejustbyobservationofthechildwhilethe
historyisobtained,beforeanyformalexaminationisperformed.
Recognition of the features in the history and in the clinical signs that indicate that a child is
significantly ill is a skill that must be developed by anyone caring for children. While they do
becomeillquicklyand,ifuntreated,deterioratemorerapidlythanadults,childrenalsorepairand
recoverquickly.Itisimperativethatifanychildisnotimprovingatatimewhenimprovementis
expected, an immediate investigation into what might be complicating the situation must be
instigated.

1AustralianBureauofStatistics2005
397
398
2-E
The Paediatric Consultation



While many illnesses seen in children also occur in adults (for example, asthma) the requisite skill is knowing the
variations that must be considered in the child which will influence future management. An example is the method of
administering bronchodilator therapy in young children. While the same medications are used, their methods of
administration are very different, with small volume spacers and masks designed for very young children.
There are also many conditions that are specific to children, for example hypertrophic pyloric stenosis and cystic
fibrosis although the latter has now become an adult disease, and adult practitioners need to be aware of
management of this condition. While intussusception can occur in adults in later years, secondary to a number of
bowel lesions, 'primary' intussusception is a very prominent condition in infants, which must be considered in any
infant presenting with colicky abdominal pain. In this country, the majority of children seen in clinical practice present
with relatively minor complaints, but practitioners must always be vigilant for the circumstances when they should be
considering more complex conditions specific to infants and children. Such situations are usually recognised from the
history or the appearance of the child on examination. Beware of the listless infant or child who allows you to perform
any examination you wish. This is usually a very sick child.
Prescription of drugs is also different in children: most drugs are given on a mg per kg basis up to a certain weight and
age. Similarly intravenous and oral fluids are calculated on a mL per kilogram basis and need to be calculated
carefully for each child. Specific pocket handbooks with this information are published by several of the major
Australian paediatric teaching hospitals.
The needs of children to develop their potential and to remain in good health are legion. While we practise in a
so-called developed country, we still have a significant percentage of our child population who live in extremely
adverse circumstances, whether these relate to poor nutrition, poor socioeconomic circumstances, poor parenting
skills, harmful emotional environment or even deprivation. Many primary schools arrange breakfast for their pupils as
for one reason or another the children leave home for school having not eaten.
As each year goes by, the needs of the child change. Many parents find it difficult to provide for those needs, to the
detriment of the child's development. The general practitioner is often an appropriate person to assess this situation.
Unfortunately all too commonly children in our country are the subjects of abuse, whether it be physical, sexual or
emotional, and the medical practitioner needs to be alert to this possibility, especially when the presentation is at
odds with what is observed. Australian law mandates that in each suspicious case, the relevant appropriate
authorities are to be notified.
As is typical of the industrialised countries, the spectrum of conditions seen by medical practitioners has changed
dramatically over the last couple of decades. Rather than malnutrition, many of the problems we see are related to
inappropriate nutritional habits and inactivity leading to obesity.
Emotional and behavioural problems are common and often relate to the child's life experiences.
1
Some situations
that may influence these are:
many children under two years of age participate in formal child care while parents work, almost a necessity for
maintaining a suitable income:

1 Practical Paediatrics Ed. MJ Robinson, DMRoberton 5th Ed. Churchill Livingstone 2003 p2.
399
2-E
The Paediatric Consultation
the extended family in many communities is scattered and less accessible, as young adults move freely around the
country seeking employment;
one in five children will experience divorce of their parents before mid teens;
a significant percentage of children live in one-parent families; many in two-parent families live in very unhappy
circumstances;
with the higher divorce rate, blended families, where children from previous marriages live together, can often be a
source of major conflict. Having two 13-year-old females suddenly living together can be quite trying; and
tobacco and/or alcohol use, especially binge drinking, is common in teenagers, and most high school students are
aware of where they can obtain cannabis and other drugs.
Years ago, Dr Howard Williams of Melbourne, a mentor to many practising Australian paediatricians and one of the
forerunners of paediatrics in this country, used to urge his postgraduate students to be aware of 'the new morbidity'. He
stated that his generation had overcome much of the infectious disease morbidity and mortality with antibiotics and
immunisation, but that behaviour problems, disrupted families and the effect on the children involved would be a major
part of the work of the modern practitioner. How right he was, as much of the consultation time of paediatricians in this
country is taken up with oppositional defiant behaviour, attention deficit disorder, and other developmental behavioural
problems.
Much of this may be related to the sociological change in child rearing. Children of today largely depend on artificial
media for entertainment in their spare time television, cinema, electronic games, many of which require little if any
intellectual skills and commonly have a strong base of violence and aggression. Add to this a high divorce rate and the
loss in many instances of the extended family and the scene may be ripe for acting-out behaviour.
The physical and emotional needs of the growing child must therefore be kept to the fore when children are being
assessed. The presentation of a child with an emotional problem can be quite varied and commonly may be
organically based, so that the practitioner must be very alert to this possibility.
The care of the disabled child, whether it is a physical or intellectual disability, often falls to the primary practitioner for
day-to-day events. Detailed knowledge of rare conditions is not generally necessary, but the support given to parents
as they advocate for their offspring can be a major role asked of the practitioner. Advice concerning screening
procedures and genetics is also a common question, which the practitioner should refer to a higher authority, as the
explosion of knowledge in these fields is occurring at such an alarming rate that it has outstripped the ability of most of
us to keep up-to-date.
The internet has revolutionised the practice of medicine, including paediatrics, as parents consult the internet for
advice on conditions their children are reported to have. Often parents may self-diagnose based on this information,
but generally present with their downloaded information asking for explanation of the contents. Many sites
unfortunately are inaccurate and anecdotal. Hence the practitioner's role often is to sift through this information and to
give an accurate prcis of the particular condition.
A complete history, examination and discussion with a parent of a child's problem can take considerable time. In the
AMC MCAT examination, only certain aspects will be examined in any one scenario. For example, the task may
involve coming to a diagnosis from the information supplied and then counselling a parent on the management of the
child's condition. Or it may be taking a focused history to determine the cause of the presentation.
400
2-E
The Paediatric Consultation



Candidates therefore are assessed on their ability to relate to a worried parent of a sick child at a standard
expected by the Australian community.
It can be seen then that working with children, while it can be a complex business, is generally quite ordered and
rewarding if aware of the various factors that influence the development both physically and emotionally of the
child, as well as being mindful and knowledgeable about the specific conditions that are peculiar to children.
Children are fun to work with, are honest and much less complex than most adults. They do however have special
needs and are afflicted by many conditions specific to their age group, whether it be neonate or teenager, and the
competent practitioner needs to be aware of these conditions in order to consider them, no matter how minor the
complaint appears to be. The ability to counsel worried parents in an empathie manner is paramount for successful
paediatric practice.

Peter J Vine
401
2-E
ThePaediatricConsultation



2-EThePaediatricConsultation
CandidateInformationandTasks
MCAT074-077
74 Neonataljaundiceinthefirstdayoflife
75 Immunisationadvicetotheparentofa6-week-oldbaby
76 Darkurine,facialswellingandirritabilityina5-year-oldboy
77 Feverandsorethroatina5-year-oldboy
402
074-075
Candidate Information and Tasks



Condition 074
Neonataljaundiceinthefirstdayoflife
CANDIDATEINFORMATIONANDTASKSHistory
Youareaskedtoseeaninfant,Jessica,born24hoursago,forjaundice.Sheisthefirstchildof
ahealthymother,whosepregnancywasnormal.Deliverywasatterm,byamidwife,andwas
uneventful.Theinfantweighed3700gatbirth.Jaundicewasnoticedsoonafterbirth,withinthe
first24hours.Theinfanthasbeensuckingwellatthebreast.Themotherwantstogohomeas
soonaspossible.
ExaminationFindings
Theinfantisclinicallyjaundicedbutotherwisewellandactivewithnohepatosplenomegalyor
otherabnormalphysicalsigns.
Youhaveobtainedallrelevantfindingsonhistoryandexamination.

YOURTASKSARETO:
Asktheobservingexaminerforresultsofanyinvestigationsyouconsidernecessary.
Advisetheparentondiagnosisandmanagement.

ThePerformanceGuidelinesforCondition074canbefoundonpage405

Condition 075
Immunisationadvicetotheparentofa6-week-oldbaby

CANDIDATEINFORMATIONANDTASKS
YournextpatientisbabyLaurabroughtbyhermothertoageneralpracticeatsixweeksofage,
aspartofroutinepostnatalfollowup.Lauraisthecouplesfirstchild.Thebabeisbreastfedand
gainingweightnormally.Hermotherwantstoknowwhatyouwouldadviseaboutimmunisation
becausesheandherhusbandhaverecentlyheardconflictingviewsexpressedinthemedia.
General examination of the baby reveals no abnormality. She was given her first hepatitis B
vaccinationsoonafterbirth.

YOURTASKSARETO:
Outlinethecurrentimmunisationprotocolyouwouldrecommendandwhatdiseasesthe
programmeisprotectingagainst.
Discussanyconcernstheparentshaveaboutimmunisation.
Youwillnotbeexpectedtotakeanyadditionalhistoryoraskforexaminationfindings.The
PerformanceGuidelinesforCondition075canbefoundonpage408
403
076-077
CandidateInformationandTasks



Condition 076
Darkurine,facialswellingandirritabilityina5-year-oldboy
CANDIDATEINFORMATIONANDTASKS
A five-year-old boy is brought to the Emergency Department because of swelling around the
eyes.Hehasonlybeenpassingsmallamountsofurine,whichisdarkincolour.Inthepast12
hourshehasbecomerestlessandirritable.
The child had school sores (impetigo) three weeks ago, treated successfully with a topical
antibioticcream,buthashadnootherpriorillnesses.
Bothparentsarewell.Thechildisanonlychildandhasalwayskeptingoodhealth.

YOURTASKSARETO:
Asktheexaminerfortherelevantphysicalfindingsyouwishtoelicit.
Discusswiththeparentyourprovisionaldiagnosis.
Advisedetailsofanyinvestigationsthatarerequiredandadvisetheparentofthetreatment
thatwillbeneeded.

Youdonotneedtotakeanyfurtherhistory.
ThePerformanceGuidelinesforCondition076canbefoundonpage412

Condition 077
Feverandsorethroatina5-year-oldboy
CANDIDATEINFORMATIONANDTASKS
Peter,afive-year-oldboyisbroughttoyouinageneralpracticesettingbyhisparentwithafever
of 40 C that developed overnight. He complains of an intensely sore throat and finds it sore
henheswallowsfoodorfluid,althoughheisabletodoso. w

YOURTASKSARETO:

Indicatetotheexaminertheclinicalexaminationyouwouldperformtodiagnosetheproblem.
Theexaminerwillgiveyoutheresultsofthephysicalexamination.
Discusswiththeparentanyinvestigationsyoufeelarenecessary.
Explainyourdiagnosisandsuggestmanagementtothemother.
Youdonotneedtotakeanyfurtherhistory.
ThePerformanceGuidelinesforCondition077canbefoundonpage414
404
2-E
The Paediatric Consultation



2-EThePaediatricConsultation
PerformanceGuidelines
MCAT074-077
74 Neonataljaundiceinthefirstdayoflife
75 Immunisationadvicetotheparentofa6-week-o!dbaby
76 Darkurine,facialswellingandirritabilityina5-year-oldboy
77 Feverandsorethroatina5-year-oldboy
405
074
Performance Guidelines



Condition 074
Neonatal jaundice in the first day of life

AIMS OF STATION
To assess the candidate's knowledge of causes of neonatal jaundice occurring in the first 24
hours after birth, and the appropriate management of the condition.

EXAMINER INSTRUCTIONS
This scenario illustrates the common problem of ABO blood group incompatibility with the classic
combination of a mother group O Positive and a baby A Positive, and a strongly positive Coombs
test.
The baby's bilirubin level has reached a total of 250 umol/L at 24 hours of age. Phototherapy is
required, which should prevent further rise in bilirubin, but will be needed for several days.
The problem is compounded by the mother's disappointment. She is a young professional
woman who wanted a completely natural delivery and management and is disappointed that she
is not allowed to go home as her infant requires treatment.

Investigation results/details to be gi ven to candidate by examiner on request
Tests performed:
Mother's blood group 0 Rh positive.
Infant's blood group A Rh positive. Direct Coombs test strongly positive.
Infant's Hb 170 g/L.
Blood film microspherocytes.
Bilirubin Total 250 umol/L
Conjugated 6 umol/L at 24 hours.
The biochemist indicates that this is abnormal, but below the range at which exchange
transfusion is indicated.

The examiner will have instructed the parent as follows:
You gave birth to your first child 24 hours ago.
You believe childbirth is a natural phenomenon, and resent medical intervention.
You are well educated, and were recently a middle-level manager in a successful company.
You insisted your obstetrician allowed you to have a natural childbirth with appropriate
assistance from a midwife, and that you could go home on day two.
Now that your babe has become jaundiced, you are confused and upset.
After appropriate discussion, you will accept the doctor's recommendations if they are given
clearly and empathically.
406
074
Performance Guidelines



Questions to ask or statements you could make:
I expected everything to be normal. '
Why do I need to stay longer in hospital? I want to go home. '
Is treatment really necessary?'
What would happen if no treatment were given?'
'Are there any side effects of this light treatment?'
EXPECTATIONS OF CANDIDATE PERFORMANCE
The candidate should explain the following:
Jaundice occurring in the first 24 hours after birth is not due to immature liver function, but usually due to
haemolysis consequence upon blood group incompatibility. In a primiparous woman, ABO incompatibility would
be the most likely cause.
ABO incompatibility this has been confirmed by the tests done.
Consequence of severe neonatal jaundice and the need for phototherapy and monitoring.
Exchange transfusion unlikely to be required but could be an option if jaundice worsens despite phototherapy.
The technique of phototherapy, its side-effects and reassurance regarding aspects which could cause anxiety:
~ Jessica's bowel motion may be a loose green/black colour while under lights.
~ Her eyes will be covered while she is under lights to protect her eyes
~ Baby is only under lights when not feeding and is sleeping.
~ Phototherapy may be able to be given in the room where mother is staying in hospital.
Excellent prognosis.
Arrange continued stay in hospital for mother and infant with facility for mother to continue breastfeeding.
Followup developmental assessment and audiometry not usually discussed at this first consultation.
KEY ISSUES
Recognition of haemolytic disease of newborn and its immediate treatment.
Empathie but realistic communication with new parent.
Ability to relate to mother's disappointment with need for medical intervention.
CRITICAL ERROR
Failure to recognise haemolytic disease of newborn and failure to advise phototherapy.

407
074
Performance Guidelines



COMMENTARY
The dominant cue in this example is neonatal jaundice in the first day of life.
Jaundice occurs frequently in the neonatal period, but when seen on the first day of life, it indicates a pathological and
potentially dangerous rise in bilirubin level. In the common, so-called 'physiological jaundice', the serum bilirubin rises
more slowly, and jaundice is not apparent on the first day.
Haemolytic disease is the most common cause of potentially dangerous neonatal jaundice. As it is readily treatable,
and complications potentially avoidable, early diagnosis is mandatory. Hyperbilirubinaemia is likely to reach a
maximum level around the third day of life.
There is a diagnostic rule that 'jaundice on the first day of life is haemolytic unless
proven otherwise' This originated in the days when Rh haemolytic disease was a common cause. The jaundice of
the affected infant could increase rapidly, so immediate diagnosis and often exchange transfusion were required to
avoid the serious complication of kernicterus or later nerve deafness. The candidate familiar with this rule will
immediately refine the cue to: 'J aundice on the first day of life, probably haemolytic' This scenario is an example
of the need for pattern recognition where urgent diagnosis is required.
Toframe the problem more clearly, the clinician needs to seek evidence confirming the existence of haemolysis and
defining the degree of hyperbilirubinaemia. The protocol states that hepatosplenomegaly is not present. This makes
the severe intrauterine haemolysis seen in some cases of Rh haemolytic disease less likely, but does not exclude less
dramatic forms of haemolysis.
The crucial laboratory tests in establishing the diagnosis are examination of blood group of mother and infant, and
direct Coombs test. The scenario of a group O Positive mother and a group A Positive infant indicates the potential for
the infant's blood to be harmed by maternal anti-A antibody. The positive direct Coombs test confirms that the infant's
red cells have been sensitised by antibody and establishes the diagnosis of haemolytic disease due to AO
incompatibility.
In deciding management and providing further confirmation of the diagnosis, estimation of serum bilirubin level
(direct-reacting and indirect-reacting) should be performed. Bilirubin is derived from the catabolism of haeme proteins
produced in the breakdown of red blood cells. Unconjugated (indirect-reacting) bilirubin is converted in the liver to
conjugated (direct-reacting) bilirubin, and excreted into the bile. Conjugated bilirubin is not reabsorbed once it enters
the intestinal tract. In the present scenario, the level of bilirubin is insufficient to warrant exchange transfusion, but the
clinical picture, combined with laboratory confirmation of an unconjugated hyperbilirubinaemia exceeding 240 umol/L,
confirms the need for treatment with phototherapy.
The degree of haemolysis should be defined by measuring the infant's haemoglobin, though simple transfusion for
correction of the anaemia will rarely be required in AO haemolytic disease and is not needed here.
In this scenario, discussion of other haemolytic or nonhaemolytic causes of neonatal jaundice is not required once the
problem is correctly framed as 'jaundice on the first day of life'.
As indicated in the examiners' 'Performance Criteria', the candidate is not only expected to make the diagnosis, but to
provide information in a persuasive and lucid manner to justify medical intervention, while recognising the mother's
disappointment.
408
075
PerformanceGuidelines



Condition 075
Immunisationadvicetotheparentofa6-week-oldbaby

AIMSOFSTATION
Toassessthecandidate'sknowledgeofthecurrentlyrecommendedimmunisationprogramme
in Australia, knowledge of side effects and the latest information concerning claims of
associationswithseriousmedicalconditions.

EXAMINERINSTRUCTIONS
Theexaminerwillhaveinstructedtheparentasfollows:
Youarethemotherofasix-week-oldbaby,Laura.
Youareaneducatedparentinastablemarriage.
Youare widelyreadandtakeaninterestin popularmedicalarticlesespeciallyduringyour
pregnancy,asyouwereconcernedaboutsomeinformationyouhadreadandheardabout
immunisationanditspossibleadverseeffects.
Youaretakingtheopportunityofthesixweekvisittohavetheseconcernsclarified.
Questionstoaskunlessalreadycovered:
'What vaccines or injections will Laura need to be immunised with, up to school
age?'
' Whatproblemsmightshehave?'
' Arethereanychildrenwhoshouldn'thavethesevaccines
9
'
'I'veheardthewhoopingcoughvaccinecancausebraindamage.Canweleaveit
out
9
'
' Whatothersideeffectshappenwiththesevaccines
9
'
' What if we didn' t give these vaccines can' t you just treat any infections with
antibioticsanyway?'
Ihave a friend who goes to a homoeopath and he gives the same vaccines, but
verydiluted,bymouthandtherearenosideeffects.Isthatanalternative?'
' Whataboutotheralternativevaccines?'
' I' ve heard babies can get high fever and be guite sick after some of these
injections.Canyoudoanythingtoeasethesideeffects?'
' I' ve also heard about a vaccine for chicken pox. Is this available and do you
adviseit?

EXPECTATIONSOFCANDIDATEPERFORMANCE
Thecandidateshouldgivetheparentasuccinctandaccurateregimenfortheimmunisationthat
iscurrentlyrecommended,withspecialreferencetothosegivenattwomonths.Ifuncertain,the
candidateshouldbeawareofthecurrentNHMRCImmunisationGuidelinesandhowtoaccess
them.
Examinersshouldbeawarethattherecommendedschedulehaschangedannuallyoverthelast
severalyearsasnewvaccineshavebeenintroduced.Inthefuture,combinationvaccineswhich
will reduce the number of injections are expected and small variations to the schedule will be
neededtoaccommodatethese.Toreducerelianceonsinglesuppliers,severalversionsofthese
combination vaccines are likely to be approved and thus variations as currently seen in the
schedulewillbecomemorecommon.Becauseofthis,candidatesshoulddemonstratefamiliarity
withthebasicprinciplesoftheimmunisation
409
075
Performance Guidelines



schedule rather than detailed knowledge of precise recommendations, particularly for States
other than the one in which they work.
Examiners should also be aware that the level of understanding expected should match the
current edition of the immunisation handbook rather than vaccines introduced subsequent to
publication of the handbook. The following would generally be recommended:
At birth: Hepatitis B (hepB).
At 2 months and 4 months: Acellular diphtheria, tetanus, pertussis (DTP
a
); H.influenzae
type B (Hib); oral or inactivated polio vaccine (O/IPV); hepatitis B (hepB); 7-valent
pneumococcal conjugate vaccine (7
V
PCV).
At 6 months: Acellular diphtheria, tetanus, pertussis (DTP
a
); oral or inactivated polio
vaccine (O/IPV); (hepatitis B [hepB] in NSW, QLD, SA. NT); 7-valent pneumococcal
conjugate vaccine (7
V
PCV).
At 12 months: Measles, mumps, rubella (MMR); H.influenzae type B (Hib);
meningococcus (MenC): (hepatitis B [hepB]) in VIC, WA, TAS).
At 18 months: Varicella zoster virus (VZV); 23-valent pneumococcal polysaccharide
vaccine (23
V
PPV).
At 4 years: Acellular diphtheria, tetanus, pertussis (DTP
a
); measles, mumps, rubella
(MMR); oral or inactivated polio vaccine (O/IPV).

Examiners should note that there are slightl y different recommendations for the
immunisation schedule from State to State. What is being tested in this scenario is
whether the candidate is aware of the general principles for DTP, Hib, polio, hepB, MMR,
meningococcus, pneumococcus, chicken pox.
Candidates should address specific concerns that the mother may have regarding possible side
effects and the incidence of these. If the parent has no particular concerns, the candidate should
discuss known side effects and how these can be reduced, but stress that these are few and
minor and that vaccinations are safe. Also stress the marked decrease in the incidence of side
effects with the use of acellular vaccines for pertussis.
Candidates should discuss the few absolute contraindications to vaccination. These are
encephalopathy within seven days of a previous DTP-containing vaccine or an immediate severe
or anaphylactic reaction to vaccination with DTP. A simple febrile convulsion or preexisting
neurologic disease are not contraindications to pertussis vaccine.
Children with minor illnesses, i.e. without systemic illness and providing the temperature is less
than 38.5 "C, may be vaccinated safely. With a major illness or a high fever, the vaccination
should be postponed until the child is well.
Live vaccines (MMR, oral poliomyelitis, rubella, chicken pox) should not be administered to
immunocompromised patients. An anaphylactic reaction to egg is not a contraindication to MMR
vaccine, but many authorities recommend that in such a case it should be administered in an area
where resuscitative equipment is available and the child be observed for 4 hours.
The following are NOT contraindications to any of the vaccines in the standard schedule:
family history of adverse reactions to immunisation;
family history of convulsions;
previous pertussis-like illness, measles, mumps or rubella infection;
prematurity (immunisation should not be delayed);
075
410
PerformanceGuidelines
stableneurologicalconditions(e.g.cerebralpalsy,Downsyndrome);
contactwithaninfectiousdisease
asthma,eczema,atopy,hayfever,'snuffles';
treatmentwithantibiotics;
treatmentwithlocallyinhaledorlowdosetopicalsteroids;
child'smotherispregnant;
childisbreastfed;
historyofjaundiceafterbirth;
overtheagerecommendedinvaccinationschedule;
recentorimminentsurgery;and
replacementcorticosteroids.
Candidates may mention the reported association of measles vaccination with autism The
knowledgeable candidate will be aware that no association has been convincingly
demonstratedandseveralstudiesshownolinkatall.Candidatesshouldstatethatthereisno
evidenceforthe efficacyofalternative(homoeopathic)oralvaccinesgivensublingual^Latest
vaccination now available for varicella-zoster virus (the cause of chicken pox) and
meningococcusshouldbediscussedandrecommended.
Candidatesshouldstressthatmanyofthesediseasesarestillprevalentinthecommunity(e.g.
pertussis, pneumococcal and meningococcal infections, and varicella). Candidates may
suggestparacetamolforfeverandpainaftervaccinationasnecessary,includingasingledose
about30minutespriortoDTP
a
prophylacticallyandforsubsequentimmunisationifsignificant
reactionwithfeverwithfirstorseconddose.

KEYISSUES
Knowledgeofbasicprinciplesofcurrentimmunisationregimens.
Explanationandaccuratenformationregardingbenefitsofimmunisation
Explorationofparentalconcerns.

CRITICALERRORS
Candidateprovideswrongadviceregardingcontraindicationstoimmunisation.
Recommendationoracceptanceofsublingualhomoeopathicvaccines.

COMMENTARY
This scenario is concerned with counselling a young mother on the advantages and
disadvantagesofimmunisation.Thisrequiresofcandidatesasoundknowledgeofthetopicand
an ability to give the information to the parent in a manner that gives a balanced overview,
withoutdomineeringwiththeirownpersonalfeelings.Thisisaverycommonsituationingeneral
practicewherepatientswilloftenattendtodiscusswiththedoctor,beliefstheyhave,ortoseek
further information on a topic. Doctors should not hesitate to admit that they do not know a
particular answer but should offer to seek the answer and communicate it at a later date.
Updated immunisation schedules such as the one illustrated are available from paediatric
hospitals.
Candidatesshouldbeawareoftheabsolutecontraindicationstothestandardvaccinationsand
alsothefalsecontraindicationswhicharesooftenquoted.
Immunisation schedule guidelines adapted from Royal Children's Hospital, Melbourne, 2006
CONDITION 075. FIGURE 1.

411
412
076
PerformanceGuidelines



Condition 076
Darkurine,facialswellingandirritabilityina5-year-oldboy

AIMSOFSTATION
To assess the candidate's ability to recognise that this child most likely has acute post-
streptococcalglomerulonephritis(PSGN)whichrequireshospitalisationinviewofhypertension
andrecentirritability.

EXAMINERINSTRUCTIONS
Theexaminerwillhaveinstructedtheparentasfollows:
Youarethe parentofafive-year-oldboy.Youareparticularlyconcernedaboutthe darkurine
andswellingoftheboy'sface.Nothinglikethishaseverhappenedbefore.Thechildhasnever
beenreallysickbefore.Thisillnessisallveryunusualandworrying.
Questionstoaskunlessalreadycovered:
' Whyhasthishappened?'
' Willmysonbeallright?'
'Whatisgoingtohappennow?' (Ifhospitalisationisrecommended)
' Whatisgoingtobedonetomysoninhospital?' (Ifhospitalisationisrecommended)
' Whataretheylookingforwiththesetests?'
' Howlongwillittaketogetresults?'
Ifkidneybiopsyismentioned,becomeevenmoreconcerned.
Isthatreallynecessary?'
' Whyisbloodtestingnotenough?'

Relevantphysicalfindingstobegiventothecandidateonrequest
Restingbloodpressure 145/90mmHg.Noposturalhypotension.
Temperature 36.5C.
Pulse 90/minregular.
Periorbitaloedemapresentnooedemaelsewhere.
noascitesorpleuraleffusions.
Cardiovascularsystem normal.
Liveredge palpablejustbelowthecostalmargin.
Opticfundi normal.
ENTexamination normal.
Urinedipstick stronglypositive(++)forbloodandprotein.

EXPECTATIONSOFCANDIDATEPERFORMANCE
The candidate should explain the cause of the child's clinical picture in terms the parent can
understand,andwithoutmedicaljargon.Thiswouldincludethattheoriginalskinstreptococcal
infection(impetigo)hastriggeredanimmunereactionofthebodyagainsttheorganismandthatthis
reactionisoccurringinthekidneyscausingamajoreffecton

413
076
PerformanceGuidelines



theirfunction.Thisthenleadstoafallinurineoutput,andsaltandfluidretentionthatcausesthe
swellingofhiseyesandraisedbloodpressure.
Hospitaladmissionisdesirableinviewoftheacutepresentationandhypertension
Investigationsrequiredwillincludebloodandurineteststoconfirmtheprovisionaldiagnosisof
PSGN.
Teststobeorderedshouldinclude:
Ureaandelectrolytes,creatinine,inflammatorymarkersC3,C4,ASOT,DNAaseB
Urinemicroandculture,fullbloodexamination
Immediatemanagement
Admissiontohospital.
Strictfluidbalanceandrestrictedfluidintake
Testallurine-four-hourlybloodpressureandothervitalsigns.
Dailyweight.
Lowprotein,lowsalt/highcarbohydratediet.
Antihypertensivetreatment.
Penicillintherapymaybesuggestedbutisnotessential
Renalbiopsyisnotneededfordiagnosisatthisstage.
Futuremanagement
Monitorbloodpressureandrenalfunctionweekly/monthly/quarterlyasneededas
convalescenceprogresses.
Regularurinalysis(microscopichaematuriamaypersistforuptotwoyears).
Longtermprognosisisexcellentwithaverylowincidenceofsequelae;sopositiveand
sympatheticreassuranceisrequired.
KEYISSUES
DiagnosisofacutePSGN
Abilitytospecifyappropriateplanofinvestigations.
Developmentofcoherenttreatmentplan

CRITICALERROR
Failuretoadmittohospital

COMMENTARY
This scenario involves diagnosis, from clinical signs and appropriate investigations, and an
empathieexplanationoftreatment.Fromtheinformationgiven,thecandidateshouldbeableto
arriveatthecorrectdiagnosisandinvestigateandtreatappropriately.Failingtodosoputsthe
patient at risk. While classical poststreptococcal glomerulonephritis has become rare in many
parts of Australia, knowledge of the condition is important in considering the differential
diagnosisofthischild'ssymptoms.
077
414
Performance Guidelines



Condi t i on 077
Fever and sore throat in a 5-year-old boy

AIMS OF STATION
To assess the candidate's ability to diagnose and treat a child with tonsillitis most likely due to
Group A -haemolytic Streptococcus. Some investigations to confirm this are indicated.

EXAMINER INSTRUCTIONS
The examiner will advise the parent as follows:
You are the mother of a five-year-old boy, Peter, who has become unwell overnight with a very
sore throat and has difficulty swallowing food and drink. He most likely has an acute tonsillitis
which should be treated with penicillin. The candidate may suggest some basic investigations
to help confirm the diagnosis. You are worried that Peter has tonsillitis, and are concerned by
his high temperature.
Famil y history
Both parents are well. Father is an office-worker, mother is at home. Three-year-old sister at
kindergarten is well. Your son has had no previous antibiotic reactions.
Questions to ask unless already covered:
'How would he get this infection
9
'
'Is he likely to get it again?'
'What causes this infection
9
'
'I've heard that this sort of infection can damage your heart or your kidneys or something - is
that right?'
How long will he take to get better?'
'Is antibiotic therapy reguired?'
Examination findings to be given by examiner to candidate on request
A flushed child, tonsils acutely inflamed with follicular exudate, with moderately enlarged
and tender cervical lymph nodes on both sides.
The appearance of his oropharynx and tonsils are shown in the illustration (Figure 1). which
the examiner will show to the candidate.
There is no evidence of neck stiffness, no hepatosplenomegaly, rash or lymph-adenopathy
elsewhere.
Temperature is 40 C, blood pressure 110/70 mmHg, respiration rate 24/min, pulse rate
110/min
Tympanic membranes are normal on otoscopy.
Examination is otherwise normal.
077
Performance Guidelines
1. CONDITION 077. FIGUR
Acute tonsilitis
E
EXPECTATIONS OF CANDIDATE PERFORMANCE
The candidate should enquire as to the important clinical findings on examination allowing confident diagnosis of
acute tonsillitis, most likely bacterial. A throat swab could help confirm this prior to antibiotic treatment. The candidate
should be able to explain in simple terms the diagnosis and its associated complications in a manner that the parent
can understand; and should also arrange for further review in a few days to ensure the expected recovery is occurring
and if not, review and possibly seek other aetiologies.
Explanation of diagnosis
Acute tonsillitis, probably streptococcal. Reassure that with appropriate treatment this should resolve completely.
Immediate management
The candidate may wish to perform a throat swab for culture (appropriate but not obligatory). There is no need for any
other investigations at this stage. Check whether antibiotic reaction previously, and prescribe oral penicillin. Advise
the mother of need for frequent fluids. Prescribe analgesics.
Although the problem might be viral and settle without antibiotics, the majority opinion would be that penicillin therapy
is indicated because of the high likelihood of the diagnosis being streptococcal tonsillitis. If the candidate does not
recommend antibiotic therapy the mother should ask whether antibiotics are needed.
Antibiotic therapy would not only treat the streptococcal sore throat but would probably reduce the likelihood of
serious poststreptococcal complications.
Future management
Suggest review in few days or earlier if concerned and if the child has not responded as expected. If this is the case,
other aetiologies (e.g. infectious mononucleosis) should be sought. Stress that a full course of 10 days penicillin
treatment is required. Indicate that viral infection may cause similar features, as might acute infectious
mononucleosis.
415
416
077
PerformanceGuidelines



KEYISSUES
Appropriateexaminationinterpretation,withappropriatediagnosis.
Adequatetreatmentplan.
Appropriateexplanation.

CRITICALERRORS
Failuretoconsiderstreptococcaltonsillitisasthediagnosis.
Failuretodiscussfollowupandscreeningforotherconditionsifthereisnoinitial
improvement.

COMMENTARY
Thisscenarioassessestheabilityofthecandidatetocometoalogicalconclusionastothe
most likely diagnosis (acute bacterial tonsillitis) in this situation based on the information
provided and knowledge of the natural history of disease processes. The scenario tests
diagnosticacumenbyshowinghowseveralconditionscanbesafelyexcludedbecauseofthe
historyandthetimeframeandgivesscopetoconsideringotherdiagnosesiftheprovisional
diagnosisisnotconfirmed.
2-F: The Obstetric and Gynaecologic
Consultation

Roger J Pepperell
'Man endures pain as an undeserved punishment; woman accepts it as a natural heritage.'
Anonymous
Although in clinical practice obstetric and gynaecologic consultations may
involve a consideration of a complex set of symptoms and history which can
include relevant past history, medical history and social history, the clinical
scenarios used as part of the MCAT examination are much more focused
and restricted to fit in with time constraints. The scenarios reflect conditions
which should be able to be appropriately assessed and managed by a final
year medical student or a doctor working as an intern in a public hospital or in
community practice.
Some involve the candidate taking an appropriately focused history to enable
the diagnosis to be made. Because only eight minutes are allocated
for the assessment, and the history-taking will represent only a fraction of the
total time spent, the history-taking must concentrate on relevant issues and
not be generalised, verbose and largely irrelevant.
Some of the stations involve the candidate requesting the examination
findings they would look for if assessing such a patient to allow the examiner to assess whether the candidate knows
what examination findings are particularly relevant and important in assisting the candidate make the correct
diagnosis in this circumstance.
Although it would be unusual for
you to have to examine the
abdomen of a pregnant woman, or
perform a pelvic examination in the
actual clinical MCAT examination,
you clearly need to know how to do
such clinical examinations and may
well need to do such assessments
on models which have been
specifically designed and produced
for this purpose.
Where investigations are required to assist in making a diagnosis or starting treatment, the candidate is again
expected to show perspective rather than ordering a large number of irrelevant and inappropriate tests.
If the candidate needs to advise the patient on the initial management plan, this should be provided to the patient in lay
language, in terms she can readily understand, with perspective and with empathy and compassion. In obstetrics and
gynaecology, all of the options of management which might be appropriate need to be provided to the patient, to
enable her to decide which option she will accept, and ultimately to give the clinician informed consent to proceed with
the option chosen.
In clinical practice today, particularly in obstetrics and gynaecology, communication with the patient, and if appropriate
with her partner, is mandatory, and unless done in a manner which is acceptable to the patient, can result in the
candidate being reported to the relevant medical board or health complaints commission.
Where the clinician is not prepared, on religious grounds, to follow through a particular treatment which might be
appropriate, such as a pregnancy termination because the fetus has a lethal congenital abnormality, the clinician has
a responsibility to explain the options available to the patient, and has an obligation to offer to refer her to an
appropriate physician who would provide the treatment she has accepted as being most appropriate. Personal beliefs
should not restrict the matters discussed with the patient although they may affect what the clinician is actually
prepared to do in terms of actual management.
417
418
2-F
The Obstetric and Gynaecologic
Consultation


Clinicians must preserve a nonjudgmental and supportive approach in discussion and must not impose their own
religious or other nonmedical views on a concerned patient.
Although it would be unusual for candidates to have to examine the abdomen of a pregnant woman, or perform a
pelvic examination in the actual clinical MCAT examination, they clearly need to know how to do such clinical
examinations and may well need to do such assessments on models which have been specifically designed and
produced for this purpose.
The various scenarios cover aspects of the female reproductive system including normal development and
disorders of uterus, tubes, ovaries, vagina, fertility and contraception, hormonal influences, pregnancy,
labour, abortion, obstetrical toxaemia and haemorrhage, menopause, pelvic infection, vaginal discharge,
dyspareunia, haemostasis and bleeding disorders.

Roger J Pepperell
419
2-F
TheObstetricandGynaecologic
Consultation


2-FTheObstetricandGynaecologicConsultation
CandidateInformationandTasks
MCAT078-082
78 Breechpresentationinlabourat38weeksina25-year-oldwoman
79 Vaginalbleedingina23-year-oldwoman
80 Cessationofperiodsina30-year-oldwomanontheoralcontraceptivepill(OCP)
81 PositivecultureforGroupBstreptococci(GBS)at36weeksofgestationina26-year-old
woman
82 Vaginalbleedingafter8weeksamenorrhoeainawomanwithpreviousirregularcycles
420
078-079
Candidate Information and Tasks



Condition 078
Breech presentation in labour at 38 weeks in a 25-year-old woman

CANDIDATE INFORMATION AND TASKS
You are working in the Emergency Department of a general hospital. This patient is 3 25-year-old
woman in her second pregnancy, at 38 weeks of gestation and is in early labour. Vaginal
examination unexpectedly reveals a breech presentation: the legs of the fetus are apparently
both extended. The cervical dilatation is 4 cm. The previous pregnancy resulted in a normal
cephalic vaginal delivery of a 4 kg baby at 41 weeks of gestation. The current pregnancy has
been uneventful to date and the fundal height is 38 cm above the pubic symphysis at the time of
admission in labour at 38 weeks.

YOUR TASK IS TO:
Advise the patient of the possibilities in regard to subsequent management and the pros
and cons of these.

You may take any further relevant history you require, but do this briefly as the essential features
have been provided above.
The Performance Guidelines for Condition 078 can be found on page 424

Condition 079
Vaginal bleeding in a 23-year-old woman

CANDIDATE INFORMATION AND TASKS
You are working in a hospital Emergency Department. Your next patient is a 23-year-old
nuliiparous woman who has been trying to conceive, and believes she is pregnant. She has
developed vaginal bleeding after eight weeks of amenorrhoea.

YOUR TASKS ARE TO:
Take any further relevant history you require.
Ask the examiner about the findings you would look for on general and gynaecological
examination and the results of any tests you would expect to be available at the time you
are seeing the patient.
Advise the patient of the probable diagnosis and subsequent management you would
institute, including any further investigations you would arrange.

The Performance Guidelines for Condition 079 can be found on page 427
421
080-081
Candidate Information and
Tasks



Condition 080
Cessationofperiodsina30-year-oldwomanontheoral
contraceptivepill(OCP)

CANDIDATEINFORMATIONANDTASKS
Your patient is a 30-year-old woman who is taking the oral contraceptive pill (OCP). She has
cometoseeyouinageneralpracticebecauseshedidnothaveaperiodfollowingthelasttwo
coursesofpills

YOURTASKSARETO:
Takeafurtherfocusedhistory.
Asktheexamineraboutthefindingsyouwishtoelicitongeneralandgynaecological
examination.
Advisethepatientofthediagnosisandsubsequentmanagement(includingany
investigationsyouwouldarrange).

ThePerformanceGuidelinesforCondition080canbefoundonpage430

Condition 081
PositivecultureforGroupBstreptococci(GBS)at36weeksofgestationina
26-year-oldwoman
CANDIDATEINFORMATIONANDTASKS
Your next patient is a 26-year-old woman who is now at 37 weeks of gestation in her first
pregnancy. You have been looking after her pregnancy in a shared care arrangement in a
generalpracticesetting.Allhasbeennormal,andat36weeksyouorderedavaginalandrectal
swab for Group B streptococcal (GBS) testing. This test has shown GBS organisms were
detectedinthelowervagina.Shehasreturnedtoreceivetheresultsandanyimplicationsifthe
testispositive.
YOURTASKSARETO:
AdvisethepatientoftheresultsoftheGBStest.
Adviseheraboutthesubsequentmanagementyouwouldadvise
There is no need for you to take any further history or to request any examination findings or
investigationresultsfromtheexaminer
ThePerformanceGuidelinesforCondition081canbefoundonpage432
422
082
Candidate Information and Tasks



Condition 082
Vaginal bleeding after 8 weeks amenorrhoea in a woman with
previous irregular cycles
CANDIDATE INFORMATION AND TASKS
Your patient is a 25-year-old married nulliparous woman who presents to you in a general practice
with vaginal bleeding after eight weeks of amenorrhoea. Her cycles are often irregular with the
periods occurring at intervals of 4-8 weeks.

YOUR TASKS ARE TO:

Take a further focused history.
Ask the examiner about the findings you wish to elicit on general and gynae- |
cological/obstetric examination.
Advise the patient of the probable diagnosis and subsequent management, including any
investigations you would arrange.

The Performance Guidelines for Condition 082 can be found on page 434
423
2-F
TheObstetricandGynaecologicConsultation

2-FTheObstetricandGynaecologicConsultation
PerformanceGuidelines
MCAT078-082
78 Breechpresentationinlabourat38weeksina25-year-oldwoman
79 Vaginalbleedingina23-year-oldwoman
80 Cessationofperiodsina30-year-oldwomanontheoralcontraceptivepill(OCP)
81 PositivecultureforGroupBstreptococci(GBS)at36weeksofgestationina26-year-old
woman
82 Vaginalbleedingafter8weeksamenorrhoeainawomanwithpreviousirregularcycles
078
424
PerformanceGuidelines

Condition 078
Breech presentation in labour at 38 weeks in a 25-year-old woman

AIMS OF STATION
To assess the candidate's ability to appropriately advise a patient concerning the advantages
and disadvantages of vaginal breech delivery or Caesarean section when the fetus is found to
be presenting by the breech in early labour at 38 weeks of gestation.

EXAMINER INSTRUCTIONS
The examiner will have instructed the patient as follows:
Breech presentation has not previously been diagnosed and all your tests and progress have
been normal, as was your previous pregnancy.
Opening statement:
'So it is a breech, doctor. Does that cause any problems?'
List of appropriate answers to questions by the candidate:
Your desires in relation to mode of delivery are as follows:
~You would prefer vaginal delivery if possible but would accept Caesarean section if this is
recommended as necessary or very much more preferable.
~You had no problems with delivery of the first baby at 41 weeks of gestation. Forceps
delivery was not required.
~Only a very small episiotomy was necessary, despite the baby weighing 4 kg.
~Your antenatal course in this pregnancy has been normal.
~There is no family history of diabetes or other problems.
Questions to ask if not already covered:
'What are my options regarding delivery?'
'Are there any significant risks to the baby or me if I have my baby normally?'
What are the potential problems to the baby of vaginal delivery versus Caesarean section?'
Examination findings
The candidate may ask for specific components of the examination, but no additional findings in
addition to those outlined in the candidate's instructions need to be given.
Investigation results
None is to be provided or available.
078
425
Performance
Guidelines

EXPECTATIONSOFCANDIDATEPERFORMANCE
Advicetopatient(thecandidateshouldconveythesubstanceofwhatfollowsto
thepatient):
Diagnosis - breech presentation in early labour. The type of breech presentation (extended
legs)isafavourableone.andassheiskeentoavoidaCaesareansectiondelivery,anattempt
atvaginalbreechdeliverywouldbeappropriate.
X-raypelvimetryisunnecessaryinviewofthesizeofherpreviousbaby(4000g)whichwas
born at 41 weeks of gestation. Although second babies are likely to be bigger than the first
one. the current baby is being delivered three weeks earlier than the preceding one which
meansitshouldbesmallerthanthepreviouschild
Cardiotocography(CTG)monitoringisnecessaryinassociationwithbreechpresentationas
thereisanincreasedriskofcordprolapseassociatedwiththisabnormalpresentation.Vaginal
examination as soon as the membranes rupture, to exclude cord prolapse and confirm the
typeofbreechpresentation,isalsomandatory.
Assheisinlabour,ultrasoundexaminationwillprobablybedifficulttoarrangeurgentlyHad
thebreechpresentationbeendiagnosedpriortolabour,ultrasoundwouldhavebeenofvalue
tocheckfetalsize,typeofbreechpresentation,andwhetherthefetalneckwasextended.
Caesarean section would be indicated if there was slow progress of labour, or if breech
extractionwasconsideredrequiredtoeffectdeliverybecauseoffetaldistressorinadequate
progress,orasignificantCTGabnormalityoccurredinthefirststageoflabour.
Asuccessfuloutcomeoflabourcanbeanticipatedwiththefindingswhichareevidentinthis
patient.Howeverabout3-5%ofpatientsdohaveproblemsduringthelatterstagesofdelivery
duetodifficultydeliveringthelegs,armsorhead.

KEYISSUES
Abilityofthecandidatetoadviseandcounselapatientofthecurrentoptionsinregardto
breechdeliverybyvaginalorCaesareandelivery.

CRITICALERRORS
Failuretoadviseoftheappropriaterisksofvaginalbreechdelivery
Recommendingthatexternalcephalicversionshouldbeattempteddespitethefactsheisin
labour.
Indicatingtothepatientthatvaginalbreechdeliveryisabsolutelycontraindicateddespiteher
desires.

COMMENTARY
Approximately4%of allbabiespresentbythebreech,andvaginaldelivery issafeinselected
patients.Thisparticularlyapplieswherethebabyisofnormalsize(between2.5and4.0kg);the
breechpresentationisacompletebreechorabreechwithextendedlegs:thefetalneckisnot
extended;wherelabouroccursspontaneouslyandprogressesattheappropriaterate;andwhere
thepelvicdimensionsarenormal.
426
078
Performance Guidelines



There are risks to the baby of vaginal delivery however, and the risks are higher than when the baby is delivered by
Caesarean section. These aspects were well reported in the Term Breech Trial published in 2000.
In this patient it would be appropriate to recommend a trial of vaginal delivery with appropriate monitoring.
Caesarean section recommendation at this stage would be appropriate depending on the patient's responses and
concerns after discussion.
The recent trial of vaginal breech delivery as compared to Caesarean section delivery clearly showed the risk of
vaginal delivery was higher than that associated with delivery by Caesarean section. Despite this, and the general
recommendation that all babies presenting by the breech should be delivered by Caesarean section, some patients
will still prefer a vaginal delivery.
If the candidate suggests external cephalic version should be attempted at this time, when she is clearly in labour,
this is contraindicated and clearly WRONG.
Common problems likely with candidate performance are:
Failure to advise of the actual care in labour which would be given.
Failure to advise that the risk of vaginal breech delivery is higher than that of delivery by Caesarean section with
the risk being approximately doubled.
427
079
Performance Guidelines



Condition 079
Vaginal bleeding in a 23-year-old woman

AIMS OF STATION
To determine the ability of the candidate to assess and appropriately manage a patient in early
pregnancy with eight weeks of amenorrhoea which was then followed by vaginal bleeding.

EXAMINER INSTRUCTIONS
The examiner will have instructed the patient to repl y to questions from the candidate as
follows:
Your periods are usually regular and normal and your last menstrual period was eight weeks
ago. You think and hope that you are pregnant; this is your first pregnancy. You checked via a
chemist two weeks ago, and had a urine pregnancy test which was positive.
You and your husband have been trying to conceive since stopping the pill.
You ceased the oral contraceptive pill (OCP) five months ago, and have had regular menstrual
cycles since then until recent amenorrhoea.
No bleeding since last menstrual period until yesterday. Light loss then. Total loss is much
less than a normal period. Bleeding seems now to have stopped.
You have minimal pelvic discomfort.
Breasts sore and nipples tender for last six weeks no reduction in these symptoms recently.
Blood group O Rh negative.
Questions to ask if not already been covered:
'Will my baby be OK?'
'Can you give me something to make sure I don't lose this pregnancy?'
'What will happen if I miscarry?'

Physical examination findings to be gi ven to the candidate on request
General examination
Pulse 80/min and regular.
Blood pressure 120/80 mmHg, not distressed
Pelvic examination cervix closed and firm, no blood in vagina.
Uterus retroverted, enlarged to the size of an eight-week pregnancy.
Adnexae no mass or tenderness
Previous investigation results to be gi ven on request
Pregnancy test positive previously, confirmed on spot urine testing now.
Blood group O, Rhesus negative
079
428
Performance Guidelines



EXPECTATIONS OF CANDIDATE PERFORMANCE
Advice to Patient
The substance of what follows should be communicated to the patient in lay terms.
She needs an ultrasound of the pelvis to enable the pregnancy to be sited, to confirm the gestation, to check the
sac size, liquor volume, and the presence or absence of fetal heart activity. These findings would be expected to
confirm and define the diagnosis of threatened abortion (miscarriage). The candidate should ask for these
investigations to be done and should explain to the patient that if everything was normal, the pregnancy was in
the uterus, and the fetal heart activity was present, the diagnosis is a threatened miscarriage, with a good
prospect of a continuing viable pregnancy.
Other investigations required: checking the haemoglobin and check indirect Coombs as patient is Rh negative. If
indirect Coombs is negative, give anti-D if abortion occurs. (Anti-D is often not available in Australia for a
threatened abortion).
Immediate Management
Treat conservatively and rest. No specific therapy is effective in improving the pregnancy outcome.
Chance of successful outcome of pregnancy prior to performance of the ultrasound the chance of success
was only 50%. Providing the ultrasound examination is perfectly normal, the chance of a successful pregnancy is
somewhere between 90%and 95%.

KEY ISSUES
Ability to define the diagnoses needing to be considered in the presence of eight weeks of amenorrhoea.
Ability to appropriately investigate a woman with these symptoms.

CRITICAL ERRORS
Failure to confirm pregnancy by pregnancy testing
Failure to arrange ultrasound to check site and viability of pregnancy.
Failure to consider use of anti-D in view of Rhesus negative state.
429
079
Performance Guidelines



COMMENTARY
In all cases of bleeding in early pregnancy, the most critical examination findings are those of uterine size, the state of
the cervix and the presence or absence of pelvic tenderness. The reliance upon ultrasound examination alone is
inappropriate. Ultrasound in this case will enable the viability of the pregnancy to be assessed, thus enabling the
patient to be reassured with a degree of confidence. The other aspect of this case is the fact that the patient's blood
group is O Rhesus negative. Common problems likely with candidate performance are:
When taking the history, not being focused enough to the actual problem, but asking for information such as
irrelevant past history, social history etc. This just takes time to do and reduces the time available for the remaining
tasks.
Failure to examine the patient appropriately (cervical closure or opening status was not requested, uterine size was
not asked for. possible signs suggesting an ectopic pregnancy were not asked for).
Failure of candidate to advise the patient of the likely prognosis for this pregnancy, following performance of the
ultrasound examination and assuming confirmation of normal findings.
080
430
Performance Guidelines



Condition 080
Cessation of periods in a 30-year-old woman on the oral
contracepti ve pill (OCP)

AIMS OF STATION
To assess the candidate's ability to take an appropriate history and to assess findings to define
the cause of amenorrhoea developing while on the OCP, and then to appropriately counsel the
patient.

EXAMINER INSTRUCTIONS
The examiner will have instructed the patient to repl y to questions from the candidate as
follows:
You were married six years ago. You have been on the OCP since then. You will probably
want to conceive in about two years time.
Your menarche was at 14 years of age. When not on the OCP, your cycles were 28 days long
and you bled for three days, but lightly.
You have been on Microgynon 30 for six years. Initially the periods were normal, but they
have become lighter and lighter. About six months ago the periods were only lasting for one
day. Since then they have been shorter and lighter, and no period occurred at all at the end of
the last two packs of pills. You have not missed any pills in the last six months (give the
information of progressive reduction in menstrual loss only in response to specific request
from the doctor).
No problems with sexual activity, usually active 3-4 times per week.
No recent nausea, vomiting, breast enlargement, or nipple discomfort (so nothing to suggest
a pregnancy).
No relevant past, medical, surgical, family or social history You have never had a curettage.
Questions to ask unless already covered:
'Does it matter if I don't have a period at the end of the pill month?'
'Will I be able to have a baby when I want to do so?'

Examination findings gi ven to the candidate on request:
General and abdominal examination:
normal
Speculum examination: normal
Pelvic vaginal examination: uterus retroverted and of normal size and
mobility
Adnexae: normal

431
080
Performance Guidelines



EXPECTATIONS OF CANDIDATE PERFORMANCE
The candidate should convey the substance of what follows to the patient:
The diagnosis is endometrial atrophy due to the progestogen component of the OCP
(Microgynon 30
M
).
There is no real problem with the progestogen-induced secondary amenorrhoea except for the
anxiety it produces in the patient about whether she is pregnant. When the oral contraceptive
pill is ceased, all will return to normal, including her fertility.
A pregnancy is most unlikely but a p-hCG estimation should be done to confirm this for the
patient.
If she is really worried about the amenorrhoea, the pill could be changed to either a
higher-oestrogen-containing pill (such as Microgynon 50), or a triphasic pill (Triquilar), and
the menstrual loss may increase. The other option would be for her to have a break from the
oral contraceptive pill and use some other method of contraception.
If the assessment of oestradiol, FSH, LH or prolactin levels is suggested, this would
suggest little or no insight into the cause of the amenorrhoea or the effect of the OCP on
these hormone test results.

KEY ISSUES
Ability to diagnose the cause of amenorrhoea when on the OCP.
Ability to counsel the patient appropriately.

CRITICAL ERROR
Failure to perform a pregnancy test (/J -hCG) to exclude the unlikely possibility of a pregnancy
occurring whilst taking the OCP.

COMMENTARY
The reduction in the amount of of withdrawal bleeding whilst a patient is on the oral contraceptive
pill is not uncommon. The cause is due to a progressive endometrial atrophy (progestogen
-induced) over the period of time the patient is taking the pill. The key to the situation is generally
the history of gradual reduction of menstrual flow over a period of time prior to the complete
cessation of withdrawal bleeding. Whilst the likelihood of pregnancy is very low, a pregnancy test
is appropriate to reassure the patient as conception is possible whilst a patient is taking the
oral contraceptive pill. Common problems likely with candidate performance are:
Inadequate history concerning the progressive reduction in the menstrual loss whilst on the
OCP.
Inadequate advice concerning the natural history of this symptom after cessation of the OCP.
432
081
Performance Guidelines



Condi t i on 081
Positive culture for Group B streptococci (GBS) at 36 weeks of gestation in a
26-year-old woman

AIMS OF STATION
To assess the ability of the candidate to counsel a patient concerning the significance of the finding of vaginal GBS
organisms late in pregnancy and the subsequent management required.
EXAMINER INSTRUCTIONS
The examiner will have instructed the patient as follows:
You will be advised of the results of the recent GBS screening and what the doctor advises in regard to treatment. You
have no history of allergies to penicillin.
Questions to ask unless already covered:
'What are these GBS organisms?'
'Why are these bugs there?'
'Will these bugs do any harm?'
'Why don't you just give me antibiotics now and get rid of them?'
EXPECTATIONS OF CANDIDATE PERFORMANCE
The candidate should convey the substance of what follows to the patient:
The significance of GBS organisms in the vagina is:
~ the organism will not usually produce a problem for the mother, and 10-15%of
pregnant women may carry this organism, at this stage of the pregnancy;
~ the risk of the baby being colonised is 40-50%, if mother is GBS positive, and
delivers vaginally, and is not given antibiotics in labour;
~ the risk of the baby becoming 'infected' under the above circumstances is 1%but
this infection can be very severe;
~ by the time the neonatal diagnosis is able to be made clinically in the infant, it may
be too late to treat effectively and mortality is high; and
~ the important principle is therefore to prevent the baby getting infected.
GBS cannot be eradicated from the vagina with certainty by treating with penicillin or amoxycillin during
pregnancy.
Having found that she is GBS positive, it becomes important to treat the mother in labour, to prevent fetal
infection. Although a low risk situation, consensus best practice is to treat all GBS positive patients during labour.
Treatment with parenteral penicillin should be commenced in labour or if membranes rupture prior to onset of
labour. The antibiotic crosses the placenta and protects the baby. It is extremely unlikely that the baby will become
infected under such a regimen.
Some obstetric units only give antibiotics to 'high risk' patients in labour, such as those in premature labour, those
with premature rupture of the membranes, or where there is a maternal fever. Candidates should be aware that
such a management protocol does however put about 0.5%of babies at significant risk where the mother is GBS
positive.
081
433
Performance
Guidelines
Ifallergictopenicillin,useerythromycin.
Parenteralpenicillintothebabyafterbirthisoptionalunlesssignsofinfectionensueorinhigh
risksituations(suchasprolongedrupturedmembranes).

KEYISSUES
Definingthemanagementplan.
Counsellingthepatientastowhyantibiotictreatmentinlabourisrecommended.

CRITICALERRORS
FailuretoadvisepatientofthesignificanceofGBSorganismstomotherandherbaby.
Failuretoadviseantibiotictreatmentofthepregnantwomanifthemembranesrupture,or
whenlabourcommences,toprotectthefetusfromtheriskofsevereinfection.

COMMENTARY
Thiscaseillustratesthenowalmostuniversalpracticeofroutinelyscreeningallpregnantwomen
at34-36weeksgestationforthepresenceofGBScolonisationofthevagina.Itisimportantto
know that approximately 10-15% of pregnant women will be colonised with Group B
streptococcusorganismsatthisstage.Thecriticalaspectofthemanagementofthissituationis
that antibiotics are given to the mother only when she presents in labour and not at any time
duringthepregnancywhenthecolonisationisdiscovered.Itisimportanttocounselthemother
thatcolonisationwiththisorganismposestittle,ifany,risktothemotherbutmayaffectthebaby.
ItisimportanttostresstheserioussignificanceofGroupBstreptococcalinfectionintheneonate.
Commonproblemslikelywithcandidateperformanceare:
Recommendingadministrationofantibioticsduringpregnancy(antenatally)andassuming
thatsuchtreatmentwoulderadicatetheGBSorganism.
Believingthattreatmentofaninfectedbabyissoeffective,thatprophylacticantibiotictherapy
tothemotherinlabourisunnecessary.
Believingantibiotictreatmentofthemotherisnecessarynowbecauseoftheadverseeffects
GBSorganismswillhaveonher.
434
082
PerformanceGuidelines



Condition 082
Vaginal bleeding after 8 weeks amenorrhoea in a woman with
previousirregularcycles

AIMSOFSTATION
Toassessthecandidate'sabilitytoappreciatethesignificanceofvaginalbleedinginawomanwithirregular
cycleswhereearlypregnancyisapossibility.

EXAMINERINSTRUCTIONS
Theexaminerwillhaveinstructedthepatienttoreplytoquestionsfromthecandidateas
follows:
Youarea25-year-oldmarriedwomanwithoutprevioussignificantillness.
Yourlastmenstrualperiodwaseightweeksagoandwasnormal.Yourperiodsareoftenirregularwith
cyclesvaryingbetweenfourandeightweeksduration.
Youdonotusuallyidentifymidcyclemucustorecognisethetimeofovulation.Youhavenoticedsome
breastdiscomfortandnausearecently.
Thecurrentbleedingisminimalandbrightincolour.Itcommencedyesterdayspontaneouslyandislike
daytwooftheperiod.Notissuehasbeenpassed.Thebleedingwasnotrelatedtoanysexualactivity.
Noabdominalorpelvicpainhasbeenassociatedwiththebleeding.
You use condoms for contraception. You would not mind if you were pregnant although you were not
planningtobecomepregnantforanothercoupleofyears.Nopreviouspregnancies.
Nopastmedicalorsurgicalhistoryofrelevance.Nomedications.
LastPapsmearwassixmonthsagoandwasnormal.Youhaveneverhadanabnormalsmeartest.
YourbloodgroupisORhpositive.
Questionstoaskunlessalreadycovered:
Doyouthi nkI' mpregnant?'
' IfIampregnant,whyamIbl eedi ng?Wi l l thebabybeOK?'
Isthereanytreatmenttostopthebl eedi ng?'

Examinationfindingstobegiventocandidateonrequest
Patientlookswellbutisoverweight(90kg)
Bloodpressure 120/80mmHg
Pulse 70/min
Abdominalexamination nomassorviscuspalpable,notenderness
Speculum cervixclosedandnormal;someminimal
bloodloss
Pelvicvaginalexamination uterusisnotobviouslyenlarged,andis
retroverted
Adnexae normal,notenderness
082
435
Performance
Guidelines



EXPECTATIONSOFCANDIDATEPERFORMANCE
The history needs to define the normal cycle regularity and length, enquire about pain, and
enquireaboutsymptomssuggestiveofpregnancy.
Thecandidateshouldadvisealongthefollowinglines:
Thediagnosisisunclearfromthehistoryandexamination.Investigationswillneedtobedone
to confirm or exclude pregnancy and then define whether the pregnancy, if present, is
progressingsatisfactorily.
Anappropriateplanofinvestigationswouldbe:
-hCG tocheckifpregnant.
If-hCGisnegative,thediagnosisisjustalateperiod,thereforeobserve.Ifperiodsremain
irregular,hormonalteststoseeiffertilitytreatmentisrequiredmaysubsequentlyneedtobe
considered(suchasFSH,LH,PRL,TFTs).
If-hCGispositive,check-hCGleveltoassessusefulnessofultrasoundexamination.
If-hCGispositiveandgreaterthan1000U/L,sheneedsanultrasoundtocheckthesiteand
normalityofthepregnancy,andthegestationandduedate.
Whenalloftheseresultsareknownitwillbenecessarytoreviewher.Theprognosisregarding
thepregnancycanbediscussedwhenitisknownwhattheresultsare.
If she is pregnant, the diagnosis is probably a threatened miscarriage, and no hormonal
therapyislikelytobeofvalue.

KEYISSUES
Abilitytoevaluateapatientwithbleedingafteramenorrhoea.
Abilitytoconfirmorexcludepregnancyasacause.

CRITICALERRORS
Failuretoconsidernon-pregnancyaswellaspregnancycauses.
Failuretoarrangeultrasoundifpregnantand-hCGisgreaterthan1000U/L.

COMMENTARY
Thisisasituationwherebleedingoccurssomeeightweeksafterapreviousperiodbutwherethe
patientoftenhasanirregularcyclewhenamenorrhoeamaylastuptoeightweeks.Itistherefore
importanttodifferentiatewhetherthiswomancouldbepregnant,orwhethershesimplyishaving
oneofherlonger,irregularmenstrualcycles.Therefore,symptomssuggestingpregnancy,and
testsforpregnancy,mustbediscussedinthemanagementofthiscase.Itisalsoimportantto
remember that where pregnancy is proven not to exist, further investigations for the irregular
menstrualcyclesshouldbeconsidered.
Commonproblemslikelywithcandidateperformanceare:
Failuretotakeanadequatehistorytodefinethepreviousmenstrualcyclefrequencyandto
checkforsymptomsofpregnancy.
Failuretodescribeappropriatemanagementandinvestigativeplans.
Failuretoadviseappropriateendocrinetestsifsheisfoundtobenotpregnantandthe
irregularcyclespersist.
436
2-G: The Psychiatric Consultation

Frank P Hume
'The care of the human mind is the most noble branch of medicine.'
Aloysius Sieffert (c. 1858)
A psychiatric assessment is a structured clinical conversation, complemented by observation and mental
state examination and supplemented by a physical examination and the interview of other informants when
appropriate. After the initial interview, the clinician should be able to establish whether the individual has a
mental health problem or not, the nature of the problem and a plan for the most suitable treatment. A
thorough initial assessment may take an hour or more to complete, but when time is short it may be
necessary to focus on the immediate problems at first and schedule a longer followup appointment to round
off the evaluation. In the context of the AMC assessment, with the time constraints imposed, the tasks are
split and focused to allow completion in the time period allowed.
For trust and rapport to develop, the clinician must display tact, empathy and genuine respect for the
individual's dignity throughout the interview. A private setting is crucial with comfortable seating and
ambience and freedom from interruptions. Confidentiality is central, given the personal and intimate nature
of the material to be talked about, but it is not absolute when the safety and interests of the patient or others
are at issue, or in medicolegal consultations.
Comprehensive and contemporaneous case notes are essential. Whenever possible, notes should be taken
during the interview rather than relying on recall afterwards. However note-taking should be delayed at the
outset until the patient feels that he or she has the clinician's attention. If the patient is highly anxious,
agitated, hostile or paranoid it maybe sensible to defer note-taking until after the interview and limit the
amount of factual information at the first interview.
Clinicians should begin by welcoming patients by name, introducing themselves if unknown, greeting
companions if the patient is accompanied and explaining how long they may have to wait and whether they
will be interviewed. It is usual to interview the patient alone first and other informants afterwards, with the
patient's consent. Let the patient know from the outset how long the interview is likely to take and that you will
be taking notes at some stage (which are confidential)
Begin with basic census data: contact details, education, occupation and languages spoken. If the interview
is to be conducted in a language other than English, then a trained health service interpreter should be
used rather than an accompanying relative or friend, depending on the sensitivity and intimacy of the
information to be gathered. Experienced interpreters will repeat patient's replies word for word, even if they
are obviously delusional or thought-disordered, whereas well-meaning relatives may paraphrase or
substitute replies to compensate for confused or disordered responses. When using an interpreter, direct
your attention and your enquiries to the patient and not the interpreter.
The interview should commence with the history of the presenting complaint by asking an open-ended
question such as 'please tell me about your problems in your own words'. The patient should then be allowed
to talk spontaneously and without interruption for several minutes, with the clinician maintaining appropriate
eye contact, paying attention to the factual content whilst simultaneously monitoring the patient's verbal and
nonverbal behaviour. Encouraging the patients to 'go on' or 'tell me more are simple strategies to put them at
ease, as are nodding, leaning forwards, expressing concern or repeating key
437
2-G
ThePsychiatricConsultation

phrases,forexample,' soyour sl eeppr obl emhasbeenget t i ng wor se?' Avoidusing


specialised language: for example, anorexia, insomnia, anhedonia. Comments which make
patientsrealisethattheyarebeinglistenedtoandunderstoodwillincreasetheirconfidenceand
deepenrapport.
Astheinterviewunfolds,thenmoredirectivequestionsaimedatclarifyingsymptomsandtheir
evolutionareused,askingformorespecificexamplesofsymptomsorexperiencesInterviewing
isanactiveanddynamicprocess:initialhypothesesorroughideasaremodifiedcontinuouslyas
more information is collected. When there is time pressure or urgency to make treatment or
admission decisions, then the clinician may need to be more active or directive from the
beginningandusemoreclosedquestions(requiringjust'yes'or'no'answers)
Goodcliniciansshouldbeableto:
putananxiouspatientatease;
gainsufficienttrusttoencourageanunwillingorsuspiciouspatienttodiscussrelevantissues
bydisplayingtact,patienceandencouragement:
becomfortable,tolerantandempathiewhenapatientbecomestearfulduringtheinterview;
knowhowtosetlimitsifpatientsbecomeangry,hostileorabusive;
recognise and respect patient-clinician boundaries especially with dependent, disinhibited,
overfamiliaroradulatorypatients;
politely interject and refocus garrulous patients, explaining that because of time restraints it
maybenecessarytobreakintotheirflowofconversationfromtimetotimetoconcentrateon
thepointsthatareimportantforplanningtreatment:
rapidly identify patients who are demented, disorganised, disorientated, intoxicated, grossly
psychoticordysphasicandforwhomotherinformantswillbeimperative;and
becomeawareofandmonitortheirowncountertransferenceresponsestoparticularpatients.
By being aware of your own prejudices, weaknesses, blind spots and personal vulnerabilities,
andrecognising whenpatientsarousestrongfeelingsof anger,boredomsexualexcitementor
' r escue' fantasiesinyou,youareaccordinglylesslikelytoreactinappropriatelytothem
Apsychiatricassessmentdiffersfromothermedicalinterviewsinthatmoreattentionneedstobe
paid to the patient's psychological and social influences. Accordingly, patients' cultural and
spiritual backgrounds, formative influences, important relationships, significant life events and
theirreactionstothem;theirattitudes,valuesandbeliefsaboutthemselves,otherpeopleandthe
worldmayallbeexploredinthecourseofanassessment.
A vertical time line can be used to summarise key events in a person's life from what follows
(whichisnotcomprehensive).
Afterthehistoryofthepresentcomplainthasbeenestablished,thenthefamilyhistoryshouldbe
reviewedwithafamilytreeorgenogramdevelopedasfarbackasthegrandparents.Thequality
of the relationship of each family member with the patient and its stability over time, parental
occupations, family status and atmosphere, familial diseases and illnesses, family psychiatric
disorders(andtreatment)shouldallberecorded.
438
2-G
ThePsychiatricConsultation



The patient's personal history may begin with conception, but there may have already been
significantfamilyorparentaleventsthathaveoccurredthatwillinfluencetheirdevelopmentand
shapetheirdestiny(e.g.thepriordeathofsiblings,maternalrape,incest,immigration,domestic
violence,IVF,theHolocaust).
Maternalpregnancyandbirth:abnormalities,earlydevelopmentandnutrition,milestones.
Childhoodmilieu:separations,illnessesandhospitalisations:anxietytraitsandbehavioural
problems;educationandschooling,learningdifficulties,experienceofbullying,examination
successandageofleavingschool.
Adolescentpressures:puberty,peergroups,rebelliousness,drugandalcoholtaking,fantasy
life,psychosexualidentityanddysphoria,dietandexercise.
Occupational history in chronological order: training, competence, satisfaction, ambition,
experienceinarmedforcesorwar.
Marital history: length of courtship; age, occupation and personality of partner: quality and
stability of relationship, fidelity, previous relationships, divorce, separations, violence or
abuse.
Psychosexual development from childhood: sexual orientation, sexual dysfunction,
deviationsorfetishes,satisfaction,currentlibido,contraception.
Children: stillbirths, miscarriages, terminations, childhood deaths, age, sex, health and
tempertantrumsofthesurvivingchildren,attitudestochildrenandfurtherpregnancies.
Pastmedicalhistory:shouldincludesignificantchildhoodillnesseswhichmayhaveaffected
brain development or function; operations, hospitalisations, accidents; menstrual or
menopausal symptoms and chronic physical illnesses including fatigue, eating disorders,
obesity,neurologicaldisorders,headinjury.
Previous mental health: includes self-harm, mood disorder, anxiety symptoms, somatic
concerns, behaviour disorders and insomnia, with details about treatment or not; duration
andseverityofsymptomsandperiodsofhospitalisationandoutcome.
Use and abuse of drugs and alcohol: includes tobacco, caffeine, cannabis, stimulants,
sedatives, analgesics or narcotics and whether prescribed or not; and the chronological
historyofuseandthequantitiesinvolvedandpatternsofusageovertime,attemptstogiveup
andtheireffectsonhealth,relationships,workandfinances.Gamblinghistoryshouldalsobe
explored.
Forensic history: includes delinquency, arrests, convictions, imprisonment, probation and
anyhistoryofviolence,assaultorpropertydamageincludingfiresettingandarson.
Current life situation: involves a description of family, housing, social, work and financial
circumstances.Relationshipswithneighbours,peersandcolleagues,friendsandrelatives,
employers and superiors. Recent life stresses, bereavement, losses, disappointments,
promotionsandthepatient'sreactiontothem.
Personalityreferstothehabitualattitudes,behavioursandphysicalcharacteristicsthatdefinea
person as an individual to oneself and others. Psychiatric disorders may change a person's
personality,thusotherinformants,aswellasthepatient,canhelptodescribethefollowing:
Attitudestoothers:insocial,family,workandsexualrelationships.
Attitudestoself:e.g.vain,critical,self-conscious,realistic,self-critical.
Moralandreligiousattitudes:e.g.rigid,permissive,andrebellious.
Predominantmood:andwhetherstableorchanging.
2-G
The Psychiatric Consultation
Leisure acti vities: hobbies and interests; creative, physical, solo or team.
Fantasy life: includes daydreams and nightmares.
Resilience: in the face of adversity.
It is neither essential nor desirable to enquire exhaustively about all of the above with each
patient. Common sense and experience should inform the clinician about what is relevant to each
patient as a picture emerges during the interview.
In structured assessments at undergraduate level, scenarios must be selecti ve and
focused with clear aims and guidelines, to enable appropriate candidate assessment over
a brief eight minute doctor-patient encounter.
MENTAL STATE EXAMINATION
History-taking deals with the past while mental state
examination is a systematic review of the patient's present
symptoms and observed behaviour during the interview. It is a
cross-sectional view of the patient and is one of the essential
elements of psychiatric practice. The principles of the mental
state assessment can readily be incorporated into the
examination of any patient.
439
Recording mental state begins with:
Appearance and behaviour
A comprehensive, accurate and lifelike word-picture
of how the patient looks in terms of appearance, body size,
grooming, dress, posture, movement and facial
expressiveness. Behaviour refers to cooperation, body language and gestures, psychomotor
function and general activity and social relatedness during the interview. Orientation, mood,
anxiety, hallucinations and medication may all influence appearance and behaviour.
Mental state examination is a
systematic review of the
patient's present symptoms
and observed behaviour
during the interview. It is a
cross-sectional view of the
patient and is one of the
essential elements of
psychiatric practice. The
principles of the mental state
assessment can readil y be
incorporated into the
Speech
The rate, volume, quality, quantity and tone of speech are recorded. Dysarthria or dysphasia are
noted. The form of the patient's talk is considered rather than the content: spontaneity,
pressure, slowness, hesitancy, coherence, looseness, response latency unusual sentence
construction e.g. Yoda the J edi in Star Wars, neologisms (made up words), repetitions and
distractibility.
Mood and affect
Mood refers to a person's usual or longterm feeling state. Affect is a more short-term and
immediate feeling state and refers to what is observed by the clinician during the interview.
Mood is to affect as climate is to weather. Clues to mood assessment arise from the patient's
appearance, mobility, posture and behaviour. Patients could be asked
How do you feel in yourself?' or
'What is your mood like?' or
How about your spirits?'
To assess depression ask about unhappiness, sadness, tearfulness, pessimism about the
present, shame or guilt about the past and hopelessness about the future.
440
2-G
The Psychiatric Consultation



To assess suicidal ideation, begin with the first question, and then progress tactfully:
'Have you ever felt so bad/desperate that you have wanted to end it all?'
'Have you ever thought of harming/or actually harmed... yourself?'
'Do you feel unsafe at the moment?'
'Do you feel desperate enough to kill yourself?
' Do you think you are suicidal?
'Do you have a 'Plan B'?'
Asking about suicidal ideation or plans does not make patients suicidal, nor does it put the idea
into their heads. Patients who are depressed and suicidal may be alarmed and frightened by their
thoughts and are relieved that someone cares enough to ask what they may be thinking about.
Most suicidal patients do not want to kill themselves and find the thoughts repugnant, but may feel
it is the only solution to their anguish. Some genuinely suicidal patients may deny being suicidal
when asked, because they are determined to succeed and do not wish to be thwarted or prevented
from doing so.
To assess elation or hypomania ask
'Do you ever/often feel in unusually good spirits?'
'Do you ever/often feel on top of the world or full of energy?'
'Do you ever/often get racing thoughts?'
'Do you ever/often go on uncontrolled spending sprees that leave you in debt?
'Do you often feel unusually confident, inventive, fabulous or famous?'
Other mood states that may be specifically enquired about include anxiety, anger, irritability, envy,
suspiciousness and perplexity.
The range of mood should be described as normal, increased, labile, restricted or blunted. Also
note whether it is constant or stable.
Appropriateness of affect means that the current emotional expression matches what is being said
at the time.
Depersonalisation and derealisation experiences are difficult for patients to describe. They may
describe feeling unreal or detached, emotionless and numb, or as //'they are acting a part or being
like a robot. Alternatively they may describe their environment as colourless, lifeless, artificial or
cartoon-like. The feelings may vary from mild to severe, but are always seen as alien, unwanted
and unpleasant. All or any part of the body may be involved and the feelings may be intermittent or
persistent. They are usually accompanied by anxiety and/or depression.
Thought form
Abnormalities of thought can only be inferred from what patients say or write and may be
influenced by mood or psychosis. Depressed patients may have slowed speech with no rhythm or
cadence and only give limited or monosyllabic replies after a pause and with a limited range of
topics or themes. Manic patients speak very rapidly and their train of thought may shift repeatedly
(flight of ideas). They may be difficult to interrupt and their flight of ideas may be triggered by a pun
or a clang association where the sound of a word is rhymed with another word midsentence to
produce a different set of ideas.
Loosening of associations is the classic formal thought disorder of schizophrenia. A patient may
say a lot but it is impossible to grasp the meaning of what is being said. Attempts to clarify with
followup questions often only deepen the puzzle, because there is a loss of the normal clarity and
structure of thinking. Examples of disorganised speech should be recorded verbatim.
441
2-G
The Psychiatric Consultation



Circumstantiality infers a lengthy and garrulous response to a question, often to the extent that the patient forgets
what the question was. Obsessional patients may be anxious not to leave any doubt about their replies and qualify and
exhaustively explore every detail and nuance before they get to the point.
Tangentiality is an oblique or irrelevant response to a straightforward question.
Concrete thinking is a literal and restricted response to a basic question, for example, 'How
are you feeling today?' .................' With my fingers and toes, as usual! ' ........................... or
1
You 're the doctor.
you tell me!'
Thought content
A delusion is a false belief which is out of keeping with an individuals educational, cultural, religious and social
background which is held with extraordinary and unshakeable conviction and absolute certainty. Subjectively, it is
indistinguishable from a true belief and it is not influenced by rational argument or evidence to the contrary. A delusion
may arise spontaneously (out of the blue), or be a secondary response to a patient's mood hallucinatory experiences
or false memories.
Paranoid (persecutory) delusions are the most common. Grandiose, guilty, nihilistic, jealousy, religious,
hypochondriacal, sexual, control, and referential delusions also occur. Specific delusions about one's thoughts,
involving either thought insertion, withdrawal or broadcasting are pathognomonic of schizophrenia.
An overvalued idea is usually a solitary abnormal belief which dominates a patient's life and causes disturbed
functioning and suffering to the person or others. The patient's whole life may revolve around this one idea (e.g.
anorexia nervosa, body dysmorphic disorder, transsexualism), and cause irreparable harm to significant relationships
in the patient's life.
Obsessions are recurrent, intrusive, irrational thoughts, impulses or images that persist despite efforts to exclude or
resist them. They are recognised as being self-generated and nonsensical and usually deal with issues that the patient
finds disturbing or unpleasant (e.g. dirt, germs, violence, sex, illness or religion).
Perception
Perception is the process of integrating input either from the sense organs or from imagery and fantasy (which are
self-generated). It is influenced by mood: mania heightens perception, particularly of colours; anxiety may intensify
sound; depression mutes sound and dulls colour. Schizophrenia may affect olfaction and taste.
An illusion is the misinterpretation of a real stimulus and is more likely to occur when attention and concentration are
unfocused, or when anxiety is high.
Hallucinations are false perceptions in the absence of a stimulus. They have the full force and impact of a real
perception and occur spontaneously and cannot be controlled or terminated by self-will. Topatients, hallucinations are
normal sensory experiences. They may be simple: experience of bangs, rattles, whistles or flashes of light: or
complex: hearing voices, music, faces, animals or scenes.
Auditory hallucinations are characteristic of schizophrenia, but can also occur in alcoholism, amphetamine psychosis
and affective disorder. Voices in schizophrenia may be single or multiple; whisper or shout or speak in normal
conversational tone: give a running commentary on the patient's behaviour; argue with each other or appear to speak
or echo the patient's thoughts out loud. Usually the patient is referred to in the third person (he or she), but
occasionally commands and orders are given in the second person (you).
442
2-G
The Psychiatric Consultation



In organic disorders and depressive states when voices are heard they may adopt an abusive or critical tone and use
the second person or simple words or brief sentences.
Auditory hallucinations may occur in normal individuals under stress (e.g. sensory deprivation, bereavement) and
when falling asleep or on waking up. Blind or deaf people may hallucinate images or sounds.
Visual hallucinations usually signify organic illness and are uncommon in schizophrenia. Substance abuse
(hallucinogens, glue, alcohol), neurological disorders affecting the visual pathways or the occipital lobe,
postconcussional states, temporal lobe epilepsy and various forms of dementia may cause visual hallucinations.
Tactile, haptic (touch-related) or somatic hallucinations may occur in acute psychosis and often have a bizarre
complexity. Perceptions of heat, touch, water dripping (or blood or bodily fluids); visceral sensations of severe pain
caused by 'knives' or 'demons': or of formication (the sensation of ants or insects crawling on or under the skin), have
all been described. They may be more indicative of benzodiazepine, cocaine or alcohol withdrawal. Olfactory and
gustatory hallucinations are rare, usually occur together and although they may occur in schizophrenia or depressive
disorders, are more likely due to temporal lobe phenomena or neurological lesions of the olfactory pathways.
Cognition
Cognitive function should be assessed briefly in every patient and interpreted in relationship to age, education and
intelligence. Orientation, attention, concentration and short-term, recent and remote memory should be tested. If
impairment is suspected or revealed, then a more structured and objective review of cognitive functioning, such as the
Mini-Mental State Examination, should be performed.
Insight
Insight refers to the self-awareness of morbid experiences (symptoms), and their effect on personal functioning and
relationships. It also encompasses attitudes to assessment, mental disorder and treatment. Insight is not simply
present or absent, but depends on the degree to which patients acknowledge, or are aware of. phenomena that other
people (including the clinician) have drawn to their attention. Insight depends next on the degree to which patients
recognise that the phenomena are abnormal, or may have a psychological or psychiatric cause, and finally whether
patients are willing to have treatment or be hospitalised. In psychotic disorders, insight is usually absent or partial at
best.
Rapport
Rapport refers to the degree of relatedness between the patient and the clinician during the interview and is a
measure of the quality of the communication and trust achieved. Difficulty in establishing rapport may be symptomatic
of the patient's illness or the clinician's countertransference and lack of empathy. Rapport predicts whether a patient
will engage in and continue treatment.
Risk
Depending on the circumstances, the clinician's assessment of the patient's suicidalityand risk of violence to others
should be noted and then acted upon, if appropriate.
Reliability
Whenever there is any doubt about the veracity of the patient's account of symptoms or behaviour change and other
autobiographical details, other informants should be interviewed
443
2-G
The Psychiatric Consultation



to corroborate the history. This applies particularly to patients who are cognitively impaired, disorganised, intoxicated,
significantly thought-disordered or affectively distressed.
THE MINI-MENTAL STATE EXAMINATION (MMSE)
The MMSE was developed by Marshal Folstein and colleagues from the Johns Hopkins School of Medicine in
Baltimore. Since its publication in 1975 it has become the most commonly used instrument for bedside cognitive
function screening. Its purpose is not to make a diagnosis, but to indicate the presence of cognitive impairment due
to delirium, dementia or head injury. It is brief, easily administered, has high inter-rater reliability and may be used to
monitor progress or fluctuations in these disorders.
The thirty items in the MMSE measure orientation, attention, registration (immediate memory), recall (short-term
memory), language and visuo-spatial function
Performance in the MMSE is affected by age; years of education; socioeconomic status; ethnicity and whether English
is the first or second language. Better educated people may score well on the test despite having significant cognitive
impairment.
Scores of 25-30 out of 30 are considered normal; 18-24 indicate mild to moderate impairment and scores of 17 or less
indicate severe impairment. Once patients reach the more advanced stages of disease or dementia, their scores are
so low that progression cannot be assessed.
The MMSE was developed primarily to quantify cognitive functioning in elderly patients with delirium and
dementia and may not be reliable in every patient in all situations. It may be useless at detecting focal
cerebral lesions (aphasia, amnesia), right hemisphere disorders and frontal lobe deficits.
It is essential to consider the performance profile of the subsets of the test as well as the overall score. Patients with
Alzheimer disease may perform worst on recall, orientation and drawing, whereas subcortical dementias may primarily
affect attention and concentration.
The MMSE may be supplemented by specifically testing frontal lobe functioning via
verbal fluency tests: for example, naming as many words as possible in one minute starting with the letter F,then
the letter A, then S. Normal is 15 words per letter or 30 words in total for the three. Once again allowance must be
made for age, education, ethnicity and command of English. Alternatively ask for as many examples as possible
from semantic categories: such as animals, fruit or vegetables in one minute (10 or less abnormal); or list as
many items as possible that can be bought in a supermarket (15 or less is abnormal);
The interpretation of proverbs or sayings: for example, a bird in the hand is worth two in the bush', 'a stitch in
time saves nine]
similarities and differences: e.g. apple and banana, table and chair, child and dwarf, ice and glass: and
motor sequencing tests: either rapidly alternating hand movements or the Luria three-step hand movements:
fist-edge-palm; are all easily incorporated adjuncts to a more comprehensive screen of frontal lobe function.
Patients with frontal lobe dysfunction score poorly, perseverate and become disorganised under the time pressure
of these simple tests.
Frank P Hume
2-G
ThePsychiatricConsultation



MINI-MENTALSTATUSEXAMINATION(MMSE)
1
DateofExamination:Name:
DateofBirth: Sex:
Handedness:

Occupation(previous):
EducationalLevel:General
RemarksHearing:Vision:
Recordexactreplies.Enterthescoresoutofthemaximumshown.Nohalfmarksawarded.
1. Orientation
Whatisthe Year?
Season?
Date?
Month'?
Dayoftheweek
7

Wherearewe
7
State
City
Suburb
Hospital
Floor(ifathome,thestreetnumberandname)
2. Registration
Namethreeobjects,takingonesecondtosayeach,thenaskthepatienttorepeatthem.
e.g.Apple Table Coin
Repeatthemuntilthepatientlearnsallthree.
Askpatienttorememberthem
Numberoftrialsrequiredtolearntheanswers
3. Attention/Concentration
(a) Seri al Sevens(take7seriallyfrom100-93;86;79;stopafterfive
answers):deductonemarkpererror
Or
(b) SpellWORLDbackwards(e.g.dlrow=5,drolw=3)
4. Recall
AskthepatienttonamethethreeobjectsinQuestion2

444
1AdaptedfromFolsteinM..FolsteinS.andMcHughP..1975.J.PsychiatricResearch12pp189-198.

5. Language
(a) Askthepatienttonamethefollowingasyoupoint:' pen' ....' watch/2
(b) Havethepatientrepeat:' Noi ts.ands,orbuts/1
(c) Havethepatientfollowathreestagecommand:
1
Takethi spaperi nyourri ghthand.Fol dthepaperi nhal f./3
Putthepaperdowni nyourl ap' .
(d) Havethepatientreadandobeythefollowing:CLOSEYOUREYES /1
(e) Havethepatientwriteasentence(containingsubject,verb,object) /1
6. Construction
Askthepatienttocopythisdesign

/1

Score=/30
445
446
2-G
The Psychiatric Consultation



2-GThePsychiatricConsultation
CandidateInformationandTasks
MCAT083-089
83 Medicationchangesfora35-year-oldwomanwithchronicschizophrenia
84 Demandforurgenttreatmentfor'suddenhairloss'froma29-year-oldman
85 Poorworkperformanceina30-year-oldfemalepoliceofficer
86 Lifestylestressina45-year-oldman
87 Bingedrinkingina25-year-oldman
88 Nausea,headacheandfeeling'jittery'ina30-year-oldbankclerk
89 Collapseofa30-year-oldwomanonthewaytoacourtattendance
447
083
CandidateInformationandTasks



Condition 083
Medicationchangesfora35-year-oldwomanwithchronicschizophrenia

CANDIDATEINFORMATIONANDTASKS
You are working in a general practice. A longterm patient of the practice has attended for a
repeatprescriptionofthioridazine(Melleril).Thepatientisa35-year-oldwomanwhohasbeen
receiving thioridazine 200 mg daily for chronic schizophrenia over the past 15 years. Due to
recent adverse publicity and concern about the effects of longterm thioridazine on cardiac
conductivity(asreflectedinaprolongationofthecorrectedQTintervalontheECG),youwish
tochangehertoaneweratypicalantipsychotic.
Beforedoingso,youwillneedtodiscusswithhertherisksandbenefitsofhercurrenttreatment
andtherisksandbenefitsofthecommonlyavailableatypicalantipsychotics.

YOURTASKSARETO:
Explainyourconcernsaboutcontinuingonthioridazine.
Educatethepatientabouttherisksandbenefitsoftheneweratypical
antipsychotics.
Explainthesideeffectsofthemostcommonatypicalantipsychotics.
Respondappropriatelytothepatient'squestions.
Thereisnoneedforyoutotakeanyfurtherhistoryfromthepatient.
ThePerformanceGuidelinesforCondition083canbefoundonpage456
084
Candidate Information and Tasks



Condition 084
Demand for urgent treatment for 'sudden hair loss' from a
29-year-old man

CANDIDATE INFORMATION AND TASKS
You are the duty Hospital Medical Officer (HMO) in a busy city hospital clinic attached to the Emergency
Department. It is early evening.
The patient you are about to see is a neatly dressed, well-groomed 29-year-old man who has brought an envelope
containing some hair strands to the front desk, asking if they can be 'examined under a microscope'. He appeared
to be guite anxious and restless whilst waiting to be seen by you and the triage nurse has told you that he has
visited the toilet facilities for lengthy periods of time on several occasions.
The triage assessment states that he is worried that he is suddenly going bald because he has begun to lose his
hair. He has brought some of his hair to the hospital to be examined to find out what the problem is and have
treatment urgently' because he believes that his hair loss is affecting his 'prospects for promotion at work'.
He is single and lives at home with his parents. He has worked as a financial analyst in a merchant bank for the
past six years. He admits to not having any social life and is a nonsmoker and nondrinker.
He appears to have a normal full head of hair as illustrated below.
YOUR TASKS ARE TO:
Establish rapport.
Take a sensitive, focused and relevant history.
Reach a diagnostic conclusion, and discuss this with the patient.
Discuss management briefly with the examiner.
CONDITION084. FIGURES 1 AND 2.
The Performance Guidelines for Condition 084 can be found on page 459
448
449
085
CandidateInformationandTasks



Condition 085
Poorworkperformanceina30-year-oldfemalepoliceofficerCANDIDATE
INFORMATIONANDTASKS
Youareworkinginageneralpractice.Thepatientisa30-year-oldPoliceOfficerwhohasbeen
advisedtoseekmedicalhelpbythehumanservicesofficer(staffcounsellor),fortheStatePolice
Service.ThePoliceServicehasbecomeconcernedthatshedoesnotappeartobefunctioning
aswell intheworkplace.Shehasbeenhavingan increasing numberofsick
1
days, whichare
oftenonthefirstdayofanewsetofrosteredshifts.
YOURTASKSARETO:
Takeafocusedhistoryyouhavesixminutestodothis.
Informtheexaminerofthethreemostlikelydiagnoses.
Answerquestionsfromtheexamineraboutoneormoreofthesediagnoses

ThePerformanceGuidelinesforCondition085canbefoundonpage463
450
086
Candidate Information and Tasks



Condi t i on 086
Lifestyle stress in a 45-year-old man

CANDIDATE INFORMATION AND TASKS
You are working in a general practice. The patient has come to see you after having insurance medical
examinations at work. It was recommended that he see a general practitioner, to monitor his general health, and
that he may be 'just stressed'. He did not really think of being a 'stress type' before now, and has no symptoms
except for headaches towards the end of the day. Other family members attend your practice, but this patient has
not previously consulted you.
The patient has been told that cholesterol, blood sugar, and resting ECG are normal, but on two occasions in the
last month when tested by the insurance doctor, the BP reading was high (160/80 mmHg) but eventually settled to
normal levels. On those days, the patient had come from particularly difficult meetings.
The patient is upset by these findings, believing that he has always been in perfect health. The insurance doctor
said 'there was nothing to be concerned about really', but now he is worrying about having a heart attack and can't
get that out of mind over the past couple of nights. This worry has been reinforced by several episodes of stabbing
chest pain each lasting only a few seconds, unrelated to exertion. Last night he took a sleeping tablet, normally only
used on long plane trips, to get some sleep, and feels much better nowthe patient is now thinking he may have
been suffering from stress over the past couple of years.
You have obtained the information as listed below in the patient profile, and you have just completed examining the
patient, including performing an ECG. No abnormality has been found. Blood pressure today is 130/70 mmHg.
Patient profile

Marital status Married, wife an artist
Household Wife, three teenage children
Occupation Finance Manager
Smoking habits 3-5 cigarettes daily
Alcohol use two whiskies/sherries most nights
Drug sensitivities Not known
Family history Eldest of five siblings, all alive and well. Father died aged 65

of a heart attack; mother alive and well, although a worrier
Past medical history No serious illnesses or operations
Major continuing

health problems None known
Current medication None

451
086
CandidateInformationandTasks



YOURTASKSARETO:
Discusshishealthconditionandrelevantmatterswiththepatient.
Advisethepatientofyourdiagnosisandproposedmanagement.
Answeranyquestionsthepatientasksyou.

ThePerformanceGuidelinesforCondition086canbefoundonpage466
452
087
Candidate Information and Tasks



Condition 087
Binge drinking in a 25-year-old man
CANDIDATE INFORMATION AND TASKS

You are working in a general practice. This patient is well known to you. He is a 25-year-old sole parent of a
six-year-old girl the mother left soon after the girl's birth and there has been no contact since. The patient works
full time as a local delivery truck driver.
He regularly drinks heavily at the weekends.
He intermittently presents on Monday 'feeling seedy' requesting a certificate for the day off.
He is otherwise in good health, but has had frequent presentations for minor sporting injuries.
He is not taking any medication; smokes 10 cigarettes per day; there is no other drug history.
He does not have any history of psychiatric illness.
He is generally a good and caring parent he has no other regular help with child-care.
His relationship with his family is strained they blame him for his wife leaving.
His father was a violent, heavy drinker during the patient's childhood, and still drinks, but not to excess.
The patient came to the practice today for the removal of sutures to a small scalp laceration, well-healed, sustained
eight days ago in a fall at the pub after the football. He was briefly unconscious.
He was taken, intoxicated to the Emergency Department of the local hospital at 1:00 am eight days ago, the wound
was repaired and he was discharged several hours later. Your nurse has just removed the sutures. She has alerted
you to discuss the patient's drinking and parental responsibilities.
YOUR TASKS ARE TO:
Discuss with this patient his pattern of drinking and its harmful consequences.
Make appropriate recommendations for dealing with the problem.

The Performance Guidelines for Condition 087 can be found on page 470
453
088
CandidateInformationandTasks



Condition 088
Nausea,headacheandfeeling'jittery'ina30-year-oldbankclerk

CANDIDATEINFORMATIONANDTASKS
Your next patient is a 30-year-old bank clerk, who is consulting you in the Emergency
Department of a general hospital. She is complaining of severe nausea, headaches and the
'jitters'.
Sheattendedheretwodaysago.BriefnotesintheEmergencyDepartmentpatientrecordstate
thatshewasthencomplainingofbackpainwhichwasdiagnosed as'musclepain'.Thenotes
also state that she had been taking the selective-serotonin-release-inhibitor (SSRI) Prozac
(fluoxetine)20mgdailyfordepressionforthreeweeksontheadviceofherlocaldoctor,without
muchimprovement.AnalternativeSSRIZoloft(sertraline)100mgdailywasprescribedwhen
sheattendedtheEmergencyDepartment.

YOURTASKSARETO:
Takeafurtherfocusedhistoryrelatedtothissituation.
Asktheexaminerfortheappropriateexaminationfindingsyourequiretoassistin
diagnosis.
Informtheexaminerofyourdiagnosis.
Counselthepatientaboutthelikelycauseofhersymptoms,theirtreatment,andwhatyou
recommendwithregardtofurthermanagementofherdepression.

ThePerformanceGuidelinesforCondition088canbefoundonpage474
454
089
CandidateInformationandTasks



Condition 089
Collapseofa30-year-oldwomanonthewaytoacourtattendance

CANDIDATEINFORMATIONANDTASKS
This patient was brought to the Emergency Department complaining of a sudden inability to
walk.Shehadcollapsedonthewaytocourtwhereherhusbandwasduetoappearonfraud
charges.Thechargesrelatedtoembezzlementtocoverthehusband'sgamblingdebts.
Sheisa30-year-oldhousewifewhowasfullyactiveyesterdayandcarryingouthereveryday
lifeupuntilthismorning.Youhavereviewedthecaseandfoundthepatientpresentedwitha
similar condition a year ago at the time the fraud was first alleged. At that time, she was
admittedtohospital,andinvestigationsincludingcomputedtomographyofthespineandhead
werereportedasnormal.Aftertwoweeksinhospitalsherecoveredtheabilitytowalk.

YOURTASKSARETO:
Examinethelowerlimbswithattentiontotheneurologicalsystemyouhavesix
minutestocompleteyourexamination.
Reportyourfindingstotheexaminerasyouproceed.Alsotakenoteofthepatient's
generalbehaviouranddemeanour.
Answerthequestionswhichtheexaminerwillaskyouaboutthisproblem.
Providealikelydiagnosistotheexaminer,andgiveyourreasonsforselectingthe
diagnosis.
ThePerformanceGuidelinesforCondition089canbefoundonpage478
455
2-G
The Psychiatric Consultation

2-GThePsychiatricConsultation
PerformanceGuidelines
MCAT083-089
083 Medicationchangesfora35-year-oldwomanwithchronicschizophrenia
084 Demandforurgenttreatmentfor'suddenhairloss'froma29-year-oldman
085 Poorworkperformanceina30-year-oldfemalepoliceofficer
086 Lifestylestressina45-year-oldman
087 Bingedrinkingina25-year-oldman
088 Nausea,headacheandfeeling'jittery'ina30-year-oldbankclerk
089 Collapseofa30-year-oldwomanonthewaytoacourtattendance
456
083
Performance Guidelines



Condition 083
Medication changes for a 35-year-old woman with chronic schizophrenia
AIMS OF STATION
Toassess the candidate's ability to explain the need for antipsychotic medication change.
EXAMINER INSTRUCTIONS
The examiner will have instructed the patient as follows:
You are a 35-year-old woman with chronic schizophrenia characterised by chronic paranoid delusions about being
'spied upon by people who live in your roof cavity'. Occasionally you can hear them whispering amongst themselves
or 'operating a computet system that enables them to track you with electromagnetic radiation '.
Although you live at home with your aging parents, you are socially isolated and rarely go out. You have been taking
thioridazine under protest for 15 years and your condition is relatively stable. You have only limited insight into your
illness. You are convinced that a family does live in your roof and you only take medication because your mother
supervises this and insists that taking medication daily is a condition of you continuing to live at home. You cannot
realistically move out, as you cannot afford to live anywhere else. Your only income is the disability support pension.
Your weight is 66 kg and your height is 1.5 metres (BMI 29 kg/m2).
You are attending your general practitioner to receive a repeat prescription of your thioridazine (Melleril). It is a
drug that has given you a mild dry mouth, blurred vision and constipation for years, so that when the doctor tells you
it is time for a change, you are relieved, but sceptical unconvinced that you need medication at all.
Listen carefully to what the doctor says about the new medication options and respond appropriately depending on
what is said.
EXPECTATIONS OF CANDIDATE PERFORMANCE
The candidate should advise the patient of the need for medication change by appropriately:
Outlining recent evidence linking her current medication with potentially life-threatening arrhythmias.
Describing the benefits and side effects of alternative medications.
Obtaining informed consent for medication change.
Outlining the management plan as detailed in the commentary.
Responding to patient's queries.
KEY ISSUES
Explanation of risks of continuing current medication.
Explanation of benefits and side effects of recommended alternative medications.
Monitoring and followup during medication changeover.
457
083
Performance Guidelines


CRITICAL ERROR - none defined
COMMENTARY
In 2002, Australian prescribers were alerted to the recently established link between thioridazine and prolongation of
the QTc interval of the heart. There is a danger of life-threatening ventricular tachycardia if the QTc interval is longer
than 500 milliseconds.
Pre-existing cardiac pathology, electrolyte abnormalities, thyroid disease, cerebrovascular disease, severe
bradycardia and many commonly prescribed drugs may all lengthen the repolarisation phase of the ventricular
myocardium, which could trigger the polymorphic tachycardia known as torsade de pointes.
This arrhythmia is usually self-terminating but can progress to ventricular fibrillation or sustained tachycardia.
Dizziness, syncopal episodes, cardiac arrest or death may result. Blockade of cardiac potassium channels may be the
mechanism and genetic factors may play a part. Women are at greater risk.
Thioridazine has also long been known to cause lenticular opacities when used in high dosage for long periods. But it
is the anticholinergic effects which cause most subjective discomfort. Sedation, postural hypotension and weight gain
are other well known side effects. Extrapyramidal toxicity with thioridazine is uncommon, but not rare.
Since 1992 there have been several atypical antipsychotic drugs available for prescription in Australia. They are
'atypical' in the sense that their mechanism of action is not solely to block CNS dopamine D2 receptors and they are
thus less likely to cause tardive dyskinaesia or other extrapyramidal syndromes. Their efficacy is equivalent to
conventional 'typical' antipsychotics.
The most important emerging side effects of the atypical agents are weight gain, metabolic disturbances and
hyperprolactinaemia. Risperidone, olanzapine, amisulpride and quetiapine are available for prescription on the
Pharmaceutical Benefits Scheme for schizophrenia.
Weight gain liability is not confined to the atypicals, but olanzapine, quetiapine and risperidone (of the drugs
available for prescription by general practitioners) are associated with faster and greater weight gain than typical
antipsychotics. Adolescents may be particularly susceptible to this side effect. Weight gain is not dose-dependent, but
patients who were relatively underweight prior to treatment may put on the most weight.
The mechanism of weight gain may be blockage of the histamine H, receptor (which also causes sedation) and
antagonism at 5HT2A receptors, as well as impaired feedback of the adipose tissue-leptin loop.
There is an increased risk of Type 2 diabetes mellitus in patients with schizophrenia independent of treatment, but
weight gain and inappropriate dietary choice increase the risk further. Atypical antipsychotics, particularly olanzapine,
heighten the prevalence of Type 2 diabetes especially in overweight or obese patients, by increasing insulin
resistance. The risk is less with risperidone and quetiapine.
Triglyceride levels, but not cholesterol, may also be significantly increased by olanzapine. Data about other atypicals
are limited.
458
083
Performance Guidelines



Hyperprolactinaemia is a well known side effect of typical antipsychotics due to removal of inhibition of prolactin
secretion by hypothalamic dopamine receptor blockade. Risperidone, of the atypical antipsychotics, has the greatest
risk of hyperprolactinaemia and hence amenorrhoea, galactorrhoea. decreased libido, impotence and anorgasmia.
Prolonged uninhibited prolactin release may cause hypogonadism and decreased oestrogen and testosterone
secretion, which in turn increases cardiac morbidity and osteoporosis, and gives an increased risk of breast cancer.
Hyperprolactinaemia, however, is a laboratory finding and is not always associated with clinical symptoms.
Clozapine is the atypical antipsychotic with the most adverse side effect profile, including agranulocytosis, but it is
available only through specialist clinics.
Amisulpride is the atypical antipsychotic most likely to cause extrapyramidal side effects, although it is claimed to
preferentially block limbic dopamine receptors rather than those in the striatum.
The decision to change a patient from a typical to an atypical antipsychotic depends on the risk/benefit ratio. A
patient on typical antipsychotics, who is stable and whose symptoms have reached equilibrium with minimal side
effects, should not necessarily be changed to an atypical agent just because there is a choice. There is an increased
risk of relapse requiring hospitalisation during the changeover period and patients need to be warned of the above
major side effects.
Lifestyle and dietary advice, weight monitoring and an agreed exercise program are all essential elements of the
preswitch counselling process. Baseline weight, body mass index, blood pressure must be measured as well as a
12-lead ECG. thyroid function, fasting blood sugar and lipids, prolactin, full blood count, electrolytes, urea, creatinine
and liver function tests.
Once the decision has been made to change a patient from one antipsychotic to a new one, then for a nonacute
patient, a crossover period of 1-2 weeks is recommended by reducing the dose of the previous medication and
gradually increasing the dose of the new medication. Patients and caregiver need to be alert to the 'early warning
signs' or relapse signature' symptoms which herald the return of acute psychosis.
Nonspecific discontinuation symptoms may persist for several weeks after changeover. These include: nausea,
headache, restlessness and an influenza-like syndrome.
In this patient, because of the length of time on a typical antipsychotic, any change toa newer atypical agent may
require dose-titration over several weeks and doses towards the higher end of the therapeutic range may be
necessary.
459
084
Performance Guidelines



Condi t i on 084
Demand for urgent treatment for 'sudden hair loss' from a 29-year-old
man
AIMS OF STATION
This is primarily a diagnostic and communication skills station, assessing the candidates ability to take an empathie
and relatively quick psychosocial history and to rapidly reach the correct diagnostic conclusion.
EXAMINER INSTRUCTIONS
The examiner will have instructed the patient as follows:
You are a 29-year-old financial analyst in a large city merchant bank. It is a competitive environment. You have been
with the company for six years since graduation from university, but your career progress has stalled. Although you are
conscientious and reliable at work, it is three years since your last promotion. Fellow cadets from your intake cohort
seem to have left you behind. You have just had a performance appraisal interview and have once again been passed
over for promotion. Your explanation for this is that you are losing scalp hair and must be going bald and that it is your
hair loss that has cost you your promotion.
Since your adolescence you have been concerned about your appearance and grooming. It began with a belief that
your face was asymmetrical which you believed was obvious to other people and this led to you checking your facial
features in a mirror several times a day. Then you began to notice facial skin flaws and different shades of
pigmentation. If you developed a pimple or shaving rash you would touch, pick at and constantly inspect the lesion in a
mirror on an hourly basis.
Checking your facial appearance on a regular basis during the course of a day has now become part of your daily life.
Each mirror checking episode lasts several minutes with you having to reassure yourself that no new blemishes have
appeared or that any existing blemishes are improving or fading. Whilst at work you can only do this every two hours
for a few minutes, but at home and on weekends it may take you at least half an hour to complete a thorough
inspection of your entire head and face region, which you repeat three times a day. If asked, you will concede that the
total amount of time you spend checking your appearance in front of the mirror, touching, examining and picking at
almost every skin pore or hair follicle, could be four hours a day on weekends. The amount of time you spend
monitoring your appearance is slowly increasing.
You still live at home with your parents, who are both school teachers approaching retirement. Your mother suffered
from agoraphobia during her twenties before her marriage. They have become accustomed to you constantly asking
them 'Howdo I look?' or, 7s there something wrong with my face/skin/mouth/eyes/hair?' Their unfailing reassurance
that there is nothing wrong with your appearance does not reduce your concerns as you are sure they are only saying
that to humour you.
In the past decade, you have spent a small fortune on male beauty treatments, face packs, facial massages, hair
styling and allergy-free soaps, shave creams and cosmetics in an attempt to treat or camouflage your skin defects. It is
your main interest. Your concern for the care of your skin means that you do not like to socialise at parties or clubs
where people may smoke. You avoid crowds and public transport to avoid embarrassment of strangers
460
084
PerformanceGuidelines



subjectingyourfeaturestoclosescrutiny.Youavoiddirectsunexposure.Youdonotsmokeor
drinkalcohol.
Overthepast fewmonthsyouhavebeenmonitoring thenumber ofyourscalphairsyouhave
foundontheflooroftheshowercubicleafterashower.Althoughitmayonlybeoneortwo,you
havecometobelieveseveralthings.Thefirstisthatthecumulativelossofhairsmeansthatyou
aregoingbaldandthattheresultantchangeinyourhairdensityandthicknessisobvioustoother
people.Youarecertainthatthisobvioushairlosshasinfluencedyouremployersnottopromote
youbecausetohaveemployeeswiththinninghairisnotgoodforthebank'simagewhendealing
withclients.
Finding out that you have yet again not been promoted has driven you to seek advice, about
diagnosisandtreatmentforyourhairloss.Asyoudonottrustyourfamilygeneralpractitioner,
whohasdismissedyourconcernsandsaidthereisnothingwrong,youhavesoughtasecond
opinionfromanunbiaseddoctoratthecityhospitalclosesttowhereyouwork.
Howtoplaytherole
Youmustbeneat,welldressedandhaveafullheadofhair.Youwillhaveanenvelopewitha
coupleofstrandsofyourhairinit.Youwillbeanxiousandsomewhatirritable.Ifthereis
7
amirror
intheconsultingroom,theninsistonshowingthedoctoryour'recedinghairline'atanearlystage
oftheinterview.(Itwouldbeusefultohaveasmallmirrorasaprop)
Openingstatements
Doctor,you' vegottofindoutwhyI' mlosingmyhair!'
Iwantthesehairsofmineexaminedunderamicroscopebyaspecialist!'
'LetmeshowyouwhereI' mgoingbald'.
' You' vegottodosomething!'
Your subsequent behaviour and emotional reactions will be shaped by the way the interview
unfolds.Ifthedoctorrushestojudgmentanddismissesyourconcernswithouttact,empathyor
appropriatediscussion,thenyourirritabilityandexasperationmayincrease.
If the doctor realises what your underlying problem is. effectively establishes rapport, and the
extent of your difficulties and hypochondriacal concerns is realised, then be defensive and
sceptical,butbepreparedtolistenandinteractappropriately.
Donotwillinglyvolunteerhistoryofyourritualsorcheckingbehaviouruntilasked.Thesehave
beenbehavioursyouhavekeptsecretforyears,butyoumayberelievedthatatlastsomeoneis
abletoencourageyoutotalkaboutthem.
Aftersixminutes,theexaminerwillinterrupttheconsultationandask' Whatisyourprovisional
diagnosis?Describebrieflypossiblemanagementplanstome.'
EXPECTATIONSOFCANDIDATEPERFORMANCE
ThepatienthasaformofBodyDysmorphicDisorder(BDD)presentingwiththeconvictionof
impendingbaldness,whenobjectivelyandclinically,thereisnosupportingevidence.
BDDisaformofhypochondriasiswhichispartoftheanxietydisorderspectrum.Inthisinstance
therearemanyobsessivecompulsivedisorderfeatures.BDDisaconditionthataffectsabout1%
ofthepopulation,butisinfrequentlydiagnosedbecauseofthelackofawarenessbyclinicians
andpatients'secrecyabouttheirbodilypreoccupations.
461
084
Performance Guidelines



The candidate will be expected to establish the diagnosis, the associated behaviours and the
complications in this case, as well as being aware of the common comorbid psychological
disorders. Knowledge about the effectiveness of the serotonin reuptake inhibitor antidepressants
and cognitive behavioural therapy in this condition would be desirable.
Key questions the candidate should ask of the patient would be:
'Apart from your hair, have you ever been very worried about your appearance in any other
way?'
(If yes): 'Can you tell me what your concern was?'
'Did this concern preoccupy you? Do you think about it a lot and wish you could worry about it
less?'
'What effect has this preoccupation with your appearance had on your life?'
'Has it affected your social life, family relationships, friends, job or other activities?'
Do you wish to do anything about your concerns?'

KEY ISSUES
Ability to take a focused psychosocial history and to come to an appropriate diagnosis.
Ability to communicate with a patient with body dysmorphic disorder.

CRITICAL ERRORS - none defined

COMMENTARY
'Body Dysmorphic Disorder, or dysmorphophobia, is a chronic preoccupation with an imagined
defect in one's appearance. Even if a slight physical anomaly is present, the person's concern is
markedly excessive. The preoccupation causes significant distress or impairment in the person s
social, occupational and other important areas of functioning.
Typical complaints commonly involve imagined or slight flaws of the face or head such as thinning
hair, acne, wrinkles, scars, vascular markings, paleness or redness of the complexion, swelling,
facial asymmetry or disproportion or excessive facial hair.
Other common preoccupations include the shape, size or some other aspect of the nose, eyes,
eyelids, eyebrows, ears, mouth, lips, teeth, jaw, chin, cheeks or head. Any other body part may be
the focus of concern (the genitals, breasts, buttocks, abdomen, arms, hands, feet, legs, hips,
shoulders, spine, overall body size or body build and musculature).
The preoccupation may focus simultaneously on several body parts and although it may be often
specific: 'a hooked nose'; it may also be vague: 'a flat chest'; or general: 'I'm just ugly'.

From Diagnostic and Statistical Manual 4 - Text Revision
Most individuals with this disorder experience marked distress over their perceived deformities.
They find their preoccupations difficult to control and may make little or no attempt to resist them.
Many hours of the day may be spent thinking and worrying about their 'defect' and these thoughts
may dominate their lives, leading to significant impairments in functioning and avoidance of work,
social and public situations. Repetitive and
462
084
Performance Guidelines



time-consuming behaviours are undertaken to reduce their distress, which have no or only minimal benefit.
Reassurance that there are no visible defects has no lasting effect on their abnormal beliefs.
The core irrational belief in BDD is that the person is somehow defective and unattractive and this is accompanied
by low self esteem, shame, embarrassment and fear of rejection. The condition is common if it is looked for and
asked about.
The most common associated behaviours are mirror-checking, touching, comparing the defect' with other people's
body parts either directly or with pictures in magazines, excessive grooming, camouflaging, constantly seeking
reassurance and questioning others about their alleged defects or ugliness and then seeking dermatological or
cosmetic surgical treatments.
Social impairment is the norm. They are socially avoidant and will not willingly visit restaurants, shopping centres,
beaches or go to parties or functions because of their self-consciousness about their appearance. Insight is usually
poor or partial and their beliefs may become delusional. It is their self-referential ideas, i.e. that other people are
taking special notice of their 'defect' and will talk and gossip and laugh about it, that contribute to their social isolation
and intensify their suffering to the point of despair, self-harm and sometimes suicide.
Psychiatric comorbidity is universal. Major depression is the most common (60%) but social anxiety/phobia,
obsessive compulsive disorder, substance abuse and avoidant personality disorder are highly prevalent.
There is a roughly equal sex incidence and similar clinical features. Perhaps women are more likely to focus on their
skins, lips, and weight, whereas men are more preoccupied with overall physique, their genitals and hair loss or
excess. The condition typically begins in adolescence, but may not present or be diagnosed until the thirties. The
course is chronic and relapsing.
Most patients with BDD seek costly dermatological or cosmetic surgical consultations and treatments, but remain
dissatisfied with the results. They may then become litigious or violent. Rarely patients perform their own
procedures after consulting internet web sites.
Management
The specific serotonin reuptake inhibitor antidepressants and clomipramine are often effective. The dosages need
to be in the higher range and it may take three months to get a response, but 70%of patients report improvement.
Augmentation with antipsychotics may increase the response rate. Treatment must be continued longterm as
relapse is common if treatment is discontinued.
Cognitive behavioural treatments including psychoeducation, cognitive challenge and restructuring, exposure and
response prevention, as well as anxiety management training, can supplement drug treatment and increase
response rates to over 80%. Severe comorbid depression may need hopitalisation and/or lithium carbonate
augmentation with antidepressants.
Trying to convince patients with entrenched ideas that their beliefs are irrational or that they look normal is unlikely to
persuade them to accept psychiatric treatment or referral. With the patient's consent, family involvement in
psychoeducation and treatment planning and supervision of response prevention strategies and the removal of
mirrors from the family home may be valuable.
463
085
PerformanceGuidelines



Condition 085
Poorworkperformanceina30-year-oldfemalepoliceofficer

AIMSOFSTATION
Toassesstheabilityofthecandidatetodiagnoseastress-related depressionassociatedwith
increasedalcoholintake.
Toassesstheabilityofthecandidatetodeterminewhenandwhytheproblemstarted,andwhat
theramificationsoftheproblemmightbeatthistimeorinthefuture.

EXAMINERINSTRUCTIONS
Theexaminerwillhaveinstructedthepatientasfollows:
You do not wish to be examined by the candidate and resent having been advised to see the
generalpractitionerbythePoliceService'sstaffcounsellor.Youshouldpresentasanxiousand
initiallyreluctanttoadmitthatyouarehavingdifficultiesatwork.Whenthecandidatequestions
youfurther,answerthefollowingorsimilarquestionsdirectly,butvolunteeradditionalinformation
onlyifappropriatelyled.
After candidates introduce themselves, insert the following into your first answer to them: I
di dn' t r eal l ywant t ocomet oday. I amonl yher ebecauset hePol i ceSer vi ce
t hi nks I have a pr obl em. '
Ifyouareaskedifthereareanyproblemsatwork,atfirstsay:Ido my j ob wel l t her e
have been no compl ai nt s about what I do at wor k. '
Youareapoliceconstablewithavarietyofserviceattachmentsinthefieldandatofficetasks.
Youlivealone.Nocurrentromanticrelationships.Toobusy at wor k. '
Yourfamilyisinterstate.
Youhavel ot s of f r i ends' , buthaven'tseenthemmuchlately.
Beenill?' Not t hat much mor e t han ot her s. I ' ve had si x days of f i n t he l ast
mont h. '
Daysoffwhenshiftchanges?IguessI ' vehadoneor t wodaysof f ont hel ast f our
t i mes t he shi f t changed. '
Whathasbeen wrong?' I ' ve been f eel i ng r eal l y j umpy and on edge. I ' ve been
f i ndi ng i t har d t o concent r at e and j ust don' t have any conf i dence. '
Whendidthisstart?' Si x mont hs ago. ' ' One day I had t o go t o a f at al mot or
vehi cl eacci dent wher eanadul t andt wochi l dr enwer eki l l ed. Thever ynext
dayI wascal l edt oanar medhol d- upandbot hmywor kcol l eagueandI wer e
shot at by t he of f ender . I was OK, but my wor k col l eague r ecei ved ser i ous
i nj ur i es and he has not r et ur ned t o wor k. '
Whenhastheanxiousfeelingbeenworse?' WhenI t hi nkabout t hoseexper i ences,
or when I have t o deal wi t h anot her si t uat i on wher e peopl e mi ght get hur t .
I t ' s al so wor se when I ' m goi ng back t o wor k af t er some days of f . '
085
464
Performance Guidelines



What is the anxiety like? Volunteer any two of the following features if asked a general question:
~Feel terrified something awful will happen.
~Racing heart (palpitations), breathing faster, and perspiring a lot.
~Nightmares of both incidents most nights.
~Can't watch television or read magazines with pictures or articles of car accidents or Police Officers
being shot at.
What will happen in the future? 'It is only a matter of time before I'm injured or killed in the line of duty. '
Alcohol? Only answer direct questions 'Over the last six months my alcohol intake has increased from 1-2
glasses of wine per week to 6-7 glasses of wine each day on most days. '
What does alcohol do for you? 'It relaxes me and lowers the anxiety. It's the only thing that helps me
"unwind" after work and enables me to get to sleep. '
Does your use of alcohol bother you? 'I am worried that I have to have a drink to control the anxiety. '
Suicidal thoughts? 'No. ' If pressed further, add 'I have sometimes thought that life is not worth living. '
Any plans to commit suicide or past attempts to harm yourself? 'No. Never. I don't want to end my life. '
What about your gun? 'I have not handed this in nor have I been asked to do so. '
Examiner's questions
Towards the end of six minutes, the examiner will ask the candidate to describe three conditions which
should be included in the differential diagnosis. Appropriate responses are:
1. Anxiety disorder: post-traumatic stress disorder, panic disorder or generalised anxiety disorder.
2. Depressive disorder: adjustment disorder, major depressive disorder.
3. Al cohol dependence/abuse, or just problem drinking.
Next questions for the examiner to ask should be:
'What is a safe level of alcohol consumption for this patient?' Low risk =maximum of 20 grams per day (two
standard drinks) and two alcohol-free days per week (NHMRC Levels for Women).
'What are the short term risks associated with the patient's current level of alcohol use? You should expect
at least four of the following:
~hangover effects headaches, anorexia, tremor:
~gastritis;
~impulsive acts including suicide attempts;
~impaired decision-making:
~accidents, including with firearms;
~worsening of mood/depression;
~potentiation of anxiety or post-traumatic stress disorder symptoms; or ~social or occupational problems.
NOTE: If the candidate has not identified alcohol dependence/abuse as a problem, these questions should
NOT be asked. Instead ask 'Are there any other possible diagnoses?

465
085
Performance Guidelines
KEY ISSUES
Ability to take a focused history to define the potential cause of the current problem.
Knowledge of the causes of the problem
Knowledge of the short-term effects of alcohol excess, and the NHMRC recommendations of alcohol abuse.

CRITICAL ERRORS
Failure to identify the excess alcohol consumption.
Failure to ask about suicide.

COMMENTARY
This case concerns a patient with a work experience that has exposed her to severe stress, leading to an alcohol
abuse problem to help relieve a post-traumatic stress syndrome with anxiety. The patient is also at risk of comorbid
depression and suicide. She has a responsible job and failure to help her may also result in her colleagues or other
members of the community being at risk.
Candidates must both take an adequate history of alcohol consumption, and also explore the factors that have led to
the problem, namely the fatal car accident and the shooting of her work colleague the next day. They should ask about
the key features of post-traumatic stress, assess subjective mood and risk of suicide and. particularly in this case, her
access to firearms. Identifying the alcohol problem alone is insufficient for a pass. Candidates should be familiar with
at least four of the short-term risks or consequences of excess alcohol use and the NHMRC recommendations
concerning alcohol consumption. If they are not aware of at least four, a pass mark is unlikely to be given.
466
086
Performance Guidelines



Condi t i on 086
Lifestyle stress in a 45-year-old man
AIMS OF STATION
To assess the candidate's ability to recognise and to communicate to the patient, common behavioural,
physiological, psychological and emotional concomitants of lifestyle stress, and to formulate and implement an
appropriate immediate and preventive management plan,
EXAMINER INSTRUCTIONS
The candidate needs to have sufficient skills in evaluation of cardiovascular disease to recognise this is not
ischaemic heart disease, and advise the patient accordingly. If candidates attempt to seek further history,
examination or investigation of cardiovascular disease, the examiner is to inform them (once) they are to proceed on
the basis of a normal examination. No further prompts. The candidate who pursues a physical cause to the exclusion
of the psychological matters should be marked down.
The examiner will have instructed the patient as follows:
You have been recommended to see a general practitioner to review your health after an insurance assessment. You
have been told that cholesterol, blood sugar, and resting ECG are normal, but on two occasions in the last month
when tested by the insurance doctor, the BP reading was high (160/80 mmHg) but eventually settled to normal levels.
On those days, you had come from particularly difficult meetings.
You have never thought about having serious physical illness, and believe people who complain of stress 'just aren't
motivated enough or don't work out their goals properly'. You enjoy being challenged by work and sports, and people
coming to see you for advice. You never take a day off work, even with a bad cold or jetlagged from a trip, and avoid
taking tablets, even a Panadol if you have a headache at the end of the day.
The only health problem you have noticed is more frequent headaches towards the end of the day. which you put
down to eyestrain.
You have never felt depressed, or at any time recognised yourself as being anxious, and you would still enjoy all your
usual activities if there was time to do them.
You feel rather shocked by what you have been told and by the way you have felt over the past couple of weeks. You
have planned to retire at 55. You work 12-15 hour days. Since you were promoted 18 months ago, you have been
taking work home on weekends more frequently and there is a lot more pressure. You sometimes feel like escaping,
but think of it only being a few more years and anyway you feel happier at work than at home these days. It is just too
noisy with three teenage children, and you get irritated with your eldest son who dropped out of university last year,
and 'just sits around playing music with his mates'. You did not go on the family holiday this year for the same reason,
blaming work. Your spouse is usually very understanding, but got mad with you about that and things have been
tense the last few months. You have been more irritable at home, and your spouse complains you criticise the
children too much.
467
086
Performance Guidelines



Younormallygetonwithafewhourssleep,oftenthinkingthroughworkproblemsonceyouarein
bed.Youneverfeeltired,butthelastyearorsoyou' catchupwithnaps' onweekends.Thelast
coupleofweeks,youhavesleptbadly,andfeeltiredinthemorningThereislittletimeforsex,but
youaresureitisnotyourlackofinterestthatistheproblem.
Youusedtoexerciseregularlyatthegymandwithweeklytennisbuthavenotdonesointhelast
fewyearsbecauseyouarejusttoobusyatwork,andrunningaroundwiththechildren'sactivities
onweekends.Now,youwalkupthestairswhenyouhavetime.
Youhavegainedafewkgoverthepastfewyears,butdonotconsideryourselfoverweight,or
unfit. You have always eaten a balanced diet, except when on overseas trips and at business
lunches.
You stopped smoking 15 years ago when your father, who was a very heavy smoker and ate
badly,diedat65ofaheartattack,butstartedagainsmoking3-5cigarettesadayinthepast12
monthsbecauseithelpskeepyougoingthroughtheday.Youhaveneverworriedaboutsuffering
your father's fate, because you have looked after yourself so well. You consider yourself a
moderatesocialdrinker.Younoticeyouaredrinkingmorethesedays.
You feel uncomfortable in an unfamiliar environment, and somewhat embarrassed about the
problem. Answer any questions from the candidate in a straightforward manner. Seek
reassuranceabouttheriskofheartattack,butdonotlabourthepoint;andreadilyacceptfurther
investigations,ifoffered.
Expect to be provided with an effective solution and definite results, the sooner the better, but
acknowledgethestresslevelsandsourceswhenidentified,andseekadviceregardingchange.
Youaremoreacceptingofastraightforwardsolution(forexample,regularrecreation,exercise,
andsleeppattern).
Questionstoaskunlessalreadycovered:
Openingquestion:7sthisstress,orsomekindofabreakdown?'
' Coulditbethebeginningofhearttrouble,orcauseaheartattack?'
' Ifmyheartisokay,whydidIhavethosepainsinmychest?'
'Whatwouldhelpthemost?'
' WhatdoyousuggestIdo?'
'Howlongwillthisalltaketomakeadifference?'
' WhathappensifIneedthesetabletsallthetimetosleep?Aretheyaddictive?'
' WhatelsecanIdotosleep?'
EXPECTATIONSOFCANDIDATEPERFORMANCE
Thecandidateshould:
Informthepatientaboutcommonsymptomsofstress,bothpsychologicalandphysiological.
Explain the mechanism of physiological symptoms headaches resulting from muscle
tension,sympatheticarousalcausingbloodpressureriseandinitialsleepdisruption.
Reassureaboutnatureofstabbingchestpain.
Identifyforthepatientthesourcesofstress(overwork,absenceofleisureandexercise)and
the compensatory measures which increase physical and emotional burden (alcohol and
smoking,reducedsleep).
468

086
Performance
Guidelines
Educateaboutthenegativeroleofstressincardiovasculardiseaseandmentalhealth.
Assistthepatientinidentifyingrealisticchangesforhealthierwork/personalbalance(e.g.
increasedrecreationthroughregularexercise,holidays,andsleeppattern).
Initialmanagementplan
Appropriateadviceregardinglifestylechange,includingdiscussionwithspouse
Avoidanceofintensiveorinvasivemanagement.

KEYISSUES
Patientcounsellingexplanationofdiagnosisandpatienteducationandinitialmanagement
plan.
Provisionofanadeguateexplanationaboutstress,itsoriginsanditsphysical,behavioural
andpsychologicalseguelaeandcomplications,acuteandchronic.
Reassuranceaboutbloodpressureandchestpain.

CRITICALERROR-nonedefined

COMMENTARY
This patient has obsessional personality characteristics of perfectionism, mental and
interpersonal control, propensity to overwork, and inflexibility (not personality disorder, on
availableinformation).Asacopingstyle,ithasbroughtoccupationalandpersonalsuccess,but
inthecontextofpromotionandincreasinglycomplexlife,especiallyfamilydemands,itisnow
beingoverused;healthycompensatorymechanismssuchaspursuitoffitnessandcompetitive
sportshavebeendiscarded,anddysfunctionalhabitssubstituted.Thedegreeofdysfunctionis
sufficient to produce somatic and behavioural symptoms increased headache, chest pain,
labile hypertension, increased intolerance and isolation. The patient is stressed and does not
haveapsychiatricorphysicalillness,butinterventionisnowneededtomodifythosebehaviours
which increase risk of cardiovascular disease and psychiatric illness, as well as relationship
breakdown. Recognition of the personality style enables realistic intervention brief,
behaviourallyorphysicallymediated,withextensiontoincludespouseforinterpersonal/family
issues.
Management
Theessentialmanagementistoprovidethispatientwithappropriatelyfocusedbriefintervention
tomodifybehavioursandlifestyle,afterengagementthroughadequatereassuranceregarding
physical illness, followed by education of the physiological mechanisms and identification of
stressors. Use of hypnotherapy for sleep disturbance (short-term) is acceptable but other
measuresarepreferable,suchasaregularsleeppattern,andprogressivemusclerelaxation.
Modificationoflifestylerestrain workinghours,alcoholandcigaretteconsumption.Resume
exercise,increaseleisureactivities,includingfamilypursuitsthesearetheinterventionsmost
abletobeimplemented.
Includethespouseinsupportinglifestylemodificationsandenablediscussionofinterpersonal
andfamilyissues.
Followupandongoingmonitoringofbloodpressureandcardiovascularhealthisindicated.
469
086
Performance Guidelines



Communication skills
Ability to take this opportunity to engage a reasonably informed, currently anxious, but usually pragmatic and busy
person in appropriate lifestyle change.
Ability to combine the tasks of identifying stressors, with providing psychoeducation, through using direct questions
around the main activities of daily living and habits, and empathie listening.
Attitude to alcohol and cigarette use should be nonjudgmental and proportionate to use.
Consideration to inclusion of spouse in further discussion.
470
087
Performance Guidelines



Condition 087
Binge drinking in a 25-year-old man

AIMS OF STATION
To assess the candidate's ability to recognise the specific risks of the patient's drinking pattern
and to counsel him accordingly.

EXAMINER INSTRUCTIONS
This is problem drinking of a binge drinking type with consequential exposure to risk in this
instance, to personal physical injury, neglect of parental responsibilities and potential harm to
his daughter. The binge drinking pattern risks impaired judgement and injury to self and others.
Other potential problems, some of which he has manifested in the past, are of illness, financial,
work, relationship, social, psychological and legal complications. Brief interventions such as this
consultation provide a vital opportunity to initiate change in critical patterns of dysfunctional
behaviour.
The examiner will have instructed the patient as follows:
Opening statement
(You are embarrassed and feel tense but attempt to make a joke of it) I deserve this knock
and the team lost! I think its put some sense into my head, but I'll listen to your lecture just in
case!'
You have no recollection of your fall, or how much you had to drink.
Your mates said you were 'playing up'.
Respond to further questions about the amount you drink, or any other attempt to estimate it
(e.g. by cost), defensively 'not any more than my mates at the club don't do anybody
any harm.'
In response to specific questioning:
You drink because it helps you unwind, and makes you more sociable; you need a break by
the end of the week with work and taking care of your daughter.
Admit to being stressed by the demands of being a single parent and not being able to call on
your family for help because they hold you responsible for your former partner leaving. Now
you no longer have a babysitter because the next door neighbour who has filled that role now
refuses to do so because of the events last week. She had to go home and left your daughter
asleep, alone.
You regard yourself as a responsible and caring parent and had intended to be home. If
specifically asked, you admit this has happened before but rationalise your lapses by saying
' The girl is such a good sleeper, she never wakes and would never know.'
You do not feel anxious or depressed.
You do not have any symptoms of panic or phobia of any kind.
You have never had any medical complaints, apart from minor injury like today.
You do have 'blackouts' (episodes of amnesia) quite often.
You do not have any fits, faints, withdrawal symptoms (sweats, tremor and palpitations).
You have had no period of abstinence longer than two weeks.
You have not noted any change in tolerance.
087
Performance Guidelines
471
You do not drink in the morning.
You have had one drink driving charge with no loss of licence several years ago. No other forensic history.
Questions which may be asked with appropriate responses
'Are you saying I'm an alcoholic?' The candidate should indicate there are ranges of consequential problems
to excessive drinking, and binge type drinking is associated with increased risk-taking and acute harm
events.
'Look at my father he's always been a drinker and he's okay' There are various responses, including
simply accepting this statement without comment, to a reminder that his father's drinking was associated
with violence and family dysfunction, which the patient would want to avoid for his daughter.
'Howcan I relax if I don't have a few drinks with my mates?'The candidate could respond with an undertaking
to discuss this further, or the option of controlled drinking (less feasible with a binge drinking pattern), or
some introduction of relaxation techniques.
Further instructions:
You know this doctor quite well, and generally feel comfortable here. You are embarrassed about your drinking
problem being addressed directly and you are initially tense and defensive, especially about any risk or harm to
your daughter or doubt about your parenting capacity. However, unless the doctor is unduly critical, you are
prepared to listen to the advice and respond in a positive way to changing your drinking habits.

EXPECTATIONS OF CANDIDATE PERFORMANCE
Recognise that alcohol overuse binge pattern is the primary problem.
Discuss the problem and actual consequences.
Seek out whether there are any aggravating as yet undisclosed issues or current
stressors.
Counsel the patient about the risks to his daughter and his relationship with and care of
her.
Advise reduction or cessation of alcohol use discuss.
Demonstrate knowledge of hazardous/harmful drinking levels (NHMRC guidelines for men, safe up to six
standard drinks per day. no more than three days a week).
Be able to communicate concern in a nonjudgmental and nonthreatening way so as to maintain rapport and
ensure engagement in ongoing review and case management.
The candidate is expected to diplomatically but firmly advise that the patient has a habit of binge drinking on
weekends, and that change is necessary, for his own wellbeing and his daughter's. The candidate should
approach this by discussing this incident and injury and asking about any current problems or stresses, and
encouraging the patient to talk about his views about his drinking and other potential problems.

KEY ISSUES
Discussion about binge drinking and consequential harm.
Highlight risk to daughter discuss potential referral to child protective services or equivalent.
472
087
Performance Guidelines



CRITICAL ERROR
Not addressing the issue of his daughter's wellbeing, protection and care management in any way.
COMMENTARY
Alcohol abuse and dependency are linked with genetic and developmental predisposition, developmental,
environment, personality traits and other psychiatric illness, especially mood and personality disorders, Sociocultural
factors are also significant. The evaluation of all of these elements is important in the individual case, especially
identification of concurrent psychiatric illness which requires treatment in its own right, and of interpersonal and
sociocultural factors which trigger or maintain the behaviour.
Hazardous drinking of excessive quantities of alcohol intermittently is a subtype of alcohol abuse, which is less likely
to be associated with addictive/withdrawal symptoms than a daily drinking pattern, and more likely to be associated
with injury and other social and interpersonal sequelae of impaired judgement and poor impulse control.
The recognition of hazardous drinking depends less on an estimation of the quantity consumed than defining a
pattern of drinking, often rapidly, to severe intoxication and consequential risk-taking. The symptom of a 'blackout', a
brief period of amnesia without loss of consciousness during a drinking episode, is associated with the rapid
consumption/absorption of alcohol and is a useful indicator of this pattern. Incidents of accidental injury to self or
others, disinhibition and sexual or aggressive acts, and neglect of self or others are other frequent reasons for
intervention. Episodic neglect of financial, occupational and social responsibility is common, such as regularly
missing work after weekends.
Effectively intervening in such a problem requires identification of the reason for the adverse consequences, sensitive
but frank communication about them and the underlying problem and appropriate offers of assistance, including
education, assessment of other psychiatric, medical, social, legal and interpersonal problems, motivation for change
and ongoing review.
In this case, there is a specific need to appropriately address the risk to the patients daughter which must include his
responsibility for parental care and protection, and the responsibility of the doctor to monitor and ensure her
wellbeing. including consideration of referral to child protective services.
In addition to counselling this young man about his hazardous drinking, the doctor may have a statutory obligation to
inform an authority such as the Department of Families, Community Services and Indigenous Affairs of his daughter's
situation. She is potentially at risk of abuse by neglect. Rather than being punitive or restrictive, the Department's
caseworkers should be able to advise him about local child-care and parent support services, as well as possible
child-minding options. Consider checking whether notification was made at the time of his presentation to the
Emergency Department.
Most communities will have single parent support groups and possibly men-only support groups which may be of
interest to him. A good general practitioner will either have relevant pamphlets on hand or be able to print the
information off line.
087
Performance Guidelines



In addition to practical support, this man would benefit from understanding simple problem-solving in relation to his
daughter's ongoing child-care. For example, through her school he could advertise for part-time child support or
pressure the partners of his team mates to care for his daughter during the game and its aftermath.
His drinking pattern may jeopardise his ability to drive and hence his livelihood if he should lose his driver's license or
have a serious accident. Consider at what point should a clinician notify the local license-issuing authority about
CONDITION087. FIGURE 1.
Alcohol content of standard drinks*
'Many thanks to Drug and Alcohol Services South Australia for this poster
473
473
hazardous drinking.
474
088
PerformanceGuidelines



Condition 088
Nausea,headacheandfeeling'jittery'ina30-year-oldbankclerk

AIMSOFSTATION
Toassessthecandidate'sabilitytorecognisethatthehistorystronglysuggestsamildserotoninsyndrome.
The candidate should ask for findings consistent with the serotonin syndrome, namely: sweating, tremor,
elevatedbloodpressure,increasedpulserate,increasedmuscletoneandhyper-reflexia.Ifthecandidatehas
notstartedtoaskforthesefindingssixminutesintotheexamination,theexaminershouldadvisetoproceed
withthetasksrequired,namelyadvisingthepatientofthediagnosisandmanagementofboththeimmediate
problemandherdepressiveillness.

EXAMINERINSTRUCTIONS
Theexaminerwillhaveinstructedthepatientasfollows
Youarea30-year-oldbankclerk.Yousufferfromrecurrentdepression.Yourmedicationhasrecentlybeen
changed (see below). You have come back to the Emergency Department of a nearby general hospital
becauseofseverenausea,headacheandthejitters.
Your opening statement to the doctor should be: I feel awful doctor I' m nauseated, I' ve got a
headacheandafeel i ngofthej i tters.Ital l startedyesterday' .
Without prompting go on and tell the doctor: ' Three weeks ago my l ocal doctor started me on
Prozac (fl uoxeti ne) 20 mg dai l y for a rel apse of the depressi on I get. It di dn' t seem to be
hel pi ngmuch' .
TwodaysagoyoufeltsomebackpainandattendedthehospitalEmergencyDepartment.Yousawadoctor
and mental health nurse and were told it was due to muscle strain'. They were more concerned with your
depressionandprescribedZoloft(sertraline)100mgdaily.YoutoldthemaboutbeingonProzacwhich
youthenstoppedtakingbecauseyouknewthatbothdrugswereantidepressants.
Inanswertofurtherquestionswhichmaybeasked:
Youfeelanxiousand'aroused'.Youarestillsleepingpoorlyandwakingabout4-5am.Youthinkyour
appetitewasimprovingbeforethenauseastarted.
Youhaveneverhadsuicidalideasbutyouremainpessimisticaboutthefutureandfindithardto
concentrateatwork.
Yourbackpainhasresolved.
Youhavehadnoothersymptomsandyourlastperiodwastwoweeksago.
Youarewonderingifthemedicationisthereasonyoufeelsounwell.Whenthecandidateexplainsthatthisis
thecase,youarerelieved.Youareangrythatthehospitaldoctorsdidnotwarnyouofthis,butontheother
handyouthinktheremaywellhavebeensomeconfusionoverwhatwassaidtowhomastheyallseemed
verybusyanddistractedandyoufeltabitsorryforthemandwantednottobotherthemtoomuch.Youare
preparedtofollowthedoctor'sadviceaboutfurthertreatment,butdonotwanttohaveanotherreactionlike
this.Youcouldsaysomethinglike' Couldthishappenagain?'
The candidate is not expected to conduct a physical examination, but will askfor appropriateexamination
findingsfromtheexaminer.
475
088
PerformanceGuidelines



Neartheendoftheexam,ifthecandidatehasnottoldyoutostopthetreatmentwithZoloft,you
shouldask' Shoul d I cont i nue wi t h t he cur r ent dose of Zol of t ?'

Examinerwillprovidedetailsofphysicalexaminationonrequestasfollows:
Pulserate90/min,regular.
Temperature36.8C.
Bloodpressure130/80mmHg
Shehasatremor,herpalmsfeelsweaty,andtoneandreflexesinlimbsarebriskandmildly
hyperactive.

EXPECTATIONSOFCANDIDATEPERFORMANCE
Serotoninsyndromeshouldbediagnosed.
TheappropriateadviceregardingmanagementistostopZoloft(sertraline)andwaituntilthe
symptomsresolve,inabout24hours.AsProzac(fluoxetine)hasalonghalflifethecandidate
should recommend waiting at least another week before reintroducing sertraline, at a lower
dose,forexample,25-30mg.Areasonablealternativeistoreintroducefluoxetine,whichhas
notyethadanadequatetherapeutictrialinthispatient.Thecandidateshouldcontinuetotreat
thedepressionandshouldarrangefollowupwiththeEmergencyDepartmentorthepatient's
general practitioner the next day, and advise the patient to contact the after-hours service
immediatelyifsymptomsworsen.Supportwillneedtobeprovidedforthepatientduringthe
'washout'periodassheisstilldepressed.

KEYISSUES
AbilitytodiagnosetheserotoninsyndromeduetosideeffectsofaSelectiveSerotonin
ReuptakeInhibitor(SSRI)drug.

CRITICALERROR
FailuretorecognisetheneedtostoptheZoloft(sertraline)medication.

COMMENTARY
This scenario is a timely reminder about aspects of psychopharmacology. Side effects are
common with most psychotropics because they may be prescribed too enthusiastically and in
dosages that are inappropriately high, especially in management of' depr essi on' which is a
complexmultifactorialcomplaint inour modernsociety.Notallpatientswith' depr essi on' or
depressivesymptomsneedantidepressants,butlikeantibioticstheyareoftenprescribedreflexly
by doctors under time pressure as a quick fix it cant do any harm panacea for a patient in
distressorintears.Oftenitisthedoctor'shelplessnessthatisbeingtreatedbytheprescription
becausethereisneverenoughtimetoestablishwhythispatientisdepressedonthisoccasion.
476
088
Performance Guidelines



Another common error is to start with too high a dose if the patient is 'really, really distressed {more must be better and
will work faster'). Antidepressants and antipsychotics take 3 4 weeks to work. If the patient's symptoms improve within
that time there may be other factors which explain the improvement, such as reduction in anxiety or insomnia or the
benefits of a sensitive interview with the discussion of issues and problems, or relief that the problem has been
identified and that something is being done.
Often it is not symptoms per se that cause patients or relatives to seek treatment. Patients present to doctors when
they are worried or anxious about symptoms or behaviours, or someone else is, who influences the patient to attend
the consultation. Anxiety intensifies ALL symptoms including 'depression' and is accompanied by typically
exaggerated and catastrophic cognitions about the conseguences and outcome of whatever is causing their distress.
Is it fatal/terminal? Will I go mad/drop dead etc?'
An effective initial consultation with a patient who is 'depressed which attempts a biological-psychological-
sociocultural approach and allows sufficient time for the patient to be listened to, to be understood and to be taken
seriously, will in itself relieve a major part of the intensity of the symptoms. This will only enhance the effectiveness of
whatever is subseguently recommended or prescribed.
Many people with 'depression' have mood fluctuations on a cyclical basis which are subthreshold or relatively mild.
These people are more likely to present at their peaks or troughs when they are symptomatic in response to a life
event or ongoing environmental stress. Their symptoms may be naturally or temporally transient. If these people (as
patients) are then prescribed psychotropics, including antidepressants, when symptomatic (instead of being managed
expectantly), and they improve after a few days, they and their clinician may mistakenly attribute their response to the
medication. This may commit them to a future psychological dependence on medication rather than learning to
tolerate temporary oscillations in mood and biological symptoms by using nonchemical coping strategies.
Some doctors and patients have become brainwashed by pharmaceutical companies into believing that any degree of
distress or suffering reguires a chemical solution that is quick and effective (but freguently expensive and
unnecessary). When a patient has been started on an antidepressant and is appropriately reviewed a week later and
reports no improvement, the inexperienced or unaware clinician may recommend doubling the dose and seeing the
patient a week later. At two weeks, when there is still no major improvement or cure, the dose will be increased or
doubled again. By the third week when the patient reports some improvement at last, this is wrongly attributed to the
increase in dosage and not the latent response to the initial dose.
SSRIs are potent drugs even in low dosage. Once the dosage increases then side effects and toxicity will increase
significantly. Most patients take such medications erratically or in fits and starts (i.e. if they are having ' a good day'
they will skip a dose: if it's a bad day', then they will double the dose). Some patients are extremely somatically
focused and will develop toxicity just by reading the package inserts about product information.
This patient feels aggrieved that she has been mismanaged and ill-served by the doctors who have unknowingly
contributed to her serotonin syndrome. Patients deserve better and clinicians must ensure that they are aware of both
the risks and benefits of the drugs they prescribe. As patients become better informed, they will not tolerate scenarios
like this one lightly. Neither will their legal advisers.
477
088
Performance Guidelines



The serotonin syndrome is caused by excess serotonin in the central nervous system, commonly because of
drug-drug interaction, in this case inadequate washout between a long half life agent (fluoxetine) and a high starting
dose of a second SSRI (sertraline). The syndrome usually presents with changes in mental state (confusion, irritability,
labile mood), restlessness, myoclonus, hyper-reflexia. fever, sweating, shivering and tremor and diarrhoea.
Hypertension, convulsions, and death have been reported. Treatment is to cease the medication and provide
symptomatic care (e.g. cooling blankets). Referral to an emergency specialist may be necessary in more severe
cases.
478
089
Performance Guidelines



Condition 089
Collapse of a 30-year-old woman on the way to a court attendance

AIMS OF STATION
To assess the candidate's ability to conduct an examination of the lower limbs focusing on the
neurological system.
To make a diagnosis based on the neurological findings, observations of the patient's behaviour
and the history provided.


The examiner will have instructed the patient as follows:
The candidate is to do a neurological examination of your legs and ask you to stand and
walk.
Be polite, calm and cooperative. Exhibit a lack of concern for your condition.
Although your spouse is presently in court facing charges linked to his gambling debts, and
you cannot walk, behave in an unconcerned manner.
If asked directly, you say that you are a little worried for your spouse but have no concerns
for yourself. You have confidence in the hospital that they will be able to help and you will get
better.
Candidates are not required to ask any further history from you, and will be directed away
from that course by the examiner, should they do so.
During the examination, which involves tests of movement and coordination of your legs,
follow the candidate's instructions in a straightforward way while you are on the examination
couch you are not required to simulate any dysfunction or discomfort.
When requested, you are able to lift your legs, and sit over the edge of the examination
couch, but you cannot stand, even with support, and firmly decline to walk.
You will not need to ask any specific questions to the candidate.

EXPECTATIONS OF CANDIDATE PERFORMANCE
When the candidate commences any sensory examination, or asks to do this, the examiner will
say that sensation does not need to be tested.
The candidate must test: passive and active movement, power, tone, reflexes, coordination and
should attempt to test gait. If the candidate attempts to test the plantar reflex, indicate this is
down-going.
Tone, coordination and reflexes will be normal.
Inconsistent findings should be noted by the candidate: normal active and passive move-
ments in supine and sitting position, but patient unable to stand or walk, aided or unaided.
At six minutes, instruct the candidate to stop the examination, ask the candidate to summarise
the findings, and ask the following questions (appropriate answers in brackets):
'What would you expect to find on sensory examination, given your findings thus far?

(Normal sensation)
'What is your likely diagnosis?' (Somatoform conversion disorder or similar term)
'What has led you to that conclusion?' (Physical findings inappropriate for organic illness)
479
089
Performance Guidelines



Ifthereistime,andthecandidatehasnotcommentedonthesefeatures,theexaminercouldask:
Whathaveyounoticedaboutthepatient'sattitudeandgeneralbehaviourandwhat
doesthatsignify?'
What is your understanding of the psychological reasons for this patient' s
presentationatthistime?'
Thecandidateshouldbeableto:
Performasystematicmotorexaminationofthelowerlimbs.
Recognisethepresenceofincongruousaffect,beingblanddisconcern(' belleindifference' ),
anditssignificancefordiagnosis.
Formulate a likely diagnosis, being a physical problem developing in an individual under
stress: conversion disorder, somatoform disorder, abnormal illness behaviour, sick role
behaviour. Malingering is not an acceptable diagnosis because there is no personal gain.
Anxiety/stressorothersuchdiagnosisbyitselfisnotanacceptablediagnosis.
Demonstratefamiliaritywithtypicallyassociatedfindingssuchasnormalsensoryexamination
findings.
Utiliseanonjudgmentalapproach,inthefaceofabnormalillnessbehaviour.
Hypothesise that the 'belle indifference' and physical disability are defences against an
overwhelmingemotionsuchasanxiety,angerorshame.

KEYISSUES
Abilitytoconductanappropriatefocusedneurologicalexaminationofthelowerlimbsand
identifyasomatoformconversiondisorderwithabnormalillness(sickrole)behaviour.

CRITICALERROR
Failuretoconductathoroughneurologicalexaminationasinstructed

COMMENTARY
This station assesses the ability of the candidate to recognise abnormal illness behaviour, to
correctlyidentifyconversiondisorderandalsoconductanexaminationofthelowerlimbs.Itisan
integrated station in that it is assessing both clinical skills in neurological examination and
recognitionofapsychiatricsomatoformdisorder.Itisthusunacceptableforthecandidatetodoa
cursoryorincompleteneurologicalexaminationandequally,itisunacceptableforthecandidate
to conclude the problem is 'stress-related', 'an anxiety disorder' or other such ill-defined
diagnosis.Useoftermsthatareinpsychiatricclassificationschemes,otherthanthemostrecent
versions of the International Classification of Diseases ICD10 or American Psychiatric
AssociationDSM-IV,suchashystericalconversion,abnormalillnessbehaviourandsick
role behaviour are acceptable. Stronger candidates may present a more sophisticated
diagnosis with formulation, thus correctly linking the conversion disorder to the unresolved
emotionalconflictsaroundtheimpendingfraudchargesandthecandidate'sextremeshameand
anxietyinregardtothis.
480
089
Performance Guidelines



The common feature of somatoform disorders is the presence of one or several symptoms or physical signs that
suggest an organic or physical illness but which are not explained by any medical condition, or side effects of any
medication or substance, or by another psychiatric disorder (e.g. schizophrenia or panic disorder). The symptoms
must cause significant distress, impairment or predicaments in the patients social and occupational functioning.
These disorders are common and typically first present in general medical or neurological settings. Presentations
vary from mild to severe and may be symptom-focused (hypochondriasis) or sign-focused (conversion disorder).
This patient has a conversion disorder. Her symptoms are confined to the voluntary central nervous system. The
acute disruption in her ability to walk was not associated with any known infection, trauma or physical injury and does
not conform to any known neurological damage pattern. Psychological factors associated with the drama and turmoil
of her husband's court appearance are highly relevant. Whilst multiple sclerosis, myasthenia gravis and idiopathic
periodic paralysis could be considered, they are improbable.
The psychological pathogenesis of a conversion symptom is that the individuals somatic symptoms represent a
symbolic resolution of an unconscious conflict, thus reducing otherwise overwhelming affects (anger/rage, anxiety,
depression, psychosis) and hence keeping the conflict out of conscious awareness ('primary gain'), but at a price.
The external benefits of the symptom/illness behaviour may include avoidance or exemption from anxiety-provoking
or threatening life experiences (e.g. a court appearance, an exam, a wedding, a job interview), an escape from
responsibility, or financial compensation ('secondary gain'). Unlike malingering or the deliberate feigning or faking of
symptoms of illness, which is done consciously and with intent, in conversion disorder the motivation is unconscious
and unintentional.
Typically the neurological findings are bizarre and atypical and do not conform to any known neurological disease,
but may reflect the individual's beliefs about how neurological disease may present. Recent functional magnetic
resonance imaging and positron emission tomography studies of lower limb psychogenic paralysis, show activation
of inhibitory centres in the orbitofrontal and cingulate gyrus areas of the brain with associated nonactivation of the
primary motor cortex, when patients were actively trying to move paralysed limbs. Psychogenic paralysis is worse
when patients consciously try to move a paralysed limb and are attending to the task, but improves when their
attention is distracted.
Previous followup studies with conversion disorders suggested that 30%may subsequently develop organic central
nervous system disease. With more thorough examination and modern sophisticated investigative technologies
missed organic disease may occur in less than 10%nowadays. 'La belle indifference has no diagnostic validity and is
nota criterion for diagnosis, nor is there any association with histrionic (or hysterical) personality traits, or repressed
sexual conflict in the genesis of most instances of conversion disorder.
Diagnosis depends on a careful history and linking a significant life event or interpersonal stress temporally to the
onset of the symptoms. Previous episodes of conversion disorder, alexithymia (lack of psychological mindedness).
increasing age, lower social class, a family history of physical illness and hence role models and a fear of
stigmatisation if psychological disorder is acknowledged, may all predispose to a conversion disorder
481
Performance Guidelines



Many conversion disorders resolve spontaneously over a short period of time, especially if the onset was acute, with
no specific treatment other than explanation and targetted suggestion (after a thorough history, examination and
appropriate investigative workup). Other patients will need active rehabilitation, either as inpatients or as an outpatient,
with physiotherapy as the mainstay of treatment. Patients with chronic or multiple sensorimotor disturbances may
need treatment in specialised multidisciplinary units or psychiatric units. Patients should be told that their symptoms
are real and genuine and that their neurological deficits result from loss of conscious control over the affected function
due to a neurochemical disturbance. An expectation of full and complete recovery is provided in conjunction with
multimodal therapy, which may include cognitive behavioural treatments antidepressants and physiotherapy. Nontoxic
technologies such as ultrasound and transcranial or direct muscle magnetic stimulation may produce dramatic 'cure' of
paralysis.
An essential part of the treatment is a therapeutic alliance and rapport that allows the patient to recover with dignity
and no loss of face. The underlying affects may then emerge and become more florid and obvious, as the weakness or
other physical symptoms improve. These patients have exguisite somatic sensitivity and may develop side effects just
from reading the manufacturer's package inserts from any medication that is prescribed. Adjunctive psychotropics,
usually an SSRI antidepressant and/or an atypical antipsychotic, should usually be prescribed in iow fractional
dosages initially and increased very gradually to the normal therapeutic range. Up to 80%of patients make a complete
or substantially complete recovery from an individual episode, but 50%may relapse within 5 years.

482
483
3 Clinical Management (M)


3-A:ManagementObjectives:Therapeutics,
Prevention,&PublicHealth


AlanTRose,MichaelRKidd,andRonaldMcCoy
' Di seasehassoci al aswel l asphysi cal ,chemi cal andbi ol ogi ccauses.'
HSigehst(1891-1957)
Thetermmanagementcanbeusedbroadlytodescribewhatthedoctordoesoncethehistory
and physical examination have been completed. At that stage, a firm diagnosis may not have
been reached so that the next step in management may be to proceed with investigations or
referral.Someofthe illustrative managementcasesinthisbook begin withastatementofthe
diagnosis which is provided to the candidate. Others provide sufficient information for the
candidate to formulate diagnostic and management plans. In others the information provided
directs the candidate to undertake investigations to facilitate diagnosis, or the patient may be
returningforasecondconsultationwithresultsofinvestigationsnowtohand.
Once the cause of the clinical presentation has been fully identified and treatment is about to
begin,theemphasisisnowontheformulationof amanagementplanbythecandidate,which
mayincludepatienteducationandreassurance,adviceandcounselling,prescribingmedication,
recommendingaprocedure,onwardreferral,arranginghospitaladmission,preventivecare,and
followup. Separation of diagnostic and management phases allows parts of what would be
regardedasalongcase'innormalclinicalpracticetobecompletedwithinthetimeallowed.
Managementwillofteninvolveothersbesidethepatient:aparent,spouse/partner,carer,other
familymembersorothers,suchasemployers.Theethicalboundsofconfidentialitymustalways
bekeptinmind.Incomplexcases,theclinicianmaybeconfrontedwithnumerousconditionsto
dealwithsomeofwhichmaybeself-limiting,someinsoluble,andotherswithalowpriority.Thus,
one may have to use techniques of 'selective attention' and 'selective neglect' whereby a
consciousdecisionismadetofocusonsomeproblems,butnotonothers,puttingasidesomefor
exploringatalaterconsultation.
OBJECTIVESOFTHEMANAGEMENTPHASEOFTHECONSULTATIONARETO:
treatappropriatelythepatientandpresentingcondition;
educatethepatientaboutthecondition;
involvepatientsasfaraspossibleinthemanagementoftheirownconditionsandailments;
achievecomplianceintherapy;
emphasisepreventiveopportunities;and
provideappropriatesupportandreassurance.
484
3-A
Management Objectives: Therapeutics,
Prevention, and Public Health


THE MANAGEMENT PROCESS
The following guidelines provide a sequence of steps for the management process. Not all are appropriate to every
case, and there will be different emphases according to the nature of the clinical problem.
The doctor should:
Tell the patient the diagnosis.
~ The medical term as well as the common or lay name should be given (e.g. Herpes zoster/shingles). An offer to
write the medical term down should be made. The use of medical abbreviations should be avoided (e.g. AMI).
Reassure the patient if the condition is minor, or advise the patient of appropriate serious concerns in a
nonthreatening and supportive way.
~ An anxious patient may not absorb detailed information adequately before being given time to consider the
implications of an important or serious diagnosis
Establish the patient's knowledge, understanding and attitude regarding the condition. ~ Patients are usually
reticent to admit ignorance of a condition and need encouragement to make inquiries (because of concern about
wasting the doctor's time).
Educate the patient about the condition.
~ Correct any incorrect beliefs.
~ Supplement existing knowledge to a level appropriate to the needs of the patient and the doctor.
~ The use of a diagram, model, or the patient's X-rays, ultrasounds or scans, will often facilitate effective
communication.
Answer the patient's questions. These may reveal misunderstandings which require further explanation. The
doctor should exhibit tolerance to repetition of questions, and be prepared to repeat or complete the provision of
information at a subsequent consultation.
Propose therapy within an appropriate timeframe.
~ Immediate, including when no action is required. Hospital admission may be the most important action to be
taken.
~ Longterm, if the illness is chronic or recurrent.
~ Preventive, which may be specific and may require lifestyle change.
Refer as required to a medical specialist or allied health professional. The standard of written referrals is often
inadequate. Referrals can be given to the patients to carry or sent separately. Enclosing copies of relevant
investigations and other medical reports is advisable. When considering the degree of urgency of the referral it
should be remembered that few patients feel they have minor problems in which long delays are acceptable.
Candidates in the AMC clinical examination need to recognise that referral to a specialist is not sufficient action,
unless the candidate indicates why the referral is necessary, and what action the specialist is likely to take.
Supplement and reinforce the information already given during discussion by providing written instructions, or by
supplying leaflets and brochures.
Counsel the patient or relative as required (see also Section 1-A).
Arrange followup this may include an offer to see another member of the patients family, with the patient's
consent. Patients should leave the consultation with a clear understanding of whether the doctor wishes to review
this episode of illness, by what means and when.
485
3-A
Management Objectives: Therapeutics,
Prevention, and Public Health


If investigations have been ordered, it is important for the doctor to review these, and to inform the patient of the
results, at a subsequent consultation, or by telephone or letter. Unexpected adverse results should be conveyed by
further interview, rather than by telephone. Overlooking to inform a patient of serious adverse results can have
medicolegal complications, even when the patient has been instructed to ring to check for results and failed to do so.
THERAPEUTICS
Therapeutics is the selection and use of pharmaceutical agents in the
treatment and prevention of ill health and in the maintenance of an
individual's health status.
Australians are fortunate to have a Pharmaceutical Benefits Scheme
which provides federal government subsidies to patients for prescribed
approved medicines (outside of public hospitals) by registered medical
practitioners who hold a preserver's number issued by the federal
health department. Costs to patients are further reduced by
concessions to certain income and age groups and ex-service
personnel 'safety net' levels also apply to individual's/family's annual
expenditure on prescribed medicines.
Candidates for the AMC examinations are advised to be familiar with
the 'Schedule of Pharmaceutical Benefits' handbook updated three
times a year by the Australian government and issued free to registered
doctors, dentists and pharmacists. It contains a list of all subsidised
pharmaceuticals classified by disease categories, similarities of
actions, generic and trade names, form, prescription quantities and
their cost to the government.

The use of pharmaceuticals in
therapy usually begins once a
positive diagnosis has been
made. Exceptions are where relief
of severe pain is necessary
before the diagnosis has been
established or proven (renal
colic, biliary colic), when a
therapeutic trial using medication
can confirm a strongly suspected
diagnosis (gout, polymyalgia
rheumatica), or when medication
is given before a diagnosis has
been confirmed because early
treatment is necessary to avoid
serious, even life-threatening
consequences (temporal arteritis,
meningococcal septicaemia).
Some items require government approval before prescribing because of their high cost or risk of adverse reactions.
All these considerations have an effect on what pharmaceutical item is selected by the doctor for the treatment of a
particular condition.
Candidates are also referred to a series of publications entitled 'Therapeutic Guidelines', each covering a different
body system. They contain regularly updated advice from dedicated consensus groups on the current therapy for most
diseases, and are invaluable aids.
Australians also self-medicate with 'over-the-counter' items available from pharmacies and health food or natural
lifestyle outlets. Knowledge by the doctor of the use of these substances by patients can assist in understanding the
patient's attitude to sickness and health, awareness of possible drug interactions and the possibility of noncompliance.
486
3-A
Management Objectives: Therapeutics,
Prevention,andPublicHealth


Patient perception of the effectiveness of medications (e.g. the use of antibiotics for viral
infections),variesmarkedly.Patientsoftenattributetheirrecoverytomedicationsratherthanto
spontaneousresolution.Also important inconsidering drugtherapy,istheneedto treatmore
than one condition in the same patient (e.g. a diabetic patient who is hypertensive and has
hyperlipidaemia). Polypharmacy is common in older age groups where several chronic
conditionsarebeingtreatedsimultaneously,orwhenapatientonlongtermmedicationdevelops
an acute illness which demands additional medication. Patients may be confused when
medicationsareadded,existingregimensarealteredordosageschanged.Patientsmayalso
beuncertainwhichoftheirmedicationsrelatestowhichoftheirailmentsandbemistakenabout
the timing or frequency regimens of their medication. These shortcomings need to be
understoodby thedoctor whomustinstructthe patientcarefully. Thismayinclude writingthe
instructionsdown.Thesituationiscompoundedbypatientsattendingmultipledoctors.Patients
will not necessarily volunteer what medications they are on to a second doctor, even when
referredforspecialistcare.Thecreationofmedicationlistsonapatient'scomputer-generated
medical record has not replaced personal communication and enquiry about medication from
thedoctortothepatient.
Pharmacists have limited capacity to detect inappropriate prescribing because patients use
different pharmacies, and because it is unethical for a pharmacist to ask a patient what
diagnosishasbeenmadebythedoctor.
Iatrogenic illness has steadily increased because of polypharmacy, drug interactions, and
patient confusion over medication dosage, together with adverse reactions, allergies and
idiosyncrasieswhichmaybeassociatedwiththeuseofanymedication.
TAKINGA'DRUG'HISTORY
Diagnosticconsultationsoftenrequirea'drug'history,asfollows:
Doyoutakenoworinthepastanyprescriptionorover-the-countermedicines?'
' Whatfor,andweretheyhelpfulordidyouhaveanyside-effects?'
' Doyouhaveanyallergiestomedications?'
Ifahistoryofallergyisgiven,establishdetailsandseveritytodetermineriskofanaphylaxis,for
exampletopenicillin.
Always consider whether reactions, side effects or drug interactions may be contributing to
symptoms (for example, asthma, claudication or cardiac failure from Beta receptor blocking
agents; or cough from ACE inhibitors). Of particular importance are the effects of some
antibiotics on the efficacy of oral contraception, and drugs which accentuate or decrease
anticoagulantactionsofwarfarin.TheproprietarypublicationMIMS,updatedsixtimesannually
andcirculatedtomedicalpractitionersisaveryusefulguidetosucheffects,asaredatabases
onofficepersonalcomputers.
Assessmentofknowledgeoftherapeuticsfrequentlytakesplaceinmanagementconsultations
Theuseofpharmaceuticalsintherapyusuallybeginsonceapositivediagnosishasbeenmade.
Exceptions are where relief of severe pain is necessary before the diagnosis has been
establishedor proven(renalcolic,biliarycolic),whenatherapeutictrialusingmedicationcan
confirmastronglysuspecteddiagnosis(gout,polymyalgiarheumatica),orwhenmedicationis
given before a diagnosis has been confirmed because early treatment is necessary to avoid
serious,evenlife-threateningconsequences(temporalarteritis,meningococcalsepticaemia).
487
3-A
Management Objectives: Therapeutics,
Prevention, and Public Health


The first step is for the doctor to select the correct therapeutic agents and name them by generic or proprietary name
and then comply with any government restrictions on use decide on dose, form, route and time of administration and
ensure that a sufficient supply (which may include repeats) is ordered. These decisions are either written or processed
onto personalised numbered prescription forms supplied by the government. These are the instructions which the
patient takes to a pharmacy to obtain the medicine. As emphasised above it is the doctor's responsibility to be aware of
significant side effects (unwanted symptoms from the drug, potential adverse reactions or drug interactions), and to
advise patients or relatives accordingly. It is at the doctor's discretion as to how much of such information should be
discussed with individual patients, but pharmacists are required to offer this information to patients, sometimes in
printed form.
A major objective of the doctor in therapeutics is to achieve patient compliance in the use of medication. The doctor's
function of prescribing medication is a complex process dependent not only on knowledge of available therapies for
specific diseases, but also on aspects of patient behaviour.
Often a wide range of therapeutic agents is applicable to a single condition (e.g. hypertension) where therapeutic
pharmacology is complex. Fifty different generic products (classified into 22 different categories) are listed on the PBS
for the treatment of hypertension. Most of these agents have several proprietary names. In the MIMS therapeutic
classification index, 21 different system and function groupings are listed, each with numerous (up to 14)
subcategories. The pharmaceutical companies employ many strategies for marketing their particular product to
doctors. More authoritative and referenced information about new drugs, indications, side effects, adverse reactions
and interactions is available in 'Australian Prescribe? (also online) circulated free to all doctors in Australia by the
Commonwealth Government.
Drug usage and dosage need modification in the following circumstances:
Pregnancy drugs potentially harmful to the fetus, particularly those with teratogenic potential, must be avoided.
Children correct dosage is especially important in infants and small children and must be individualised and based
on weight.
Elderly patients drug tolerance is reduced increasing risks of overdosage (e.g. postural hypotension from
antihypertensives).
Impaired organ function especially liver failure or renal insufficiency.
Previous portal-systemic shunting operations.
The most commonly prescribed drugs cover a wide range, and include blood lipid-lowering agents, antiangina
agents, antihypertensives, ACE inhibitors, proton-pump inhibitors, nonsteroidal anti-inflammatory drugs (NSAIDs),
bronchodilator aerosols, diuretics, sedatives and anxiolytics, together with antibiotics.
488
3-A
Management Objectives: Therapeutics,
Prevention, and Public Health


PUBLIC HEALTH MANAGEMENT
Medical practitioners have a central role in public health and the prevention of disease
Tobe effective in this role, clinicians need to be
opportunistic in offering preventive care when patients present with other problems or concerns;
anticipatory in routinely assessing the preventive care needs of their patients;
proactive in targeting preventive care most intensively to high risk individuals; and
reaching all of their patients, especially those who are least likely to seek out assistance.
This involves looking beyond the individual consultation to the population of patients we serve. For example, to be
effective in immunisation or screening, clinicians must reach a large proportion of patients in their practice or
community.
Todo this effectively can be difficult. Each preventive activity uses up some of the clinicians' available time to spend
with their patients, and therefore each activity must be based upon sound research evidence of what is effective.
Clinicians therefore need to understand which preventive activities are recommended as the costs may outweigh the
benefits when assessed by carefully designed research studies.
Candidates can access Guidelines on Preventive Activities from the Royal Australian College of General
Practitioners website (www.racgp.org.au). These guidelines only include activities of relevance where research has
shown a demonstrated benefit.
One of the challenges for clinicians is to ensure access to preventive care for all their patients. Some groups have
increased risk of diseases because of social or other factors.
The links between poor health and socioeconomic disadvantage include a relationship between mortality, social
class and how connected people are to their communities. The opportunities for health are affected by where people
live, their skills, their communities and lifestyles. Poorer health makes disadvantaged groups major users of general
practice and they are also the lowest users of preventive care services.
Social and economic factors influencing health include: level of education; occupational status; employment status;
income; place of residence and migration.
Candidates need also to understand the special public health and preventive issues facing Aboriginal peoples and
Torres Strait Islanders. The poor health of Aboriginal peoples and Torres Strait Islanders has many causes including
social and economic factors and the history of colonisation, and is also exacerbated by poor access to preventive
treatment and late intervention, with many cases of chronic disease only diagnosed when complications are already
present.
Notification of infectious diseases is an important public health responsibility for all clinicians. Candidates should
know which diseases are notifiable to public health authorities and how they are to be reported. Recent Australian
experience with SARS and related epidemics have emphasised the importance of knowing how to contact local
public health authorities. Candidates should be aware of how to contact relevant public health authorities in the event
of requiring assistance and advice in issues of public safety.

Alan T Rose, Michael R Kidd and Ronald McCoy
489
3-A
ManagementObjectives:Therapeutics,
Prevention,andPublicHealth


3-AManagementObjectives,Therapeutics,
PreventionandPublicHealth
CandidateInformationandTasks
MCAT090-100
90 Acuteright-sidedpainandhaematuriaina25-year-oldman
91 Faecalsoilingina5-year-oldboy
92 Psoriasisina30-year-oldman
93 Temporalarteritisina58-year-oldwoman
94 Acuteidiopathicfacialnervepalsy('BellPalsy')ina40-year-oldman
95 Dysuriaandurinaryfrequencyina40-year-oldman
96 Eclampsiaina22-year-oldprimigravidaat38weeksofgestation
97 Anabnormalglucosetolerancetest(GTT)ina34-year-oldprimigravida
98 Bed-wettingbya5-year-oldboy
99 Acutegoutina48-year-oldman
100 Requestforrepeatbenzodiazepineprescriptionfroma25-year-oldman
490
090
CandidateInformationandTasks



Condition 090
Acuteright-sidedpainandhaematuriaina25-year-oldman

CANDIDATEINFORMATIONANDTASKS
This25-year-oldmanisbeingseeninthehospitalEmergencyDepartmentwithafirstepisode
ofsevereright-sidedabdominalpain.Thepaincameontwohoursearlierandwassosevere
thatthepatientwrithedinagonyunabletorelievehissymptoms.Thepainstartedintheright
sideofhisbackandradiatedintohisrightgroinandtesticle.
Heisnowfreeofpain.Hehashadnopainlikethispreviouslyandhasbeeningoodgeneral
health.
Physicalexaminationfindingsarenormal,exceptthathisurineispositiveforbloodonchemical
testing.Thereisnoloinorothertenderness.
Youhavejustfinishedexamininghim.
\
YOURTASKSARETO:
Determinethemostlikelydiagnosisanddiscussinitialinvestigationswiththeexaminer.
Explainthediagnosistothepatient.
Outlineyourmanagementplan,andanyfurtherinvestigationsrequired,tothepatient.

Youwillnotneedtotakeanyadditionalhistory.Thereisnoneedforyoutoasktheexamineraboutanyother
findingsonclinicalexamination.
ThePerformanceGuidelinesforCondition090canbefoundonpage500
491
091
CandidateInformationandTasks



Condition 091
Faecalsoilingina5-year-oldboy
CANDIDATEINFORMATIONANDTASKS
Mark,afive-year-oldboy,isbroughttoseeyouinageneralpracticesetting,becauseforthepast
sixweekshehasbeensoilinghispants,withincreasingfrequency,withfoul-smellingsemifluid
faeces.Itisnowhappeningalmosteverydayandheisbeingteasedatschool.
Hisparentcannottellyoumuchabouthisbowelhabitsashenowattendstohisowntoiletneeds
whenhefeelslikeit.

YOURTASKSARETO:
Takeafurtherfocusedhistoryfromtheparent.
Asktheexaminerfortheappropriatefindingsonexaminationofthechildwhichwouldbe
relevanttoyourdiagnosis.
Explainyourdiagnosistotheparentandadviseonmanagement.

ThePerformanceGuidelinesforCondition091canbefoundonpage503
092
Candidate Information and Tasks



Condition 092
Psoriasis in a 30-year-old man

CANDIDATE INFORMATION AND TASKS
You are working in a general practice. You are seeing a 30-year-old man who works as a bank teller.
He has consulted you about a rash on the extensor surfaces of both elbows and both knees, over the sternal
and lower back areas, and in the scalp. It first appeared after a motor accident six months ago in which he
suffered a fractured femur. The patient remembers that his father, now deceased, used to be bothered by a
chronic rash.
It has been getting steadily worse over the last few months with some improvement following the use of cream
obtained from the local pharmacist (Egopsoryl TA). This has helped the rash on his body but not on elbows,
knees and in the hair.
Examination has revealed the typical lesions of plaque type psoriasis. The plaques vary in size from a few
mm to several cm. They are raised, pink and covered with a silvery waxy scale. The nails are not affected. The
level of severity for this patient's psoriasis should be regarded as moderately severe.
You are about to discuss the disease and its management with the patient. The photograph shows details of
the skin lesions on the knees.

YOUR TASKS ARE TO:
Explain the nature of his condition to the patient
Advise the patient about management.
CONDITION 092. FIGURE 1.
The Performance Guidelines for Condition 092 can be found on page 507
492
093
Candidate Information and
Tasks



Condi t i on 093
Temporalarteritisina58-year-oldwoman
CANDIDATEINFORMATIONANDTASKS
You are working in a general practice. This 58-year-old woman has consulted you about the
recentonset(twoweeks)ofright-sidedheadachegraduallybecomingmoreandmoresevere,and
which is now constant. Over the last few days, the patient has also had a tight feeling in the
musclesontherighthandsideofthefacewhenchewing.
On physical examination you noted tenderness and tortuosity over the right temporal artery as
illustrated.Itspulsationcannotbefeltaswellasthatofthetemporalarteryontheleft.Therewere
nootherabnormalphysicalfindings.Thepatientisnormotensive.

CONDITION 093. FIGURE 1.
Sideviewofrighttemple
Based on this information you believe that the most likely cause of the patient's symptoms is
temporalarteritis('cranialarteritis'or'giantcellarteritis').
BriefPatientProfile
Married, works as an accountant. Nonsmoker. No significant past or family history except for
occasionalmigraine.HasbeentakingPanadol(paracetamol500mg)fortheheadache.
YOURTASKSARETO:
Explainthediagnosis,anditsimplications,tothepatient.
Advisethepatientaboutmanagementbothimmediateandlongterm.This
couldincludeanyinvestigationsyoubelievearenecessary.

Youdonotneedtotakeanyfurtherhistory.Youhavejustconcludedyourphysicalexamination
andareabouttoadvisethepatientofyourdiagnosisandmanagementplans.
ThePerformanceGuidelinesforCondition093canbefoundonpage510
493
094
CandidateInformationandTasks



Condi t i on 094
Acuteidiopathicfacialnervepalsy('BellPalsy')ina40-year-oldman

CANDIDATEINFORMATIONANDTASKS
You are consulting in a general practice. You have just completed taking a history from and
examininga40-year-oldmanwhoisveryupsetbecauseofthesuddenonsetofparalysisofhis
face.
Hefeltdiscomfortbehindtheleftearlastnightandonwakingtodayfoundthattheleftsideofhis
face would not move. He arranged an urgent appointment with you. Examination confirms a
near complete left 7th cranial nerve facial palsy of lower motor neurone type. The
accompanying illustrations show the findings. There are no other abnormal neurological or
othersignsincludingnormalearcanalsandtympanicmembranes.Thepatientsparotidsalivary
glandsshownoabnormality.Youhavemadeaconfidentclinicaldiagnosisofacuteidiopathic
facialnervepalsy(BellPalsy')
YOURTASKSARETO:
Explaintheproblemtothepatient.
Advisethepatientofthemanagementyouwouldadvise.
Respondtoanyquestionsaskedbythepatient.Thepatientisveryupsetandconcerned
that this may be a stroke, and wishes to know the cause, whether recovery will occur,
whattreatmentandtestsheshouldhaveandhowlongitwilltaketorecover.

Timeshouldnotbewastedtakingfurtherhistoryoraskingforanyotherphysicalfindings.The
mainissuestobeaddressedarepatientcounsellingandmanagement.
Smiling Blowingoutcheeks
CONDITION094.FIGURES1-4.
Theseillustrationsshowhisfacialappearanceinreposeandwithsmilingandmovements.
ThePerformanceGuidelinesforCondition094canbefoundonpage512
494
095
CandidateInformationandTasks

Condition 095
Dysuriaandurinaryfrequencyina40-year-oldman
CANDIDATEINFORMATIONANDTASKS
This40-year-oldpostmanismarriedwithtwochildrenandhasconsultedyoutodayinageneral
practicesettingcomplainingofthegradualonsetofdysuriaandfrequencyofmicturitionoverthe
last three days. There has been no urethral discharge and no history of extramarital sexual
contact. On examination the patient is afebrile and you found no abnormality on examination,
includingrectalexaminationoftheprostate.Amidstreamurinespecimenwascollectedandthe
followingofficelaboratorytestsweredoneontheurine
Dipstixpositiveforprotein,leucocytesandnitrites:negativeforblood,glucoseandketones.
Microscopyofuncentrifugedspecimenshowslargenumbersofleucocytesandbacilli.
The patient usually keeps in excellent health. He is aware that he is sensitive to penicillin but
otherwisehispasthistory,familyhistory,habits,anduseofmedicationhavenorelevancetothis
problem.
YOURTASKSARETO:
Advisethepatientofyourdiagnosis.
Advisethepatientofyourimmediatemanagement.
Discusstheconditionandansweranyquestionsthepatientmayask.
CONDITION095.FIGURE1.
Urineteststrip
ThePerformanceGuidelinesforCondition095canbefoundonpage519
495
496
096-097
CandidateInformationandTasks



Condition 096
Eclampsiaina22-year-oldprimigravidaat38weeksgestation

CANDIDATEINFORMATIONANDTASKS
This 22-year-old primigravida has been seeing you in a general practice clinic for her shared
antenatalcaresinceearlyinherpregnancy.Sheisnowat38weeksofgestation.
The pregnancy has been progressing perfectly normally until now. Whilst in the waiting room
alongwithhermotherwaitingtoseeyouforherroutineantenatalvisit,shehashadagr and
mal fit.Shehadbroughtaurinespecimenwithhertotheappointment.

YOURTASKSARETO:
Takeanyfurtherrelevanthistoryyourequirefromthemotherofthepatient,whoisinthe
waitingroom.
Asktheexamineraboutthespecificfindingsyouwouldlookforongeneralandobstetric
examinationandanyofficetestresultswhichshouldbeavailabletoyou.
Advisethemotherofthepatient,inlayterms,ofthediagnosisandthesubsequent
managementyouwouldadviseforherdaughter.

ThePerformanceGuidelinesforCondition096canbefoundonpage522

Condition 097
Anabnormalglucosetolerancetest(GTT)ina34-year-oldprimigravida

CANDIDATEINFORMATIONANDTASKS
Thispatientisa34-year-oldobeseprimigravidawhomyouaremanaginginacountrygeneral
practice.Shehashadascreeningglucosetolerancetestperformedat28weeksofgestation.
Thisrevealedafastingbloodglucoseof7.5mmol/Landatwohourlevelof9.5mmol/L(Normal
levelsfasting<5.5mmol/L;twohour<8.0mmol/L).
Progressofherpregnancyhasuntilnowbeennormal.Nootherinvestigationshavebeendone
apartfromroutinescreeningtestsatthefirstantenatalvisitwhichwereallnormal.

YOURTASKSARETO:
Takeanyfurtherrelevanthistoryyourequire.Thisshouldbelimitedto1-2minutesonly.
Asktheexaminerforthefindingsyouwouldexpectongeneralandobstetricexamination.
Advisethepatientofthediagnosisyouhavemade.
Advisethepatientofthemanagementyouwouldgiveintheremainderofthepregnancy.

ThePerformanceGuidelinesforCondition097canbefoundonpage525
497
098-099
CandidateInformationandTasks



Condition 098
Bed-wettingbya5-year-oldboy
CANDIDATEINFORMATIONANDTASKS
Johnny,afive-year-oldboy,isbroughttoseeyouinageneralpracticebyhismotherbecauseofa
bed-wettingproblem,whichoccursnightly.Hehas
beenfullycontinentbydaysincehewasthreeyearsold:andhas
previouslybeentreatedunsuccessfullywithnightlyamitriptyline(Tryptanol).
Thewettingexasperatedhisparentsinitially,buttheynowacceptthatitisinvoluntaryandboth
parentsarekeentohelphiminanywaypossible.
YOURTASKSARETO:
Askthemotherforanyfurtherrelevanthistory.
Telltheexaminerwhatrelevantexaminationfindingsyouwouldseek.
AdviseJohnny'smotherhowyouwillfurtherassessandmanagehiscondition.

ThePerformanceGuidelinesforCondition098canbefoundonpage528

Condition 099
Acutegoutina48-year-oldman

CANDIDATEINFORMATIONANDTASKS
You are about to see a 48-year-old taxi driver who consulted you earlier today in a general
practice setting about continuous, severe, worsening, throbbing pain in the right first
metatarsophalangealjoint,whichcommencedtwodaysago.Thejointwasswollenandfelthot.
Theoverlyingskinwasredandshinyandthejointwasexquisitelytender.Thereisahistoryof
previous attacks over the last two years. These have been diagnosed as gout. Each time
responsetotreatmentwassatisfactory.Youtookbloodforserumurateestimation.Thepatient
hasreturnedtofindouttheresult(whichwas0.74mmol/L)andfortreatment.
Thenormalserumuraterangeformalesis0.20-0.45mmol/L.Urinalysisisnormal.
Thepatienthasalwayskeptingoodhealthapartfrommildhypertensiondiagnosedtwoyearsago
forwhichhetakeshydrochlorothiazide.Overthepasttwodayshehastakentwoorthreeaspirin
tabletsforthepain.
YOURTASKSARETO:
Advisetreatmentoftheacuteattack.
Discussfurthermanagementofhiscondition.
Thereisnoneedforyoutotakeanyadditionalhistoryorperformanyexamination.
ThePerformanceGuidelinesforCondition099canbefoundonpage531
498
100
CandidateInformationandTasks



Condition 100
Requestforrepeatbenzodiazepineprescriptionfroma25-year-oldman

CANDIDATEINFORMATIONANDTASKS
Youareworkinginageneralpractice.Yousawthispatientforthefirsttimeoneweekagoand
providedaprescriptionforhisusualsleepingtablet,thebenzodiazepineoxazepam(Serepax)
30mgdaily,25tablets.Atthattime,youweresatisfiedtherewerenocomorbidproblemssuch
as depression. The patient has returned today for another prescription. The patient's mental
stateisunchanged.

YOURTASKSARETO:
Evaluatethesituationbytakingafocusedhistory.
Outlinetothepatientthenatureoftheproblemyouhaveidentifiedandproposed
management.
Answeranyquestionstheexaminerasksyou.

ThePerformanceGuidelinesforCondition100canbefoundonpage534
499
3-A
ManagementObjectives:Therapeutics,
Prevention,andPublicHealth

3-AManagementObjectives,Therapeutics,
PreventionandPublicHealth
PerformanceGuidelines
MCAT090-100
90 Acuteright-sidedpainandhaematuriaina25-year-oldman
91 Faecalsoilingina5-year-oldboy
92 Psoriasisina30-year-oldman
93 Temporalarteritisina58-year-oldwoman
94 Acuteidiopathicfacialnervepalsy('BellPalsy')ina40-year-oldman
95 Dysuriaandurinaryfrequencyina40-year-oldman
96 Eclampsiaina22-year-oldprimigravidaat38weeksofgestation
97 Anabnormalglucosetolerancetest(GTT)ina34-year-oldprimigravida
98 Bed-wettingbya5-year-oldboy
99 Acutegoutina48-year-oldman
100 Requestforrepeatbenzodiazepineprescriptionfroma25-year-oldman
500
090
PerformanceGuidelines



Condition 090
Acuteright-sidedpainandhaematuriaina25-year-oldman

AIMSOFSTATION
To assess the candidate's knowledge of the natural history of urinary calculi, and ability to
diagnoseandmanageapatientwitharecenthistoryofrenalpainassociatedwithastoneinthe
ureter.

EXAMINERINSTRUCTIONS
Theexaminerwillhaveinstructedthepatientasfollows:
You are aged 25 years and about two hours ago you developed severe right-sided back and
abdominal pain. The pain was gripping, severe and constant, with episodes of increased
severityeachfewminutes.Thepainextendedfromthelointoyourrighttesticle.Youhavebeen
writhing in agony with this pain, and never experienced anything like it before. Fortunately it
gradually eased and ceased after about 30 minutes. You have come to the Emergency
Department and have seen the doctor. The doctor has listened to your story and taken a full
physicalexaminationandyouhavegivenhimasampleofurineforanalysis.
The pain has now settled. The doctor will explain to you the diagnosis and proposed
management.Youhavenotheardaboutrenal/uretericcolicorastonepassingfromthekidney
downtheureter(thetubebetweenthekidneyandbladder),buthaveheardofkidneystones.
Questionstoaskunlessalreadycoveredandappropriateresponsesfromdoctor/candidate
(Answersinparenthesesafterthequestion):
Do/needtobeadmittedtohospital?' (Notatthisstage)
' Willthepaincomeback?' (Possibly,ifthestoneisstillintheureter).
' WhydidIgetthestone?' Askthisifthecandidatesuggestsastoneisthecauseofthe
pain.
HowwillIknowifIhavepassedthestone?' (Youwillstrainyoururine).
' Whathappensifthestonedoesnotpass?' (Aninstrumentmayhavetobeinsertedto
retrieveit).
'WhendoIseeyouagain?' (Followupinacoupleofdaysforinvestigationresults).
Thecandidateshouldexplainthatyouhaveasmallstonethatispassingdownyourureter.It
maytakeonetotwodaystopassthestone.Youwillbegivenstrongpainkillersforthepainin
caseitrecurs.Moststonespassspontaneously.Youareunlikelytohavefutureproblems,but
testsonyoururineandbloodwillbedonetocheckthis.
Theinvestigationsrequiredarelikelytoinclude:
Cultureoftheurinetoexcludeinfection.
Anultrasoundofthekidneymaybeperformedlookingforevidenceofblockageoftheureter
causedbythestone.
Imagingbycomputedtomography(CT)todefinethesizeandsiteofthestone.
Examinationofthestoneafteryouhavepassedittodeterminethetypeofstonepresent.
Otherbloodtestsandspecialistreferralmayberequiredinfollowup.
501
090
Performance Guidelines



EXPECTATIONS OF CANDIDATE PERFORMANCE
The candidate should indicate the pain is almost certainly due to renal colic, due to the fact a
small stone is being passed down the right ureter. No need for pethidine now as pain has gone.
If pain returns, give pethidine or Panadeine forteor a nonsteroidal antiinflammatory drug
(NSAID).
Give patient a brief description (draw diagram) of the anatomy of the kidneys, ureters, bladder
and urethra.
Pass all urine into a container and strain (save any stones found for analysis).
Imaging by computed tomography (CT) abdomen or plain X-ray, to identify the site of the stone
and use for subsequent assessment.
Check urine for infection.
Check serum uric acid, and serum calcium, urea and creatinine.
Ultrasound to see if hydronephrosis is present is an acceptable and appropriate test, but
definitive imaging is by helical abdominal CT which will pick up very small stones.
Advise high fluid intake. Refer to urologist if stone is not passed in 48 hours or pain recurs and
worsens. He may need an open operation, endoscopic removal, or ultrasound destruction if not
passed spontaneously (depends on size and site of stone).
Followup to check progress.

KEY ISSUES
Diagnosis of renal 'colic', probably due to a stone in right ureter.
Explanation of the problem to the patient incorporating rapport with the patient and
communication skills.
Providing an appropriate plan of management (including provision for further pain relief).
Appropriate choice of investigations.

CRITICAL ERRORS
Failure to make a diagnosis of renal (ureteric) colic.
Failure to arrange appropriate investigations.

COMMENTARY
Renal and ureteric pain ('renal colic') frequently accompanies urinary calculi passing from kidney
to bladder via the ureter. Although the term, renal 'colic', is hallowed by long usage (as is biliary
'colic'), in neither instance is the pain usually of true 'colicky' type. Renal 'colic' is often an intense,
constant, agonising pain which makes the patient writhe and change position in an attempt to gain
relief, and does not wax and wane intermittently in a sine-wave pattern, rising through a
crescendo of intensity, and then diminishing over a similar period, with regular intervals of relief
from pain in between episodes of cramping pain. This is the pattern of true 'colic', as seen in
intestinal colic or uterine colic. Renal pain can vary in intensity but not with such cyclical regularity.
090
Performance Guidelines



Renal (and ureteric) pain is usually recognised by its character as described above, its site (which can be over a
wide area from posteriorly and laterally in the loin and flank, to the anterior abdomen, iliac fossa and
suprapubically). Pain can also radiate to the penis and testes, and to the upper thigh. Associated urinary symptoms
and the presence of blood in the urine (macroscopic or microscopic) help confirm the diagnosis.
Investigations of presumed renal colic due to stone are by diagnostic imaging. Most urinary calculi (80%) are
radio-opaque so plain abdominal X-ray may be diagnostic (as illustrated), however small stones may be missed
and differentiation from pelvic phleboliths impossible. The preferred investigation is a helical CT without
intravenous contrast (as illustrated), which will pick up any small stones and also identify urinary tract obstructions.
The majority of stones will pass spontaneously, so that treatment is usually expectant while providing pain relief for
recurrence of pain. Indications for intervention are large stones not likely to pass spontaneously, calculi associated
with uncontrolled infection, and persistence of pain without progress.
Radiolucent stones occur with hyperuricaemia and in some types of familial and metabolic calculi. It is prudent in
each patient with a urinary calculus to check for hypercalcaemiato pick up cases of primary hyperparathyroidism
presenting as renal calculi, a common mode of presentation of parathyroid adenoma. Full investigation for a
primary cause is mandatory in patients with a history of recurrent calculi.

502

CONDITION090. FIGURE 1.
Plain X-ray showing opaque ureteric
calculi
CONDITION090. FIGURE 2. Helical CT
showing obstructing right ureteric
calculus
503
091
Performance Guidelines



Condition 091
Faecal soiling in a 5-year-old boy
AIMS OF STATION
To assess the candidate's ability to diagnose and manage the problem of encopresis in a young
child secondary to constipation and faecal retention and to advise the concerned parent on
management.

EXAMINER INSTRUCTIONS
The examiner will have instructed the parent as follows:
The doctor is required to question you further, seeking a possible cause for your child's soiling.
Below are a series of answers to possible questions you may be asked'
You are a very concerned parent about your child's constant soiling over the last six weeks.
This is most unlike him as he was fully toilet trained by three.
He is embarrassed by it, especially at school where the other children are calling him names.
Sometimes he hides his soiled underpants.
This has been occurring for the last six weeks or so. He had an episode three months ago
when his bowel motions were hard and difficult to pass and caused him bleeding and pain
when he went to the toilet. However he seemed to get over that with some laxatives.
There is no abdominal pain.
He has had no vomiting.
His appetite is good and he has a well balanced diet with lots of fruit and vegetables.
He has not lost any weight.
His urine is normal and he has no daytime or evening urinary incontinence.
Except for the recent teasing, he enjoys school and he has lots of friends. His progress at
school is excellent and he enjoys the teaching he has had this year
He has a good relationship with his younger sibling.
His general health is excellent.
He is the elder of two children. The home situation is very stable with both parents very active
in the raising of the children.
Once the doctor has finished questioning you, the examiner will provide examination findings on
your child, then the candidate is required to explain to you what is wrong and the principles of
management.
Questions to ask unless already covered:
'What can we do to treat this?'
'Surely he must smell It when he does It in his pants? Doesn't he know he is soiling
himself?
504
091
Performance Guidelines



Examinationfindingsprovidefindingsspecificallyrequested
ashyboy;
normalheightandweighton50thcentile;
abdomenissoft;
faecalmassesarefeltinthelowerguadrants;
nootherabnormality;
anusappearsnormal,withsomefaecalstainingadjacent;
noanalfissureisapparent;and
onrectalexaminationtherectumispackedwithfirmfaeces.

EXPECTATIONSOFCANDIDATEPERFORMANCE
Thecandidateshouldexplainthat:
The most likely diagnosis is chronic constipation which from the history is most likely
secondary to the probable anal fissure he had three months previously. The constipation
leadstochronicdilatationoftherectumandlowercolon.
Liguefactionofthefaecesleadstosoilingfromoverflow,butfaecalmassesremain.
The process often starts as an anal fissure, giving pain, and the child holds on fearing
defaecationwillbepainful.
Otheraetiologiesshouldbeexplored(e.g.emotionaldisturbancesathomeorschool).None
isapparent,andsoconstipationwithoverflowisthemostlikelydiagnosis.
Management
Empty large bowel by whatever means necessary. Try high dose oral laxative, enema or
suppositories.
Explainthatifthisisnotsuccessfulhemayneedoralgastrointestinallavage(Golytely).
Nextobjectiveistomaintain regularbowelhabit byuseof laxativesandfaecalsofteners.
Therapy needs to continue for many months to allow resolution of the megacolon and to
ensurethatthepassageofmotionsisnotpainful.
Reviewtoensureconstipationisnotrecurring.
Seeregularlytoencourageandsupportparentandchildintheirefforts.
Interactionwithpatient
Explanation with appropriate language to parent to discuss the matters with the child.
Behavioural technigues such as a star chart to reward successful defaecation should be
encouraged.
Suggest regular toileting after meals for a set period of time; an egg timer is suitable for
timing.

KEYISSUES
Explanationofdiagnosis
Initialemptingofrectumandcolon.
Needforprolongedtreatmentandfollowup
505
091
Performance Guidelines



CRITICAL ERROR
Suggesting that sigmoidoscopy or colonoscopy is required at this stage.

COMMENTARY
Constipation in young children is a very common problem presenting to primary care physicians and paediatricians
alike. The great majority of cases are nonorganic, that is they are related to an episode, in many cases associated with
the passage of a hard stool, which may make the child wary of passing a bowel motion subsequently. This can last for
a varying length of time. Constipation can be associated with a mucosal tear or anal fissure which distresses the child,
further compounding the problem. If one is able to recognise this pattern early and treat it with faecal softeners allowing
the fissure to heal, the episode may be short lived.
Major problems may develop, however, if the child remains fearful of going to the toilet because of anticipation of pain
and this compounds the situation leading to chronic constipation, toilet refusal and in many cases overflow encopresis.
This may be more difficult to treat.
Young children may also develop a fear of the toilet during toilet training if they are required to perch on a toilet seat
without support and this also may lead to deferral and subsequent constipation.
In many children, constipation is secondary to an emotional upset or trauma, whether this is at home, school or
elsewhere. Careful enquiry is necessary to seek information indicating that this may exist.
The aims of management are therefore to:
Exclude any possible organic pathology.
Explore any precipitating features.
Provide adequate explanation of the processes involved for both parent and child with a plan of action to alleviate
the problem.
The hallmark of care of these children is a thorough and careful history which, in the majority of cases, will clarify the
probable aetiology. The history should include a thorough enquiry into the child's environment. While organic
conditions like Hirschsprung disease may need to be considered, Hirschsprung disease has usually presented by this
age and usually has a history of constipation from birth, often with a delay in the passage of meconium. However, if
there is doubt or suspicion of this condition, a paediatric surgical opinion should be sought to arrange bowel biopsies.
The history should include a thorough enquiry into aspects of the child's environment including diet, general health,
growth pattern, the family dynamics, progress at school, relationship with peers, and the like. From this information a
likely diagnosis may be evident.
Treatment is essentially careful reassurance and explanation to parent and child of the nature of the condition, if
organic pathology is not suspected. This should include a comprehensive explanation about how constipation has
developed, preferably with illustrative drawings. Toachieve success is very time consuming in resistant cases, and the
doctor's role is to be supportive of the efforts of the parent and child, riding with them the inevitable ups and downs
towards success.
506
091
PerformanceGuidelines

Thebowelhoweverusuallyneedstobeemptiedoftheretainedfaecesinthemostpainlessand
noninvasivemannerpossible.Thismayneedattimeseithersimplefaecalsoftenersormore
vigorous treatment with microenemas, or aperients from above. In severe cases, manual
disimpaction under general anaesthesia may be necessary. Once the bowel is empty the
faecesarekeptsoftwithfaecalsofteners,whichpreferablydonotstimulatethebowel,anda
retrainingprogramisinstigated.Thismayvaryfromchildtochild,particularlyiftoiletphobiaisa
majorissue.Manymethodsareavailableandareoftensuccessful.
Theprogrammesusedaredescribedwellinstandardtextbooks,andareavailableasClinical
GuidelinesfrommostmajorChildren'sHospitals.Parentliteratureisfreelyavailableandforthe
olderchildrenstorybooksaddressingthesituationintermstheycanunderstandareavailable.
Itcannotbeemphasisedtoomuchthattheclinician'smainroleisthesupportofthechildand
family over the period of time required to achieve success. Without this support and
encouragement,theprogrammeisdoomedtofailure.
507
092
PerformanceGuidelines



Condition 092
Psoriasisina30-year-oldman
AIMSOFSTATION
Toassessthecandidate'sknowledgeofpsoriasisanditsmanagement,andtheabilitytocounsel
thepatientaboutachronic,cosmeticallydisturbingcondition,whichcannotbecompletelycured.
EXAMINERINSTRUCTIONS
Theexaminerwillhaveinstructedthepatientasfollows:
Youaresufferingfrompsoriasis.Youhaveascalyskinrashforwhichyouhavejustconsulted
this doctor. The doctor has taken your history and examined you and will now discuss the
conditionwithyou.
You are concerned that it may spread and wonder if a recent serious motor accident was the
cause.Youfindtheailmentandtheointmentsdistastefulbecauseoftheunpleasantsmelland
staining of clothing and you are worried your wife may become infected. You are hoping to
receivereassurancethatitcanbecured.Youwonderwhetheryoushouldseeaskinspecialist.
Questionstoaskunlessalreadycovered:
' Whatcausesthis?'
' Canitbecured?'
' Willitspreadtootherpartsofmybody?'
' Isitinfectious?'
' Canitaffectmyhealthinotherways?'
'CouldIpassitontomychildren?'
Doesithaveanythingtodowithmyaccident?'
'I'veheardthatthereisachemistsomewherewhocancurepsoriasis?'
EXPECTATIONSOFCANDIDATEPERFORMANCE
Approachtopatient
Thecandidateshouldshowinterestandconcern,listentoanddealwiththepatient'sconcerns
andgiveclearinstructions,answeringquestionsdirectly.
Explanationofthediagnosis
Descriptionofwhichpartsofthebodycanbeaffected(extensorsurfacesofelbowsandknees,
sternalandsacralareas,scalpandnailsbutcaninvolveotherpartsoftheskin).
Aetiology, nature,associations,expectedcourse, availabilityoftreatments,prognosis.Good
effectofsunlightifnotoverexposed.Psoriaticarthritiscouldbementioned.
Pointoutthatphysicaloremotionalstresscancauseflareup.Diethasnoeffect.Exerciseand
reductioninalcoholintakemaybebeneficial.Useofsunscreenapplicationsduringsummer.
508
092
PerformanceGuidelines

ManagementImmediate
The candidate should exhibit a general understanding of the principles and modalities of
treatment and that these are applied according to severity. Exact details, generic or trade
names, strengths (except for steroids) and doses are not expected although candidates
shouldindicatethatthesewillbeascertainedifrequired.
The candidate should include an offer to refer to a dermatologist. If this is the only
managementadvicegiven,thepatientshouldaskformoreinformationabouttreatmentand
expectdiscussionofdifferenttypesoflocalcreamsandlotions.
ManagementLongterm
Monitoringofprogressandrepeatsofmedicationsarerequired.Reviewduringflare-upswith
closerinvolvementifsystemictherapybecomesnecessary.Reviewifsecondaryinfectionis
suspected.Liaisonwithdermatologist.
Counselling
Abilitytoachievepatientunderstandingofthechronicity,variabilityanddifficulttherapeutic
natureofthecondition.Despitethis,reassurancethattheconditioncanusuallybebrought
undercontrol.
Thisrequirestheestablishmentoftrustandconfidence,backedupbyclinicalknowledge,a
willingnesstolistentothepatient'sviewsaboutcauseandtreatmentssuggestedbyothers,
andrecognitionofwhenreferraltoadermatologistshouldbemadeusingapatient-centred
aswellasdisease-centredapproach.

KEYISSUES
Approachtopatient:mustlistentoandacknowledgepatient'sconcernsandprovidesupport
andencouragement.
Management: knowledge of principles of different local measures. Extent of treatment
proportionaltoseverity.
Counsellingandexplanationofdiagnosis:mustacknowledgechronicnatureofpsoriasisand
thattreatmentcanbedemandingandwillbeprolonged.Mustbehonestandsupportive.

CRITICALERROR
Failuretoexplainappropriateprinciplesoftreatment.
509
092
Performance Guidelines



COMMENTARY
Aetiology of psoriasis is unknown but there is a familial predisposition. Onset is most commonly between 10 and 40
years but can occur at any age. There is increased epidermal cell proliferation, with vascular proliferation and
inflammation in the upper dermis. Psoriasis affects 2-4%of the population. It is not infective and waxes and wanes in
intensity. It can be drug-induced, e.g. lithium, Beta blockers, chloroquine and hydroxychloroquine; and it can be
precipitated by infections, trauma or emotional stress. The lesions can become secondarily infected. Diet is not a
factor. There is no complete cure but in most cases psoriasis can be reasonably controlled with therapy. Treatment
should be at an appropriate level for the type, site and severity of the condition. Psoriasis can be associated with a
specific type of arthritis, mainly affecting the hands.
In mild cases, emollients or a weak topical corticosteroid may suffice, but disfiguring psoriasis may warrant the use of
antimetabolites or immunosuppressants following the use of more potent topical corticosteroids. Complicated or
difficult cases need specialist care and open lines of communication between general practitioner, dermatologist, any
other medical attendants and the patient.
For mild to moderate plaque psoriasis use topical therapy dithranol, tars, corticosteroids, keratolytics, and
emollients. Occlusive dressings increase their effect.
~ Dithranol an antiproliferative agent is very effective. Salicylic acid can be combined with tars and dithranol.
~ Tars anti-inflammatory but can stain and smell which are disadvantages. Less when in combination (e.g. with
allantoin).
~ Topical corticosteroids are more potent preparations for thicker lesions, large quantities for widespread rash.
~ Keratolytics for lifting and softening thick scale such as sulphur and salicylic acid.
~ Emollients for scaling or irritation. Harsh soaps should be avoided.
~ Calcipotriol (a vitamin D derivative that regulates growth of keratinocytes).
For psoriasis which is widespread, severe, or causing disfigurement or disability, systemic therapy is indicated such
as methotrexate or acitretin or cyclosporin. Phototherapy is also often used by dermatologists.
093
Performance Guidelines
510
Condition 093
Temporal arteritis in a 58-year-old woman
AIMS OF STATION
Toassess the candidate's knowledge of the treatment of temporal arteritis and its most important possible sequel: visual
impairment.
EXAMINER INSTRUCTIONS
The examiner will have advised the patient that she should ask about side effects of steroids and whether some
alternative medication is preferable, as she doesn't like the idea of steroids. She should also ask about further tests such
as X-ray and whether the headache might just be a simple migraine.
Questions to ask unless already covered:
'Could it be a migraine?'
'Are there any (other) complications?'
'Are you sure that my eyesight will be all right?'
'Should I see an eye specialist?'
'Isn't "cortisone" dangerous?' (If 'cortisone' or 'steroids' are recommended)
'What are its side effects?'
'How long will the headache last once treatment lasts?'
'How long will I be on "cortisone"?'
'Can this trouble affect me in any other way?'
EXPECTATIONS OF CANDIDATE PERFORMANCE
This can become an emotive situation for the patient after being informed of the nature of the condition and the
possibility of severe visual impairment.
The doctor should not withhold this information, which should be given with empathy and support. Politeness, respect
and consideration rather than an authoritarian approach should be demonstrated when discussing the threat of
blindness and obtaining compliance with the use of corticosteroids.
The doctor should listen carefully to the patient's queries and provide honest as well as accurate answers. Generating
trust and confidence and giving the correct level of reassurance are also expected.
Initial management plan
An erythrocyte sedimentation rate (ESR) or C-reactive protein (CRP) should be arranged immediately with request for a
same day report (this requires liaison with the patient who should contact the doctor later in the day).
The patient should be commenced on oral prednisolone in high dose at first (60-100 mg). A stronger nonopioid
analgesic than paracetamol should also be prescribed.
Referral to a surgeon with a view to temporal artery biopsy should be discussed and urgent referral to an eye specialist
should be advised.
093
Performance Guidelines
511
Patient Education
The aetiology and prognosis of temporal arteritis are obscure. It is a manifestation of giant cell arteritis.
Confirmation by ESR or CRP (usually markedly elevated) is essential, followed in most cases by biopsy of the
superficial temporal artery for confirmation because of the likely need for medium to longterm corticosteroid therapy.
Commencement of oral steroid therapy before completing investigations is indicated to reduce the risk of visual
impairment, especially in a case of this duration (two weeks).
This patient should be seen again within 48 hours by which time significant resolution of symptoms should have
occurred. Once symptoms are controlled and ESR levels fall, the prednisolone can be reduced to maintenance levels
(5-10 mg three times daily).
The patient should be monitored closely by continuing review of symptoms and serial ESR levels. Resolution may take
up to 2-3 years. Concomitant use of H2 receptor antagonists should be considered in patients with a history of
dyspepsia or peptic ulcer.

KEY ISSUES
Skill in conveying unpleasant news to patient in an honest and supportive manner with guarded reassurance about
the outcome.
ESR or CRP must be ordered with urgent early report requested.
Must commence prednisolone therapy immediately.
Patient counselling and education is required regarding possible biopsy, referral to eye specialist and longterm
nature of the condition.

CRITICAL ERRORS
Failure to request ESR or CRP.
Failure to commence prednisolone therapy.

COMMENTARY
Although this is not a common disorder, the high risk of preventable blindness and response to early treatment makes
it essential knowledge. A highly probable diagnosis is possible on clinical grounds alone. Involvement of the ophthalmic
artery or ciliary arteries may occur causing optic atrophy and blindness. Vision is impaired in about 50%of patients at
some stage. If blindness occurs it is usually irreversible.
Temporal arteritis may follow polymyalgia rheumatica in about 20%of cases. The condition is very responsive to
corticosteroids which should be prescribed in high doses initially.
Maintenance steroid therapy in lower dose over 2-3 years may be required, which raises the possibility of
steroid-induced complications of osteopenia, hypertension, diabetes and changed facies.
512
094
Performance Guidelines

Condition 094
Acute idiopathic facial nerve palsy ('Bell Palsy') in a 40-year-old man
AIMS OF STATION
Toassess the candidate's knowledge of Bell Palsy, its prognosis and its management, and skills in counselling an
upset and anxious patient.
EXAMINER INSTRUCTIONS
The patient has the condition as illustrated. He is very concerned, but if informed and reassured appropriately, will
accept the diagnosis and management plans.
EXPECTATIONS OF CANDIDATE PERFORMANCE
You would expect the candidate to:
Acknowledge the patient's distress about his appearance and to provide support and guarded reassurance,
particularly reassurance that the patient has not had a stroke.
Explain the diagnosis and natural history of the condition. The cause is unknown but is consistent with
inflammatory compression of the facial nerve in the temporal bone (probably viral).
Expected course about 70%of patients completely recover within two months. First signs of recovery appear
within two weeks. About 20-25%take up to six months for full recovery; and 5-10%do not recover by the end of
one year. An older patient age is associated with slower recovery.
Advise about immediate management:
~ Steroids are usually prescribed empirically: prednisolone 40-80 mg daily for three days then taper off and cease
over the next seven days. Antiviral drugs may also be given because of its presumed viral aetiology.
~ Wear patch over left eye at night.
~ May prescribe artificial tears.
~ Review within a few days for support and monitoring.
~ Investigations are not essential but CT head would be appropriate for reassurance in view of patient anxiety
about a stroke.
~ Referral to a neurologist should be offered for confirmation of diagnosis and possibly for nerve conduction
studies. Referral is also appropriate for confirmation of management, because of possibility of incomplete or
nonrecovery.
~ Arrange continuing followup to monitor progress, watch for symptoms of conjunctivitis and corneal injury.
~ Consider early referral to a physiotherapist as an aid to self-management strategies. There is no evidence that
exercises or nerve stimulation aid recovery, but they may support patient confidence in recovery.
094
Performance Guidelines
513
KEY ISSUES
Approach to patient acknowledging distress about his appearance, providing support and guarded
reassurance about recovery.
Initial management plan protection of eye and possible use of steroids and antiviral agents. Offer referral to
neurologist and physiotherapist.
Patient counselling about prognosis and natural history, stressing that complete recovery is usual (although not
invariable).

CRITICAL ERRORS
Telling patient that complete recovery always occurs.
Very unsatisfactory counselling skills displaying insensitivity in dealing with an anxious patient.
COMMENTARY
The most common cause of unilateral facial nerve palsy without a clear history of local injury is the condition of
idiopathic acute facial nerve palsy ('Bell Palsy)
Bell Palsy is of unknown aetiology and affects all ages and both sexes. Patients present with an acute or subacute
onset over a few hours. Pain around the ear is followed by unilateral facial paralysis of lower motor neurone type, with
complete or partial paralysis of muscles supplied by the facial nerve. The clinical features are consistent with a lesion
due to inflammatory oedema and compression of the nerve within the bony canal of the petrous temporal bone.
Clinical features of the lesion, if complete and all muscles equally affected are as follows (see illustrative figures):
Facial asymmetry is accompanied by loss of voluntary, emotional and associated movements.
The affected side of the face is immobile, the eyebrow drops, the lines on the forehead and nasolabial fold are
smoothed out.
The palpebral fissure is wider due to the unopposed action of levator palpebrae.
Tears fail to enter the lacrimal puncta medially because they are no longer held against the conjunctivae and the
eye weeps.
The direct corneal reflex is absent, but the patient appreciates the discomfort from testing and the indirect corneal
reflex is present (the other eye blinks). Corneal abrasion and ulceration are significant risks.
Efforts to close the eyes cause the affected globe to roll up under the upper lid (Bell reflex).
The a/a nasi does not flare or dilate with vigorous breathing.
The lips stay in contact but cannot be pursed for whistling. When smiling, the angle of the mouth on the affected side
does not move; and in repose 'wry-mouth' can be identified.
Hyperacusis in the affected ear can be troublesome when the patient is subjected to local noise.
During mastication food accumulates in the cheek and dribbling of saliva can occur from between paralysed lips.
The articulations of labial consonants (m, b, p) may be affected.
094
Performance Guidelines
514


Loss of taste sensation may be noted in the anterior tongue on the affected side.
The cosmetic and psychological effects of the disfigurement can be profound.
Patients with Bell Palsy are frequently concerned that they may have suffered a paralytic stroke.
Pathologic Anatomy
The facial nerve supplies muscles of facial expression from scalp to neck from occipito-frontalis in the scalp to
platysma below, and including those muscles governing movement of eyebrows, eye closure, mouth, cheek and
nose.
The major motor root of the 7th nerve originates in the pons from the motor nucleus and fibres run in the pons in an
unusual curving course around the 6th nerve nucleus before leaving the anterior surface of the mid-pons to enter the
internal auditory canal with the 8th (vestibulocochlear) cranial nerve. The motor root is joined at the internal auditory
meatus by the 'sensory' root, which carries afferent taste fibres from the tongue and also efferent secretomotor fibres
to the lacrimal and salivary glands. Somatic sensation to the face is subserved by the 5th cranial nerve, not the 7th.
Within the petrous temporal bone the nerve runs laterally to the medial wall of the tympanic cavity before bending
backwards abruptly (the genu). The facial nerve then runs downwards in the facial canal on the inner wall of the
tympanic cavity, giving branches to the tiny stapedius muscle of the inner ear, and giving off the chorda tympani,
before emerging from the stylomastoid foramen at the base of the skull.
The geniculate ganglion is the relay station for the secretomotor fibres for tears and the site of the sensory root
ganglion of the taste fibres. From the geniculate ganglion run the secretomotor fibres to the lacrimal gland and
submandibular salivary gland. The autonomic sensory taste fibres are carried from the tongue with the lingual nerve
(carrying ordinary sensation) via chorda tympani through middle and inner ear, to the sensory facial nerve root. Facial
nerve lesions below the chorda tympani (e.g. in the parotid gland) will not affect taste.
The chorda tympani leaves the nerve a few millimetres above the point of exit from the stylomastoid foramen, and runs
between the layers of the tympanic membrane separating outer and middle ears, and then joins the lingual nerve to
the tongue, and supplies the anterior two-thirds of the tongue with taste sensation as illustrated.
The motor branches of the facial nerve break into a spray of branches and run through the parotid salivary gland
before emerging from its anterior border to supply the facial muscles via temporal, zygomatic, buccal, mandibular, and
cervical branches as illustrated.
094
Performance Guidelines
515

f.m, foramen magnum
s.f stylomastoid foramen
s.g. submandibular ganglion
t n taste nucleus - nucleus of tractus solitarius
s.s.n. superior secretory nucleus
7* m.n. 7
th
nerve motor nucleus
rn T. motor root
s.r 'sensory' root
s stapedius
b buccinator
at chorda tympani
g.g- geniculate ganglion
i.a.m. internal auditory meatus
CONDITION 094. FIGURE 5.
Anatomy of facial nerve
094
Performance Guidelines
516
Differential Diagnoses other causes of facial nerve palsy
Patients with Bell Palsy are frequently concerned that they have had a stroke', or have a cerebral tumour.
Facial weakness due to 'stroke' is usually upper neurone in type and part of a hemi-paresis on the same side as
the facial paralysis. Movements of the upper muscles to forehead and eyes (which are bilaterally innervated from
the upper motor neurone) are spared, as may be emotional movements. However, an infarct in the pons may
produce a nuclear (lower motor neurone) lesion of the facial nerve.
In Bell Palsy the motor lesion is confined to the facial nerve alone and is lower motor neurone in type.
If hearing loss or other cranial nerve lesions are associated with facial nerve palsy the diagnosis is more likely to
be a cerebellopontine angle tumour (for example, auditory neurofibroma) or a vascular event from vertebrobasilar
insufficiency.
If vesicles within the external ear or on the palate accompany the 7th nerve palsy the condition is viral herpes
zoster infection affecting the geniculate ganglion, not Bell Palsy. This is the Ramsay Hunt syndrome, and
occasionally other cranial nerves are also affected. Such patients are often elderly. Pain may precede the facial
palsy and the associated herpetic eruption in the ear and sometimes on tongue or palate. Recovery of facial nerve
function is rare. Prompt treatment with aciclovir may improve prognosis and diminish post-herpetic neuralgia.
If the nerve is affected within the parotid gland, this is usually due to a parotid malignancy giving partial or total
lower motor palsy. Benign parotid tumours do not cause facial palsy.
Basal skull fractures of the petrous bone are another important cause of facial nerve palsy.

CONDITION094. FIGURE 6.
Left-sided facial nerve palsy
Face in repose: note widening of palpebral fissure due to unopposed action of the levator of the upper lid, smoothing
of facial lines and failure of eversion of mucosa of patient's left lower lip ('wry-mouth') due to paralysis of depressor
anguli oris.
094
Performance Guidelines
517
CONDITION094. FIGURE 7.
Left-sided facial nerve palsy
Attempted eye closure: note failure of left eye closure with rolling up of the eye under the upper eyelid, and
accentuation of the 'wry-mouth' triangular deformity.

CONDITION094. FIGURE 8. Left-sided
facial nerve palsy
Obvious deformity when smiling: note immobile left eye and mouth musculature and absence of nasolabial fold.
094
Performance Guidelines
518
CONDITION094. FIGURE 9.
Left-sided facial nerve palsy
Attempting to blow out cheeks: note failure of left buccinator muscle with flaccid paralysis of patient's left cheek.
The patient is unable to prevent air from escaping from the mouth when he tries to build up intraoral pressure.
095
Performance Guidelines
519

Condition 095
Dysuria and urinary frequency in a 40-year-old man
AIMS OF STATION
To assess the candidate's approach to a first time urinary tract infection in an adult male patient.
EXAMINER INSTRUCTIONS
The examiner will have instructed the patient as follows:
You consulted this doctor today because of the gradual onset of dysuria and frequency of micturition over the last three
days. The doctor has examined you (including rectal examination) and asked you to provide a urine sample which was
checked in the practice laboratory. You are about to receive the doctor's advice about the problem.
Questions to ask unless already covered:
'What did you find in my urine?'
'Can this infection be treated easily?'
'Where do these bacteria come from?'
Is this the same as my wife gets?'
'Tell me exactly what tests I should have done?'
'Why do I need these tests?'
'Do you think I have something seriously wrong?'
'How long will I need to take the medication?'
'Could it occur again?'
'What will the urologist do?' (Ask only if referral is advised)
You are not overly concerned about your condition because your wife has suffered from occasional urinary tract
infections over the years which have always responded well to treatment with antibiotics. She has had no other
investigations other than urine laboratory tests. You expect to recover quickly after receiving antibiotics.
If the doctor indicates that further special investigations are necessary followed by referral to a urologist, be surprised
and express some reluctance to undergo these procedures. If the doctor handles your reaction satisfactorily agree to
follow this advice.
EXPECTATIONS OF CANDIDATE PERFORMANCE
Approach to patient
This may appear to be a straightforward clinical situation but it requires care to avoid alarming the patient
concerning the need for more investigations than just a urine culture
Give a clear explanation of the nature of the condition.
Obtain compliance in use of medication (clear instruction about frequency and duration)
Emphasise importance of followup.
Explain why further investigation is essential and obtain compliance for this.
095
Performance Guidelines
520

Tell the patient what Is wrong
A variety of terms may be used to describe a urinary tract infection but the candidate should explain that it is most
likely to be in the lower urinary tract (bladder, prostate or urethra) rather than in the kidneys, because of the absence
of fever and loin pain.
Immediate management
The candidate must advise that the midstream urine specimen collected today will be sent for culture and antibiotic
sensitivity assessment.
Choice of initial treatment appropriate antibiotic for example, trimethoprim 300 mg orally, once daily; or
cephalexin 500 mg orally, 12 hourly. Urinary alkalisation may be used, for example, Ural (sodium citrotartrate) 8
hourly. Duration of therapy is 14 days. Amoxycillin is often given but is inappropriate in this patient because of the
penicillin sensitivity. Advise patient to drink extra fluids. The antibiotic therapy should be commenced today whilst
awaiting the culture results. Phone with results when through.
Early review if poor response to treatment. Followup of this episode by repeat microscopy and culture after
completion of antibiotic therapy.
Discussion of condition and advice about investigation
Significance of a urinary tract infection in males. Usually associated with underlying pathology according to age
group:
Children congenital abnormality especially vesicoureteric reflux.
Younger adults foreign body in bladder, sexually transmitted infection, including homosexual activity.
Older adults
~ calculus formation in kidney, ureter, or bladder;
~ prostatitis;
~ bladder polyps or carcinoma;
~ benign prostatic hypertrophy;
~ carcinoma of prostate;
~ urethral stricture; or
~ genitourinary tuberculosis should not be forgotten.
The clinical picture suggests a lower urinary tract infection. The main conditions to be excluded are urinary
neoplasm and calculus and prostatic pathology.

KEY ISSUES
Approach to patient.
Initial management plan.
Choice of investigations.
Patient education and counselling.
095
Performance Guidelines
521
CRITICAL ERRORS
Failure to arrange urine culture before commencing antibiotic therapy.
Failure to advise the need for further investigations.
COMMENTARY
Further investigation is essential to identify the underlying cause and to exclude malignancy.
These would be undertaken in a staged manner.
Urinary culture to define organism. Repeat culture after initial treatment.
Ultrasound of kidneys, ureters, and bladder.
Contrast enhanced CT of abdomen and pelvis.
Prostatic specific antigen (PSA) level.
Serum urea and electrolytes
Referral to a urologist who may arrange:
~ cystoscopy,
~ voiding cystourethrogram.
522
096
Performance Guidelines



Condition 096
Eclampsia in a 22-year-old primigravida at 38 weeks gestation

AIMS OF STATION
Toassess the candidate's ability to recognise that the grand mal fit is a sign of eclampsia, and the ability to manage
appropriately this particular pregnancy complication in an 'out-of-hospital' situation.

EXAMINER INSTRUCTIONS
The examiner will have instructed the mother of the patient as follows:
Your daughter has just had a fit in the waiting room. The fit occurred approximately 10 minutes ago and lasted three
minutes. She bit her tongue, and had funny movements of her limbs, and then went off to sleep.
The candidate will generally be expected to take an appropriate history from you as the mother in order to manage
the case. The list of responses below is likely to cover most of the questions you will be asked.
Your daughter has:
no past history of epilepsy, and has never had any treatment with antiepileptic drugs;
no hypertension, renal disease, or other medical problem in the past;
not mentioned any headaches or visual disturbances recently; and
noticed oedema of the legs for the last two weeks, but was otherwise well.
Questions to ask if not already covered:
'Why did she have a fit?'
'Will the fit damage my daughter or her baby?'
'Will she have any more fits?'
'What are you going to do with her now? Can I take her home?'
If the candidate suggests hospital transfer, but does not detail what will happen to the daughter following
admission to hospital, you should ask 'What treatment will they give my daughter in hospital?'
096
Performance Guidelines
523

Examination findings to be given to the candidate by the examiner on request
Drowsy, but rousable.
Generalised oedema.
BP: 180/110 mmHg.
Pulse: 80/min and regular.
Reflexes: very active, clonus evident at the knees.
Abdominal examination: uterus enlarged to 38 cm (symphysis-fundal height), lax, and non tender. The
presentation is cephalic, fixed in the pelvic brim, with three fingerbreadths palpable above the pubic symphysis.
The fetal heart is audible and normal.
Central nervous system examination apart from the conscious state and the active reflexes, this appears
normal. There are no unilateral localising signs.
Investigation results
None done except office urine testing of specimen brought with her showed proteinuria (++++). Failure to ask for
the results of urine testing would indicate inadequate care. ,

EXPECTATIONS OF CANDIDATE PERFORMANCE
The candidate should advise the mother along the following lines:
The diagnosis is eclampsia, a condition which occurs late in pregnancy generally in women having their first baby.
Providing it is well controlled, no longterm harm usually occurs to either mother or baby, although it is potentially
very dangerous to both the mother and baby.
She will need to be transferred and admitted to hospital immediately, and delivery arranged as soon as her blood
pressure and any further fitting are brought completely under control.
There is no point prolonging the pregnancy in view of the gestation of 38 weeks.
Prior to transfer to the hospital admission an anticonvulsant such as diazepam should be given intravenously in an
attempt to prevent further fitting (oral therapy is inappropriate and NOT acceptable).
The likely care provided in hospital would be:
~ an intravenous drip to be inserted and magnesium sulphate commenced in appropriate dosage to try to prevent
any further fits;
~ the blood pressure should be lowered with intravenous hydralazine or diazoxide oral agents are ineffective and
should not be used;
~ tests on patient should include: renal function tests, liver function tests, Hb and platelet count, and coagulation
profile;
~ the fetus should be checked by cardiotocography (CTG);
~ monitoring of the patient should include: pulse, blood pressure, temperature, urine output, and frequent urine
testing (predominantly for protein); and
~ the room should be prepared in case a further fit occurs with the facility to administer oxygen, to have a Guedel
airway/padded spoon available to prevent her from biting her tongue, to have the facility to place her in Sims
position, and to observe her in a slightly darkened environment.
096
Performance Guidelines
524

The mode of delivery will depend on her condition, and the cervical findings. If the cervix is very favourable, she
should be induced and monitored closely in labour, probably with the use of an epidural anaesthetic for pain relief
and blood pressure control. CTG monitoring of the fetus in labour is mandatory. If the cervix is unfavourable,
consideration needs to be given as to whether induction is appropriate, after prostaglandin priming, or whether an
elective Caesarean section is more appropriate. Obviously if the CTG is abnormal, Caesarean section is likely to
be required.

KEY ISSUES
Knowledge of the causes of fitting in pregnancy.
Ability to manage a patient who has had an eclamptic fit in late pregnancy and is not in hospital.

CRITICAL ERRORS
Failure to diagnose eclampsia and recognise risk of this to mother and baby.
Failure to sedate, and failure to transfer her immediately to hospital.
Failure to outline the three principles of management in the hospital sedation, lower blood pressure, and
delivery of baby.

COMMENTARY
In this case the most likely diagnosis is eclampsia occurring in pregnancy. A history of previous fits should be sought,
but it is unlikely that this is anything other than an eclamptic fit. It is important that the three basic principles of the
management of eclampsia are performed or arranged. These principles are: prevention of further fits; lowering of the
blood pressure; and arrangement for immediate delivery of the baby by the most appropriate route.
Common problems likely with candidate performance are:
When taking the history, not being focused enough to the actual problem, but asking for information such as
irrelevant past history, social history and so on. This just takes time to do, and reduces the time available for the
remaining tasks.
Not asking whether she had had any fits before.
Not asking for appropriate examination findings, such as hyperreflexia.
Not requesting whether there was any proteinuria present.
Failing to understand that any prolongation of the pregnancy is irrelevant as the gestation is already 38 weeks.
Administering oral instead of intravenous hypotensive drugs to reduce the blood pressure.
525
097
Performance Guidelines



Condition 097
An abnormal glucose tolerance test (GTT) in a 34-year-old primigravida

AIMS OF STATION
Toassess the candidate's ability to make the correct diagnosis of gestational diabetes, and to appropriately manage the
patient for the remainder of the pregnancy. The candidate should do this by taking a focused history, asking the
examiner for the examination findings, and then advising the patient appropriately

EXAMINER INSTRUCTIONS
The examiner will have instructed the patient as follows
The list of responses below is likely to cover most of the questions asked.
No family history of diabetes.
You have never been tested for diabetes previously.
No previous operations or illnesses
This is your first baby.
You are now 28 weeks pregnant.
Your ultrasound at 18 weeks was normal.
Questions to ask if not already covered:
What do the blood sugar levels mean?'
'Do I have diabetes?'
'How bad is my condition?'
What treatment will I require?'
Will my baby be diabetic?'

Examination findings to be given to the candidate by the examiner on request
The blood pressure is 120/80 mmHg.
No proteinuria.
Uterus is enlarged to the size equivalent to a 28 week pregnancy (symphysis-fundal height = 28 cm).
Cephalic presentation, head still mobile above the pelvic brim.
Fetal heart rate is normal.
526
097
Performance Guidelines



EXPECTATIONS OF CANDIDATE PERFORMANCE
It would be expected that the candidate would provide much of the following information.
The diagnosis is gestational diabetes
Consultation with a diabetic physician and consultant obstetrician is mandatory.
She should follow a special diet to keep the blood glucose during the day at less than 7 mmol/L. If this is not
possible, insulin therapy will probably be necessary.
Test the blood sugar 3-4 times per day, especially about two hours after a meal.
The major risks to the baby are:
~ Macrosomia (large baby size) do ultrasound at 32-34 weeks and probably deliver
by Caesarean section if macrosomic.
~ Increased risk of fetal death in utero therefore weekly CTGs should be performed until delivery, twice weekly
if on insulin, the fetus is macrosomic or polyhydramnios occurs. These should be started at 32-34 weeks
gestation.
~ Hyaline membrane disease if delivered prematurely try to delay induction until after 37 weeks. Steroid
therapy would improve fetal lung maturity, but will make gestational diabetes worse.
Risks to the mother increased risk of pre-eclampsia.
Deliver at term at the latest, unless obstetric complications indicate earlier delivery is indicated. Monitor the fetus
by continuous CTG in labour. Keep blood glucose levels stable in labour with intermittent insulin injections.
Deliver by elective Caesarean section if macrosomic (> 90th centile for weight), breech presentation, or evidence
of fetal distress.
The diabetes will almost certainly resolve following delivery. However gestational diabetes is likely in subsequent
pregnancies, and there is a 30%risk of her developing diabetes later in life. Glucose tolerance should therefore
be checked at least every 5 years for life. She must control any weight gain in the future.

KEY ISSUES
Ability to recognise that the blood sugar results are diagnostic of gestational diabetes.
Ability to appropriately assess the control of the diabetes during the remainder of the pregnancy, and to
appropriately manage the patient, in consultation with a physician and obstetrician.
Ability to recognise the need for insulin if the blood glucose levels are not reduced satisfactorily with diet alone.
Ability to recognise the increased risks to the fetus, and the need for close monitoring.
097
Performance Guidelines
527
CRITICAL ERRORS
Failure to diagnose gestational diabetes.
Failure to advise diabetic diet and testing of blood sugar levels 3-4 times daily.
Failure to arrange for consultation with a diabetic physician and obstetrician.
COMMENTARY
This case illustrates the need for the candidate to recognise the diagnosis of gestational diabetes based upon a two
hour glucose tolerance test. The most important aspects of the management of the case are to recognise the need for
assessment of the blood sugars three or four times a day; the need to consider insulin if the blood glucose levels do not
respond, and the need to include in the management of this patient a diabetic physician and an obstetrician.
Common problems likely with candidate performance are:
When taking the history, not being focused enough to the actual problem, but asking for information such as
irrelevant past history, social history and so on. This just takes time to do, and reduces the time available for the
remaining tasks.
Failing to recognise a need for special fetal monitoring because of the increased risks to the fetus.
098
Performance Guidelines
528
Condition 098
Bed-wetting by a 5-year-old boy

AIMS OF STATION
To assess the candidate's ability to diagnose and handle the common problem of bed-wetting in a 5-year-old child.

EXAMINER INSTRUCTIONS
The examiner will have instructed the parent as follows:
You are the mother of Johnny who has a problem with bed-wetting nightly since the age of three years. You were
initially exasperated by the wetting but now have accepted that the wetting is involuntary. You have not punished
Johnny despite your exasperation.
His general health is excellent, and he has had no major illnesses. He appears to be growing normally and is on the
middle line of his graph for height and weight. Since the age of 3 years, he has always been dry during the day and
never had any incontinence. He has never had a urinary tract infection.
He is embarrassed and you and your spouse are very keen to help him control his wetting. His father wet the bed until
the age of nine years. The tablets that were tried two years ago made no difference to the wetting.
Johnny is going very well at school and enjoys his teacher. He has lots of good friends. You and his father are happily
married and have no major stresses in your lives. Johnny has a 4-year-old younger sister who has been dry day and
night since the age of two and a half.
Questions to ask if not already covered:
Is there something wrong with his kidneys or bladder?'
'Does he need any investigations?'
We have restricted his fluids after dinner at night and lift him onto the toilet when we go to bed. Should we
continue to do this?'
'What about when he is asked to sleepover at a friend's place so far we haven't let him do this. Is there anything
we can do for that?'
'How does this alarm work if he has already passed urine and wet his bed before it goes off?'(If an alarm is
advised)
EXPECTATIONS OF CANDIDATE PERFORMANCE
This scenario describes a 5-year-old boy with persistent primary bed-wetting from three years of age. He is otherwise
well, has no daytime wetting or any other symptoms to suggest a pathological cause for his wetting.
His height and weight are on the 50th percentile. The boy himself is very keen to be dry, and his parents are keen to
help him achieve this. These are important points the candidate should appreciate. Amitriptyline (Tryptanol) was
tried about two years previously to no avail. Johnny is doing well at school and has lots of good friends.
098
Performance Guidelines
529
The parents are happily married and under no major stresses. His 4-year-old younger sister has been dry day and night
since the age ot 2
1
/2years. The candidate should enquire about family history; one parent was a bed-wetter until aged
nine years.
Before embarking on a plan of action, the candidate should proceed as follows:
Check the boy's urine by dipstix (Multistix) testing, and a urine microscopy and culture.
Enquire about the child's growth percentiles, which are normal.
Ensure his blood pressure is normal.
Renal ultrasound may be suggested but is probably unnecessary unless there is a great deal of parental anxiety.
Having excluded any organic pathology and having ensured that there are no serious emotional reasons to account for
the symptom, the candidate should outline an ongoing plan of management. This should include:
Empathy with the exasperating nature of the condition particularly with the excessive washing of bedclothes and
pyjamas, but enthusiasm for the interest the parents are showing in trying to help Johnny.
Reassurance that there is almost certainly no organic pathology present.
Advice that even though lifting and restriction of fluids have not been shown to be effective generally, if the parents
are keen to continue this they should feel free to do so as it occasionally does help some children.
Outlining the plan of management including use of an enuresis alarm. Explain how the alarm works as a
conditioning response to release of urine.
Explaining how to obtain the alarm (for example, through pharmacies [hiring], buying or through some Community
Health Centres or Children's hospitals)
Discussing that the success rate is much higher if the child himself is motivated to become dry (as Johnny is),
Discussing a recording star chart and reward system.
Supporting and encouraging child and parent by regular frequent review to encourage the boy on even minor
successes.
Explaining a plan of action for the use of arginine vasopressin (DDAPV) by nasal spray when it is important to
remain dry and avoid any embarrassment for school camps and sleepovers. Explain the safety of this substance if
used only as directed.
Advice that the success rate with amitriptyline (Tryptanol) is low. It can be a dangerous drug in overdose and is
rarely used now.
Advice that even with the alarm it may be some weeks before success is achieved and the alarm should be
persisted with for up to three months.
A review appointment should be made two to three weeks after the alarm has started, to review the progress.

KEY ISSUES
Empathy, support, and encouragement to both child and parent.
Enquiry about a family history of enuresis.
Exclusion of emotional stress at home or school.
Exclusion of organic pathology by the history and by arranging simple urine testing.
Advice about plan of action should be logical and clear.
098
Performance Guidelines
530

CRITICAL ERROR
Suggesting a probable organic cause for the wetting and the need for invasive investigations.
COMMENTARY
This scenario describes a five-year-old boy with persistent primary bed-wetting from three years of age. He is
otherwise well, has no daytime wetting or any other symptoms to suggest a pathological cause for his wetting. The
candidate snould appreciate that the boy himself is very keen to be dry, and that his parents are willing to help him
achieve this
531
099
Performance Guidelines



Condition 099
Acute gout in a 48-year-old man
AIMS OF STATION
To assess the candidate's ability to manage an acute attack of gout and give advice about its prevention.

EXAMINER INSTRUCTIONS
The examiner will have instructed the patient as follows:
You are suffering from severe pain in your right foot which began two days ago. You saw the doctor earlier today who
diagnosed gout (which you have had before), and arranged for a confirmatory blood test. You have returned to find out
the result and receive treatment.
You are a 48-year-old taxi driver and usually keep in good health. You do not smoke but drink three or four stubbies of
beer, after work, daily. No serious past medical problems but you are taking tablets for mild blood pressure diagnosed
two years ago. You are overweight. You have no family or social problems.
You are anxious to get relief from the pain which is preventing you from driving your taxi. You are somewhat irritated
that you were asked to have a blood test, because this was not done during previous attacks which responded well to
treatment. You have little knowledge about the cause of gout and are unaware that recurrent attacks are to be expected
and can be prevented. You have not suspected that the blood pressure tablets could have something to do with gout,
and are annoyed that you were not warned about this.
Questions to be asked if not covered
'How long before I can resume work?'
'What causes gout?'
'Can it do any serious damage to my system?'
'What is the best treatment?'
'Should I have any other tests?'
'What about having a beer after work?'
EXPECTATIONS OF CANDIDATE PERFORMANCE
Approach to patient
It is essential to establish a satisfactory relationship with this patient because of the need for compliance regarding
his use of alcohol and control of his weight, and to defuse irritation about having to have a blood test for this attack
and finding out that the antihypertensive medication has been a precipitating factor.
Initial management plan immediate
~ Discontinue diuretic and aspirin.
~ Specific treatment:
- Nonsteroidal anti-inflammatory drug (NSAID) initially in high dose: e.g. indo-methacin 25 mg capsules
50-75 mg immediately, 50 mg two hours later, 25 mg eight hourly for 48 hours, then 25 mg twice daily for one
week, would be appropriate. Other NSAID such as naproxen or ibuprofen are also effective.
532
099
Performance Guidelines



- OR prednisolone 25 mg orally, daily in the morning, reducing to zero over 7 to 10 days.
- OR colchicine 0.5 mg tablets 2-3 immediately, then 1-2 every 4-6 hours or until diarrhoea occurs.
Maximum dose 6 mg/24 hours. As pain reduces the dose of colchicine can be reduced to 0.5 mg twice
daily.
- Note that allopurinol and probenecid are contraindicated for an acute attack.
~ Additional measures:
- Increase fluid intake.
- Elevate and rest foot for 24-48 hours.
- Paracetamol (Panadol) can be used for additional pain relief if needed.
- Warn regarding possible side effects of medication: indigestion and elevated blood pressure from
indomethacin or prednisolone: diarrhoea from colchicine.
- Suggest an alternative drug to reduce blood pressure (e.g. angiotensin converting enzyme inhibitor [note
that all thiazides and Beta blockers may exacerbate gout]).
- May return to work as soon as pain is relieved should be within 48 hours.
Initial Management Preventive
~ Ensure adequate intake of water. Do not take diuretics or salicylates.
~ Reduce weight. Reduce intake of alcohol. Avoid foods rich in purine (offal, tinned fish, shell fish and game).
~ Approximately eight weeks after this attack has subsided may commence allopurinol 50-100 mg daily,
gradually increasing up to 300 mg daily (two strengths 100 and 300 mg tablets)
~ Check uric acid level after 4 weeks aim to reduce below 0.4 mmol/L
~ Colchicine (0.5 mg b.d.) can be used in conjunction with allopurinol if gout recurs during initial therapy.
~ Further assessment of patient should include review of blood pressure, serum lipids, fasting blood sugar and
urea and electrolytes. Other renal function tests are not indicated at this stage. X-ray of the affected area is not
required.
Patient education and counselling
~ Gout is a metabolic disturbance with an inherited tendency in which there is decreased renal clearance of urate
causing hyperuricaemia with deposition of urate crystals in joints, soft tissue (tophi) and urinary tract (urate
stones). It is frequently associated with hypertension, dyslipidaemia, and Type 2 diabetes. Thiazide diuretics
also predispose to diabetes. Gout:
- Particularly occurs in the great toe following minor trauma.
- Can follow any surgical operation.
- Can be precipitated by alcohol excess, and diuretics which inhibit sodium reabsorption.
- Is aggravated by diet high in purines.
- Is prone to recurrence.
- Exhibits a prompt response of the acute attack to appropriate treatment (24-48 hours).

KEY ISSUES
Appropriate choice of drug therapy for initial management.
Appropriate patient education and counselling regarding prevention of further attacks.
099
Performance Guidelines
533
CRITICAL ERROR
Failure to advise change of antihypertensive medication (thiazide diuretic).
COMMENTARY
Gout (uric acid arthropathy) may present as acute arthritis or be associated with a chronic destructive arthropathy. Most
cases of primary gout are due to excessive synthesis of uric acid while one-third relate to reduced renal clearance of
urate. Acute gout commonly affects the great toe metatarsophalangeal joint, although other foot joints and the ankle are
frequent sites. It may affect any joint in the body or, unusually, it can present as a polyarticular arthritis mimicking other
systemic rheumatic conditions. The joint can be extremely painful, red and tender and the patient may be intolerant of
even a sheet touching the foot. If fluid can be obtained from an affected joint, it will contain needle-shaped crystals that
are negatively birfringent on phase-contrast microscopy. Plasma urate concentration may not be elevated in the
course of an acute attack, so hyperuricaemia is not a necessary diagnostic criterion.

CONDITION 099. FIGURE 1. CONDITION 099. FIGURE 2.
Acute gout Chronic tophaceous gout
It is important to treat hyperuricaemia in order to avoid chronic gouty arthritis, tophaceous gout and renal complications
(calculi, chronic renal failure due to interstitial nephritis). Hyperuricaemia is commonly exacerbated by excess alcohol
intake and drugs. It is common in clinical practice to come across an elderly patient who has been on longterm diuretic
therapy with chronic tophaceous gout and renal impairment. This should be considered an iatrogenic disease.
Hyperuricaemia is an independent risk factor for cardiovascular disease. Therefore, an attack of gout provides an
opportunity for the prescriber to review the cardiovascular risk factors (for example, smoking, hyperlipidaemia,
hypertension, obesity), and to recommend appropriate management.
In this station, the candidate is confronted with a very common clinical problem, an eminently treatable condition and an
opportunity, through patient education, to institute a longterm management plan to reduce the frequency of attacks. It
also provides the practitioner with the opportunity to address significant lifestyle issues with the potential for improved
cardiovascular health.
100
Performance Guidelines
534
Condi t i on 100
Request for repeat benzodiazepine prescription from a 25-year-old man

AIMS OF STATION
Toassess the candidate's ability to identify benzodiazepine dependency and counsel the patient accordingly.

EXAMINER INSTRUCTIONS
The station assesses the candidate's awareness that longterm benzodiazepine use is a problem (abuse), capacity to
recognise overuse in this case by simple calculation (dependency), and preparedness to intervene and address this
with the patient. It also assesses ability to evaluate the problem further by taking a focused history, communication
skills in engaging the patient and ability to make an appropriate management plan including an awareness of
potential problems following sudden cessation.
A brief survey of mood is all that is required in this case. The history does not suggest any other psychiatric diagnosis
except substance dependency.

At five minutes, if candidates have not already done so, interrupt and ask them to give their conclusions and
make their recommendation to the patient.
At six minutes, ask candidates whether they would immediately stop the prescription and why.
You may not need to ask this question if the candidate has already addressed this issue.

The examiner will have instructed the patient as follows:
You are a 25-year-old, divorced salesman. You were first prescribed this anxiolytic four years ago when your
marriage broke down. At the same time, your business failed and you were having problems going for job interviews
because you had lost so much confidence. You eventually found the anxiolytic was very good at helping you get off to
sleep quickly, and you have continued to use the anxiolytic since. Your work, relationships and home life are generally
okay now, but you expect this is at least partly because you can get to sleep without fail and feel calm throughout the
day.
Opening statement:
'This is just a quick one for you, Doc. I just need a refill of my sleeping tablets, thanks, and then I will be on my way'.
If the doctor indicates in any way that you should not be taking the Serepax, or that the doctor is going to stop or
substantially change the dose immediately, respond as follows:
I know I need to continue the Serepax because on occasions I have taken only half a tablet for a few days, or
missed taking them for one to two days, and I ended up feeling edgy, jittery, shaky and was unable to sleep or
concentrate on my work. On one occasion, I thought I was going to have a heart attack because my chest was
pounding. As soon as I get back to my usual dose, I feel fine. I do not use, and have never used, any other drugs.'
100
Performance Guidelines
535

You are generally in good health. You do not smoke and rarely drink alcohol. You have not had any serious illnesses,
injuries or accidents, including car accidents. You have never had any fits, falls or faints or any loss of consciousness.
You did feel depressed when your marriage broke up and your business failed, but that has been the only time.
Give the following responses and answers to further questions:
When or if the candidate mentions addiction or dependency, say that you are aware of some information about
these tablets being addictive, but you don't think it applies to you (for no particular reason), and also your symptoms
on ceasing are evident to you that you have a 'real need'.
When, or if, the candidate asks about the schedule of your use. respond by saying that you use them only to get to
sleep.
When or if the candidate goes on to demonstrate to you that you have used more than one per day in the past week,
say that there has been extra pressure at work, and admit that you have taken a few extra. You can expect the
candidate to pursue the issue of quantities used. If the candidate approaches this diplomatically, be cooperative in
uncovering the overuse problem. If the candidate is critical or blaming, or wishes to refer you immediately to a
substance abuse unit or report you to the Health Department, take offence and say you do not want to continue with
the consultation: 'Just give me the script and I will go' or 'I'll sort this out myself.
When the candidate explains a proposal of a period of monitoring your use and moods or stress, followed by graded
reduction, along with regular appointments, support and resource materials or groups, agree with this management
plan.
If the candidate refuses to prescribe, take offence and respond as above: 'I'll sort this out myself.
If the candidate agrees to provide a prescription with no suggested measures or comments such as 'we will talk next
time', accept that and say 'What days do you work. Doc. so that I can be sure to see you next time?'
Be quite at ease and be pleasant. Provide your background history and the development of your habit in a
straightforward way. Respond to questions about your present use as outlined previously.
Questions to ask if not already covered:
'So what's the problem with taking these tablets, really?'
EXPECTATIONS OF CANDIDATE PERFORMANCE
The candidate should:
Take a focused history of the patient's use of the benzodiazepine anxiolytic (Serepax) and history of other
substance and alcohol use and patient mood.
Advise the patient about the problems of benzodiazepine dependency and outline a plan of management that
includes a gradual reduction in use, along with regular appointments, support and resource material and followup.
536
100
Performance Guidelines



KEY I SSUES
Identification and preparedness to address the issue of dependency and overdose.
Appropriate language and attitudes in taking the history and discussing the problem.
Knowledge of biological and psychosocial management of benzodiazepine dependency.
Awareness of the risks of sudden cessation, such as acute withdrawal states, fits, agitation, exacerbation of
anxiety and treatment failure.
CRITICAL ERROR - none defined

COMMENTARY
This station assesses the candidate's ability to identify inappropriate benzodiazepine use and dependency, and to
counsel a patient appropriately. The doctor is presented with the problem of being drawn into maintaining a longterm
benzodiazepine use habit, with clear evidence of over-use (approximately 75%greater consumption than the
prescribed dose). Benzodiazepines are recommended for short-term use only. While they have some place in the
longterm management of chronic severe anxiety, other treatments, including antidepressant medication and
psychological treatments (relaxation techniques, cognitive behavioural or interpersonal therapies) must be applied
first. Longterm prescription needs are to be closely supervised and monitored for over-use, such as in this case.
This case challenges candidates in a number of ways. Chiefly, it requires them to actively and constructively
intervene, not just to provide the prescription (with or without advice) or to just refuse the prescription, thus provoking
the patient to seek out another source, or risking precipitating a withdrawal state. The satisfactory candidate, in
addition to managing the immediate consultation needs, will be aware of community support and self-help groups.
Candidates should demonstrate that they understand the problems of both prescribing further medication without any
review or plan for reduction, and also suddenly stopping the medication. Thus, simply advising the patient that they
will 'talk next time', or refusing to prescribe with no other measures put in place, are both unsatisfactory. Similarly,
referring the patient immediately to a substance abuse unit would be unsatisfactory and counterproductive.
3-B:ClinicalProcedures

PeterGDevittandBarryPMcGrath
' Thehandi sthecutti ngedgeofthemi nd.'
JacobBronowski
Thetermmanagementimpliesanintegratedapproachtopatientcare,focusingoninvestigation
andtreatment.
This section concentrates on aspects of management that involve practical aspects of patient
care. There are certain procedures with which the commencing intern is expected to be fully
conversantandcompetent.Theseincludeatleast:
establishmentofelectiveoremergencyvenousaccessfor
infusionoffluids;
Thejuniorgraduatecommencing
internshipisexpectedtobefully
conversantandcompetentwith
certaincoreskills,including
antisepticandaseptictechnique,
basicfirstaidtechniquesand
primarywoundcare.
Graduatesareexpectedalsoto
understandtheprinciplesofmore
specialisedclinicalskills,andtobe
abletoexplaintheseclearlytoa
patient.
venesectionforcollectionofbloodsamples;
insertionandremovalofaurinarycatheter(maleand
female);
intramuscular,subcutaneousandintradermalinjections;
cardiopulmonaryresuscitation(CPR);
interpretationofECGsandbasicimagingfilms;
useofaglucometer:
useofinhaler,spacerandnebuliser;and
understandingofspirometryandpulmonaryfunction
testing.
Thereareotherskillsthattheindividualisexpectedtoacquireprogressivelyundersupervision
during internship and should have some understanding of at graduation. Some of these are
ward-basedskillsandsomemaybelearntintheemergencyoranaestheticdepartment.These
skillsinclude:
simpleskinsuturing;
useofadefibrillator;
maintenanceofanadequateairway;
insertionofanasogastrictube
performanceofanelectrocardiogram;
collectionofasampleforarterialbloodgasanalysis;
centralvenouspressuremeasurement;
pleuralaspirationandperitonealtap(needlethoracentesis,needleperitoneocentesis);
nasalpacking;
slitlampexamination;
anoscopy(proctoscopy/rigidsigmoidoscopy);and
endotrachealintubation
537
538
3-B
ClinicalProcedures



Another group of skills are those which the intern will be expected to observe and perhaps
performundersupervision,butnotnecessarilydemonstratefullyindependentcompetenceat
thisstageoftraining.Itisimportantthattheprinciplesoftheseproceduresareobservedand
understoodandcanbeexplainedtopatients.Theseskillsinclude,amongothers:
indirectlaryngoscopy;
lumbarpuncture;
useofatonometer;and
advancedlocalanaestheticandfieldblockincludingintravenouslocalblock(Bier),and
epiduralblockcommonlyusedinobstetrics.
Afourthgroupofskillsincludethosewhicharenotconsideredsuitableforthoseatinternlevel,
butwhichmayhavebeenobservedbyindividualswhilstmedicalstudentsorinterns.Thereisa
commonmisconceptionthatsuchproceduresneedtobelearntatinternship,wheninrealitythe
procedures should preferably only be undertaken independently by individuals with already
developedspecialistskills.Again,theprinciplesoftheseproceduresshouldbeunderstoodat
internlevel,suchthattheycanbeexplainedtopatients.Theseproceduresinclude:
cricothyroidotomy;
insertionofacentralvenousline;
insertionofanintercostaldraintube;
peritonealdialysis/lavage;and
haemodialysis/haemofiltration.
Missingfromtheselistsareanumberoftaskssometimesthoughtofasprocedures,butwhich
in reality should be considered an essential part of the examination of the patient. These
include:
ophthalmoscopy;
measurementofbloodpressure;
otoscopy;and
urinalysis.
Specialistandgenericendoscopicskills
Theseareusuallyconfinedtoperformancebyspecialistsinvariousdisciplinesandinclude:
nasolaryngoscopy,bronchoscopy;
uppergastrointestinalendoscopy--oesophagoscopy,gastroscopy;
lowergastrointestinalendoscopyflexiblesigmoidoscopy,colonoscopy;and
arthroscopy,thoracoscopy,laparoscopy.
Thissectioncontainssomeexamplesofprocedureswhichtheinternisexpectedtobeableto
performcompetentlyandbeabletoexplaintoapatient.
ThefollowingtableisanAMCcompositechecklistof'ClinicalProceduralSkills'.Studentsare
expectedbygraduationtounderstandtheprinciplesofallofthese,andtobeabletoexplain
themclearlytoapatient.
Competencelevelsexpectedasregardsperformancethusvariesbetweenthosetheyshould
be able to perform and interpret independently (A), perform under supervision and with
guidance(B),orobserveandunderstandprinciples(C).
3-B
Clinical Procedures
539

Many University Clinical Schools make use of similar student log books indicating acquisition of skills and certification
of their performance under supervision of a clinical tutor. We are grateful to the University Clinical Schools of Adelaide,
New South Wales and of Monash for access to their skills lists, which have been amalgamated into the AMC table.
Other important core skills and principles often violated or overlooked relate to appropriate antiseptic and aseptic
techniques, including such basic aspects as care in handwashing between each patient contact; and skills in regard
to basic first aid techniques and primary wound care, together with knowledge of operating theatre protocols
including donning of gowns, gloves and masks and simple splintage and plastering techniques
Clinical Procedural Skills Competency Expectations Final Year Key to Competency
Levels
A = ability to perform and interpret independently and explain principles to patient
B = ability to explain principles, perform, and/or interpret with prompting and guidance
C = observe and understand principles, explain to patient
Core skills (Competency Level A)
At the end of final year the student should be able to perform the following age-appropriate skills competently and
without supervision
Competency Level A
Measure and record vital signs: oral, aural and axillary temperature/pulse/respiratory rate/blood pressure
Testing of urine (urinalysis)
Venepuncture (including knowledge of appropriate containers for common tests)
Insertion of peripheral intravenous cannula
Simulated administration of a drug intravenously
Setting up of an intravenous infusion including blood
12 lead ECG procedure
12 lead ECG interpretation
Use of auroscope/ophthalmoscope
Basic first aid, including CPR
Application of a simple dressing
Basic trauma management
Basic wound care including burns
Basic life support on a mannikin
Application and removal of a forearm plaster
Insertion of a nasogastric tube
Insertion of a urinary catheter (male and female)
Removal of a urinary catheter
Nasopharyngeal aspiration
Administration of immunisation and knowledge of appropriate schedules
Calculation of common drug dosages
Administration and prescription of oxygen
Give a drug orally
3-B
Clinical Procedures
540
Competency Level A (continued) Give a
drug via eyedrops Give a drug sublingually
Give a drug via an inhaler, a spacer and a nebuliser
Give a drug rectally
Give a drug vaginally
Give a drug intranasally
Give a drug transdermally
Give a drug via intramuscular injection
Give a drug subcutaneously
Hand cleaning, asepsis, awareness of sterility and standard precautions Surgical
scrub procedures
Aware of occupational health and safety (OH&S) procedures about blood and body fluid exposure and
needlestick
Diagnose death, complete a death certificate and a cremation form
Demonstrate knowledge of requirements for reporting deaths to the coroner
Demonstrate knowledge of requirements for detention under the Mental Health Act
Demonstrate knowledge of requirements for notification of infectious diseases
Complete a Workcover certificate
Write a discharge letter from a hospital to a general practitioner/local medical officer
Write a referral letter to a specialist Write a hospital consult request
Write an investigation request form
Interpret standard laboratory medicine reports
Interpret commonly used imaging (CXR, AXR, pelvis, spine and long bones)
Specimen handling
Write a prescription (including Authority and SP) simulated
Demonstrate use of relaxation therapy
Lifting and patient handling skills
Use of infiltrative local anaesthesia
Simple nerve block, e.g. ring block/peripheral digits
Suturing of a simple wound
Remove skin sutures and staples
Skin biopsy
Wound swab
Throat swab
Skin swab
Cervical smear/use of speculum
Perform the following components of advanced life support on a manikin intubation Understand principles of
advanced life support on a manikin defibrillation
3-B
Clinical Procedures
541

Supervised Core Skills (Competency Level B)
Supervised core experience (at the end of final year student has performed or assisted with the age-appropriate skill
under direct supervision).
Competency Level B
Normal vaginal delivery
Arterial puncture
Femoral venepuncture
Management of epistaxis by nasal packing
Syringe an ear
Uncomplicated removal of foreign bodies from eyes, ears and nose
Use of slit lamp microscope
3-B
Clinical Procedures
542

Procedural Skills (Competency Level C)
Observed core experience (at the end of final year student has observed and may have assisted with the skill or
procedure and is able to describe and explain it to a patient simply and clearly).
Competency Level C
Advanced trauma management, including defibrillation/revival
Measurement of central venous pressure Insertion of CVP line
Pleural tap, insertion intercostal catheter/chest drain management of underwater drainage
Abdominal paracentesis/diagnostic tap of ascitic fluid; peritoneal dialysis/lavage
Suprapubic bladder tap
Lumbar puncture and measurement of CSF pressure
Simple skin excision
Handling an aggressive patient
Joint aspiration (including practice on a model) knee, shoulder, elbow
Endoscopy upper Gl tract
Colonoscopy
Flexible sigmoidoscopy
Colposcopy
Fine needle aspiration cytological biopsy for common conditions, e.g. breast, thyroid and lymph node
Liver biopsy
Audiometry
Post mortem examination (autopsy)
Bone marrow biopsy Exercise ECG
Echocardiogram
Coronary angiogramand angiographic procedures
Bronchoscopy
Spirometry formal lung function testing
MRI and CT imaging
Ultrasound (abdominal, musculoskeletal, breast and obstetric)
Nuclear medicine scans Electroconvulsive therapy (ECT)
Application of a lower limb traction splint

Peter G Devitt and Barry P McGrath
3-B
Clinical Procedures
543


3-B Clinical Procedures
Candidate Information and Tasks
MCAT 101-104
101 Resuscitation of a 24-year-old man after head and chest injury
102 Fluid balance assessment in a 50-year-old patient after abdominal surgery
103 Evaluation of lung function by spirometry in a 22-year-old man
104 A suspected fractured clavicle in a 20-year-old man
101
Candidate Information and Tasks
544
Condition 101
Resuscitation of a 24-year-old man after head and chest injury

CANDIDATE INFORMATION AND TASKS
You are a Hospital Medical Officer (HMO) in the hospital Emergency Department A 24-year-old man is carried by his
friends into the department, having just driven his car into a telegraph pole immediately outside the hospital. He is
unconscious, barely breathing and with limited chest movement. He vomits as he is carried into the room. His
forehead is bleeding from a head injury from being thrown into the windscreen.

YOUR TASKS ARE TO:
Indicate what measures you would undertake to resuscitate him, describe these to the examiner and
perform them on the manikin.
Following your initial resuscitative measures his colour improves initially, but another several minutes later it
becomes very difficult to ventilate him even squeezing the breathing bag very firmly, and he becomes deeply
cyanosed. Indicate to the examiner what further physical findings you would seek, and what measures you
would then undertake.

The Performance Guidelines for Condition 101 can be found on page 549
102
Candidate Information and Tasks
545
Condition 102
Fluid balance assessment in a 50-year-old patient after
abdominal surgery
CANDIDATE INFORMATION AND TASKS
You are a Hospital Medical Officer (HMO) on a surgical ward; you have been asked to see a middle aged patient who
had a laparotomy for a perforated duodenal ulcer six hours ago. He has a nasogastric tube in position and an
intravenous line administering 5%dextrose. Serum electrolytes and creatinine done prior to surgery showed no
significant abnormality. Operative findings as recorded were an anterior wall duodenal ulcer perforation which was
oversewn without complication.
The nursing staff are concerned that the patient has not passed any urine since the procedure was completed six
hours ago. The patient had been in good health up to the time of the perforation and had been in severe pain for 12
hours before the operation, and had vomited several times. He has had 500 ml_ of 5%dextrose intravenously since
admission. He has a nasogastric tube in situ draining to the bedside. He does not have a urinary catheter.

YOUR TASKS ARE TO:
Assess the patient, and determine the cause of his problem.
Initiate a plan of action: this will include the writing up of any drug, or intravenous or nursing orders.
Outline your plan and orders to the observing examiner.
You have six minutes for your assessment and two minutes to describe your management plan and diagnosis.
.

The Performance Guidelines for Condition 102 can be found on page 551
103
Candidate Information and Tasks
546
Condition 103
Evaluation of lung function by spirometry in a 22-year-old man

CANDIDATE INFORMATION AND TASKS
You are seeing this 22-year-old man in a general practice for an examination for insurance purposes. You have
completed the history-taking and the only finding is a history of mild asthma until the age of 11 years. There have
been no symptoms in recent years and no need for treatment. You have examined the respiratory system which you
find is normal. You are asked to evaluate the patient's lung function using a Vitalograph spirometer as illustrated
below. You are also asked to determine the FEX^ (forced expiratory volume in 1 second) and FVC (forced vital
capacity) and then compare these with normal values. After this you are asked to explain the results to the patient.
YOUR TASKS ARE TO:
Perform spirometry on this patient.
Calculate the FEV, and FVC.
Compare the patient's FEV^ and FVC with predicted values.
Discuss the results with the patient.

CONDITION103. FIGURES 1-3.

The Performance Guidelines for Condition 103 can
be found on page 558
104
Candidate Information and Tasks
547
Condition 104
A suspected fractured clavicle in a 20-year-old man.
CANDIDATE INFORMATION AND TASKS
You are a Hospital Medical Officer (HMO) in an Emergency Department. Your patient is a young adult male who fell
heavily onto the point of his right shoulder earlier today.
He presented with pain in the area and you suspected a fracture of the clavicle on that side because of swelling, mild
deformity and localised tenderness.
An X-ray of the area has just been done.

YOUR TASKS ARE TO:
Examine the X-ray provided.
Inform the patient of your diagnosis and outline a management plan and answer any specific questions asked
by the patient.
Commence treatment using the materials provided.
CONDITION104. FIGURE 1.
The Performance Guidelines for Condition 104 can be found on page 561
3-B
Clinical Procedures
548

3-B Clinical Procedures
Performance Guidelines
MCAT 101-104
101 Resuscitation of a 24-year-old man after head and chest injury
102 Fluid balance assessment in a 50-year-old patient after abdominal surgery
103 Evaluation of lung function by spirometry in a 22-year-old man
104 A suspected fractured clavicle in a 20-year-old man





















101
Performance Guidelines
549
Condition 101
Resuscitation of a 24-year-old man after head and chest injury

AIMS OF STATION
Toassess the candidates knowledge and ability to clear the patient's airway, perform assisted ventilation, cardiac
massage if necessary and circulatory support for patients with severe trauma.
Tosuspect, diagnose and perform emergency decompression of a tension pneumothorax.

EXAMINER INSTRUCTIONS

EXPECTATIONS OF CANDIDATE PERFORMANCE
A patient who has a severe combined head and chest injury needs urgent resuscitation:
Clear the airway remove aspirate, vomitus.
Perform assisted ventilation mouth to mouth, oral airway, ventilate with bag if available, consider endotracheal
intubation.
If initial improvement is followed by significant deterioration and inability to ventilate the patient with an
endotracheal tube in place, the clinician must suspect a tension pneumothorax is present.
Physical findings to be sought:
Percussion and auscultation of the chest reveals tympany, absent or reduced air entry, supported by evidence of
mediastinal and tracheal shift. This confirms the clinical diagnosis of tension pneumothorax on that side and
necessitates immediate insertion of a large bore (18 gauge) needle into the second or third intercostal space anteriorly,
followed by elective passage of an axillary intercostal underwater drainage tube. The candidate should indicate the
tube connections to the underwater drainage bottle.
KEY ISSUES
Knowledge of the AIRWAY, BREATHING. CIRCULATION priority sequence in management.

CRITICAL ERRORS
Failure to follow an appropriate priority sequence in management.
Failure to consider a tension pneumothorax.
Failure to recommend immediate needle decompression thoracentesis.
101
Performance Guidelines
550

COMMENTARY
Emergency management of severe trauma (EMST) involves primary surveys assessing adequacy of airway
breathing, and circulation by initial assessment; and correcting life-threatening complications by procedures such as
those illustrated in this scenario.
The possibility of neck injury must be born in mind, but adequate ventilation must be achieved or the patient will not
survive. Manual holding of the neck to prevent any further injury during the airway management, is clearly necessary.
102
Performance Guidelines
551
Condition 102
Fluid balance assessment in a 50-year-old patient after
abdominal surgery
AIMS OF STATION
Toassess the candidate's ability to manage a common problem postoperative fluid balance and monitoring.

EXAMINER INSTRUCTIONS
The examiner will arrange the station and will have instructed the patient as follows:
The patient is a middle-aged man in pyjamas, lying in a hospital bed or on a trolley.
A nasogastric tube is in position from nares to bedside, taped appropriately to the face, leading to a nasogastric bag
containing 600 ml_ of 'gastric juice' (malt vinegar).
An intravenous line and cannula is in place taped to run beneath a dressing to left forearm, from a full bag of 500 mL
5%glucose (dextrose) in water.
The fluid balance chart and nursing observations sheet appended is at the foot of the bed This will show the following:
1. Pulse and blood pressure in the normal range since admission and surgery 6 hours ago. Temperature is normal.
2. The nasogastric drainage output is 600 mL since surgery (corresponding with the volume in the bag which has not
been emptied since surgery).
3. Intravenous fluid intake since admission has been
500 mL isotonic saline started during the operation and subsequently just finished.
Intravenous orders as currently written are
~ 500 mL 5%dextrose over 12 hours, then
~ 500 mL 5%dextrose over 12 hours then review
The patient's abdomen has a gauze and plastic dressing over the upper abdomen to cover the simulated vertical upper
midline incision.
Analgesics have been ordered on an as required basis morphine 10 mg four hourly sub-cutaneously.
Instructions given to the patient are:
You are recovering from surgery from a perforated ulcer. You have some abdominal pain but this has been reasonably
controlled by an injection 30 minutes ago.
You have not passed urine since the operation. If asked, you gave a specimen prior to surgery; you do not feel
currently a need to pass urine. The nurses have asked the doctor to see you because you haven't yet passed urine.
The doctor will look at your charts and examine you. You have a painful cut in the upper abdomen covered by a
dressing and any undue handling will be distressing. You have a tube in your nose ostensibly passing to your stomach
to drain the gastric fluids. You have an intravenous drip in your left arm
If the doctor palpates your lower abdomen say that it is not particularly uncomfortable and you do not feel the urge to
void if abdominal compression is performed. However, palpation of your upper abdomen will be painful as you have
just had an upper abdominal procedure performed.
102
Performance Guidelines
552

If asked other questions say you feel reasonably well apart fromthe pain in your wound, and you feel rather thirsty
but appreciate that you cannot yet drink.
EXPECTATIONS OF CANDIDATE PERFORMANCE
The candidate would be expected to check the nursing and fluid balance observations and note the absence of
voiding in combination with minimal intravenous replacement over 6 hours since surgery and in the presence of
continuing nasogastric losses.
Optimal acquisition of data would be by checking intraoperative and postoperative fluid charts, to establish input
since admission and to check for extrarenal losses via gastric tube. The patient should be asked if he feels like
voiding. The normal vital signs should be noted; there are no concerning abdominal signs relevant to surgery. The
bladder is not palpable, gentle suprapubic pressure and absence of desire to void would be checked by
knowledgeable candidates. Checking that the jugular venous pressure is not elevated and examination of tissue
turgor for signs of 'dehydration' would be expected (none is present). The JVP is normal and no peripheral oedema
is present.
Appropriate treatment would be to increase intravenous fluid intake after acquisition of all data, with presumptive
diagnosis of insufficient fluid and electrolyte replacement.
The preferred intravenous fluid to order initially would be an isotonic electrolyte solution (saline, Hartmann
solution, 4%dextrose in 1/5 isotonic saline), rather than dextrose alone, but a dextrose bolus would be acceptable
although not the preferred fluid.
Immediate treatment should be intravenous saline 500 mL over 30-60 minutes or so, with review after this.
These instructions should be stated and written.
Some hospitals possess proprietary (ultrasound-based) equipment which the ward staff can use to make
estimation of bladder volume. This will help determine if the failure to void is due to retention of urine or to
oliguria associated with inadequate fluid replacement.
Candidates may suggest passing a urethral catheter, which would be acceptable with review of amount obtained
and subsequent urine output.
Preferably one would review the patient after giving an initial bolus of fluid. If the patient has not produced any urine
within an hour, a urethral catheter should be passed. Urine obtained would be measured for volume and, if available,
a urinary sodium level should be sought to aid diagnosis.
Complete anuria would be a highly unlikely finding under these circumstances, and the finding of a low urinary
sodium level (less than 20 mmol/L) in a small volume of urine would confirm prerenal circulatory insufficiency with
renal conservation of sodium and oliguria. In subsequent monitoring of his urinary output, one would aim at an output
of more than 30 mL hourly.
Appropriate responses are as outlined. In particular candidates should appreciate that appropriate volume
replacement (after checking for signs of volume overload venous filling, peripheral oedema, auscultate lung
bases) would preferably be by increased intravenous electrolyte solution replacement (isotonic saline. Hartmann
solution, 4%dextrose in 1/5 isotonic saline) rather than dextrose/water alone. Administration of a diuretic prior to
fluid replacement would indicate a clear fail.
If no commentary or plan is forthcoming by six minutes the examiner will ask:
What is your diagnosis?'
What orders will you give?'
102
Performance Guidelines
553
KEY ISSUES
Assess the candidate's ability to determine the most likely cause for apparent postoperative oliguria with failure of
early voiding after surgery by examination of nursing and fluid balance data and patient assessment.
Assess the candidate's ability to take appropriate corrective action and to recognise the need for continued
monitoring and review.
CRITICAL ERRORS
Administration of a diuretic without previous fluid replacement.
Failure to order an increased rate of intravenous replacement of appropriate type.
Failure to suggest continuing review after trial of more fluids.
COMMENTARY
Surgical operations have the potential to upset the normal water and electrolyte balance of the body, in which daily
intakes and outputs are usually finely balanced and equivalent, as shown below in an adult:
CONDITION102. TABLE 1.
Water Input

Water Output
Ingested Liquids 1.500 mL = Urine 1,500 mL
Water of food 1,000 mL = Insensible losses 1,000 mL
Water of metabolism 100-200 mL = Faecal losses 100-200 mL
TOTALS 2.5-3 litres =

2.5-3 litres
This simplification is administratively and clinically convenient:
In normal circumstances measured ingested water balances and equals measured urine production;
unmeasurable water in food is balanced by unmeasurable insensible losses from lung and skin; and
unmeasurable water produced within the body, by metabolism of ingested foodstuff, is balanced by unmeasured
water loss in faeces.
Urine output can be monitored daily or more frequently. A urethral catheter is necessary in the severely ill or shocked
patient. A urine output of 40-50 mL per hour is reasonable evidence of satisfactory renal perfusion; urine output relates
closely to renal blood flow in shock states Accordingly, urine output monitoring is one of the most useful guides
we have in following progress of shocked patients. A 24-hour urinary output of 1000 mL or greater is within the
normal range.
Early diagnosis of postoperative oliguria is extremely important. Incipient acute renal failure can often be recognised
early and reversed.
102
Performance Guidelines
554


Changes in Fluid Balance during the Postoperative Period
There is increased secretion of many hormones after surgery (catecholamines, Cortisol and adrenocorticotrophic
hormone (ACTH), aldosterone, antidiuretic hormone (ADH), glucagon, growth hormone) and decreased secretion of
others (e.g. insulin). The net result is that both salt and water are retained temporarily by the kidneys. This results in
a transient and minor fall in urine flow for 24-48 hours after major injury and a dilution of electrolytes in serum and
extracellular fluid. Sodium is also retained, but not to the same degree as water, so sodium is also temporarily diluted.
The reality of this is easily checked by looking at virtually any serum electrolyte measurement after an operation, and
seeing that the levels of ions such as sodium often seemto fall below their normal range, and plasma osmolality is also
lowered. Remember that this effect is a dilutional effect and does not imply a need for giving more ions quite the
opposite in fact, since ions are already being retained by the kidneys.
After the first 24-48 hours, the hormonal effects on the kidneys progressively disappear, and the kidneys can be
relied on to make good the electrolyte adjustments required by the body provided they are given enough fluid and
electrolytes with which to work.
Postoperative Water Balance
Obligatory water losses are around 1500 mL per 24 hours (1000 mL insensible loss from lungs and skin, 500 mL
minimal obligatory urine volume). Obligatory water losses may increase after surgery because of tachypnoea or
fever. However, renal losses diminish often to between 700-1000 mL in the first 24 hours. Therefore, 2.5 L per 24 hr
is usually adequate fluid replacement for adults in the first 24 hours after surgery (not 1,5 litres as is currently ordered
in this man).
As the kidneys begin to excrete more water, the amount of fluid should be increased to approximately 3 L per 24
hours. This is the baseline replacement for adults of average spectrum of size and weight. But it must be remembered
that any other losses (e.g. nasogastric suction), must be replaced in addition to the 2.5 or 3 L; and this patient has
already lost an additional 600 mL of gastric aspirate.
Postoperative Electrolyte Balance Sodium
Renal excretion of Na+ diminishes in the first 24 hours after surgery due to increased aldosterone and Cortisol
secretion so there may be no need to give Na+ in this period. However, unidentified losses of Na
+
can occur with
injury or surgery (third space losses); and because of the uncertainty, it is more common to see sodium being given
than withheld during this period up to one litre daily, and usually this causes no problems. Nevertheless, it is well to
remember that this is probably in excess to body requirements and sodium should be withheld or given
with caution in elderly patients, or those with failing hearts. This patient has no such problems and we have
evidence of continuing gastric electrolyte losses of sodium.
Once the first 24 hours is past, then sodium requirements return towards normal, and a figure of 150 mmol per day is
adequate (which conveniently is represented by 1 L of normal saline).
With the hormone-driven retention of sodium in the first 24 hours, there tends reciprocally to be increased K+ loss in
the urine. However, this loss is small, and not until after this first 24-hour period is it important to replace K+. Note also
that potassium given intravenously
102
Performance Guidelines
555
immediately after surgery will just tend to be excreted by the kidney. After the first 24 hours 2gm of K+ in a litre gives
26.8 mmol of K; and so if given in 3 L, provides around 80 millimoles K
+
a day, which is quite adequate.
Intravenous Fluids Used in Surgery
a) 5% Dextrose in Water: This is isotonic (isosmolar) with extracellular fluid, and is therefore the solution to use if you
think a patient is dehydrated (i.e. deficient in water alone for example by fever or absence of normal intake) and
needs water. Giving 500 mL of 5%Dextrose intravenously over a short period, say an hour, is a useful manoeuvre
if you think dehydration (i.e. pure water deficit) is the cause of a patient's low urine output. This can be thought of as
giving the patient a drink intravenously. This patient needs a drink of salty water, however he is being
'de-salinated' by his previous vomiting and peritonitis and by continuing gastric losses.
b) Normal (isotonic) Saline: This is 0.9%NaCI and in one litre contains approximately 150 mmol of Na+ and 150
mmol of CI"; so at 300 millimolar concentration it is roughly similar in equivalence to extracellular fluid it might be
regarded as unsophisticated extracellular fluid replacement. This form of fluid and electrolyte depletion (ECF) is
by far the one most commonly seen in surgical patients, and gives classical signs (firm woody tongue, decreased
tissue turgor, sunken eyes).
c) 4% Dextrose in 1/5 normal Saline: This is an isotonic solution comprising 4/5 dextrose and 1/5 saline; 1/5 isotonic
Saline (equals 30 millimolar Na) is mixed with 4%Dextrose to increase the tonicity back to that of plasma. Three
litres daily of this solution gives 90 mmol sodium which is usually quite adequate for daily requirements.
d) Solutions such as Ringer-Lactate and Hartmann solution use lactate to increase their tonicity and more closely
mimic extracellular fluid composition. They also contain some of the other cations and anions of extracellular fluid
(K, Ca. Mg. CI) in roughly the concentration found in extracellular fluid. Therefore, they can be regarded as
sophisticated extracellular fluid replacement. However remember that lactate is metabolised in the liver, with the
release of bicarbonate. Thus in acidotic situations it makes good sense to give such solutions; but not if the patient
is alkalotic.
e) Potassium-containing fluids: Potassium is the main cation of intracellular fluid. Potassium requirements are
broadly similar to sodium. But in contrast to sodium, extracellular and intravascular concentrations of potassium are
low (3-5 mmol/L). and high concentrations are cardioplegic and can cause cardiac arrest. So NEVER give
potassium in a higher concentration than 30-40 mmol/L, and preferably do not exceed 26.8 mmol/L (2 gm KCI in 1
litre).
f) Haemaccel: This is used in situations where there has been rapid blood loss and when it is important to maintain a
normal blood volume while awaiting blood for replacement. It consists of sulphated gelatine (produced from
crushed horses' hooves, so it can be allergenic) with an average molecular weight of 30,000 Daltons. It is more or
less isotonic and the gelatine has a half life of between 30-120 minutes, so its use is very much an interim measure
to swell blood volume.
The patient needs isotonic replacement for suspected insufficient ECF ('Saline') replacement together with
replacement of continuing losses (i.e. gastric aspirate), which can be thought of as roughly equivalent to the same
volume of isotonic saline. Intravenous hydrochloric acid cannot be given readily, and gastric juice contains a lot of
sodium as well as hydrogen ion and chloride.
102
Performance Guidelines
556
Distinguishing
acute prerenal postoperative oliguria due to prerenal circulatory insufficiency from
established acute oliguric renal failure due to acute tubular necrosis, is very important, as the first responds to
volume loading, and in the latter case the kidneys are unresponsive. Apart from the important clinical signs and
the initial response to a bolus loading, help may be obtained from urinary electrolyte findings (Table 2).
CONDITION102. TABLE 2.
Urinary findings expected in postoperative oliguric patient

Prerenal oliguria (renal circulatory
insufficiency)
Intrinsic acute oliguric renal failure
(acute tubular necrosis)
Urine sodium Low (< 20 mmol/L) High (> 50 mmol/L)
Urine osmolality or specific
gravity
High (> 1,500 mmol/L) (S.G >
1020)
Low (< 1,000 mmol/L) (S.G <
1050)
Urine urea and creatinine
High Low
The urine composition in the instance of intrinsic renal failure resembles an ultrafiltrate of plasma due to impaired
tubular function.
This would be a highly unlikely diagnosis in the current circumstances, with no preoperative evidence of pre-existing
renal impairment, and no evidence of intraoperative complication.
Composition of Abnormal Losses Seen in Surgical Patients
Gastrointestinal fluids all contain significant amounts of sodium and varying amounts of chloride and bicarbonate
(Table 3). For unsophisticated and usually satisfactory replacement of abnormal gastrointestinal fluids one can use
simple isotonic 0.9%saline. The only caveats are with small children whose kidneys are not yet fully capable of
handling sodium loads, and with elderly frail patients with cardiac insufficiency.
Gastrointestinal Secretions Daily

Fluid
(litres)
Sodium
(mmol/L)
Potassium
(mmol/L)
Chloride
(mmol/L)
Bicarbonate
(mmol/L)
Saliva 1.0 50 20 40 50
Gastric juice 1.5 100 15 200
-
Bile 1.0 100 5 40 40
Pancreatic juice 1.5 150 5 40 120
Intestinal juice 2.0 150 5-15 110-150 5-30
CONDITION102. TABLE 3.
102
Performance Guidelines
557

Note that it isotonic saline is used to replace prolonged bowel fistula losses, deficiencies of potassium and of
bicarbonate are likely to be induced so that more sophisticated intravenous fluids, including special hypertonic
intravenous parenteral nutrition fluids delivered by a central line, may be necessary. For temporary intravenous fluid
management over only several days, simple isotonic intravenous solutions given by peripheral limb lines are all that is
usually required.
103
Performance Guidelines
558
Condition 103
Evaluation of lung function by spirometry in a 22-year-old man
AIMS OF STATION
To assess the candidate's ability to perform and interpret spirometry in a subject with previous asthma.
EXAMINER INSTRUCTIONS
The examiner will have instructed the patient as follows:
You are a 22-year-old accountant. You had occasional mild attacks of asthma until the age of 11 years. You required
only occasional bronchodilator treatment with Ventolin. You have had no symptoms since the age of 11 years. You
jog regularly and play tennis without any limitation. You are otherwise very well.
You have not had your lung function measured previously. Listen carefully to the doctor and do as you are asked.
When performing the first blow do it incorrectly by taking less than a full breath in and blowing out slowly for six
seconds before giving a maximal effort. These manoeuvres should be practised with the examiner before the start of
the first candidate. If the doctor does not give you appropriate instructions repeat the same error.
Only give a maximal effort if instructed to do this.
Questions to ask if not already covered:
What do the results mean?'
If told results are normal, ask 'Does it mean I no longer have asthma?'
EXPECTATIONS OF CANDIDATE PERFORMANCE
The candidate is expected to:
Use a nose clip.
Insert a clean mouthpiece.
Advise the patient to:
~ undertake a maximum inspiration;
~ give maximum effort during expiration;
~ ensure expiration continues for six seconds, or until candidate comments that the technique was faulty when
the effort can be abandoned; and
~ repeats the manoeuvre until acceptable and reproducible efforts have been achieved; reproducible efforts are
less than 200 mL or less than 5%variation for both FEV1 and FVC.
Identifies FEV1 (forced expiratory volume in 1 second) and FVC (forced vital capacity) correctly (do not penalise
for use of wrong scale).
Compares patient's FEV1 and FVC with normal values and calculates FEV1 as a percentage of FVC.
Disposes of used mouthpiece.
103
Performance Guidelines
559
Explains test results which are very likely to be normal. Normal results do not indicate that asthma has remitted; they
indicate that at present there is no evidence of airway obstruction

KEY ISSUES
Approach to patient gives clear advice and corrects errors with constructive advice.
Familiarity with test equipment
Performance of procedure or task detects errors and obtains reproducible efforts.
Interpretation of investigation accurately calculates FEV1 and FVC.
Patient counselling explains test results.

CRITICAL ERRORS
Failure to recommend maximal inspiration
Failure to recommend maximal expiratory effort.
Failure to calculate FEV1.

COMMENTARY
The diagnosis and ongoing management of asthma require objective measurements of reversible airways obstruction
using pulmonary function tests. The diagnosis of asthma is usually confirmed by spirometry measurements before and
after the administration of inhaled, short-acting -agonist bronchodilator.
Although spirometry is ostensibly a simple test that can be performed in a medical practitioner's office and requires the
recording of a forced expiratory volume-time curve using a spirogram, there are some pitfalls that must be avoided to
ensure that valid and accurate results are obtained.
Patient education is critically important to ensure compliance and best effort by the patient. Forced expiratory volume
measurement must be recorded from a state of maximal inspiration. All expired air must be captured and nasal air
escape or escape of expired air around the mouthpiece due to poor sealing of the lips around the mouthpiece, or
partial tongue obstruction of the mouthpiece, can give false readings
The two key components of the test result are the forced expiratory volume that is produced in the first second (FEV1)
and the percentage this contributes to the total expiratory volume or forced vital capacity (FEV1/FVC). Typical
examples are shown in the figures.
103
Performance Guidelines
560
CONDITION103. FIGURE 4.
Most patients with asthma will show FEV1 < 80%predicted normal value and FEV1/FVC < 80%predicted normal
value plus a significant (>10%) increase in FEV1 after bronchodilator. The following factors are taken into account in
determining predicted normal values: age, gender, weight, height, surface area.
Some patients with asthma will have normal spirometry test results, but sensitivity of airways to bronchoconstrictor
stimuli and additional pulmonary function tests, including lung volume and diffusing capacity measurements, can be
assessed further in a specialised pulmonary function laboratory.
CONDITION103. FIGURE 5
104
Performance Guidelines
561
Condition 104
A suspected fractured clavicle in a 20-year-old man
AIMS OF STATION
To assess the candidate's ability to recognise a fractured clavicle on an X-ray film of the shoulder area (the degree of
deformity is mild so that open reduction/fixation is not indicated), and to advise and initiate appropriate treatment by
clavicular restraint.
EXAMINER INSTRUCTIONS
The examiner will have instructed the patient as follows:
You fell heavily onto the point of your right shoulder earlier today. The area has been painful since. Movement of the
shoulder increases the pain. You have sought medical advice the doctor has examined you and arranged for an
X-ray of your shoulder. This has just been done.
The doctor will now discuss the injury and its treatment with you. Ask the doctor to show you the fracture on the X-ray.
Opening statement
Is a bone broken, doctor?'
Questions to ask unless already covered (answers in parentheses after questions):
'How long will I need to keep the arm in the sling?' (Until review at two weeks)
'Can I remove the sling to exercise my elbow?' (Yes)
'What about in bed?' (Elevate shoulder and arm on pillows)
'Do I have to have it X-rayed again?' (Not if clinical union proceeds normally)
'Will there be any disfigurement?' (Minor residual bony thickening)
'When will I be able to go back to work?' (Depends on work and progress in regaining use of arm often 2-4
weeks)
EXPECTATIONS OF CANDIDATE PERFORMANCE
The candidate is expected to diagnose the clavicular fracture and to begin appropriate conservative treatment with a
supportive sling or other clavicular restraints.
The candidate should:
Tell the patient what is wrong and what will happen including a time frame for treatment.
Commence treatment demonstrating skill in maintaining alignment of the clavicular fragments using methods which
will pull the whole shoulder girdle upwards and backwards (figure of eight bandaging or clavicular restraint shoulder
rings tied together at back): and which immobilise and support the shoulder by the use of a sling. A well applied sling
elevating and supporting the arm would be the most appropriate form of treatment with this minimally displaced
fracture.
Avoid complications by testing for excessive tightness of the bandage/rings, advising finger, wrist and elbow joint
exercises, removal of the sling after 7-10 days, and the bandage/rings if used, after three weeks when there is
clinical evidence of union beginning (unduly prolonged immobilisation can result in a stiff shoulder).
104
Performance Guidelines
562


KEY ISSUES
Ability to recognise fracture of clavicle on X-ray.
Skill in treatment by optional use of figure of eight bandage or clavicular restraint shoulder rings; sling for arm
and forearm (supporting shoulder).

CRITICAL ERRORS
Failure to recognise fracture on X-ray.
Failure to apply some form of appropriate shoulder and arm support

COMMENTARY
Fractures of the clavicle are common in younger patients from falls on arm or shoulder. The usual break occurs at
the midclavicle and the outer fragment is carried with the arm downwards and forwards, while the inner fragment
usually rides up giving a step deformity. Healing is usually rapid with conservative treatment nonunion can occur
but is rare.
Treatment consists of elevating and supporting the arm with a sling which helps relieve discomfort and helps to
minimise displacement. Application of a sling is illustrated (Figures 2-5)
Additional clavicular restraints can be used such as a figure of eight bandage, or shoulder rings tied together from
behind to brace back the shoulders into the 'position of attention'. All of these tend to loosen quickly and to require
frequent adjusting. Union is usually accompanied by a localised boss of exuberant callus. Remoulding occurs in the
young with time and the cosmetic result is usually acceptable. Return of full arm function is also usual.
104
Performance Guidelines
563
CONDITION104. FIGURE 2. CONDITION104. FIGURE 3.

CONDITION104. FIGURE 4. CONDITION104. FIGURE 5.

Application of a supportive sling

564
Integrated Diagnosis and
Management (D/M>
4
4-A:ClinicalPerspectivesandPriorities


BryanWYeo
' Experienceisthemotheroftruth;andbyexperiencewelearnwisdom.'
WShipperton(1736-1808)

INTRODUCTION
Clinical perspective is an essential skill that every clinician
muststrivetodevelop.Itisneverfullyachieved,butshouldbe
steadily improved and refined throughout the lifetime of the
clinician. Proper perspective facilitates patient management,
avoids unnecessary delay, allows better cost-effective
treatmentandachievesbettercommunicationwiththepatient,
especiallywhentheprognosisispoor.
Common things are common. This must be kept in mind as it
helps priority-oriented management. Dysphagia associated
with an oesophageal stricture could conceivably follow a
lightningstriketoaswordswallowerbutismuchmorelikelyto
be due to one of two common causes chronic reflux
oesophagitisoracarcinomaoftheoesophagus.
Pain in the right iliac fossa always raises the possibility of
appendicitis.Episodesofspontaneouslyresolvingnonspecific
abdominalpainwithminimallocalsignsarecommonlyseeningeneralpractice,butpersisting
acutelocalpainwithtendernessandguardingintherightiliacfossaoftheabdomenismostlikely
duetooneofthreeconditionsappendicitis,atubal-ovarianproblem(ovariancyst,salpingitis,
ectopic pregnancy), or a urinary tract problem (urinary calculus or infection). These should be
consideredfirstinthediagnosisbeforeotherlesscommoncauses(ofwhichtherearemany).A
22-year-oldmanwithcentralabdominalpainwhichshiftstotherightiliacfossa, withanorexia
andlocaltendernessandreboundtendernessatMcBurneypointneedsanappendicectomy,and
notawhitecount,CTorultrasoundoftheabdomenorabatteryofotherunnecessarytests.This
basic maxim regrettably continues regularly to be overlooked in hospital emergency
departments.
A22-year-oldmanwithcentral
abdominalpainwhichshiftsto
therightiliacfossa,with
anorexiaandlocaltenderness
andreboundtendernessat
McBurneypointneedsan
appendicectomy,andnota
whitecount,CTorultrasound
scanoftheabdomenorabattery
ofotherunnecessarytests.This
basicmaximregrettably
continuesregularlytobe
overlookedinhospital
emergencydepartments.
Perspectiveskillsimprovewithaccurateknowledgeoftheunderlyinganatomyandphysiology.A
forty-year-oldwomanwithabdominalpainandsignsofperitonitisintherightupperquadrantof
theabdomenismostlikelytohaveacutecholecystitisassociatedwithgallstones.Butalsokeep
inmindacommonconditionsuchasappendicitisoccurringinalesscommonsiteliketheupper
rightquadrant.
Simply diagnosing that a patient has 'gallstones' is not enough. The site of the gallstones is
criticaltothepatient'sconditionandapriorityindiagnosis.Correctsitingdeterminesthecorrect
timing of surgical treatment with the least risk of serious complications. A patient with severe
acute cholecystitis due to a stone obstructing the cystic duct should undergo cholecystectomy
withinfivedaysoftheonsetoftheattack.Apatientwithpancreatitisdue
565
4-A
Clinical Perspectives and Priorities
566


to gallstones passing through the common bile duct is best managed initially with conservative treatment of the
pancreatitis; cholecystectomy is performed when the pancreatitis has subsided in the second or third week of the
initial admission. Thus, the particular complication caused by the gallstones will prioritise the timing of the surgery.
Never is there a more important need for clinical perspective than in discussion with a patient who has surgery for a
malignant disease. After resection of a colonic carcinoma, the subsequent development of malignant ascites
associated with peritoneal metastases is an indication that the patient is unlikely to be helped by further surgery in
the form of an open laparotomy. Furthermore, this later occurrence of ascites would have been predicted as a likely
future complication after the first operation when the histopathology had revealed that the carcinoma had spread
through the full thickness of the bowel into the omentum and pericolic tissue. Discussion of the prognosis with the
patient after the first operation would be guarded and with discretion, based on the spread of the carcinoma identified
in the histopathology of the resected bowel. Similarly, a patient with carcinoma of the pancreas and back pain due to
lymphatic or perineural spread of the carcinoma is unlikely to be cured by a pancreatic resection.
Clinical perspective is enhanced by knowing the frequency of a condition in a particular region. In a country where
tuberculosis is widespread, drug treatment may be appropriately commenced before waiting for the results of
cultures that may take several weeks. An experienced clinician correlates the frequency of the disease or condition
in a particular location where the patient lives or has been, with cognisance of the patient's age and sex, and paying
close attention to the anatomy and function relevant to the site of the clinical features. In this way, clinical perspective
and priority management are enhanced.
CLINICAL REASONING PERSPECTIVES
Occam's razor often provides useful diagnostic perspectives, applying the principle of diagnostic
parsimony/unification, thus explaining multiple phenomena by a unifying single cause.
Example 1 Melioidosis:
A 50-year-old surgeon felt quite well after returning from an interstate congress, until performing poorly at his usual
game of recreational tennis (he served three double faults in a row, felt cold, and unduly tired despite minimal
exercise).
He lay down for a rest. His wife noted an hour later that he was ashen-faced, clammy and shivering uncontrollably.
She summoned an ambulance and he was taken to hospital. His temperature was 40 C, pulse 130/min, blood
pressure 110/60 mmHg. He looked very ill. The only positive findings were basal lung crackles. X-ray showed
bronchopneumonia changes. Arterial blood gas analysis showed gross hypoxia and desaturation. Blood cultures
were taken and immediate treatment with broad spectrum antibiotics was begun.
After 24 hours his condition had improved somewhat but fever and chest signs persisted. The respiratory physician
in charge was not satisfied that the diagnosis was of pneumonia alone. Further detailed history showed that the
patient and some colleagues had gone on a far northern Australia tour immediately after the conference. While
wading in a lagoon he had abraded his heel drawing a bead of blood. Shortly after the improvement in his chest
signs, his heel was noted on re-examination to be slightly swollen and tender, with an inconspicuous area of
subcutaneous induration in the adjacent ankle region as illustrated.
4-A
Clinical Perspectives and Priorities
567
SECTION 4-A. FIGURE 1. SECTION 4-A. FIGURE 2.
Site of heel abrasion Subcutaneous induration
around ankle
The diagnosis of melioidosis was subsequently confirmed on serum antibody testing. He had continuing antibiotic
treatment for several weeks with progressive improvement. One month later he had fully recovered.
Melioidosis is caused by a free-living, small, motile, aerobic Gram-negative saprophytic bacillus (Burkholderia
pseudomallei, formerly called Pseudomonas pseudomallei) found in soil and ponds in tropical environments. Infection
is through soil contamination of abrasions followed by subcutaneous infections, bacteraemia and lesions in the lung
and elsewhere. The organism is endemic in northern tropical Australia. Fulminating infection can be rapidly
progressive and requires life-saving prompt antibiotic and resuscitative treatment without awaiting confirmation from
culture or immune antibody tests. Careful clinical assessment in this instance led to linkage between the primary site
of infection (initially nonsymptomatic and overshadowed by the subsequent prostrating illness), and the associated
pneumonia to identify a single cause.
Example 2 Breast cancer:
A 60-year-old woman presented with persisting back pain. She had breast cancer surgery two years before. Think of
bony metastasis ahead of osteoporosis.
Example 3 Melanoma:
A 40-year-old man presented with a firm nodal groin lump. Direct questioning was required to reveal the history of
removal of a small mole from the lower leg two years before. The diagnosis was metastatic melanoma.
Occam's razor is likely to be of particular benefit when applied to the systemic effects of malignancies, or linked
manifestations of widespread septic, vascular or metabolic disease.
4-A
Clinical Perspectives and Priorities
568


Example 4 Melanoma:
A 45-year-old woman had a malignant melanoma excised from her thigh. Two years later she presented with a
mechanical small bowel obstruction requiring surgical relief. She had no previous history of abdominal surgery. The
preoperative diagnosis, which was confirmed at surgery, was based on knowledge of the natural history of modes of
spread of melanoma. She had a metastatic melanotic deposit in the small bowel submucosa causing an intus-
susception a classical although infrequent association.
Example 5 Lymphoma:
A 75-year-old man presented with a slowly growing, painless subcutaneous lump in the left cheek. Examination
findings were of a focal firm discrete lump 2.5 cm in diameter, in the substance of the parotid gland. There was no
facial nerve involvement and no intraoral extension. No other neck lumps were apparent, nor were there any
suspicious skin lesions of scalp or neck. The clinical features were consistent with the diagnosis of a pleomorphic
adenoma of the parotid, the most common parotid neoplasm, requiring superficial parotidectomy.
Further questioning revealed that five years previously he had had a left groin lump removed, which on histology was
a low-grade non-Hodgkin follicular lymphoma. No other manifestations of lymphoma were found at that stage. He
was treated expectantly after local excision, and had had no intervening symptoms.
Computed tomography (CT) of the head and neck confirmed a 2.5 cm focal solid lump of homogeneous texture
within the superficial left parotid, without other abnormality.
Fine needle aspiration cytology (FNAC) was performed and showed a uniform cellular aspirate of small lymphoid
cells consistent with a low-grade lymphoma. Flow-cytometryon the aspirate confirmed this as a low-grade B cell
monoclonal lymphoma consistent with the previous groin node pathology.
He was treated by intravenous infusion of the monoclonal antibody rituximab (Mabthera) with rapid halving in size
of the lump. Over the subsequent five years no change in the parotid lump
occurred; a few nodes in the upper cervical chain became palpable but remitted
after two further infusions.
He remained well and symptom-free for five years.
He then developed acute dysphagia for solids and liquids, and an oesophageal
carcinoma was suspected. Oesophagoscopy showed narrowing of the lower
oesophagus with benign histology on endoscopic mucosal biopsy.
A chest CT and barium swallow showed extrinsic compression of the lower
oesophagus by an enlarged posterior mediastinal lymph node mass, over which
the oesophagus was stretched with dilatation of the oesophagus above the
stricture (Figure 3). After another course of rituximab infusions his symptoms
rapidly improved and a repeat barium swallow six weeks later showed a normal
oesophagus with resolution of the narrowing and shrinkage of the mediastinal
nodal mass.
SECTION 4-A. FIGURE 3.
4-A
Clinical Perspectives and Priorities
569


He continues symptom-free and has resumed active professional and
recreational activities (Figure 4).
The above five examples illustrate the useful application of Occam's
razor in systemic infective and neoplastic disease.
However, 'Patients may have both fleas and lice (Tannhauser)'. With
longer lifespans comes a greater tendency for multiple unrelated
clinical problems to occur: so the razor is perhaps less sharp than it
used to be.
Example 6 Diagnosis of small bowel obstruction:
A 50-year-old woman had previous abdominal gynaecologic surgery for
fibroids five years ago and presented with a recent onset of clinical
features suggesting small bowel obstruction. Initially treated
conservatively as postsurgical adhesive obstruction, the diagnosis proved to be an obstructing caecal cancer when
she later came to surgery
SECTION 4-A. FIGURE 4.

Bryan W Yeo
4-A
Clinical Perspectives and Priorities
570

4-A Clinical Perspectives and Priorities
Candidate Information and Tasks
M CAT 105-112
105 Abdominal pain and vaginal bleeding in a 39-year-old woman after 9 weeks amenorrhoea
106 Recent insomnia in a 25-year-old man
107 Dandruff or head lice in a 6-year-old girl?
108 Recent orchidectomy for a testicular neoplasm in a 28-year-old man
109 Postnatal fatigue and exhaustion in a 28-year-old woman
110 Fundus greater than dates in a 26-year-old woman at 30 weeks gestation
111 Tiredness and anaemia in a 55-year-old woman
112 Colonoscopy findings in a 24-year-old man with chronic diarrhoea
105
Candidate Information and Tasks
571
Condition 105
Abdominal pain and vaginal bleeding in a 39-year-old woman after 9 weeks
amenorrhoea
CANDIDATE INFORMATION AND TASKS
Your patient is a 39-year-old woman who has been married for 12 months and suffered a spontaneous abortion at
eight weeks of gestation six months ago. This was her only previous pregnancy.
An ambulance has brought her to the hospital today because of severe lower abdominal pain and heavy vaginal
bleeding for the last 12 hours. Her last period was nine weeks ago. You are seeing her in the Emergency Department
at the local hospital

YOUR TASKS ARE TO:
Take any further relevant history you require (it should not take you more than 3-4 minutes to do this).
Ask the examiner for the appropriate findings you would look for on general and gynaecological examination,
and then ask for any investigation results you feel are necessary to enable you to make a diagnosis.
Advise the patient, in lay terms, of the diagnosis and the subsequent management required.

The Performance Guidelines for Condition 105 can be found on page 579
106
Candidate Information and Tasks
572

Condition 106
Recent insomnia in a 25-year-old man
CANDIDATE INFORMATION AND TASKS
You work in a family medical centre. You are seeing the son of a recently deceased patient. You have known the
family for many years, and have seen the son on many occasions before with childhood illnesses. You saw him last
six months ago (prior to him taking flying lessons) at which time a full general examination was normal.
The son is now 25 years old, single and a university student. He is consulting you because of difficulty sleeping over
the past month since the unexpected death of his father. Prior to the death there had been no problems sleeping,
except immediately prior to the university examinations. There is no relevant past history and his general health
previously has been excellent.

YOUR TASKS ARE TO:
Take a focused and relevant history, to enable you to diagnose the cause of the insomnia.
Develop a management plan.
Counsel the patient.

The Performance Guidelines for Condition 106 can be found on page 582
107
Candidate Information and Tasks
573
Condition 107
Dandruff or head lice in a 6-year-old girl?

CANDIDATE INFORMATION AND TASKS
You have just examined 6-year-old Jodie, who has been brought to a general practice by her mother because of an
itchy scalp and what appears to be dandruff in her hair.
Her parent is concerned that she may have lice in her hair which have been reported among other children at her
school. Jodie has excellent health otherwise. She has a 4-year-old brother who occasionally scratches his scalp. On
inspection of her scalp you have found 'white specks' which are firmly stuck to the hairs as illustrated
Eyelashes and eyebrows show no abnormality. There is no scalp dermatitis, though some redness consistent with
scratching is evident.

YOUR TASK IS TO:
Explain your diagnosis and management plan to the parent, who is concerned as to whether this is lice or dandruff.
CONDITION107. FIGURE 1.
The Performance Guidelines for Condition 107 can be found on page 585
108
Candidate Information and Tasks
574
Condition 108
Recent orchidectomy for a testicular neoplasm in a 28-year-old man
CANDIDATE INFORMATION AND TASKS
You are working in a metropolitan general practice. A 28-year-old university student recently had his testis removed
(three weeks ago) because he had noticed a painless lump in the scrotum. The surgeon had told him that it was a
tumour of the testis and gave him photographs of the tumour that has been resected (see below). The student has
now moved interstate because of his studies and he has consulted you. his new general practitioner, concerned
about his future management. The surgeon said that he would forward details of the operation once informed of the
new general practitioner's name, and that the patient would need further tests and followup. You have just finished
examining the patient. He has a well-healed right inguinal incision, the remaining testis feels normal, there are no
abnormalities on examination of abdomen, chest, or head and neck.

YOUR TASK IS TO:
Discuss his followup and answer his questions and concerns.

There is no need for you to take any additional history or perform any examination

CONDITION108. FIGURE 1.
CONDITION108. FIGURE 2.
The Performance Guidelines for Condition 108 can be found on page 587
109-110
Candidate Information and Tasks
575
Condition 109
Postnatal fatigue and exhaustion in a 28-year-old woman

CANDIDATE INFORMATION AND TASKS
You are working in a general practice. The patient is a 28-year-old mother of two children who presents with a
two-week history of broken sleep, fatigue and exhaustion. She is accompanied by her husband. Peter, who is a
30-year-old manager of a travel agency. They live locally with his parents whilst they save for a house. They have a
10-week-old son, Thomas, who is breastfed, and 30-month-old daughter.
She is a nonsmoker and nondrinker.
Your practice records show that both pregnancies and deliveries were normal and uneventful.

YOUR TASKS ARE TO:
Take a further appropriate history to evaluate the possible causes of her symptoms. You do not need to perform
a physical examination.
Provide a likely diagnosis to the examiner and give your reasons for selecting this diagnosis.
Outline briefly your management approach to the examiner.

The Performance Guidelines for Condition 109 can be found on page 589

Condition 110
Fundus greater than dates in a 26-year-old woman at 30 weeks gestation

CANDIDATE INFORMATION AND TASKS
Your patient is a 26-year-old primigravida. She has been attending the general practice where you are working and
seeing the doctors there in a shared care arrangement with a specialist in a major city 30 km away. She is not due to
see the specialist again for a further six weeks. All appeared to be normal up to and including her last visit at 26 weeks
of gestation, when the symphysis-fundal height was 28 cm. Today, four weeks later at 30 weeks of gestation, the
symphysis-fundal height is 40 cm, and a weight gain of 6 kg has occurred during the four week time interval.

YOUR TASKS ARE TO:
Take any further relevant history you require.
Ask the examiner about the relevant findings on examination and the results of specific previous investigations
which you believe would have been performed.
Advise the patient of the diagnosis and subsequent management.

The Performance Guidelines for Condition 110 can be found on page 593
111
Candidate Information and Tasks
576
Condition 111
Tiredness and anaemia in a 55-year-old woman

CANDIDATE INFORMATION AND TASKS
You are working in a general practice. Your next patient is a 55-year-old woman who consulted you two days ago
complaining of recent fatigue and mild breathlessness on exertion. There were no other symptoms. Past and family
histories were not helpful. The patient is a nonsmoker, rarely drinks alcohol, and has a normal diet. She is two years
postmenopausal.
On clinical examination the only abnormal finding was pallor. She had a resting pulse rate of 88/min (regular).
Cardiovascular system was otherwise normal. Abdominal and rectal examinations were normal. Urine examination
was normal.
You suspected anaemia and arranged a full blood examination. The results are as follows:

Haemoglobin 97 g/L (120-150)
WCC 8.5 x 109/L (4.5-11)
MCV 74 fL (80-97)
Film The red cells show some microcytosis, anisocytosis and
poikilocytosis.

YOUR TASKS ARE TO:
Tell the examiner what further blood tests are required to confirm the type of anaemia. The results will be
provided.
Explain the problem to the patient.
Advise the patient about what should be done.
Respond to any questions asked by the patient.

You will not be required to take any additional history or perform any further clinical examination.
The Performance Guidelines for Condition 111 can be found on page 596
112
Candidate Information and Tasks
577
Condition 112
Colonoscopy findings in a 24-year-old man with chronic diarrhoea

CANDIDATE INFORMATION AND TASKS
You are working in a general practice. A 24-year-old man with chronic diarrhoea had a recent colonoscopy for the
investigation of his diarrhoea. For the previous six months he was passing bloody stools six to seven times per day
without any mucus or slime. He was told he had inflammatory bowel disease but could not remember any specific
names. The specialist who did the colonoscopy did however give him a copy of the photographs taken of the colon;
and told him that the appearance was similar for the whole of the large bowel.

YOUR TASKS ARE TO:
Interpret the colonoscopy illustrations.
Advise the patient which disease you believe is likely, and what treatment is likely to be required.

You are not required to take any further history. The endoscopist has sent you a detailed letter but it has not yet
arrived.
CONDITION112. FIGURE 1. CONDITION112. FIGURE 2.
The Performance Guidelines for Condition 112 can be found on page 599
Clinical Perspectives and Priorities
4-A Clinical Perspectives and Priorities
Performance Guidelines
MCAT 105-112
105 Abdominal pain and vaginal bleeding in a 39-year-old woman after 9 weeks amenorrhoea
106 Recent insomnia in a 25-year-old man
107 Dandruff or head lice in a 6-year-old girl?
108 Recent orchidectomy for a testicular neoplasm in a 28-year-old man
109 Postnatal fatigue and exhaustion in a 28-year-old woman
110 Fundus greater than dates in a 26-year-old woman at 30 weeks gestation
111 Tiredness and anaemia in a 55-year-old woman
112 Colonoscopy findings in a 24-year-old man with chronic diarrhoea
578
105
Performance Guidelines
579
Condition 105
Abdominal pain and vaginal bleeding after 9 weeks amenorrhoea, in a 39-year-old
woman

AIMS OF STATION
To assess the candidate's ability to define the possible causes of lower abdominal pain and vaginal bleeding, and to
manage a patient with these symptoms in the presence of 'cervical' shock.

EXAMINER INSTRUCTIONS
The examiner will have instructed the patient as follows:
You are attending the Emergency Department of the local hospital because of severe lower abdominal pain and heavy
vaginal bleeding. Your blood pressure is low. You are lying on a couch and the doctor is taking your history. The list of
responses below is likely to cover most of the questions you will be asked.
List of appropriate answers:
You have been trying to conceive since the last miscarriage and have not used contraception.
Your last period occurred nine weeks ago; you had hoped you were pregnant.
You normally have periods usually every four weeks, lasting five days.
Bleeding and pain started 12 hours ago.
You have already used more pads than you usually use in a whole period; and have passed a number of clots and
possibly pieces of tissue.
The pain is in the lower abdomen. The pain is coming and going every 5-10 minutes. There is no radiation of pain to
the back or elsewhere.
There is no dizziness.
You do not know your blood group.
Questions to ask unless already covered:
'Am I pregnant again?'
'Can anything be done to save my pregnancy?'
'If I'm losing the pregnancy why is this occurring?'
'This would be my second pregnancy lost why has this occurred?'
105
Performance Guidelines
580
Examination findings to be given to the candidate by the examiner on request
General appearance: She Is clammy, pale, obviously distressed and in pain.
Cardiovascular: Pulse 90/min
Blood pressure 80/50 mmHg
Abdominal examination: Lax, non-tender. No mass or viscus palpable.
Speculum examination: Cervix open, products of conception (POC) in cervical os.
(If the candidate does not ask if the cervix is open or
closed, don't give this information but comment on
uterine size alone).
Pelvic examination: The uterus is enlarged to the size of an eight week
pregnancy, anteverted and mobile. Cervical os is open and easily admits
one finger. Products of contraception are felt in the cervix. No adnexal
masses or tenderness.
Investigation results (Give these results if the specific test is requested):
Blood group O Rh negative, indirect Coombs test negative.
Hb 112 g/L.
If candidate requests coagulation studies, ultrasound examination or pregnancy test, these are unavailable and
unnecessary, and should be considered so by the examiner and the candidate marked down accordingly.

EXPECTATIONS OF CANDIDATE PERFORMANCE
The candidate should advise the patient along the following lines.
This is an incomplete abortion (miscarriage)
The products of conception (POC) need to be removed from the cervix immediately to treat the shock and reduce
the bleeding. This should be done in the Emergency Department An intravenous line should be inserted,
simultaneously with blood sampling for blood grouping and holding of serum for cross-typing if shock persists after
removal of POC.
Give ergometrine or a similar oxytocic intravenously to reduce blood loss.
Give anti-D as patient is Rh negative. Candidates not asking to determine the blood group, should be marked down
accordingly.
She needs curettage as soon as possible to ensure uterus is empty and to control the bleeding.
Send curettings for histologic examination although the cause for the miscarriage probably will not be established
with certainty by this examination.
If BP still remains less than 110 systolic when the POC have been removed from the cervix, further intravenous fluid
replacement (with possible blood transfusion) will be required
In discussing causes of an incomplete miscarriage in a 39-year-old woman when the emergency has been dealt
with, and prior to discharge from hospital, the candidate should indicate that the loss was probably because of a
chromosomal abnormality of the fetus, and that such miscarriages unfortunately become more common as women
get older The likelihood of a miscarriage the next time she gets pregnant is probably about 25-30%.
105
Performance Guidelines
581
In the next pregnancy an ultrasound should be done at about seven weeks of gestation to provide reassurance that all
is normal, maternal serum screening is advised, nuchal fold assessment at 11-12 weeks of gestation is advised, and a
decision ultimately made, if all is apparently going normally, whether a chorion villus biopsy or amniocentesis should be
performed in view of her advanced maternal age and increased risk of a fetal chromosome abnormality.

KEY ISSUES
Ability to define the likely cause of the symptoms and the low blood pressure found
Ability to recognise that no investigations are required in this patient other than urgent determination of blood group,
as the diagnosis can clearly be made on clinical grounds.
Ability to define the subsequent management plan.

CRITICAL ERRORS
Inappropriate investigation requested such as pregnancy test after results of physical examination are known,
ultrasound examination, or coagulation screen.
Failure to recognise the need to remove POC from cervix.
Failure to check blood group to see if anti-D antibody was indicated.

COMMENTARY
This case confirms the importance of the performance of a speculum and vaginal examination in a woman who
presents with a history of bleeding and lower abdominal pain after a period of amenorrhoea. This case in particular
demonstrates the need to recognise POC trapped in the cervical os and causing 'cervical' shock. The importance of
removing the products of conception from the cervical os and administering ergometrine to contract the uterus and
prevent or reduce further bleeding cannot be stressed too highly.
Common problems likely with candidate performance are:
When taking the history not being focused enough to the actual problem but asking for information such as
irrelevant past history, social history and so on. This just takes time to do, and reduces the time available for the
remaining tasks. Adequate diagnosis and resuscitation have first priority in this case
Failing to determine that there are POC in the cervical os, and that a diagnosis of incomplete miscarriage is
therefore able to be made without the need for further investigation. If the cervix is still closed, ultrasound is required
to define whether the pregnancy is in the uterus, and whether it is viable.
Failing to advise the probable cause of the miscarriage and the likely outcome of future pregnancies.
106
Performance Guidelines
582
Condition 106
Recent insomnia in a 25-year-old man

AIMS OF STATION
To assess the candidate's ability to evaluate a recent onset of a sleep problem by taking a focused history and then
to counsel the patient. This sleep disturbance is part of a normal grief reaction and possibly post-traumatic stress
related to a failed resuscitation attempt at the time of his father's death.

EXAMINER INSTRUCTIONS
The examiner will have instructed the patient as follows:
You are a second year architecture student at the local university. You passed all your subjects last year without any
difficulty and did not anticipate having problems this year. You enjoy your studies and do not find them difficult. You
have your next set of university exams in three weeks time and have some concerns about these as you did very
little study after your father died, and haven't managed to catch up with your assignments and coursework yet.
You haven't advised the Faculty Office, or any of your lecturers about the death of your father one month ago, nor
explained the time you were absent from your course (2 weeks), or that you have had difficulty studying (one month).
Your father died suddenly at home a month ago at the age of 49 years. He had never had an illness in his life, and
was sitting watching the TV when he suddenly collapsed. You were studying at the time in your bedroom, but your
mother called you to help with the resuscitation. Both you and she did this, getting advice over the telephone having
dialled '000', until the Intensive Care Ambulance arrived.
At no stage were you able to get a pulse going. A Coronial postmortem examination was done and showed the major
artery to the main ventricle of the heart was blocked, and this was the probable cause of death. You have never had
difficulty sleeping in the past, except for the night before an examination where you felt your preparation might have
been inadequate and have never taken any drug treatment for this.
You have never been seriously unwell in your life, have never used drugs of any sort, but have not played any sport
since starting your university studies. You are still living at home because you cannot afford to move into a flat, and
haven't had time to take on a regular job except during the long university vacations. You are also an only child and
your parents encouraged you to stay at home to save money.
Since your father's death, your mother has coped reasonably well despite being initially depressed, and expects to
receive a pension from your father's previous employer (Australia Post). She thinks this will be enough for you to be
able to continue with your studies, as the house is fully paid off and her pension will be about $500 per week. On
specific enquiry, deny feelings of hopelessness, despair or guilt. You have felt sad but have been able to be cheered
up by your friends for example when they insisted on taking you out to a movie recently.
You have no suicidal thoughts. After the death you did not eat very well but your appetite has since improved. You
have not suffered panic attacks. You are anxious about the impending examinations, as you do not think you are well
enough prepared for them You have no history of mental illness or psychological problems.
106
Performance Guidelines
583
Questions to ask unless already covered:
'Would it be worth taking some sleeping pills?'
Is my heart going to be OK?'
EXPECTATIONS OF CANDIDATE PERFORMANCE
The history must include relevant information about the father's death, the involvement of the son at the time, how the
rest of the family are coping with the death itself and with any problems which have resulted. How much time away from
university studies resulted and whether any special consideration has been applied for must also be determined.
Concern that the patient has about the possibility of genetic predisposition to early age heart disease should be
discussed, including possible referral to a cardiologist for a full checkup (although it is unlikely anything would be found
to be wrong as the flying examination was normal six months ago).
Blood lipid studies would normally be performed. The candidate is expected to exclude depressive illness, anxiety
disorder or other psychiatric conditions. Absence of pre-existing problems and the onset of sleeping problems at the
time of bereavement, as well as symptom profile should exclude these.
Short-term treatment of insomnia due to the acute stress of bereavement, utilising hypnotics such as temazepam
(10-20 mg) at night, may be indicated.
Explanation that their use to break a vicious cycle is unlikely to result in dependence when their use does not exceed
two weeks, is important. Antidepressant drugs with sedative properties are not generally indicated unless a depressive
illness is the principal problem Empathie advice should also be given about the normal nature of this response to loss
Candidates may suggest coping strategies such as using behavioural techniques; cognitive strategies to cope with
intrusive thoughts; general measures such as regular exercise, avoidance of heavy meals, alcohol and
caffeine-containing drinks between the evening meal and bedtime should be stressed.
The candidate should advise that review of heart disease risk factors should be delayed until later, after he is over the
current problem. The candidate should offer to check progress, offer to counsel other members of the family as well
and should also recommend to the patient that it would be wise to notify his Year Coordinator in the Architecture
Faculty of the death and his disturbance in studying. Such advice needs to be given prior to the actual examinations if
'special consideration' is ultimately necessary.

KEY ISSUES
Displaying empathy with patient.
Appropriate history to determine diagnosis of the cause of insomnia.
Displaying appropriate counselling skills when advising the patient about the management of this problem, including
other common problems which can occur.

CRITICAL ERRORS
Prescribing antidepressants at this stage.
Omitting supportive interaction.
106
Performance Guidelines
584

COMMENTARY
Grief is the emotional cost after the loss of someone or something precious and meaningful. Grief is painful and
causes psychological anguish, waves of unpleasant dysphoria and distress, physiological arousal and a mixture of
physical symptoms. It is a normal function of the human condition, and is not an illness.
Insomnia in the early days and weeks following a loss is part of the adjustment process and should be managed
expectantly in most instances. Individuals can cope and adapt if they realise that grief is part of the natural response
to an emotional wound, and like a bruise, it will resolve, given time. In some people who cannot afford daytime
fatigue or somnolence even for a few days after a restless night, then short-term treatment with a hypnotic, either a
short-acting benzodiazepine such as temazepam, oxazepam or lorazepam. or a nonbenzodiazepine such as
Zolpidem or zopiclone may be prescribed judiciously for up to two weeks. Rebound insomnia may still occur when
the course of the hypnotic is tapered off. Grief-associated insomnia treated with hypnotics may be the start of chronic
or lifelong psychological dependence on a benzodiazepine and requests for repeat prescriptions should usually be
declined, with the focus switched to sleep hygiene strategies instead.
Grief reactions such as anorexia and weight loss are common as is poor concentration, fatigue, absent-mindedness,
auditory and visual or olfactory hallucinations involving the deceased, crying, rocking, wearing the clothing of the
deceased, searching behaviours, feeling that life is not worth living, catastrophic cognitions about life in the future
without the deceased ('How will I cope without. . . ?' ) are also normal and not indicative of depression or indications
for psychotropic treatment.
It is normal to feel diminished and bereft when someone revered dies, but not to feel personally worthless, useless
and hopeless with lowered self-esteem and irrational guilt or shame. Suicidal ideation is not part of normal grief, but
deliberate self-harm afterwards may be cultural, or an expression of extreme dysphoria and not indicative of an
intention to die. Marked functional impairment, prolonged anorexia and weight loss, persistent insomnia, significant
psychomotor retardation, hypomania or manic behaviour or frank psychosis are all indications for antidepressant
treatment, mood stabilisers including lithium carbonate, or atypical antipsychotics. Specialist referral may be
necessary once grief overlaps with psychiatric illness.
It would not be unusual for this young man or his mother to experience some elements of post-traumatic stress
following their unsuccessful attempt at resuscitation. Anxiety, guilt, helplessness, horror and despair may be
understandable sequelae to the incident, but fortunately he is not experiencing any of those feelings at this stage.
Perhaps after his examinations are over or on subsequent anniversaries of the death, unpleasant and intrusive
memories with nightmares and insomnia may appear, but that is for another scenario.
585
107
Performance Guidelines



Condition 107
Dandruff or head lice in a 6-year-old girl?

AIMS OF STATION
To assess the candidate's recognition and practical treatment of head lice succinct, accurate, decisive
information-giving is required. This should be done in a reassuring and supportive manner to reduce anxiety about the
source, transmission and stigma associated with the infestation.

EXAMINER INSTRUCTIONS
The examiner will have instructed the parent as follows:
You are the parent of 6-year-old Jodie and are concerned whether she has head lice because you have heard it is in
the school, but you have never seen the condition before. If it is lice, you are concerned by the stigma of poor hygiene
being deemed causative, as you regard Jodie's hygiene as being very good. You wonder if it will spread to other family
members and whether the children can go to school, and want advice on treatment.
Questions to ask unless already covered:
'What is wrong with her, Doctor? Why is she scratching all the time?'
'How would she get this?'
'We are so careful with our hygiene!'
'What treatment can she have? Will it get rid of it?'
Her little brother scratches his head all the time. Could he have it as well?'
How do we treat it? Does she have to have her head shaved?'

EXPECTATIONS OF CANDIDATE PERFORMANCE
The candidate should explain:
the diagnosis is head lice;
the eggs (nits) can be seen; they can be mistaken for dandruff;
the infestation is usually contracted at school;
it spreads from person to person by direct contact;
it can spread quickly within the family;
it is not a sign of poor hygiene, and is common in our society; and
all family members need to be checked, and treated only if nits or lice are present.

The candidate should advise and prescribe:
laundering of pillowcases using hot water or hot clothes dryer; and either
~ Application of pyrethrin-based scalp preparation to thoroughly cover hair, thorough rinsing, combing with a
fine-toothed comb and drying with a clean towel.
~ Repeat in seven days.
OR alternatively:
~ Conditioner and comb every two days until no lice are seen for 10 days.
107
Performance Guidelines
Hair does not need to be shaved or cut short.
School exclusion after treatments is not required: the child can return the day after treatment.
Followup to check for resistance to the medication, then try malathion-based preparation.
KEY ISSUES
Ability to make the correct diagnosis.
Succinct, accurate, decisive information-giving.
Ability to describe the correct treatment as outlined above.
Ability to reduce parental concern.
CRITICAL ERROR
Failure to diagnose head lice.
COMMENTARY
This case tests the candidate's recognition of a common problem, and the ability to provide succinct and empathie
guidance on treatment. The dominant cue here is the presumptive end-diagnosis already provided by the parent,
namely head lice. Because this is a common condition, the candidate will be expected to recognise the pattern of
small white spots, or 'nits', which are the eggs of the infesting lice. The diagnosis should be made almost instantly, as
there are no features of any associated dermatitis which could be suggested as an alternative.
Candidates are expected to be able to explain this common condition, prescribe an appropriate regimen of treatment
for head lice and achieve rapport with the parent in order to ensure compliance, reduce anxiety and achieve a
satisfactory outcome.
CONDITION 107. FIGURE 2.
'Nits' on hair
586
587
108
Performance Guidelines



Condition 108
Recent orchidectomy for a testicular neoplasm in a 28-year-old man

AIMS OF STATION
To assess the candidate's ability to recognise the pathology specimen as an
orchidectomy for neoplasm.
To assess the candidate's knowledge of the investigations required and the adjuvant
therapy used for management of testicular tumours, as well as prognosis following
treatment.

EXAMINER INSTRUCTIONS
The examiner will have instructed the patient as follows:
Opening statement
'I've forgotten the name the surgeon gave to my ondition?'
Further questions to ask
'Why was the incision made in the groin and not in the scrotum?'
'My uncle, who is much older, recently noticed a lump in his scrotum, but his
operation was done through an incision in the scrotum. Would the diagnosis of my
uncle's condition have been the same or different?'
'What other tests will I need?'
'Is any other treatment now necessary for me?'
'Do I need any further followup? If so, what would this involve?'
'What is my outlook for the future?'
EXPECTATIONS OF CANDIDATE PERFORMANCE
The candidate should recognise that the specimen is of a malignant testicular
neoplasm showing uniform macroscopic appearance consistent with a seminoma.
Appreciation that optimal treatment is excisional surgery as has been done, with
additional chemoradiotherapy after referral
Investigations required for tumour markers and imaging for possible metastases.
Appreciation of excellent prognosis when the tumour is confined to testis.
Knowledge of specific details of chemoradiotherapy is not essential.

KEY ISSUES
The candidate should understand the spread of testicular tumours to the para-aortic
retroperitoneal lymph nodes and the favourable prognosis with combined treatment
of surgery radiotherapy and chemotherapy.

CRITICAL ERRORS
To suggest that testicular tumours should be removed through a scrotal approach as
used for benign lesions.
To suggest that no further tests or followup are required.
108
PerformanceGuidelines


COMMENTARY
Thetesticulartumourasphotographedhasthetypicalfeaturesofaseminoma,whichisa
malignanttumour.Othermalignanttesticulartumoursincludeteratomasandmixedtumours.
Spreadofthesetumoursmayoccuralonglymphatics,whichdraintotheparaaorticlymphnodes
initially(notinguinal).Retroperitoneallymphnodeenlargementmaybeevidentonacomputed
tomogram(CT)oftheabdomen.Tumourmarkers(cx-fetoproteinand/J-hCGestimations)are
helpfulinbothinitialdiagnosisandsubsequentfollowupafterprimarytreatment,tohelp
diagnosetumourrecurrence.
Surgeryisperformedthroughaninguinalincision,excisingthetestisandthecordtothelevelof
thedeepinguinalring.Ascrotalincision(orscrotalneedlingofthetumour)iscontraindicated
becauseofthepotentialriskoftumourimplantationinthescrotalwallthusinvolvinganother
lymphaticfield.Scrotalincisionsaresatisfactoryforbenignscrotalconditionssuchas
hydrocelesoranepididymalcyst.Boththeclinicalsignsandanultrasoundexaminationofthe
scrotumhelpclarifywhetherthelesionisahydroceleoranepididymalcystoramalignant
testiculartumour.
Adjuvantradiotherapyand/orchemotherapyaftersurgeryhasgivenabettersurvivalratefor
patientswithseminomaandteratoma,theformerhavingabetterprognosis(5yearsurvivalrates
greaterthan85%).
Followupwithclinicalexamination,tumourmarkertestsandCTisrequired.Thispatientneeds
referraltoaradiationoncologist.FurtherimagingofhisabdomenandchestbyCTisrequiredto
stagethetumourfully.Heislikelytorequiretreatmentbycombinedchemotherapyand
abdominalradiation.

588
109
Performance Guidelines

Condition 109
Postnatal fatigue and exhaustion in a 28-year-old woman
AIMS OF STATION
To assess the candidate's ability to diagnose postnatal depression with psychotic features and
suicidal ideation, requiring urgent intervention.
EXAMINER INSTRUCTIONS
The examiner will have instructed the patient as follows:
You are a 28-year-old woman who has been urged by your husband, Peter, to seek advice and
treatment for poor sleep, increasing daytime fatigue and exhaustion over the past two weeks.
You should give the following history without prompting initially in a hesitant manner, but with
increasing conviction as you proceed. Don't try to give all the history at once, but give it in reply to
questions from the candidate. You should start by giving the information in the paragraph below.
About ten weeks ago, you gave birth to your second child, Thomas, in the local maternity unit.
The pregnancy, labour and delivery were uneventful. It was a planned pregnancy and everyone
else is delighted that Thomas seems normal and healthy. You have breastfed him since birth.
You have been happily married to Peter, who is 30, for five years and continue to live with his
parents while you save for a home of your own. Although Peter is loving and supportive, he is
committed to building up his travel agency business and is also doing a part time business
degree, which keeps him busy six days a week.
As soon as you arrived home from the hospital with Thomas, you began to have niggling doubts
that there was something wrong with him. You had so much wanted to have a perfect child, the
son that your husband and family would be so proud of. Despite the delight and reassurances of
Peter, your extended families and the community baby health clinic nurses, you have become
preoccupied with Thomas's health and wellbeing and your adequacy as a mother.
Admit to 'possibl y' being depressed if asked. Volunteer the following 'I'm really not coping
well I don't know what Is wrong with me I feel I've lost my confidence I can t make
decisions I'm having difficulty sleeping. It's been going on ever since I got home but much
worse this past two weeks. I am really anxious the baby may stop breathing, like in SIDS. So I'm
trying to wake every hour or so to check.'
You have been constantly afraid that Thomas may stop breathing and suffer a sudden infant
death, so you have forced yourself to sleep for only brief periods. You have become increasingly
worried that Thomas is not feeding properly and is failing to thrive because of your insufficient
supply of breast milk. So you have been weighing him at every opportunity, at the clinic, at home
and weekly at the local hospital Emergency Department. Even though his weight appears to be
increasing, you have become convinced that the various weighing scales are wrong and that, as
his mother, you KNOW that he is not gaining weight.
589
109
PerformanceGuidelines



Asfarasyouareconcerned,itreflectspoorlyonyourcompetenceandadequacyasamother.
You cannot convince people that there is something wrong with your son, that his breathing
fluctuatesandhemaydieatanymomentandthatyouwillbeblamedforthisandthatthequality
ofyourbreastmilkissopoorthanyoursonisnotgrowingproperly.
Youhavecometobelievethatyouareabadmotherandafailureandthatasaresultyourson
anddaughterwillbetakenawayfromyouandthatyourstorywillbehighlightedinthemedia.
You have begun to think that life is not worth living and that the shame and gossip from
neighboursandfriendswhichyouareSUREisgoingon,wouldbestoppedifyoujustgoaway
somewhereandkillyourself.Thenyourfamilywillbebetteroff,becausethatwillprovetothem
howmuchyoulovedyourson,ashishealthproblemsareyourentirefault.
Increasingly over the last two weeks, you have not been sleeping well. Your appetite has
diminishedandyouhavebeenlosingweightsinceThomaswasborn.Youhavelost6kginthe
pasttendays.Youaretiredallofthetime,youfeelterribleinsideandyoufeelanxious,agitated,
unhappy,guiltyanddesperate.
Your anxieties about your son's health and wellbeing have not been allayed by reassurances
from your husband, your extended family, the clinic nurses or the hospital Emergency
Department. Now you are worried that your husband's insistence on you coming to see the
doctormeansthatheisplanningtodivorceyou,becauseyouhavefailedasamotherandawife
andbroughtshametohisfamilyname.
PSYCHOSOCIALHISTORY
Youarethemiddleofthreechildrenwithanolderbrotherandyoungersister.
Parentsarealiveandwell,butlive100kmaway.
Averagestudent,goodatArt.CompletedYear12.
Workedinadvertisingandstudiedgraphicdesignatcollege.
Methusbandthroughwork.Yourfirstandonlyseriousrelationship.
Married5yearsago.Happyinitially.
Describeyourselfasaquiet,placid,sociallyanxious.Veryhomeandfamilyorientated
obsessionalandconscientious.
Nonsmoker.Nondrinker.
Bothpregnanciesplannedandrelativelyuneventful.
Nopostpartumproblemswithfirstpregnancy.
Nopastorfamilyhistoryofpsychiatricdisorderorillness.
APPEARANCE/MENTALSTATE
Nomakeup,neat,casuallydressed,butrumpled,uncombedhair.
Anxiousandanguishedappearanceandmanner.
Occasionallywringinghands.
Softlyspoken,limitedaffectiverange.
Nottearfulbutsad,gloomyattimes.
Notthought-disordered.Noperceptualabnormalities.
Thoughtformnormal,butconvincedyouareafailureandthatyoursonissufferingbecause
ofyourinadequacies.
Admittoruminatingaboutself-harmifasked,butinsistyouwouldNEVERharmyourson,
daughter,husbandoranyoneelse(youmightcutyourownwristsorstabyourself).
590
109
Performance Guidelines

EXPECTATIONS OF CANDIDATE PERFORMANCE



Approach to the patient
Establishment of rapport.
Empathic, sensitive questioning.
History-taking
Exploration of presenting symptoms.
The evolution of her mood disorder over time.
Premorbid and psychosocial factors.
The clarification of her beliefs.
The sensitive uncovering of her suicidal ideation.
Ask also to interview her husband and check health of children with him.
Diagnosis
Postnatal depression with melancholic features in a woman with premorbid anxious and
obsessional traits.
Treatment
Urgent hospitalisation, if necessary invoking the powers of the relevant Mental Health
legislation. This is a psychiatric emergency with a deepening depression. The risks of
attempted or successful suicide and infanticide are real.
Once in hospital, anxiolytics, antipsychotics and antidepressants should be commenced.
Electroconvulsive therapy may be the treatment of choice initially with lithium carbonate
maintenance. Whilst a mother and baby unit would the optimal hospital placement, very few
areas of Australia offer this service. Therefore the mother should be admitted alone to a
secure unit, where her safety and prompt treatment are the primary concerns.
Breastfeeding should be terminated forthwith and agents administered (dopamine agonist) to
reduce breast enlargement. Supervised visits with her infant son and other family members
should be a daily occurrence. Prognosis is very good for complete recovery. Relapse likely in
the future both postpartum or not, but lithium carbonate prophylaxis will improve prognosis
and reduce likelihood of relapse.

KEY ISSUES
Diagnosis of postnatal depression.
Potential suicidal ideation requiring urgent intervention.

CRITICAL ERRORS
Failure to diagnose severe postnatal depression.
Failure to appreciate urgency of situation (including possible suicide) requiring
hospitalisation.

591
592
109
Performance Guidelines

COMMENTARY
The diagnosis is postnatal depression with melancholia features, in a woman with premorbid
anxious and obsessional traits.
Her beliefs about her infant and her competence as a mother could be interpreted as
overvalued ideas (and therefore not psychotic): or delusions (fixed, irrational false beliefs not
amenable to evidence or persuasion to the contrary) which would suggest severe depression
with psychotic features.
Her depression began with hypochondriacal anxiety about her son's well being and this
progressed to affect her view of herself as a mother and as a wife. She has comorbid anxiety
symptoms, but her primary diagnosis is depression.
Her suicidal ideation and her cognitions of guilt, shame, failure and inadequacy are of serious
concern and require urgent intervention.
This is not dysthymia, obsessive-compulsive disorder, post-traumatic stress disorder or
schizophrenia.

110
Performance Guidelines

Condition 1 1 0
Fundus greater than dates in a 26-year-old woman at 30 weeks gestation

AIMS OF STATION
To assess the candidate's knowledge of the causes of a pregnant uterus being much larger than
expected, and to convey the implications of this to the patient.

EXAMINER INSTRUCTIONS
The examiner will have instructed the patient as follows:
List of appropriate answers to the questions likely to be asked:
Weight at the start of pregnancy was 56 kg. You now weigh 76 kg (height 160 cm).
No family history of diabetes, no glycosuria during the pregnancy, you had a glucose
challenge test (GCT) at 26 weeks, you believe it was normal.
You had ultrasound examination at 18 weeks which confirmed the gestation and showed a
single pregnancy, which was apparently normal.
No oedema, no headaches or visual disturbance, no proteinuria in pregnancy thus far.
Blood group O Rhesus positive, no blood group antibodies found (indirect Coombs test
negative).
No one has ever commented about the possibility of uterine fibroids.
Questions to ask unless already covered:
'Why am I so big?'
'Will my baby be OK?'
'Will I be able to have a normal delivery?'

Examination findings to be given to candidate by the examiner on request
Examination showed the symphysis-fundal height was 40 cm.
The uterus was tense, but not tender.
A fluid thrill was evident.
The presentation was cephalic, with the head being freely mobile above the pelvic brim.
Fetal heart was heard and was normal.
General examination showed no evidence of oedema, and the blood pressure was 120/80
mmHg.

593
110
PerformanceGuidelines




Investigationresultstobegiventocandidatebytheexamineronspecificrequest
BloodgroupORhesuspositive,indirectCoombstestnegative.
Urinetesttodaynegativeforproteinandglucose.
GCTat26weeksofgestationhadshownthebloodglucoselevel,1houraftera75g
glucoseload,wasnormal.
Haemoglobinat26weekswas114g/L.
Ultrasoundat18weeksshowedasinglebabyofasizeconsistentwithdates.

EXPECTATIONSOFCANDIDATEPERFORMANCE
Advicetopatient(thecandidateshouldconveythesubstanceofwhatfollowstothepatient):
Thediagnosisisthatofauteruswhichislargerthanitshouldbe,probablyduetoanexcessive
amountofamnioticfluid(polyhydramnios).
~Thisdiagnosisneedstobeconfirmedwithanultrasoundwhichwouldalsolookforthe
possiblecause.
Othermattersthecandidatemaycoverareasfollows:
~Othercausesofafundusgreaterthandatesare:
- Multiplepregnancythiswouldhavehadtobemissedatthetimeoftheprevious
ultrasoundexaminationat18weeksofgestation.Thiswouldappearunlikely.
- Amacrosomic(largesize)babyunlikelyasnoevidenceofgestationaldiabetes.
- Uterinefibroids,wrongdatesbothoftheseshouldhavebeenabletobediagnosed
onthe18weekultrasoundexamination.
~Otherconcerningcausesofpolyhydramnioswhichneedtobelookedforinclude.
- Fetalmalformationsofthecentralnervoussystem,gastrointestinalsystem,
abdominalwall,orelsewhere.
- Diabetesthereforedoaglucosetolerancetest,eventhoughGCTwasnormal.
- Chorioangiomaoftheplacentathisshouldbelookedforonultrasound.
- Fetalinfectionwithcytomegalovirusortoxoplasmosis.Bothofthesecanresultin
polyhydramnios.
Polyhydramnioscancauseproblemsinlatepregnancyandlabourincludingmalpresentation,
premature rupture of the membranes, premature labour, and placental abruption following
membrane rupture. As she is now at 30 weeks of gestation, she should be advised that she
wouldprobablybenefitfromextrabedrest,butthatprematuredeliveryislikely.Sheshouldbe
informedtoattendearlyifshethinksshemightbeinprematurelabour.Speculumexamination,
or pelvic examination to assess the cervix, is sometimes appropriate. The candidate should
indicatethatreferralbacktothespecialistisnecessarynow(orwithinthenextfewdays).There
wouldbeaplaceforprophylacticsteroidtherapytoreducethelikelihoodofrespiratorydistress
inthebabyifitisdeliveredpriorto34weeksofgestationbutthespecialistshouldmakethis
decision.
594
110
Performance Guidelines

KEY ISSUES
Knowledge of the causes of excessive uterine enlargement in pregnancy.
Diagnosis of polyhydramnios.
Determining the cause of polyhydramnios in this patient.
CRITICAL ERRORS
Failure to consider and confirm that the problem is most likely due to polyhydramnios.
Failure to consider the possible causes of polyhydramnios and failure to arrange the
appropriate investigations, or failure to refer to a specialist for these within the next few
days.

COMMENTARY
This case tests the candidate's ability to understand and be able to diagnose the causes of a
fundus being larger than dates at any specific gestation. It also is important to understand the
risks associated with conditions such as polyhydramnios and to be able to counsel the patient
accordingly. The complications of polyhydramnios (such as premature labour and premature
rupture of the membranes) require the cooperation of the patient to enable the correct
management and therefore the information given to the patient about such complications is
critical.
Common problems likely with candidate performance are:
Failing to enquire about the possibility of diabetes in this pregnancy.
Failing to enquire about the results of the 18-20 week ultrasound examination.

595
111
PerformanceGuidelines

Condition 111
Tiredness and anaemia in a 55-year-old woman

AIMS OF STATION
To assess the candidate's knowledge of iron deficiency anaemia and how to deal with a patient
of this age with this type of anaemia.

EXAMINER INSTRUCTIONS
The examiner will have instructed the patient as follows:
You are a 55-year-old housewife. You are seeing the doctor today to find out the results of a
blood test which was done to investigate the cause of your recent tiredness and mild
breathlessness on exertion.
Opening statement:
' Wel l doctor, what di d my bl ood tests show?'
Questions to ask if not already covered
ask these questions if the topic has been raised but not clarified adequatel y:
' What could cause thi s type of anaemi a?'
' But I' ve had no bl eeding from the bowel .'
' Do I lack iron i n my di et?'
' Do you thi nk I mi ght have an ulcer?'
Do you think I mi ght have cancer?'
' Coul d I just try the i ron tabl ets first?'
You understand that anaemia means lack of blood. You find it hard to accept that there could be
bleeding into the upper or lower digestive system without you noticing it. Become anxious if the
doctor advises endoscopy. Behave as if you are somewhat averse to such tests and that you
would prefer to try iron tablets first. If the candidate suggests only one test (either upper or lower
gastrointestinal endoscopy), then ask if anything else should be done after that.
EXPECTATIONS OF CANDIDATE PERFORMANCE
Approach to the patient achieve compliance about invasive investigations whilst
minimising patient anxiety about the possibility of a serious cause of the anaemia.
Type of investigations required to define type of anaemia serum ferritin serum iron,
total iron-binding capacity (TIBC) and percentage saturation. The candidate should be given
a report with the results of these tests, as follows:
Serum Iron 4 /jmol/L (10-30)
TIBC 370 pmol/L (45-80)
Fe (transferrin) saturation 8% (15-50)
Serum ferritin 2 /jg/L (10-200)
If serum folate and B
12
levels are requested, the examiner should respond that these are
normal.
596
111
Performance Guidelines

The candidate should recognise that these results clearly confirm an iron deficiency anaemia.
Investigations to determine the cause of an iron deficiency anaemia in this patient
there are several options, but only one is correct: the candidate is expected to advise both
upper and lower gastrointestinal tract endoscopy (i.e. gastroscopy and colonoscopy), even if
the one selected first reveals benign pathology such as a peptic ulcer or colonic polyp.
Colonoscopy is usually done first. If the right side of the colon cannot be visualised, barium
air-contrast enema should also be done.
Initial management plan the essential process is to exclude carcinoma of the right side of
the colon or peptic ulcer which are the most important causes of iron deficiency anaemia in
this patient's age group. Other tests such as faecal occult blood or abdominal ultrasound are
not required. A therapeutic trial of oral or parenteral iron instead of full investigation is wrong.
Deferring investigation until the effect of iron therapy is apparent, is also wrong. Blood
transfusion is not immediately indicated. Transfusion is unlikely to be needed before surgery if
this is found to be necessary. Commencement of oral iron therapy e.g. Ferro-gradumet
(ferrous sulphate dried 350 mg sustained release) one daily or similar is acceptable (may need
to be stopped prior to colonoscopy). Warn about side effects (constipation, colour of stools).
Commencement of an H2 receptor antagonist, whilst awaiting endoscopy is also acceptable.
Patient education and counselling explain the significance of the iron deficiency
anaemia and possible causes:
~peptic ulcer, reflux oesophagitis;
~ benign tumour of the stomach or large bowel;
~malignant tumour of the stomach or large bowel;
~dietary cause is unlikely (the patient is not a vegetarian);
~adverse drug reaction e.g. nonsteroidal anti-inflammatory drug (NSAID), aspirin: and
~candidate should know that thalassaemia minor is extremely unlikely with the blood film and
iron studies obtained.

KEY ISSUES
Determine type of anaemia and its cause
Interpretation of investigations to confirm presence of iron deficiency anaemia
Identification of possible causes of iron deficiency, particularly an occult colonic cancer.
Patient education and counselling, explaining problem, answering patient questions and
advising further investigation.

597
111
PerformanceGuidelines

CRITICAL ERRORS
Failure to diagnose iron deficiency anaemia.
Failure to consider gastrointestinal blood loss.
Failure to advise both upper and lower gastrointestinal tract endoscopy.
COMMENTARY
Patients with a mild anaemia may or may not be symptomatic depending on the rapidity of
development. In this patient iron deficiency anaemia is strongly suspected by the low mean
corpuscular volume (MCV) and the microcytic hypochromic blood film. The low serum ferritin
and low transferrin saturation confirm the diagnosis of iron deficiency.
This station requires the candidate to interpret the clinical picture of symptomatic anaemia in a
middle-aged postmenopausal woman and the full blood examination and blood film presented.
Circulating iron is bound to transferrin, the iron transport protein. Iron in excess of the amount
needed for haemoglobin synthesis binds to a storage protein apoferritin forming ferritin. Serum
ferritin levels below 10g/L are virtually diagnostic of absent body iron stores.
It is expected that the candidate will know that the blood film changes suggests iron deficiency
and proceed to ask for iron studies. Once iron deficiency is confirmed, the key clinical diagnosis
is that the patient almost certainly has occult gastrointestinal blood loss and therefore requires
both upper and lower gastrointestinal endoscopies. It is not acceptable clinical practice in a
postmenopausal woman with iron deficiency anaemia to treat the iron deficiency with iron
therapy and do nothing further.

598
112
Performance Guidelines
Condition 1 1 2
Colonoscopy findings in a 24-year-old man with chronic diarrhoea
AIMS OF STATION
To assess the candidate's knowledge of endoscopic findings in inflammatory bowel disease, the
differences between ulcerative colitis and Crohn disease, and the management of a patient with
inflammatory bowel disease.
EXAMINER INSTRUCTIONS
The examiner will instruct the patient that he has come to see the doctor for explanation of his
condition and advice on his treatment. If any part of the doctor's explanation seems unclear, ask
for clarification.
EXPECTATIONS OF CANDIDATE PERFORMANCE
The candidate should diagnose inflammatory bowel disease.
Inflammatory bowel disease includes ulcerative colitis and Crohn disease (granulomatous colitis).
Both can present with diarrhoea. Ulcerative colitis involves the large bowel -mainly ulceration of
rectal and colonic mucosa (bloody diarrhoea); whereas Crohn disease is a transmural
inflammation of the whole wall of the small and/or large bowel (usually causing mucous diarrhoea)
and may lead to intestinal obstruction as well. 'Skip' lesions and fistulae occur in Crohn disease.
Longterm inflammation is associated with the development of carcinoma of the colon in a small
proportion of patients with ulcerative colitis, usually those with total colonic involvement. Malignant
change is less likely in Crohn disease.
Management includes the treatment of nutritional deficiencies and anaemia in severe or chronic
diarrhoea. Drug therapy includes the use of sulfasalazine, prednisolone (local enemas or
systemic) and immunosuppressive therapy (e.g. azathioprine or mercaptopurine). Surgery is
indicated for severe, chronic ulcerative colitis or Crohn colitis not responding to medical therapy.
Surgical treatment for ulcerative colitis necessitates a proctocolectomy with an ileo-anal pouch
procedure or ileostomy. Urgent surgery is very occasionally needed if acute toxic dilatation occurs.

KEY ISSUES
The candidate should understand the differences between the pathologies of ulcerative colitis
and Crohn disease, and should demonstrate an understanding of the medical therapies used and
the indication for surgery.

CRITICAL ERROR - none defined

599
112
PerformanceGuidelines

COMMENTARY
This is becoming an increasingly common scenario. Patients are often provided with
photographicevidenceofaproceduretheyhaverecentlyundergoneandthisbecomespartof
their travelling record, along with packets of X-rays, letters from medical practitioners and
laboratoryresults.Whilstallthiscanbeofconsiderablehelptothedoctorwhoisreviewingthe
patient,thereisanexpectationbythepatientthatthedoctorwillbeabletointerpretthedata.
This may not always be the case, particularly if the patient has returned to see the general
practitioner before the specialist has had the opportunity to communicate with the referring
doctor.
Inthiscase,thepatienthasphotographsofapartofthecolonandtheareaofmucosaonview
shows erythema, oedema and ulceration. The patient has given a history of six months'
duration, so infection as a cause of this colitis is unlikely but stool cultures and microscopy
shouldnotbeomitted.Inflammatoryboweldiseaseisadiagnosiswhichcanonlybemadewith
certaintyoncethebiopsyresultsareavailable.
Whilstawaitingtheletterfromthespecialistandthehistopathology,itwouldbereasonableto
explain to the patient what is meant by the term 'inflammatory bowel disease' and provide a
brief overview of Crohn disease and ulcerative colitis. At this stage, the general practitioner
needs to reassure the patient that in most instances both diseases can be managed quite
satisfactorily with medical therapy. The exact medications will depend on which disease is
present and the specialist will direct therapy. In most cases the disease will run an indolent
courseandbeeasilykeptundercontrol.Thepatientwillbekeptunderregularreviewbyboth
thegeneralpractitionerandthespecialist.Bothdiseasesarelifelongafflictions.
If the underlying problem is Crohn disease, the small as well as the large bowel could be
involved. The patient will be treated on a symptomatic basis, but must be aware that
complications can occur. These include flare-up of the colitis (diarrhoea, haemorrhage,
abdominalpain,andsystemicillness),acutecolonicdilatation,anorectalfissures,abscessand
fistulae. If the patient has involvement of the small bowel, intestinal obstruction is another
complication to be considered. Fistulae may form and the patient may present with a
complicationrelatedtothefistula(e.g.pneumaturia).
Ulcerative colitis is confined to the mucosa of the large bowel and most patients are well
controlled on medical therapy. Complications may include a flare-up of the colitis, acute
dilatationofthecolonandmalignancy.
At this stage in the management process it would not be appropriate to bombard the patient
withallthisinformationandcauseunnecessaryconcern,particularlywithtalkaboutmalignancy
ortheplaceofsurgery.Atthisstage,thepatientwillbeseekinganametotheproblemandthe
reassurancethattheconditioncanbetreated.

600
4-B: Life-threatening Emergencies Priorities of Treatment
Bryan W Yeo
'Extreme remedies are most appropriate for extreme diseases.'
Hippocrates (7460-377BC)

The ability to recognise when a patient is severely ill, rather than just unwell,
is potentially life-saving, even if a precise diagnosis is not immediately
apparent.
In severely life-threatening illness, treatment and diagnostic plans must
proceed simultaneously and rapidly if a successful outcome is to be
achieved. Emergencies requiring such early recognition include:
CARDIAC EMERGENCIES cardiac arrest, arrhythmias, myocardial
infarction, pericardial tamponade.
Inseverelylife-threateningillness,
treatmentanddiagnosticplans
mustproceedsimultaneouslyand
rapidlyifasuccessfuloutcomeis
tobeachieved.
RESPIRATORY EMERGENCIES respiratory arrest, airway obstruction and asphyxiation, tension
pneumothorax, flail chest, sucking chest wounds.
CIRCULATORY EMERGENCIES shock due to volume loss requires refilling of the pipes and turning off
the tap. Refractory shock requiring specific management may be cardiogenic, obstructive, septic or
anaphylactic.
NEUROLOGICAL EMERGENCIES patients who are comatose, stuporous or paralysed need rapid
assessment and treatment with exclusion of treatable cerebral or spinal compression.
OVERWHELMING SEPSIS specific infections (e.g. meningococcaemia) require early diagnosis and
treatment to avoid a fatal outcome.
BIRTHING EMERGENCIES problems are compounded when two lives mother and babe are at
stake.
EMERGENCY MANAGEMENT OF SEVERE TRAUMA (EMST) management of severe trauma is aided
by effective triage leading to efficient and rapid protocols of primary (ABCDE: Airway, Breathing, Circulation,
Disability, Exposure) and secondary surveys where treatment and diagnosis proceed hand in hand.
POISONING AND ENVENOMATION urgent diagnosis and management will be required for circulatory,
respiratory, neurologic and haematologic effects.

Bryan W Yeo

601
602
4-B
Life-threateningEmergenciesPrioritiesofTreatment


4-BLife-threateningEmergencies
CandidateInformationandTasks
MCAT113-118
113 WheezingandAseverelyill4-month-oldbabygirlwithfever
114 Alethargicfebrile2-year-oldboywitharash
115 breathingdifficultyina5-year-oldgirl
116 Cutstothewristofa25-year-oldman
117 Severepostpartumhaemorrhageina25-year-oldprimigravida
118 Emergencymanagementofasnake-biteina20-year-oldman
113
CandidateInformationandTasks
603
Condition 113
Aseverelyill4-month-oldbabygirlwithfever

CANDIDATEINFORMATIONANDTASKS
You are a junior Hospital Medical Officer (HMO) working in the Emergency Department. A
4-month-oldfemaleinfantisbroughtinbyherparentat0300hours.
Thebabyhasbeenincreasinglyunwellfor24hourswiththefollowinghistory:
Poorfeedsonlyonebreastfeedinthattime.
Feverish.
Decreasedurineoutput.
Excessivedrowsinessanddifficulttowakeforfeeds.
Therehasbeennovomitingordiarrhoea,andnocoughorwheeze.
The baby is the first born infant of healthy parents who are both currently well. She was
delivered normally at term after an uneventful pregnancy. She has been thriving and
developingnormallyuntilnow.
YOURTASKSARETO:
Asktheexaminerfortheclinicalfindingsyouwouldwishtoelicitonexamination.
Discussthelikelycausesoftheinfant'sillnesswiththeveryanxiousparent.
Explainyourplanofmanagementtotheparent.

Nofurtherhistoryisrequired.
ThePerformanceGuidelinesforCondition113canbefoundonpage610
114
CandidateInformationandTasks
604
Condition 114
Alethargicfebrile2-year-oldboywitharash

CANDIDATEINFORMATIONANDTASKS
Atwo-year-oldboyisbroughtbyhisveryupsetparentstoageneralpracticeinasmallcountry
town,50kmfromaregionalcity.Thechildhasbecomelethargicandfebrileinthelastfourhours.
He has had a mild upper respiratory tract infection for the last three days. Now he has a high
fever, is uninterested in food, irritable and has very cold skin. He is an only child of healthy
parents.
Onexamination,helooksunwellandhasafinenonspecificmacularpetechialrashonthetrunk
andlegsasillustratedbelow.Elsewheretheskiniscoldandpale,especiallyovertheextremities.
Vitalsigns:
Temperature 40"C(tympanicmeasurement)
Respiratoryrate 48/min
Pulse 150/min
Bloodpressure 90/60mmHg
Neckstiffness Noneapparent
YOURTASKSARETO:
Explainthediagnosticpossibilitiestotheparent.
Outlineyourmanagementplan.
CONDITION114.FIGURE1.
ThePerformanceGuidelinesforCondition114canbefoundonpage612
115
CandidateInformationandTasks
605
Condition 115
Wheezingandbreathingdifficultyina5-year-oldgirl

CANDIDATEINFORMATIONANDTASKS
A5-year-oldgirlisbroughttotheEmergencyDepartmentinbyherworriedparent.Shehasbeen
well until yesterday when she developed a cold. Last night she was coughing and woke with
wheezeandbreathingdifficulty.Shewentbacktosleeponandoffthroughthenightbutonwaking
thismorningstillhasanobviouswheeze.Hergeneralhealthhaspreviouslybeengood,apartfrom
anepisodeofmildwheezingassociatedwitharespiratoryinfectionwhenshewas3-years-old.
Shealsohasa pasthistoryofmildflexuraleczemabuthasnoknownallergies. Herfather had
severalepisodesofwheezinguntiltheageofeightbutnonesince.Hedoes,however,sufferfrom
seasonalhayfever.

Examinationfindings
Sheappearstobeinsomerespiratorydistressandhasanaudiblewheeze.
Temperature 37.5C
Pulse 110/min
Respiratoryrate 25/min
Nocyanosis
Clearnasaldischarge
Throat slightlyreddened
Chest nodeformity,percussionresonant.Generalisedmarked
inspiratoryandexpiratorywheezes.Nocrackles.
Nootherabnormality
YOURTASKISTO:
Explainthediagnosisandyourplanforfurthermanagementtotheparent.
Thereisnoneedforyoutotakeanyadditionalhistoryorperformanyfurtherexamination.
ThePerformanceGuidelinesforCondition115canbefoundonpage614
116
CandidateInformationandTasks
606
Condition 116
Cutstothewristofa25-year-oldman
CANDIDATEINFORMATIONANDTASKS
You are working in a general practice setting. Your patient is a 25-year-old fulltime university
studentlivingathomewithhisparentswhoarebothteachers,andthreeyoungersiblings.
Youhavejustseenthisyoungman,previouslyunknowntoyou,todealwithcutstohisleftwrist
andhand.Hetoldyouhedidnotknowhowhesustainedhisinjuries.Youdidalimitedpsychiatric
assessmentinthesutureroom,andestablished:
Hehasbeenupsetoverthepastthreedaysandhasbarelyslept,havingquiteatypicallyfailed
hissemesterexams.
Heisconvincedhehadaninfalliblemethodofpredictingthequestionstobeaskedinthe
exams,onlystudiedthosequestionsandcannotunderstandhowitdidnotwork.
Hedeniesanysuicidalthoughtsorimpulses,anddeliberateself-harm.
Hetoldyouthathehadbeen' doi ngsomedrugs' butnootherdetailswereabletobeelicited.
Hedeniesanypasthistoryofpsychiatricillness.
Hisgeneralhealthisgood,andheisnottakinganymedicationofanykind.
Younotedheisactivelyhallucinatingandthought-disorderedandconcludedhewasacutely
psychotic.
Hedoesnotconsiderhimselfunwell,andjustwantstogohometosleep.
Thepracticenursehasdressedhiswoundsandyouarenowgoingtoseehimandaddress
thepsychiatricmanagement.

YOURTASKSARETO:
Informthepatientofyourevaluationofhisproblemandtreatmentrecommendations.
Informtheexaminerofthereasonsforyourproposedmanagementandotherpossible
options.
Explainyourdecisiontothepatient.

ThePerformanceGuidelinesforCondition116canbefoundonpage618
117
CandidateInformationandTasks
607
Condition 117
Severepostpartumhaemorrhageina25-year-oldprimigravida
CANDIDATEINFORMATIONANDTASKS
This25-year-oldprimigravidahadanormalvaginaldeliverybythemidwife20minutesagoina
countryDistrictHospitalinwhichyouareaHospitalMedicalOfficer(HMO),andcurrentlyoncall
for the Obstetric Unit. The pregnancy had been perfectly normal. The labour was of 14 hours
duration. Only one dose of analgesia had been required. The estimated blood loss at delivery
wasonlyabout250mL.Howeverafurther1500mLofbrightbloodhasbeenpassedinthelast
15minutes.Themidwifehasjustphonedyoutoadviseyouofthesefacts,andtoaskyoutocome
andhelpwiththepatient'scare.

YOURTASKSARETO:
Ask the midwife the appropriate questions to define the probable cause of the
haemorrhageandtoassistyoutodefinewhatcareisnowrequired.Youshouldnottake
morethanfourminutestodothistask.
Advise the midwife of what she should do between now and when you will arrive in the
deliverysuite.Youarecurrentlyatyourflatwhichistenminutesfromthehospital.
Advisetheexaminerofthemostprobablecauseofthehaemorrhage,andwhatyouwilldo
whenyouarriveinthedeliverysuite.

ThePerformanceGuidelinesforCondition117canbefoundonpage622
118
CandidateInformationandTasks
608
Condition 118
Emergencymanagementofasnakebiteina20-year-oldman

CANDIDATEINFORMATIONANDTASKS
YouareworkinginacountryhospitalEmergencyDepartment.A20-year-old-manisbroughtto
the hospital by car after having been bitten by an unidentified snake 30 minutes ago. He was
bareleggedwhilewalkingneartheriveranddidnotnoticethesnakelyingon thepathconcealed
by shadows. He accidentally stepped on the snake which was brown in colour and about two
metresinlength,whichslitheredawayafterbitinghimonceabovetheankle.
Hewasdrivenstraighttohospitalbyhisfriendwhowaswalkingwithhim.Hehassomepainat
thesite,helooksanxiousandworried.Heishaemodynamicallystable.Thesiteof thebiteisas
showninthephotographbelow.Afewdropsofvenomremainontheskinnearby.

YOURTASKSARETO:
Applyafirstaiddressingtothelimbusingthematerialsavailable.
Telltheexaminerwhatyouaredoingandwhy.Neartheendofthetimeperiod,the
examinerwillaskyouoneortwoquestions.
CONDITION118.FIGURE1. CONDITION118.FIGURE2.
ThePerformanceGuidelinesforCondition118canbefoundonpage625
609
4-B
Life-threateningEmergencies
PrioritiesofTreatment


4-BLife-threateningEmergencies
PerformanceGuidelines
MCAT113-118
113 Aseverelyill4-month-oldbabygirlwithfever
114 Alethargicfebrile2-year-oldboywitharash
115 Wheezingandbreathingdifficultyina5-year-oldgirl
116 Cutstothewristofa25-year-oldman
117 Severepostpartumhaemorrhageina25-year-oldprimigravida
118 Emergencymanagementofasnakebiteina20-year-oldman
PerformanceGuidelines
Condition 113
Aseverelyill4-month-oldbabygirlwithfever
AIMSOFSTATION
Toassessthecandidate'sabilitytodiagnoseandmanageacutelife-threateningsepsisininfancy.

EXAMINERINSTRUCTIONS
Theexaminerwillhaveinstructedtheparentasfollows:
Youaretheanxiousparentofa4-month-oldinfant,Emma,whohasbecomeveryunwelloverashort
periodoftime.Initiallyyoudidnotthinkthattherewasanythingseriouslywrong.Yousubsequently
becamealarmed,asthebabyhasbecomelethargicandunresponsive.Youarenowverydistressed
byyourbaby'sappearanceandfrightenedshemightdie.
Questionstoaskunlessalreadycovered:
' WhatiswrongwithEmma?'
'Couldmybabydie?'
'Wherewouldshegetaninfection?' (Askthisifinfectionissuggestedasthecauseofthe
problem)
Isthisbecausewehaven' tbeenlookingafterherproperly?'
' Whataretheseinvestigationsyouwanttodo?'

Examinationfindingstobegiventothecandidatebytheexamineronrequest
Drowsy,poorlyresponsivebutwell-nourishedinfant
Thebabylooksdehydratedandill,withablotchycyanosisoftheextremities
Temperature39.5C
Pulserate160/min
Respiratoryrate20/min
Earsandthroatareslightlyinjected
Anteriorfontanelleissoft
Nootherabnormalityisfoundonfullexamination

EXPECTATIONSOFCANDIDATEPERFORMANCE
The candidate should realise that the condition is life-threatening from history and examination
findings,andshouldexplainthat:
Babyneedstobeadmittedtohospitalimmediatelyforinvestigationandtreatment.
Itcanbedifficulttodeterminetheexactcauseoftheinfant'sillnesswithout investigationsasbabies
presentinadifferentmannerfromadults.
Infection is highly likely and there are multiple investigations required, which include: blood
cultures,fullbloodexamination,inflammatorymarkers,urinemicroscopyandculture,chestX-ray,
lumbarpuncture.Explainthereasonsforeachinvestigation.
610
113
PerformanceGuidelines
Thisisnotduetosomethingtheparentshavefailedtodofortheirbaby.
Inform the parents that an intravenous line will be inserted and once the investigations and
specimensaretaken,antibioticswillbecommenced.
Immediatemanagement
Admitimmediatelytohospitalforinvestigation(ortransfertomajorpaediatrichospitalifwithin30
minutestransfertime).
Commence intravenous fluids and antibiotics (e.g. cefotaxime and gentamicin) or other suitable
broad spectrum antibiotic cover until diagnosis confirmed. If intravenous access cannot be
obtainedrapidly,giveintramuscularantibioticinitially.

KEYISSUES
Recognitionofclinicalfindingsindicativeofacutelife-threateningsepsisininfancy.
Recognitionofanextremelysickinfantrequiringimmediatediagnosticworkup.
Recognitionthatinvestigationandtreatmentmustproceedtogether.
Recognitionthatparenteral(preferablyintravenous)antibiotics(broadspectrumcoveragetocope
withallpossibilities)willberequired.
CRITICALERRORS
Failuretoadmittheinfanttohospital.
Failuretoundertakeurgentinvestigations.
Failuretorecommendimmediateantibiotictreatment
COMMENTARY
Thisscenariodescribesa4-month-oldbabygirlwhopresentsfebrileandobviouslyveryunwell.The
diagnosisismostlikelytobeseverebacterialsepsiswithnosignssuggestingthecausaloriginofthe
infection.Thisinfantrequiresurgentadmissiontohospitalforinvestigationandtreatment.
Inthissituationacauseofoverwhelmingsepsismustbesoughtandthiswillinvolveafullworkupof
investigations to determine the probable origin and spread of infection. This will include full blood
examinationincludingbloodcultureandinflammatorymarkers,chestX-rayandexaminationofurine,
preferablybysuprapubicaspiration,andcerebrospinalfluidvialumbarpuncture.
Infants present in a very nonspecific manner when infected: physical signs that may be present in
olderchildren,forexampleneckstiffness,areabsent.Themedicalattendantthereforemustseekout
multiplepossibleinfectionsources.
Havingobtainedtheseinvestigationsrapidly,thesafestandmostappropriateactionisthentotreat
the infant with antibiotic therapy preferably intravenously, providing a broad cover for appropriate
organisms(e.g.cefotaximeandgentamicin).Thereisnoplaceinthissituationforobservationwhile
awaitingresultsastheinfantmaydeterioraterapidlyandirretrievably.Undermostcircumstancesone
orotheroftheaboveinvestigationswillrevealthesourceofinfection.
Younginfantsbecomeillquicklywithbacterialinfectionandwilldeterioraterapidlyifnotinvestigated
andtreatedurgently.Appropriatetreatmentoftengivesrapidimprovement.
611
114
PerformanceGuidelinesCondition 114

Alethargicfebrile2-year-oldboywitharash
AIMSOFSTATION
To assess the candidate's ability to recognise a paediatric emergency order appropriate treatment
andexplainthemanagementempathicallytoananxiousparent.
Thisscenarioisofafebrileillchildwithashorthistorywhohasarash.Theclinicalpicturestrongly
suggests septicaemia, probably meningococcal, though H. i nfl uenzae is a possibility. Few other
diagnosesneedbeentertainedatthisstage.
EXAMINERINSTRUCTIONS
Theexaminerwillhaveinstructedtheparentasfollows:
Openingstatement
' Helooksawful,doctor.Ithappenedsoquickly'.
Howtoplaytherole
Youareextremelyconcernedaboutyourverysickchild.Heisyourfirstchild,andyouhavehadno
illness in the household. The doctor has just examined your child and is explaining to you what is
likelytobewrongandwhatmanagementisadvised.
Questionstoaskunlessalreadycovered:
' Howseriousisit?'
Isitjustanastyviralinfection?'
' Coulditbemeningitis?'
' Doesheneedtogotohospital?'
'Doesheneedanytreatmentbeforehegoestohospital?'
'Whattestsdoesheneed?'
EXPECTATIONSOFCANDIDATEPERFORMANCE
Thecandidateshould:
Explainthediagnosis,whilearrangingforurgentadmissiontohospitalasfollows:
thechildisveryill;
heurgentlyrequiresimmediatetreatmentbeforeheleavesheretogotohospital:
hemostlikelyhasabacterialinfectionwhichcanrapidlycausemarkeddeteriorationwithavery
seriousoutcomewithouturgenttreatment(onethirdofpatientshaveafulminatingcourse);
meningococcaemiaisadefinitepossibility;and
immediatetreatmentwithantibioticsisofparamountimportance.
Indicateamanagementplan
Takebloodforcultureandadministerintravenouscephalosporin(preferred)orbenzylpenicillin.If
intravenousadministrationcannotrapidlybeperformed,giveantibioticby intramuscularroute.
612
114
PerformanceGuidelines
Arrangeimmediatehospitalisationinachildren'sunitfordefinitiveinvestigationandmanagement
(telephonetoEmergencyDepartment,paediatricianorinfectiousdiseaseconsultant).
Explaintoparentthemostlikelymanagement:bloodculture,lumbarpuncture,urinemicroscopy
andintensiveantibiotictherapy.
KEYISSUES
Recognitionofaseriouslyillchild,whorequiresurgenthospitalisation.
Suspicionofmeningococcaemia.
Recognitionofurgencyoftreatingpotentialsepticaemia.
Recognitionofneedforurgentimmediateadministrationofantibioticspriortotransfer
Empathicexplanationofseriousdisease.
CRITICALERRORS
Notrecognisingthepossibilityofmeningococcaldisease.
Failuretoensureimmediateadministrationofcephalosporinorpenicillinandfailuretoarrange
urgenthospitalisation.
COMMENTARY
TheDominantCueforthisscenarioistherashinachildwhohastheunusualcombinationofpallor
andfever.
Theusualpatternoffebrileillnessinchildhoodisthatofaflushedchild,whofeelshot,andprotests
whenhandled.Thepresentscenariodeviatessignificantlyfromtheusualpattern,asthechildispale
anduninterestedwhenhandledthepatternofanunusuallyandseverelyillchildAnysuchchildwho
has a rash, which may in the early stage be very nondescript, should be regarded as having
septicaemia until proven otherwise. This will probably be meningococcal. Before introduction of
routineinfantimmunisationagainstHaemophi l us i nfl uenzae, haemophilusinfluenzalinfectioncould
also present in this manner. Immunisation with meningococcal vaccine is likely to lessen the
frequencyofthissortofpresentation,butanychildwhopresentslikethis,evenifimmunised,should
betreatedurgently.
PatternRecognitioninthisscenariopromptsurgentaction,whichinthiscaseisimmediatetreatment
with intravenous or intramuscular antibiotics. These must not be withheld while the child is
transferred,asthatcouldbefatalThirdgenerationcephalosporinsarethepreferredchoicebecause
ofthepossibilityofH. i nfl uenzae, butpenicillinGwouldbeacceptableforinitialtreatment.
Whilenotappropriateatthisstage,theclosecontactsofthischildwillrequireantibioticprophylaxis
oncethechild'streatmenthasbeenarranged.Ifclinicalsuspicionofthediseaseisstrong,thisshould
becommencedassoonaspossible.
Meningococcaldiseaseintheearlystagescanbedifficulttorecogniseandanycluetosuggestthatit
couldbepresentmustbefollowedup.Theappearanceofarashinasickfebrilechildshouldalertthe
clinician to this possibility, no matter how atypical the rash may appear. While other organisms,
especiallyviruses,maycauserashesinfebrilechildren,oncemeningococcaemiaiswellestablished
itmaybeimpossibletocontain.
613
115
PerformanceGuidelines
614
Condition 115
Wheezingandbreathingdifficultyina5-year-oldgirl
AIMSOFSTATION
To assess the candidate's ability to make a diagnosis of asthma precipitated by infection in a
child, to explain the diagnosis and its implications, prescribe appropriate treatment, and to
providetheparentwithappropriateguidanceforafutureepisode.

EXAMINERINSTRUCTIONS
Thegirlhasacuteasthma:supportedbyherpasthistoryofwheezewitharespiratoryinfection
when3-years-old,andofflexuraleczema.
Theexaminerwillhaveinstructedthepatientasfollows:
YouaretheconcernedparentofSarah.Youhavebeenupmostoflastnightwithherbecauseof
hercoughing,breathingdifficultyandwheezing.Youdonotknowmuchaboutasthmaandyou
arealarmedifthediagnosisissuggestedforSarah,asaneighbour'schildrecentlyhadsevere
asthmarequiringanintensivecareunitadmission.
Openingstatement
' She had a cold yesterday doctor and now she has got this nasty wheeze still. Does
sheneedanantibiotic?'
Questionstoaskunlessalreadycovered:
' Whatiscausingthesewheezingepisodes?'
' Whatcanwedoaboutit?'
'Willshealwayshavethisproblem?'
' CouldyouexplaintomehowIgivethesemedications?'
Ihadadreadfulnightwithherlastnight!'
' WhatshouldIdoifshehasanotherepisodelikethis?'
' Shouldwebebuyingoneofthosepumps?'
' Howdothesemedicationswork?'
Ifsteroidsarementioned,ask' Arethereanysideeffects?Ihaveheardaboutsteroids
andthoughttheyaredangerous.'
115
PerformanceGuidelines
615
EXPECTATIONSOFCANDIDATEPERFORMANCE
Thecandidateisexpectedto:
Makethediagnosisofacuteasthma.
Explain the mechanism of acute asthma attacks in children. They are most commonly
precipitated by viral infections causing bronchospasm. Also aggravated by cold weather,
exercise, change in weather, smoke and inhaled allergens. Less commonly due to food
allergens.
Ensure immediate relief of symptoms, by administering salbutamol or a similar agent via
metereddoseinhaler(MDI)andspacer/mask/mouthpieceorbynebuliser.
Ifnoreliefafter20-30minutes,repeattreatmentandrefertohospital.
Mayrequireinpatienttreatmentifnotrespondingtotreatment.
Ifhospitalisationrequired,candidateshouldexplainthetreatmentSarahwillhavetnhospital.
Futuremanagement
Enquireifparentssmokeandifso,encouragethemtoquitoratleastsmokeoutdoors.
Explainhometreatmentwithaerosolbronchodilatorusingaspacerasillustratedandinstruct
parentsastohowthismaybegiven.

CONDITION115.FIGURE1.
Inhalerattachedtospacer
Explainprobabilityofrecurrenceespeciallywithviralupperrespiratorytractinfections.
Reassureherparentsofthegenerallygoodprognosis.
Requestreviewattendancewithinafewdays.

KEYISSUES
Confidentexplanationofthediagnosisofasthma.
Appropriatemanagementofthisattackofacuteasthma.
Candidateshouldoutlineanasthmatreatmentplanforthechildforfurtherattackasoutlinedin
theactionplantable.
Supportandreassurance.
115
PerformanceGuidelines
616

EXAMPLEOFSIMPLEASTHMAACTIONPLANS
SYMPTOMBASEDPLAN
Whenwell
Youwi l l
Befreeofregularnight-timewheezeorcoughorchesttightness
Havenoregularwheezeorcoughorchesttightnessonwakingorduringtheday
Beabletotakepartinnormalphysicalactivitywithoutgettingasthmasymptoms
Needrelievermedicationlessthan3timesaweek(exceptifitisusedbeforeexercise)
Whennotwell
Youwi l l
Haveincreasingnight-timewheezeorcoughorchesttightness
Havesymptomsregularlyinthemorningwhenyouwakeup
Haveaneedforextradosesofrelievermedication
Havesymptomswhichinterferewithexercise(You
mayexperienceoneormoreofthese)
Ifsymptomsgetworse,thisisanacuteattack
Youwi l l
Haveoneormoreofthefollowing:wheeze,cough,chesttightnessorshortnessofbreath
Needtouseyourrelievermedicationatleastonceevery3hoursormoreoften
Dangersignsrequiringimmediateaction
Yoursymptomsgetworseveryquickly.
Wheezeorchesttightnessorshortnessofbreathcontinueafterusingrelievermedication
orreturnwithinminutesoftakingrelievermedication
Severeshortnessofbreath,inabilitytospeakcomfortably,bluenessoflipsImmediate
actionisneeded:dial000foranambulanceandsay'severeasthmaattack'
IF AMBULANCE IS DELAYED GET SOMEONE TO DRIVE YOU TO THE
NEARESTHOSPITALEMERGENCYDEPARTMENT

CRITICALERROR
Failuretodiagnoseasthma.
115
PerformanceGuidelines
617
COMMENTARY
Inthisscenarioa5-year-oldchildispresentedbyanexhaustedparentwhohasbeenstruggling
throughthepreviousnightwithherchild'sacutebreathingdifficulty.Acarefulhistorydetermines
thatthemostlikelydiagnosisisasthmaandexaminationrevealsthatthechildstillhassignificant
respiratorydistress.
Recognitionoftheanxietycreatedbythisveryworryingsituationfortheparentisimportantand
reassuranceshouldbegiventhroughouttheconsultationastotreatmentandasubsequentplan
ofaction.
There are several important issues that must be addressed in this scenario. Firstly, the child
needsappropriateacutetreatmenttorelievehersymptomsandthismayrequirehospitalisation
inthefirstinstanceifshedoesnotrespondtoinitialtreatmentinthepractice.
Secondly, the parent needs to be educated about asthma and the strong associated genetic
element, as well as what the common precipitating causes are in children, for example, viral
infections.
Thirdly,theparentneedstobecounselledonfactorsthatmayaggravatethechild'sasthma,for
example,cigarettesmoke,andanappropriateplanofactionoutlinedfortheparentintheeventof
the child developing a further asthma attack. The clinician must ensure that the parent is
confidentinthismanagementandhashadappropriateinstructionintheuseofbronchodilators
andtheequipment,forexample,spacers,associatedwiththeiruse.
Acommonomissioninpracticeisadequateinstructionforparentsinmethodsofadministering
medications and the care of equipment. Some practitioners are confident in instructing the
parentsintheiruse,butifnot,thehelpofanasthmaeducatorshouldbesought,ifavailable.
Mostparentsareabletovery adequatelycope withtheirchild'sasthmaiftheyunderstandthe
pathogenesisandareconfidentthattheyknowexactlywhattodointheacutesituationandwhen
theyshouldcallforhelp.Itisimportanttoemphasisethateventhoughtheyhaveaplantofollow,
they should immediately seek help at any time irrespective of the plan if they are concerned
abouttheirchild'scondition.
116
PerformanceGuidelines
Condition 116
Cutstothewristofa25-year-oldman
AIMSOFSTATION
Toassessthecandidate'sabilitytoidentifythatthispatientisathighriskoffurtherharmandneeds
acutepsychiatrictreatment.Theriskisthroughpossibledeliberateself-harm,accidentalharmwhilst
intoxicated, or violence driven by psychotic phenomena. This is a psychiatric emergency, and the
patient needs to be constantly observed pending psychiatric inpatient admission or uptake by a
psychiatriccrisisassessmentteam,
EXAMINERINSTRUCTIONS
Theexaminerwillhaveinstructedthepatientasfollows:
Youarea25-year-oldfulltimeuniversitystudentwhohasjustseenthedoctortorepaircutstoyour
leftwristandhandyoudonotknowhowyousustainedthisinjury.
Openingstatement
ThenursesaidIhadtoseeyoubeforeIleave/toldyouthatIjusthavetohaveasleep!'
The candidate is expected to inform you that you need to have urgent psychiatric treatment, and
discuss with you the possible options, including the possibility of involuntary treatment if after
discussion about your condition, available supports, and intent, you do not accept voluntary
treatment.
Expectedresponsestocandidateenquiries:
Whenaskedaboutunderstandingofpresentcondition:' Thereisnothingwrongwithmeonce
Ihavefiguredoutwhathasgonewrongwiththissystem,I' llbeabletosleep.Will youhelp
mewithanappealaboutthemark?Thatmustbewhereithasgonewrong!'
Wheninvoluntarytreatmentisdiscussed:' I f youaretellingmethatyouaregoingtosendme
tohospitalwhetherIlikeitornot,thenImightaswellgobecauseIwouldhavetoanyway!
CanIgohomefirstandtellmyparents?'
Questionswhichshouldbeasked,withappropriateresponses:
' CanIgohome?'
Thecandidateshouldadvisethatitwouldbebesttophoneparentsandseeiftheycancomehere
ormeetyouatthehospital.
' Whataretheygoingtodoformethere?'
The candidate should advise that at the hospital, the doctors will decide on medication as the
patientneedssomethingforsleep,andwillperformteststohelpsortoutwhatisgoingon,aswell
asactingonresults.
Furtherinstructionstopatient
Youaretired,perplexedabouteverythingthatisgoingon,preoccupiedwithyourownthoughtsand
experiencingauditoryhallucinations('hearvoices' )thatyoulocateasoriginatingintheairventin
theroomandtowhichyouintermittentlymutterinaudibly.Ononeoccasion,saytotheareaofthe
vent'Can' t you shut up and let me think?' Resisttheideathatyoumighthaveanypsychiatric
illness,butaccepttheinevitablewhenyouhearthatyouwillbehospitalisedonaninvoluntarybasis.
If candidate does not discuss that, do not accept treatment and do not ask for your parents to be
contacted;andsayyouwishtoleave.
618
116
PerformanceGuidelines
619
EXPECTATIONSOFCANDIDATEPERFORMANCE
This patient in his current condition is certifiable/recommendable if his agreement with
recommended treatment is not obtained (terminology varies from state to state for involuntary
treatment,andtheexamsiteandcandidateworkexperiencewilldeterminetheexactwording).It
is acceptable for the candidate to inform the patient of this evaluation if so it should be
communicatedinanonthreateningway.
Thecandidateisexpectedtocommunicatetothepatientthatheispsychiatrically/emotionally
unwell,andthathisthoughts/feelings/experiencesaresodistressingandunpredictablethatthey
placehimatriskoffurtherinjury,thattreatmentisrequiredandwillhelp.Itisacceptabletouse
thecomplaintofsleeplossasafocusofinitialsymptomreliefintreatment.Itisessentialthatthe
candidateisexplicitthattreatmententailseitherpsychiatricinpatienttreatment,'hospitalinthe
home'orcrisisteammanagement.

Thecandidateisexpectedtocommunicatetotheexaminerthereasonsforthetreatment
decisionifthishasnotalreadybeenmadecleartothepatient:
Thepatientprobablyhasamajorpsychiatricillness.
Heisatriskofself-harm.
Herequiresurgenttreatment.
And the decisive point for involuntary certification in this case is that treatment cannot be
administeredinalessrestrictiveway.
Otherpossibleoptionsareto:
Contacttheparentforacollateralhistory(afterpatientconsent).
Administersedationwithbenzodiazepineorantipsychoticifnegotiationsfail.
Consideralternatives(e.g.sharedcare,communitytreatmentorderunlikelytobesecure
enough).

KEYISSUES
Abilitytounderstandanddirectlyaddressthefactofseriousmentalillness,riskofharm,and
needfortreatment.
Knowledgeofpsychiatriccrisistreatmentoptionsincludinginvoluntarytreatment.
Abilitytocommunicateandnegotiatewiththepatient,includingflexibilityandinclusivenessof
significantothers,suchasparents,withoutneglectingdutyofcare.

CRITICALERROR
Notensuringurgentreferralforpsychiatricassessmentandcrisiscare.
116
PerformanceGuidelines
620

COMMENTARY
Thisisamanagementtasktoimplementcrisiscareforayoungmanwithanacutepsychosis
andinjuryofunknownorigin.
The patient presents with a minor injury in the setting of acute psychosis. Such an injury may
havebeenaccidental(forexample,whilstintoxicated),duetodeliberateself-harm,ordrivenby
psychotic phenomena (for example, by command hallucinations or delusions). These
mechanismscarrydifferentdegreesofriskanddeterminingthespecificmechanismisimportant
in the understanding and management of each individual case. In crisis management in a
primarycareoroutpatientsetting,however,thefactofinjuryandpotentialforfurtherharmisthe
overridingprincipaldeterminantrequiringfurtheraction.
Presentationwithconsequentialadverseevents,suchasunexplainedself-injuryoroccupational
andsocialproblems,iscommoninacutepsychoticillness,asispresentationwithrequestsfor
reliefofsomaticorvegetativesymptomssuchassleepdisturbance.
Itisimportantfordoctorsinallareas,especiallythoseworkinginprimaryoremergencycare,to
beawareofthisphenomenon,andtohavethecapacitytoeffectivelyintervenewithappropriate
responsenotonlytothepresentingproblembutalsotheunderlyingpsychiatricillness,evenifnot
fully elucidated. There is a significant risk of serious injury through self-harm if this type of
presentationisignored.
There is accumulating evidence that early intervention in psychotic illness plays a vital role in
diminishingchronicityandseverityofoutcomeinarangeofbiopsychosocialareas.
Thus,appropriatedoctorresponseentails:
Decidinguponthelevelofrisktothepatientandothersbasedonobtainedinformationand
observation.
Preparednesstoactonthedecisionwithintheframeworkofthementalhealthlegislationand
crisispsychiatricresourcesintheregionofthedoctor'spractice.
Applying knowledge of psychopharmacology and skill in chemical restraint. In this case,
undertaking to ensure the patient has sleep relief whilst his condition is further assessed is
important.
Using communication skills to manage a difficult-to-engage patient; in this case responding
positivelytothepatient'srequestsforsleepreliefandassistancewithhisacademicproblems
istherapeutic,whereasaggressivelypursuingthepossibilityofillicitdruguseatthistimeis
not.
Applying good judgement with respect to privacy/confidentiality in communicating with
significantothers.Inthiscase,seekingconsenttoinformtheparentsofthecrisisisimportant,
butdisclosingspeculativeinformationsuchaspossibleillicitdruguseisnotacceptable.
Self-harm or unexplained injury in the context of acute psychosis is a serious psychiatric
emergency.Thedoctorinthisscenariohasadutyofcaretothisyoungmantoensurethathe
immediately receives a specialist mental health/psychiatric assessment and that he is
continuously supervised until he has had that assessment. He remains the doctor's
responsibilityuntiltheassessmentoccurs.
116
PerformanceGuidelines
621

Inreality,oncethisyoungmanisinformedthathehasaseriousmentalillnessandthathewill
havetogotoahospitaloranotherclinictoseeapsychiatrist,heislikelytobecomeanxiousand
frightened.Hemayattempttoeitherrunawayimmediatelyorusesomedistractingstrategyto
evade his chaperone and escape. Typically he will ask to go to the toilet; to go outside for a
cigarette,acoffee;tomakeaphonecall;gototheATM;getamagazineornewspaper;movehis
carHemaythenrunoffanddisappearinastateofpanic.Theriskoffurtherself-harmmaybe
exaggeratedoncethepatientbecomesawarehehasaseriousmentalillness
Hisperplexityanddenialofknowledgeofhisinjuriesandhisactivepsychosiswithgrandiosity
makehim atextreme riskof furtherself-harm.Thementalhealth crisisteam orthepoliceand
ambulanceneedtobesummonedtothedoctor'sgeneralpracticeassoonaspracticablerather
than allowing him to be taken to the clinic/assessment centre/ hospital in a private car
accompaniedbyafriendorwithhisparents.
Toomuchsedationgivenbythedoctoratthegeneralpracticeinthissituationwillonlyconfound
or prolong the subsequent specialist assessment if the young man is rendered senseless or
unable to communicate. Oral diazepam 5-10 mg and/or olanzapine 5-10 mg would be
appropriateinitialchoicesforsedationbythegeneralpractitioner.Diazepammustneverbegiven
intramuscularly.
117
PerformanceGuidelines
Condition 117
Severepostpartumhaemorrhageina25-year-oldprimigravida

AIMSOFSTATION
To assess the candidate's ability to diagnose and manage a patient having a serious primary
postpartumhaemorrhage.

EXAMINERINSTRUCTIONS
Theexaminerwillhaveinstructedthemidwifeasfollows:
YouarethemidwifespeakingtothedutyHospitalMedicalOfficer(HMO)onthetelephone.Youwill
needtoanswerquestionswhichareaskedaboutthepatient,andaskforanyrecommendationsfor
treatmentuntiltheHMOarrivesinthedeliverysuite.
Thefollowingisalistofappropriateanswersthesearetobeprovidedfollowingspecificrequestsby
thedoctor.
Vitalsigns:bloodpressure85/50mmHg,pulserate120/min.
Thelabourlasted14hours.
Thedeliverywasspontaneousandvaginal.
Thebabyweighed3800gandwasingoodconditionatdelivery.
Anepisiotomywasnotcut,andthereisnotearintheperineum.
Thebloodisjustpouringoutandisbrightred.
IfaskedIsitclotting?answerIdon' tknow' .
Shehashadnopreviousuterinesurgery.
Noergometrineoroxytocin(Syntocinon)hasbeengiven,atthepatient'srequest
Theplacentaweighed800gandlookedcompleteandnormal.Itwasdeliveredfiveminutesago.
Ihavenotpassedaurethralcatheter.
Theuterusiscentralinposition.
Theuterusappearsslightlylaxandextends2cmabovetheumbilicus.
Totalmeasuredbloodloss:1500mLsofar.
Haemoglobinoneweekago:120g/L.
Bloodgroup:ORhpositive.
Shedoesnothaveanintravenousdriprunning.

EXPECTATIONSOFCANDIDATEPERFORMANCE
Thepossiblecausesofthehaemorrhagetobeconsideredrapidlyare:
Uterineatony.
Genitaltractlacerationvagina,cervix,uterinerupture.
Retainedplacentalfragment,cotyledonormembranes.
Acoagulationdisorder.
Thequestionsaskedbythedoctorofthemidwifeshouldattempttoclarifywhichofthesecausesis
mostlikely.Theappropriatequestionsandresponsesarelistedintheexaminerinstructionstothe
midwife.
622
117
PerformanceGuidelines
623

Followingthequestionsandresponses,thecandidateshouldgivethefollowingadvicetothe
midwife:
Becausethemostlikelydiagnosisisahaemorrhageduetouterineatony,theinitialinstructions
foractiontobetakenbythemidwifeneedtobe:
Massagethefundus,
Gainintravenousaccessifyoucan.
Give intravenous or intramuscular ergometrine (0.25 mg) or intramuscular ergometrine and
oxytocin (Syntometrine) immediately. Some candidates may suggest the use of a
prostaglandinpreparationgivenperrectum.
Starttransfusionrapidlywithsaline,Hartmannsolution,orHaemaccel.
Takebloodforcross-matching(goodcandidateswouldalsorequestbloodshouldbetakenfor
coagulationstudiesaswell).
Passaurethralcathetertoensurethebladderisempty.
Observeanybloodwhichisbeingpassedtocheckwhetheritisclotting.
Commentarytoexaminershouldbealongthefollowinglines:
Thecausecouldbeanyoftheabovediagnoses,butthemostlikelyisuterineatonybecauseof
thelongishlabour,thelackofadministrationofoxytocicsandtheabdominalfindings.Although
other causes cannot be excluded at this time the initial treatment should be the treatment of
uterineatony.
AssoonasIarrive,Iwilldothefollowingunlessthebleedinghassubstantiallydecreased.
~Insertanintravenousdripifnotcompletedbymidwifeandcommenceorcontinue
crystalloidfluidsorHaemacceluntilbloodisavailable.
~Inspecttheplacentamyselftoensureitlookscompleteandnormal.
~Doaspeculumexaminationtoensurethereisnovaginalorcervicallacerationwhich
mayrequiresuturing.
~Regularlymonitorthevitalsignstocheckthepatient'scondition.
~Giveoxygenbymask.
~Givebloodassoonasavailableandcross-matched.
Ifthebleedingcontinues,addSyntocinon20-50units/litretotheintravenousfluidsbeing
administered,andnotifytheobstetricconsultant;asitishighlylikelyanexaminationunder
anaesthesia(EUA)willberequiredtoensuretherearenoretainedproductsofconception
(RPOC),theuterusisintactandthereisnoevidenceofapartialuterineinversion.Ifpossiblethe
EUAshouldnotbeperformeduntilabloodtransfusionisrunningandsheishaemodynamically
stable.
It would generally only be after an EUA has been performed that the use of intramyometrial
prostaglandinpreparationssuchasPGF2a,orinvasiveandmajorsurgerysuchasinternaliliac
ligation(orevenhysterectomy)wouldbeconsidered.

KEYISSUES
Knowledgeofthecausesofprimarypostpartumhaemorrhage.
Abilitytorecognisethemostlikelycauseofaprimarypostpartumhaemorrhage.
Abilitytoadvisethemidwifeoftheinitialmanagementrequired.
Abilitytounderstandthestepsrequirediftheinitialmanagementisnotsuccessfulin
controllingthehaemorrhage.
117
PerformanceGuidelines
624

CRITICALERRORS
Failuretodefinethatuterineatonyislikelytobepresentontheinformationobtainedfromthe
midwife.
Failuretoaskforinformationregardingthevitalsigns.
Failuretoadvisethemidwifetoinitiatetheinitialactionsdetailed.Theperformanceof
coagulationstudiesisnotmandatorybuttheotheractionsare.

COMMENTARY
Severepostpartumhaemorrhageisalife-threateningconditiontothemotherandtheimmediate
management of this condition must be known. The four causes of postpartum haemorrhage
(genital tract trauma, retained tissue, blood clotting abnormalities, and uterine atony) must be
consideredinthiscase.Thevarioustreatmentsforeachoftheseconditionsmustbediscussed.
Thevariousdrugsusedtomanageuterineatonymustbeknown.
Commonproblemslikelywithcandidateperformanceare:
Failuretoconsiderthepossibilityofacervicalorvaginallacerationbecausetheperineumwas
intact,andfailuretoindicateaspeculumexaminationofthevaginaisnecessary.
Failuretoconsiderthepossibilitythatsomeretainedproductsofconceptionmaystillbeinthe
uterusandthatexaminationunderanaesthesiawouldberequiredifthebleedingcontinued,
eveniftheplacentaappearstobecompletewheninspected.
118
Performance Guidelines
625


Condition 118
Emergencymanagementofasnakebiteina20-year-oldman
AIMSOFSTATION
Toassessthecandidate'sabilitytoapplythepressure-immobilisationtechniqueoffirstaidtoa
victim of a snake bite from an unidentified snake; and to test recognition of symptoms of
envenomationanditstreatment.

EXAMINERINSTRUCTIONS
Theexaminerwillhaveinstructedthepatientasfollows:
He was bitten on the anKle half an hour ago by an unidentified snake. No symptoms of
envenomationarepresentatthisstage,butheisconcernedandanxious.
The patient is lying on a couch wearing shorts. The bite marks are visible and a few drops of
venomareontheskin.
The candidate is expected to employ first aid by applying a pressure-immobilisation crepe
bandagetothelimb,andtoimmobilisethelimbbyappropriatesplintage.Nospecificlocalwound
toiletortreatmentisindicated.Knowledgeablecandidateswillrecognisethepresenceofvenom
ontheskinandcollectsomeforidentificationwiththeVenomDetectionKitwhichisavailablein
mosthospitals.Anysuggestiontoapplysuctionortoinciseorexcisethewoundisincorrectand
wouldbemarkeddownaccordingly.
The appropriate technique is to use a wide crepe bandage from the equipment provided,
applyingitfirmlytothewholelimbfromtoestoaboveknee,applyingthebandagewiththedegree
ofpressureappropriateforasprainedankle,thenapplyingasplintandattachingthebandaged
legtothesplintimmobilisingboththefootandknee.
Theavailabilityofoxygentherapyandappropriateresuscitativeequipmentshouldbecheckedby
the candidate. Use of a venous tourniquet above the bandage is not obligatory but would be
appropriate.Theuseofanarterialtourniquetisincorrectandinappropriate.
Afterthecandidatehascompletedthedressing,theexaminerwillask;
' What symptoms and si gns of envenomati on woul d you l ook for i n further fol l owup?'
' How woul d you treat the pati ent i f envenomati on i s di agnosed or suspected?'
Indications for antivenom administration are the presence of symptoms and signs of
envenomation developing over a period of observation in hospital with facilities for oxygen
administrationandforventilatoryandcirculatorysupportavailable.
Initial symptoms are often headache, nausea and vomiting, which may be accompanied by
circulatory collapse, clouding or loss of consciousness, muscle paralysis and coagulation
disturbance.
Initial signs of muscle weakness often include diplopia, local muscle paralysis and breathing
difficultyprogressingtoapnoeaduetotheeffectofneurotoxins.Somevenomshaveadditional
haematologic effects causing a coagulopathy with local and systemic bleeding with
thrombocytopenia.Severalcausesystemictoxicityaswell,withprogressiontorenal,respiratory
andmultiorganfailure.
118
PerformanceGuidelines
626
LocaltissuenecrosisatthesiteofthesnakebiteisnotafeatureofAustraliansnakevenomsas
distinct from the effects of venoms from exotic species such as the American rattlesnake.
IdentificationofthesnakecanbeaidedusingVenomDetectionKits(VDK)withswabbingofany
residualvenomfromtheskinsurfacearoundthebite.Soinfirstaid,venomshouldnotbewiped
orwashedfromtheskinnorshouldantisepticbeapplieduntilcheckingforresidualvenomhas
been done at the hospital. Venom present on the skin will NOT be absorbed and is NOT a
continuing risk envenomation is due to absorption of venom injected subcutaneously by the
fangsatthetimeofthebite.
Choiceofantivenom.Itispreferabletouseaspecificantivenomifthesnakeoritsvenomcanbe
positivelyidentified.Ifthesnakecannotbeidentified,thenormalprocedureistogivepolyvalent
antivenom (except in Victoria or Tasmania, where use of combined specific venoms for tiger
snakeandforbrownsnakeisrecommended).Expeditioususeof antivenomcanbelifesaving.
Antivenom is always administered intravenously. Most antivenoms are prepared from horse
serum.Eachampouleispreparedtocontainenoughantivenomtoneutralisetheaverageoutput
fromabiteoftheparticularsnake,butwithsevereenvenomationmultipleampoules,upto10or
more,mayberequired.
Because of the risk of allergic reaction to the antivenom, it is recommended that prior to
commencingantivenomadministration,subcutaneousadrenaline(0.25mginadultsand0.01mg
perkginchildren)shouldbegiven;andtheinfusionofantivenom(diluted1in10inHartmann
solution)commencedfiveminutesafteradrenalinehasbeengiven.
Loadedsyringeswithfurtheradrenalinesolution(1mgpermL),antihistamine,andsteroidshould
alsobeonhand,asshouldoxygenandfullresuscitationfacilities.
Afterobservationforreactionstothefirstdose(whicharerare),twoormoreantivenomampoules
shouldbeinfusedintravenously.
Prednisolone (50 mg orally) is also recommended daily for five days if antivenom is used, to
minimiseriskofdelayedserumsickness.Steroidsarestoppedafterfivedays,taperingthedose
isnotrequired.
KEYISSUES
Abilitytoapplyanappropriatefirstaiddressing.
Knowledge of features of envenomation and appropriate principles of management of
envenomation. The antivenom ampoules contain instructions as above, so that detailed
knowledgeofdosageandspecificprotocolsarenotrequired.
CRITICALERROR
Inabilitytoapplyanappropriatefirstaiddressing.

COMMENTARY
Australian venomous snakes are among the most dangerous in the world in terms of venom
toxicity and output particularly the taipan and the small scaled (fierce) snake; but other
venomous species are widely distributed and include tiger snake, common brown snake,
copperhead,deathadder,mulga,andred-belliedblacksnake.
118
PerformanceGuidelines
627
CONDITION 118. FIGURE 3.
Tigersnake
CONDITION118.FIGURE4.
Countrygraveofsnakebitevictim
Fatal attacks in humans are fortunately now rare, and effective pressure immobilisation
dressingsandantivenomshavesavedmanylives.Antivenomsrequirerefrigeratedstorageand
haveashelflifeofaroundthreeyears.Polyvalentantivenomcontainsvenomfromfivesnakes
andrequiresthereforethelargestvolumeofantivenomtobeused.SnakeVenomDetectorKits
(VDK)canhelpidentifytheneedforspecificantivenom.
CONDITION 118. FIGURE 5.
Venomdetectorkit
628

Legal, Ethical and


Organisational (LEO)

629
5
Kerry J Breen
' OnlyoneruleinMedicalEthicsneedconcernyouthatactiononyourpartwhich
bestconservestheinterestofyourpatient'
MartinHFischer(1879-1962)

5-A:Legal,EthicalandOrganisationalAspects:Ethicaland
LegalDilemmas
As emphasised in the introduction to this book, effective and compassionate practice of medicine
requires a combination of medical knowledge, clinical competence and professional attitudes and
skills. In the past, the last of this combination was called a 'good bedside manner' and was not
formally taught. It seemed to be assumed that young doctors would somehow acquire such skills,
perhaps by observation and experience. In Australia, for at least the last twenty years, the
medical profession, especially those responsible for the education and training of new doctors,
have identified professional skills as something which can and must be taught. Professional
skills or 'professionalism' covers a wide range of elements, including communication
skills, knowledge of medical ethics, personal and professional development, awareness
of relevant laws pertaining to medical practice and cultural awareness in our multicultural
society. Above all 'professionalism' includes an assumption that a person wishing to
practise medicine effecti vel y will bring to the task positi ve attitudes to all the roles
invol ved in being a doctor.
These new approaches to medical education and medical practice have been driven both by the
profession and by society. A landmark document which reflected the wish of the profession to be
responsive to our changing society was the 'Doherty report'
1
which recommended sweeping
changes to the medical student curriculum. The Australian community has gradually changed its
attitude to the traditional emphasis the medical profession had placed on beneficence (which
was increasingly seen to represent authoritarianism or paternalism). Through better knowledge
and understanding of science and medicine and an increasing desire for the information
necessary for health care decisions, the community has asserted the rights of patients to the
ethical principle of autonomy. These altered attitudes have also been reflected in decisions
made by Australian courts in regard to such matters as the adequacy of information to which
patients are entitled when giving consent to treatments which carry risks
2,3
. In the space of less
than
1 AustralianMedicalEducationandWorkforceintothe21stCentury.ReportoftheCommitteeofInquiryinto
MedicalEducationandMedicalWorkforce.Canberra,1988
2 RogersvWhitaker(1992)175CLR479
3 GeneralguidelinesformedicalpractitionersonprovidinginformationtopatientsNHMRCCanberra.1993
http://www.nhmrc.gov.au
5-A
630
Legal, Ethical and Organisational Aspects:
EthicalandLegalDilemmas

onegeneration,thekeyethicalpillarsformedical practiceofbeneficenceandnonmaleficence
have been overshadowed by the principle of patient autonomy bringing the expectation that
doctors will show respect for patients, share information willingly and ensure the adequacy of
consenttotests,treatmentandprocedures.
These changes have not been easily or readily adapted to by all members of the medical
profession. Putting some of them into practice can require considerable wisdom and skill,
especiallywherethebalanceofthoseethicalprinciplesmayneedtobemodifiedtoadjusttothe
wishes of older Australians who may still want their doctor to make decisions for them, or to
adjust to the needs of newly arrived citizens with different expectations. For doctors who
themselves are newly arrived in Australia, especially if they have come from countries where
more traditional approaches to the authority of the medical profession have been maintained,
adjustment may also be difficult. Adjusting to a different approach to professionalism will be
compounded should the health care system, and the disease and illness patterns, be vastly
differentinAustraliafromthoseofthedoctor'scountryoforigin.
TheAustralianMedicalCouncilisawareofthechallengeswhichfacedoctorswhoobtainedtheir
initialtraininginothercountries.TheCouncilhasstriventoseethateducationandtrainingare
available in the relevant areas of medical professionalism. It has altered its examinations to
incorporatekeyelementsoftheseaspectsofmedicalpractice.Thissectionprovidesexamplesof
howthishasbeendone.However,itisveryimportanttonotethattheseparationofthissection,
on professional skills and knowledge of law, ethics and the organisation and structure of the
healthcaresystem,fromthesectionsonconsultingskills,communicationskillsandclinicalskills
is artificial. To be able to practise medicine in Australia effectively, these professional skills all
needtobeappliedindailypractice.
The ethical principle of respect for the autonomy of patients brings the need for doctors to
examinethepersonalandprofessionalvalues(includingreligiousvalues)theymightbringtoa
consultation and consciously avoiding imposing their own values in problematic clinical
situations.ThisisnotjustanissuefordoctorsnewtoAustralia.Ourmulticulturalsocietyprovides
similarchallengesfordoctorswhohavetrainedinAustralia.
Thereisnosingledefinitionofwhattheterm'ethics'meansinhealthcare.Onewidelyaccepted
viewisthatitrepresentsthewayinwhichasocietydetermineshowitwillliveafterconsidering
carefullyalltheissues.Thussomepeoplesaythatethicaldecisionsare'all-things-considered'
decisions. The dominant ethical principles applying to the practice of medicine in Australia at
present reflect a combinationof historicalinfluencesdatingbacktothetimeofHippocratesas
wellastheJudeo-ChristianthinkingwhichthefirstEuropeansbroughtwiththemalittleovertwo
hundred years ago. The values of Aboriginal and Torres Strait Islander peoples have not yet
beenfullyrecognisedbynon-indigenousAustraliansalthoughguidelinesdevelopedrecentlyfor
the purpose of considering health research involving Aboriginal and Torres Strait Islander
peoples provide considerable relevant insights into the values and ethical views of these
Australians
1
.Viewsonethicsmaychangeovertimeandarecontext,timeandculturespecific.A
society's jointly held view on ethical issues will also influence personal values and views of
ethicalmatters,especiallyduringaperson'sformativeyears.
Laws are specific, ethics are conceptual. For doctors in Australia, several ethical codes of
practiceexist.ThemostwidelyknowncodeisthatoftheAustralianMedicalAssociation
2
,

1 ValuesandEthics.GuidelinesforethicalconductinAboriginalandTorresStraitIslanderresearch.NHMRC.
Canberra,2003.http://www.nhmrc.gov.au
2 AustralianMedicalAssociationCodeofEthics2003,http://www.ama.com.au
5-A
Legal,EthicalandOrganisationalAspects:Ethical
andLegalDilemmas
631
but ethical codes also exist tor the various medical colleges. In addition, medical registration
boards,whicharestate-based,provideguidancetodoctorsoverarangeofprofessionalmatters
includingstandardsofpracticeandlegislationrelevanttomedicalpractice.Medicalboardsare
notresponsibleforsettingthehighstandardstowhichdoctorsshouldaspire,butexisttoprotect
the community from doctors whose performance is alleged to have fallen below minimally
acceptablestandards.
Another way of expressing the expectations placed upon doctors (especially those who are
registeredtopractisemedicinewithoutsupervisioninAustralia)tohavesufficientknowledgeand
awarenessoftheprofessionalskillsrequired,istolistsomepracticalexamplesofethical,legal
andprofessionaldilemmasdoctorsmightfaceineverydaypractice.Itmustbepointedoutthatno
practisingdoctorinAustraliaisexpectedtohaveimmediatelyavailabletheinformationtohandle
allofthesedilemmas.However,doctorsareexpectedtobeabletorecognisetheissuesinvolved
and be willing and motivated to turn to relevant sources of advice and help. The following are
examples of some common matters relating to ethics, law and the organisation of medicine.
These are also issues which frequently result in complaints against doctors being lodged with
medicalboardsandhealthcomplaintscommissions.
Medicalcertificates:Writingamedicalcertificateisaserioustaskasthecertificatewillbeused
byemployers,insurancecompaniesandeventhecourtstodetermineaperson'sentitlementto
financial or other benefits. A doctor should never write anything on a certificate to which the
doctorcouldnotattesttounderoathinacourtoflaw.
Consent from minors: The definition of a minor is a legal matter, but in medical practice it is
possibleforolderchildrentogiveconsenttotreatmentandminorprocedures.
Ill and impaired doctors: Doctors may become ill and their capacity to practise impaired. In
addition, doctors are at risk of misuse and dependence on drugs of addiction also leading to
impairment. Doctors must be aware of the relevant state or territory laws relating to reporting
possibleimpairmentordrugmisuse.
Sexualmisconduct:Itisaseriousethicalbreach,oftenleadingtoderegistration.touseone'srole
asadoctortoestablishanimproperorsexualrelationshipwithapatient.
Notifiable diseases: There are a range of diseases, mostly infectious, which by law must be
notifiedtohealthauthorities.Muchofthisresponsibilityistakenonbypathologylaboratoriesbut
individualdoctorsarestillultimatelyresponsible.
When can confidentiality be breached? There are very few situations in which it is legally or
ethically permissible to breach patient confidentiality, and all doctors are expected to have a
workingunderstandingofthismatter.
Who is responsible for the professional conduct of your practice staff? In a medical practice,
ultimatemedicalandlegalresponsibilityrestswiththedoctor.Thusdoctorsmusttakestepsto
ensure that staff are trained in matters including those relating to privacy and confidentiality,
securityofrecordsandprescriptionpads,promptpassingonofmessagesandthelimitstothe
useoftheirpersonaljudgement.
Drugs of addiction: There are important laws that doctors must be familiar with covering the
securestorage,prescribinganduseofdrugsofaddiction.
Medicare fraud: Most general medical practice in Australia is funded by payments to patients
from an Australian Government statutory body, Medicare Australia. Systems are in place to
detect irregular patterns of patient and doctor access to benefits paid by Medicare. To avoid
unintendedabusesofthissystem,doctorsmustmakethemselvesfamiliarwiththeregulations
involved.
5-A
Legal, Ethical and Organisational Aspects:
EthicalandLegalDilemmas
632

Communicationfailure:Mostcomplaintsaboutdoctorsaswellasmuchlitigationfornegligence
haveastheirbasisafailureofcommunicationbetweendoctorandpatient.Medicalstudentand
postgraduate training programmes in Australia now place great emphasis on the acquiring of
good communication skills. Courses are also offered via the medical colleges and medical
indemnityorganisations.Abriefadvicedocumentongoodcommunicationbetweendoctorsand
theirpatientsisavailable.
6
Collegial support: In Australia, now and in the past, medical students and trainee medical
graduates are continuously relating to older more experienced doctors in what is partly an
apprenticeship'system.Asaresult,thereisahighlydevelopedsensethatdoctorsareallpartof
a profession that works together to support each member. This support extends to the ready
availability of free advice from senior colleagues when a doctor becomes aware of an ethical,
professionalorlegaldilemma.NewdoctorstoAustraliashouldnothesitatetoseekadvicefrom
colleagues.Thereisalsoastrongawarenessonbehalfofpatientsoftheirrighttochangedoctors
ortoseekasecondopinion.Thewillingnessofdoctorstoadmitfallibilityandtoseeksuchhelpis
highlyrespectedbyAustralianpatients.

KerryJBreen
6Communicatingwithpatients.Adviceformedicalpractitioners.NHMRC,Canberra.2004.www.nhmrc.gov.au
5-A
Legal,EthicalandOrganisationalAspects:Ethical
andLegalDilemmas
633
5-AEthicalandLegalDilemmas
CandidateInformationandTasks
MCAT119-124
119 Amanrequestingdisclosureofhiswife'smedicalcondition
120 Obtainingconsentforlegamputationina35-year-oldmanafteramotorvehicleinjury
121 Severalbonefracturesina9-week-oldbaby
122 Aparentrequestingsterilisationofherintellectuallydisableddaughter
123 Bloodtransfusionconsentfora33-year-oldpregnantwomanwithasevereAPHat7
monthsgestation
124 End-of-liferequestfromaterminallyillpatient
119
CandidateInformationandTasks
634

Condition 119
Amanrequestingdisclosureofhiswife'smedicalcondition

CANDIDATEINFORMATIONANDTASKS
Youareadoctorworkinginageneralpractice.Yournextpatient,Bill,aged67yearshasbeen
attendingthepracticeforsometime.Billpresentstoyoutodayforhisannualcheckup.Youhave
completed your examination when Bill tells you he is concerned about his wife's medical
conditionandwantstoaskyouabouthowyoufoundherwhenshesawyouafewdaysago.
Background
YoulastsawBill'swife,Ann,aged65,threedaysago.Youhavebeenlookingafterherformany
years.AnncametoseeyouconcernedthatBillkeepstellingherthatsheisbecomingforgetful
andvague.Youtookahistoryandperformedamentalstateexaminationandageneralphysical
examination,withnosignificantabnormalfindings.Anntoldyouatthattimethatshewasworried
thatBillistellinghersonsanddaughterthathewillneedtoputherinahostelornursinghome,
thoughshefeelsthatsheandBillaremanagingwellathomewithoutanyassistance

YOURTASKISTO:
RespondtoBill'squestionsandrequestsabouthiswife'scondition.
ThePerformanceGuidelinesforCondition119canbefoundonpage641
120
CandidateInformationandTasks
635

Condition 120
Obtainingconsentforlegamputationina30-year-oldmanafteramotor
vehicleinjury
CANDIDATEINFORMATIONANDTASKS
You are working as a doctor in an Emergency Department of a large teaching hospital. A
30-year-old man, Brian, has just been brought in by ambulance following a motor vehicle
accident.Heisconsciousandhasnotsustainedanyheadinjuries.Youhaveexaminedhim,he
hasbeenassessedbyyourregistrarandtheorthopaedicregistrarhasalsoseenhim.Hissole
injuryhasbeentohis leftleg,whichwascrushed under thedashboard. Hehasaverysevere
opencrushinjurytohisleftleg,andimmediatesurgeryhasbeenrecommended.Heisstableat
themomentandhasbeengivenpainrelief.
Youhavebeentoldbytheregistrarthatbelowkneeamputationofthelegislikelytoberequired
astheymaynotbeabletosavethedistallimb.Youhavebeenaskedtoobtainavalidconsent.
Theillustrationbelow,takenintheEmergencyDepartment,showsthestateoftheleg.
He has no significant past medical or surgical history. He has been fit and in good health. He
worksasalabourerandistrainingforselectioninthestaterowingteam.

YOURTASKSARETO:
Obtainavalidconsentfromthepatientforthesurgeryplanned.
Aftersixminutes,answerquestionsfromtheexaminer.
YouareNOTrequiredtoexaminethepatient,ortakeanyadditionalhistoryfromhim.
CONDITION120.FIGURE1
ThePerformanceGuidelinesforCondition120canbefoundonpage644
121
CandidateInformationandTasks
636

Condition 121
Severalbonefracturesina9-week-oldbaby
CANDIDATEINFORMATIONANDTASKS
Youarea HospitalMedicalOfficer(HMO)inahospitalEmergency Department.A30-year-old
motherpresentswithhernine-weekoldbabyboyGregory.Themothersaysthatthebabyhasa
tenderlumpinhisrightthighandisnotkickinghisrightleg.Hecrieswhenhisnappyischanged.
Sherecallsthatherolledoffthechangetablethepreviousday.
YourexaminationfindingsofGregoryare:
Normalpercentiles.
Afebrile.
Anteriorfontanellepressurenormal.
Tenderswellinginthemiddleoftherightthigh,mostlikelyinvolvingthefemur.
Somefingerprintbruisingontheleftupperlimb.
Oldbruisesonchestwallposteriorly.
YouarrangedX-raysofhislegandchest.TheX-raysshowafreshspiralfractureoftheright
femurandthreeposteriorribfracturesapproximatelythreeweeksold.

YOURTASKSARETO:
ExplaintheX-rayresultstothemotherandtakeanyfurtherhistoryyourequirefromher.
Explainthefurthermanagementofthisproblemtoher.
CONDITION121.FIGURE1.
ThePerformanceGuidelinesforCondition121canbefoundonpage647
637
122
Candidate
InformationandTasks


Condition 122
Aparentrequestingsterilisationofherintellectuallydisableddaughter

CANDIDATEINFORMATIONANDTASKS
Youareworkinginageneralpractice.MrsDavisisawidowandhasbeenapatientofyoursfor
approximatelytwelveyears.
Shehasa14-year-olddaughterEvelyn,whohasasignificantintellectualdisability.Evelynhas
alsobeenapatientunderyourcaresincethefamilymovedtothesuburb12yearsago.Evelyn
alsosuffersfromepilepsy,hasanataxicgaitandsignificantbehaviouralproblems.
WhileabletodressandfeedherselfEvelynrequiressignificantassistancewithwashingandis
notcapableofanyformofindependentliving.
Mrs Davis, now 54 years old, is concerned that Evelyn's behavioural problems will be
exacerbatedwiththeonsetofmenstruation.MrsDavisisalsoextremelyanxiousastoherown
abilitytocareforEvelynduringhermenstrualcycleandhastheclearviewthatEvelynwouldbe
intellectually incapable of understanding the physiological changes to her body in addition to
beingphysicallyincapableofmeetingherownhygieneneeds.
AsEvelynhasbeencaredforsolelybyhermother,andrequireshighlevelsofsupportivecare,
Mrs Davis has decided that she will approach you to organise for Evelyn to undergo a
hysterectomyandoophorectomy.
Today Mrs Davis has made an appointment to talk to you about this operative procedure for
Evelyn.

YOURTASKISTO:
Respondtothemother'squestionsandprovideinformationtoherabouttheconsent
requiredpriortotheoperativeproceduressheisseekingforherdaughter.

ThePerformanceGuidelinesforCondition122canbefoundonpage649




















123
CandidateInformationandTasks
638

Condition 123
Blood transfusion consent for a 33-year-old pregnant woman with a severe
APHat7monthsgestation

CANDIDATEINFORMATIONANDTASKS
A33-year-oldwoman,Miriam,hasjustcometoseeyouintheEmergencyDepartmentofamajor
urban hospital with a severe antepartum haemorrhage. She is seven months (30 weeks)
pregnant,andpriortothistime,hasbeenfitandwell.Thisishersecondpregnancyherfirst
babyisaliveandwell.
Onexamination,sheisconsciousandabletospeak.Herappearanceispaleandsweaty,sheis
tachycardic and her blood pressure is 80/45 mmHg. The haemorrhage is continuing. An
emergencyultrasoundsuggestsacentralplacentapraevia.
Youstarttotakeappropriatemeasures,includinginsertionofanintravenouscannula,andtaking
blood for grouping and cross matching. You have begun transfusion with Hartmann balanced
electrolytesolution.Youexplaintoherthatshewillneedanemergencybloodtransfusionaspart
ofhertreatment,andthatthiswillstartassoonaspossiblewhilepreparationsaremadeforan
emergencyCaesareansection.
ShesaysthatsheisaJehovah'sWitnessandwillnotacceptabloodtransfusion.Herhusbandis
alsoaJehovah'sWitness.Heisoverseasatthemomentandcannotbecontacted.

YOURTASKSARETO:
Ascertainfullythepatient'sviewsabouthertreatmentbybloodtransfusion.
ExplaintherisksandbenefitsofthesuggestedtreatmentforbothMiriamandherbaby.
Aftersixminutes,answertheexaminer'squestions.ThePerformance
GuidelinesforCondition123canbefoundonpage652
124
CandidateInformationandTasks
639



Condition 124
End-of-liferequestfromaterminallyillpatient
CANDIDATEINFORMATIONANDTASKS
You are working in a general practice. Sally, aged 65 years, has been your patient for
approximatelytenyears.
OneyearagoSallywasdiagnosedwithpancreaticcancer.Sincethattimeyouhavecontinuedto
beinvolvedinhertreatmentandhavemaintainedaclosedoctor-patientrelationshipwithherand
herfamily.
Despiteactivemedicalintervention,Sallyisnowattheendstageofherdiseaseandhaschosen
towithdrawfromfurthertreatmentandremainathometodie.Youhaveorganisedtheoncology
andcommunitynursestovisitdaily.
Sally has intravenous therapy, an indwelling catheter and a nasogastric tube i n si tu. You are
maintainingregularcontactwithSally,herhusband,andtheirtwoadultchildren.
SoonaftertheinitialdiagnosisSallyapproachedyoutoaskifyouwouldassisthertobringanend
to her life. However, at the time you were evasive and attempted to counsel her in taking a
positiveapproachtohertreatmentandcondition.
Today you are making a house call to Sally to assess her general condition and revise her
medicationregime
YOURTASKSARETO:

RespondtoSally'squestions.
Aftersixminutes,answerquestionsfromtheexaminer
ThePerformanceGuidelinesforCondition124canbefoundonpage655
Legal,EthicalandOrganisationalAspects:Ethicaland
LegalDilemmas
640

5-AEthicalandLegalDilemmas
PerformanceGuidelines
MCAT119-124
119 Amanrequestingdisclosureofhiswife'smedicalcondition
120 Obtainingconsentforlegamputationina35-year-oldmanafteramotorvehicleinjury
121 Severalbonefracturesina9-week-oldbaby
122 Aparentrequestingsterilisationofherintellectuallydisableddaughter
123 Bloodtransfusionconsentfora33-year-oldpregnantwomanwithasevereAPHat7
monthsgestation
124 End-of-liferequestfromaterminallyillpatient
119
Performance Guidelines
641
Condition 119
Amanrequestingdisclosureofhiswife'smedicalcondition
AIMSOFSTATION
To assess the candidate's ability to recognise patient rights to confidentiality and to reconcile
theserightswiththeconcernsofthepatient'sspouseabouthercapacitiesandcompetence.

EXAMINERINSTRUCTIONS
Theexaminerwillhaveinstructedthepatientasfollows:
YouareBillaged67yearsandhavebeenapatientofthisgeneralpracticeformanyyears.You
thinkthatyour65-year-oldwife,Ann,whoisalsoapatientofthispractice,isgettingconfused.
Youhavebeennagginghertogetacheckupandshecametoseethedoctor(thecandidate)
earlier this week. You are worried about the future and think you should list her for a nursing
homeplacement.
Annconsidersherselfneitherforgetfulnorconfusedandwillnotdiscusswithyouwhathappened
atherappointmentwiththedoctor.
Background(disclosetocandidateondirectquestioning):
Youhavebeenmarriedforalongtime.
YourrelationshipwithAnnisstablebuttherehavebeentimesofconflictthroughoutyour
marriage.
Youarefindingthatitisbecomingincreasinglydifficulttotalktoherandhavefoundthatlately
youhavebeenveryirritatedwitheachother.
Allofyourchildrenareadultsandnonelivesathome.
Youliveinyourownhomeanddonothaveanyfinancialproblems.
Starttheinterviewbysaying:
Doctor, I' m really worried about Ann. She' s been behaving differently lately She' s
beenveryforgetfulandseemsconfused.Shecametoseeyouthisweek.Whatdidyou
thinkother?'
Ifthedoctorsayshecannottellyouwhattheydiscussedorhisfindings,orsuggeststhatyouask
Annforanyinformationabouttheconsultationask:
' Whynot?I'mherhusband,surelyIshouldknow?or' Shewon' ttellmeanything'
Ifthecandidatestilldoesnotprovidetheinformation,say:
' Why not? We live together. I' m worried she could leave something on and cause a
fire.Whatifsheforgetssomethingwhenshe' slookingafterthegrandchildren'
Inresponsetospecificquestionsfromthedoctor:
Annhasnothadanyfalls,beenlost,forgottenwhereshewasorwhatshewasdoing.
Whilethegrandchildrenhavebeenwithyounoincidentshaveoccurred,butaddbut
somethingcouldhappen' .
119
PerformanceGuidelines
642

Intheeventofalternativeresponsesfromthedoctor:
The candidate should not divulge the nature of the consultation with Ann. However, if the
candidatedoesandsaysthatnothingabnormalwasfound,thenask:
' The children and I have discussed it and I' m sure they think we should list her for a
nursing home placement.Weneed areferralfromyoutogether assessed.Canyou
writeoneplease?'
Ifthecandidatedoesnotagreetowriteareferral,ask:
' Wellifwecan' tgetitfromyou,wherecanIgetonefrom?'
EXPECTATIONSOFCANDIDATEPERFORMANCE
Thisscenarioillustratestheethicalproblemsofmaintenanceofpatientconfidentialityintheface
ofthehusband'sconcerns.
The candidate should demonstrate recognition of the rights of the patient (Ann) to have her
medicalinformationkeptconfidential.Thecandidateshouldalsoprovidesomeexplanationasto
thebasisuponwhichthereisaconflictinthatbothAnnandBillarepatientsofthepractice.Both
arethuspatientsofthecandidateanddisclosureanddiscussionsoftheirindividualconditions
cannotbegiventotheotherwithoutmutualconsent,suchasbyajointattendanceforamedical
consultationwithoutcoercion.
Thecandidateshouldbeabletoidentifyboththelegalandethicalpositionintermsofaccessing
information in circumstances where the husband has serious concerns about the wife's
competency/capacity.

KEYISSUES
Abilitytomaintainpatientconfidentiality.
Abilitytocounselthehusbandwherethereisaconflictofinterest.
CRITICALERROR
DisclosinginformationaboutAnnwithoutherconsent.
COMMENTARY
Whatisconfidentiality?
Thedutyofconfidentialityrequiresthatdoctorskeepconfidentialallinformationtheyaregivenby
patients and/or which they discover or learn about patients through their professional
interactions.Veryoccasionally,andclearlynotapplicabletothisinstance,confidentialitymustbe
breachedbymandatedreportingforexample,incasesofchildabuse.Patientscannormally
expectthat:
allinformationpatientsdisclosetothedoctorwillnotbepassedontoathirdpartywithoutthe
expressconsentofthepatient;and
thedoctorwilltakereasonablestepstoprotectpatients'information.
119
PerformanceGuidelines
643

Whyisconfidentialityimportant?
Confidentiality offers benefits for individual patients, both in terms of their likely usage of
healthservicesandthequalityofcaretheymayreceive.Forexample,lackofassuranceabout
confidentialitymaypreventpeoplefromseekinghealthcare(youngwomenwithcontraceptive
needs,peoplewithpotentiallystigmatisingillnessessuchasHIV). Thiscanlimitmedicalcare,if
doctorsreceiveonlypartialaccountsofpatients'histories.
Confidentiality maintains trust in the doctor-patient relationship. Trust is a foundation value,
necessary for full and effective communication, sharing information between patient and
doctorandseekingappropriatehealthcare.
Maintainingconfidentialitymaintainscommunitytrustinhealthcare,withoutwhichthewhole
systemwouldbeinjeopardy.
Confidentiality expresses respect for patients' autonomy. Respecting patient autonomy
acknowledges that patients have aspects of their lives that they should be able to keep
confidentialiftheychoose.
Confidentiality is part of the inherent obligation of becoming a medical practitioner. When a
personbecomesadoctor,thecommunityexpectstobepromisedthatpatientinformationwill
remainconfidential.
Inthissituation,maintainingAnn'sconfidentialityrequiresgoodcommunicationskillstoexplain
toBillwhyAnn'smedicalissuescannotbediscussedwithhimwithoutherconsent.Thisrequires
explaining the concept of confidentiality in terms that Bill can understand, and being able to
defusemattersandrespondappropriatelyifBillbecomesangry.
120
PerformanceGuidelines
644

Condition 120
Obtaining consent for leg amputation in a 30-year-old man after a motor
vehicleinjury
EXAMINERINSTRUCTIONS
Theexaminerwillhaveinstructedthepatientasfollows:
You are a 30-year-old labourer, Brian, who has been fit and well. You are training for selection in
the state rowing team. You have just been involved in a motor vehicle accident and have a major
crush injury to your left leg. Surgery has been recommended and the candidate has been asked
to obtain your consent for surgery. The registrar has told the candidate that your leg may have to
be amputated. You have been given pain relief and are not experiencing pain at the moment. You
can't feel the leg at the moment. The candidate will not be examining you at all. You are lying on
a hospital couch with your leg covered by a sheet and dressing.
Opening statement
' What ' s goi ng t o happen?'
When the candidate says that surgery has been recommended, ask
Is t hat r eal l y necessar y?'
When the candidate explains the seriousness of the injury and the possibility of amputation, ask
for clarification of any terms you don't understand (including 'amputation') and then say
' You can' t do t hat ! ' Be emphatic that you don't want to lose your leg. Say ' I t doesn' t hur t
that much i t can' t be t hat bad' .
Ask; I ' m not goi ng t o l et you cut of f my l eg, so what ar e you goi ng t o do?'

EXPECTATIONS OF CANDIDATE PERFORMANCE
The candidate should:
Give adequate information in broad terms about the injury and about the required surgery, the
preparation for surgery and the likely outcomes, including what is likely to happen if Brian
refuses amputation at this time. This must include any risks which wouldn't be obvious to the
patient. The doctor's explanation should cover the following points:
~All possible means will be considered during the surgery to try to save the leg.
~The main indications for amputation are irreparable damage to the limb and its
tissues from crushing, tearing or other injuries, and this is the concern with Brian's leg
that the local damage may be too great to salvage.
~Broken bones, or severed blood vessels or nerves or tendons, can be sewn together
and repaired and this will be done if possible in a bid to save the limb. Even if a leg has
been wholly amputated in the accident, it can sometimes be replaced by skilled teams
such as this hospital has, members of which will be involved in Brian's surgical care but
this is only possible if the amputated tissues can be restored to useful life and function by
restoring full blood flow. If the tissues are already crushed and dead they cannot be
brought back to life, and this is the concern with Brian's leg.
120
Performance Guidelines
645

~Abelowthekneeamputation,(whichiswhatislikelytoberequired)cangive,in
combinationwithamodernprosthesis,averyeffectiveandnearfully-functioninglimb.He
mighthaveheardofexampleslikefighterpilotslosinglegsandreturningfullytoflying
duties.Therearemanyotherexamplesamongprominentsportsmenandwomen,suchas
theYoungAustralianoftheYearamputeewhobecameanoutstandingpara-olympian
medallist,
~Ifirretrievablydeadanddyingtissueisnotremovedbyamputation,theendresultisnotjust
auselesslowerlimb.Thereareveryseriousriskstoyourhealthandlife.
- Thedeadtissuescanreleasetoxinsandotherwasteproductsintothecirculation
whichcanseriouslyaffecttheheartandcausesuddencardiacarrest.
- Thedeadtissueswillinevitablybecomeinfected,whichcanbeoverwhelmingand
spreadfromdeadtissuethroughthebloodandtootherbodysystems,called'gas
gangrene',whichisamajorcauseofdeathintheseinjuriesunlessdeadtissueis
promptlyremoved.
~Ifhestillcannotfaceuptothefactthatamputationmayberequired,andwillnotconsentto
this possibility, the surgeon will respect his wishes, but this would be Brian's decision,
althoughmadeagainsttherecommendationsofhistreatingmedicalandsurgicalteam.He
ismakingthisdecisionhavingbeenforewarnedofthepossiblerisksandsequelae.
~Afterexplanationalongtheselines,thecandidateshouldrepeattherequest,' Once again
Bri an,Iaskyoui fyouwi l l gi veconsentforamputati ontobedoneasal astresortandonl y
i fal l el sefai l stosaveyourl eg?'
Clearexplanationofthepossibilityofamputationandrequestforconsentshouldbemadeas
detailedabove,
Ensurethepatientisanadultofsoundmind.
Considerthatthenarcoticmaypotentiallyinterferewithhiscapacitytogiveavalidconsent.
Identify that doctors can only carry out procedures that they have obtained consent to
undertake,inapatientsuchasthiswhoisconsciousandcompetenttomakedecisions.
Identify that if he refuses the procedure the surgeon (or surgical team) cannot override that
refusal.
Consentcannotbeobtainedthroughmisrepresentation,threatorduress.
Aftersixminutestheexaminerwillaskthefollowingquestions:
' You now need to phone your registrar, what are you going to tell him about this
consent?'
' Do you think there is anything about the condition of this patient that causes you
concern in relation to his competency to give a valid consent?' (Consider effect of
narcotic,patient'sunderstandingoftheprocedureandtheterminologyused,givenhehashad
nopreviousexposuretothehealthcaresystem).

KEYISSUES
Abilitytounderstandtheelementsnecessarytoobtainavalidconsent(legalandethical).

CRITICALERROR
Threateningthatthesurgicalteamwilloverridethepatient'sobjectionsandproceedto
amputationifdeemedsurgicallynecessary.
120
PerformanceGuidelines
646

COMMENTARY

Ethicalfoundationsofconsent:
Informed consent is a mechanism for people to control what happens to them, forming an
importantpartoftheethicalobligationofrespectforpatientautonomy.Themainaimofinformed
consent is to ensure that people are acting freely, without coercion or deception. Informed
consent is the process through which people can authorise what happens to them or who
touches them, making medical treatments morally acceptable. The law strongly upholds this
moral position: before you can examine, treat or care for a competent adult person, you must
obtain a legally valid consent. Without consent, touching a patient, taking blood, or doing an
operationispotentialbattery,eveniftheactionisintendedtohelpratherthanharmthepatient.
The conditions for a valid consent try to ensure that any decision the patient has made is
autonomousbyposingthreequestions:

Isthispersoncompetent tomakethedecision?'
' Doesthepersonunderstand thenecessaryinformation?'
Isthepersonmakingthedecisionfreely? '

Competencehastodowiththeabilityofapersontomakedecisionsthatreflecttheirvaluesand
their concern for their own well being. Competence is determined by a person's age and
intellectualormentalcapacitytomakealegallybindingdecision.Whenassessingcompetenceit
is important to make sure that the person understands the nature, purpose and effects of the
proposedtreatment.Inthiscase,useofjargon,presenceofnarcoticanalgesiaandshockmayall
potentiallyaffectBrian'scompetence.Asecondpartofcompetencerelatestothepatient'sability
to comprehend and retain relevant information. Assessing this would require the doctor to ask
Briantoexplain backtothedoctorwhatthe likelyconsequencesareifBriandoesordoesnot
agree to amputation. A final part of competence is assessing whether the patient believes the
information and hasweigheditinthebalance withotherconsiderationswhenmakingthefinal
choice.Again,thisrequirescarefuldiscussionwithBrian,andaseriousattemptonthepartofthe
doctor to understand why Brian is refusing treatment. Allowing Brian to see the degree of
damage suffered by the limb may help his understanding; but ultimately if Brian remains
adamant, then the surgeon would proceed to wound debridement as necessary, short of
amputation,andmonitorprogresscarefullyinthepostoperativeperiod.
The issue of understanding in relation to obtaining a legally valid consent is where patients
require information about the consequences of the proposed treatment, and any alternative
coursesofaction.
Thelegalstandardinrelationtoinformationtobecommunicatedtopatientsisdeterminedbythe
relevantcaselawandlegislationineachofthestatesandterritories.
Fromalegalpointofview,adoctorwhodoesnotgivethepatientenoughinformationis
potentiallyliableinactionsinnegligenceortrespasstotheperson.
Making decisions free of coercion requires the doctor to explain the alternatives and their
consequencesinplainEnglish,tailoringfurtherresponsestoanyquestionsthatthepatientmay
ask,andavoidingpotentiallythreateningbehaviourssuchasimplyingthatthepatientismakinga
stupid decision or that the patient 'deserves' any adverse consequences from not following
medicaladvice.
121
PerformanceGuidelines
647

Condition 121
Severalbonefracturesina9-week-oldbaby
AIMSOFSTATION
Toassessthecandidate'sabilitytorecogniseandmanagenonaccidentalinjury(NAI)inaninfant.
This is a diagnostic and management scenario and will require the candidate to elicit further
historyconcerningpossibleabuseandalsothecurrentstateofthemother.Thesecondpartof
thescenarioinvolveshowthecandidatecopeswiththisdifficultclinicalsituation,andwhetherthe
candidateisawareofthedoctor'sresponsibilitiesundercommonlaw.
EXAMINERINSTRUCTIONS
Theexaminerwillhaveinstructedtheparentasfollows:
Youarea30-year-oldwomanwhosefirstchild,Gregoryis9weeksold.Yourhusbandisajunior
partnerinalargecitylawfirmandworkslonghours.Yourpregnancywasuncomfortablewiththe
deliveryneedingforceps.Gregoryhasneverfedwellandestablishinglactationhasbeendifficult.
Youhavemovedawayfromfamilyandfriendsbecauseofyourhusband'sposition,andhaveleft
yourownsuccessfuljobbecauseofthis.Gregorycriesalotandyouhavebeenverydepressed,
angryandbewilderedbyyoursituation.Youbecomeveryupsetandtearywhenhecries.
Youarefearfulthatyoumayhaveinjuredhimwhenyoubecameangrywithhimyesterdaywhile
changinghisnappyandrationalisethatherolledoffhischangetable.
InitiallyyouattempttocoverupthebackgroundbutwhenconfrontedwiththeX-rayresults,you
becomeveryanxious,agitatedanddistressed,breakdownandcrysayingthat' noonerealises
howdifficultithasbeenandhowawfulIfeel'.
Questionsyoumayaskorstatementsyoumaymake:
Ifeelsoaloneandnobodyhelpsme.Cananyonepleasehelpme?'
' Willtheytakemybabyawayfromme?'
' NobodyknowshowdifficultithasbeenandhowawfulIfeel' (thiscanberepeated
overandover),
EXPECTATIONSOFCANDIDATEPERFORMANCE
Thecandidateshould:
Informthemotherthatthereisafractureofthethighandalsooldfracturesoftheribs.
Showconcernabouthowthevariousinjuriesmighthaveoccurred.
Enquireaboutthepregnancy,birth,puerperiumandthemother'sfamilyandsocialsituation.
Organiseurgentadmissionofthebabytohospitalasaplaceofsafetyimmediatelyfor
investigationastothecauseofthefractures.
InformmotheroftheneedtonotifytherelevantchildwelfaredepartmentintheStateor
Territoryasrequiredbylaw.
Besupportivetoherandindicateunderstandingofhowdifficultshemusthavefoundittrying
tocopewithherbabyandherproblems.Donotbejudgmentalorthreatening.
121
Performance Guidelines
648

Stressthatduringtheadmissionseveralpeoplewillbeworkingwithhertohelpherthrough
herfeelingsofdepressionandinadequacy.Thereisnoplantoremoveherbabyfromherbut
rathertohelpandsupportherthroughoutthedifficultperiodsheishaving.Herneedsaswell
asthoseofherinfantwillbeaddressed.
Herspousewillalsobeinterviewedtoarrangecounsellingforhimaswell.
Stressthatthesemeasuresaredesignedtopreventasimilarepisodefromoccurringagain.
Explain that a variety of services will be available to help now and in the future. At time of
discharge,supportingpeoplewillhavebeenincontactwithherto arrangeongoinghelpfor
her.
KEYISSUES
Empathicattitudetoadistressedmotherwhoisaskingforhelp.
Recognitionthatsuspectedchildabuse(NAI)isrequiredtobenotifiedbylawinmost
Australianstatesorterritories.
Arrangingforthechildtobeinaplaceofsafety(hospital)fortreatment,andarranginghelpfor
themother.
CRITICALERRORS
Failuretodiagnosenonaccidentalinjury.
Failuretoadvisenecessityofhospitaladmission.
COMMENTARY
Nonaccidental injury to children is not rare in the Australian community and presents in many
ways.Itaffectsallsocioeconomicgroupsandmayrangefromrelativelyminorinjuries,throughto
majortraumaandevendeathorsevereneurologicaldamage.
The medical attendant must be alert to the possibility of NAI in any presentation where the
explanationfortheinjurydoesnottallywiththeinjurysustainedorwherethereareunexpectedor
unexplained minor injuries (e.g. bruises). A thorough and careful history must be taken and
accuratelyrecorded.
With possible physical (and sexual) child abuse, the law in all Australian states requires the
medicalpractitionertoreportthesuspectedabusetotherelevantgovernmentdepartmentwho
will investigate the situation and arrange appropriate followup. In large teaching hospitals a
specificChildAbuseteammaybeinvolved.Thechildmustbeplacedinaplaceofsafety.Inthe
caseofinfants withfractures,thisusuallymeans hospital, whilehelpis beingarrangedforthe
family.Oftenfurtherinvestigationsmayberequiredtodeterminetheextentofinjuriesandmay
include skeletal survey and blood investigations. This commonly reveals that abuse has been
occurringoveraperiodofweeksorevenmonths.
Themedicalpractitioner'srolealsoinvolvescontinuingsupportforthefamilywhiletheprocessof
investigationisproceedingandafterdischargefromtheplaceofsafety.Thissituationmayoften
bedifficultandmaybeassociatedwithconsiderableanger,frustration,accusationanddistress,
allofwhichneedtobeaddressed.
649

Condition 122
Aparentrequestingsterilisationofherintellectuallydisableddaughter

AIMSOFSTATION
Toassessthecandidate'sabilitytoappropriatelycounselaparentrequestingthesterilisationof
herintellectuallydisableddaughter.

EXAMINERINSTRUCTIONS
Theexaminerwillhaveinstructedtheparentasfollows:
You are the 54-year-old mother of a 14-year-old girl named Evelyn. Evelyn has a significant
intellectual disability an ataxic gait and suffers from epilepsy. In addition, Evelyn has
progressively demonstrated significant behavioural problems that have included episodes of
uncontrollableviolentoutbursts.
Sincethedeathofyourhusband12yearsagoyouhavebeenthesolecarerofEvelyn.Youhave
nofamilysupportandwhileyouhaveanumberoffriendstheyarenotabletoassistyoutoany
significantextentwithEvelyn'scare.
WhileEvelynisabletofeedanddressherselfsheisnotcapableofwashingherself,beingleft
alone,orindependentlycaringforherself.
YouareincreasinglyconcernedthatEvelyn'sbehaviouralproblemsarebecomingmorefrequent
andofgreaterintensity.Youhaveformedtheviewthattheseepisodeswillbeexacerbatedwith
theonsetofmenstruation.
IndeedyouhaveseriousconcernsthatEvelynisneitherintellectuallycapableofunderstanding
thephysiologicalchangesthatwilloccurtoherbodynorphysicallycapableofundertakingher
hygiene needs. You have, on a number of occasions attempted to talk with Evelyn about
menstruationandsexualitybutshedidnotunderstandwhatyouwereattemptingtodiscusswith
her.
YouhavemadethedecisionthatEvelynrequiresanoperativeprocedurethatwillresultinhernot
commencingmenstruation.Youhavediscussedtheissuewithafamilyfriendanddecidedthat
Evelynshouldundergoahysterectomyandoophorectomy.
You believe that the ovaries need to be removed to prevent her having premenstrual tension
symptoms.Todayyouhavemadeanappointmentwiththegeneralpractitionertoorganisethe
admissionofEvelynintoahospitalsothatshecanhavetheprocedureperformed.
Starttheinterviewbysaying:
' Doctor, as you know Evelyn is reaching the age where she will commence
menstruation.Shewon' tbeabletolookafterherselfandIreallythinkthatthechange
in her hormoneswill make it too difficult for me to care for her. I am already finding
her care difficult. I have attempted to talk to her about her periods and about
pregnancy but she doesn't have aclue what I am saying to her. I have spoken with
afriendandIthinkthebestthingtodoistohavehersterilised.Iwantyoutoarrange
for her to be admitted to a private hospital sothat she can have a hysterectomy and
her ovaries removed. What do I have to do?
122
PerformanceGuidelines
650

Thecandidatemayexpressinabilitytoarrangesuchaprocedure.Ifsosay:
'Whycan' tyou?YouhavebeentreatingEvelynfor12yearsatthispractice.Youknow
that she can' t look after herself. You know that she is never going to be able to look
afterababy!Whycan' tshejustbesterilisedandthenIcanmanageherasIhavebeen
doing all theseyears.Ifshestarts tohaveherperiods I'm sure thatshe will become
moreaggressiveandIwon'tbeabletomanageher'.
Ifthecandidatestillmaintainsinabilitytoassistinorganisingtheprocedure,ask:' Whycan' tyou
have her admitted for the surgery? When Evelyn needed her appendix out you
admittedhertohospital' .
Questionstoaskunlessalreadycovered:
' Whycan'tyouhaveheradmittedtothehospitalandorganisethesurgery?'
' Why can' t I consent to Evelyn having the operation? I am her carer and the only
familyshehas.'
' Whycantshebesterilised?'
' Whycan' tyou,asherdoctor,consenttoherhavingthesurgery?'
' WellhowdoIgetconsenttohavetheprocedureperformedonEvelyn?'
EXPECTATIONSOFCANDIDATEPERFORMANCE
Thecandidateshould:
recognise that Evelyn herself lacks the capacity to give a valid consent to undergo
sterilisation.
identifythatsuchconsentforthetreatmentofEvelynmustcomefromanadultwhoisofsound
mind and therefore deemed through the legislation or the case law (in the relevant
jurisdictions)tohavetheauthoritytogiveavalidconsent.
expressly recognise that Mrs Davis, the mother, does not have the legal authority to give
consenttothesterilisationofEvelyn.
appreciate that sterilisation (except where it is an incidental result of surgery performed to
cureadiseaseorcorrectamalfunction)isoutsidethescopeofaparent'slegalauthorityto
validlyconsenttomedicaltreatmentofthechild.
acknowledgethatEvelyn,thoughshehasanintellectualdisability,hasthelegalandethical
right to be treated medically as would any other patient. That is, a procedure would not be
carriedoutunlessitwasmedicallyindicatedasbeinginthe'bestinterests'ofthepatient.
Thecandidatemustarticulatethatincircumstanceswhereayoungperson,withan intellectual
disability, is incapable of giving a valid consent, a procedure such as sterilisation cannot be
carried out lawfully without the authority of the Family Court, or similar legal body such as a
GuardianshipBoard.
KEYISSUES
Theabilityto:
recognise that medical procedures like sterilisation, are different in nature from those
proceduresforwhichaparenthasthelegalauthoritytoconsentonbehalfoftheirchild.
identifythatparentsdonothavethelegalauthoritytoconsenttosterilisationoftheirchild.
122
PerformanceGuidelines
651
identifythatthedoctordoesnothavethelegalauthoritytoconsenttosterilisationofthechild.
recognisetheneedforavalidconsentbeforetheprocedurecanbeundertaken.
recognisethatonlytheAustralianFamilyCourtorinsomestatestheGuardianshipBoardhas
thejurisdictiontoauthorisethecarryingoutoftheprocedureintheparticularcircumstances.
articulatethe'harm'thatpotentiallyflowsfromthesterilisationofaminorandtherecognitionof
theissuesassociatedwithdiscriminationinrelationtoindividualswithdisabilities.

CRITICALERRORS
ToagreeEvelyn'sparentcangivealegallyvalidconsentforEvelyntoundergosuchsurgery.
Toagreethatthecandidate,asthetreatingmedicalpractitioner,cangivealegallyvalid
consenttotheprocedure.
TofailtorecognisethateventhoughEvelynhasanintellectualdisability,avalidconsentis
necessary.

COMMENTARY
Thecaseraisesthelegalandethicalissuessurroundingconsent.Asageneralpropositionthe
parentsofachildhavethelegalauthoritytoconsenttotreatmentonbehalfoftheirminorchildren
provided they do so in 'the best interests' of the child (Family Law Act [Commonwealth]).
Howevertherearelimitationstothisauthority.IntheAustraliancaseofReMari ontheparentsof
a14-year-oldintellectuallydisabledchildappliedtotheFamilyLawCourtforanorderpermitting
ahysterectomyandoophorectomytobecarriedoutontheirchildoradeclarationthattheycould
lawfullyconsenttothesurgicalprocedure.Theissuebeforethecourtwaswhethertheparentsof
thechildhadthelegalauthoritytoconsenttotheirchild'ssterilisation?Themajoritydecisionof
the High Court of Australia determined that the procedure required the authority of the court,
expresslystatingthattheperformanceofasterilisationprocedurewasa'specialcase'thattook
thedecisionoutsidewhatwouldbeconsideredastheordinaryscopeofaparent'sauthorityto
consent.
Theethicalconsiderationsincluderecognitionoftheprincipleofautonomy andthefactthatin
circumstanceswherepatientcapacityiscompromised,patientsmaynotbeabletoparticipatein
decisionsabouttheirhealthcareasautonomousdecision-makers.Thedegreetowhichtheyare
abletomakeautonomousdecisionswillbedeterminedbyboththeirlevelofincapacity(interms
oftheirabilitytounderstand)andthedegreeofcomplexityorinvasivenessoftheprocedure.The
ethical principle of 'harm' and the right of all patients to the protection of their body integrity is
pivotaltothefactspresentedinthescenario.Patients,regardlessoftheirlevelofintellect,are
entitledtomedicaltreatmentthatisbaseduponsoundandknowledgeableclinicalpractice.All
patientsareentitledtobetreatedequallyandnotbediscriminatedagainstbasedonfactorssuch
astheirlevelofdisability.
123
PerformanceGuidelines
652
Condition 123
Bloodtransfusionconsentfora33-year-oldpregnantwomanwithasevere
APHat7monthsgestation
A I M S OFSTATION
Toassessthecandidate'sabilitytocounselappropriatelyapatientrefusingabloodtransfusion
onreligiousgrounds.
EXAMINERINSTRUCTIONS
Theexaminerwillhaveinstructedthepatientasfollows:
YouareMiriam,a33-year-oldJehovah'sWitness.Youaresevenmonthspregnantandhavinga
life-threateninghaemorrhage.Youbelievethebabyisstillalive,butdonotknowwhetheritwill
survive.
Openingstatement
IdonotwantabloodtransfusionasIamaJehovah' sWitness.'
Thecandidateshouldcounselyouthatabloodtransfusionisnecessaryinordertoavertavery
highlikelihoodofdyingfrombloodlossforbothyouandyourbaby.DONOTagreetothisatany
pointduringtheinterview.Remaindeterminedtorefusethetransfusion.
Questionstoaskunlessalreadycovered:
'Cantyousavemeandthebabywithoutabloodtransfusion?'
' Cantyoudeliverthebabynowwithouttransfusingme?'
Aftersixminutes,theexaminershouldaskthecandidate:
' Summarisethelegalandethicalissuesinthissituation.'
' Whatwillyoudohowwouldyoumanagethissituation?'
EXPECTATIONSOFCANDIDATEPERFORMANCE
The candidate should understand the ethical issues of autonomy and the patient's right as an
adultofsoundmindtoself-determinehertreatment.
However a patient's right to be autonomous is restricted by the potential to damage a third
person.
Ifcandidatesarenotabletocounselthemotherinrelationtothepotentialdamagetotheinfant
andareunabletoobtainconsentfortransfusiontheycouldseekanemergencyorderfromthe
Courttocommencethetransfusion,althoughtheyaremostunlikelytoreceivepermissiontodo
so.
InAustralia,thefetushasno'rights'justifyingtreatmentbeingforcedonthemotheragainsther
wishes,asuntilthebabyisbornitismostunlikelyanyCourtwouldforceanytreatmentonthe
mothertoimprovethechancesoffetalsurvival.
Oncethebabyisborn,thepossibilityoftransfusingthebabychangescompletely.
123
Performance Guidelines
653
The Human Tissue Act (1982) in Victoria allows children to be transfused without parental
consentprovidingthatwithoutatransfusionthechildwaslikelytodie,andprovidingtwomedical
practitionersconcurredinthatopinionbeforetheadministrationofthebloodtransfusion.Similar
Actsinotherstatesallowthesame.
Forothertreatmentsanapplicationcanbemadetoacourttohavethedeliveredbabymadea
'WardofCourt'andsubsequentlytreatedasnecessary,butthisrequirementdoesnotexistfor
bloodtransfusionitself.
Thecandidatethereforeneedstocoverthefollowing:
UrgentdeliverybyCaesareansectionisrequiredasthebestmeasuretosavethemother'slife
(and baby). The baby would not die of blood loss but of hypoxia (lack of oxygen delivery
carriedbyredcells)duetomaternalshock.
Blood transfusion is clearly advisable in a shocked patient with continuing blood loss, to
improvethemother'sandbaby'schangeofsurvival,andwouldnormallybegivenbeforeand
duringtheoperation.
Transfusionofproductsnotderivedfrombloodarenotnearlyasusefulastheydonotcarry
oxygen, although various synthetic blood substitutes (Haemaccel, Macrodex, etc.) can
improvethebloodvolumeandreducetheapparentshock;butclearlyhypoxiabecomesthe
limitingfactorwithcontinuousbloodlossof40%bloodvolumeormore.
Ifbloodtransfusionisabsolutelyrefusedthechanceofmaternaldeathismarkedlyincreased,
althoughthefetusmaysurviveiftheoperationisdoneimmediately.
Themotherhastherighttorefuseabloodtransfusion.ThereisacaseinAustraliawherethe
husband, who was also a Jehovah's Witness, ultimately gave permission for a blood
transfusion to be given to his wife, because she had lapsed into unconsciousness and was
unable to give consent or continue to refuse blood transfusion at the time. A court recently
upheldthe appropriatenessofthehusband'srighttomakethedecisionandfor the hospital
stafftothentransfusethewoman.Thiswouldnotbepossibleinthiscase,asthehusbandis
not contactable, so the previous wishes of the woman would have to be accepted and
followed.

KEYISSUES
Abilitytodealwithstrongreligiousviewsinarespectfulmanner.
Abilitytorecognisetheprioritiesinthisemergencysituationandrespondappropriately
Recognising that urgent Caesarean section is required as the bleeding is continuing and
causing persisting shock and urgent operation is the only way of controlling persisting
bleeding,despitetheriskswhichanoperationwouldentail.

CRITICALERRORS
NotfindingoutMiriam'sdefinitivewishesabouttreatment.
IndicatingtheywouldtransfuseMiriamwithoutobtainingavalidconsent(eitherfromMiriamor
fromherhusbandifMiriamwasnolongerconsciousandcapableofprovidinganyopinion,for
exampleafterMiriamhadlostconsciousness).
123
PerformanceGuidelines
654
COMMENTARY
Thiscaseraisestheissuesofalegallyvalidconsent.Obtainingavalidconsentisaprocessfor
ensuringthatpatientscanhavecontroloverwhathappenstothem.Oneimportantpartofthisis
ensuringthatpatientsarecompetentandhavesufficientinformationsothattheycanunderstand
the implications of their decisions. Another important part of informed consent is that patients
have the right to make their medical decisions free from undue controlling influences,
manipulationorcoercion.
These can be hard to define, as many decisions that patients make are influenced by their
circumstances, or the wishes or feelings of their relatives. For example, elderly patients may
refuseelectivesurgerybecausetheyaretheprincipalcarerfortheirfrailpartneranddonotwish
torelinquishthisrole,howeverbriefly.Inthissituation,whereMiriamisfacedwithaife-or-death
choicewithwellknownreligiousimplications,itisveryimportantthatsheshouldbeofferedthe
opportunity to discuss her decision about treatment in privacy. From an ethical perspective,
Miriam'srighttobeanautonomousdecision-makerisnotactuallyrestrictedbyanyimpacther
decisionwillhaveonherbaby,althoughitwouldberestrictedifthedecisionwasoneinvolving
bloodtransfusiontoherbabyfollowingdeliveryifthiswasnecessarytopreservelife.Australian
lawdoesnotprotecttheunbornchildfromapparentlyinappropriatematernaldecisionsregarding
care,andpractitionersinAustralianeedtobeawareofthis.
124
PerformanceGuidelines
655
Condition 124
End-of-liferequestfromaterminallyillpatient
AIMSOFSTATION
To assess the candidate's ability to understand and communicate the legal and ethical issues
thatarisewherethereisanend-of-liferequestfromaterminallyillpatient.
EXAMINERINSTRUCTIONS
Theexaminerwillhaveinstructedthepatientasfollows:
You are Sally and you have terminal pancreatic cancer. You have been suffering from this
diseaseforapproximatelyoneyear.Youhavehadseveralperiodsofhospitalisationandhave
beentreatedaggressivelyforthelasttwelvemonths.
Duringyourlastperiodofhospitalisationyouweregiventhealternativesofcontinuingwiththe
treatment,whichmayextendyourlifeforafurthersixmonthsorstoppingthetreatmentandbeing
dischargedhome.
Youhavechosentostopthetreatmentandspendyourremainingtimeathomewithyourfamily.
Youhaveanindwellingcatheter,nasogastrictubeandacannulaforintravenoustherapyinsitu.
Youareathomewithyourhusbandandtwoadultchildrenwhoarecaringforyou.Youhavebeen
verypleasedwiththecareyouhavereceivedfromyourgeneralpractitioner(whohasbeenyour
doctorforthepasttenyears).
Now that you are at home the general practitioner has assumed the responsibility for co-
ordinating your medical care. To that end, you are seen daily by the oncology and community
nurseswhocarryoutyourdailyhealthcare.
Your general practitioner has arrived to assess your condition and review your medications.
Openingstatement
' Doctor,weallknowIamdying.Ihavethoughtaboutthisveryseriouslyformonths.I
knowyou' regivingmesomethingforthepainbutIwantyoutoprescribesomethingI
canusetoendmylife' .
Ifthecandidateexpressesinabilitytodothat,say:
I told you right from the start I would not be able to endure this. I have thought it all
throughverycarefullyIhavehadenoughofthis.Idon'twanttoliveanylonger.Iam
nevergoingtogetbetter.Iwantyoutogivemesomethingtobringallthistoanend'.
Ifthecandidatestillexpressesinabilitytoassistinbringingaboutyourdeath,say:
I' manincreasingburdenoneveryone.Myhusbandandchildrenareexhaustedfrom
caringforme.TheyareupsetallthetimeandweallknowthatIamgoingtodie.We
mightaswelljustgetitoverwith.Idon' twantthemtoremembermeasaburdenon
them.Thethoughtofthemhavingtolookaftermealldayandallnightismakingmy
lastdaystooupsettingforallofus' .
Ifthecandidatecontinuestodeclinetoassist,say:
' Well,whycan' tyou?You' resupposedtobemydoctorandIamnotgoingtorecover.
Iamscaredofthepain.Whatareyougoingtodo?Youhavetodosomethingforme'.
124
PerformanceGuidelines
Alternativeresponses
Thecandidatemayaskthepatient,Sally,aboutherlevelofpainandwhetherthemedication
currentlyprescribedisrelievingherpain.Ifthisquestionisasked,Sallyshouldrespond:' The
painisincreasingandIamconstantlynauseated.MythroatisverysoreandIcan't
swallowwithoutalotofdifficulty.'

AFTERsixminutes,theexaminershouldaskthecandidate:
'Whataretheethicaldilemmasandlegalconsequencesofassistingapatientto
bringaboutherowndeath?'
' Whatalternativescanyouofferher?'

EXPECTATIONSOFCANDIDATEPERFORMANCE
Thecandidateshould:
expressly acknowledge the patient's request and clarify what it is the patient is asking the
doctortodo.
recognisethatthepatientisaskingthedoctortoassisthertobringaboutherowndeath.The
candidateshoulddeclinetodoso.
beabletoidentifythattherequestisforanadditionalmedicationintendedtobringabouther
deathandnotbasedonprescribingforpainrelief.
provide an explanation which shows an understanding of the law in relation to murder and
manslaughterwheretherequestfromthepatientinvolvesbringingaboutherowndeath.
demonstrate an understanding of the ethical principles underpinning 'good' and 'harm' in a
clinicalcontext.
becapableofidentifyingthepotentialforethicaldilemmasincircumstanceswhereapatient
exercises the right to be an autonomous decision-maker in relation to the health care
decisionswhichruncountertothelaw.
addresstheethicalconceptsof'good'and'harm'.
identifyoptionsforpalliativecareandpainrelief(forexample,communityandpalliativecare
nursingservices,adequatepainreliefregimens,respitecare).
KEYISSUES
Abilityto
confirmthatmedicalassistanceisnotgivenforthesolepurposeofbringingaboutthedeath
ofapatient.
articulate the legal position in relation to murder/manslaughter where euthanasia is an
issue.
identifytheconflictbetweenthepatient'srighttobeautonomousinmakingherownhealth
caredecisionsandtheethicalobligationsofbeneficence(doinggood)andnon-maleficence
(nottocauseharm).
656
124
PerformanceGuidelines
CRITICALERRORS
AgreeingtoprovideSallywiththedrugssheisrequestingforthesolepurposeofendingher
life.
Failure to address, in a constructive manner, the issue of pain relief. That is, the candidate
must provide information as to how the patient's pain relief will be managed in a manner
responsivetoherneedsasherconditiondeteriorates.
COMMENTARY
657
Classification Example Moral justifications Legal status
Suicide Self-killing by means such as
hanging, drug overdose or carbon
monoxide poisoning. No
involvement of others.
Right of individuals to
self-determination:
may be prudent or
courageous
Illegal:
Criminal Law
Physician-Assisted
Suicide
Provision of means for patient to
kill themselves, such as a
prescription for self-poisoning, or
insertion of an intravenous line for
a patient to inject lethal drugs.
Requires involvement of doctor.
Right of individuals to
self-determination:
assisting patient to achieve
self-determination;
compassion for suffering of
individual
Illegal:
Criminal Law
Passive Euthanasia I:
Refusal of treatment by
competent person
Refusal of antibiotics in advanced
malignant disease, or advance
directive refusing resuscitation.
No direct involvement of others.
Right of individuals to
self-determination;
duty of doctors to respect
wishes of competent
patient
Legal:
Consent and refusal
of treatment
Passive Euthanasia II:
Withdrawing or
withholding life-sustaining
treatment from
incompetent patient
Turning off ventilator in person with
massive stroke, or withholding
nutrition from a severely brain-
damaged patient.
May require involvement of others.
Avoidance of burdensome
or futile treatment;
relief of suffering;
best interests of patient to
cease treatment;
fair use of medical
resources
Legal:
Consent and refusal
of treatment
CONDITION124.TABLE1.
Voluntarydeathandmedicalaidindying:Ethicalandlegalperspectives*
124
PerformanceGuidelines
658
Themoralconflictscanclearlybequitecomplex.Notonlyaretheretheautonomouswishesofa
competentpatienttoconsider,thedoctormayalsofeelthatitisinSally'sbestintereststorelieve
her suffering, even if this includes ending her life. Obviously candidates must comply with the
law,butsomeacknowledgementanddiscussionofthemoraltensionsmaydisplayawarenessof
theseissues.
As stated, the candidate should constructively address legal avenues for relief of suffering,
includingadequateanalgesiaandpossibleuseofantidepressants.Inthecontextofdiscussing
analgesia,somefamiliarityofthe'DoctrineofDoubleEffect'maybeexpected.
TheDoctrineofDoubleEffectisanargumenttojustifymedicalactionswhich,whileintendingto
relievesuffering,mayalsohastendeath.
Theactitselfmustbemorallypermissible.
Theilleffect,whileforeseen,mustbeunintended.
Theilleffectmustnotbedisproportionatetothegoodeffect.
Theilleffectisnotthemeansbywhichthegoodeffectisachieved.
Theclassicexampleisthatofadministeringmorphinetoaterminallyillperson,withtheaimof
relieving suffering but in the knowledge that the accompanying respiratory depression will
hastendeath.Thisfulfilsallofthecriteriaofthedoctrine:
~Theadministrationofmorphineforpainreliefisamorallypermissibleaction.

*AdaptedfromRogersWAandBraunack-MayerAJ,Practical Ethics for General Practice. OxfordUniversityPress,


2004.
Classification Example Moral justifications Legal status
Active voluntary
euthanasia
Doctor administering lethal dose of
drug with aim of causing
immediate death, at patient's
request
Right of individuals to
self-determination:
assisting patient to achieve
self-determination;
compassion for suffering
Illegal:
Criminal Law
Active nonvoluntary
euthanasia (person
usually incompetent)
Doctor administering lethal dose of
drug in absence of any request, for
example actively killing severely
disabled neonate.
Avoidance of burdensome
or futile treatment;
relief of suffering;
best interests of patient to
cease treatment;
fair use of medical
resources
Illegal:
Criminal Law
Doctrine of Double Effect Doctor administering drugs with
aim of relieving suffering, knowing
that side effect may be to hasten
death
Compassion for suffering;
death foreseen but
unintended
Legal:
Criminal Law
(May vary
between states)
CONDITION124.TABLE1.
Voluntarydeathandmedicalaidindying:Ethicalandlegalperspectives*(continued)
1
PerformanceGuidelines
659

~Themorphineisintendedtorelievepainratherthancauserespiratorydepression.
~Thegoodofrelievingpainoutweighsthelossoflifeinapatientwhoisalreadydying.
~Thegoodeffect,ofrelievingpain,isachievedbytheactionofthemorphineratherthanby
thepersondying
The doctrine rules out active euthanasia, as in euthanasia the act (administering a lethal
substance)isimpermissible,deathisanintendedratherthananunintendedconsequence,and
thegoodeffect(reliefofsuffering)isachievedbymeansoftheilleffect(killingthepatient).Both
themedicalprofessionandthelawappearcomfortablewiththeDoctrineofDoubleEffect.
660

661
MCAT Trial Examinations

MCATTrialExaminationsTwo
Papers

PREPARATORYINSTRUCTIONS

RogerJPepperell
I had scarcel y passed my twel fth bi rthday when I entered the i nhospi tabl e regi ons of
exami nati ons, through whi ch for the next seven years I was desti ned to j ourney These
examinationswereagreattri al tome.'
Wi nstonSChurchi l l (1874-1965)My
Earl yLi fe,Col l i ns(1930)
Test1-16(125-140)
(NumbersT1,T2,T3,T4,T5,T6,T7,T8,T9,T10,T11,T12,T13,T14,T15,T16)

Retest1-8(141-148)
(NumbersR1,R2,R3,R4,R5,R6,R7,R8)
MCAT
TrialExaminationsTwoPapers
662
MCATTRIALEXAMINATIONSPREPARATORYINSTRUCTIONS
In sitting for the clinical examination of the Australian Medical Council, a number of basic
principlesshouldbeborninmind.Theseare:
The tasks are indicated to you at the entrance to the station. These must be read
CAREFULLYtoensureyouareawareofexactlywhatyouhavetodo,sothatyouareableto
plan and use the eight minutes of your contact time with the patient and examiner
appropriately,andensureyoucoverallofthetaskslisted.
Ifoneofyourtasksistotakeafocusedhistory,thisisexactlywhatyoushoulddo.Thismeans
that a full, detailed history is not expected, but that you should concentrate on the matters
whicharerelevanttoyourtasks.Inmanystations,youareadvisedoftheapproximatetime
youshouldspendtakingthishistory,beforeyouproceedtotheremainingtasks.
Therealpatientsortherole-playingstandardisedpatientshavebeenadvisedinadvanceof
how they should perform in the consultation, the questions they are likely to be asked, the
repliesexpectedtothosequestions,andthequestionstheyshouldaskforclarificationofa
point. Appropriate questions have been designed to determine the clinical ability of the
candidateinthetaskareasdefined.
Ifoneofyourtasksistoperformafocusedexamination,againthisiswhatyoushoulddo.An
examinationofthecardiovascularsystem,forexample,wouldclearlyincludeassessmentof
thepulserate,bloodpressure,jugularvenouspulse,heart,andlungs.Itwouldalsoinclude
examinationofthehands,fingers,andankles.Youshouldpreferablydescribeyourfindingsto
theexaminerastheexaminationproceeds,orsummarisetheseattheendoftheexamination,
butbewareofrunningoutoftimeinthelattercase.
Ifyouareadvisedtoasktheexaminerfortherelevantandfocusedclinicalfindingsyouwould
lookforonphysicalexaminationtoaiddiagnosis,youneedtoindicatepreciselywhataspects
ofphysicalexaminationyouwishtodo,whatpositiveornegativefindingsyouwishtolookfor,
andasktheexaminerspecificallyforthese.
Ifyouareadvisedof,or wishtoasktheexaminer for, theresultsofprevious investigations
which you would expect to be available at the time of consultation, again you will need to
requestspecificallywhichresultsyourequirefromtheexaminer.Theexaminermayaskyou
whyyourequiretheseresults.Ifthetestshavenotbeendone,youwillbeadvisedofthis.
Officetestswhichyouwouldperformatthetimeofconsultationandwhichwouldcontributeto
thediagnosisshouldbeaskedfor,suchasurinalysis,haemoglobin,ECGfindings,glucometer
readingsandurinarypregnancytesting.
In some stations, you will need to request other appropriate tests to assist you to make a
diagnosis. If this is the case, you may or may not be given the results of these tests it
depends how the scenario has been designed as to whether the results are available and
giventoyou.Usually,resultswillonlybeavailablefortestspreviouslyperformed,
In responding to patients' and relatives' questions, remember to avoid medical terms which
thenonmedicalquestionerwouldnotbeexpectedtounderstand.Avoidmedicaljargon.Ifyou
persistentlyusemedicalterminologyandnotlayterminology,despitethepatientrequesting
clarificationinlayterms,youwillalmostcertainlybemarkedadversely.
Insomescenarios,youneedtorespondbrieflytospecificquestionsfromtheexaminerabout
diagnosisormanagement,ortogiveacasesummary.Beclearandconcise:rememberthe
patientisalsolistening.
Your performance will be assessed by the examiner, who will take into account your
performanceinanumberofmarkingcategories.Currently14suchcategoriesexist,with
663
MCAT
TrialExaminationsTwoPapers

4-5usuallybeingthemaximumforeachcasescenario.Keyissuesareoutlinedmatchingthe
domains assessed. For some stations, there will be predetermined critical errors which, if
committed,wouldusuallyresultinthecandidatebeinggivenaFAILgradeforthatparticular
station.Thestationsaredesignedtoassessbroadclinicalconsultativeabilitiesoverareasof
communication,diagnosisandmanagement,associatedreasoningskills,andethicalaspects
ofmedicalpractice.
Thecurrentmarkingdomainsareasfollowsandaredetailedinfullforeachconditionnearthe
endofthebook:
Approachtopatientandresponsestopatient'squestions
Patientcounselling/education
History-taking
Physicalexaminationchoiceandtechnique
Physicalexaminationaccuracy
Choiceofinvestigations
Interpretationofinvestigationresults
Diagnosis/Differentialdiagnosis
Initialmanagementplan
Clinicalprocedureexplanation
Clinicalprocedureperformance
Clinicalprocedurefamiliaritywithtestequipment
Commentarytoexaminer
Answerstoexaminer'squestions
Eachmarkingdomainisdesignatedasakeyornon-keydomain,andeachmustbemarkedin
oneofthefouroptionsprovided.
VerysatisfactoryPass
SatisfactoryPass
Unsatisfactory Fail
VeryunsatisfactoryFail
AfinalglobalmarkofPASSorFAIListhengivenforeachstation.
In general if the candidate gets one very unsatisfactory grade in a key area, or two
unsatisfactorygradesinkeyareas,thefinalglobalmarkwillbeaFAIL.
Intheactualclinicalexamination,candidateshavefourreststationsinadditiontothe16active
stationswhicharemarked.Eachofthereststationsisjustthat.Noprereadingorwritingcan
bedoneatthesereststations,andcandidatesareadvisedtousethemasarest,andnotthink
aboutwhattheymighthavesaidordoneatapreviousstation,asitisnotpossibletoretrace
stepstothatstationandbere-marked!
Threeofthestationsgenerallycoverproblemsinobstetrics/gynaecology,threeinpaediatrics,
twoinpsychiatry,fiveinmedicineandthreeinsurgery.
Topassthis16-stationexamination,candidatesmustpassatleast12stations,andthismust
includeatleast1ofthe3obstetrics/gynaecologystations,andatleast1ofthe3paediatric
stations. Passing only 10 or 11 stations (including at least 1 obstetric/ gynaecology and 1
paediatricstation)wouldqualifyforthe8-stationretest.Failingallobstetric/gynaecologyorall
paediatric stations or passing 9 or less stations is a clear fail grade in this 16-station
examination.
Bestwishestocandidatesfortheexaminations.
664
MCAT
TrialExaminationsTwoPapers



MCATTrialExamination
CandidateInformationandTasks
MCAT125-140(T1-T16)
125 [T1] Meconiumstainingofliquorinlabourina25-year-oldprimigravida
126 [T2] Aheartmurmurina5-year-oldgirl
127 [T3] Vigorousvomitingbya3-week-oldboy
128 [T4] Urinaryincontinenceina50-year-oldwoman
129 [T5] Migraineina30-year-oldwoman
130 [T6] Pasthistoryofhipdislocationina35-year-oldman
131 [T7] Tirednessina45-year-oldman
132 [T8] Reviewoflungfunctiontestsina65-year-oldmanwithshortnessofbreath
133 [T9] Assessmentofa28-year-oldprimigravidaat34weekswithfundusless
thandates
134 [T10] Deliriumina25-year-oldmanafteraburninjury
135 [T11] Chronicdiarrhoeaina45-year-oldman
136 [T12] Fever,irritabilityandeardischargeina2-year-oldboy
137 [T13] Reviewofcytologyafteraspirationofabreastlesionina28-year-oldwoman
138 [T14] Nocturnalhanddiscomfortina35-year-oldschoolteacher
139 [T15] Anattackofasthmaina25-year-oldman
140 [T16] Preparinga30-year-oldwomanwithsuspectedacuteappendicitisfor
surgery
125 (T1)
CandidateInformationandTasks
665
Condition 125 (T1)
Meconiumstainingofliquorinlabourina25-year-oldprimigravida

CANDIDATEINFORMATIONANDTASKS
Yourpatientisa25-year-oldprimigravidawhoisinearlylabourat41weeksofgestationSheisin
thelocaldistricthospitalwhereyouareattendingasageneralpractitioner.Thehospitalhasgood
facilitiesbutaconsultantobstetricianisnotavailable.Pelvicexamination30minutesagoshowed
thecervixwas3cmdilated,welleffaced,andwellappliedtothepresentingpart.Thecephalic
presentationwaspositionleftoccipitotransverse(LOT),atzerostation,withnocaputormoulding
evident.Themembraneswerestillintactandallowedtoremainso.Spontaneousruptureofthe
membranesthenoccurredandrevealedprofuse,thickmeconium-stainedliquor.Thepregnancy
hadbeenuneventfultodate,andbloodpressureandurinetestinghavebeennormalinlabour.
Thefetalheartrate,asdefinedusingauscultation,hasbeenbetween130and140/min.

YOURTASKSARETO:
Takeanyfurtherrelevanthistoryyourequire.
Asktheexamineraboutrelevantfindingslikelytobeevidentongeneralandobstetric
examination.
Advisethepatientofthediagnosisandsubsequentmanagementduringandafterdelivery.
ThePerformanceGuidelinesforCondition125(T1)canbefoundonpage680
126 (T2)
CandidateInformationandTasks
666
Condition 126 (T2)
Aheartmurmurina5-year-oldgirl
CANDIDATEINFORMATIONANDTASKS
Youhavejustseenahealthy5-year-oldgirlinageneralpractice.Herfamilyrecentlymovedto
your area, and her previous doctor said that she had a heart murmur and should be reviewed
from time to time. She is an active child, who has had occasional sore throats (about once a
year).Sheishavingadentalcheckinoneweek'stime.Herparentsand3-year-oldbrotherarein
goodhealthandthereisnofamilyhistoryofheartdisease.

Examinationfindings
Active, well-grown child. Vital signs normal, blood pressure 100/65 mmHg (upper and lower
limbs). Pulse normal volume and rate. Apex beat easily palpable in 5th left intercostal space
(LICS) in midclavicular line. Systolic thrill palpable at lower left sternal border. Heart sounds
normal.Secondsoundatpulmonaryareanormallysplit.Long,grade4/6,harshsystolicmurmur
heard, maximal at lower left sternal border, widely conducted over the whole precordium.
Examinationotherwisenormal.

YOURTASKISTO:

Explainthelikelydiagnosisandyourmanagementplantotheparent.

ThePerformanceGuidelinesforCondition126(T2)canbefoundonpage683
127 (T3)
CandidateInformationandTasks
667
Condition 127 (T3)
Vigorousvomitingbya3-week-oldboy

CANDIDATEINFORMATIONANDTASKS
Youhavejustseena3-week-oldbabyboyinametropolitangeneralpracticeofficeconsultation.
Heisthefirstbornsonofhealthyparents,andhasbeenbreastfeedingvigorously.Twodaysago
hevomitedforthefirsttimeandyesterday,hisparentsaid,' thevomi ti ngwenteverywhere....I' ve
neverseenababyvomi tl i kethat' .Thevomitinghascontinuedsothatheisnowvomitingsoon
aftereveryfeed,yetheseemshungryallthetime.

Examinationfindingsyouhaveelicitedare:
Well-nourishedbaby.
Temperature37C.
Pulserate 140/min.
Respirationrate48/min.
Upperabdomendistendedduringfeeding.
Gastricperistalticwavesareseenpassingfromlefttorightacrosstheupperabdomen.
Nomasspalpable.
Nosignsofdehydration.
Nootherabnormality.
Large,forcefulvomitofmilkimmediatelyafterthefeed,

YOURTASKISTO:
Explainyoursuspecteddiagnosisandyourmanagementplanfortheparent.
ThePerformanceGuidelinesforCondition127(T3)canbefoundonpage685
128 (T4)-129 (T5)
CandidateInformationandTasks
668
Condition 128 (T4)
Urinaryincontinenceina50-year-oldwoman

CANDIDATEINFORMATIONANDTASKS

Your next patient is a 50-year-old woman with three children aged 29, 25, and 22 years. She
comes to see you in a general practice setting because of a problem of urinary incontinence
necessitatinghertowearapadallthetime.

YOURTASKSARETO:
Takeanyfurtherrelevanthistoryyourequire.
Asktheexamineraboutrelevantfindingslikelytobeevidentongeneraland
gynaecologicalexamination.
Advisethepatientofthediagnosisandsubsequentmanagementincludingany
investigationsyouwouldadvise.

ThePerformanceGuidelinesforCondition128(T4)canbefoundonpage688

Condition 129 (T5)
Migraineina30-year-oldwoman
CANDIDATEINFORMATIONANDTASKS
Thispatientwasseenaweekago.Atthattime,shehadasevereheadache,andthefollowing
historywasobtained.Sheisamarriedschoolteacheraged30yearswithtwochildren.Thiswas
the second episode of headache in the last month. The headache was felt over both
temporofrontalregionsandwaspresentonwakingup,becomingthrobbingasitprogressed.It
lasted the whole day and prevented the patient from going to work. It was associated with
vomitingandphotophobiaandavisualaura.Panadol(paracetamol)gaveonlylimitedrelief.A
physical examination at that time was normal. A provisional diagnosis of migraine was made,
howeveracomputedtomography(CT)ofthebrainwasarrangedbecauseofthepatient'sanxiety
aboutthepossibilityofacerebraltumour.Thiswasnormal.Youarenowseeingthepatientfor
followupinageneralpracticesetting.Thepatienthasindicatedtoyouthatnofurtherheadaches
haveoccurredsincethelastappointmentoneweekago.

YOURTASKSARETO:
TellthepatienttheresultofthebrainCTscan.
Discussthefuturemanagementofhermigraine.
Thepatientwillaskyouquestionsaboutthetreatmentwhichyouadvise.Theexaminer
mayrequestclarificationofthisinformation
Thereisnoneedforyoutotakeanyadditionalhistoryfromthepatient.ThePerformance
GuidelinesforCondition129(T5)canbefoundonpage691
130 (T6)
CandidateInformationandTasks
Condition 130 (T6)
Pasthistoryofhipdislocationina35-year-oldman

CANDIDATEINFORMATIONANDTASKS
You are working in a hospital followup outpatient clinic. This 35-year-old patient sustained a
posteriordislocationoftherighthipinamotorvehicleaccidentfiveyearsagoillustratedinthe
X-ray below, taken at the time. There were no associated injuries, the dislocation wasreduced
and he had a period of bed rest with traction followed by graduated ambulation and weight
bearing. He has been well since and has noted no problems apart from occasional aching on
prolongedexercise.Hepresentstoyouforacheckupforinsurancepurposes.

YOURTASKSARETO:
Demonstrateyourexaminationofthehipstocheckwhetherhipfunctionisnormal.
Givearunningcommentarytotheexaminerasyouproceed,orattheendofyour
examination.
Advisethepatientofyouropinionabouttheconditionofhiship.
Advisethepatientofanyfurthertestswhicharerequired
Nofurtherhistoryisrequired.
CONDITION130(T6).FIGURE1.
Previousfilmofposteriordislocationrighthip
5yearsagobeforereduction
ThePerformanceGuidelinesforCondition130(T6)canbefoundonpage694
669
131 (T7)
CandidateInformationandTasks
670
Condition 131 (T7)
Tirednessina45-year-oldman
CANDIDATEINFORMATIONANDTASKS
The45-year-oldpatientwhomyouareabouttoseeisconsultingyouinageneralpracticewitha
presenting complaint of being tired. You have not seen this patient before. The patient was
advisedtoseeadoctorbytheworkplaceHealthandSafetyOfficer.Asyougreetthepatient,you
notice that he is middle aged, neatly dressed in a professional mode, looks worried, smells of
alcohol,hasafinetremoronhandshakeandhasnicotinestainingoffingersofhisrighthand.

YOURTASKSARETO:
Takeanappropriatelydetailedandfocusedhistory.
Aftersixminutes,theexaminerwillinformyouofthephysicalexaminationfindings.
Informtheexaminerofyourprovisionaldiagnosisandwhatfurthertestsyouwillorderto
aidconfirmation.

ThePerformanceGuidelinesforCondition131(T7)canbefoundonpage696
132 (T8)
CandidateInformationandTasks
Condition 132 (T8)
Review of lung function tests in a 65-year-old man with
shortnessofbreath

CANDIDATEINFORMATIONANDTASKS
Yourpatient,JosephThomas,isa65-year-oldformercourierdriver,whowasaheavysmoker
untiltwoyearsago(30perdayfor40years).Hestoppedworkingtwoyearsago.Forthreeyears,
he has had a cough productive of small amounts of clear sputum only, and associated with
increasingexertionaldyspnoea.Dyspnoeaisnowevidentwalking100metresontheflat.When
examined in the hospital primary care clinic two days ago, he was overweight (BMI 35 kg/m
2
,
waist circumference 115 cm) and had an audible wheeze. The chest was hyperinflated,
auscultation revealing reduced breath sounds and occasional crackles as well as wheezes
bilaterally.Therewerenosignsofrightheartfailure.Youarrangedpulmonaryfunctiontestsand
thepatienthasnowreturnedfordiscussionoftheresults.Theseareseenbelow(Table1).
YOURTASKSARETO:
Interprettheresultsofthelungfunctiontests.
Explainbrieflytothepatientyourdiagnosisandplanofmanagement.
Aftersixminutes,youwillbeaskedbytheexaminertodemonstratemethodsofusingthe
pharmacologicalagentsprescribed.

For the purposes of this task there will be no need for you to request any additional history or
examinationinformationrelevanttothispatient.
671
Name: THOMAS,Joseph Age: 65years

Weight: 108.5kg Height: 176cm

BMI: 35

PRE POST NORMALRANGE

BR0NCH0DILAT0R

SPIROMETRY

Forcedvitalcapacity(FVC;L) 4.4 4.6 (3.5-5.3)
Forcedexpiratoryvolume(FEV^L) 1.8* 2.0*Low (2.5-4.0)
FE^/FVC(%) 41" 43*Low (65-84)
LUNG VOLUMES

Totallungcapacity(TLC;L) 7.4 7.8*High (5.4-7.4)
Residualvolume(RV;L) 2.9 3.4*High (1.6-3.2)
RWTLC(%) 39 41 (26-52)
DIFFUSING CAPACITY


Diffusingcapacityforcarbon
monoxide(DLC0)(mDmmHg/min)
14* 15*Low (17-33)
CONDITION132(T8).TABLE1.
PulmonaryFunctionTests
ThePerformanceGuidelinesforCondition132(T8)canbefoundonpage700
133 (T9)-134 (T10)
CandidateInformationandTasks
672
Condition 133 (T9)
Assessmentofa28-year-oldprimigravidaat34weekswithfunduslessthan
dates

CANDIDATEINFORMATIONANDTASKS
Yournextpatientisa28-year-oldprimigravidawhoworksasanurseinarenaltransplantunit.
You have been looking after her pregnancy since the first trimester. You are seeing her in a
general practice setting in a shared care arrangement with the local obstetric hospital. All
appeared to be normal up to and including her last visit at 30 weeks of gestation, when the
symphysis-fundalheightwas28cm.Today,fourweekslater,thesymphysis-fundalheightis29
cmandthereappearstobeareducedamountofliquorpresent.

YOURTASKSARETO:

Take any further relevant history you require. Ask the examiner about relevant findings
likely to be evident on general and obstetric examination and available investigation
results.
Advise the patient of the diagnosis and subsequent management including any further
investigationsyouwouldarrange.

ThePerformanceGuidelinesforCondition133(T9)canbefoundonpage705

Condition 134 (T10)
Deliriumina25-year-oldmanafteraburninjury

CANDIDATEINFORMATIONANDTASKS
Youareanightinterninageneralhospital.Thepatientyouareabouttoseeisan25-year-old
malestudentwhowasadmitted24hoursagowith20%partialthicknessburnssustainedwhen
throwing fuel over a camp fire. The burns, involving all the limbs, are being managed
conservatively and have been dressed under intravenous ketamine neuroleptanalgesia. You
havebeencalledbecausethepatientisunabletosleep,restlessanddistressedandhaspulled
outtheintravenouslinedeliveringpatient-controlledanalgesia(morphine1mg/hour).

YOURTASKSARETO:

Determinethecauseofthesleepingproblembytakingarelevantfocusedhistory,and
performinganappropriatepsychiatric(mentalstate)assessment.
Explaintothepatientthenatureoftheproblemandwhatcanbedonetohelp.
ThePerformanceGuidelinesforCondition134(T10)canbefoundonpage708
135 (Til)
CandidateInformationandTasks
673
Condition 135 (T11)
Chronicdiarrhoeaina45-year-oldman
CANDIDATEINFORMATIONANDTASKS
Youareworkingasaninterninthegastroenterologydepartmentofahospital.Youareaboutto
see a 45-year-old insurance agent who is presenting again because of chronic diarrhoea. He
attendedtheclinictwomonthsagowithahistoryofchronicdiarrhoea.Atthatstagehisrecords
indicatethattherewerenoabnormalitiesonphysicalexamination,andexaminationofhisstools
for giardia, Clostridia and other infestations were negative. Colonoscopy was done and was
normal to the caecum. He was diagnosed as having irritable bowel syndrome and prescribed
loperamide(Imodium)andreferredbacktohislocaldoctor.
Briefpatientprofile
Marriedwithtwochildren
Nonsmoker
Moderatedrinker(twostandarddrinkstwoorthreetimesaweek)
Nopersonalfamilyorfinancialproblems.
Pasthistory
Noseriousillnesses.
Familyhistory
Nilsignificant
Noboweldisorders.

YOURTASKSARETO:

Takeafurtherfocusedhistory.
Asktheexaminerfortherelevantsignsyouwouldlookforonrepeatphysicalexamination.
Discussyourfurtherplansinregardtoinvestigationswiththepatient.
Explainthelikelydiagnosistothepatient.

ThePerformanceGuidelinesforCondition135(T11)canbefoundonpage712
136 (T12)
CandidateInformationandTasks
Condition 136 (T12)
Fever,irritabilityandeardischargeina2-year-oldboy

CANDIDATEINFORMATIONANDTASKS
Alexander is a 2-year-old boy who presents to the general practice officewith an acute febrile
illness.Hehashadarunnynoseforafewdaysandhasbecomefebrileandirritableoverthepast
24hours.Hedidnotsleepwellthepreviousnightandhasvomitedtwicetoday.Shortlybefore
presentation his mother noticed a discharge from his right ear. There is no past history of
significantillness.
Examinationfindings
Onexaminationheisfebrile(temperature38.8C)andflushed.Thereisclearrhinorrhoea.
Therightearcanalisfullofpus.Thelefteardrumisintenselyredandbulgingasillustrated.He
hasnoneckstiffness.
Therearenootherabnormalphysicalsigns.
YOURTASKSARETO:
Explainthelikelydiagnosistotheparent.
Explaintotheparentthemanagementyouwouldrecommend.
You have obtained all the relevant findings on history and examination.
FurtherhistorytakingorexaminationisNOTrequired.
CONDITION136(T12).FIGURE1.
Bulgingredleftdrumonotoscopy
ThePerformanceGuidelinesforCondition136(T12)canbefoundonpage716
674
137 (T13)
CandidateInformationandTasks
Condition 137 (T13)
Reviewofcytologyafteraspirationofbreastlesionina
28-year-oldwoman
CANDIDATEINFORMATIONANDTASKS
Thepatientyou areseeinginthegeneralpracticeoffice isasinglefemalesecretary,aged28
years.Shepresentedtoyoutwoweeksagowithahistoryofpainfulbreasts,cyclicalinnature.
Shehashadnopregnancies,isonnomedicationsapartfromtheoralcontraceptivepill,andhas
hadnopreviousbreastproblems.Hergeneralhealthhasbeennormal.Youfoundnoabnormality
onexamination,exceptforsomemildgeneralisedbreasttenderness,butinviewofherconcerns,
youarrangedabreastultrasoundwhichshowedasolitaryabnormalitydeepintherightbreast.
An ultrasound guided fine-needle aspirate was done, and sent for cytologic assessment (fine
needleaspirationcytologyFNAC).
Theultrasoundandthecytologyreportareasillustratedbelow(Figure1,Table1).Sheis
now returning to discuss the results of the ultrasound and FNAC with you, and for counselling
aboutherfutureconcerns.
Hermotherwasrecentlydiagnosedwithbreastcanceratage50yearsandwastreatedwithlocal
surgery followed by radiation therapy. Because of her mother's problems, the patient was
concernedthatshealsomayhavebreastcancer.
YOURTASKSARETO:
Examinetheultrasoundandthecytologyreport,andadvisethepatientofthediagnosis.
Advisethepatientaboutfuturemanagement,andaddressherconcerns.
CONDITION137(T13).FIGURE1.
Ultrasoundoverareaofconcerninrightbreast
675
137 (T13)
CandidateInformationandTasks
676
ThePerformanceGuidelinesforCondition137(T13)canbefoundonpage718
FINENEEDLEASPIRATION
CYTOLOGYREPORT
MS AB aged 25 years
Aspiration Cytology from right breast lesion.
ReportThe cvtoloqv shows scattered groups of
benign-appearing ductal cells and occasional
macrophages.
There is no evidence of malignancy.
CONDITION137(T13).TABLE1.
677
138 (T14J-139 (T15)
Candidate Information and
Tasks



Condition 138 (T14)
Nocturnalhanddiscomfortina35-year-oldschoolteacher
CANDIDATEINFORMATIONANDTASKS
Youareworkinginageneralpractice.Yournextpatientisa35-year-oldschoolteacherwhois
consultingyouaboutdiscomfortinherrightarmandhand,worseatnight.
YOURTASKSARETO:
Takeafocusedhistoryfromthepatient.
Performaregionalexaminationrelevanttothepatient'shistory.
Explainthediagnosistothepatient.
Discusstreatmentwiththepatient.
ThePerformanceGuidelinesforCondition138(T14)canbefoundonpage721

Condition 139 (T15)
Anattackofasthmaina25-year-oldman
CANDIDATEINFORMATIONANDTASKS
Youareseeingthispatientpriortodischargefromhospital.
You are the duty Hospital Medical Officer (HMO) on your morning round. The patient was
admitted to the hospital Intensive Care Unit (ICU) last night because of a sudden very severe
attackofasthma,theworsthe hadeverexperienced.Hehasrespondedpromptlytointensive
treatment with nebulised salbutamol and hydrocortisone intravenously, and subsequent oral
prednisoloneovernight.Hewantstobedischargedtoday.
Youwouldpreferthepatienttoremaininhospitalforanotherday.Thepatient'speakexpiratory
flowreading(PEF)thismorningis450L/min(expectedvalueforage/heightforthispatientis625
L/min). There is minimal expiratory wheezing on auscultation of the chest. He is a 25-year-old
whohasworkedasanautomotivespraypaintersinceheleftschoolattheageof15years.He
hasneversmokedandtakesalcoholmoderatelyatweekendsonly.
Hehashadoccasionalmildasthmaattackssinceage16years.Theywereneversevereuntilthis
attack. His only treatment has been Ventolin (salbutamol) metered dose aerosol used
irregularly,prescribedbythefamilydoctor.Hehashadnootherseriousillnesses
YOURTASKSARETO:
Explainthenatureandsignificanceofhisasthmatothepatient.
Specifywhatmedicationthepatientshouldtakeafterdischargefromhospital.
Discussthefuturemanagementofthepatient'sasthma.

ThePerformanceGuidelinesforCondition139(T15)canbefoundonpage724
678
140 (T16)
CandidateInformationandTasks



Condition 140 (T16)
Preparinga30-year-oldwomanwithsuspectedacuteappendicitisforsurgery

CANDIDATEINFORMATIONANDTASKS
You are the intern on duty in the Emergency Department in a small country hospital. You have just seen this
30-year-oldwomanwhohaspresentedwithasixhourhistoryofacuteworseningabdominalpainwithvomiting.
Thepatientisnormallyingoodhealthandhashadnomajorillnessesinthepast.Shehasregularnormalperiods,
thelastbeingthreeweeksago.Sheisontheoralcontraceptivepill.
Examinationfindings:
Temperature37.9C
Pulse 100/minregular
Bloodpressure125/85mmHg
Tongue coated, breath offensive, tenderness and guarding in the right iliac fossa with localised release
tenderness.
Rectalexaminationfoundtendernesshighupontheright.
Urinenormalonchemicaltesting.
Youstronglysuspectthatthepatientissufferingfromacuteappendicitis.
YOURTASKSARETO:
Advisethepatientofthediagnosis.
Counselherontheneedforoperation.
Explaintheimplicationsoftreatmenttothepatientinobtainingpatientconsentforsurgery.
Answeranyquestionsthatthepatientmighthave.
Thereisnoneedforyoutotakeanyadditionalhistoryorperformanyfurtherphysicalexamination.
ThePerformanceGuidelinesforCondition140(T16)canbefoundonpage728
679
MCAT
TrialExaminationsTwoPapers

MCATTrialExamination
PerformanceGuidelines
MCAT125-140(T1-T16)
125 [T1] Meconiumstainingofliquorinlabourina25-year-oldprimigravida
126 [T2] Aheartmurmurina5-year-oldgirl
127 [T3] Vigorousvomitingbya3-week-oldboy
128 [T4] Urinaryincontinenceina50-year-oldwoman
129 [T5] Migraineina30-year-oldwoman
130 [T6] Pasthistoryofhipdislocationina35-year-oldman
131 [T7] Tirednessina45-year-oldman
132 [T8] Reviewoflungfunctiontestsina65-year-oldmanwithshortnessofbreath
133 [T9]Assessmentofa28-year-oldprimigravidaat34weekswithfundusless
thandates
134 [T10]Deliriumina25-year-oldmanafteraburninjury
135 [T11]Chronicdiarrhoeaina45-year-oldman
136 [T12]Fever,irritabilityandeardischargeina2-year-oldboy
137 [T13]Reviewofcytologyafteraspirationofabreastlesionina28-year-oldwoman
138 [T14]Nocturnalhanddiscomfortina35-year-oldschoolteacher
139 [T15]Anattackofasthmaina25-year-oldman
140 [T16]Preparinga30-year-oldwomanwithsuspectedacuteappendicitisfor
surgery
125 (T1)
PerformanceGuidelines
680
Condition 125 (T1)
Meconiumstainingofliquorinlabourina25-year-oldprimigravida

AIMSOFSTATION
Toassessthecandidate'sabilitytomanageapatientinlabourwhohasthickmeconium-stained
liquorwhenthemembranesrupture.

EXAMINERINSTRUCTIONS
Theexaminerwillhaveinstructedthepatientasfollows:
Thelistofresponsesbelowislikelytocovermostofthequestionsyouwillbeaskedastheyare
likelytoberelevanttothecasepresented.
The baby was due one week ago. The antenatal course of the pregnancy has been quite
normal.Youhadacardiotocogram(CTG)threedaysagobecausethepregnancywasgoing
overdue.Youweretolditwasquitenormal.
Fetalmovementshavebeenfrequentandhavenotdeclined.
Therehasbeennovaginalbleeding.
Contractionsarenowoccurringevery3-4minutesandlastabout45seconds.
Questionstoaskunlessalreadycovered:
' CanIhaveavaginaldeliveryifthebabyisallright?'
IsmybabygoingtobeOK?'
' DoIneedaCaesarean?'
' Whatdoesthegreenfluidmean?'
'HowcanyoucheckifthebabyisOK?'
' Willmybabyneedanyspecialcareafterbirth?'

Examinationfindingstobegiventocandidatebytheexamineronrequest
These examination findings refer to findings done following the spontaneous rupture of the
membraneswhenthickmeconiumstainingbecameevident.Themeconiumwasdarkgreen
andthick.
Allotherfindingswereunchangedfromthose30minutespreviously,i.e.thecervixisstill3cm
dilated,andwelleffaced.Thepositionofthefetalheadisstillleftoccipito-transverse(LOT),at
zerostation,withnoevidenceofcaputormouldingfound,andnoevidenceofcordprolapse.
Bloodpressurewas120/80mmHg.Fetalheartrateandrhythmareunchanged.Investigation
resultsnildoneasyet.
125 (T1)
681
Performance
Guidelines



EXPECTATIONSOFCANDIDATEPERFORMANCE
Thecandidateshouldconveythesubstanceofwhatfollowstothepatient:
Meconiumstainingoftheliquoriscommoninpost-termlabour.Sometimesitmeansthebaby
is having problems, but in most instances the baby is perfectly healthy. It is necessary to
assess the fetal condition using cardiotocography. If cardiotocography is abnormal, further
actionmayberequired.
Thecandidateshouldindicatethatapelvicexaminationisrequiredtoassesstheprogressof
labourandtomakesurethecordhasnotprolapsedandisthecauseofthemeconiumstaining
of the liquor. In the absence of any CTG abnormality, or abnormal PV finding on vaginal
examination,thelabourshouldbeallowedtoprogress.Providingnormalprogressoccurs,and
theCTGremainsnormal,thelabourshouldbeallowedtoproceedsothatvaginaldeliveryis
ultimatelypossible.IfaminorabnormalityintheCTGrecordingoccurs,itispossibletoassess
the significance of this by measuring the fetal scalp pH or lactate level. If this is clearly
abnormal,Caesareansectionorvaginalmanipulativedeliverymaybenecessary.
Atthetimeofvaginaldelivery,adequateaspirationofthemouth,pharynx,andnasalcavityis
essentialinanattempttopreventanyinhalationofmeconiumintotherespiratorytractofthe
baby.Visualisationofthevocalcords,immediatelyafterbirth,withaspirationofmeconiumin
anyadjacentregionisalsorequired.Apaediatricianisdesirableatdelivery,ifoneisavailable,
to ensure that the resuscitation is adequate, and to commence any further therapy which
mightberequiredshouldthebabyinhaleanymeconium.Ifthecandidatefailstoprovidethe
information concerning the need for aspiration of meconium from the mouth and nasal
passages at the time of delivery, the examiner should indicate that the patient is suddenly
readyfordeliveryandask' whatwi !l youneedtodoatdel i very?'
Amnioinfusionisbeingadvocatedinsomecentresinanattempttopreventmeconiumaspiration
syndrome.Thishasstillnotachievedwidespreadacceptance,however,andwouldnotusually
beincludedaspartofclinicalpracticebymostobstetricians.

KEYISSUES
Understandingtherelevanceofthemeconiumstainingoftheliquor.
Understandingthemanagementrequired.

CRITICALERRORS
Failuretodovaginalexaminationtocheckcervicaldilatationandexcludecordprolapse.
FailuretoadequatelymonitorthebabythroughouttherestofthelabourusingcontinuousCTG
assessment.
Failuretoaspiratethemouthandpharynxadequately,atdelivery,toreducetheriskof
meconiumaspiration.
125 (T1)
PerformanceGuidelines
682

COMMENTARY
Thiscaseassessestheunderstandingofthesignificanceofthepresenceofmeconiumstaining
of the liquor in full term (or slightly past-term) pregnancies. It is important to understand that,
although the presence of meconium may be a normal accompaniment of term (or slightly
post-term) pregnancies, the sudden appearance of meconium when the membranes rupture
mandatesthesubsequentnecessityofmonitoringofthebabythroughouttheremainderofthe
labour. Itshould alsoberememberedthatthatbaby'spharynxandmouthmustbe adequately
aspiratedatbirthtopreventmeconiumaspirationsyndrome.
Commonproblemslikelywithcandidateperformanceare:
An apparent lack of understanding that meconium staining of the liquor is common in
post-termlabouranddoesnotimplythebabyisinextremedifficulty.
PelvicexaminationtoexcludecordprolapseandcontinuousCTGmonitoringarerequiredto
determine whether the baby is at risk of hypoxia. Only occasionally is Caesarean section
required.
126 (T2)
683
PerformanceGuidelines



Condition 126 (T2)
Aheartmurmurina5-year-oldgirl

AIMSOFSTATION
Toassessthecandidate'sabilitytodiagnoseventricularseptaldefectinachild,andtoexplain
thetreatmentandprognosisinanempathicwaytoaparent.

EXAMINERINSTRUCTIONS
Theexaminerwillhaveinstructedtheparentasfollows:
Youaretheparentof5-year-oldAlyssa.Youhavejustmovedtotheareaandareattendingthis
general practice forthefirsttime.Yourdaughter'smurmurwasfirstfoundattwoyearsofage.
You were not particularly concerned, as you had previously been told that the murmur would
probablygoaway.You aremuchmoreconcernednowifyouaretoldthatthisis a holeinthe
heart.
OpeningStatement
' Our previous doctor said this murmur might go away, and not to worry about it, but
haveitcheckedeachyear'
Questionstoaskunlessalreadycovered:
'Whatdoyouthinkiscausingthis,doctor?'
AretherespecialprecautionsweshouldtakewithAlyssa?'
' Shouldsheplaysport?'
' Canyouexplaintheseinvestigationsyouarerecommending?'
' Aretheypainful?'
Ihavejustbookedthewholefamilyintoseeadentist.DoIneedtomentionthisto
him?'
EXPECTATIONSOFCANDIDATEPERFORMANCE
Explanationofdiagnosis
Thecandidateshouldexplainthefollowinginlaytermsthattheparentcanunderstand:
Themurmurisduetoaconditionthatrequiresspecialistinvestigation,mostlikelya'holeinthe
heart'(smallventricularseptaldefect).Itsfeaturesshouldbeexplainedbriefly.Theprognosisis
usually very good. In some cases spontaneous closure can occur although this becomes less
likely by the age of five. No restriction of activity is required and normal activities should be
encouraged.Special investigationsareneededtomakethediagnosis-chestX-ray,ECGand
echocardiogram.Thesenoninvasiveproceduresshouldbeexplained.Referraltoanappropriate
specialistpaediatrician/paediatriccardiologistifpossibleisrequired.Regulardentalcareis
important;appropriateantibioticprophylaxiswillberequiredforpotentiallycontaminateddental
workandsurgicalprocedures.Yearlyreviewwithcardiologistmostlikelytoberequireduntilitis
determinedwhetherthelesionwillcloseandhowlargeitis.Thesignswouldsuggestitissmall
andmaynotwarrantoperativeclosure.
126 (T2)
684
PerformanceGuidelines


KEYISSUES
Diagnosisofventricularseptaldefectordiagnosisthatthisisanorganicmurmurprobablya
congenitallesion.
Referraltopaediatricianorpaediatriccardiologist.
Explanationandreassurance.

CRITICALERROR
Candidateregardsthisasaninnocentfunctionalmurmuranddoesnotrecommend
referral.

COMMENTARY
Thiscasefeaturesahealthy5-year-oldchildwhoisbeingreviewedpriortodentalcheckup.She
has a known murmur considered innocent, but a careful cardiological examination points to a
pathological murmur due to a potentially serious heart condition, most likely ventricular septal
defect(VSD).Knowledgeofthekeyclinicalfeaturesofchildhoodcardiacmurmursisexpected.
Explanationoftheindicationsforprophylacticantibioticinthepresenceofaheartmurmurisalso
required.
Heart murmurs are common in children and over 40% of healthy 4-year-olds may have a soft
innocent heart murmur if careful auscultation is carried out. The medical practitioner therefore
mustrecognisethefeaturesthatdistinguishinnocentfromsignificantmurmurs.Thepresenceof
aloudpansystolicmurmurwithathrillindicatesanorganiclesionandwarrantsinvestigation.
Theechocardiogram,anoninvasiveinvestigation,isusuallyabletoclearlydefinethelesionand
no other investigations are necessary. However, most cardiologists also perform an ECG and
chestX-rayaspartoftheworkup.AsmallVSDmaybesosmallastonotwarrantanysurgical
closureasitisnothaemodynamicallysignificantandwillcausenoclinicalproblemtothepatient.
However, the patient and family must be warned and constantly reminded of the need for
appropriateantibioticprophylaxisforanyoperativeordentalprocedure.
Alldoctorscaringforchildrenmustbecognisantofthefeaturesofaninnocentmurmurandthe
importantdifferencesbetweenafunctionalandanorganicmurmur.
127 (T3)
PerformanceGuidelines
685
Condition 127 (T3)
Vigorousvomitingbya3-week-oldboy
AIMSOFSTATION
Toassessthecandidate'sabilitytodiagnoseinfantpyloricstenosis,andtoexplainthediagnosis,
managementandprognosistotheparentinanempathicmanner.
EXAMINERINSTRUCTIONS
This case involves a 3-week-old healthy infant who for two days has developed projectile
vomiting suggestive of congenital hypertrophic pyloric stenosis. The typical physical signs
reinforcethisdiagnosis.Althoughamasscannotbefelt,thismayrelatetoclinicianexperience
andshouldnotnecessarilynegatethediagnosis.
Theexaminerwillhaveinstructedthepatientasfollows:
Youareaveryanxiousparent,convincedthatthebabyhassomeunusualandsevereillness.
Youhaveneverheardofpyloricstenosisifthisismentionedandrequireittobeexplainedtoyou
indetail.
OpeningStatement
I' veneverseenababyvomitlikethatbefore,doctor.Theremustbesomethingreally
wrongwithhim.'
Questionstoaskunlessalreadycovered:
Whydoeshevomitallthetime?'
Issomethingwrongwithmymilk?'
' Willheneedtohaveanytestsdone?'
'Whatwilltheydowhenhegoestohospital?' (Ifhospitalisationsuggested)
' Doesheneedtohaveanoperation?'
'WhenwillIbeabletostartbreastfeedinghimagain?'
' Howlongwillheneedtobeinhospital?' (Ifhospitalisationsuggested)
'Canyoudrawmeapicturetoshowmewhatthisisallabout?'
EXPECTATIONSOFCANDIDATEPERFORMANCE
Explanationofdiagnosis
Thecandidateshould:
Indicatethatpyloricstenosisistheprobablediagnosis.
Explainthemechanismofthecondition,ideallywiththeaidofadiagram.
Explaintheproblemsofobstructiontogastricoutflow.
Explaintheseriousnatureoftheproblemifuntreatedandthatsurgeryisnormallyfully
curative.
127 (T3)
686
PerformanceGuidelines



Immediatemanagement
Explain the need for admission to a hospital experienced with childhood surgical
emergencies.
Indicatethediagnosiswillbeconfirmedafterobservationandtestfeedinginhospital,butmay
requireinvestigationbyultrasoundorbariummeal(eitherisacceptable,thoughsomecentres
have a reservation about a barium meal because of the risk of vomiting and aspiration).
Ultrasoundisonlyasgoodastheoperator.
Explaintheneedforbloodteststolookforelectrolyteandacid-basedisturbance.
Explainthelikelyneedforintravenousfluidtherapybeforesurgery.Thiswilldependonthe
clinicalstateofthebabyandtheresultsofthebloodtests.
Explain the need for surgery (referral to paediatric surgeon for surgery when diagnosis
established)andtheprincipleofpyloromyotomy.
Indicatethattheprognosisisexcellent.
Reassure that a person skilled in giving anaesthetics to young infants will give the
anaesthetic.
Indicatethataftersurgery,babywillbeabletofeedfromthebreastwithinafewhours.Hemay
need to have the intravenous line i n si tu for 24 hours. Generally he should be able to be
dischargedafter24hours.

KEYISSUES
Diagnosisofpyloricstenosis.
Earlyreferraltohospital/paediatricsurgeon
Explanationincleartermsofthecondition.

CRITICALERRORS
Failuretodiagnosepyloricstenosis.
Failuretorefertohospitalorpaediatricsurgeonforfurtherassessment.

COMMENTARY
Vomitingintheneonatalperiodandbeyondiscommon.Thegreatmajorityofbabiespossetafter
feeds (they bring up effortlessly small amounts of milk, occasionally associated with burping).
Thereare however importantpathologicalconditionsthatmustbeconsidered and excluded in
anybabywhohaspersistentvomiting.
The important features in this scenario are the forceful or projectile vomiting that the parents
describe, and that the baby appears hungry all the time. As the history has been for only two
days,thispictureiscompatiblewithpyloricstenosis.Withpyloricstenosis,afterseveraldaysof
vomiting and with progressive dehydration and electrolyte imbalance, the baby will become
lethargicanduninterestedinfeeds.
While other diagnoses may be entertained, for example, oesophageal reflux or urinary tract
infection, the clinical features usually differentiate these conditions. With reflux, although
projectile vomiting can occur, it is not persistent. Urinary infection is often associated with an
unwellinfantandpossiblyfever.
127 (T3)
PerformanceGuidelines
687
Whenpyloricstenosisissuspected,theclinicalexaminationshouldinvolveatestfeedandduring
thefeedtheclinicianlooksforactiveperistalticwavestravellingfromlefttorightacrosstheupper
abdomen. The best time to feel the tumour, is when the wave reaches the right side of the
abdomenanditcommonlywillbefeltimmediatelyafterthebabyhasvomited.
Investigationstoconfirmthediagnosismaybehelpful,butexceptforelectrolyteandacid/base
status,maybeunnecessaryifthetumourcanbeeasilyfeltandtheclassicalhistoryispresent.
These investigations however may include ultrasound that may demonstrate the tumour and
bariumswallowwiththeclassic'string'signofanarrowedpyloriccanal.
Themostimportantinvestigationstoassessthebaby'sgeneralclinicalstateareelectrolytesand
urea/creatinine and acid/base studies. These will usually show significant abnormalities with
hypochloraemicalkalosis.Theseabnormalitiesshouldbecorrectedwithappropriateintravenous
fluidsbeforethebabyundergoescorrectivesurgery.
The effective treatment is Ramstedt pyloromyotomy where the thickened pylorus is split
surgicallydowntothemucosabutnotthroughthemucosa.Thistreatmentresolvestheproblem
veryeffectively.
The parents should be advised that the baby will be able to resume feeding soon after the
surgeryandusuallycanbedischargedfromhospitalwithin24hoursofoperation.Thecondition
isoftenassociatedwithastrongfamilyhistory,whichifpresent,shouldalertthecliniciantothe
possibilityofthisdiagnosis.
128 (T4)
688
PerformanceGuidelines

Condition 128 (T4)
Urinaryincontinenceina50-year-oldwoman
AIMSOFSTATION
To assess the candidate's ability to diagnose and manage the common forms of urinary
incontinencelikelytobeseeninamiddle-agedwoman.

EXAMINERINSTRUCTIONS
Theexaminerwillhaveinstructedthepatientasfollows:
The doctor will question you about further details of your urinary incontinence. The following
responsesshouldcovermostofthequestionsasked.
Listofappropriateanswerstospecificquestionsasked
Menstrualhistoryperiodswereregularuntilfouryearsagowhentheyceasedsuddenly.You
had some hot flushes in the next 12 months but did not take any hormone replacement
therapy(HRT).
Noflushesatpresent.
Noearlyonsetheartdiseaseorosteoporosisinyourfamily.
Nopreviousoperationsorillnessesexceptthreenormaldeliveriesofbabieseachweighing
about3400g.
Notonanymedications.
Urinaryincontinencefeatures.Thiscommencedabouttwoyearsagoandnowyou
wetyourselfeveryday.Thismostcommonlyoccursifyoucoughorlaugh,whenyoulosea
smallamountofurine,butalsooccurswhenyoudoyouraerobicstokeepyourselffit.There
hasbeennoincontinenceatnight,unlessyoucoughorlaugh.Whenyougetthefeelingthat
youneedtoemptyyourbladder,youhavetoruntothetoiletoryouwetyourselfwithalarge
volumeofurine.Youhavehadnourinarytractinfectionsthatyouknowof,butdohavesome
urinaryfrequencyatnight.
Yousmokeabout20cigarettesaday,andweigh87kg(BMI32).Youarenotkeentohave
anyoperationunlessitisabsolutelynecessary.
Questionstoaskunlessalreadycovered:
'Can' tyougivemeanythingtostopthisincontinence?'
' DoIneedanoperation?Ireallydon' twantone.'
' Whyhasthisincontinencestarted?'
128 (T4)
PerformanceGuidelines
689
Examinationfindingstobegivenonrequesttothecandidatebytheexaminer
She is overweight, weighing 87 kg (BMI 32), but looks otherwise well. Full general
examination is normal. Blood pressure 150/90 mmHg. No evidence of any abnormal
massesintheabdomen.
Speculum examination: The vaginal epithelium appears atrophic. There is minimal
prolapseoftheanteriorvaginalwall,butnoprolapseoftheuterusorposteriorvaginalwall.
When she was asked to cough, loss of a small amount of urine from the urethra was
demonstrated.
Pelvic vaginal examination (PVE): uterus is not enlarged, and both adnexae appear
normal.
Investigationresults:nonedone.
EXPECTATIONSOFCANDIDATEPERFORMANCE
Thecandidateshouldconveythesubstanceofthefollowingtothepatient.
Thediagnosisisalmostcertainlythatofmixed'urge'incontinence(associatedwithasudden
needtovoid)andstressincontinence(associatedwithstrainingstresses).
Sheshouldbeadvisedthatthefollowingplanofmanagementisrequired:
~ A urine microscopy and culture should be performed to exclude a urinary tract infection,
which could be the cause of the urinary frequency. If this was found antibiotic therapy
wouldberequired
~Duetotheapparentmixednatureoftheincontinence,sheneedsreferralforaurodynamic
assessment.Thesestudiesshouldbeexpectedtoconfirmthisdiagnosis,andtoseewhich
componentappearstobeworse.
~Inviewofherdesiretoavoidanoperation,whentheresultsoftheurodynamicassessment
aretohand,decisionscanbemadeonnonoperativecare.
~ If there is proven genuine stress incontinence (GSI), there is a place for pelvic floor
exercises,andeventheuseofapessarytocorrecttheanteriorwallvaginalprolapseIn
additionsheshouldbeadvisedtoloseweightandstopsmoking,asthelatterwillalmost
certainlyreducehercoughingepisodes,andweightlossbacktoidealbodyweightusually
improvesGSI.
~ If there is clear evidence of detrusor instability, the use of drugs such as propantheline
(Probanthine) and imipramine (Tofranil), or oxybutynin (Ditropan) should be
considered.HRTmayalsoimproveurinaryurgencyaswellandcouldbetriedfor1-2years
with minimal risk to the patient. Retraining of the bladder, so it accepts a larger volume
beforeemptying,mayalsobeimportantifdetrusorinstabilityisthemajorproblemdefined
intheurodynamictesting.
~ If medical treatment is ineffective, and the predominant problem is genuine stress
incontinence,itmayultimatelybenecessaryforhertohavesurgery.Principlesonlyneed
to be discussed, techniques are designed to elevate, support, and buttress the bladder
neck.
128 (T4)
PerformanceGuidelines
690
KEYISSUES
Knowledgeofthecausesandtypesofurinaryincontinencewhichoccurinmiddleaged
women.
Knowledgeoftheinvestigationsrequiredtoassesssuchincontinenceandthegeneral
principlesofthemanagementprogrammesavailable.
Abilitytorecognisetheprobabilitythatthisisnotjuststressincontinence,thaturodynamic
assessmentisadvisable,andtreatmentwilldependonresultsobtained.
CRITICALERROR
Failuretoarrangeurinemicroscopyandculturetoexcludeurinaryinfection.
COMMENTARY
Urinaryincontinencewithurgencyina50-year-oldwomanmaybeduetoanumberofcauses.
Thesimplestisaurinarytractinfection,whichmustbeexcluded.Othercausesmayinvolvean
unstablebladderorgenuinestressincontinenceassociatedwithvaginal
prolapse. Urodynamic studies should be performed to identify the cause, as the management
maybeeithermedicalorsurgical.
Nonsurgical treatments include bladder retraining and pelvic floor exercises, weight loss,
cessationofsmoking,andpharmaceuticalagentstoimprovedetrusorinstability,whichinclude
HRTandantispasmodics/anticholinergics.
Surgical treatments for female urinary stress incontinence can be grouped into several
procedural types the multiplicity of surgical procedures described is indicative that no single
procedureisuniformlyeffective:
Retropubicortransvaginalsuspensionprocedurestosupportthebladderneckandurethra
(Burch,Marshall-Marchetti-Krantzprocedures)
Anteriorcolporrhaphywithbladderneckbuttressing
Slingprocedures(Aldridge)
Localinjectionsofcollagen
Commonproblemslikelywithcandidateperformanceare:
Failuretotakeanadequatehistoryofthefeaturesoftheincontinence.
~Largevolumessuggesturgencyincontinence.
~Smallvolumesonstrainingsuggeststressincontinence.
~Continuouslosssuggestsfistulaoroverflow.
Failuretorecogniserelationshipoftheincontinencetothemenopauseandtoenquireabout
theuseofHRT.
Failuretoadviseaboutnonoperativemodalitiesoftreatment.
129 (T5)
PerformanceGuidelines
691
Condition 129 (T5)
Migraineina30-year-oldwoman
AIMSOFSTATION
Toassessthecandidate'sabilitytoprovideclearinformationaboutthenatureofmigraineandits
expectedcourse;toobtainthetrustandconfidenceofapatientwithrecentonsetmigrainewith
potentialfor recurrence; effectivelycommunicatethenecessarytreatmentmeasures,including
dual management of pain and nausea; and to counsel regarding principles of management of
migraine,includinglifestyle.
EXAMINERINSTRUCTIONS
Candidatesareexpectedtoknowtheprinciplesoftreatmentbutshouldnotbepenalisedfornot
knowingthedetailsoflesscommonlyusedmedications.
Theexaminerwillhaveinstructedthepatientasfollows:
Youareaged30years.Youattendedaweekagoaboutasecondattackofsevereheadachethat
occurred over the last month. You were unable to go to work because of the attacks. The
headachewaspresentwhenyouwoke,feltoverthefrontandonbothsidesofyourhead,lasted
the whole day and was throbbing. You preferred to avoid light, felt nauseated, and vomited
severaltimes.Youhadaccompanyingflashesandspecksoflightbeforeyoureyes.Youtooktwo
Panadol(paracetamol)everyfourhours,withoutmucheffect.
Thedoctorwhoexaminedyoususpectedmigraine,butdecidedtobecautiousandarrangeda
brainscantoexcludemoreseriousconditions.Thiswasinlinewithyourwishes.Youhavenow
cometofindouttheresult,andreceivefurtheradviceaboutthetreatmentofyourheadache.
Youwereconcernedaboutthecauseofyourheadacheandthepossibilitythatyoucouldhavea
brain tumour. You understand you may have developed migraines, like your mother, but are
puzzledaboutwhyithasstartedatthisstageinyourlife,andalsowhatcausesit.Youremember
thattreatmentwasnotveryeffectiveforyourmother.Youleadaverybusylifeandfindyourwork
as a schoolteacher increasingly demanding. Your two young children are involved in many
activities,which aredemandingofyourtime.You havelittleopportunityto relax.Thereareno
marital,familyorfinancialproblems.
PatientProfile
Youareahappilymarriedteacherwithtwochildren.
Drugsensitivitiesnil
Knownfamilyhistorymotherusedtosufferfrommigraine.
Pastmedicalhistorynomenstrualproblemsorknownrelationshipofheadachetomenstrual
cycle.Youarenotonthecontraceptivepill.
Currentmedicationnil,exceptforPanadol(paracetamol)forheadache.
Habitsnonsmoker,youhave2-3standardalcoholicdrinksoccasionally.
Yourpartnerhashadavasectomy.
129 (T5)
PerformanceGuidelines
Questionstoaskunlessalreadycovered:
WhatshouldIdowhenIhaveanattack?'
CanIpreventtheattacksaltogether?'
Dothesedrugshavesideeffects?' (Askthisifdrugtreatmentissuggested)
ArethereanynaturalremediesorotherthingsIcoulddo?'
Isthemigrainegoingtooccurmorefrequently?'
' AreyousureIdon' thaveabraintumour?'
Whatcausesmigraines?'
Canitleadtoanythingmoreserious?'
Whyhasitcomeonnow?'
'Doesanythinginparticularbringonanattack?'
' HowlongwillIgoonhavingmigraine?'
' Arethechildrenlikelytogetmigraines?'
EXPECTATIONSOFCANDIDATEPERFORMANCE
Firsttellthepatienttheresultofthebrainscan(seetasks).
Thendiscussthefollowingwiththepatient:
The mechanism and epidemiology of migraine, and knowledge of its treatment, trigger
factorsandpreventionmalignancywouldnotexplainthesesymptomsandatumourhas
beenexcludedbythecompletelynormalCT.
Explanation of mechanism functional disturbance, possibly episodic serotinergic/
vasomotorchanges,affectingintracerebral/extracerebralarteries.
Discussion of prevalence affects approx 10% of population, more common in females,
onsetinyoungadults(canbechildren),reducesinfrequencyovertheageof50years.
Trigger factors fatigue, stress, menstrual cycle, foodstuffs especially alcohol, drugs
(especially vasodilators). Counsel against using codeine-containing analgesics regularly
(riskofdependence,constipation,reboundheadaches).
Satisfactorymanagementrequiresfollowupofthepatientbyarrangingreviewofprogress.
Treatmentofacuteattack
CandidateMUSTknowthefollowingprinciplesofmanagement:
Restinquiet,darkenedroomandavoidmovement,reading,watchingTV:applycoldpacks
tohead.
Commenceappropriatemedicationinadequatedosesearlyinanattack.
Appropriatedrugtherapyisselectedaccordingtoseverityofattack,previousefficacy,side
effects.
~Mildsolubleaspirin600-900mg4hourly:orparacetamol500mg2-3tabs4hourly(4g
daily): Maxolon (metoclopramide) 10 mg; Stemetil (prochlorperazine) 5mg; or
Panadeine (paracetamol/codeine), or Mersyndol (paracetamol/codeine/ doxylamine
succinate).
~ModerateErgodrylMono(ergotaminetartrate)1mg,repeatin1hourifnecessary;or
Cafergot(ergotaminewithcaffeine)twotabletsinitiallyor2mg/100mgsuppository;or
Naproxen(250mg)750mgtomaximumof1250mgin24hours.-
~Severeorergotaminetrialfailed Imigran(sumatriptan)50mgtabletrepeatin1hour,
orinjection(sumatriptancontraindicatedwithin24hoursofergotamine).
692
693
129 (T5)
Performance
Guidelines



Preventivetherapy
Thefollowingisasummaryofcurrentpreventivetherapyformigraine.Thecandidateshouldnot
be penalised for lack of detailed knowledge of preventive measures, but should know of their
availabilityandpotentialsideeffects.Ongoingsupportisappropriateinallcases.
Attentiontolifestyleandothertriggerfactors.
Preventi vemedi cati onisnotrequiredinthispatientwithonlytwoattacks;butisindicatedfor
three or more attacks a month, selecting from: beta blockers/pizotifen/
cyproheptadine/tricyclic antidepressants/methysergide/calcium channel blockers. All these
medicationscanhavesignificantsideeffects.
~betablockerscontraindicatedinasthmatics;adverseeffectsoftirednessandbradycardia;
~pizotifenadverseeffectsofdrowsinessandweightgain;
~cyproheptadineadverseeffectsofdrowsinessandweightgain;
~tricyclicantidepressantsimpairedalertness,anticholinergic,autonomicsideeffects;
~methysergidesixmonthsuseonly;adverseeffectsincludevasospastic
syndromes,retroperitonealfibrosis.
~calciumchannelblockersindicatedinsevere,life-disruptingcasesonly,because
ofadverseeffects;useisforalimitedtimeonlytobreakthecycle;cangive
improvementinmajorityofcases.
KEYISSUES
Counsellingandpatienteducation.
Appropriatedrugtherapyaccordingtoseverityandincludingpreventivemeasures,

CRITICALERRORS
Failuretoadequatelyaddressthetreatmentofacuteattacks.
Failuretoconsidertriggerfactors.
COMMENTARY
Migraine is a common disorder encountered in everyday clinical practice by both general
practitionersandspecialists.
Adviceaboutitsmanagementrequiresadequateconsultingtimeincontrasttoitsdiagnosis.
Effective patient education and counselling are required to achieve a successful outcome
including patient acceptance of likely recurrences this requires satisfactory use of
communicationskillsbythecandidate.
Therapyinvolvesbothattentiontolifestylefactorsandchoiceofmedicationfromawiderangeof
alternatives.
A correct choice of therapy depends on the level of severity of the migraine and this initial
consultationprovidesastartingpointforongoingcare.
Theinformationincludedunderexpectationsofcandidateperformanceprovidesasoundbasis
forfuturepracticebythecandidate.
130 (T6)
PerformanceGuidelines
694
Condition 130 (T6)
Pasthistoryofhipdislocationina35-year-oldman
AIMSOFSTATION
Toassessthecandidate'sabilitytoexaminethehipforpossiblepostdislocationhiparthritis.
EXAMINERINSTRUCTIONS
Optimal points that the candidate should cover by examination are detailed below and in the
commentary.Thecandidateisnotexpectedtocoverallaspectstoacquireasatisfactorypass.
Theexaminerwillhaveinstructedthepatientasfollows:
You are a 35-year-old man who sustained a posterior dislocation of the right hip in a motor
vehicleaccidentfiveyearsago.Therewerenoassociatedinjuries,thedislocationwasreduced
undergeneralanaesthesiaandyouhadaperiodofbedrestwithtractionfollowedbygraduated
ambulation and weight bearing. You have been well since and have noted no problems apart
fromoccasionalachingonprolongedexercise.Youarepresentingforacheckupforinsurance
purposes.Thisstationinvolvesthecandidateexaminingyourhipjoints.Becooperative.Youmay
beaskedtowalkandtostepuponacouchorstool.
Pleaserespondnaturallytotherequestsofthedoctorastheexaminationproceeds.
EXPECTATIONSOFCANDIDATEPERFORMANCE
Thecandidateisexpectedtoperformawellorganised,logicalandsequentialexaminationofthe
right hip region, comparing signs with the normal left hip. There will be no abnormal physical
findingsinthiscase.
Trendelenburg test/gait performs and describes competently. The test is positive if the
oppositebuttocksags,ratherthanbeingelevated,whenthepatientstandsontheaffectedleg
(Trendelenburg test 'sound side sags'), or during walking (Trendelenburg lurching gait to
affectedside).
Limb length with patient supine should level pelvis with legs symmetrically placed, in
anatomicalposition.Checksforanyapparentorrealdiscrepancyinleglength,usingatape
measure. Avascular necrosis of femoral head could cause true shortening above greater
trochanter.
Hip joints each side observes for deformity Thomas test for fixed flexion deformity
(exposedbyflexingoppositehiptoflattenlumbarlordosis).
Observesforwasting/tenderness
Observesrangesofmovement(activeandpassive)checksforpain/crepitus.
~ Flexion/extension forward from 0 anatomical position (normal range 0-100), normally
limited by apposition of thigh and abdomen, backward extension while prone from 0
anatomical position (normal range 0-20), normal range of flexion-extension
approximately120.
~ Abduction/adduction pelvis must be fixed. Checks abduction from anatomical position
(0-40range);adductionacrossotherleg(normalrange0-20orhalfwayupthigh),normal
rangeofabduction/adductionapproximately60.
130 (T6)
PerformanceGuidelines
695
~External/internalrotationmustcheckmovementisconfinedtohip.
- Whilesupinerollsthigh(femur)inwardsandoutwards.
- Whilepronebendskneetorightangleandrotates(passive/active)
- Externalrotationrangeapproximatelyfrom0-50.
- Internalrotationrangeapproximately0-40.Normalrangeofexternal-internal
rotationisapproximately90.
Sciaticnervefunction--checksnopost-dislocationweaknessorsensoryloss.
Further tests required could recommend imaging to check for any signs of accelerated
arthritis.Thereisnoobviousproblemdetectedonclinicalexamination,andnosymptoms;a
baselineplainX-rayorcomputedtomograph(CT)wouldbeacceptablebutnotobligatory.
KEYISSUES
Techniqueofexaminationofhips.
Confirmationofnormalfindings.
Qualityofcommentarytoexaminer.
CRITICALERROR
Failingtoadequatelyassesstherangeofmovementsofthehipjoint.
COMMENTARY
The hip joint is designed for stability combined with a flexible range of movements, of which
flexion has the greatest range and extension the least. The bony configuration and strong
ligamentssurroundingthejoint,particularlytheverystronganteriorligaments,protectthejoint
against dislocation under normal strains. Support is weakest posteriorly which is the most
common site for traumatic dislocations. The sciatic nerve is at risk posteriorly. The most
concerning longterm complication is accelerated arthritis due to interference with the blood
supplyoftheheadofthefemurortotraumaticchondromalacia.
131 (T7)
PerformanceGuidelines
696
Condition 131 (T7)
Tirednessina45-year-oldman
AIMSOFSTATION
To assess the candidate's ability to recognise and evaluate a moderately severe major
depressive episode complicated by substance abuse (alcohol, cigarettes, analgesics and
caffeine).
EXAMINERINSTRUCTIONS
Theexaminerwillhaveinstructedthepatientasfollows:
Youareaged45years.Wearstreetclothes.Becooperativeifaskedquestionsortoelaborate
symptoms,butotherwisenotforthcomingorspontaneous.Appearsometimesvague,askingfor
questions to be repeated, as if finding concentration difficult. Avoid eye contact keep eyes
downcast.Appearsad,withslowedmovementsandspeech,andbehumourlessandunreactive.
OpeningStatement
' Doctor,Iamjustexhaustedallthetime,andI' mabouttolosemyjobifIdon' tgetmy
acttogether.Isthereanythingyoucangivemetohelp?'
Continueafterrequestformoredetailsofsymptomswithoutspecificprompts.
Youarejustnotcoping.Youfeeldrainedandawful,especiallyinthemornings.Youwishyoudid
nothavetogotowork,becauseyouhavesomuchtroubleconcentrating,butyoudon'tfeelany
betteronweekendseither.Youhavefeltthiswayforaboutsixmonths,anditisgettingworse.
Inresponsetospecificquestions:Youarenotsleepingwell.Yoursleepisbroken,andoverthe
pastfewweeks,youwakeupat4am,feelingrestlessandbotheredbyallkindsofthoughts,and
donotgetbacktosleep.YouoftentakeaPanadeine(paracetamol/codeine)toaidsleep.You
feelbetterlaterinthedayandafteryouhavehadaglassofwine.Youhavestarteddrinkingat
lunchtimenow.Youcannotbebotheredwithotherpeople,andhavegivenupyourregulartennis.
Youhavenointerestinsex.
Youthinkyourlifeisnotworthliving.Youhavehadpassingthoughts.oftakinganoverdoseor
gassingyourself.Youhavenoactiveplansorsuicidalimpulses.Youmissyourchildrenalot,and
worryaboutyourandtheirfuture.
Responsestodirectquestions
Review of general health: you have had weight loss of about 5 kg in six months; no previous
episodesofweightlossorgain.Poorappetite.Constipationisaproblem,forthefirsttimeinyour
life.
Reviewofrelevantsystems:negativeresponsetoallothersystems.
SocialHistory
Yourwifeleftyousixmonthsago,takingthechildren,andyouhavenothadanyregularcontact
withthemsince.Yourbosshashadtalkswithyouinrecentweeksaboutyourworkperformance,
and yesterday you received a written warning from the company regarding your performance
whichyouunderstandmeansdismissalisunderconsideration.
131 (T7)
PerformanceGuidelines
697
Patientprofile
Recentlyseparatedandlivingalone.
Youareajournalistforametropolitannewspaper.
Youhavetwochildrenaged17and13,andyourestrangedwifehasmadeanyregularcontact
difficult.
Youareahabitualsmokerofabout35-40cigarettesdaily
Youareahabitualdrinkerof-1bottleofwinedaily,increasedoverthepastfewweeksto1-2
bottles.
Noillicitdruguse.
Nomajorillnessesoroperationsinthepast.
Notundertreatmentforanycondition.
Yourdiethasbeenreasonablyhealthyuntilrecentmonths.
Youdrinkatleast8cupsofcoffeedaily.
Nocurrentmedication,apartfromself-administeredPanadeine,2-4tohelpyousleep,most
nightsforseveralmonths.
You had a previous episode of depression when aged 20, when you failed your university
exams, the same year your father disappeared. You refused a psychiatric referral, and the
localdoctorgaveyouValium(diazepam);yougotbetterafteraboutayear,whenyoumet
yourwife.
Nopastsuicidalityorviolence.
Familyhistory
Yourmotheris72andinpoorhealth,livingwithyoursister.Yourfatherwasaheavydrinkerand
disappearedwhenyouwere20.Yourtwooldersistersaremarriedandwell,andaresupportive
butbusy.Noknownfamilyhistoryofdepressionorotherpsychiatricillness.
Aftersixminutes,theexaminershouldinformthecandidatethatphysicalexaminationshowsno
abnormality, and ask: What is your provisional diagnosis and differential diagnosis?
Whattestswillyounoworder?'
EXPECTATIONSOFCANDIDATEPERFORMANCE
This patient has longstanding excessive use of alcohol, cigarettes and caffeine, and recent
pathologicalalcoholintakeandinappropriateuseofanalgesics.Healsohasamajordepressive
illness with psychomotor retardation, vegetative symptoms, anhedonia, depressed mood and
suicidalideation.
Thecandidateisrequiredto:takeahistoryfocusingonthesymptomsandalliedproblemsand
tellyouthelikelydiagnosisanddifferentialdiagnosis,andwhattestsshouldbedone.Candidates
must organise their approach to this station to allow sufficient time to address all the tasks
adequately.Candidatesarenotexpectedtobeabletoeliciteverydetailofthehistoryprovidedin
thescript,butthehistorymustincludedetailsofcurrentandhabitualalcoholandothersubstance
use,especiallycigarettesandanalgesics,andasystematicscreening fordepressivedisorder,
commencingwiththepresentingvegetativesymptoms.Riskassessmentforsuicidalitymustbe
included.
Rapportcanbeestablishedwiththispatient,providedanonjudgmentalapproachistakentothe
substanceabusehistory,includingevidenceofrecentalcoholconsumption,andtotheslowness
ofresponses.Directquestioningisnecessarytoelicitthedepressivesymptoms.
698
131 (17)
PerformanceGuidelines
The proposed tests should include a full blood examination (looking for anaemia and
macrocytosis),thyroidfunctiontests,liverfunctiontestsandserumlipids.
Themainproblem-solvingabilityistorecognisethedepressiveillnessasaprimarydiagnosis,
andnotsolelyanassociatedsymptomofsubstanceabuse,orsolelyatemporaryresponseto
work and family problems. History must elicit key symptoms of major depressive illness
includingpsychomotorretardation,sleepdisturbance,earlymorningwakening,weightloss,
anhedoniaandsuicidality.
Must obtain history of recent excessive alcohol intake and habitual consumption, cigarette
andcaffeineoveruse,andrelevantsystemreview.Mustobtainpsychosocialhistoryofmarital
breakdown and social isolation. Psychiatric examination must include suicide risk
assessmentandbriefreviewofcognitivefunction,(e.g.byelicitingacohesivehistory,butnot
by formal testing of cognitive function [Mini-Mental State Examination MMSE]).
Diagnosis/Differential Diagnosis expected is major depressive episode with alcohol
abuse/dependency.
Depression secondary to organic disease needs to be considered especially frontal lobe
impairment, anaemia, malignancy, hypothyroidism, alcohol related hepatic/ gastrointestinal
disease,whichwillbeassessedfurtherbyinvestigation.
KEYISSUES
Identificationofmajordepressionasprimaryillness.
Appropriatehistory-takingskills.
Riskassessmentofsuicide.
Recognitionofexcessivealcoholintake.
Appropriate investigations for complications of alcohol abuse and for organic causes of
depression.
Recognitionofpotentialsuicidalityrisk.
CRITICALERRORS
Failuretodiagnosismajordepression.
Failuretoidentifyexcessalcoholconsumption.
Failuretoaskaboutsuicidalthoughtsorplans.
131 (T7)
PerformanceGuidelines
699
COMMENTARY
The main clinical reasoning in this case is the ability to take an accurate history of depressive
symptoms and recognise depression as a primary diagnosis, and not solely an associated
symptomofsubstanceabuse,orsolelyatemporaryresponsetowork/familyproblems.Intaking
the history the candidate needs therefore to elicit the key symptoms of major depression,
including psychomotor retardation and slowing with fatigue, sleep disturbance, early morning
wakening,weightloss,poorconcentration,anhedoniaandsuicidalthoughts.
The candidate must also obtain the history of present alcohol abuse, habitual and excess
consumption,detailsofothersubstancemisuse,andarelevantsystemsreview.Thecandidate
musttakesomepsychosocialhistory,namelythemaritalbreakdownandsocialisolationasthese
are highly relevant to the case and further management. Suicide risk assessment is critical. A
formalextendedcognitivestateexaminationisnotpossiblebutthecandidateshouldbeableto
commentoncognitivestateandmemoryasreflectedinthehistoryprovided.
Thecasealsoinvolvesconsiderationofrelevantinvestigations.Theseincludephysicalproblems
that may present as depression (e.g. hypothyroidism, frontal lobe impairment, anaemia, or
malignancy) and physical consequences of alcohol abuse (particularly hepatic and
gastrointestinaldisease).
At first glance this may seem a complicated case, but both depression and alcohol abuse are
verycommonmentalhealthproblems,withhighlevelsofdisability;andtheyoftencoexist.
132 (T8)
PerformanceGuidelines
700
Condition 132 (T8)
Reviewoflungfunctiontestsina65-year-oldmanwith
shortnessofbreath
AIMSOFSTATION
Toassessthecandidate'sabilitytodiagnoseandmanagechronicobstructiveairwaysdisease.
EXAMINERINSTRUCTIONS
Theexaminerwillhaveinstructedthepatientasfollows:
Youarea65-year-oldpatientwhosmokedheavily(30perdayfor40years)untiltwoyearsago,
andhavebeenseeingyourgeneralpractitionerforchroniccoughwithwheezeandshortnessof
breath.Yourshortnessofbreathcomesonwhenshoweringordressingorwhengoingforshort
walks on level ground. The doctor arranged breathing tests (spirometry and assessment of
diffusingcapacity).Youhavereturnedtodayforfurtheradviceandinformationabouttheillness
andtreatment.Youhavebeenacourierdriverallyourlife,deliveringgoodsfromafactorytothe
customer.Youretiredtwoyearsago.
Questionstoaskunlessalreadycovered:
' Whathascausedthisproblem?'
' Canyoufixmylungs?'
'WillitbeharmfultoexertmyselfifIneedto?'
' WillIneedtotaketreatmenteveryday?'
'Didanythingatworkcausethisproblem?'
CanIcontinuetoplaygolf?'
' WillIgetcancer?'
Be very concerned about the future, the effectiveness of treatment and the possibility of
developinglungcancer.
EXPECTATIONSOFCANDIDATEPERFORMANCE
The candidate should correctly interpret the pulmonary function tests as showing significant
obstructiveairwaysdiseasewithsignificantincreaseintotallungcapacityandresidualvolume.
ThereissomechangeintheFEV^FVC(ForcedExpiratoryVolume/ForcedVitalCapacity)ratio
followingabronchodilator.Thereissignificantimpairmentofdiffusioncapacity.
Thecandidateshouldcoverthefollowing:
Cause of the problem almost certainly the heavy smoking of cigarettes. Environment
exposure (dust exposure) would also be important if relevant and prolonged exposure to
vehicle exhaust emissions may be a relevant additional factor in this patient. Central
abdominalobesityisanadditionalcontributingfactor.
132 (T8)
701
PerformanceGuidelines



Management principles assessment of disease severity (given by lung function tests):
moderate airway obstruction based on FVC, FEV
1
and FEV,/FVC ratios; moderate
emphysemabasedontotallungcapacity(TLCincreased)anddiffusingcapacityforcarbon
monoxide(DLCOdecreased)withsmallresponsetobronchodilatortherapy.
~Educationregardingsmoking,includingpassivesmokinganddustexposure.
~Educationregardingaerobicexercise(walkinginparticular).
~Possiblereferralforrehabilitationprogramme.
~Prophylaxiswithannualinfluenzavaccineandfiveyearlypneumococcalvaccine.
~Earlyantibiotictreatmentofexacerbationswithamoxycillin(AugmentinorRulide).
~Trialoftherapywithbronchodilators.
Discussionofappropriatedrugtherapyshouldincludetheuseof:
sympathomimetics(e.g.Ventolin[salbutamol],Bricanyl[terbutaline]);
anticholinergics(e.g.Spiriva[trotropium],Atrovent[pratropium]);
inhaledcorticosteroid(e.g.asinhaledbetamethasone[QVAR],budesonide[Pulmicort]or
fluticasone[Flixotide]);and
roleoflongacting/^-agonistssuchasSerevent(salmeterol)orOxis(eformoterol)should
bementionedaspossibletreatmentoptions.
Roleofoxygentobealludedto:
acuteattack:importanttomodifydoseaccordingtobloodgas(p0
2
andpC0
2
)levels;
homeoxygen:basedonarterialbloodgasesandrelatedfactors.
Need for chest X-ray if not already done in preceding 12 months or so (in view of symptoms,
previous cigarette smoking and risk of lung cancer). The risk of lung cancer in this patient will
reducewithtimeifheremainsoffcigarettesandavoidsexposuretopassivesmoking

Aftersixminutes,examineristosaytothecandidate' Please explain to the patient the
use of the metered dose inhaler and the spacer device.' Closed or open mouth
techniqueisacceptable.Slowinhalationand10secondbreathholdshouldberecommended.
Spacershouldberecommendedifmultidoseinhaler(MDI)techniqueisunacceptable.
KEYISSUES
Interpretationofinvestigationscorrectinterpretationofpulmonaryfunctiontests.
Counsellingexplanationofdiagnosisofobstructiveairwaysdisease,anditscause.
Managementplanuseofmedicationinthemanagementofchroniclungdisease
(emphysema),familiaritywithequipmentpatienteducationinuseofMDIandspacer
device.

CRITICALERROR
Failuretocorrectlyinterprettherespiratoryfunctiontest.
132 (T8)
PerformanceGuidelines
COMMENTARY
ChronicObstructivePulmonaryDisease(COPD)
This is a heterogeneous disorder that includes emphysema, chronic bronchitis, obliterative
bronchiolitis and asthmatic bronchitis. It is defined as 'a disease state characterised by airflow
limitationthatisnotfullyreversible.Theairflowlimitationisbothprogressiveandisassociated
with an abnormal inflammatory response of the lungs to noxious particles or gases'. The
diagnosis should be considered in a patient with symptoms of cough, sputum production and
dyspnoea, and/or with a history of exposure to risk factors. The clinical course of COPD is
influenced by multiple factors: genetic susceptibility, low birth weight, maternal smoking,
infections in the first year of life, asthma in childhood, and exposure to inhaled irritants in the
workplaceandenvironmentalairpollution.Morethan80%ofcasescanbeattributedtosmoking.
Alpha-1 antitrypsin deficiency occurs in about 2% of patients with emphysema and patients
should be screened early in the course of the disease, particularly if there is family history of
eitherlungorliverdisease.
Spirometry confirms the diagnosis and is useful for assessing severity. The GOLD guidelines
1

classifypatientsin4stageslinkedtoforcedexpiratoryvolumein1second(FEV
1
)andforcedvital
capacity(FVC):
Stage0:Atriskcharacterisedbychroniccoughandsputumproduction.Thelungfunctionis
normal.
StageI:MildCOPDwithmildairflowlimitationwithanFEV
1
/FVC<70%butFEV
1
is>80%of
predicted.Somehavecoughandsputumproduction.
Stage II: Moderate COPD characterised by worsening airflow limitation: HA with an FEV
1

50-80%; IIB 30-50% of predicted. Usually there is progression of symptoms with shortness of
breathtypicallyonexertion.Theremaybefrequentexacerbationswithan impact onqualityof
life.
Stage III: Severe COPD characterised by severe airflow limitation with an FEV
1
< 30%
predicted or the presence of respiratory failure or clinical signs or right heart failure. These
patientsusuallyhaveapoorqualityoflife.
LungVolumeMeasurements
Total lung capacity comprises a number of components: tidal volume (TV), inspiratory reserve
volume(IRV),expiratoryreservevolume(ERV),andresidualvolume(RV).InCOPDtotallung
capacity (TLC) is increased largely because of increased RV and functional residual capacity
(FRC).Vitalcapacity(VC)isusuallydecreasedbutmaybenormal.
Management
ManagementofCOPDinvolves:
Educationofpatientsandtheirfamilies.
Retardingtheprogressionbyearlydetectionandavoidingriskfactors.
Minimisingairflowlimitationbyreducingtheproductionandincreasedeliminationof
secretions.
Utilisingbronchodilatorseffectively.
1PauwelsRA.BuistAS,CalverleyPM.JenkinsCR.HurdSSandtheGOLDScientificCommittee.Global
strategy for the diagnosis, management and prevention of chronic obstructive pulmonary disease.
NHLBI/WHOGlobalInitiativeforChronicObstructiveLungDisease(GOLD)Workshopsummary.American
Journal of Respiratory Critical Care Medicine. 2001,163:1256-76.
702
703
132 (T8)
PerformanceGuidelines
Correctingthesecondaryphysiologicalalterationsincludinghypoxaemia,hypercapnia.and
pulmonaryhypertension.
Optimisinglungcapacitybyexerciseconditioning,reducingobesity,goodnutrition,muscle
trainingandpsychosocialrehabilitation.
Smoking cessation (see condition 008) and avoidance of occupational and environmental air
pollutantsareessential.
Bronchodilator medications are central to symptomatic management of COPD. Frequently, a
long-acting
2
-agonist(salmeterol,eformoterol)iscombinedwithananticholinergic(ipratropium
bromide,trotropium).Additionaluseofshort-acting
2
-agonist(fenoterol,salbutamol,terbutaline)
isoftenrequiredforsymptomrelief.Theuseoforaltheophyllineislimitedbyitspotentialtoxicity,
particularlycardiactoxicity,andifused,druglevelsshouldbemonitored.
ProlongedtreatmentwithinhaledcorticosteroidsdoesnotmodifythelongtermdeclineinFEV
1
.
TheGOLDguidelinesrecommendregularuseofinhaledsteroidsinthosepatientswithFEV
1
<
50%ofpredictedorthoseinStageIIIwithfrequentexacerbationsandinthesepatientsthiscan
reduce exacerbations and preserve quality of life. A therapeutic trial of up to three months is
recommended.
Prolonged treatment with oral glucocorticoids (prednisolone) is NOT recommended in COPD
andmayleadtoseveresideeffects,inparticularsteroidmyopathyandosteoporosis,whichcan
severely affect patients with COPD. Short courses, however are very useful in acute
exacerbations.
Combinationmedicationsthatmaybeespeciallyusefulinappropriatepatientswithmoresevere
disease,includeipratropium-salbutamol,fluticasone-salmeterol.
Oxygentherapy
InpatientswithCOPDitisimportanttomonitoroxygensaturationlevels.Arterialbloodgasesare
usually obtained at baseline for comparison during subsequent exacerbations. In patients with
more severe disease, severe arterial hypoxaemia, respiratory acidosis and hypercapnia are
commonasissecondarypolycythaemiaandpulmonaryhypertension.Itisimportanttoevaluate
patientswithmoreseverediseaseforsuitabilityofhomeoxygentherapy.
PatienteducationinuseofMDIandspacerdevice
See also http://www.asthmaaustralia.org.au. There are 2 types of aerosol inhalers: 'Puffer' a
pressandbreathedeviceand'Autohaler'abreath-activateddevice.
Itisrecommendedthatpuffersareusedwitha'spacer'.
Usinga'puffer':Removecoverfrommouthpiece.Holduprightandshakevigorously.Breatheout
andtiltchinup.Put'puffer'mouthpieceinmouthandsealwithlips.Starttobreatheinthrough
mouth,thenfireonepuffofmedicationandcontinuetobreathein.Holdbreathfor10seconds,
thenbreatheoutvianose.Rinsemouthoutafteruseofinhaledsteroid.
Usinga'spacer':Attach'puffer'toendof'spacer'.Place'spacer'mouthpieceinmouthandclose
lipsaroundit.Ifusingwithafacemask,placefacemaskovermouthandnosetoensureagood
seal.Pressdownon'puffer'canisterasillustratedoncetoreleasemedicationintothe'spacer'.
Breathe in and out normally for about 4-5 seconds. 'Spacer' should be cleaned regularly with
warmsoapywaterandallowedtodripdry.
132 (T8)
PerformanceGuidelines
CONDITION132.FIGURE1.
Instructinginuseofaerosolandspacer
704
133 (T9)
PerformanceGuidelines
705
Condition 133 (T9)
Assessmentofa28-year-oldprimigravidaat34weekswithfunduslessthan
dates
AIMSOFSTATION
To assess the candidate's ability to diagnose and manage probable oligohydramnios and
associatedintrauterinegrowthrestriction.
EXAMINERINSTRUCTIONS
Theexaminerwillhaveinstructedthepatientasfollows:
Youarea28-year-oldprimigravidaat34weeksofyourpregnancy.Thelistofresponsesbelowis
likelytocovermostofthequestionsyouwillbeasked.
Nocigarettesmokingoralcoholintake.
Nohypertensionatanytimeandnopreviousrenaldisease.
Noproblemwithlupusorarthritisinthepast,andnothrombosesinthepast.
Nocontactwithdogsorcats,andyoudonoteatrawmeat.
Fetalmovementsarestillpresent,buttheyarecertainlylessthantheywere.
Thedatesarecertain,andthesewereconfirmedbythe18-weekultrasoundwhichwas
apparentlynormal.
Questionstoaskunlessalreadycovered:
'Whyhasn' tmyuterusincreasedinsize?'
EveryonesaysIlooksosmall.IsmybabygoingtobeOK?'
WhendoyouthinkI'llcomeintolabour?'
' WillIstillbeabletohavemybabynormally?'

Examinationfindingsandinvestigationresultstobegiventothecandidatebytheexamineron
request
Symphysis-fundalheightis29cm,cephalicpresentation,headthreefingerbreadths
abovethepelvicbrim
Fetalheartaudible,normalrate.
Bloodpressure120/80mmHg.
Nooedema.
Tests:Investigationresultsnoproteinuriaonwardtesting.Cytomegalovirus(CMV)
antibodieshadbeentestedpriortopregnancybecauseofherworkintherenalunit(IgG
antibodieswerepositive).Shehasnotbeenscreenedfortoxoplasmaantibodies.
706
133 (T9)
PerformanceGuidelines



EXPECTATIONSOFCANDIDATEPERFORMANCE
Thisisaclinicalproblemofintrauterinegrowthrestriction(IUGR).ThereasonforthisIUGR,and
itsseverity,needtobeassessedbyspecialinvestigations,andthetimingandmodeofdelivery
shouldbediscussed.
Investigationsrequiredshouldinclude:
Ultrasoundexaminationtoconfirmthesizeofthebaby,lookfortheamnioticfluidvolume,and
to see whether there is any obvious congenital abnormality which might explain the clinical
IUGR.Thisexaminationwillprobablyneedtoberepeatedeach2-3weeks.Bloodflowinthe
umbilicalcordshouldalsobeassessedbyDopplerstudies.
Cardiotocographicevaluation(CTG)twiceaweekfromnowuntilthetimeofdelivery.

CausesofIUGRwhichneedtobethoughtaboutareasfollows:karyotypicabnormalities:renal
disease: pre-eclampsia; congenital infections (such as CMV or toxoplasmosis); or placental
dysfunction(thiswouldincludeaconcealedplacentalabruption).
TheappropriateinvestigationscertainlyincludeultrasoundexaminationandCTG,asreferredto
above,butalsoserumurea,uricacidandcreatininelookingforevidenceofrenalcompromise,
and measurement of the lupus anticoagulant and anticardiolipin antibody would be advisable
(knowledgeable candidates may suggest all these tests). The antibodies for toxoplasmosis
shouldbesoughtand,ifamniocentesisisbeingperformedtoassessthekaryotypeofthebaby,
aswouldoftenbeadvised,toxoplasmosisshouldberuledoutonthisspecimenaswell.
The mode of delivery will probably be by Caesarean section, and almost certainly prior to the
estimatedduedate.Thetimingofthiswilldependontheultrasoundevaluation,theCTGrecord,
and the amount of growth which occurs subsequently. Providing the baby is not allowed to
become hypoxic and acidotic, and does not have a congenital malformation or a congenital
infection,thelongtermprognosisforthebabyshouldbesatisfactory.

KEYISSUES
Understandingthecausesofoligohydramnios.
Managementofagrowth-restrictedfetusinlatepregnancy.

CRITICALERRORS
Failuretorecognisethattheclinicalpictureisofintrauterinegrowthrestriction.
Failuretoarrangeappropriateassessment(orreferraltospecialist).Attheleastultrasound
andserialCTGmustbedone.
133 (T9)
707
Performance Guidelines



COMMENTARY
Thiscaseassessesthecandidate'sabilitytounderstandthecausesofafundusmeasuringless
thanthatexpectedforthegestationalage.Themostcommonoftheseisthatthefetusissuffering
from intrauterine growth restriction (IUGR) with usually a degree of oligohydramnios. The
importantaspectofthiscaseistohaveanunderstandingofthemanagementofafetuswhichis
growthrestrictedinlatepregnancy.Themeansofmonitoringthefetusfortheremainderofthe
pregnancymustbeknown,andalsotheoptionsforsafedelivery.Referraltoaspecialistmaybe
advised,butthecandidatewillbeexpectedtohaveanunderstandingofwhatthespecialistwill
advisethepatient.
Commonproblemslikelywithcandidateperformanceare:
Failuretoconsiderthevariouscausesofapparentoligohydramics.
Failuretoadvisethatdeliverypriortotheestimateddateofconfinementwillprobablybe
requiredandthechanceofthisneedingtobebyCaesareansectioniscertainlyincreased.
134 (T10)
PerformanceGuidelines
708
Condition 134 (T10)
Deliriumina25-year-oldmanafteraburninjury
AIMSOFSTATION
Toassessthecandidate'sabilitytoassessandinitiallymanageayoungacutelyillpatientwho
has a delirium/acute brain syndrome/acute confusional state, and to identify the nature of the
physiologicalproblem.
Theexaminerwillhaveinstructedthepatientasfollows:
Wait for the candidate to take the lead. You are a 25-year-old university student, living in a
share-house.Youareusuallyingoodhealth,yourstudiesaregoingwell.Youdrink6-8alcoholic
drinksatparties,butdonotdrinkonadailybasisorhaveanyalcohol-associatedproblems.
You do not use drugs. You have never been in hospital before. You have had no other
operations, illnesses, accidents or injuries of any kind. There is no family history of any
psychiatricorneurologicalillness.Anyotherbackgroundhistory,sayIcan' tbebotheredwi thal l
that now.'
Youwereoutcampingwithmates.Thefirewasslowtostart,soyouthrewsomepetroloverit.
You cannot remember exactly how long ago that was, maybe a day or so, or how you got to
hospital.Youknowyouareinhospital.Youdonotknowwhichone,orforhowlong.Youfeelabit
nauseated,nothungry,andyouareinpain.Youfeelasifyouhavenotbeensleepingatall,but
youdon'tknowhowthetimehaspassed.Whenyoudofallasleep,youaresuddenlywokenby
vivid,frighteningdreamsofbeingchased,beinginafire,orsomeotherdangeroussituation.
Thelasttime,youthoughtitwastheangelofdeath.Whenyouwake,youseestrange-looking
peopleintheroomwithyou,andthentheysuddenlydisappear.
Younowfeeltoofrightenedtocloseyoureyes,afraidthatsomethingbadwillhappen.Youare
frightened that someone has given you the wrong drugs by mistake, and you don't think you
shouldtrustthenursingstaffbecausetheydon'tseemtoknowwhattheyaredoing.
Whemaskedfurtherquestions:
~ Be quite cooperative, but subdued, answer the questions, and look a bit suspicious and
perplexedaboutitall.
Whenthecandidatetestsyourcognitivefunction(theymaynotcheckallofthis):
~Youknowyouareinhospital(don'tknowthename);youknowyourname(own),address
(own),andage(25)anddateofbirth:(ownday&monthcurrentyearminus25).Givethe
wronganswerfortoday'sdayanddate,butcorrectformonthandyear.Givetheincorrect
season(withrespecttothedayoftheexam).
~Beapologeticyoufeelyouarejustguessing.Rememberonlytwooftheitemsyouare
askedtorepeat,nomatterhowmanyrepetitions,andnonewhenaskedaftera
gapintime:' No, sorry, it is just gone' . Serial7s:100....93....85....70 .' What was I
upto?' 'World'backwards:drIwo.
~Nameanyobjectsyouareaskedcorrectly,repeatanyphraseyouaregivencorrectly,and
completethefoldedpapertestorsimilarreasonablycorrectly.
~Writeasentence'Iamfeelingsick'.
134 (T10)
PerformanceGuidelines
709
-Obeythewritteninstructionyouaregiven.Sayyouaretoosicktocopythediagram.When
thecandidateasksyouaboutseeingthings,describethevisionsthatwakenyou,andthe
creaturesyouseeintheroom.Youarenothearingvoices,anddonothaveanyotherodd
orunusualexperiences.
Youarefeelingafraid,andarebeginningtothinksomethingmusthavegonewrong,youare
notbeinggiventherightdrugs,orthenursesdon'tknowwhattheyaredoing,butyouarenot
convincedofthis,andcanbetalkedoutofit.
Questionstoaskunlessalreadycovered:
'Am I losing it?'
If/whenthecandidateexplainsyouhaveadelirium/acutebrainsyndrome/acuteconfusional
state,andgivesyouagoodexplanationoftheconditionanditsmanagement,expressrelief
andaskifyoucanhavesomethingtosleepnow.
The explanation should be similar to the following: this is a common complication of major
injuries and their treatment, such as your burn. It will most likely get better along with your
recovery, but there are some common problems which need to be checked, ike infection,
changesinyourfluidbalance,anditmaybeasideeffectofyourpainrelief.Thevisionsand
fearsyouhavearepartofthedelirium,anddonotmeanyouhaveapsychiatricillnesssuchas
schizophrenia.Ifthenursingstaffknowsaboutyourfears,theycantakeextracaretoexplain
their treatments to you, and they will also be careful about lighting, noise etc, as those
environmentalmatterscanbeirritating.Inthemeantime,therearethingsthatcanbedoneto
relieveyoursymptoms,suchasadditionalsedation,andchangingyouranalgesic.

EXPECTATIONSOFCANDIDATEPERFORMANCE
Thecandidateisexpectedtoidentifythenatureofthepsychologicproblemby:
Taking a history about the presenting problem (inability to sleep), which should easily lead
themtothesymptomsofvisualhallucinations,ahallmarkofanorganicconditionassociated
with heightened arousal, nonspecific fear, with vague persecutory ideation, as well as
short-termmemorydeficits.
Confirmingthediagnosisbyperformingaformalcognitiveassessment(i.e.byassessmentof
orientation/attention/concentration/memory; could use Folstein Mini-Mental State
Examination[MMSE],orotherscreeningtoolbutnotessential).Thecognitiveassessmentwill
confirm that the patient has significant deficits in orientation, attention concentration and
short-term memory. These are the hallmark indicators of an acute brain
syndrome/delirium/acute confusional state, especially in association with perceptual
disturbance, typically visual or tactile hallucinations, and disturbances of thinking, including
persecutoryideation.
Intheexplanationto thepatient oftheproblem,candidates must useappropriate clearand
plainterminology,andmustindicatetheyknowthatthereisaneedtoassessthecause(s)of
thedelirium.Whilstthisaspectisnotbeingfurtherassessedatthisstation,theyareexpected
tosaytheywouldstartwithareviewofthelikelyphysicalcausesofdelirium,andthatthemost
likelycauseistheketamineormorphine.Knowledgeablecandidateswillfocusinparticularon
the ketamine, which is often associated with disturbed sleep and vivid and concerning
dreams.
In this station, candidates are not required to proceed to do the investigations or change
medication.Iftheyattempttodoso,informthemthattheydonotneedtoproceedfurther.
134 (T10)
710
PerformanceGuidelines

Ataboutsixminutes,ifthecandidatehasnotalreadytoldthepatientthediagnosis,askthemto
do so. If the candidate offers a diagnosis or differential diagnosis of functional psychosis (e.g.
schizophrenia),donotaskthemifitcouldbeanythingelseorhowcertaintheyare,unlessthey
indicatetheyarenotentirelysure(i.e.don'thinttheymightbewrong).
The candidate must stipulate some likely causes of the condition, again using clear and plain
terminology,andmayspecifymedicationtobeused,butascheduleisnotrequired.Appropriate
specifictreatmentsare:
Discussionwithnursingstaffaboutexplanation,reassurance,environmentalcontrol.
Cessationofmorphinebythis stageafterburn injuriesfurtheranalgesiadoes notusually
requireopioids.
Avoidfurtherdrugtherapyunlesssymptomspersist.Ifrequired,useanadequatedoseofa
mediumactingbenzodiazepine(e.g.diazepam,lorazepam),withdosetitratedtoresponse
possibly a low dose of a sedative antipsychotic (e.g. olanzapine, being mindful of acute
extrapyramidalsideeffects).

KEYISSUES
Abilitytomakeadiagnosisofdelirium,bytakingbothanadequatehistoryandperformingan
adequatementalstateexamination.
Abilitytoformulateandcommunicateaninitialmanagementplan,whichincludesaddressing
thecauseofthedelirium,andrelievingthesymptoms.

CRITICALERRORS
Failingtomakeadiagnosisofdelirium/acutebrainsyndrome.
Tellingthepatienthehasschizophreniaorlikeillness.

COMMENTARY
Delirium(acutebrainsyndrome)isanacuteconfusionalstateassociatedwithacuteimpairment
ofconsciousness.Disorientation,restlessness,agitation,illusionsandhallucinations
arecommon.
Deliriumandconfusionmaybeprecipitatedbychangefromafamiliarhomeenvironmenttothe
unfamiliaranddisturbingenvironmentofthehospitalward.Confusionisoftenworseatnight.
Theconditionisoftenparticularlydistressingforconcernedrelativesandfriends.Itisimportantto
rapidlyobtainasadequateahistoryasispossiblefromthepatient,relatives,andnursingstaff,
andtoundertakeafocusedclinicalexaminationlookingforprecipitatingfactorsandidentifiable
disorders, so that effective treatment can be instituted as quickly as possible, and so that the
patientisnotatcontinuingriskfromphysicalinjuryarisingfromhisconfusedstate.
134(T10)
PerformanceGuidelines
711
Commoncausesinhospitalisedsurgicalpatientsinclude:
Hypoxiacommoninpostoperativeandpost-traumaticpatientsandthosewithatelectasis.
Drug intoxication or withdrawal common drug intoxications can occur from sedatives,
analgesicsandopiates,anaestheticorneuroleptanalgesicagentssuchasketamine,andfrom
withdrawalofalcohol.
Sepsis - chest, wound and urinary infections are the most common, particularly with
accompanyingbacteraemia.
Metabolic disturbances water and electrolyte disturbances (hyponatraemia
hypernatraemia), hypoglycaemia or hyperglycaemia, anaemia, uraemia, hyperthyroidism or
hypothyroidism,hepaticdisease.
Vulnerability to the above produced by an underlying central nervous system or psychiatric
disorder(Alzheimerdisease,cerebraltumour,psychosis).
Key features include disturbed consciousness and confusion with disorientation in time and
place, which fluctuate over a 24-hour period with episodic lucid intervals. Perceptual
abnormalities (hallucinations); sleep-wake cycle abnormalities (typically inverted pattern
awake all night, dozing briefly during the day); mental sluggishness; poor attention and
concentration with distractibility; affective lability including fear, anxiety, depression or
grandiosity and aggression; ideas of reference and misinterpretations leading to paranoid or
suicidal/nihilistic ideation and delusions; psychomotor disturbance: either restless agitation or
somnolentretardation,anyorallmayoccurinanycombination.Onrecovery,mostpatientshave
no recall of their period of delirium. Some patients experience the features of so-called
'near-deathexperiences'.
Mostpatientsinhospitalwithdeliriumhavemultiplefactorswhichcontributetotheirconfusion.
Advancingageand'cerebralinsufficiency',drug/alcoholeffects,braininjuryorinfectionandany
conditionrequiringintensivecarearepredisposingfactors.
Treatment involves recognition of the diagnosis and then identifying and correcting the
contributingfactors.Pain,sleepdeprivation,exhaustion,panic/terrorandenvironmentalfactors
must not be overlooked. Assigning the same staff each shift to care for the patient and
encouragingfamilyandfriendstositwiththemarehelpful,asisaquietroomwithfamiliarobjects
fromhomenearby.Subduedlightingatnightassistsinorientationandreducesthelikelihoodof
misinterpretations.
Haloperidolinlowdosagetwicedailycanbeusedforagitationinitiallyandcanbesafelyused
parenterally. Olanzapine is now available as an intramuscular alternative. The newer atypical
antipsychoticsare'notapproved'foruseindeliriumorinpatientsunder18orover65.
Short acting benzodiazepines lorazepam, oxazepam and alprazolam may be preferable
sedativesfor useatnightand inpatientswithliverdisease.Diazepammayprolongorworsen
daytimeconfusion.Hypoalbuminaemiamaymeanthedosageofallprotein-bounddrugsmustbe
reduced, including psychotropics, to limit medication toxicity from free drug in the circulation.
Oncethepatienthasbeguntoimprovethepsychotropicsshouldbereducedgraduallyandnot
stoppedabruptly.
135 (T11)
PerformanceGuidelines
712
Condition 135 (T11)
Chronicdiarrhoeaina45-year-oldman
AIMSOFSTATION
Toassessthecandidate'sabilitytoelicitthecauseofchronicdiarrhoeainanadultandselectthe
essentialtestsforitsinvestigation,includingscreeninganddiagnostictestsforcoeliacdisease.
EXAMINERINSTRUCTIONS
Theexaminerwillhaveinstructedthepatientasfollows:
Youarea45-year-oldinsuranceagentsufferingfromdiarrhoea.Youhavebeenreferredbackto
thegastroenterologydepartmentofthehospitalbecauseyouarenotimproving.
Openingstatement
' I' vestillgotdiarrhoea.IwastoldbythelastdoctorIsawheretocomebackifitdidn' t
clearup.'
Withfacilitationandminimalpromptingbythedoctorstate:
You have to go to the toilet urgently five or six times every day and the bowel action is either
'explosive'orjust'poursout'likewater.Youalsopassalotofwind,andnoticealotofgurgling
noises in your abdomen. Your abdomen also feels bloated on and off. The medicines you are
takinghavenotstoppedthediarrhoea(KaomagmaandImodium[loperamide]).Thetrouble
started about six months ago and has been gradually getting worse. Three months ago you
developedathroatinfectionandweregivenacourseofantibiotics.Thediarrhoeawasworsefor
threeweeksafterwards.Atfirstyourlocaldoctorthoughtthatthiswasthecauseandadvisedyou
to eat yoghurt and less fatty food. However, the diarrhoea persisted and two months ago you
werereferredtothisclinicandhadtestsofyourfaecesforinfectionsbutthesewereclear.
You were referred for colonoscopy and advised that this was also normal; to take Imodium
capsules as-required to control the diarrhoea, which was most likely due to irritable bowel
syndrome. You are now 'fed up' and want something done about your problem. Your back
passage has become pretty sore and you are sick of the inconvenience. You are also feeling
moretiredthanusualandlackenergy.
Inresponsetodirectquestioning,providethefollowinganswers:
Noabdominalpain,neveranybloodorslime(mucus)inthemotion.
Themotionispalerincolourthanbefore,stickstothepan.andtendstofloat.Ithasamore
offensivesmellthanusual.
Whentheurgecomesonyouhavetogotothetoiletimmediately.Thisfeelinghaswokenyou
atnight.Sofaryouhavenotsoiledyourself.
Onrisinginthemorningyouneedtogotoopenyourbowelsimmediately.Youusuallyhaveto
gosoonafterameal.
Thediarrhoeadoesnotseemto berelatedtothetypeoffoodyoueatexceptthatitisless
frequent when you eat less. The Imodium does help but you do not take it regularly. The
Kaomagmadoesnotmakeanydifference.
713
135(T11)
PerformanceGuidelines



You had not been overseas prior to the onset. Appetite is not affected but you have lost
weight. You have had no rashes. Negative responses to questions about other body
symptoms.
Reviewofgeneralhealth
Youhavelostweightfourtofivekilogramssincethediarrhoeabegan.
Youhavefeltmoretiredthanusualandlackenergy.
Youstillhaveagoodappetite.
Be cooperative and pleasant but dissatisfied you now want 'something done' for your
problemwhichwasexpectedtoeventually'settledown'.Youarenotcancerophobic.Youdo
notregardyourselfasanervouspersonandyoudonothaveanyparticularlifestresses.
Questionstoaskunlessalreadycovered:
'WhydoyouthinkIstillhavethisdiarrhoea?'
' Isn' tthereanothertestwhichwillgivethediagnosis?'
EXPECTATIONSOFCANDIDATEPERFORMANCE
Thecandidatewouldbeexpectedtocoverareasonablecross-sectionofthefollowing:
History
~Appropriatelydetailedhistoryforchronicdiarrhoeatoincludeduration,frequency,nocturnal
occurrence,stoolcharacter,associatedbloodormucus,abdominalpain,weightloss.
~Anyrecenttravelhistory,recentantibioticcourse.
~Previousinvestigations,adviceandtreatmentfromlocalmedicalofficerandhospital.
Examinationfindingswhichshouldbeaskedforfromtheexaminer
~Vitalsigns(allnormal)
~Signsofnutritionaldeficiency/malabsorption(skincolour,tongue,nailsnoabnormality).
~Checkskinforrash(nonepresent).
~Conjunctivaeforanaemia(noabnormalfindings).
~Thoroughabdominalexamination(nolocalisedtenderness,nomass).
~Rectalexaminationfortenderness,mass,stoolcolour(noabnormalfindings).
~Theexaminerwillsummarisebystatingthatphysicalexaminationisnormaland
noncontributorytothediagnosis.
Discussionwiththepatient
~ Wide range of possible causes for the diarrhoea mostly excluded by previous
investigations.Diagnosisofcoeliacdiseasemaybesuspectedbutcannotconfidentlybe
madeatthisstage.
~ Physical findings (or lack thereof) support possibility of coeliac disease or some other
malabsorption syndrome, whilst reducing possibility of other serious causes of the
diarrhoea.
~ The candidate should acknowledge the patient's concerns and dissatisfaction that his
conditionhasnotimproved.
135 (T11)
714
PerformanceGuidelines



Furtherinvestigationiswarranted,asfollows
~Repeatstoolmicroscopyandculture.
~Fullbloodexamination,checkingformicrocyticandmacrocyticanaemia.
~ImmunologicaltestsessentialIgAandantigliadinantibodies.
~Smallbowelbiopsyistheonlyspecificconfirmatorytestforcoeliacdisease.
~Repeatcolonoscopyisacceptableprovidingthattestsforcoeliacdiseasearealsoadvised.
~Bariumenemaislessappropriate;abdominalcomputedtomographacceptable.

KEYISSUES
Abilitytotakeafocusedhistoryrelevanttothepatient'sproblemofchronicdiarrhoea.
Recognitionoftheimportantexaminationfeaturestobelookedforincomingtoadiagnosis.
Diagnosis/differentialdiagnosisdependsonknowledgeofcoeliacdiseasebutthecandidate
shouldrecognisethechronicityofthediarrhoeaandpossiblecauses,andthesignificanceof
the negative results of previous investigations, suggesting that additional investigations
shouldbedone.
Choice of investigations recognition that immunological screening for coeliac disease
shouldbedone,andthatsmallbowelbiopsyshouldbediscussed.

CRITICALERROR
Failuretoconsiderthediagnosisofcoeliacdisease

COMMENTARY
The main diagnostic challenge in this case is to distinguish a malabsorption state from
inflammatory bowel disease. Differential diagnosis includes: coeliac disease, irritable bowel
syndrome, ulcerative colitis, Crohn disease, giardiasis, thyrotoxicosis, carcinoma colon
(previouslyoverlookedatcolonoscopy),lactoseintolerance,antibiotic-induceddiarrhoea.Other
causes of malabsorption are less likely: tropical sprue, Whipple disease, intestinal
lymphangiectasia,pseudomembranousenterocolitis,HIV.
Coeliac disease is defined as the permanent intolerance to gluten (present in wheat, rye, oats
and barley) resulting in small bowel villous atrophy and malabsorption. Symptoms improve by
removingglutenfromthediet.
Thefrequencyofcoeliacdiseaseisdisputed,butinEuropeans is possiblyupto1 in300. Itis
likelythatmanymildcasesescapediagnosis.Theconditionisfamilial.Thereisanincreasedrisk
of lymphoma. Dermatitis herpetiformis may be associated. Common presenting features of
coeliacdiseaseare:fatigueandweightloss(70%);diarrhoea(70%)onsetofsymptomsoften
follows use of antibiotics; flatulence and bloating (60%); and iron or folate deficiency (70%).
Serological tests for coeliac disease are now readily available and knowledge of these is
expected. Total IgA is included, as IgA deficiency often occurs in coeliac disease. Antigliadin
antibodies(gliadinisacomponentofgluten)areusefulinmonitoringdietarycompliancebutare
notspecific.Falsepositiveantigliadinantibodies,particularlyIgG,canoccurwithCrohndisease,
post-infectionmalabsorptionandsomeautoimmunediseases.
135(711)
PerformanceGuidelines
715

All suspected patients should have the diagnosis of coeliac disease confirmed by small bowel
biopsy,asthisistheonlyspecificconfirmatorytestforthisdisease.Becauseofspecificradioand
television programs about adult coeliac disease and the activities of the Coeliac Society of
Australia,thereisanincreasingawarenessoftheconditioninthecommunity.
136 (T12)
PerformanceGuidelines
716
Condition 136 (T12)
Fever,irritabilityandeardischargeina2-year-oldboy
AIMSOFSTATION
To assess the candidate's ability to diagnose and manage otitis media with perforation of the
drum,andtheabilitytoachieverapportwithananxiousparent.
EXAMINERINSTRUCTIONS
Theexaminerwillhaveinstructedtheparentasfollows:
You are the mother of 2-year-old Alexander. You have just told the doctor about him and the
doctorhasjustexaminedhim.Youarefeelingguiltythatyoudidnotbringthechildyesterday,
andworriedaboutthepossibilityofsomepermanentdamagetotheear,ifyouaretoldthatthe
righteardrumisruptured.Youfeelthatitisyourentirefaultthatthishappenedandthatnowhis
hearingwillbepermanentlycompromised.
Openingstatement:
' Should I have brought him yesterday, doctor?'
Questionstoaskunlessalreadycovered:
' Do you think he needs antibiotics?'
' Will it get better on its own?'
Isthere any other medicine which might help?'
'Will the hole in his ear drum heal?' (Askifthecandidateindicatesthedrumhas
ruptured).
' Will it affect his hearing?'
'Will his hearing be affected permanently?'
' Should he see a specialist?'
EXPECTATIONSOFCANDIDATEPERFORMANCE
Explanationofdiagnosis
Thecandidateshouldexplainthat:
Thechildhasacuteotitismediawhichisacommonchildhoodillness,oftenprecededbya
viralupperrespiratorytractinfection.
Aperforatedeardrumcomplicatesasmallproportionofcases.
Thespontaneousdrainageofthemiddleearafterperforationoftheeardrum/tympanic
membraneleadstoreliefofpainandresolutionoffever.
Theholeinthetympanicmembraneisusuallysmallandhealsspontaneouslywithinafew
weeksinthemajorityofcases.
Mostchildrendonothavesignificant,longtermdeficitinhearing,buttestingofthehearing
followinghealingofthedrumcouldbeperformedtoallayanysubsequentconcerns.
Immediatemanagement
Oralantibiotics,preferablyamoxycillininappropriatelyhighdose(90mg/kg/day).Cefaclor,and
amoxycillinwithclavulanate(Augmentin)arealsowidelyusedandwouldbeacceptable.
717
136 (T12)
PerformanceGuidelines

Simple analgesia (e.g. paracetamol) should be given for pain. Decongestants are of no
establishedbenefitinacuteotitismedia.Acuteotitismediawilloftenresolvewithouttheuseof
antibioticsbutthepresenceofapurulentdischargefromtherightearandabulginglefteardrum
would dictate their use in this case, and a culture of the discharge may be helpful but is not
obligatory.
Futuremanagement
Arrangetoreviewthechildinoneweekforfurtherexaminationandinparticularinspectionof
therighteardrum.
Specialistreferralisnotnecessaryatthisstage,butreferraltoanENTspecialistwillbe
requirediftheperforationdoesnothealspontaneously.
Ongoingobservationisnecessarytoensurethattheperforationhashealed.
Considerhearingcheckineightweeks.
KEYISSUES
Managementofacuteotitismediainchildhood.
Abilitytoachieverapportwithanxiousparent,whoisblamingherselfasshefeelsherinaction
hascompromisedherchild'shearingpermanently.
COMMENTARY
Whilethisscenarioaddressesotitismedia,acommonconditioninyoungchildren,theimportant
issuetobeaddressedaswellistheguilttheparentfeels(inappropriately)thatherinactionhas
caused her child to be permanently deaf. This is not the case and spontaneous perforation is
quitecommon.Reassurancethatthiscanoccurinanycaseofotitismedia,andoftenisbeneficial
inthatitrelievessymptomsofpainanddiscomfortimmediatelyshouldbeemphasised.Bythe
endoftheconsultation,thesefearsoftheparentshouldbeallayedbyappropriatereassurance.
Perforation does not necessarily mean major damage to the tympanic membrane, which
generallyhealscompletelywithoutanyhearinglossordamagetothemiddleear.Inthiscase,
there is evidence of significant infection in the other ear and antibiotics are considered
appropriateinthiscase.Gentleeartoiletwithcottonbudstoremoveexcessdischargeisallthat
is necessary for the right ear and antibiotic drops are not necessary. The parent should be
cautionedaboutvigorouscleansingoftheexternalcanal.
Most cases of otitis media in young children have a viral aetiology and antibiotic therapy is
usually unnecessary. Otitis media is usually part of a generalised upper respiratory tract
infection.
Whiletheconditionusuallyresolvescompletelywithoutanysequelae,itisappropriatetoreview
for any complications and to arrange for a hearing test, especially if the parents feel that his
hearingdoesnotseemnormalfollowingtheillness.Theyshouldbecounselledtopresenthim
againifthisisthecase
Ifantibioticsaretobeused,itisimportanttoensurethatanappropriatedrugisusedinadequate
dosage.
137 (T13)
PerformanceGuidelines
718
Condition 137 (T13)
Reviewofcytologyafteraspirationofabreastlesionina
28-year-oldwoman
AIMSOFSTATION
Toassessthecandidate'sabilitytomakeadiagnosisofasmallbenignbreastcystafterlookingat
the investigation results, and to counsel the patient appropriately on results of cytology, and
futuremanagement.
EXAMINERINSTRUCTIONS
Theexaminerwillhaveinstructedthepatientasfollows:
You are 28-years-old, single and work as a secretary. Your mother, now aged 51 years, had
mammographic screeninglastyear whichfoundanearlyandnonpalpablebreastcancer. This
has since been treated by wide local excision using needle localisation, axillary lymph node
dissection,andlocalradiationtotheremainingbreast.Shecontinuestobewell12monthsafter
theoperation.Youareworriedaboutyourownrisksofbreastcancer.Youhadnotedsomeminor
breastpain,especiallylateinyourmenstrualcycle,andsawyourlocaldoctortwoweeksagofor
this.Youweretoldtherewasnothingworryingonexamination,butanultrasoundofthebreasts
wasarranged.Thedoctordoingtheultrasoundexaminationfoundacystinyourrightbreast,and
aspiratedthisafterconsultingwithyourdoctor.
Youarenowseeingthedoctoragaintodiscusstheresultsoftheultrasoundexaminationandthe
findingsofthemicroscopicexaminationonthecystfluidwhichwasaspirated.Youareworried
whetherthecystcouldbeacancer,orcouldturnintoacancerinthefuture.Youhaveanolder
sister,nowaged30years,andabrotheraged28years.Botharewell.
Questionstoaskunlessalreadycovered:
' Whatdidtheultrasoundshow?'
' Whatwastheresultofthepathologyexamination?'
' Canacystlikethisturnintocancer?' (Ifthecandidateidentifiesabreastcystasthe
diagnosis)
'ShouldIhaveanoperation?'
'Whatcausesacysttocome?'
'ShouldIstoptheoralcontraceptivepill?'
' DoesmumhavingabreastcancermeanthatI' mgoingtogetonetoo?'
Howoftendoyouwanttoseeme,andwhattestsshouldIhave?'
'Whataboutmyoldersisterdoessheneedteststoo?'
137 (T13)
PerformanceGuidelines
719
EXPECTATIONSOFCANDIDATEPERFORMANCE
The ultrasound film shows a simple benign small breast cyst. The result of the fine needle
aspirationcytology(FNAC),asshowninthereport,clearlyindicatestherearenomalignantcells
seen.Thepathologyismostlikelyabenignbreastcystwithnorelevancetoherfuturerisksof
breastcancer.Thepathologyreportshowsbenignductalcellsandmacrophagesonlyconsistent
withabreastcyst.
If the candidate believes the lesion could be a malignancy and wishes a mammogram to be
performed,thiswouldbeinappropriatecareatthisyoungageandshouldbemarkedaccordingly.
Thecandidateneedstoinformthepatientclearlyofthediagnosisandthemanagementreguired
now.Thisshouldbealongthefollowinglines:
Diagnosisbenigncyst,nofuturecancerrisksonthisaccount.
Managementobservationandperiodicreview(perhapsreferraltospecialistbreastsurgeon
forfurtherreassurance).
Thecandidateshouldthenaddresstheconcerns raisedbythepatientinthequestionsshe
hasasked.
' Canacystlikethisturnintocancer?' (No).
' ShouldIhaveanoperation?' (Unnecessary,unlessarecurrentcystoccursnotresponsive
torepeataspiration).
' Whatcausesacysttocome?' (Producedbythebreastcellswhichrespondabnormallyto
thehormonesofthemenstrualcycle).
' ShouldIstoptheOCP?' (No).
' DoesmumhavingabreastcancermeanthatI' mgoingtogetonetoo?' (No,butyou
areataslightlyincreasedriskbecauseofthefamilyhistory).
'Howoftendoyouwanttoseeme,andwhattestsshouldIhave?' (Unlessyoudevelop
alump[andyoushoulddobreastself-examinationeachmonthafteryourperiodhasfinished]
youshouldseemeonceayear.Whenyoureachtheageofabout35yearswewillstartyearly
mammograms. Otherwise ultrasounds may be necessary depending on what is found on
examination).
' Whataboutmysisterdoessheneedteststoo?'(Sheneedstobemanagedthesame
asyou).
KEYISSUES
Abilitytointerprettheimagingtomakeacorrectdiagnosisofabreastcyst,almostcertainly
benign.
Abilitytocommunicate,inappropriatelanguage,theinvestigationfindingsofultrasoundand
FNAC,andtheirinterpretationandsignificance.
Discussiononfuturefollowupandneedforscreening.
CRITICALERRORS
Failuretoadequatelycounselthepatientaboutfibrocysticchangeandbreastcancerrisks.
Recommendingmammographyinapatientofthisage.
720
137 (T13)
PerformanceGuidelines



COMMENTARY
Fibrocysticchangeofthebreastisacommonfindinginwomenduringthereproductiveperiod.
Although less common after the menopause, breast cysts can occur at any age. Most are not
clinically apparent or relevant; they can show a cyclical pattern and are frequently seen as
incidentalfindingsonscreeningmammographyasfocalroundeddiscreteparenchymalnodules
whichoftenmeasureonlyafewmillimetresindiameter.Thediagnosisandthebenignnatureof
the nodules can be confirmed by ultrasound if required. In some instances a breast cyst can
present clinically as a well-defined palpable discrete ump with a smooth outline. In
premenopausalandyoungerwomen,suchcystsareacommoncauseofpatientspresentingwith
a breast lump. The cysts are lined by a simple epithelial layer. The fluid varies in colour from
serousyellowthoughshadesofturbidgreenandbrown('dishwater'appearance).
Cytological examination shows no evidence of malignancy and usually consists of debris and
macrophages mixed with normal parenchymal cells. Breast cysts have no demonstrated
associationwithcancerandarenotprecancerous.
Nonpalpablebreastcystsdiscoveredincidentallyusuallyrequirenoactivetreatmentotherthan
reassurance. Aspiration is used only when exclusion of cancer is a concern in view of family
historyorpatientanxiety.
Breast cysts presenting clinically as dominant palpable lumps are treated by percutaneous
aspiration.Iftypicalfluidisobtainedwithdisappearanceofthelump,andwithnorecurrenceon
followup, no further treatment is required. Surgery is reserved for those complex cysts with a
bloody aspiration fluid or with suspicious findings on cytology or with a residual lump despite
aspiration,orwithequivocalimagingfindings.
138(114)
PerformanceGuidelines
721
Condition 138 (T14)
Nocturnalhanddiscomfortina35-year-oldschoolteacher
AIMSOFSTATION
Toassessthecandidate'sabilitytodiagnoseandadvisetreatmentforcarpaltunnelsyndrome
(compressionofthemediannerveatthewrist).
EXAMINERINSTRUCTIONS
Theexaminerwillhaveinstructedthepatientasfollows:
Openingstatement
I've got these funny feelings and tingling in my right hand and arm (indicate lateral
aspectofrightforearmandhand).Itgetsworseatnight.'
Respondtothedoctorasfollows,withoutprompting:
Aboutsixweeksagoyoufeltthesefunnyfeelingsinyourrighthandandalsoapaininpartofyour
arm(indicatelateralaspectofrightforearm).Youaskedapharmacistaboutitandhesuggested
sometablets(Nurofen[ibuprofen])buttheyhavenothelped.Infactyouthinktheymayhave
madeitworse.
Inresponsetoappropriatequestioning
By 'funny feeling' you mean 'pins and needles' sensation with a feeling of numbness and
tingling.
Only the thumb, index, middle and part of the ring finger are affected. Most marked in the
pulpsofthedigits(patientsare oftenunawareoftheabsenceof pain inthelittlefinger and
knowledgeablecandidateswillseekthisinformationbydirectquestioning).
Thefeelingislessenedbyvigorousshakingofyourhand(whichyouhavedoneespeciallyat
night)
Itisnotpresent allthetime. Occursoccasionally duringtheday (especiallyafteruseofthe
hand,suchasgardeningattheweekend).
Haswokenyouduringthenightonthreeorfouroccasionsoverthelastweek.
Thesymptomsaregettingworseandyouarenowdescribingthefeelingaspainfultingling.
Youkeepinexcellenthealthandrarelyseekmedicaladvice(exceptforroutinePapsmears).
Youarenotonanyothermedicationthanibuprofen(Nurofen).
Personaldetails,familyhistoryandpastmedicalhistoryrespondasforyourself,exceptfor
occupation(schoolteacher).
The doctor may test for sensation (by pinprick or cotton wool). When this is done, you should
state that the sensation over the thumb, index, middle and ring fingers of your right hand is
reducedandyoucanonlyjustfeelthepin(incomparisonsensationinallthefingersoftheleft
hand and the right little and ring finger are normal). You will be told by the examiner how to
respondtoothertestsdonebythecandidate(TinelandPhalensigns).
138 (T14)
PerformanceGuidelines
722
Questionstoaskifnotalreadycovered:
' Why are some fingers not affected?'
' Why is it worse at night?'
' Could it be my neck?'
' Why didn t the Nurofen
0
" help?'
' Should I have any tests done?'
' Could my hand get weak?'
' What do you believe would be the best treatment?'
EXPECTATIONSOFCANDIDATEPERFORMANCE
Historyshouldinclude:
descriptionanddistributionofparaesthesia;
diurnalvariation;and
worseningfactorsnonsteroidalanti-inflammatorydrugs(NSAID),exertion.
Examination
signsofmediannervecompression
~tappingoverflexorretinaculum(Tineltest);
~maximalflexionofwrist(Phalentest)bothpositive;and
~testsforsensoryandmotordisturbanceinmediannervedistribution.
Requesturinetestforglycosuria.
Explanationofdiagnosis
Compressionofmediannerveinnarrowspaceatwrist.Maybeassociatedwithfluidretention
and with certain medications (NSAID a possible precipitant in this case) and rarely some
generalmedicalconditions(diabetes,hypothyroidism,andacromegaly).
CandidatesmaysuggestinvestigationswhichcouldincludeX-rayofcervicalspine.X-rayof
wrist,thyroidfunctiontests,fastingbloodsugarandnerveconductionstudiesThesearenot
necessaryatthisstageofmanagementinthispatientandarenotkeyissues.Theirusewill
depend on the doctor's confidence in the accuracy of diagnosis based on clinical grounds
alone. If the diagnosis is in doubt, investigations are warranted. In this case, proceeding to
management without nerve condition studies is acceptable, and possibly the preferred
course. However candidates should not be penalised if they choose to proceed with
confirmatoryinvestigationsornerveconductiontests.
Immediatemanagement
The treatment options should be briefly outlined. Cessation of NSAID should be advised.
Theseoptionsare:
~splintingofwristatnight;
~useofadiuretic;
~localcorticosteroidinjection(somerisk,usuallydonewithultrasoundguidance):and
~surgicaldecompressionbydivisionofflexorretinaculum.
Inthiscase,withtheclear-cuthistoryandexaminationfindings,thelastoption istheonlyone
whichislikelytoprovidepermanentrelief,butuseofanightsplintpriortosurgicalreferralwould
be appropriate and diagnostically helpful (failure of relief of symptoms makes the diagnosis of
carpaltunnelsyndromelesslikely).
138 (T14)
PerformanceGuidelines
KEYISSUES
Abilitytotakeafocusedhistory.
Abilitytoperformarelevantexamination.
Appropriatediagnosisandmanagementplans.
CRITICALERROR
Failuretosuggestcarpaltunnelsyndromeasmostlikelydiagnosis.
COMMENTARY
Carpal tunnel syndrome due to compression at the wrist of the median nerve under the flexor
retinaculum is the most common of the nerve entrapment syndromes. Others include the
common peroneal nerve, the lateral cutaneous nerve of the thigh, the obturator nerve, the
posterior tibial nerve and the sciatic nerve and plantar digital nerves (Morton metatarsalgia).
They can result from direct axonal compression, be secondary to vascular problems, or result
fromfluidretentionasoccurswiththeuseofNSAIDorduringpregnancy.Thecommonfactoris
increased pressure on a small segment of the nerve where it passes through a narrow space
surroundedbyrigidstructures.
Painisoftenworseatnight,perhapsbecausenocturnalfluidretentionincreasespressureonthe
nerve.
Spontaneousresolutionoccurswithsomeentrapmentsthoughlesscommonlywithcarpaltunnel
syndrome,apartfromwhenoccurringinpregnancy.Nerveconductionstudiesarediagnosticbut
are only necessary when the diagnosis is in doubt. Wrist splinting at night usually gives
symptomaticreliefandisthepreferredinitialtreatment.Itshouldbestressedthatthediagnosis
depends on the classical clinical features. Testing the power of palmar abduction by abductor
pollicisbrevisishelpful.Thenarmuscleatrophymaybeapparentinlatecases.

CONDITION138.FIGURE1.
Thenaratrophyincarpaltunnelsyndrome
Local steroid injection is potentially hazardous and certainly not indicated at this stage.
Indications for surgical decompression include persisting troublesome symptoms of
paraesthesia,orobjectivesignsofthenarmuscleatrophy.Operationisusuallycurativerisksof
median nerve injury are small but must be appropriately explained in obtaining consent for
surgery.
723
139 (T15)
724
PerformanceGuidelines

Condition 139 (T15)


Anattackofasthmaina25-year-oldman
AIMSOFSTATION
To assess the candidate's ability to manage this patient's asthma and skill in achieving
compliancewithrecommendedmanagement.

EXAMINERINSTRUCTIONS
Theexaminerwillhaveinstructedthepatientasfollows:
Youarea25-year-oldsingleautomotivespraypainterlivingwithyourparents.Sincetheageof
16,youhavehadoccasionalmildattacksofasthma.However,lastnightyouhadaverysevere
attackrequiringhospitaladmission.
Youhaverespondedverywelltotreatmentandyouwishtobedischargedtoday.Youareabout
to be reviewed by the ward doctor. You have heard from the nursing staff that the doctor will
suggestthatyoustayanotherdayinhospital.
Youarerelievedtohavehadtheattackcontrolledsopromptly,butpuzzledwhytheattackcame
on. You have not considered that exposures at work may be making your asthma worse. You
admitthatyouhadnottakenyourasthmaseriously.
Youarereluctanttostayinhospitalanylongernowthattheasthmahasbeencontrolled;andare
notconcernedaboutavoidinganotherattack.Youarenotkeentobetaking'drugs'allthetime.
Youlackknowledgeaboutasthma,itspossiblecausesanditstreatment.Howeverinresponseto
appropriate communication skills exhibited by the candidate, gradually become more
cooperative and indicate that you are prepared to take your asthma and its treatment more
seriously,andtofollowadvicegiven.
Questionstoaskunlessalreadycovered:
WhencanIgobacktowork?'
Whyhasmyasthmabecomesevererecently?' (Donotaskspontaneouslyifyourwork
maybecausingyourasthmaormakingitworse).
' Areallthesemedicinesnecessary?'
Dotheyhaveanysideeffects?'
' WhatshouldIdoiftheasthmacomesonagain?'
' ShouldIseeanasthmaspecialist?'
EXPECTATIONSOFCANDIDATEPERFORMANCE
Thecandidatewouldbeexpectedtocovermostofthefollowingpoints:
Ascertainthepatient'sunderstandingofasthma.
Asthma is a longterm problem but can be controlled by correct treatment. It can be
life-threatening.Summarisepossiblecauses,includingoccupation.Explaineffectonairways.
Willneedfurtherdiscussionatfollowupconsultations.
139 (T15)
PerformanceGuidelines
725
Adviseaboutimmediatemanagement:Prednisoloneorally30-50mgdailyovernext5-7days,
thenstop(taperingalsoappropriatebutnotessential).Regularuseofinhaledcorticosteroid
(QVAR,PulmicortorFlixotide);anduseofinhaled
2
-agonist(salbutamolorterbutaline)
asrequired.Discusstheuseofaerosolandspacerifaerosoltechniqueinadequate.
Adviseself-monitoringwithpeakflowmeter.Provideanasthmaactionplanasillustrated.
Adviseaboutfuturemanagement.
Responsibilityofpatientformonitoringcondition.
Educationoffamily.
ContactwithAsthmaFoundation.
Referral to allergist or respiratory physician or occupational health physician through local
doctor(checkingoccupationalcause).Followupbylocaldoctorwithin48hours.
Returntoworkaccordingtoprogress.
Useofeffectivecommunicationskillsincounsellingthispatient:thecandidateshouldimpress
uponthepatientthepotentiallylethalnatureofsevereasthmaandthatmostfatalitiesresults
from inadequate or delayed management rather than overuse of aerosols in the acute
situation. This advice needs to be tempered in order to avoid engendering undue anxiety.
Keepterminologysimple.Assistpatienttolearnbycarefulexplanation.
Candidatesshouldencouragequestionsandcheckunderstanding;and demonstrateuseof
devicesappropriately.
Avoidoverloadingpatientwithinformation.Stresstreatmentpriorities.Displayconcerneven
thoughthisisthefirstsevereattack.Reassurethatcontrolisreadilyachievablebutindicate
potentialforseriousoutcome.
Focusoncompliance.Avoidconfrontation.


EXAMPLESOFSIMPLEASTHMAACTIONPLANS
PEAKFLOWBASEDPLAN
Bestpeakflowis600(example)
Ifpeakflowis
Below480(or80%ofbest)doubledoseofinhaledbronchodilatorandmeasure
peakflowevery-1hour.
Below360(or60%ofbest)startprednisolonewith25mgdoseandcontact
doctorurgently
Below240(or40%ofbest)continuebronchodilatoranddial000for
ambulance.
Say'severeasthmaattack.'Whilstwaiting,use
bronchodilatorcontinuously.
IFAMBULANCEISDELAYEDGETSOMEONETODRIVEYOUTO
THENEARESTHOSPITAL
139 (T15)
PerformanceGuidelines
726
KEYISSUES
Approachtopatientavoidconfrontationandensurecompliance.
Initialmanagementplanknowledgeofappropriatemedicationanditsdelivery.
Patientcounsellinguseofpeakflowmeterandasthmaactionplan.
CRITICALERRORS
Failuretorecommendcontinuinguseofanaerosolsteroid.
Failure to consider that occupation may be causing or aggravating the patient's asthma.
COMMENTARY
Definition: Asthma is a chronic inflammatory disorder of the airways in which many cells and
cellularelementsplayarole,inparticular,mastcells,eosinophils,T-lymphocytes.macrophages,
neutrophilsandepithelialcells.
Commonclinicalfeaturesinsusceptibleindividualsinclude:
Recurrentepisodesofwheezing,breathlessness,chesttightnessandcoughingparticularlyat
nightorearlyinthemorning.
Episodesareusuallyassociatedwithwidespreadbutvariableairflowobstructionthatisoften
reversibleeitherspontaneouslyorwithtreatment.
Inflammationalsocausesanassociatedincreaseintheexistingbronchial
hyper-responsivenesstoavarietyofstimuli.
Asthmaisalife-threateningbutpreventabledisease.
AsthmaisalsoacommondiseaseinEmergencyDepartmentsandthegeneralcommunity,being
amongstthetoptenconditionsseenbygeneralpractitioners.
Severe life-threatening attacks are more likely to occur in patients with inadequate medical
supervision
Theseverityofairflowobstructionwillbeoftenunderestimatedunlesslungfunctionismeasured
regularly.
Earlyinterventionisthebeststrategytorelieveanasthmaattackandpreventdeterioration.
Allpeoplewithasthmashouldhaveashort-acting
2
-agonistforsymptomreliefandbeinstructed
aboutitsuseastheirownguidetocurrentcontrol.
Commencepreventivetherapyifthepatientrequires
2
-agonistrelievermorethan3-4timesper
week,excludingthattakenbeforeexercise.
139(T15)
PerformanceGuidelines
727
Peoplewithasthmashouldbeencouragedto:
takecontinuingresponsibilityfortheirmedication;
makeappropriatechangeswhennecessary;
contacttheirdoctor/pharmacist/asthmaeducatoriftheyhaveconcernsorqueriesregarding
theirasthmamanagement;
attendforregularreview;
understandthedifferentrolesoftheirrelieverandpreventivemedications,andsymptom
controllersifprescribed;
bringtheirinhalerdevicetotheconsultationsothattheirinhalertechniquecanbechecked;
and
understandthebasicpathophysiologyandnaturalhistoryofasthma.
Allergyisanimportantcauseofasthmainadultsandchildren.
patientsshouldbeencouragedtoceasesmoking;
educationwithdrugtherapyandaneffectivetreatmentplanreducemorbidityandmortality;
and
poorlycontrolledasthmarestrictsparticipationinnormalphysicalandsocialactivities.
140 (T16)
Performance Guidelines
728

Condition 140 (T16)
Preparing a 30-year-old woman with suspected acute appendicitis for surgery
AIMS OF STATION
To assess the candidate's ability to manage a patient with suspected acute appendicitis, and
prepare her for surgery.
EXAMINER INSTRUCTIONS
The examiner will have instructed the patient as follows:
You will be lying on a hospital couch wearing a hospital gown.
You have had worsening abdominal pain with vomiting. The pain woke you up six hours ago and
is getting worse. You are still in severe pain and choose to lie still on the couch to minimise the
pain. You are very tender in the right hand side of your abdomen. You were brought to this
hospital and have just been seen and examined by the duty doctor, who has just returned to see
you after checking an examination of your urine and speaking to a surgical consultant by
telephone.
You are apprehensive. You have never had to face a serious illness and have never considered
that you might need surgery. You wish to know if there is any alternative treatment. The doctor is
expected to discuss a number of matters with you these are detailed in the section
Expectations of Candidate Performance
Opening statement:
'Please tell me what the problem Is.'
Questions to ask unless already covered:
'Will this be done by keyhole surgery?'
'Don t you need to X-ray to diagnose the cause?'
'What if it's not my appendix?'
'Isn t surgery a risky undertaking?'
'When will I be able to go home?'
'How long will I be away from work?'
140 (T16)
729
Performance Guidelines



EXPECTATIONS OF CANDIDATE PERFORMANCE
The candidate should explain the following information to the patient:
The diagnosis and its implications (left untreated, the risk of perforation).
The high probability of diagnosis (high clinical specificity of her signs).
Other possible causes for the illness (particularly gynaecologic ectopic pregnancy, other
tubal-ovarian causes).
The candidate may indicate that additional tests could assist in the diagnosis, but in this
scenario diagnosis should be made on clinical grounds without requirements for further
diagnostic tests.
The recommended treatment (appendicectomy).
Risks of the procedure (anaesthetic, laparoscopy or laparotomy).
Potential complications (particularly infection).
Likely postoperative course (analgesic requirements, length of stay in hospital, time off work).
Should at least mention anaesthetic risks, intraoperative complications, wound infection
which are individually and cumulatively small in this situation.
The candidate should be able to show appropriate communication skills and provide
information in a reassuring manner about the condition, reasons for the proposed treatment
and what actually will be done.

KEY ISSUES
Counselling the patient regarding preparation for appendicectomy.

CRITICAL ERROR
Failure to adequately explain the indications for and potential complications of surgery.
140 (T16)
Performance Guidelines
730
COMMENTARY
Appendicitis is common: one in seven individuals in Australia will develop the condition. For most
sufferers, the diagnosis will be readily evident on clinical grounds and they will proceed through
an uncomplicated appendicectomy. Appendicitis can occur at any age and has its greatest
morbidity amongst the very young and the very old. In the former group, the diagnosis is difficult
to make and is often made late, and in the latter group, comorbidities compound the acute
abdominal problem. The natural history of appendicitis is that the disease will progress to
perforation if untreated in 30% of cases. The remainder will resolve after a bout of acute
inflammation. The practical problem facing the clinician is deciding which patients will proceed to
perforation and this is not possible to predict with any degree of certainty. In other words, once
a confident diagnosis of appendicitis has been made, the patient should be advised to undergo
appendicectomy.
If the clinician has a high index of suspicion of appendicitis on clinical grounds, there is little need
for any investigation. It the diagnosis is in doubt, or there are other important conditions to
exclude (e.g. ectopic pregnancy), it may be prudent to investigate further. A white cell count is
occasionally helpful. A pregnancy test may be appropriate and in some circumstances imaging
may be indicated. Ultrasound and computed tomography are fallible and should only be
interpreted in context with the clinical condition.
Once a decision has been made that the patient should proceed to appendicectomy, informed
consent must be obtained. This involves explaining the nature of the proposed treatment to the
patient. This is best done by prefacing the discussion with an explanation of the natural history of
appendicitis. The explanation must be given in plain language and the patient given the
opportunity to ask any questions that she might have. Any potential side effects and
complications of the procedure must be described. The patient with appendicitis should be
reassured that for most patients the procedure is uneventful and normal activities will be resumed
within a week or two of leaving hospital. Appendicectomy may be performed either by open
operation or laparoscopy, the choice usually resting with the surgeon's preference and
sometimes the patient's wishes. The laparoscopic approach is favoured by most surgeons for
most patients, but in the long term, there is little difference in outcome between the two
procedures. The laparoscopic approach will allow a better view of the abdominal cavity and this
can be advantageous where there is a degree of diagnostic doubt such as in young women of
childbearing age.
The risks to be discussed include those related to the anaesthetic, laparoscopy and surgery. The
risks include infection (chest, wound, intra-abdominal), perforation of aviscus and venous
thromboembolism. In a young, fit individual, such as the woman in this case, the risks must be put
in perspective and the patient reassured that the chances of complications are low. The risks will
be reduced by the use of prophylactic antibiotics, calf compression devices and early
mobilisation. Most patients will be mobile the day after the procedure, and home once gut
function has returned.
731
MCAT
Trial Examinations Two Papers



RETEST TRIAL EXAMINATION

I should have liked to be asked to say what I know. They always tried to ask what I did
not know. I did not do well in examinations.'
Winston S Churchill (1874-1965) My Early
Life, Collins (1930)

R1-R8 (Conditions 141-148)
This is an 8 active-station clinical examination, with 2 additional rest stations. There is:
1 Obstetric/Gynaecology station,
1 Paediatric station,
1 Psychiatry station,
3 Medical and
1 Surgical stations.
To pass the retest trial, candidates must pass 6 of the 8 active stations, but there is no
requirement to pass specific individual discipline stations.
732
MCAT
Trial Examinations Two Papers



Retest Trial Examination

Candidate Information and Tasks

MCAT 141-148 (R1-R8)
141 [R1] Intravenous cannula insertion for antibiotic prophylaxis
142 [R2] Heartburn in a 35-year-old man
143 [R3] Spontaneous bruising and nosebleed in a 3-year-old boy
144 [R4] Nausea and vomiting in the first trimester in a 25-year-old primigravida
145 [R5] Visual difficulties in a 50-year-old man
146 [R6] Cognitive state assessment of a 50-year-old barman
147 [R7] Jaundice in a 25-year-old man
148 [R8] Assessment of prominent leg veins in a 38-year-old woman

141 (R1)
Candidate Information and Tasks
733

Condition 141 (R1)
Intravenous cannula insertion for antibiotic prophylaxis
CANDIDATE INFORMATION AND TASKS
You are working in a hospital primary care clinic. Your next patient is to have intravenous
antibiotics for prophylaxis against endocarditis. He has a prosthetic heart valve and is about to
have a dental procedure.
YOUR TASKS ARE TO:
Advise the patient of the forthcoming procedure to obtain informed consent.
Make an appropriate selection from the equipment on the table.
Demonstrate how you would insert an indwelling intravenous cannula in the forearm or
hand up to the point of insertion; BUT DO NOT INSERT THE CANNULA OR NEEDLE
AND DO NOT REMOVE ANY OF THE PLASTIC COVERINGS.
Describe to the examiner what you are doing and why as you proceed.
The Performance Guidelines for Condition 141 (R1) can be found on page 741

142 (R2)
Candidate Information and Tasks
Condition 142 (R2)
Heartburn in a 35-year-old man

CANDIDATE INFORMATION AND TASKS
You are working as a Hospital Medical Officer (HMO) in a country base hospital. Your next
patient is a 35-year-old man who consulted you recently because of worsening heartburn.
The symptoms have been most severe in bed at night when there has been occasional water
brash' (regurgitation of bitter fluid into the mouth) and bouts of dry coughing. A drink of milk or
Mylanta(aluminium hydroxide and magnesium hydroxide) gives some relief.
The patient is overweight (BMI 28 kg/m
2
) but otherwise well. You arranged for upper
gastrointestinal endoscopy which showed a small sliding hiatus hernia and oesophagitis with
some moderately severe ulceration above the gastro-oesophageal junction as demonstrated in
the illustration below. A helicobacter breath test was also done and proved negative. The
specialist advised there was no evidence of malignancy nor of another peptic ulcer, and that the
symptoms were due to reflux oesophagitis. The patient was advised to see you for further advice
and treatment.
Brief profile of this patient Married. Has one daughter. All of his working life has been spent in
the police force. Smokes 15 cigarettes per day. Uses alcohol at weekends when off-duty.
Occasionally drinks to excess (mild intoxication). Only drinks beer.
Past History Mild migraine 3 or 4 times a year for which he uses aspirin.
Family history father died of pancreatic cancer, aged 61 years.
YOUR TASKS ARE TO:
Discuss the results of the endoscopy with the patient.
Advise on management.
Answer any questions the patient may ask.

There is no need to take any further history from the patient, or perform any examination.
CONDITION142 (R2). FIGURE 1.
The Performance Guidelines for Condition 142 (R2) can be found on page 744
734
143 (R3)
Candidate Information and Tasks
Condition 143 (R3)
Spontaneous bruising and nosebleed in a 3-year-old boy

CANDIDATE INFORMATION AND TASKS
You are working as an intern Hospital Medical Officer (HMO) in a country base hospital. Robert, a
3-year-old boy, who had a cold two weeks ago, from which he has recovered, has come to see
you for review. Over the last few days his parents have noticed widespread unusual bruising of
his skin. He had a small nosebleed yesterday and he was brought to see you then.
Brief inspection showed an alert well developed 3-year-old. Numerous fresh bruises were noted
on the trunk and limbs and on his hand as illustrated below; and scattered petechiae were noted
between the bruises, maximal on lower limbs. Vital signs were normal. You found no
hepatomegaly or splenomegaly and no lymphadenopathy. You arranged for an urgent full blood
examination (FBE) and he and his mother have returned today to discuss the results, which are
shown below.

YOUR TASK IS TO:
Outline your provisional diagnosis and discuss your plan of management with the parent.

CONDITION 143 (R3). FIGURE 1.
Bruising of hand

REPORT: Full Blood Examination
Haemoglobin: 125 g/L (100-145 g/L)
No abnormality of the red cell series is seen
White cell count: Total Count 11.2 x 109/L (7-14)
Differential count normal for age
No immature white cells seen: white cell morphology normal
Platelets: 15 x 109/L (160-250 x 109)
Comment: the platelet count is greatly reduced,

The Performance Guidelines for Condition 143 (R3) can be found on page 748
735
144 (R4)-145 (R5)
Candidate Information and Tasks
736
Condition 144 (R4)
Nausea and vomiting in the first trimester in a 38-year-old primigravida

CANDIDATE INFORMATION AND TASKS
Your next patient is a 38-year-old woman who has come to the general practice because of
severe nausea and vomiting for the last two weeks in this, her first pregnancy. She claims that
she has been unable to keep foods or fluids down. Her last menstrual period was eight weeks
previously, and pelvic examination by your colleague in the general practice two weeks ago
showed the uterine size was appropriate for gestation and a pregnancy test was positive. She
has had no previous operations or illnesses.

YOUR TASKS ARE TO:
Take any further relevant history you require.
Ask the examiner about relevant findings evident on general and obstetric examination
which would assist you in making a diagnosis.
Advise the patient of the likely diagnosis.
Advise the patient of the care you would advise for her, including any investigations you
would arrange.
The Performance Guidelines for Condition 144 (R4) can be found on page 750
Condition 145 (R5)
Visual difficulties in a 50-year-old man

CANDIDATE INFORMATION AND TASKS
Your 50-year-old patient presents to you at the hospital Emergency Department with a nine
month history of finding it difficult, while driving, to see cars approaching from the sides at
intersections. He has also noted bumping into things.
YOUR TASKS ARE TO:
Assess visual acuity.
Assess the eye movements.
Assess the visual fields.
Tell the examiner what investigations should be done for this patient.
Describe your findings to the observing examiner as you proceed. You can talk to the
patient and give instructions as you examine him but this is an examination station and
further history is not expected.
Indicate the most likely cause of any abnormality you define.
The Performance Guidelines for Condition 145 (R5) can be found on page 753
146 (R6)
Candidate Information and Tasks
737
Condition 146 (R6)
Cogniti ve state assessment of a 50-year-old barman

CANDIDATE INFORMATION AND TASKS
This 50-year-old man works as a barman and is attending an appointment at a general practice,
which you arranged after a conversation with his wife. He has a history of consumption of up to 10
standard alcoholic drinks on most days, over the last 10 years.
In attempting to contact the patient at home, you spoke with his wife who said that the patient has
been quite forgetful and unreliable for some months, causing family and work colleagues to be
irritated and concerned.
The patient is in your consulting room. You have completed your history and you are now
proceeding to test his cognitive function.

YOUR TASKS ARE TO:
Examine the cognitive state using the modified Folstein Mini-Mental State Examination
(MMSE) provided.
Explain to the patient what you are going to do and why.
Summarise for the examiner, the normal and abnormal MMSE findings.
Tell the examiner your interpretation of the results, including what condition these results
signify.

The Performance Guidelines for Condition 146 (R6) can be found on page 756
147 (R7)
Candidate Information and Tasks
738

Condition 147 (R7)
Jaundice in a 25-year-old man

CANDIDATE INFORMATION AND TASKS
You are working in a medical outpatient clinic. Your next patient is a single 25-year-old sales
assistant who has consulted you about general malaise, anorexia, nausea and fever gradually
worsening over the last 10 days. Yesterday he noted dark urine and pale stools.
On examination you have found icteric conjunctivae, a temperature of 38.8 C and tender
enlargement of the liver. Urinalysis was positive for bilirubin and urobilinogen. There are no other
abnormal findings on examination.
YOUR TASKS ARE TO:
Take any further relevant history you require.
Tell the examiner what investigations you would arrange.
Explain the nature of the illness and possible cause to the patient, indicating which you
consider to be the most likely cause.
Advise the patient concerning immediate management.

The Performance Guidelines for Condition 147 (R7) can be found on page 760
148 (R8)
Candidate Information and Tasks
Condition 148 (R8)
Assessment of prominent leg veins in a 38-year-old woman
CANDIDATE INFORMATION AND TASKS
You are a hospital intern admitting a patient for varicose vein surgery tomorrow. She is a 38-year-old woman with
visible venous varicosities, who has had progressively increasing leg aching and heaviness over the past two years
She has five children aged from 6 to 19 years. She had a tubal ligation five years ago. She has been in good health
with no other relevant or significant past history.
You do not need to take any further history. The appearance of her legs on standing is as illustrated below.

YOUR TASKS ARE TO:
Perform a focused physical examination to assess and document the venous circulation status in the left lower
limb.
Give a running commentary to the examiner as you proceed.
CONDITION148 (R8). FIGURE 1. CONDITION148 (R8). FIGURE 2.
The Performance Guidelines for Condition 148 (R8) can be found on page 763
739
740
MCAT
Trial Examinations Two Papers



Retest Trial Examination

Performance Guidelines

MCAT 141-148 (R1-R8)
141 [R1] Intravenous cannula insertion for antibiotic prophylaxis
142 [R2] Heartburn in a 35-year-old man
143 [R3] Spontaneous bruising and nosebleed in a 3-year-old boy
144 [R4] Nausea and vomiting in the first trimester in a 25-year-old primigravida
145 [R5] Visual difficulties in a 50-year-old man
146 [R6] Cognitive state assessment of a 50-year-old barman
147 [R7] Jaundice in a 25-year-old man
148 [R8] Assessment of prominent leg veins in a 38-year-old woman

141 (R1)
Performance Guidelines
Condition 141 (R1)
Intravenous cannula insertion for antibiotic prophylaxis
AIMS OF STATION
To assess the candidate's performance of the procedure of insertion of an intravenous cannula
for antibiotic prophylaxis using appropriate equipment
EXAMINER INSTRUCTIONS
The examiner will have instructed the patient as follows:
The patient is wearing short sleeves, and is lying on a couch
Candidates are asked only to DEMONSTRATE how to insert an indwelling intravenous cannula
in the forearm or hand UP TO THE POINT of insertion the needle will NOT be inserted and will
not be removed from its coverings.
Candidates have been told to describe to the examiner what they are doing and why as they
proceed.
EXPECTATIONS OF CANDIDATE PERFORMANCE
Ability to explain the procedure of intravenous cannulation to the patient and obtain consent
for the procedure.
Knowledge of appropriate equipment.
Knowledge of appropriate technique.
Competence in performance of the procedure.
Candidates would be expected to: explain to the patient what they are about to do and obtain
consent (verbal); choose an appropriate intravenous cannula (20 or 22 gauge): wash hands and
use gloves; use antiseptic skin preparation appropriately; choose a suitable vein in the upper
limb; describe the procedure satisfactorily to the examiner. Some candidates may use local
anaesthetic but this is not mandatory.
Correct equipment to be selected from various items (Figures 1 and 2):

CONDITION 141 (R1) FIGURE 1 CONDITION 141 (R1). FIGURE 2.
Box of gloves (nonsterile)
Intravenous cannulae Pink 20 GA, Blue 22 GA (J elco, Dwell-Cath)
Chlorhexidine swab stick
Baxter interlink injection site
741
141 (R1)
742
Performance Guidelines
Local anaesthetic (1% Xylocaine) ampoule
BD Ultra-fine insulin syringe 1 mL
Sodium Chloride injection BP 0.9% (Saline)
10 mL latex free syringe
Blunt plastic cannula
Tourniquet
Sharps container
Skin cleansing swab (Alcowipes)
Cotton wool balls
Transparent dressing with label

KEY ISSUES
Explanation of procedure to patient with verbal consent.
Appropriate knowledge and proficiency of the technique of intravenous cannulation at
undergraduate level.

CRITICAL ERRORS
Failure to explain procedure adequately in obtaining verbal consent.
Failure to select appropriate equipment.

COMMENTARY
Cannulae and needles for intravenous therapy are now almost always inserted percutaneously.
The technique of venous cannulation first requires choice and preparation of the site.
The best sites are where visible or palpable subcutaneous veins are present which can be
needled without risk to any neighbouring important structures liable to inadvertent puncture. Thus
the dorsum of the hand is ideal, or the cephalic vein on the radial side of the forearm just above
the wrist. The median cubital and basilic veins at the elbow are prominent and readily accessible,
but are much less desirable choices because of the very occasional (but real), risk of inadvertent
needling (or even worse, of infusing) the underlying brachial artery or median nerve.
Once the site is selected, the veins should, if necessary, be rendered more prominent by
warming or by application of a venous tourniquet. The desired equipment is selected, the skin
prepared by application of an alcohol wipe to deal with vegetative skin organisms, and
needle/cannula insertion performed wearing gloves and using a suitable technique.
Intradermal injection of a bleb of local anaesthesia (1% Xylocaine) is optional. In children or in
apprehensive patients application of local anaesthetic cream or spray can be helpful.
Simple butterfly needles are suitable for intravenous access for drugs and for administering
single dose brief injections.
141 (R1)
Performance Guidelines
743

Otherwise insertion of a plastic cannula of inert material (Teflon, silastic) is preferred to
minimise dangers of extravasation of infusion fluids by inadequate fixation of needles allowing
laceration of the vein wall from movement during the infusion period. 'Cannula-over-needle' is the
most widely used type of cannula many proprietary preparations are available (J elco,
Dwell-Cath, etc). The needle is removed after successful insertion and the cannula in the vein
remains flexible and pliant at body temperature. Insertion may be done with the needle and
cannula initially attached to a syringe, but in experienced hands this is not usually necessary.
The alternative technique of 'cannula-through-needle' is usually reserved for insertion of
cannulae at central sites like insertion of a central venous cannula. Whatever technique is used, a
simple transparent adhesive dressing maintains the cannula in place while the infusion is
completed.
Most antibiotics are irritant to vein endothelium and are given diluted in a buffered iso-osmolar
solution to minimise risks of chemical thrombophlebitis.
142 (R2)
Performance Guidelines
744
Condition 142 (R2)
Heartburn in a 35-year-old man
AIMS OF STATION
To assess the candidate's knowledge of the management of gastro-oesophageal reflux and skills
in motivating a patient to achieve lifestyle change.
EXAMINER INSTRUCTIONS
The examiner will have instructed the patient as follows:
You consulted this doctor recently because of worsening 'heartburn' (pain behind the lower
breast-bone) over recent months. The symptoms have been worse in bed at night when
recumbent and you have occasionally had regurgitation of bitter fluid into your mouth and bouts
of dry coughing. You get some relief by drinking a glass of milk or taking a dose of Mylanta
(aluminium hydroxide and magnesium hydroxide).
You are otherwise well and have no other symptoms. You are overweight. The doctor arranged
for endoscopic examination of your gullet and stomach (as a day patient) and you have returned
to find out more about treatment.
You have also had a breath test for a possible infection in your stomach which was negative. The
specialist told you that he found a hiatus hernia, reflux and inflammation which has caused the
pain but nothing more serious. Your personal details, which are known to the doctor, are as
follows: You are married with an 8-year-old daughter. You smoke 15 cigarettes per day. You
drink on weekends. Occasionally you have a little too much (beer) with mild intoxication
Your father died of pancreatic cancer aged 61 years. You have no history of serious illnesses or
operations. You have mild migraine three or four times a year for which you take aspirin. You
were an active sportsman but not now. You still go to the football. Gardening is your other main
activity.
You should listen carefully to the doctor's explanation but interrupt if there are confusing
statements or if inappropriate medical jargon is used. Your main concern is to get rid of your
symptoms. You have not been concerned about a serious cause for your symptoms, until now.
Display some anxiety if there is emphasis on the most serious complications (malignant change,
oesophageal stricture or rupture). If the doctor starts to discuss alternative treatments for your
migraine in any detail, indicate that this is not a significant problem and that you will be able to
cope without aspirin. If the doctor's approach is satisfactory from your point of view, then agree to
follow the advice about behavioural change. Otherwise be resistant to the doctor's advice about
weight, smoking and alcohol use.
142 (R2)
Performance Guidelines
745
Questions to ask unless already covered:
'Why have I got reflux?'
'Does this mean an operation?'
'Do I need any further tests?'
'Can this develop into cancer?'
'What else can be done to help it?'(\i lifestyle issues such as weight loss are not raised)
'What about alcohol?'
'Will I have to have another of these examinations ('endoscopy,)?'
Is cancer inherited?'
EXPECTATIONS OF CANDIDATE PERFORMANCE
Approach to patient the doctor's main objective should be to have the patient stop smoking,
lose weight, modify his use of alcohol and accept responsibility for these lifestyle changes.
The doctor's approach should be supportive and nonthreatening.
Explanation of condition reassurance that the condition is not serious and is reversible at
this stage. No relationship to his father's cancer.
Description of anatomy of upper gastrointestinal tract explanation of terms 'reflux', 'hiatus
hernia', 'oesophagitis', and mechanism of reflux.
Cause of symptoms (oesophagitis) association with hiatus hernia which is not causative.
Reference to complications which may occur ulceration, bleeding, stricture, rupture,
malignant change (Barrett oesophagitis). Use of diagram would assist.
Immediate management prescription of proton pump inhibitor (e.g. Losec[omeprazole],
Nexium[esomeprazole]), taken at night. The patient should also be given lifestyle advice
along the following lines: Advise
~smoking cessation;
~weight reduction, and moderation in use of alcohol. Only have a light meal in evening (avoid
late bedtime meals), avoid spicy foods, wine (especially red wine), coffee and chocolates;
~trial of elevation of head of bed or use of several pillows: and
~ cease use of aspirin. Other nonsteroidal anti-inflammatory drugs (NSAID) should be
avoided.
Discuss alternative treatment for migraine (details not required). Reduce coffee, fatty foods
which reduce competence of oesophageal sphincter. The role of surgery could also be
mentioned but is not an option at this stage. A repeat endoscopic examination will be required
in followup.
If the candidate attempts to use the time for discussion of smoking cessation or weight loss in
detail, the examiners will interrupt and say these are appropriate but could be dealt with at a
further consultation.
142 (R2)
Performance Guidelines

KEY ISSUES
Approach to patient ability to achieve compliance by explanation and support.
Interpretation of investigations symptoms are caused by the reflux, not the hiatus hernia per se.
Initial management plan use of proton pump inhibitor.
Patient counselling and education nature of reflux and future course.
CRITICAL ERROR
Failure to prescribe treatment which will reduce gastric acid secretion.

COMMENTARY
Gastro-oesophageal reflux is common and most people will experience 'heartburn' and regurgitation
at some time. For most individuals these episodes are a physiological event and do not warrant
therapy or investigations. For those who get frequent reflux, over-the-counter medications (antacids
or H2-receptor antagonists) may be all that is required in terms of management. If the patient has
symptoms of such severity that medical advice has been sought, then a decision must be made on
whether any investigations are required. If there are obvious precipitating factors such as obesity,
alcohol or cigarette abuse, then advice on lifestyle matters and empirical therapy (a proton-pump
inhibitor or H2-receptor antagonist) may be all that is required.
For individuals with persistent symptoms, or symptoms resistant to therapy or those with 'alarm'
symptoms, investigations need to be considered. Alarm symptoms include dysphagia, haematemesis
and weight loss. The preferable investigation is endoscopy, which will allow a thorough inspection of
the mucosal aspect of the upper digestive tract and biopsy of any suspicious lesion. If the patient's
problem is thought to be related to an oesophageal hiatus hernia, a contrast study may provide
additional information. In large, complex hernias (para-oesophageal), a contrast study may be more
helpful than endoscopic examination.
The finding of a 'hiatus hernia' per se does not equate with symptomatic reflux and it is probable that
most hiatus hernias do not cause symptoms. Similarly, many patients with symptomatic and
demonstrable reflux do not have any anatomical defect at the hiatus.
Frequently, patients will present with a clear history of gastro-oesophageal reflux, yet have no
abnormalities on endoscopy. These patients may need to have their pH profile studied. This is the
gold standard of measurement in reflux disease. A fine bore probe is introduced into the lower
oesophagus and the patient encouraged to go about usual activities over the next 24 hours. The pH
probe is attached to a constant monitoring device and the patient is asked to document any episodes
of 'reflux' that occur during the study. This allows subsequent correlation of symptoms with episodes
of reduction of pH in the lower oesophagus. A total fall in pH to below 4.0 for more than 4% of the 24
hours is considered pathological. A pH study is usually combined with manometric assessment. This
will document the peristaltic activity of the oesophagus, the lower oesophageal sphincter pressure
and function, and the pattern and rate of clearance of acid from the oesophagus.
746
142 (R2)
Performance Guidelines
747
Provided no abnormalities such as a columnar-lined epithelium have been detected on
endoscopy and the patient responds well to lifestyle adjustment and any prescribed medical
therapy, then no further action is required. Perhaps 10% of individuals with reflux will have some
form of mucosal abnormality detected at endoscopy. If there is a columnar-lined epithelium, this
must be carefully documented in terms of the extent of the abnormal epithelium and any
histological changes present. Any dysplastic changes must be documented and the degree of
surveillance tailored to the severity of the dysplasia. High grade dysplasia can merge into in-situ
carcinoma and these individuals require close monitoring and surgical consultation.
Uncomplicated Barrett oesophagus with no metaplastic mucosal changes needs no more than
biennial endoscopic examination
For longterm management of symptomatic gastro-oesophageal reflux, a 'step-down therapy is
frequently recommended. Many patients may require a proton-pump inhibitor to gain control of
their symptoms, but this therapy does not necessarily need to be maintained and therapy can
often be stepped-down after a few weeks to a lower dose of the medication or a change to an
H2-receptor antagonist. Likewise, if the step-down approach results in symptoms reappearing,
the medications can be modified in type or dosage.
Few individuals with gastro-oesophageal reflux disease need to be considered for surgery.
Those that do include patients whose symptoms are resistant to medical therapy, those who
have troublesome side effects from the medications (e.g. diarrhoea), those with large anatomical
defects (a para-oesophageal hernia), those with complications (ulceration or stricture formation)
and those who do so for personal choice. Regurgitation (with or without respiratory
complications) is a frequent indication for fundoplication, as medical therapy offers little help in
the presence of gross reflux. At one stage it was thought that successful surgery would help with
the regression of abnormal epithelium in Barrett oesophagus, but such regression rarely occurs
with either medical or surgical therapy. As with any procedure, patients contemplating surgery for
gastro-oesophageal reflux disease need to be fully conversant with the side effects and potential
complications. The side effects include bloating, early satiety and a reduced ability to belch or
burp.
143 (R3)
Performance Guidelines
748
Condition 143 (R3)
Spontaneous bruising and nosebleed in a 3-year-old boy
AIMS OF STATION
To assess the candidate's knowledge about the diagnosis of acute thrombocytopenia of
childhood based on the history and laboratory findings, and ability to outline an appropriate plan
of treatment.
EXAMINER INSTRUCTIONS
The examiner will have instructed the parent as follows:
You are the mother of 3-year-old Robert who has developed bruising after a recent cold which is
caused by a reaction to the virus he had which has affected the platelets in his blood. This has
caused him to have the spontaneous bruising you have noticed. He may or may not need active
treatment which will be explained to you by the doctor.
You are a very anxious parent and are worried about Robert possibly having leukaemia.
Robert has been a very well child and seems well now except for the bruising. You are frightened
by the appearance of the bruising.
Questions you must ask unless already covered:
'How does this condition occur?'
Why has he got so much bruising?'
Is this contagious?'
What treatment is there for this?'
Will he get this again?'
EXPECTATIONS OF CANDIDATE PERFORMANCE
The candidate should:
Explain diagnosis of acute thrombocytopenia, as a probable reaction to virus infection with the
production of cross-reacting antibodies against platelets.
Arrange hospital admission for assessment, as there is risk of spontaneous haemorrhage in
the early phase of the disease.
Explain that immediate treatment is generally not required because of the probability of
spontaneous remission of disease, but will be commenced if progress is unsatisfactory
Indicate that bone marrow examination is usually not performed unless there are features
which suggest significant underlying disease.
Advise that treatment (intravenous gamma globulin or steroids) is available if bleeding occurs.
Explain that the prognosis is good, with over 90% resolution (75% having a spontaneous
remission over a four to six week period). In a further 10 to 15%, remission occurs in up to six
months. About 10% of children develop chronic thrombocytopenia, which maybe helped by
splenectomy (in over 90% of these children). While this condition can resolve and then
subsequently recur, this would be very unusual: and would indicate investigations looking for
an underlying autoimmune cause, such as lupus.
143 (R3)
749
Performance Guidelines



KEY ISSUE
Explanation of diagnosis of acute thrombocytopenia and its management.

CRITICAL ERRORS
Making a diagnosis of leukaemia on the basis of the blood test.
Indicating a poor prognosis for the condition.

COMMENTARY
This scenario assesses the knowledge of a candidate in relation to the condition of isoimmune
thrombocytopenia of childhood. The candidate is expected to interpret the pathology result
accurately and recognise that the only abnormality is the low platelet count hence making more
sinister pathology very unlikely. An adequate discussion of the possible causes and the methods
of treatment should follow. The candidate should be able to reassure the parent that there is a
very unlikely possibility of any other pathology and should also be able to indicate that the natural
history of this condition is of spontaneous remission and resolution in over 90% of cases. While
this condition can resolve and then subsequently recur, this is unusual and should indicate a
need for further investigation to exclude conditions like lupus.
Hospital admission is only advised if the platelet count is very low and the child is actively
bleeding; or when parental concern about complications or alternative diagnosis is high, as in this
case.
The candidate is also assessed on the manner of reassurance given to the parent of her worry
that her child has leukaemia.
144 (R4)
Performance Guidelines
750
Condition 144 (R4)
Nausea and vomiting in the first trimester in a 38-year-old primigravida

AIMS OF STATION
To assess the candidate's knowledge about the diagnosis and management of hyperemesis
gravidarum.

EXAMINER INSTRUCTIONS
You are 38 years old and have had severe nausea and vomiting for the last two weeks. You are
now eight weeks pregnant in this, your first pregnancy.
The examiner will have instructed the patient as follows:
Appropriate answers to likely questions are:
No symptoms of urinary frequency, or pain with voiding. You need to get up once a night to
empty your bladder.
No vaginal bleeding.
Urine output satisfactory and apparently normal.
You have kept some fluids down today but have had difficulty keeping any fluids or food down
for the last week or two.
No family history of multiple pregnancy.
This is a spontaneous pregnancy and fertility drugs have never been used. You have been
trying to conceive for the last 12 months.
You are aware of the increased risk of Down syndrome due to your age, and will want tests
done to ensure this baby does not have Down syndrome. If it did, you would wish the
pregnancy terminated.
Questions to ask unless already covered:
'Why am I sick?'
'How long is the sickness going to last?'
'Will my baby be OK?'

Examination findings to be gi ven to candidate by the examiner on request
She looks unwell and drawn. Her tongue is dry and firm. Tissue turgor of the skin is
diminished.
Pulse 110/min.
Blood pressure 120/80 mmHg.
Temperature 36.8 C
Abdominal examination, uterus not palpable
No loin tenderness.
Pelvic examination not repeated.
144 (R4)
751
Performance Guidelines



EXPECTATIONS OF CANDIDATE PERFORMANCE
The candidate should convey the substance of what follows to the patient:
Hyperemesis gravidarum is a common condition in pregnancy, with significant nausea and
vomiting in up to 10% of patients, a small number of these requiring admission to hospital for
intravenous fluids and antiemetics.
In order to assess the severity of the process, a urine specimen needs to be tested for
ketones. If ketones are present, admission for intravenous rehydration is usually required. If
the candidate indicates the need for such ketone testing, advise that this was strongly positive.
As the ketone test was strongly positive, admission is required.
Although hyperemesis gravidarum can be a complication of a normal pregnancy, it occurs with
increased frequency in association with multiple pregnancies, hydatidiform mole, and in
association with urinary tract infections. Ultrasound examination is therefore required to rule
out the former diagnoses, and a midstream urine specimen should be collected and subjected
to culture to rule out a urinary infection.
Treatment in hospital consists of rehydration with saline solutions and additional dextrose,
usually with vitamin B supplementation. Pyridoxine, or antiemetic therapy with Maxolonor
Stemetilcan also be used in an attempt to resolve the vomiting.
Investigations such as assessment of the serum electrolytes and urea, and liver function tests
should be performed to assess the degree of effect of the vomiting on the maternal bodily
function.
Usually the hyperemesis settles spontaneously, often having reached a maximum at about 70
days (ten weeks) of pregnancy, and by the time of 100 days (14 weeks) most of the symptoms
will have resolved.
If nausea and vomiting do not settle satisfactorily, or following initial therapy, other causes
such as small bowel obstruction, cerebral tumour or Addison disease need to be excluded.
As she is 38 years old, and is therefore at increased risk of a chromosomal abnormality of the
baby, genetic counselling should be arranged with advice given concerning the usefulness of
the quadruple maternal serum screening and nuchal fold thickness assessed by ultrasound
examination (screening procedures only) or the use of chorion villus biopsy (CVB) or
amniocentesis to assess the fetal karyotype and actually rule out Down Syndrome. As she has
indicated this diagnosis needs to be ruled out, the definitive tests of CVB or amniocentesis
should be advised following advice to her concerning the potential complication rates of each
of these procedures (risk of abortion due to the procedure is 0.5% for amniocentesis at 15-16
weeks, and about 1.5% for CVB at 11-12 weeks).

KEY ISSUES
Ability to investigate and treat a woman with hyperemesis gravidarum.
Recognition of the need for genetic counselling in view of advanced maternal age.

CRITICAL ERRORS
Failure to recognise the need for hospitalisation.
Failure to do ultrasound and urine examination to check pregnancy, diagnose twins, molar
pregnancy, urinary infection, and the presence of urinary ketones.
144 (R4)
Performance Guidelines

752


COMMENTARY
Nausea and vomiting in the first trimester of pregnancy is common. The management depends
upon the degree of vomiting and therefore the potential diagnosis. The candidate should have
knowledge of the conditions commonly associated with hyperemesis gravidarum. In this
particular case, the history provided should be carefully read as the patient is 38 years of age and
the question of counselling for chromosomal abnormalities in a woman of this age must be
covered.
Common problems likely with candidate performance are:
Failure to advise that the vomiting will usually cease or be markedly reduced by the time the
pregnancy has reached about 14 weeks of gestation.
145 (R5)
Performance Guidelines
Condition 145 (R5)
Visual difficulties in a 50-year-old man

AIMS OF STATION
To assess the candidate's ability to test eye movements and visual fields in a patient with a visual
field defect.

EXAMINER INSTRUCTIONS
The examiner will have instructed the patient as follows:
You have lost vision on each side but not straight-ahead. Your eye movements are normal. This
means that the outer field of vision of each eye is deficient, as instructed and demonstrated by the
examiner.

EXPECTATIONS OF CANDIDATE PERFORMANCE
The candidate should identify the findings as normal acuity of vision and eye movements, with a
bitemporal hemianopia.

Examiner's question to candidate:
After seven minutes, if not covered by candidate description of findings to you, ask the
following:
'Summarise your findings' (normal acuity and ocular movements; bitemporal hemianopia.)
'What would be your next investigation?' (CT or MRI of head looking for pituitary
space-occupying lesion, and endocrine investigations of pituitary function.)

KEY ISSUES
Adequate and accurate technique of examination of eye movements and clinical assessment
of visual fields.
Identification of bitemporal hemianopia.
Suspicion of pituitary origin.

CRITICAL ERROR
Failure to demonstrate the visual field defect.
753
145 (R5)
Performance Guidelines
COMMENTARY
A bitemporal hemianopia, as has been found in this patient, indicates involvement within the optic
chiasma of the midline decussating optic tract fibres from the nasal halves of each retina which
subserve the temporal visual fields on each side. Central lesions, such as pituitary tumours, are
the usual cause of lesions of the optic chiasma. The first fibres to be compressed are the fibres
supplying the inner, lower part of the retina the candidate should therefore usually commence
the assessment with the upper outer quadrant of each eye.

CONDITION 145 (R5). FIGURE 1.
Visual field study showing bitemporal hemianopia
CONDITION145 (R5). FIGURE 2.
Plain X-ray showing expanded pituitary fossa
754
145 (R5)
Performance Guidelines
CONDITION145 (R5). FIGURE 3.
MRI showing pituitary tumour
Assessment of visual field by confrontation:
Use a red target (e.g. a pin with a red top). Instruct the patient to cover the left eye with the palm
of the hand (without pressure on the eye), and fix the gaze on the examiner's right eye.
The target is presented in the midperiphery of each quadrant of the field.
The subject is instructed to identify the colour of the target red or not red.
A defect in optic nerve, optic radiations, or visual cortex will produce a field defect with a vertical
edge. (A retinal defect will produce a field defect with a horizontal edge an altitudinal defect).
Then instruct the patient to cover the right eye and fixate on the examiner's left eye. and repeat
the test.
Rationale of using a red pin head for field testing: in any comprehensive or other lesion of the
visual pathways, colour perception is the first visual property to be lost, before the development of
an absolute scotoma. Assessing the field by waggling fingers requires the patient to make
several decisions:
Are they fingers?
Are they moving?
This is unnecessary information. All that is needed is to demonstrate a difference in retinal
sensitivity in different areas, and testing colour perception is the most sensitive way of doing this.
755
146 (R6)
Performance Guidelines
756
Condition 146 (R6)
Cogniti ve state assessment of a 50-year-old barman
AIMS OF STATION
To assess the candidate's ability to administer a Folstein Mini-Mental State Examination (MMSE),
summarise the results (impaired registration and recall, performance on other tests normal),
identify the deficient cognitive function (isolated impaired new learning/short-term memory), and
interpret its significance.
EXAMINER INSTRUCTIONS
The examiner will have instructed the patient as follows:
Be cooperative, but uninterested, follow the candidate's instructions literally (even if given the
wrong instructions according to the test), do not be helpful. These are the responses you must
give to the test the candidate gives you.
Give correct information on your name, year, day and date, season and location of
consultation.
On being asked to repeat three words, take three attempts to learn them all.
On being asked to subtract 7s from 100 say '93, 86, 79, 72, 65...' then pause, ask 'what was I
taking off?' If instructed to continue with the task, do so to the best of your ability.
On being asked to spell WORLD backwards, answer correctly DLROW.
On being asked to recall the three words, say only one of the three, no matter how many
prompts. Say you can't remember any of the others.
Name the watch and pen/pencil (or other objects the candidate indicates) correctly.
Repeat the sentence 'no ifs, ands or buts'correctly, as you hear it said by the candidate.
Follow the three-stage command (paper test) correctly.
Read and follow the written instruction 'close your eyes' correctly.
Write the following sentence I like my garden'.
Copy the design accurately. Any other questions, (e.g. date of birth) give your own, or say you
don't know. The examiner will interrupt if the candidate seeks to obtain further history.
EXPECTATIONS OF CANDIDATE PERFORMANCE
The candidate is expected to administer and interpret the Folstein MMSE in a clinical setting,
where the patient has chronic excessive alcohol consumption, and a collateral history of impaired
memory and behaviour.
The station tests the candidate's skill in administering a screening tool for cognitive impairment,
and interpreting the results, given the relevant clinical information of chronic excessive alcohol
use, and poor memory interfering with everyday activities.
The candidate is given the Folstein MMSE form (Table 1) and asked to proceed with testing the
patient's cognitive state.
The candidate is required to explain the test and its purpose clearly to the patient, and proceed
through it in an orderly fashion, paying due regard to the patient's comfort and level of
understanding.
146 (R6)
757
Performance Guidelines



Candidates are not required to calculate the score, and for that reason, the score marks have
been deleted from the sheet they are given. If they attempt to calculate a score, tell them this is
not required.
If candidates attempt to obtain a further history or do other examinations, either physical or
general psychiatric (e.g. mood, psychotic symptoms), stop them.
Examiner questions to candidate:
At fi ve minutes ask the candidate to summarise the findings and any abnormalities, and what
they signify.
The candidate is expected to identify both normal and abnormal results on the MMSE, in order
to interpret the problem as one of impaired new learning/short-term memory function.
Responses that identify abnormalities in other than registration and recall are incorrect and
unsatisfactory.
A response that the patient is confused/delirious and has dementia or a bad longterm memory
is incorrect and unsatisfactory.
At six minutes:
If the candidate has not already told you that this screen shows other cognitive functions
are normal, ask if other functions are normal, and what is the significance of this (i.e. not an
acute brain syndrome as in a withdrawn state, no definite diagnosis of dementia).
Knowledgeable candidates may know that the focal impairment may be an amnestic
syndrome, and the association with Wernicke-Korsakoff syndrome (but this is not
obligatory to pass).
If the candidate has not told you the likely cause, ask for this (i.e. alcohol-induced brain
injury).
KEY ISSUES
Skill in using a screening tool for cognitive impairment.
Ability to interpret the results and to identify there is a focal problem of short-term memory
impairment.
Knowledge that this is likely to be an acquired brain injury as a complication of hazardous
alcohol use.

CRITICAL ERROR
Failing to identify the specific short-term memory deficit.
758

146 (R6)
Performance Guidelines

COMMENTARY
The Folstein MMSE is a screening tool assessment of cognitive function impairment broadly
speaking, delirium/acute brain syndrome/acute confusional state and chronic brain
syndrome/dementia. Abnormal results indicate the need for further neuropsychiatry/
neuropsychological and medical evaluation to make an exact diagnosis.
The cognitive performance deficits as scripted are in (2) Registration and (4) Recall these are
short-term memory functions; the deficits are almost certainly due to acquired brain injury from
chronic alcohol abuse. The cognitive deficits are confined to short-term memory functions. It is
very relevant that all other responses are correct, and that there are no indications with this
screening tool of a diffuse chronic brain syndrome/dementia of whatever kind, or an acute brain
syndrome/delirium/acute confusional state. In particular this patient is fully oriented and alert, and
hence he does not have an acute brain syndrome/concentration deficit in answering the doctor's
questions; the candidate is expected to know this.
This short-term memory deficit picture, with preservation of longterm memory and other
intellectual function, in the absence of an acute brain syndrome, is a focal cognitive impairment of
an amnestic (Greek without memory) type, and is one type of the cognitive decline found with
chronic alcohol abuse. Further neuropsychological testing is required to exclude more diffuse
impairment (e.g. dementia). It is possible the patient has or has had an episode of Wernicke
encephalopathy, and the knowledgeable candidates will appreciate this.
CONDITION 146 (R6). TABLE 1.
MMSE Examination

FOLSTEIN MINI-MENTAL STATUS EXAMINATION (MMSE)
Date of Examination:
Name:
Date of Birth: Sex: Handedness:
Occupation (previous):
Educational Level:
General remarks:
Hearing:
Vision:
146 (R6)
Performance Guidelines
759

FOLSTEIN MINI-MENTAL STATUS EXAMINATION (MMSE) (continued)
Record exact replies.
1. Orientation

What is the Year'

Season?

Date?

Month?

Day of the week?

Where are we? State

City

Suburb

Street
2. Registration

Name three objects, taking one second to say each, then ask the patient to repeat them.

e.g. Apple Table Coin

Repeat them until the patient learns all three. Ask patient to remember them for later.

Number of trials required to learn the answers
3. Attention/Concentration

Serial Sevens, beginning at 100, count backwards serially by 7 (stop after five answers)

Spell WORLD backwards
4. Recall

Ask the patient to name the three objects in Question 2
5. Language

Ask the patient to name the following as you point: 'pen'.... 'watch'

Have the patient repeat: 'No ifs, ands, or buts'

Have the patient follow a three stage command:

Take this paper in your right hand. Fold the paper in half.

Put the paper down in your lap'.

Have the patient read and obey the following printed sentence: CLOSE YOUR EYES

Have the patient write a sentence (containing subject, verb, object)
CONDITION146 (R6). TABLE 1.
MMSE Examination
6. Construction
Ask the patient to
copy this design
760
147 (R7)
Performance Guidelines



Condition 147 (R7)
Jaundice in a 25-year-old man

AIMS OF STATION
To assess the candidate's skill in defining the most likely type of hepatitis from the history,
choosing appropriate investigations and explaining these to the patient, and advising on
treatment whilst awaiting the results of the investigations.

EXAMINER INSTRUCTIONS
The examiner will have instructed the patient as follows:
You are aged 25 years. You noticed yellow skin and dark urine yesterday. You have felt unwell
for 10 days.
Opening statement
'Well doctor what do you think is wrong with me?'After answering this question the doctor will
seek more information from you. Respond to the relevant questions as indicated below.
Alcohol use: social/occasional (up to three standard drinks per week)
Drug sensitivities: none known
Past medical history: childhood illnesses only
Current medication: none
You went on two weeks holiday in Thailand returning a month ago.
You have a steady girlfriend who is well, condoms are used for contraception. She was on
holiday with you in Thailand.
You have never used illicit drugs.
You have never been immunised against either hepatitis A or B.
You are not a blood donor, and have never had a blood transfusion.
Until you noted the change in colour of your urine and stools you thought you were suffering from
'just a virus'. When the doctor is discussing hepatitis, your main concern is whether you will
recover fully and how long this will take.
Questions to be asked if not already covered
'Have I got hepatitis?'
'What type of hepatitis could it be?'
'How did I get it?'
'Can I be treated at home?'
'Could I pass it onto others?'
'What should I do whilst waiting for the results of the tests?'
'Will I recover completely?'
'How long will I be away from work?'
147 (R7
761
Performance Guidelines



EXPECTATIONS OF CANDIDATE PERFORMANCE
Further history should include:
Hepatitis A and B immunisation status.
Travel and sexual history.
Current medication including use of illicit drugs.
Identification of persons who have been in close contact with the patient.
Explanation to patient should include:
Hepatitis is an infectious disease of the liver with several possible viral causes -hepatitis A, B,
C, Epstein-Barr virus (EBV) infection. The disordered liver function causes jaundice. Hepatitis
A is the most likely cause in this case, based on travel history (incubation period 15-45 days)
and negative responses to risk factors for hepatitis B and C.
Need for confirmatory investigations. These must include liver function tests and hepatitis
serology.
Complete recovery expected if hepatitis A. which can usually be treated at home with care
concerning infectivity.
Immediate management whilst awaiting for the results of the tests the patient should
be advised to have:
Initial bed rest at home, up for shower and toileting.
Fat-free diet and extra fluids
No alcohol or paracetamol
Special care with personal and interpersonal hygiene to minimise risk of transmission of
infection by faecal-oral route.
Immunoglobulin for other members of household and girlfriend if hepatitis A is confirmed.
Review after investigation results have been received. If he becomes progressively unwell, he
may need hospitalisation, although this is unlikely.
Investigations which should be requested to be stated to the examiner
Liver function tests, international normalised ratio (INR), full blood examination, serology for
hepatitis A, B, C and EBV

KEY ISSUES
Further history travel, sexual history, intravenous drug use, blood donor?
Diagnosis hepatitis A is most likely but must be confirmed by serology.
Immediate management bed rest, fluids, diet, no alcohol or paracetamol.
Investigations liver function tests and hepatitis serology are mandatory.
Patient education and counselling method of transmission, infectivity, prognosis.

CRITICAL ERROR
Failure to order hepatitis serology.
147 (R7)
Performance Guidelines
762

COMMENTARY
Hepatitis A is a major public health concern worldwide. Transmission is largely by the faecal-oral
route but food and waterborne outbreaks occur commonly. Incubation is 2 to 6 weeks. Infectivity
is greatest just before the onset of jaundice when faecal excretion of virus is at its peak. A high
proportion of patients have subclinical infections. Symptoms of hepatitis A are nonspecific
malaise, anorexia, nausea, vomiting, low-grade fever and right upper quadrant pain. Patients
may notice the onset of dark urine and accompanying jaundice in the icteric phase.
Hepatitis B and C also occur in Australia and are endemic in South East Asia. The transmission
of these is via parenteral and sexual modes. They have a longer incubation period (1-6 months).
Acute infections with these are largely asymptomatic and presentations are almost always in the
chronic phase or with serological testing of at-risk ndividuals.
A number of other viruses, such as EBV, cytomegalovirus and herpes simplex may also cause
acute hepatitis, usually in the context of a systemic illness.
Liver function tests and hepatitis serology are required to evaluate acute hepatitis. The definitive
laboratory test of acute hepatitis A is serum hepatitis A viral immunoglobulin (HAV IgM). The
urgency of evaluation of hepatitis and consideration for hospital admission depend on the
functional status of the patient, the extent of hepatic dysfunction, the changes in liver enzymes
and evidence of hepatic synthetic dysfunction (encephalopathy, prolonged INR).
In this patient management at home is likely, with attention to the principles of rest, good
hydration, alcohol and potential hepatotoxin (e.g. paracetamol) avoidance, with further
monitoring of liver function. Advice about how to avoid faecal-oral transmission to others and
offer of immunoglobulin therapy should be discussed. The issue of travel health should be raised
with the patient for educational purposes.
763
148 (R8)
Performance Guidelines



Condition 148 (R8)
Assessment of prominent leg veins in a 38-year-old woman
AIMS OF STATION
To asses the candidate's ability to evaluate venous circulatory status of the lower limb.

EXAMINER INSTRUCTIONS
The patient is a real patient playing herself. The examiner will check the specific details of the
venous circulation prior to the examination and ask the candidate to confine detailed examination
to one leg (the one with the most significant findings).
EXPECTATIONS OF CANDIDATE PERFORMANCE
The candidate should be able to:
Identify and describe the sites of visible and palpable varicosities.
Identify which varicosities are clearly within the distribution of the long or short saphenous
system.
Check appropriately for evidence of venous valvular incompetence between the superficial
and deep venous system.
~At level of saphenofemoral junction.
~At lower sites in the thigh and leg.
Check for evidence of chronic venous circulatory insufficiency affecting the deep venous
system.
Identify any evidence of superficial thrombophlebitis.

KEY ISSUES
Ability to assess the superficial venous system.
Ability to assess the deep venous system.

CRITICAL ERROR - none defined

COMMENTARY
Varicose superficial veins (dilated tortuous veins) commonly affect the legs, particularly in
women. Predisposing factors include:
pregnancy
a family history
arteriovenous communications
pelvic masses
incompetent valves (either primary or secondary to pregnancy, or to previous deep venous
thrombosis)
148 (R8)
Performance Guidelines
764

Venous drainage of the lower limbs is by a dual compartment system:
A superficial thick walled low pressure system in subcutaneous fat, with major embryological
preaxial (great or long saphenous vein) and postaxial (lesser or short saphenous vein)
draining conduits entering the deep venous system respectively at the saphenous opening in
the groin and in the popliteal fossa. Valves normally prevent reflux from the deep system.
A deep high pressure multivalved drainage system via axial venous channels below the deep
fascia adjacent to the main arteries, emptied during muscle action by the squeezing action of
the 'muscle pump', which also siphons flow from superficial to deep channels.
Reversal of the muscle pump siphon occurs when valvular incompetence develops in the deep
system (often from previous deep venous thrombosis), with incompetence involving the main
communications between superficial and deep systems (at groin and in popliteal fossa, in the
thigh, and from posterior and anterior arch veins below the knee). Continuing ambulatory high
pressure reflux causes maintained ambulatory venous hypertension in the superficial veins. This
predisposes to the development of stigmata of chronic deep venous insufficiency, with signs of:
brownish skin pigmentation with desquamative dermatitis associated with extravasation of red
cells and haemosiderin deposition;
subcutaneous indurated woody tissue oedema from sclerosing periangiitis;
incipient or previous ulceration particularly in the 'gaiter' area; and
cutaneous venous 'flares'.
Physical examination should check for presence of incompetence of saphenofemoral opening
using visual and manual assessment checking for a venous impulse or thrill over the long
saphenous vein in the groin just medial to the femoral artery, and using a venous tourniquet
appropriately (Trendelenburg tests). These tests are of most value in patients with easily visible
palpable varicosities. In patients with fat legs they are far less reliable and adequate specialist
assessment of the venous circulation by Doppler ultrasound studies is required for accurate
diagnosis.
The following illustrations (Figures 3 and 4) show another male patient with varicose veins of
right leg. Complete control of the varices is obtained by compression of the long saphenous vein
in the upper thigh and having the patient stand, as illustrated. No varices appear below the
compression point, indicating incompetence of the long saphenous vein at the saphenofemoral
junction or in the subsartorial canal.
The next illustrations (Figures 5 and 6) indicate use of a Doppler probe combined with a venous
tourniquet to clarify levels of incompetence.
148 (R8)
Performance Guidelines

CONDITION148 (R8). FIGURE 3. CONDITION148 (R8). FIGURE 4.
Obvious long saphenous system Incompetence fully controlled by
incompetence on standing pressure over long saphenous
vein in upper thigh

CONDITION148 (R8). FIGURES 5 AND 6.
Testing incompetence with tourniquet and hand held Doppler probe
765

766
Interactive Clinical Assessment Other
Methods and OSCE Modifications

Peter G Devitt and Heather G Alexander
There are nine and sixty ways of constructing tribal lays, and every single one of them is right'
Rudyard Kipling (1865-1936)
The aim of any clinical assessment process is to test and measure clinical competence. Ideally,
the tool used should be able to sample widely, be objective and capable of assessing applied
knowledge, attitudes and the skills of history-taking, physical examination, together with the
interpretation and synthesis of clinical data.
The long and short cases
The proponents of the long case would argue that this format meets all the requirements of a test
of clinical competence and allows the assessment of the whole interaction of the student and the
patient in a real world setting. Done well, the long case allows the examiner to assess the
candidate's interaction with a patient, the ability to communicate, take an appropriate history,
perform a thorough, yet efficient examination in a professional manner, integrate the information
obtained to arrive at an appropriate diagnosis and conclude the process with a discourse on the
problems and how the condition should be managed.
Long cases and viva voce examinations have been used for many years to assess clinical
competence, but for educational and logistic reasons, many medical schools and examining
bodies have moved away from these methods in recent years. Too often the long case is prone to
the biases of observer error, a lack of standardisation, but most importantly, a small sample size.
Doctors and students perform differently on different cases and so the sample size of one in a
long case leads to low reliability. Even when combined with short cases, the range of material in
such an examination is limited. The short cases are designed to supplement the long case and
ensure that the candidate is examined on a variety of clinical material. In most instances the
number of cases is relatively small, and this risks reducing the examination process to a lottery. If
the candidate is completely unfamiliar with one of the cases, the candidate may fail the
assessment. Attempts have been made to improve the long case through the use of observed
long cases and through more structured approaches to unobserved cases, for example, the
objective structured iong examination record (OSLER).
1
Logistic reasons have also influenced
the move away from using long cases. It is difficult to find suitable patients and sufficient
examiners to allow this sort of testing to be done on a broader range of cases and for a large
number of candidates; and most medical schools have now moved to alternative formats.
The viva voce
The viva voce has a limited role in clinical assessment. It is mainly a test of knowledge and
application of knowledge, but can also be used to make a measure of some of the other
ingredients of clinical competence such as ethical values, honesty and tolerance of ambiguity.
The viva voce is subject to the same weaknesses as the long and short case process, including
low reliability and subjective bias. With the development of the objective structured clinical
examination (OSCE), the viva voce has lost popularity as an examination tool, particularly at the
undergraduate level. It still has a role in critical evaluation. Whilst the viva is subjective and
variable, it is these characteristics that make it a favoured format in certain situations. These
include the assessment of individuals being considered for
1 Val Wass, Van der Vleuten C. The long case. Medical Education 2004, 38: 1176 1180
767
Interactive Clinical Assessment Other
Methods and OSCE Modifications
distinctions or prizes or for others who are potentially fail candidates. The viva gives the
examiner the ability to probe and explore, looking for candidate's strengths and weaknesses and
it provides the candidate with the opportunity to more fully explain, discuss or justify an issue.
The objecti ve structured clinical examination (OSCE)
The development and use of a multistation, objective and structured assessment process was
reported by Harden in 1979.
2
Since then, the OSCE has taken off as probably the most popular
form of assessment of clinical medicine. It is used mainly by medical schools, but specialist
colleges and other postgraduate bodies are slowly moving from the long and short case
scenarios to some type of OSCE format. Provided that a sufficient number of stations are used,
the OSCE can be used to sample a broad range of clinical scenarios and be a reliable and valid
examination process. Most OSCE contain between 15-20 stations and each station may be 5-10
minutes. Stations can be designed to test a range of clinical skills, including psycho-motor,
problem-solving, and attitudinal and communication skills. It has been suggested that to be
suitably reliable and valid an OSCE should contain at least 150 minutes of examination time.
3,4,5,6

The OSCE marking system -assuming that the examiners adhere to it ensures objectivity and
reproducibility. The reliability of the process can be increased if two examiners are used for each
station.
Formal comparisons of the OSCE with other formats, such as the viva voce, show that whereas
the OSCE appears to be a fair reflection of overall student performance throughout the course,
the viva voce correlates poorly.
Most forms of clinical assessment provide little in the way of feedback to the teachers and
assessors. The OSCE can be used to provide such information and a well constructed station
with a proven pedigree can be used to identify areas of weakness in student learning or
understanding or misconceptions that may exist.
The OSCE has attracted criticism in that its rigid structure does not allow adequate discernment
of student performance. This is largely a reflection on the system of marking according to a check
list. A check list may not take into account the logic and cohesion with which the task is
performed, but rather reward the candidate for completion of that task, no matter how random
and disorganised. This problem can be overcome by global marking
7
'
8
, which moves beyond the
mere identification of tasks and allows the examiner to mark the candidate for the skill, ability and
coordinated approach with which these tasks are approached. While global marking does
introduce subjectivity back into the assessment, the guidance of the examiner to the domains to
be assessed has produced a necessary refinement into the OSCE format.
2 Harden RM, Gleeson FA. Assessment of clinical competence using an objective structured clinical
examination (OSCE). Medical Education 1979, 13: 41-54.
3 Langford NJ , Landray M, Martin U, Kendall MJ , Ferner RE. Testing the practical aspects of therapeutics by
objective structured clinical examination. Journal ol Clinical Pharmacy and Therapeutics 2004, 29: 263 6.
4 Newble Dl, Swanson DB. Psychometric characteristics of the objective structured clinical eamination.
Medical Education 1988, 22: 325-34.
5 Robers J , Norman G. Reliability and learning from the objective structured clinical examination. Medical
Education 1990, 24: 219 23.
6 Smith LJ , Price DA, Houston IB. Objective structured clinical examination compared with other forms of
student assessment. Archives of Disease in Childhood 1984, 59: 1173-6.
7 Allen R, Heard J , Savidge M. Global ratings versus checklist scoring in an OSCE. Academic Medicine 1998,
73: 597-8.
8 Cunnington J PW, Neville AJ , Norman GR. The risks of thoroughness: reliability and validity of global rating
and checklists in an OSCE. Advances in Health Sciences Education 1996, 1: 227-233.
768
Interactive Clinical Assessment Other
Methods and OSCE Modifications
The MiniCEX (Mini Clinical Examination Exercise)
This is a modification of the clinical evaluation exercise (CEX) developed by the American Board
of Internal Medicine (ABIM) for evaluation of clinical competence of interns and residents. The
original CEX suffered from similar disadvantages to the long case in that a limited number of
cases was examined. The miniCEX has modified the CEX by having examiners observe several
15-20 minute interactions throughout the year in different clinical settings. The examiner records
aspects of the case, including complexity, and whether the focus of the interaction was
data-gathering, diagnosis, treatment or counselling. The candidates are then scored on a 9-point
scale for interviewing, physical examination, professionalism, clinical judgement, counselling,
organisation and efficiency, and overall competence. The miniCEX has been shown to be reliable
and evidence of its ability to discriminate between different levels of performance supports is
validity. The miniCEX offers a method to evaluate residents' clinical skills through direct
observation with real patients across multiple patients and settings.
In the current age of transparency of process, coupled with increasing legal challenges, it is even
more important that assessment bodies are able to justify their
decisionmaking, particularly in regard to high-stakes examinations.
Examining bodies are increasingly adopting assessment methods that
include a broader range of clinical cases, using standardised
role-playing or real patients. No matter what examination format is used,
users must justify it in terms of validity, relevance, appropriate
standards, scope and objectivity.
9
In University Clinical Schools, much teaching and much continuous
assessment is done throughout the course during clinical ward rounds
and office consultations in an outpatient or emergency department or
community attachment and as apprentice internship during the latter
years of the course. With more extended discussions and presentations
the MCAT format can be presented as several sequential MCAT
assessing history-taking, examination, diagnostic and management
plans, patient counselling, etc.
Examples of typical clinical problems expressed as questioning and
discussions on ward rounds follow as further examples of assessment
these scenarios can be modified to suit the MCAT formatting.
The aim of any clinical
assessment process
is to test and measure
clinical competence.
Ideall y, the tool used
should be able to
sample widely, be
objecti ve and capable
of assessing applied
knowledge, attitudes
and the skills of
history-taking,
physical examination,
together with the
interpretation and
synthesis of clinical
9 Holmboe ES, Huot S, Chung J , Norcini J , Hawkins RE. Construct Validity of the MiniClinical Evaluation
Exercise (MiniCEX). Academic Medicine 2003, 78: 826-30.
769
Interactive Clinical Assessment Other Methods and OSCE Modifications
770
Interactive Clinical Assessment
Candidate Information and tasks
CONDITIONS 149-150
149 Confusion and delirium after surgery in a 50-year-old man
150 Postoperative fever in a 45-year-old woman
149-150
Candidate information and Tasks
Condition 149
Confusion and delirium after surgery in a 50-year-old man

CANDIDATE INFORMATION AND TASKS
You are the intern called to the ward to see a 50-year-old man who has become confused two
days after a left total knee replacement. A few hours earlier, he started to behave in an irrational
manner, became agitated and difficult to manage. Until this stage he had been making an
uneventful postoperative recovery. His confusion has now culminated in the patient being
disoriented, noisy and difficult to restrain.
The patient's wife is with the patient and she has been unable to help.

YOUR TASK IS TO:
Assess the situation; formulate a management plan; and counsel the patient's wife as to the
cause of the current problem.

In this scenario format, your optimal approach is as shown in the performance guidelines,
indicating questions you should be asking yourself as you proceed.
1
<
2
The Performance Guidelines for Condition 149 can be found on page 773

Condition 150
Postoperative fever in a 45-year-old woman

CANDIDATE INFORMATION AND TASKS
As part of your duties as the surgical intern, you are examining the patients from the previous
day's operating list. You are at the bedside of a 45-year-old woman who had a laparoscopic
cholecystectomy for gallstones yesterday. You note from the chart that her temperature is
38.5C.
YOUR TASKS ARE TO:

Assess the overall condition of the patient.
Provide a diagnostic and management plan.
The Performance Guidelines for Condition 150 can be found on page 776
1 Devitt, Barker, Mitchell, Hamilton-Craig (eds). Clinical Problems in General Medicine and
Surgery. Harcourt. Edinburgh, 2003.
2 Alagiakrishnan K. Delirium, http://author.emedicine.com/med/topic3006.htm
771
Interactive Clinical Assessment -Other Methods and OSCE Modifications
772

Interactive Clinical Assessment
Performance Guidelines
CONDITIONS 149-150
149 Confusion and delirium after surgery in a 50-year-old man
150 Postoperative fever in a 45-year-old woman
149
Performance Guidelines
773
Condition 149
Confusion and delirium after surgery in a 50-year-old man
SELF-QUESTIONING FOR CANDIDATES
How will you approach the problem?
Your first action must be to ensure that the patient is not a danger to himself or to others around him. This may involve
a degree of physical restraint whilst you assess the situation and obtain a history and examine him. You are unable to
obtain a history as he is irrational and does not respond coherently to questions.
Before you examine the patient, it would be prudent to ensure that intravenous access has been established and that
he is on a high flow oxygen mask. Institute pulse oximetry if available. At the same time as you insert an intravenous
cannula, collect blood samples for routine haematological and biochemical screens. In addition, collect samples for
blood sugar estimation, and blood culture, especially if he is febrile
At this stage you should look at the case notes and the nursing observations charts. In particular, you should note:
Any recent change in the vital signs (blood pressure, pulse rate and temperature).
Fluid balance.
Recent drug administration (e.g. opiates, hypnotics).
Details of the recent surgical procedure.
Past medical history (e.g. problems related to diabetes or cardiovascular disease).
Drug use (including alcohol).
Comments in the medical and nursing record (e.g. sleep patterns, any abnormal behaviour).
~ Any abnormal laboratory investigations (e.g. anaemia, macrocytosis, abnormal liver function).
Once you have obtained as much information as possible from the case notes, nursing record, patient, relatives and
personnel involved in the immediate care of the patient, you are in a position to undertake a physical examination.
How will you focus your physical examination?
Talk to the patient, reassure him and try to establish whether he is oriented and aware of what is going on. He is clearly
quite disorientated. Undertake the following:
Check the vital signs of pulse rate and rhythm, respiratory rate, temperature and blood pressure.
Examine the cardiorespiratory systems.
Look for evidence of sepsis (chest, wound).
Look for evidence of venous thromboembolism (legs, chest).
Look for any neurological deficits.
When you have completed your physical examination you can discuss the situation with the nursing staff and family
members and formulate a management plan.
It is difficult to take a history or to examine the patient as he is agitated and insisting that he be allowed up to go to the
toilet. He spends much of the time sitting on the edge of the bed.
149
Performance Guidelines
774
The chest appears to be clear, but with his confused state, your examination is less than adequate. Similarly, the
examination of the abdomen is rather difficult, although you think there may be some tenderness in the lower
quadrants. The operation site around his knee looks healthy.
The patient's wife tells you that the patient has been quite well up to the time of his recent surgery, but had become
rather anxious over the impending operation and had been drinking rather more than usual.
In the light of the history and examination:
What are the likely causes for the confusion and how you would go about differentiating
them?
The likely causes of his confusion would include:
Hypoxia: a very common cause, particularly in elderly patients. Arrange arterial blood gas analysis (by this stage
the patient should be attached to a pulse oximeter), chest X-ray and ECG (possible pulmonary embolus).
Hypotension: measure vital signs.
Sepsis: investigations should include: white cell count, urinalysis, chest X-ray and blood cultures.
Metabolic abnormalities: serum electrolytes; blood sugar estimation; arterial pH.
Cardiac disease: ECG to exclude myocardial ischaemia/infarction, arrhythmias.
Cerebrovascular event; neurological examination.
Pain: look at medication chart and nursing notes.
Opiate overdose or effect of other drugs: look at medication chart. Cease any potentially contributing medications.
Drug withdrawal, including alcohol, benzodiapines and narcotics: seek information from family and friends.
Exacerbation of pre-existing medical conditions: for example dementia, hypothyroidism check past and recent
history, arrange blood tests (thyroid function tests TFTs).
The patient becomes quieter in response to intravenous fluids and oxygen by face mask.
How would you now counsel the family?
You need to reassure the family that:
The situation is under control.
Your investigations may yield a cause for the confusion.
There could be a multitude of factors contributing to the patient's delirium.
Alcohol withdrawal is a common cause of post-operative confusion and should be easily controlled and the
problem self-limiting.
You will return to review the patient progressively, particularly when the results of the investigations are through.
149
Performance Guidelines
775

Postscript
This clinical scenario was based on a real case. Little has been changed. Amongst the investigations performed was a
chest radiograph. This showed free gas under the right hemidiaphragm. Further abdominal examination of the now
calmer patient revealed clinical evidence of a lower abdominal peritonitis.
An exploratory laparotomy revealed a widespread purulent peritonitis secondary to perforated diverticular disease.
The patient underwent resection of the diseased section of sigmoid colon and made a satisfactory recovery.
In this instance, the patient's delirium was initially put down to alcohol withdrawal. By following sound clinical practice,
other causes of confusion were sought and the problem eventually tracked down to intra-abdominal sepsis and acute
diverticulitis with peritonitis.
150
Performance Guidelines
776
Condition 150
Postoperative fever in a 45-year-old woman

DESCRIBE HOW YOU WOULD ASSESS THE SITUATION?
A full assessment will focus on likely causes for her fever and will include:
Scrutiny of the case notes, including the operative details.
Appraisal of the nursing record, in particular the ward charts.
An appropriate history and physical examination.
Consideration of any diagnostic procedures.
The most likely explanation for the fever is pulmonary atelectasis. However, it is important to exclude other important
causes of postoperative pyrexia. These include:
Pre-existing problems (e.g. chest infection, immobility deep venous thrombosis [DVT]).
Sepsis or some other complication (concealed haemorrhage, bile leak) at the surgical site (operative bed, wound
drains, wounds).
Sepsis at other sites (urinary tract).
Intravenous cannula site (thrombophlebitis causing fever from intravenous cannulae and infusions is usually
inflammatory and irritative rather than infective; treatment is to remove the offending cannula).
Drug reaction (check the charts).
Febrile blood transfusion reaction (unlikely in the present circumstances of elective uncomplicated surgery but
check operation and anaesthetic notes).
The questions you will need to ask yourself include:
Is the patient septic?'
Is there evidence of a septic or inflammatory focus?'
These two questions will be answered by a careful examination of the patient record, ward observations and the
patient herself. In this instance, the fever has appeared rapidly after the operation and is most likely an inflammatory
response to partial pulmonary collapse. Pulmonary atelectasis is more likely to occur in the presence of pre-existing
lung disease, or after major thoracic or abdominal surgery where there has been poor control of pain. Physical signs
may include crackles at lung bases or evidence of more significant collapse; but with minor degrees there may be no
abnormal findings.
Septic problems related to the operation itself do not usually declare themselves for several days after the procedure.
Similarly, venous thromboembolic complications and urinary infections tend to manifest themselves after a lapse of
several days or in the second postoperative week. The major postoperative complications that give an early spike of
fever (within the first 24 hours) are inflammatory, rather than infective, and include:
Pulmonary atelectasis (the most common cause), which responds to mechanical measures to improve pulmonary
ventilation and remove inspissated sputum plugs. The major thrust of treatment is active breathing exercises and
chest physiotherapy (including postural drainage). Antibiotics have little role in the initial treatment, although many
doctors prescribe antibiotics concurrently.
A wound haematoma.
A bile leak.
Superficial chemical thrombophlebitis (cannula site).
150
Performance Guidelines
Deep venous thrombosis.
Myocardial infarction.
An allergic drug reaction or anaphylaxis.
An attack of gout.
Examination of the patient must focus on the vital signs (temperature, blood pressure, pulse rate) and the chest
(consolidation, collapse). The other conditions listed above must be considered and appropriate physical examination
undertaken.
The patient looks well and is able to tell you that she is not in pain and is moving about the bed freely. From the case
record you note she has a 20 pack-year smoking history and occasionally gets bouts of bronchitis. She is not on any
medications. She was given a single dose of a second-generation cephalosporin as antibiotic prophylaxis and has only
required paracetamol for analgesia.
On examination her pulse rate is 90/min and regular. Her blood pressure is 130/85 mmHg. The only abnormalities on
examination of her chest are some crackles at both bases. The dressings on her wounds are dry and there is no wound
drain. Her abdomen is soft. Her limbs are normal.
Provide a diagnostic and management plan for this patient
The cause of the fever in this patient is most likely to be post-operative pulmonary atelectasis. The illustrations
show typical chest X-ray findings in such patients before and after surgery (Figures 1 and 2).
In this situation the patient will require little apart from early mobilisation and chest physiotherapy. Sound advice would
be to monitor the patient (pulse oximeter) and administer humidified oxygen and a bronchodilator. Given her relative
youth and rapid recovery from the laparoscopic cholecystectomy, these measures are probably not necessary.
Obviously if the patient does not respond to your simple measures, then the treatment must be more active and include
oxygen by face mask and salbutamol by nebuliser.
CONDITION150. FIGURE 1. CONDITION150. FIGURE 2.
Preoperative X-ray Postoperative X-ray
777
150
Performance Guidelines
778

A white cell count could be performed, looking for a leucocytosis, but the findings are unlikely to
alter your management. Similarly a chest radiograph might be arranged, but you know what it is
likely to show and again, the investigation is unlikely to alter your management, although it may
be confirmatory of the diagnosis by showing areas of atelectasis in the absence of abnormal
clinical findings.
You will have checked all your medications for potential allergies, and any intravenous line or site
of intravenous injections for evidence of tender and painful phlebitis. You have checked that no
worrying abdominal signs are present. You will monitor her progress by serial review.
Postscript
With breathing exercises and chest physiotherapy her fever resolved. She subsequently made
uneventful progress and was discharged on the second postoperative day. Followup at two
weeks confirmed an uneventful recovery.
779
The Four Station Progressive OSCE

'Begin at the beginning', said the King gravely, 'go on to the end; and then stop'.
Lewis Carroll (1832-1898)
A proposed examination sequence (OSLER) is outlined below. Each MCAT station is 10 minutes and supervised by an
assessor. A global marking scheme would be used. In order of importance the aims of the MCAT/OSLER are to:
Assess the clinical skills of history-taking and physical examination, problem-solving and management.
Assess communication and counselling skills (with patient and assessor).
Assess the candidate's knowledge.
The combination of these domains comprise global clinical reasoning skills
As an example of content for the OSLER, a case of carcinoma of the oesophagus is
provided.
Station 1 The interview (communication, history and examination skills).
Station 2 Synthesis and management plan.
Station 3 Interpretation of investigations.
Station 4 Discussion of the management options with the patient (counselling).
STATION I
Objectives
A. Toestablish a likely diagnosis.
B. Toassess general state of health.
C. Toensure the patient understands the possible causes of the illness.
Scenario
Candidates will take a history and examine a 68-year-old man with a three month history of dysphagia. There has been
some loss of weight and a past history of heartburn. He smokes 20 cigarettes a day and consumes 20 g of alcohol a
day. Three years ago he suffered a myocardial infarction. Currently he takes aspirin and is on medication for
hypertension and obstructive lung disease. He had a knee replacement two years ago. complicated by a deep vein
thrombosis.
STATION 2
Objectives
A. Tosummarise the problem to the assessor.
B. Toprovide a likely diagnosis.
C. Todiscuss the management strategy and reasons for performing any investigations.
Scenario
The candidates should be able to provide a succinct and clear summary of the history and relevant normal and
abnormal examination findings delivered in a logical format. The candidate should state the relevant features from
the history (e.g. smoking, alcohol, heart and lung disease) and discuss how these may influence treatment. The
candidate needs to discuss:
Establishing the diagnosis (endoscopy/radiology).
780
151
The Four Station Progressive OSCE
Looking for tumour spread (blood tests, chest imaging by chest X-ray, computed tomograph [CT]).
Assessment of cardiorespiratory state (ECG, pulmonary function tests, echocardiography),
Assessment of haematologic status (haemoglobin and full blood examination, coagulation status).
Assessment of anaesthetic and operative risk factors (American Society of Anaesthesiologists Staging).
STATION 3
Objectives
A. Tointerpret investigations.
B. Toplan management, based on information obtained in the two previous stations.
Scenario
The candidates will be provided with the investigations appropriate to the case. In this instance, there will be blood
results, ECG, pulmonary function tests, echocardiogram and CT of the chest and abdomen, and blood and coagulation
studies. If endoscopy has been performed, the findings would also be available. His previous episode of deep venous
thrombosis (DVT) has implications in perioperative care.
Candidates will describe the findings to the assessor and outline a management plan.
STATION 4
Objectives
A. Tocounsel the patient on the results of the investigations.
B. Tocounsel the patient on a proposed plan of management.
One disadvantage of this system is the narrow range of problems covered in the examination, unless (as is commonly
the case) the patient has a number of ailments and multisystem problems. This could also be addressed by running an
eight station OSLER based on two patient scenarios.
On the positive side, this style of OSLER represents a move away from testing knowledge and allows the assessors to
focus on clinical skills and counselling abilities. Through the integration of the four stations, candidates can show their
ability to synthesise a clinical problem, put the different aspects of the case into perspective, discuss the diagnosis and
management, and interpret clinical, radiological and laboratory data in a suitably holistic manner.
The global marking scheme would allow the assessors to look at the overall performance of the candidates, rather than
concentrate on minutiae.
781
Glossary of Terms and Abbreviations

GLOSSARY OF TERMS
ABSOLUTE STANDARDS (criterion-referenced standards)
Assessment aims are to determine candidate ability to achieve defined minimum acceptable competencies and
performances over a selected broad range of knowledge, skills and attitudes. Test difficulties and candidate abilities
are integrated to derive a minimum pass score. AMC assessments follow these aims in MCQ and Clinical
assessments.
AUTONOMY
The moral right to choose and follow one's own plan of life and action, including the right to refuse treatment.
BENEFICENCE
The moral duty to help persons in need. CAPACITY
The patient's ability to understand information relevant to making a decision. Capacity determinations are made by
medical practitioners, sometimes using the help of psychiatrists and other members of the healthcare team.
CLINICIAN-ASSISTED SUICIDE
The act of intentionally killing oneself with the assistance of a clinician who deliberately provides the knowledge,
means, or both.
COMPENSATION
Compensation allows for ability of a good score in some subsets of multidisciplinary assessments to make up for a
poor score in others. The AMC applies this principle in MCQ and Clinical assessments, while identifying standards of
minimum basic competencies in each discipline.
COMPETENCE
This term is frequently used interchangeably with capacity. Strictly speaking, it is a legal term denoting the right to
make a decision. The legal presumption is that all adults are competent, and only a judge can rule a person
incompetent. Clinical competence
describes what the candidate/doctor is capable of doing.
CONSENT
The autonomous authorisation of a medical intervention by individual patients. Consent has three components:
disclosure, capacity, and voluntariness.
DISCLOSURE
The provision of relevant and material information regarding a decision by a doctor to a patient (and its comprehension
by the patient).
DISCRIMINATION
An act, practice or policy that differentiates between, or otherwise treats persons in a different way, on the basis of
such status as gender, age, nationality, religion, race, financial means or sexual orientation.
DISTRIBUTIVE JUSTICE
The obligation of clinicians to make appropriate healthcare available to their patients in a fair and equitable manner.
782
ETHICS
The discipline" dealing with principles and values defining what is good (acceptable/ appropriate/reasonable) and bad
(unacceptable/inappropriate/unreasonable), and with duties and obligations for various groups. In medicine,
application of ethical principles to complex and controversial issues and to evolving therapies can lead to consensus
codes of practice in specific areas.
EUTHANASIA
A deliberate act undertaken to end the life of another person in order to end suffering; the act is the cause of death.
FIDUCIARY OBLIGATION
The obligation to promote the best interests of persons who have entrusted themselves to the fiduciary (e.g. the
clinician); an obligation of the highest loyalty, fidelity and trust.
FORMATIVE ASSESSMENT
Assessment that is primarily intended to provide information for the learner, and thus tends to be during instruction to
allow for immediate feedback and remediation of performance if necessary. Formative assessment should be
integrated into an ongoing curriculum process to ensure that standards of performance are maintained over time.
IN-TRAINING ASSESSMENT
The process of collecting evidence about trainees during their period of training in order to judge their progress
towards achieving the required standards. Evidence may be provided by the trainee, supervisors and mentors, peers
and patients.
JUSTICE
The fair distribution of benefits and burdens within a community. MATERIAL RISKS
Risks that are common, and those that are serious, even if uncommon.
NONMALEFICENCE
The duty to refrain from doing harm.
PERFORMANCE
Clinical performance describes the application of competence: what the candidate/ doctor actually does in clinical
practice.
POSITIVE PREDICTIVE VALUE
High values indicates that there is a high probability that an event will occur when a statement is made about an
occurrence before it actually occurs.
RELATIVE STANDARDS (norm-referenced measurements/ranking)
These are established by the performance of candidates who, essentially, compete with each other to reach a pass
standard. This may be determined statistically (e.g. one or two standard deviations below the mean). The approach is
simple and involves ranking of raw scores, but has limitations in determining clinical competence and performance.
783
RELIABILITY
Where a measure will give very similar outcomes each time it is applied. At a test level this implies that there is internal
consistency so that all items are measuring the same attribute, while at the individual level it implies that a similar score
would be achieved on the re-administration of the same test.
REPRODUCIBILITY
The ability of a test to reinstate the patterns of behaviour that are initially measured in a different time or context. This
requires both validity and reliability in the tests used. These are necessary (but not sufficient) requirements as they will
be influenced greatly by the context of the measurement.
SECURITY/INVIOLABILITY OF THE PERSON
The fundamental right of individuals to expect respect for their bodies and persons and nonviolation of their bodies and
persons, be it through physical, psychological or other means.
SENSITIVITY
In statistical terminology, the true positivity rate of a test in a given population. A highly sensitive test gives a positive
result identifying most or all of the patients in the population who have the disease/condition under test. The test may
along the way give some falsely positive results if the test is excessively sensitive and thus cross-reactive with other
conditions than the one under test.
SOMATISATION
The conversion of emotional conflict and stresses into physical symptoms. SPECIFICITY
The converse of sensitivity: in statistical terminology the true negativity rate of a test. Highly specific tests for a given
disease/condition will rarely give falsely negative results: so a person testing negative is very unlikely to have the given
condition. The two properties of sensitivity and specificity are often inversely related few tests can combine absolute
specificity with absolute sensitivity.
STANDARDS
Assessments separate passers from failures using standards of competence and performance.
STATISTICAL METHODS
Those mathematical or analytical approaches, often referred to as psychometrics, that express the relationships
between quantitatively different stimuli or measures, and then seek to analyse and explain these with reference to the
attributes being measured.
SUMMATIVE ASSESSMENT
Assessment that is primarily intended for administrative purposes, such as progression to the next stage whence
'barrier assessment' or certification of competence. It is used to indicate whether a learner has reached a required
acceptable level of performance, and thus tends to be only at the end of a period of instruction with minimal feedback
to a learner.
784
VALIDITY
In short, that a measure of assessment actually measures what it was intended to measure. Differing terms are often
applied to give more specific meanings; thus face validity or content validity indicates the need for tests to be
acceptable to the stakeholders, or to appear to measure what they should be measuring. Validity can be improved by
use of a matrix or blueprint. Concurrent validity refers to one approach measuring the same attribute as another
approach, while predictive validity assumes that the measure will predict another behaviour.
VOLUNTARINESS
The patient's right to come to a decision freely, without undue pressure including force, coercion, or manipulation.
WEIGHTING
Weighting of the particular components of a global assessment can be applied when there is consensus evidence that
some components (e.g. Mastery/Key Issues) are of greater importance than other components. The AMC uses such
Mastery items in MCQ assessments, and has identified Key Issues and Critical Errors in clinical assessments
785

ABBREVIATIONS
'ABCDE' Airway, Breathing, Circulation, Disability, Exposure
ABO Major red cell blood groups
APB abductor pollicis brevis
AC air conduction
ADHD attention deficit hyperactivity (hyperkinetic) disorder
AF atrial fibrillation
ALP alkaline phosphatase
ALT alanine aminotransferase
AMA Australian Medical Association
AMC Australian Medical Council
APL abductor pollicis longus
ASA American Society of Anaesthesiologists
AST aspartate aminotransferase
AXR abdominal X-ray
BC bone conduction
BCC basal cell carcinoma
BDD Body Dysmorphic Disorder
BMI body mass index (kg/m
2
)
bpm beats per minute
BS blood sugar
CAGE' Cutting down on your drinking? Annoyed by criticism of drinking? Guilty
about your drinking, or during counselling; Eye opener needed in the
morning?
CAT clinical assessment tasks
CD computed disc
CEX clinical evaluation exercise
CM carpometacarpal
cm centimetre
CMV cytomegalovirus
CNS central nervous system
CPMC Committee of Presidents of Medical Colleges
CPR cardiopulmonary resuscitation
CRP C-reactive protein
CS Caesarean section
CT computed tomography
CTG cardiotocogram, cardiotocography
CVB chorion villus biopsy
CVP central venous pressure
786
CXR chest X-ray
DIP distal interphalangeal joint
DLCO diffusory capacity for carbon monoxide
DTP diphtheria, tetanus, pertussis
DVT deep venous thrombosis
ECG electrocardiograph/electrocardiogram
EEG electroencephalography
EBV Epstein-Barr virus
e.g. exempli gratia (Latin for example)
EMG electromyography
EMST emergency management of severe trauma
ENT ear, nose, throat
EPB extensor pollicis brevis
EPL extensor pollicis longus
ESR erythrocyte sedimentation rate
EUA examination under anaesthesia
FBE full blood examination
FCR flexor carpi radialis
FCU flexor carpi ulnaris
FDP flexor digitorum profundus
FDS flexor digitorum superficialis
FEV, forced expiratory volume in one second
FNAC fine needle aspiration cytology
FPL flexor pollicis longus
FRC functional residual capacity
FVC forced vital capacity
g gram
G6PD glucose-6-phosphate dehydrogenase deficiency
GBS group B streptococci
GCT glucose challenge test
GGT gamma glutamyl transaminase
GP general practitioner
GTT glucose tolerance test
Hb haemoglobin
hepB hepatitis B
HFE gene for haemochromatosis
Hib Haemophilus influenzae type B
HIV human immunodeficiency virus
HMO Hospital Medical Officer
787
HRT hormone replacement therapy
HSV herpes simplex virus
i.e. id est (Latin that is)
ICD international classification of diseases
IELTS International English Language Testing System
IgG immunoglobulin G
IMG International Medical Graduate (see also OTD)
INR international normalised ratio
IP interphalangeal (joint)
IUGR intrauterine growth restriction
IVP/IVU intravenous pyelogram/urogram
JVP jugular venous pressure
kg kilogram
km kilometre
LFT liver function tests
LH luteinising hormone
LICS left intercostal space
LMO local medical officer
LOT left occipitotransverse
LP lumbar puncture
m metre
MCAT multidisciplinary clinical assessment task
MCC Medical Council of Canada
MCQ multiple choice questions
MCV mean corpuscular volume
MDI metered dose inhaler
mg milligram
MIBG metaiodobenzyl guanidine
min minute
mm millimetre
mmHg millimetres of mercury
MMR measles, mumps, rubella
MMSE Mini-Mental State Examination
MP metacarpophalangeal or metatarsophalangeal (joint)
MRCP magnetic resonance cholangiopancreatography
MRI magnetic resonance imaging
MRS magnetic resonance spectroscopy
MSS maternal serum screening
NAI nonaccidental injury
788
NCS nerve conduction studies
NSAID nonsteroidal anti-inflammatory drug
NSTEMI non-ST elevation myocardial infarction
NTD neural tube defect
C degrees Celsius
OCP oral contraceptive pill
OHS occupational health and safety
OPV oral polio vaccine
OSCE objective structured clinical examination
OSLER objective structured long examination record
OTD overseas-trained doctor (see IMG)
Pap smear Papanicolaou smear
PL palmaris longus
PSA prostatic specific antigen
PCR polymerase chain reaction
Perse Latin by itself
PEF peak (maximum) expiratory flow
PET positron emission tomography
PGY1 postgraduate year 1
PICA posterior inferior cerebellar artery
PIP proximal interphalangeal (joint)
POC products of conception
PSA prostate specific antigen
PSGN post-streptococcal glomerulonephritis
PV per vaginum (Latin)
PVE pelvic vaginal examination
q.i.d. quattro in diem (Latin four times daily)
QT interval on the ECG
QTc interval on the ECG
RBC red blood cell
RPOC retained products of conception
RV residual lung volume
SARS severe acute respiratory syndrome
see squamous cell carcinoma
sic thus (Latin), inserted parenthetically to indicate a quotation with misspelling
SIDS sudden infant death syndrome
SPECT single proton emission computed tomography
SSRI selective serotonin release inhibitor
STEM I ST-elevation myocardial infarction

789
STI sexually transmissible infection
TAFE Technical and Further Education College
TIBC total iron-binding capacity
TLC total lung capacity
TOACS Task Oriented Assessment of Clinical Skills
TSH thyroid-stimulating hormone
URTI upper respiratory tract infection
USA United States of America
UTI urinary tract infection
VC vital capacity
VCR videotape cassette recording
VDK snake venom detector kit
VDRL Venereal Disease Research Laboratory
Viz. namely, that is to say (from Latin vidilecit)
VMA vanillylmandelic acid
VSD ventricular septal defect
VZV varicella zoster virus
WCC white cell count

You might also like